يرجى مشاهدة فيديو طريقة الحل قبل البدء بحل الكويز

نظام التست بانك

ملاحظة مهمة: اذا الاجوبة مو موجودة بالتست بانك , الافضل تحل باستخدام الذكاء الاصطناعي

انسخ السؤال بالتست بانك وستجد الاجابة بين قوسين

السؤال سيظهر هنا , والجواب سيكون بين قوسين
Which part of an information system consists of the rules or guidelines for people to follow?(((procedures)))
The equipment that processes data in order to create information is called the ________.(((hardware)))
In most cases, the word "software" is interchangeable with the word ________.(((program)))
What are the two major software types?(((system software and application software)))
Which of the following is a collection of programs rather than a single program?(((system software)))
Background software that helps the computer manage its own internal resources is called ________.(((system software)))
Identify the program that coordinates computer resources, provides an interface between users and the computer, and runs applications.(((operating system)))
Antivirus programs protect your computer system from viruses or malicious programs. This is an example of which type of software?(((utilities)))
System software includes all of the following except:(((browsers)))
Which of the following computers is also referred to as a server and is used to support end users for things such as retrieving data from a database or providing access to application software?(((midrange)))
Desktop, laptop, and tablet computers, and mobile devices are classified as ________.(((personal computers)))
The most widely used handheld computers are ________.(((smartphones)))
A microcomputer that is smaller, lighter, and less powerful than a notebook, and which has a touch-sensitive screen, is called a ________.(((tablet)))
This computer component holds the program and data that is currently being processed.(((RAM)))
Which of the following will not retain data after the computer is turned off?(((RAM)))
Identify the four basic categories of hardware in a personal computer system.(((system unit, input/output, secondary storage, and communication)))
This hardware houses most of the electronic components that make up a computer system.(((system unit)))
Which of the following are two important components of the system unit?(((microprocessor and memory)))
Which one of the following is not considered a form of secondary storage?(((RAM)))
________ are the most important part of any information system.(((People)))
The type of media that uses laser technology to store data and programs is ________.(((optical disc)))
Which of the following is not an example of a document file created by a word processor?(((spreadsheet)))
A financial budget would typically be created in what type of data file?(((worksheet)))
Employee records, including names, social security numbers, and related information, would typically be stored in what type of data file?(((database)))
The Internet is the largest ________ in the world.(((network)))
Which technology allows users to shift their desktop computing activities to computers on the Internet?(((cloud computing)))
________ are all about making people (end users) more productive.(((Personal computers)))
In an information system, the rules or guidelines for people to follow are called ________.(((procedures)))
________ is the component of the information system that is described as the equipment that processes data to create information.(((Hardware)))
The component of the information system that is described as raw, unprocessed facts, including text, numbers, images, and sounds, is called ________.(((data)))
________ software is a collection of programs that helps the computer manage its own resources and enables application software to interact with computer hardware.(((System)))
The ________ is the continuing development of the Internet that allows everyday objects embedded with electronic devices to send and receive data over the Internet.(((Internet of Things)))
Application software can be categorized as either general-purpose, specialized, or ________ apps.(((mobile)))
Software is another name for ________.(((programs)))
________ computers are not nearly as powerful as supercomputers, but are capable of great processing speeds and data storage and occupy specially wired, air-conditioned rooms.(((Mainframe)))
The five types of personal computers are: desktops, laptops, tablets, smartphones, and ________.(((wearables)))
Smartphones are the most widely used ________ computers and are designed to fit into the palm of one hand.(((handheld)))
________ are cell phones with wireless connections to the Internet.(((Smartphones)))
________ storage devices do not lose data or programs after the electrical power is turned off.(((Secondary)))
________ use(s) rigid metallic platters to store programs and very large data files.(((Hard disks)))
________ are secondary storage devices that use laser technology.(((Optical disks)))
A(n) ________ file might contain audience handouts and speaker notes in addition to electronic slides.(((presentation)))
Increased use of mobile or wireless communication devices and cloud computing has been termed "the ________ revolution."(((wireless)))
A(n) ________ is a communications system connecting two or more computers.(((network)))
The ________ provides a multimedia interface to the numerous resources available on the Internet.(((World Wide Web or "the web")))
________ computing uses the Internet and the web to shift many computer activities from a user's computer to computers on the Internet.(((Cloud)))
The parts of an information system are: people, procedures, software, hardware, data, and the Internet.(((TRUE)))
The purpose of software is to convert data into information.(((TRUE)))
Software is another name for a program or programs.(((TRUE)))
To be an effective and efficient computer user, you need to recognize the role of information technology in your personal and professional life.(((TRUE)))
Application software is "background" software which helps the computer manage its own internal resources.(((FALSE)))
System software is a collection of programs.(((TRUE)))
Antivirus software is a Windows utility that is unnecessary as long as you are careful browsing the Internet.(((FALSE)))
System software might be described as end-user software.(((FALSE)))
A word processor is an example of system software.(((FALSE)))
Another name for a midrange computer is "server."(((TRUE)))
Insurance companies are most likely to use personal computers to process information about millions of policyholders.(((FALSE)))
Smartphones are a type of handheld computer that do not need input, output, processing, or storage.(((FALSE)))
Two important components of the system unit are the keyboard and the hard disk.(((FALSE)))
A system unit consists of the keyboard and the monitor.(((FALSE)))
RAM will retain data when the computer is turned off.(((FALSE)))
The most common input devices are the mouse and the keyboard.(((TRUE)))
Secondary storage will retain data when the computer is turned off.(((TRUE)))
Solid-state storage uses rigid metallic platters to store data using the magnetic charges on the disk's surface.(((FALSE)))
Four common types of files are document files, worksheet files, database files, and presentation files.(((TRUE)))
Highly structured and organized data is stored in database files.(((TRUE)))
Launched in 1969 as a U.S.-funded project that developed a national computer network, the Internet was initially called ________.(((ARPANET)))
The web was introduced in ________.(((1991)))
The Internet is a ________ made up of wires, cables, satellites, and rules for exchanging information between computers connected to the network.(((physical network)))
This version of the web evolved to support more dynamic content creation and social interaction.(((Web 2.0)))
The most common way to access the Internet is through ________.(((an Internet service provider (ISP))))
Which of the following is not a well-known web browser?(((Windows Explorer)))
A web page typically contains ________, which contains the formatting instructions for displaying the web page.(((Hypertext Markup Language (HTML))))
For browsers to connect to resources on the Internet, the location of the resources must be specified through addresses called ________.(((uniform resource locators (URLs))))
In the URL "http://www.simnetonline.com," what is the domain name?(((www.simnetonline.com)))
In the URL "http://www.simnetonline.com," ".com" is the ________.(((top-level domain (TLD))))
Which programming language is used within HTML documents to trigger interactive features?(((JavaScript)))
Which of the following are files inserted into an HTML document that control the appearance of web pages including layout, colors, and fonts?(((CSS)))
This type of email account is more widely used because it frees the user from installing and maintaining an email client on every computer used to access email.(((web-based)))
Apple's Mail and Microsoft's Outlook are examples of a ________.(((client-based email system)))
Twitter is an example of what type of site?(((microblog)))
________ typically broadcast live events and use streaming technology in which audio and video files are continuously downloaded to your computer while you are listening to and/or viewing the content.(((Webcasts)))
________ do not use streaming technology and are used to download media files to your computer.(((Podcasts)))
Which of the following is a website specially designed to allow visitors to use their browser to add, edit, or delete the site's content?(((wiki)))
These special programs continually look for new information and update the databases used by search services.(((spiders)))
The ________ compares your entry against its database and returns the result, a list of sites on that topic.(((search engine)))
This type of e-commerce often resembles the electronic version of the classified ads or an auction.(((C2C)))
Online banking is an example of what type of e-commerce?(((B2C)))
Which of the following is not one of the three basic components of cloud computing?(((wireless connectivity)))
In a(n) ________ sellers post descriptions of products at a website and buyers submit bids electronically.(((web auction)))
Parents can use this type of program to block access to selected websites.(((Filter)))
This method of file transfer has been used for decades and is frequently used for uploading changes to a website hosted by an Internet service provider.(((FTP)))
This method of file transfer distributes file transfers across many different computers.(((BitTorrent)))
The continuing development of the Internet that allows everyday objects embedded with electronic devices to send and receive data over the Internet.(((IoT)))
Google Chrome is an example of a web ________.(((browser)))
The protocol ________ is used for web traffic and is one of the most widely used Internet protocols.(((https)))
In http://www.simnetonline.com, the underlined section is the ________ name.(((domain)))
HTML stands for ________.(((Hypertext Markup Language)))
References and ________ connect to other documents or web pages containing related information.(((hyperlinks)))
Special web browsers, called ________ browsers, are designed to run on portable devices such as smartphones and tablets.(((mobile)))
________ email systems do not require an email program to be installed on your computer.(((Web-based)))
Unsolicited email is called ________.(((spam)))
________ messaging is the process of sending short electronic messages using a wireless network.(((Text)))
A(n) ________ publishes short sentences that only take a few seconds to write, rather than long stories or posts.(((microblog)))
The most popular microblogging site is ________.(((Twitter)))
A ________ is a website specially designed to allow visitors to use their browser to add, edit, or delete the site's content.(((wiki)))
Wiki comes from the Hawaiian word for ________.(((fast)))
A ________ engine is a specialized program that assists you in locating information on the web and the Internet.(((search)))
________ search engines focus on subject-specific websites and can save time by narrowing your search.(((Specialized)))
When evaluating a website's content, whether or not the information is up to date is considered part of the ________ element.(((currency)))
________ cash is the Internet's equivalent to traditional cash.(((Digital)))
________ computing provides access to software, programs, and data from anywhere through an Internet connection.(((Cloud)))
An Internet ________ is a collection of utility programs designed to maintain your security and privacy while you are on the web.(((security suite)))
Knowledge of HTML and CSS is considered essential for the job of a ________.(((webmaster)))
A ________ is an example of an IoT device.(((Fitbit)))
The Internet connects millions of computers and resources throughout the world via wires, cables, and satellite.(((TRUE)))
The Internet and the web are the same.(((FALSE)))
The Apple Health application is a Web 3.0 application.(((TRUE)))
In addition to an Internet connection, users need browser software to easily display web pages.(((TRUE)))
In order to connect to a website, the browser must know only the site's domain name.(((FALSE)))
JavaScript is a language often used within HTML documents to trigger interactive features.(((TRUE)))
Cascading Style Sheets (CSS) are files inserted into an HTML document that control the appearance of web pages including layout, colors, and fonts.(((TRUE)))
PHP is a language often used within HTML documents to improve a website's interactivity.(((TRUE)))
Client-based email accounts require a special program called an email client to be installed on your computer.(((TRUE)))
Attachments to email messages can be a document or an image.(((TRUE)))
CAN-SPAM requirements have proven to be more effective than spam blockers at preventing unwanted email.(((FALSE)))
Instant messaging allows text only.(((FALSE)))
Cloud computing is an example of a social networking site.(((FALSE)))
LinkedIn is a social networking site that you should visit if you are looking for professional employment.(((TRUE)))
After a webcast is completed, there are no files from the webcast left on your computer.(((TRUE)))
Wikis allow many individuals to edit the site's content.(((TRUE)))
All information found on the web through search tools can be considered as factual.(((FALSE)))
One of the greatest challenges for the growth of e-commerce is the development of fast, secure, and reliable payment methods for buying goods.(((TRUE)))
News feed is the first page you see after logging into a social networking site.(((TRUE)))
Cloud computing has the potential to free end users from owning, maintaining, and storing software and data.(((TRUE)))
Uploading is the process of copying a file from a website to your computer.(((FALSE)))
This type of software works with end users, application software, and computer hardware to handle the majority of technical details.(((system)))
This type of software can be described as end-user software and is used to accomplish a variety of tasks.(((application)))
________ provide additional information and request user input.(((Dialog boxes)))
Most general-purpose applications use a(n) ________ that displays graphical elements called icons to represent familiar objects.(((graphical user interface)))
General-purpose applications include all the following except:(((web authoring)))
This GUI uses interrelated ribbons, tabs, and galleries.(((ribbon)))
These organize commonly used commands into sets of related activities.(((ribbons)))
To create documents that consist primarily of text, you need this software.(((word processor)))
Which would be the most appropriate type of application to create, analyze, and forecast budgets?(((spreadsheet)))
The electronic equivalent of a file cabinet is a ________.(((database)))
Programs that combine a variety of visual objects to create attractive, visually interesting presentations are called:(((presentation software)))
Programs that allow you to mix text and graphics and focus on flexible page design and layout.(((desktop publishing)))
These are specialized graphics programs for editing or modifying digital photographs.(((image editors)))
Image editors edit images consisting of thousands of pixels that form this type of image.(((bitmap)))
This type of image is made up of geometric shapes or objects.(((vector)))
These programs are used to create and edit vector images.(((illustration programs)))
________ are used to edit videos to enhance quality and appearance.(((Video editors)))
Which of the following programs would be used to create a website?(((web authoring)))
Specialized and powerful programs, called ________, are typically used to create specialized commercial sites.(((web authoring programs)))
Web authoring programs that allow you to build a page without interacting directly with HTML code are known as ________.(((WYSIWYG editors)))
Add-on programs for a variety of mobile devices, including smartphones and tablets.(((mobile apps)))
________ make up the basic structure of a relational database with columns containing field data and rows containing record information.(((Tables)))
A website that provides access to specific mobile apps that can be downloaded either for a nominal fee or free of charge.(((App store)))
Which of the following is not a popular cloud suite or online office suite?(((Lotus Symphony)))
As opposed to general-purpose applications, which are widely used in nearly every discipline, ________ applications are more narrowly focused.(((specialized)))
________ software is end-user software designed to accomplish a variety of tasks.(((Application)))
In a Ribbon GUI, ________ are used to divide the ribbon into major activity areas.(((tabs)))
________ are used to create text-based documents.(((Word processors)))
________ organize, analyze, and graph numeric data such as budgets and financial reports.(((Spreadsheet programs)))
A ________ is a collection of related data—the electronic equivalent of a file cabinet.(((database)))
________ are used to quickly create a presentation by providing predesigned styles and layouts as well as suggested content based on the type you choose.(((Templates)))
________ are programs that combine a variety of visual objects to create attractive, visually interesting presentations.(((Presentation software)))
________ programs allow you to mix text and graphics to create publications of professional quality.(((Desktop publishing)))
Digital photographs consist of thousands of dots, or ________, that form images.(((pixels)))
________ images are made up of geometric shapes or objects.(((Vector)))
Creating a website is called web ________.(((authoring)))
________ apps are add-on programs for a variety of devices including smartphones and tablets.(((Mobile)))
A(n) ________ store is typically a website that provides access to specific mobile apps that can be downloaded either for a nominal fee or free of charge.(((app)))
Office suites are also known as ________ suites.(((productivity)))
Cloud suites are stored at a ________ on the Internet, rather than on your microcomputer.(((server)))
System software can be described as end-user software and is used to accomplish a variety of tasks.(((FALSE)))
Application software includes general-purpose applications, specialized applications, and mobile apps.(((TRUE)))
Microsoft Access is an example of general-purpose application software.(((TRUE)))
A window is a rectangular area that can contain a document, a program, or a message.(((TRUE)))
Microsoft Office uses an interface with traditional menus and toolbars.(((FALSE)))
In a Ribbon GUI, a gallery displays the additional options.(((TRUE)))
In a Ribbon GUI, tabs are used to divide the ribbon into major activity areas.(((TRUE)))
Microsoft Word cannot be used to create personalized web pages.(((FALSE)))
Microsoft Word can identify incomplete sentences, awkward wording, and incorrect punctuation.(((TRUE)))
Functions, formulas, charts, and what-if analysis are common features of database management systems.(((FALSE)))
The last worksheet in a workbook is often a summary of the preceding worksheets.(((FALSE)))
What-if analysis is a very powerful and simple tool to test the effects of different assumptions in a spreadsheet.(((TRUE)))
A database is a collection of related data.(((TRUE)))
In a database, primary keys are often used to link tables.(((TRUE)))
Worksheets are only used for limited types of applications in certain professions.(((FALSE)))
Presentation software is an excellent tool to communicate a message and to persuade people.(((TRUE)))
Desktop publishing programs focus on page design and layout and provide greater flexibility for this than word processors.(((TRUE)))
Image editors are used to create and edit vector images.(((FALSE)))
Illustration programs are specialized graphics programs for editing bitmap or raster images.(((FALSE)))
Web authoring programs are used to create sophisticated commercial websites.(((TRUE)))
With faster data speeds and higher-resolution screens, smartphones and tablets are becoming a favorite way to watch TV shows, movies, or video clips.(((TRUE)))
Cloud suites are stored on your hard drive and are available anywhere you can access the Internet.(((FALSE)))
Many apps are written for a particular type of mobile device and will not run on other types.(((TRUE)))
Which of the following is not a system software program type?(((storage)))
This system software is responsible for managing your computer's resources including memory, processing, and storage.(((operating system)))
Which of the following is not a function of the operating system?(((providing Internet access)))
In many application programs, this feature is used to provide information or request input.(((dialog boxes)))
This type of boot occurs when the computer is already on and you restart it without turning off the power.(((warm boot)))
Most operating systems support the ability to switch between different applications. This is called ________.(((multitasking)))
In a network operating system, this computer coordinates all communication between computers.(((server)))
The category of operating system used for handheld devices.(((embedded)))
The operating system is often referred to as the software environment or software ________.(((platform)))
The most widely used personal computer operating system.(((Windows)))
This version of Mac OS X included new advanced utilities, improved integration with iOS devices, and better integration with cloud computing.(((Sierra)))
This operating system is widely used by servers on the web, mainframe computers, and very powerful personal computers.(((UNIX)))
This operating system is designed to run only with Apple computers.(((macOS)))
This version of UNIX is an open source operating system that is an alternative to Windows.(((Linux)))
This type of embedded operating system is used by smartphones.(((mobile)))
In a virtualized environment, this operating system runs on the physical machine.(((host)))
Which of the following is not a popular desktop operating system?(((Android)))
Which of the following is not a popular mobile operating system?(((UNIX)))
When a single physical computer runs this type of software, it operates as though it were two or more separate and independent computers.(((virtualization)))
The windows troubleshooting utility that identifies and eliminates nonessential files is called ________.(((Disk Cleanup)))
________ programs reduce the size of files so they require less storage space and can be sent more efficiently over the Internet.(((File compression)))
Before sending a large file over the Internet, you might want to use one of these programs.(((File compression program)))
A computer can become infected with viruses in all the following ways, except by ________.(((creating a new folder on your computer)))
A utility program that makes a copy of all files or selected files that are in the libraries, contacts, and favorites and on the desktop.(((File History)))
________ software consists of operating systems, utilities, device drivers, and language translators.(((System)))
A(n) ________ converts the programming instructions written by programmers into a language that the computer understands and processes.(((language translator)))
The ________ system coordinates computer resources, provides the user interface, and runs applications.(((operating)))
________ are graphic representations for a program, type of file, or function.(((Icons)))
Smartphone operating systems are a type of ________ operating system.(((real-time)))
________ operating systems are used to control and coordinate computers that are linked together.(((Network)))
Windows Server is an example of a(n) ________ operating system.(((network)))
________ operating systems control a single desktop or laptop computer.(((Stand-alone)))
The ________ is often referred to as the software environment or software platform.(((operating system)))
________ is an operating system widely used by servers on the web, mainframe computers, and very powerful personal computers.(((UNIX)))
________ refers to the process whereby a single physical computer can support multiple operating systems.(((Virtualization)))
Troubleshooting or ________ programs recognize and correct problems.(((diagnostic)))
Disk Cleanup, File History, and Disk Defragmenter are all examples of ________ programs.(((utility)))
________ programs reduce the size of files so they require less storage space.(((File compression)))
The ________ utility program identifies and eliminates nonessential files.(((Disk Cleanup)))
________ programs make copies of files that are in the libraries, contacts, and favorites and on the desktop to be used in case the originals are lost or damaged.(((File history)))
In a virtualized environment, the operating system of each virtual machine is known as the ________ operating system.(((guest)))
________ programs guard your computer system against viruses or other damaging programs that can invade your computer system.(((Antivirus)))
System software is a single program.(((FALSE)))
System software cannot handle technical details without user intervention.(((FALSE)))
Starting or restarting a computer is called tasking the system.(((FALSE)))
The operating system allows the user to interact with the application programs and computer hardware through a user interface.(((TRUE)))
In a network environment, the network server computer coordinates all communications between the other computers.(((TRUE)))
A standard desktop computer comes with an embedded operating system.(((FALSE)))
The operating system is often referred to as the software platform.(((TRUE)))
Smartphones generally include a network operating system.(((FALSE)))
Because Mac OS X market share is so large, more application programs are developed to run under Mac OS X than any other operating system.(((FALSE)))
Linux is a popular and powerful alternative to the Windows operating system.(((TRUE)))
When a single physical computer runs a special program known as virtualization software, it operates as though it were two or more separate and independent computers, known as virtual machines.(((TRUE)))
Google's Chrome OS integrates with web servers using cloud computing and storage to run applications and to perform other functions.(((TRUE)))
Based on the macOS, Android is designed for Apple's iPhone and iPad.(((FALSE)))
Programs such as word processors and spreadsheets are known as utilities.(((FALSE)))
Running File History improves system performance.(((FALSE)))
Most operating systems include utilities to organize and view your hard drive usage to identify old or unused files.(((TRUE)))
There are several ways in which you can "catch" a computer virus.(((TRUE)))
Utility suites come with one program only.(((FALSE)))
files are typically used to store students records, including names, ID numbers, and other related information(((Database)))
Which of the following behaviors will help you get the most out of a distance learning course? Click all that apply.(((اختار ثلاث خيارات Check in frequently - Have a plan in case you cannot use your computer - Find a cyber-buddy)))
In your educational life, asking questions, summarizing information, and expressing ideas are an important part of the learning process. Which communication skill would you be using for each of these activities?(((Speaking)))
Which of the following computers is most widely used type of computers.?(((Personal Computer)))
Which of the following will enhance and reinforce your listening skills?(((Asking questions to clarify points)))
Which of the following are tips to help you get the most from a search on the Internet? (SELECT ALL THAT APPLY) (((اختار ثلاث خيارات type different search terms into a new Window + determine what it is that you want answered + Use your favorite search engine)))
Which of the following are examples of poor netiquette? (Select all that apply) (((اختار ثلاث خيارات checking your phone while someone is talking + posting to your discussion board without proofreading + using headphones connected to your laptop)))
Which of the following are tips to deal with spam? Select all that apply. (( اختار ثلاث خيارات Frequently change your password + Never respond to e-mails asking for personal or financial information + Consider using two e-mail addresses ))
Application software does not include:(((Utilities)))
Application software are also described as end-user software.(((True )))
Which of the following is the reason why physical distractions are usually easier to prevent in a listening or speaking situation?(((The surroundings of a listening or speaking situation can often be changed.)))
The parts of an information system are: people, procedures, software, hardware, data, and the web.(((FALSE)))
A presentation software is not an example of system software.(((TRUE)))
Application software is the software which helps the computer manage its own internal resources.(((FALSE)))
According to the psychologist Abraham Maslow, self-actualizing needs are met through a:(((sense of achievement, competence, and creativity)))
Smartphones are a type of handheld computer that have input and output.(((FALSE)))
During classes, asking questions and expressing ideas are an important part of the learning process. Which communication skill would you be using for each of these activities?(((Speaking)))
The purpose of software is to convert information into data.(((FALSE)))
The component of the information system that is described as raw, unprocessed facts, is called ________.(((Date)))
When evaluating a website's content, whether the author have a bias is considered part of the ________ element.(((Onjectivity)))
______ are special programs continually look for new information and update the databases used by search services.(((Spiders)))
Which of the following communication skills are used when you listen to instructions and ask questions for clarification?(((Listening and speaking)))
The ability to clearly define a set of criteria for analyzing ideas is a quality of a(((Critical thinker)))
When taking an online test, you should avoid(((Taking the test on the due date.)))
Most general-purpose applications use a(n) that displays graphical elements called icons to represent familiar objects.(((Graphical user interface)))
A(n) _______converts the programming instructions written by programmers into a language that the computer understands and processes.(((Language translator)))
Specialized devices with a large display connected to a computer projector and are widely used in classrooms and corporate boardrooms.(((Digital or interactive whiteboards)))
With these systems, input and output devices are located outside the system unit.(((Desktop systems)))
An external hard disk ____.(((Usually connects to a USB or Thunderbolt port)))
_______ are used to store data and programs.(((Files)))
The statement All instructors are absent-minded intellectuals is an example of(((Stereotyping.)))
Most operating systems support the ability to switch between different applications. This is called _______.(((Multitasking)))
The most widely used audio-input device is the _______.(((Microphone)))
Smartphones use which of these storage technologies?(((Flash memory card)))
To create documents that consist primarily of text, you need this software(((Word processor)))
Type of keyboard that uses a touch screen as the input device.(((Virtual)))
Solid-state storage devices are different than hard disks in that they ________.(((Have no moving parts)))
Pat and his group are discussing a political problem in class and must offer possible solutions. He has his ideas on how to approach the problem but listens intently to what his group members have to say before he offers his approach. This is an example of(((Listening to all sides of an issue.)))
Shelly arrived in class early to do a quick review of her notes before her exam. Although she is confident that she knows the material, she has a lot of test anxiety. While she reviews, it will be most important for her to(((Intend to be positive.)))
_________ ,also known as mini notebooks, are lighter and thinner with longer battery life than most laptops(((Ultrabooks)))
In the first week of class Ed impressed his classmates with his critical and creative thinking on the discussion topics. Since then, Ed has been asked to be involved in several unrelated projects. This is an example of(((The halo effect.)))
Greg got his math test back, and although he had the correct answers, he did not get full credit. This was most likely because he did not(((Follow the instructions completely.)))
Specialized and powerful programs, called __________, are typically used to create specialized commercial sites. (((Web authoring programs)))
Typically a personal computer uses a(n) to store the operating system and software applications.(((Internal hard disk)))
Karl is faced with a plumbing problem he must solve. It would be helpful to begin to solve the problem by(((Defining the known and unknown.)))
______-input devices convert sounds into a form that can be processed by the system unit.(((Audio)))
As soon as you get the test, you should(((Write down key information you want to remember.)))
Disk Cleanup, File History, and Disk Defragmenter are all examples of programs.(((Utility)))
The most widely used audio-output devices.(((Speakers and headphones)))
The ________ is a container that houses most of the electronic components that make up a computer system.(((System unit)))
Jaqueline has worked on solving the same statistics problem for an hour and is frustrated that she cannot solve it. Right now what she needs most is(((A break)))
This type of computer is a thin slab that is almost all monitor with the system unit located behind the monitor.(((Tablet)))
Specialized and powerful programs, called _________,are typically used to create specialized commercial sites.(((Web authoring programs)))
______ are used on smartphones and other small portable devices.(((Virtual keyboards)))
The _________, also known as the system chassis, is a container that houses most of the electronic components that make up a computer system.(((System unit)))
The operating system is often referred to as the software environment or software _________.(((Platform)))
This type of mouse emits and senses light to detect mouse movement.(((Optical)))
Storage that retains its data after the power is turned off is referred to as ________.(((Nonvolatile storage)))
The electronic equivalent of a file cabinet is a _______.(((Database)))
Which of the following is not an example of secondary storage?(((RAM)))
These discs have a capacity of 50 gigabytes on one side.(((Blu-ray)))
A dedicated, mobile device for storing and displaying e-books and other electronic media including electronic newspapers and magazines.(((E-book reader)))
____________ are used on smartphones and other small portable devices.(((Virtual keyboards)))
A(n) _________hard disk is typically connected to a USB or Thunderbolt port.(((External)))
Kelsey is a film editor and loves what she does for a living. Often she will run into glitches or changes that require her to rework scenes and layers of soundtracks several times. Her greatest strength contributing to her successful career is most likely(((Persistence.)))
The hi-def largest capacity format for optical discs is the disc.(((Blu-ray)))
____ software is end-user software designed to accomplish a variety of tasks.(((Application)))
Shaimaa is confident that she knows the material to today's exam very well but she has a lot of test anxiety. She arrived in class early to do a quick review of her notes before the exam. While she reviews, it will be most important for her to(((intend to be positive)))
Ali and his team are discussing an important problem and trying to offer possible solutions. He has his ideas on how to approach the problem but listens intently to what his group members have to say before he offers his approach. This is an example of(((listening to all sides of an issue)))
CD-ROM is an example of a(n) ________ type of storage medium.(((optical disc)))
Hussein has worked on solving the same programming problem for an hour and is frustrated that he cannot solve it. Right now what he needs most is(((a break)))
Saying that all women are bad drivers, is an example of:(((stereotyping.)))
To create files that consist primarily of numeric data, you need this software.(((spreadsheet)))
Disk Defragmenter is an example of ________ program.(((utility)))
________ are lighter and thinner with longer battery life than most laptops.(((Ultrabooks)))
General-purpose applications include:(((database management systems)))
The category of operating system used for single laptop computer.(((Stand-alone)))
All men are mortal. Harold is a man. Therefore, Harold is mortal. This is an example of((Deductive reasoning))
The main purpose for analyzing and assessing your graded exam is to((Confirm your grade, determine common mistakes and patterns, and learn what to do differently next time. ))
Jaqueline has worked on solving the same statistics problem for an hour and is frustrated that she cannot solve it. Right now what she needs most is ((A break))
Partial copying from one of your classmates is considered as plagiarism ((TRUE))
Pat and his group are discussing a political problem in class and must offer possible solutions. He has his ideas on how to approach the problem but listens intently to what his group members have to say before he offers his approach. This is an example of : ((Listening to all sides of an issue))
Which of the following is NOT a part of an information system? ((Procedures))
Which of the following is system software?((Operating systems))
Mainframe computers are the most widely used type of computers,((TRUE))
_____ are created by database management programs to contain organized data Select one:((Database files))
For browsers to connect to resources on the Internet, the location of the resources must be specified through addresses called:((Uniform Resource Locators (URLs)))
The network that connects computers over the world is the:((Internet))
Acrobat Reader and Media Player are classified as plug-Ins utilities.((FALSE))
A _____ publishes short sentences that only take a few seconds to write, rather than long stories or posts.((Microblog))
Bing and Yahoo are examples of:((search engines))
Is an example of an IoT device that keeps track of human's health data?((Fitbit))
The two basic types of e-mail systems are:((Client-based and web-based systems.))
Meeting your tutor and colleagues in the class is a valuable meeting, therefore during the class:((b)Take notes and be focused.))
To control e-mail spams, it is good to:((Use two e-mail addresses; one for personal use and another one for online shopping.))
With distance learning education you can take classes offered only in the country where you live.((FALSE))
To get the most out of a distance learning course it is important to find an online partner.((FALSE))
The exchange of information through the use of written words and sentences is called ((Written communication))
Which of the following is NOT a basic component of communication?((l-attitude))
Amani's understanding of an instruction from his teacher is hampered because of the loud music in the next classroom. Which type of barrier to communication does represent?((Physical distraction.))
According to the psychologist Abraham Maslow, security needs are met through a:((Sense of achievement, competence, and creativity.))
Mr. Ali is informing his class about the time of the makeup lecture by saying: "To help you get the best out of the day, we will start the lecture at 8:00 a.m.". This is an example on:((Being a good listener))
Knowledge, comprehension, application, analysis, synthesis, and evaluation are levels associated with Bloom's Taxonomy .((TRUE))
When considering problem-solving steps, which step involves finding a strategy?((Define the problem))
A critical thinker needs to see all sides of the issue.((TRUE))
Deductive reasoning is defined by drawing conclusions based on going from the general to the specific,((FALSE))
Studying one night before the exam is good way to overcome Math and Science((FALSE))
Which of the following is the correct order of building better test habits' cycle?((Routine, Reward, Triger))
Predicting questions with your study team is a useful strategy that you need to consider before the exam.((TRUE))
Which of the following is NOT an exam question type?((PNG.))
When you use someone else's ideas in your writing without acknowledging the source, this act is called:((Plagiarism))
Acrobat Reader and Media Player are classified as plug-Ins utilities((FALSE))
The keyboard, mouse, display, and system unit are:((hardware))
Programs that coordinate computer resources, provide an interface, and run applications are known as:((operating systems))
Although not as powerful as a supercomputer, this type of computer is capable of great processing speeds and data storage.((mainframe))
Apple's watch is what type of computer?((wearable))
The type of file created by word processors, for example, memos, term papers, and letters ((document))
Uses the Internet and the web to shift many computer activities from a user's computer to computers on the Internet.((cloud computing))
The largest network in the world is [the]:((Internet))
The most popular internet activity((4.Communicating))
Maintain huge databases relating to information provided on the Web and the Internet.((2.Search))
Type of e-mail account that does not require an e-mail program to be installed on a user's computer is:((2.web-based))
The rules for exchanging data between computers.((4.Protocols))
The purpose of software is to convert data into:((Information))
The least powerful type of computers but most widely:((Personal computer))
The continuing Internet development that allows objects to send and receive data over the Internet. → IoT((TRUE))
Communication without words is called: Select one: ((Nonverbal Communication))
A communication that occurs between a tutor and a student is an example of: Select one:((Interpersonal communication.))
A possible good strategy to follow when solving math problems is to translate words into equations. Select one: ((TRUE))
File Transfer Utilities help you upload and download files to and from the Internet. ((TRUE))
With distance learning education you can become a better writer.((TRUE))
When you receive your graded exam it is important to analyze and asses it with your study team.((TRUE))
A critical thinker needs to have a positive attitude.((TRUE))
Another name for a midrange computer is "server."((TRUE))
One of the classroom netiquettes is to:((Turn off your mobile,))
Identify the four basic categories of hardware in a personal computer system.((system unit, input/output, secondary storage, and communication))
Which of the following is not a well-known web browser?((Windows Explorer))
Which part of an information system consists of the rules or guidelines for people to follow?((Procedures))
Which of the following computers is also referred to as a server and is used to support end users for things such as retrieving data from a database or providing access to application software?((Midrange ))
When evaluating a website's content, whether or not the information is up to date is considered part of the ________ element. ((Currency))
These special programs continually look for new information and update the databases used by search services.((Spiders))
Employee records, including names, social security numbers, and related information, would typically be stored in what type of data file?((Database))
In most cases, the word "software" is interchangeable with the word ________.((Program))
In the URL "http://www.simnetonline.com," what is the domain name?(( www.simnetonline.com))
Launched in 1969 as a U.S.-funded project that developed a national computer network, the Internet was initially called ________.((ARPANET))
Oral communication is the use of spoken words to exchange ideas and information.((TRUE))
Which of the following is NOT a basic component of communication?((I-attitude))
The you-attitude involves putting your reader or listener first and being considerate of the other person.((TRUE))
When you write a formal e-mail message, it is important to keep which of the following in mind?((make sure the recipient knows who you are))
Courses that combine traditional, face-to-face and online methods are referred to as: ((Blended courses.))
According to the psychologist Abraham Maslow, self-actualizing needs are met through a:((Sense of achievement, competence, and creativity.))
Which type of communication provides proof that information was exchanged?((Written communication))
What is one advantage of distance learning as a way of taking a class?((It allows you to acquire some basic technological skills.))
If you have an important point to make in an e-mail, it is a good idea to use all capital letters.((FALSE))
Miscommunication occurs when components of the intended message, actual message, or interpreted message are different for either the sender or the receiver.((TRUE))
Which of the following is not a required part of an information system? ((People))
The physical equipment that processes data in order to create information is called :((Hardware))
The word "software" is interchangeable with the word((Program))
What are the two major software types?((System and application))
Which of the following is a collection of programs rather than a single program?((Procedures))
Background software that allows the computers to work is called((system software))
Identify the program that coordinates computer resources, provides an interface between users and the computer, and runs applications((Operating system))
Which program, among the following, allows a user to eliminate unwanted file fragments and maximize on optimizing computer operations?((System software))
System software would include all of the following except ((desktop publishing))
Desktop, notebook, tablet PC, and handheld computers would be classified as:((microcomputers))
Identify the four basic categories in a typical hardware unit.((System unit, input/output, secondary storage, and communication.))
The most common output device would be a((Monitor))
Which of the following is not an example of a document file? ((a Spreadsheet))
Which among the following allows the record of highly structured and organized data?((Database files))
The capability of your microcomputer to share information with other computers is referred to as((Connectivity))
The _____ can be correlated to a giant highway that connects one to millions of other people and organizations located throughout the world.((Internet))
The Web was introduced in 1992 at((Center for European Nuclear Research (CERN).))
This enabled graphics, animation, sound, and video to be added to the Internet.((Web))
The Internet represents a _____.((Virtual network.))
These types of commercial Internet service providers are the most widely used.((National service providers.))
URL is an acronym for((Uniform Resource Locator.))
The two basic parts of a URL are((b The protocol and the domain name.))
Rules for exchanging data between computers are called:((Protocols.))
The domain name is the name of the _____ where the resource is located.((Server))
The extensions .gov, .edu, .mil, and, .net are called((Top-level domains.))
In the Web address "justincase@usource.org", what is the domain name?((usource.org))
Which among the following contains additional parts specifying directory paths, file names and pointers? ((File name))
Programs that automatically submit your search request to several search engines((metasearch engines))
Which among the following can be classified as being both Internet services and browser related programs?((с) Search engines))
Programs that automatically start and operate as a part of your browser are called((plug-ins.))
This type of software can be described as end user software and is used to accomplish a variety of tasks.((Application software))
If you need to create documents that consist primarily of text, you need this software.((Word processor))
The physical equipment that processes data in order to create information is called the ((hardware.))
The purpose of this software program is to manipulate numeric data.((a Spreadsheet))
Connections to other documents or to other locations within a Web site are((hyperlinks.))
In a computer system, data and programs are stored as Ambition is the path to success. Persistence is the vehicle you arrive in.((files.))
Which of the following is not true about cheating?((Cheating gives you confidence.))
In the first week of class Ed impressed his classmates with his critical and creative thinking on the discussion topics. Since then, Ed has been asked to be involved in several unrelated projects. This is an example of ?((al The halo effect))
Peak performers know that the best strategy for alleviating feelings of panic is to((Be prepared.))
To overcome math or science anxiety, it is important to(( Make problems physical and act them out if necessary. ))
Which of the following is the most popular social networking site? ((FaceBook))
In your educational life, wring reports and participating in discussion forums are an important part of the learning process. Which communication skill would you be using for each of these activities? (( Writing ))
What is true of wholesalers? (((Wholesalers are extremely important because of the marketing activities they perform.)))
Many service providers are considered retailers because they (((provide their services directly to consumers)))
A merger occurs when (((two companies combine to form a new company)))
A(n) ________ is a partnership established for a specific project or for a limited time. (((joint venture)))
An entrepreneur has been primarily associated with the willingness to((take risks.))
What is one of the difficulties faced by small business owners?((worries about employee problems))
What is a difference between high technology businesses and other small businesses?((High technology businesses require greater capital and have higher initial startup costs thanother small businesses))
What is true of a capitalist economic system?((Prices of goods and services are determined by supply and demand))
The quantity of goods and services that consumers are willing to buy at different prices at a specific time is referred to as((demand.))
The quantity of products that businesses are willing to sell at different prices at a specific time is referred to as((supply.))
What is a defining characteristic of public corporations?((Their stock can be bought, sold, or traded by anyone))
Which business provides a service, but is neither owned by the government nor focuses on earning profits?((a nonprofit corporation))
What is a true statement about the board members of a corporation?((They have a duty of care and loyalty to oversee the management of the firm.))
Preferred stockholders of a corporation((have a claim to profits before other stockholders do.))
In your educational life, wring reports and participating in discussion forums are an important part of the learning process. Which communication skill would you be using for each of these activities? ((Writing))
_______ allows users to shift their desktop computing activities to computers on the Internet. (( Cloud Computing ))
________ are destructive programs are usually attached to some emails. ((Computer Viruses))
Highly structured and organized data is stored in presentation files. ((False))
According to the psychologist Abraham Maslow, esteem needs are satisfied ((through feelings of self-importance or prestige.))
Which of the following computers is most powerful type of computers.? ((SuperComputers))
_______ files are typically used to store students records, including names, ID numbers, and other related information. ((Database))
Which communication skill broadens your knowledge and understanding of the world and actually makes you a more interesting person in your social life? ((Reading))
A teacher's lecture would typically be created in what type of data file? ((presentation))
Which of the following statements is true about the different types of communication? ((In written communication, immediate feedback may not always be possible.))
Laptops are classified as ________. ((Personal Computer))
In the first week of class Ahmad impressed his classmates with his critical and creative thinking on the discussion topics. Since then, Ahmad has been asked to be involved in several unrelated projects. This is an example of: ((the halo effect.))
Accordion theory suggests that retail institutions go from outlets with specialized merchandises to wide assortment and stops.((((True)))
The retail sector plays a key role in developed economies.((((True)))
Retailers do not perform traditional business activities.((((False)))
Climate change, pollution control, energy conservation, environmentally friendly ingredients and internal strategy are the main environmental factors affecting a retailer.((((True)))
A candy store specializes in selling a wide variety of candy. It even sells brands that are hard to get elsewhere. This is an example of an activity performed by retailers to _____________((((create value)))
Why is it imperative that manufacturers like HP and Procter and Gamble understand the operations of retailers?((((To encourage retailers to offer and promote their products)))
In Retail Sector, Customers, Competitors, Suppliers & Distributors together form the Macro Environment for that Business.((((False)))
The marketing environment consists of actor & forces that affect a company’s capability to operate effectively in providing products & services to its customers((((True)))
__________ describes the voluntary actions taken by a company to address the ethical, social, and environmental impacts of its business operations and the concerns of its stakeholders.((((Corporate social responsibility)))
The cash registers & laser scanners that can read a universal product code are known as____((((Electronic point-of-sale (EPOS) system)))
The differences between service and product are listed below EXCEPT of((((Flexibility)))
There are four main types of _____________ required to be a successful retail manager which are verbal and oral, written visual and non-verbal((((Communication)))
Transaction costs are costs incurred after selling the products and providing services to the government((((False)))
Aspects of visual merchandising that have most influence on consumers’ purchasing intentions are limited to color and presentation style((((False)))
Which of the following is NOT an example of political risk?((((Interest rates)))
The competition between the same type of retailers is called ______________.((((intertype competition)))
Individuals delivering services always perform in exactly the same way((((False)))
Different Products vary in _______________((((Value & price)))
Personalized services are services that are tailored to unique needs of each individual customer((((True)))
Shopping represents a basic form of consumer behavior, and retailers act as a mirror reflecting the multitude of consumer needs and desires.((((True)))
_____________ is based on the idea that individuals avoid work and without intervention by management the workforce resists, so managers need to persuade staff, reward, even punish them((((Theory X)))
Retailers are a key component in a supply chain that links manufacturers to consumers.((((True)))
Most effective way of measuring productivity is to calculate ratios of resource to results((((True)))
If a fall in price were anticipated, consumers might stockpile products to avoid the impact of lower prices((((False)))
There are three elements under ____________ forces which are demographics, culture and consumerism((((Social/cultural)))
Retailing offers entrepreneurial opportunities((((True)))
Culture sets the rules by which a society operates & applies to all of the members of the given society((((True)))
Demographic forces concern changes in products in terms of size & characteristics((((False)))
Which of the following businesses would NOT be considered a retailer?((((Procter and Gamble)))
There is an implicit, unwritten understanding between the employer and the employee that both parties have of each other through the ______________((((Psychological contract)))
Appraisal system is the process used to assess employees’ performance against selected criteria, including, job satisfaction, personal development & work-related performance((((True)))
Shrinkage increases retail costs & encourages sales((((False)))
A _____________ is a business that sells products and/or services to consumers for their personal or family use.((((retailer)))
The wheel of retailing theory states that new types of retailers begin as low-margin, low-price, and low-status operations but later evolve into higher-priced, higher-service operations, eventually become like the conventional retailers they replaced.((((True)))
Store manager’s performance is measured on ____________ skills((((Trading & management skills)))
Drycleaners, hair salons, and auto repair businesses are considered retailers.((((True)))
Which of the following terms refers to the competition between the same types of retailers?((((Intertype competition)))
Intermediaries are retailers which act as “middlemen” bringing Products produced in bulk by manufacturers to market so that consumers can buy smaller quantities((((True)))
Convenience store is a local store selling a limited variety of products open for longer hours than other retailers((((True)))
What is retailing?((((It deals with the sale of products meant for personal use.)))
Different implications of selling different Products are:((((a & c [ products vary in volume + in value and price])))
The following are examples of Economic Factors except:((((Trade restrictions and tariffs)))
___________ is a set of characteristics that identify socially acceptable patterns of behavior and social relationships within a given social group((((Culture)))
The key skills a manager needs to communicate effectively are((((All of the above)))
r is the only type of businessThe marketing environment consists of actors & forces that affect a company's capability to operate effectively in providing products & services to its customers ((TRUE))
Retailer is the only type of business to 'break bulk'(((False)))
Retailers are involved in CSR initiatives through____________((((all above)))
What is a retailer’s role in the supply chain?((((Linking manufacturers to consumers)))
A retailer’s primary competitors are other retailers that:((((use the same type of store format.)))
____________ form a part of microenvironment that influences retail management decisions.((((Competitors)))
Goods pass from manufacturer to the retailer then to wholesaler((((False)))
Competitive advantage is the achievement of superior performance. To be a success a business can create a performance differential by being;. ((((A combination of all the above))))
A buying center may contain five people performing a number of roles. The role of the buyer is: ((((The person who conducts the transaction))))
…………… provides a framework for a systematic examination of a business’s marketing environment. ((((Marketing audit))))
What is the first stage in marketing planning process ((((Where are we now?))))
Fishbein and Ajzen model suggests that an attitude towards a brand is based upon a set of beliefs about brand's attribute ((((True))))
In the consumer decision-making process the second step refers to:((((Information search))))
The model of the marketing planning process is useful as it provides a clearly defined path from the vision of what a business might be to the actual implementation. ((((True ) )))
In a High-involvement situation the consumer is not seeking info but is a passive receiver. ((((False))))
Clear focus on specialist niche market is one of the …………of SWOT analysis. ((((Strengths))))
Today retailers are brands themselves rather than distributors of manufacturer brands. ((((True ))))
............is useful for collecting information about customer buying habits through loyalty cards.((((Market research))))
.…………is the process by which companies distinguish their product offerings from competition. By developing a distinctive name, packaging & design, a brand is created. ((((Branding))))
………….. provides a framework for a systematic examination of a business’s marketing environment. ((((Marketing audit))))
…………..are non branding merchandise sold in plain packaging with low price positioning ((((Generics brands))))
According to Blackwell et al. (2005), one of the roles performed in a buying centre is the ………………….. who persuades others ((((Influencer))))
Brand equity is the core product augmented by additional distinctive features which add values & deliver specific benefits its to its users. ? (((( False))))
Clear focus on specialist niche market is one of the……………of SWOT analysis.((((Strengths))))
Competitive advantage is the achievement of superior performance. To be a success a business can create a performance differential by being:((((combination of all of the above ))))
Fishbein and Ajzen model suggests that an attitude towards a brand is based upon a set of beliefs about brand’s attributes. ? ((((True ))))
In ………………….. problem solving the consumer requires a great product information and makes a detailed examination of alternatives ((((Extended problem solving))))
In a High-involvement situation the consumer is not seeking info but is a passive receiver. (((( False))))
In the marketing orientation Companies focus on production methods before customer. ? ((((False))))
Market penetration means: ((((Selling existing products into existing markets))))
Market penetration takes existing products in existing markets((((True))))
Marketing audit is a process of constant information gathering which serves in achieving the SWOT analysis. ((((True) )))
Marketing is more than advertising, promotion & sales people ? ((((True ) )))
Production oriented companies get close to their customers so that they understand their needs and problems. ? (((( False))))
Survival and safety are considered……………type of motivation. ((((Physical motivations))))
SWOT analysis consists of all of the following elements except: ((((Organizations))))
The consumer of the product ((((The person who conducts the transaction))))
The following questions are the fundamental questions which form the starting point for the planning process EXCEPT; ((((Where were we 5 years ago?))))
The model of the marketing planning process is useful as it provides clearly defined path from the vision of what a business might be to the actual implementation. ((((True ))))
The personal influences affecting the consumer behavior are the following EXCEPT; ((((Peers))))
The possible attribute which affect store choice include the value for money, a convenient location, an ease of parking and low prices. ((((True ))))
Three types of buying situations can be identified, 1) Extended problem solving, 2) Limited problem solving and: ((((Habitual problem solving))))
Today retailers are brands themselves rather than distributors of manufacturer brands. ? ((((True ))))
Which of the following is not a role in a consumer buying center? ((((Motivator))))
Which of the following is not a role in a role in a consumer buying center? ((((Motivator))))
Which of the following stages are included in marketing planning process? ((((All of the above ))))
Which of the following stages are included in the marketing planning process ((((All of the above))))
Which of the following statement is correct?((((Marketing audit provides a framework for a systematic examination of a business’s marketing environment))))
Which one of the four statements below is correct? ((((A SWOT analysis is a method which examines the external and the internal environment of an organization.))))
In a high-involvement buying situation, marketing managers don’t need to provide a lot of information about the consequences of buying. ((False))
Awareness set is a set of brands that they seriously consider buying ((False))
The possible attributes which affect store choice include the value for money, a convenient location, an ease of parking and low prices. ((True))
the mission statement has proved to be enduring and has served to distinguish the business from its competitors ((True))
What is true of wholesalers? (((Wholesalers are extremely important because of the marketing activities they perform.)))
Many service providers are considered retailers because they (((provide their services directly to consumers)))
A merger occurs when (((two companies combine to form a new company)))
A(n) ________ is a partnership established for a specific project or for a limited time. (((joint venture)))
An entrepreneur has been primarily associated with the willingness to((take risks.))
What is one of the difficulties faced by small business owners?((worries about employee problems))
What is a difference between high technology businesses and other small businesses?((High technology businesses require greater capital and have higher initial startup costs thanother small businesses))
What is true of a capitalist economic system?((Prices of goods and services are determined by supply and demand))
The quantity of goods and services that consumers are willing to buy at different prices at a specific time is referred to as((demand.))
The quantity of products that businesses are willing to sell at different prices at a specific time is referred to as((supply.))
What is a defining characteristic of public corporations?((Their stock can be bought, sold, or traded by anyone))
Which business provides a service, but is neither owned by the government nor focuses on earning profits?((a nonprofit corporation))
What is a true statement about the board members of a corporation?((They have a duty of care and loyalty to oversee the management of the firm.))
Preferred stockholders of a corporation((have a claim to profits before other stockholders do.))
Retailing offers entrepreneurial opportunities. ((true))
Forward integration is expansion of a business so that it can distribute products directly to consumer. ((True))
Which is NOT a theory that explain patterns of retail development? (( Theories X & Y))
A company would enhance its corporate social responsibility image by ((lowering its energy consumption.))
Which of the following is considered as a Social Factor ((Consumer motivation and attitudes))
the forces from the micro environment has influence over both, the Retail Business and Macro Environment. ((False))
The set of business activities that add value to goods and services sold to the end consumer is referred to as ((retailing))
What is a retailer’s role in the supply chain? ((linking manufacturers to consumers))
What disciple do students view retailing as being a part of? (( marketing))
Political /legal forces influence retail management decisions by determining rules & responsibilities. ((True))
Retailers aim to make a profit by buying products in bulk & selling them in smaller quantities to individuals. ((True))
Disadvantage to buying online: ((Returning products that do not meet your needs))
Micro-environment: Actors in retailer's immediate environment that can affect business performance. ((True))
Low-involvement shopping: Routine -shopping can be efficient. Always go to same store in which they find all needs. Becoming familiar with layout so quickly ((True))
Managers role: organizing HR (staff) & physical aspects of the store (such as stock) in effective ways to meet strategic & tactical objectives. ((True))
Theory Y: individuals avoid work & without intervention by management, workforce resist. Managers persuade staff, reward, even punished ((False))
Habitual decision making ((involves little or no conscious effort.))
Market research: process of gathering data using techniques, e.g., questionnaire, interview or online survey, to produce information about current situation in a given market. Useful for collecting information about customer buying habits through loyalty cards( (true))
According to Blackwell et al. (2005), role performed in a buying center: ((All of the above))
Which of the following is NOT part of consumer decision making process: ((Return the product))
Which one of the following is a technical criteria that affect consumer choice: ((Reliability))
Branding: Process by which companies distinguish their Product offerings from competition, by developing a distinctive name, packaging & design a brand is created.((True))
Brand equity: measure of power & importance of a brand in marketplace based on value attributed to a brand by its customers & its assets ((True))
Dimensions affecting consumer choice of retail store: ((All of the above))
In-store atmosphere improves consumers' perceptions of merchandise quality of in store & is associated with higher prices ((True))
Consumers use non-price related cues like service offerings & quality levels to form price perceptions. ((True))
Jeffrey Pfeffer and Robert Sutton believed that evidence-based management is founded on the belief that facing the hard facts about what works and accepting the nonsense that passes for sound advice will help organizations perform better. (((FALSE)))
Peter Drucker was a 20th-century socialist, opposed capitalism and believed that managers were more of a hindrance than a help to most organizations. (((FALSE)))
Susan, a department manager, believes that ever since the corporate office eliminated certain benefits to reduce costs, her employees have not been working as hard as in the past. This reduction in effort is known as "soldiering." (((TRUE)))
Joe, a plant manager, wants to institute a compensation system in which more efficient workers earn higher wages. Joe is proposing to institute a differential rate system (((TRUE)))
The Hawthorne studies succeeded in drawing attention to the importance of "social man" and how managers using good human relations could improve worker productivity. (((TRUE)))
Olde Manufacturing, a parts supplier to the auto industry, has been using the behavioral approach to management for over 100 years. However, since that method of management is too simplistic for practical use, Olde should begin implementing the newer and more sophisticated human relations movement approach to management, which is the most contemporary approach to management. (((FALSE)))
Behavioral science relies on data from past sales to forecast human behavior to develop strategic plans and goals. (((FALSE)))
When the Americans entered World War II in 1941, they used the British model to form operations research teams to determine how to deploy troops, submarines, and equipment most effectively. These techniques have evolved into quantitative management. (((TRUE)))
The idea that two or more forces combined create an effect that is greater than the sum of their individual effects is called syncretism. (((FALSE)))
Right Office Equipment continually talks with customers after they purchase their products, and Right Office managers regularly visit customers to gather feedback on new products, product improvements, and how Right Office can better serve its customers. This is an example of an open system (((TRUE)))
When Mr. Jones, the owner of a local hardware store, asks one of his lawnmower suppliers about the total ability of their product to meet customer needs, he is questioning the quality of that mower. (((TRUE)))
A-Plus Manufacturing has a policy of using statistical sampling to locate errors by testing just some (rather than all) of the items in a particular production run. This strategy of minimizing errors is known as synergy. (((FALSE)))
A local furniture retailer routinely develops and acquires new knowledge, and the employees communicate what they learn to other employees, so the company is able to modify its behavior to reflect what they have learned. This organization is an example of a learning organization. (((TRUE)))
holds that there are few really new ideas and that "true is better" than new. (((Evidence-based management)))
Which three approaches to management are considered historical perspectives? (((classical, behavioral, quantitative)))
Within his company, Greg has set up a system with inputs, outputs, transformation processes, and feedback. He utilizes a management style that varies according to the individual and environmental situation, with a strategy for minimizing errors by managing each stage of production. Greg is utilizing a _______ perspective. (((contemporary)))
Which of the following is a key benefit of studying theoretical perspectives on management? (((They serve as a guide to action and a source of new ideas.)))
The classical viewpoint of management emphasized ways to (((manage work more efficiently.)))
Mateo, an IT manager, has been studying the actions that his workers perform in an attempt to improve their productivity. Mateo is utilizing (((scientific management.)))
Frank and Lillian Gilbreth identified 17 basic units of motion that they called (((therbligs.)))
Amy is a package delivery service manager. She is interested in implementing ___________, the management philosophy pioneered by Frederick W. Taylor and Frank and Lillian Gilbreth, which scientifically studies work methods to improve the productivity of individual workers. In this particular case, Amy will study the routes that drivers take through a congested to city in order to identify ways that delivery staff can make the most deliveries in each two-hour period. (((scientific management)))
Sal, a production manager, knows that some of his employees are upset with a new corporate policy that eliminates a tuition reimbursement program. As a result, some of these employees are participating in soldiering. To eliminate soldiering, Sal should (((carefully select workers according to their abilities and give workers training.)))
A pay structure in which more efficient workers earn higher wages, as suggested by Frederick W. Taylor, is known as a(n) _____ system. (((differential rate)))
_______ was the father of scientific management. (((Frederick Taylor)))
Tom, the manager of floral shop, is interested in implementing the principles of administrative management, which involves (((managing the total organization.)))
To better meet corporate goals this year, Donna, a CEO, is encouraging her managers to focus on the major functions of management, which are (((planning, organizing, leading, and controlling.)))
Because there are some positive aspects of bureaucracy, as proposed by Max Weber, Tammy, a restaurant manager, is interested in implementing certain aspects of the bureaucratic approach to management within her restaurant. Like Weber, Tammy feels bureaucracy is a (((rational, efficient, ideal organization based on principles of logic.)))
The management approach that emphasized ways to manage work more efficiently is the ___ approach. (((classical)))
The _______ approach to management focuses on scientific methods, time and motion studies, and job specialization with the goal of increasing productivity. (((classical)))
Greg, the IT manager, feels that his employees lack the motivation to consistently meet department goals. To better understand human actions and to motivate their employees toward achievement, Greg and his supervisors need to adopt the ____ viewpoint (((behavioral)))
Order the schools of thought in the behavioral viewpoint from earliest to most recent. (((early behaviorism, human relations movement, behavioral science)))
Hugo Munsterberg is known as (((the father of industrial psychology.)))
_______ suggested that organizations should be operated as "communities," with managers and subordinates working together in harmony. For example, in a publishing company run as a community, editors, editorial assistants, and publishers would work together to determine which books should be published. (((Mary Parker Follett)))
According to proponents of the Hawthorne effect, (((giving more attention to employees increases worker productivity, if they think managers care about them.)))
Abraham Maslow proposed the (((hierarchy of human needs.)))
Olivia, the general manager of a bowling center, believes that her workers are responsible and capable, and that they can be trusted when given responsibility. Olivia is an example of a(n) ___ manager. (((Theory Y)))
George, who owns a small grocery store, has a reputation in the community as a tough manager. Many customers have heard George yell at his employees because he feels that workers today are lazy, lack ambition, and hate to work. George is a(n) ____ manager. (((Theory X)))
Behavioral science relies on scientific research for developing theories about human behavior that can be (((used to provide practical tools for managers.)))
Which disciplines does the field of behavioral science include? (((psychology, sociology, anthropology, and economics)))
Management science stresses the use of rational, science-based techniques and mathematical models to improve (((decision making and strategic planning.)))
The new chief information officer of a national pizza chain is using mathematical tools to aid in product ordering and scheduling decisions. The CIO is using (((management science.)))
The three contemporary management perspectives are the ______ viewpoints. (((systems, contingency, and quality management)))
The systems viewpoint sees organizations as entities made up of four interrelated parts known as (((inputs, outputs, transformation processes, and feedback.)))
The systems viewpoint regards the various parts making up the whole system as (((subsystems.)))
When the sales of a certain brand of diet soda slowed, a national grocery retailer decided to drop the price of that product, which resulted in a large increase in sales of the soda. This sales increase is a type of (((feedback.)))
Right Motors calls its customers after they purchase an automobile from the dealership. In addition, every year customers are asked to complete a short survey about the car they purchased from Right Motors and their customer-service experiences with the dealership. Right Motors is an example of a(n) ___ system. (((open)))
The American economy consists of a complex combination of organizations and variables, such as countless companies differing in size and what they sell, current events, and markets (for example, the housing market and the stock market). According to ________, all of these independent factors interact with one another according to certain simple rules. (((complexity theory)))
Cheyenne is the manager of a local small hotel. Just today Cheyenne received word that a major convention will be coming to town next month, and the demand for hotel rooms is expected to skyrocket. In a conversation with the owner, she asked, "What should our approach to pricing be for the week of the convention? Should we require payment in full at the time of the reservation?" Which management method is Cheyenne using? (((the contingency viewpoint)))
When a manager assesses a particular situation and decides what to do according to the individual and environmental situation, the manager is utilizing the ____ viewpoint. (((contingency)))
Gary Hamel suggests we need to look at management as a process and then make improvements and (((ongoing and systematic.)))
The management of Prime Manufacturing is implementing a plan to minimize production mistakes by allowing teams that work in each area of the production facility to develop a plan and then monitor their area to ensure the reduction of errors. The managers are engaging in (((quality control.)))
The management of a facility that manufactures parts for car brakes has a policy of testing only some of the items in each production run to locate errors, with the ultimate goal of minimizing errors by managing each stage of production. This process is an example of the ______ technique. (((quality control)))
After returning home from a trade show, Mr. Jones, the president of a manufacturing company, called a meeting of his production department. He indicated that the company would soon be implementing a new policy of _______, which focuses on worker performance and urges employees to strive for zero defects. (((quality assurance)))
_______ believed that quality stemmed from "constancy of purpose," and that managers should stress teamwork, be helpful rather than simply give orders, and make employees feel comfortable about asking questions. (((W. Edwards Deming)))
Joseph Juran defined quality as "fitness for use," which means that (((Joseph Juran defined quality as "fitness for use," which means that )))
To gain a competitive edge this year, the upper management of a global IT company has decided to focus on customer service, employee training, and continuous quality improvement. This approach is known as (((total quality management (TQM).)))
Kiddie Village is a successful daycare center. It focuses on actively developing, finding, and communicating new knowledge to its employees so that workers can modify their behavior to reflect this new knowledge. Kiddie Village is an example of a(n) _______ organization (((learning)))
An organization in which the management builds a commitment to learning, works to generate ideas with impact, and works to generalize ideas with impact is creating a(n) (((learning organization.)))
Paul, who is normally an ethical person, has an opportunity to acquire what he feels is "easy" money in his job, and he believes it is safe to steal the money because nobody would know if he does take it. Paul rationalizes that it is fine to steal the money because he needs it more than the person whom the money belongs to. The psychological mechanism that causes Paul to want to steal the money is poor self-esteem. (((FALSE)))
Habitual cheating tends to begin with small infractions, such as illegally downloading books and songs, and then grows by increments into an ongoing deliberate strategy of deception or fraud. (((TRUE)))
Prior to a 2010 natural-gas pipeline explosion in San Bruno, California, Pacific Gas & Electric chose to use its surplus revenues to improve safety, an example of ethical decision making. (((FALSE)))
The employees of a production facility that produce parts for boats are meeting to discuss ways to control rising costs, which are affecting their bonus (incentive pay). The employees are internal stakeholders of the factory. (((TRUE)))
The general environment, or macroenvironment, includes six forces: economic, technological, sociocultural, demographic, political-legal, and international. (((TRUE)))
When the Kiddie Village Daycare Center looks at the recent increase in the number of working mothers in its neighborhood to determine how many employees to hire, its managers are studying demographic forces. (((TRUE)))
The Sarbanes-Oxley Act set up the Federal Trade Commission to ensure consumer protection and to eliminate and prevent anticompetitive business practices, with penalties of as much as 25 years in prison for noncompliance. (((FALSE)))
Companies can reduce unethical behavior by offering a case-based approach to ethics training that presents employees with ethical dilemmas and clarifies expectations. (((TRUE)))
Despite recent pressure from stockholders to increase profits, World Extraction Corp., a global petroleum organization, has maintained a practice of consistently acting ethically by being a good global corporate citizen, taking host-country and global standards into consideration when making decisions, and obeying the laws of host countries as well as international law. World Extraction Corp. provides an example of a company acting in a socially responsible way. (((TRUE)))
After the scandals at WorldCom, Tyco, Adelphia, and Enron, in which company employees lost millions of dollars and their nest eggs, U.S. companies showed less concern for corporate governance and more interest in the development of corporate social responsibility. (((FALSE)))
Shane, a sales manager, remarked that typically, her employees sincerely and positively overstate their job performance and abilities. This is an example of the ____ effect. (((holier-than-thou)))
Patsy, a registered nurse, has decided to start a home healthcare service to assist with the medical, meal preparation, and light housecleaning needs of older adults. To grow her business, Patsy is focusing her marketing efforts on the older adult population; in these efforts, she is involved with the _____ environment. (((general)))
Warm Hearts is a nonprofit organization that brings warm meals to older adults and runs errands for its clients; the board of Warm Hearts is called the (((board of regents.)))
Marie and Marti are partners who solely own M and M Florist. As owners, they can (((claim the organization as their legal property.)))
To focus on improving company customer service this year, the CEO of a pet food manufacturer outlined a list of stakeholders to concentrate on: distributors, local communities, and the mass media. These three groups are (((external stakeholders.)))
Rebecca and Donna, owners of Fine Coffee and Pastry Shop, are concerned because Panera Bread is opening a new store just one mile from their shop. Rebecca and Donna know that Panera will be offering similar products; therefore, Panera Bread will be a ____ to Rebecca and Donna. (((competitor)))
A supplier is an external stakeholder that (((provides raw materials, services, equipment, labor, or energy)))
Every Saturday Mrs. Jones and hundreds of other shoppers visit ABC Market, a grocery store, to buy food for their home or business. These shoppers are _____ to ABC. (((customers)))
Ole McDonald, a successful farm equipment supply company in the Midwest, has joined forces with A-Plus Supply, a national hardware and home retailer. As a result of this new relationship, Ole McDonald can open stores in other regions and A-Plus can now offer a high-quality product in the farm community. Ole McDonald and A-Plus Supply are now (((strategic allies.)))
Since the 1950s, the percentage of the U.S. labor force represented by unions has (((steadily declined.)))
Local communities are stakeholders of organizations because communities (((rely on the tax base from companies and the jobs the company provides.)))
The city government of Pleasantville promised significant tax breaks to a large retailer if it opened a store in Pleasantville, which would result in nearly 200 new local jobs. However, the retailer decided that it would staff the new Pleasantville store with employees from its other stores, meaning no new jobs were created. As a result, the city of Pleasantville instituted clawbacks by (((rescinding the tax breaks given to the retailer.)))
Which type of funding raises small amounts of money from many people to finance a new venture? (((crowdfunding)))
_______ are regulatory agencies, such as the SEC (Securities and Exchange Commission), that establish ground rules under which organizations may operate. (((Government regulators)))
Which forces are part of the macroenvironment? (((economic, technological, sociocultural, demographic, political-legal, and international)))
What is a synonym for the macroenvironment? (((general environment)))
Fewer children are interested in learning how to play a musical instrument, and young people having less interest in owning cars. These two trends are examples of ____ forces at work. (((sociocultural)))
When a food distributor considers changing the products it markets in the southern states because of an increasing percentage of customers from Central and South America, it is examining the ____ forces at work in the southern United States. (((demographic)))
_____ forces affect the way politics shape laws and the way laws shape the opportunities for and threats to an organization. For example, some cities in the United States (including New York and San Francisco) have set the minimum wage far above the federally mandated minimum wage. (((Political-legal)))
Ethics are (((standards of right and wrong that influence behavior.)))
To increase profitability, top management of a national retailer needs to decide if it will close several stores. Management knows that by closing the locations, it will save the company millions of dollars and benefit many stockholder groups and individuals. The benefits of closing the stores outweigh the benefits of keeping them open. Keeping the stores open would be advantageous for many fewer people than closing the stores. This logic is an example of the ___ approach to deciding ethical dilemmas (((utilitarian)))
______ guides the justice approach to deciding ethical dilemmas. (((Respect for impartial standards of fairness and equity)))
Bernard (Bernie) Madoff used money from newer investors to pay off older investors. Which type of scam was Madoff running? (((Ponzi scheme)))
The Sarbanes-Oxley Act of 2002 established requirements for ____. Noncompliance can result in penalties of (((proper financial record keeping for public companies; as much as 25 years in prison.)))
Sustainable economic development (((meets present needs without compromising the ability of future generations to meet their needs.)))
Business scholar Archie Carroll suggested that ________ is the first and primary responsibility of an organization in the global economy (((to make a profit consistent with expectations for international business)))
Which of the following is an example of a green policy or action? (((Coca-Cola has pledged to replenish (return to nature) all the water it uses to make its beverages.)))
Which of the following describes the system of corporate governance? (((governing a company so that the interests of corporate owners and other stakeholders are protected)))
Before you travel and conduct business abroad, spend some time learning about the way business is conducted in foreign countries. For example, in Asia and the Middle East, personal relationships are crucial to getting things done, so you should plan to engage in small talk and to avoid business talk during after-hours outings. (((TRUE)))
While in Italy, Tony was able to communicate important information to George, his business partner, via e-mail. Later that day, Tony remarked to George about how helpful it was that he was able to return to their office in New York in less than nine hours. The "global village" refers to this "shrinking" of time and space as air travel and the electronic media have made it easier for people around the globe to communicate with one another. (((TRUE)))
Susan, a teacher, is interested in helping children in other countries learn how to read. She has formed an international nonprofit organization, Read to the World, whose employees travel to Africa to donate books and work in schools as tutors. Read to the World is an example of a multinational corporation. (((FALSE)))
Greg, a manager at the Rome, Italy, store of an American-owned clothing company, understands the differences and similarities between American and Italian customers, customs, culture, and practices. As a result, Greg uses his understanding of Italian culture, tastes, and business practices to help him market his products to Italian customers in Rome—and he uses his understanding of American culture to sell his products to American tourists who are visiting Rome. Greg is an example of a geocentric manager. (((TRUE)))
Two managers from an Electronics Mart were talking to Ty, their district manager, about Harriet, the unpopular store manager. Susan, the assistant manager, said, "I believe I speak for other managers and employees when I say that Harriet acts like she knows it all. We all feel that Harriet does not listen to any of us because she always thinks her way is right and does not want to consider any other opinion." Harriet is an example of a geocentric or parochial manager. (((FALSE)))
Over the last two years, an American computer manufacturer has partnered with a technology organization in Japan to develop and produce computers and printers. The partnership benefits both organizations and is an example of countertrading. (((FALSE)))
There are two types of tariffs: a revenue tariff, designed simply to raise money for the government; and a protective tariff, which raises the price of imported goods to make the prices of domestic products more competitive. (((TRUE)))
NAFTA is a trading bloc consisting of the United States, Canada, Panama, and Mexico. (((FALSE)))
The "most favored nation" trading status describes a condition in which a country grants other countries favorable trading treatment, such as the reduction of import duties. (((TRUE)))
Dawn grew up helping her father work on cars in his auto repair shop and developed into an excellent auto technician. Because of stereotypes regarding women in this field, she is interested in finding a shop that minimizes gender discrimination and role inequalities. Dawn wants to work for a company that highly values gender egalitarianism. (((TRUE)))
Philippe is from the Mediterranean coast of Greece and Sasha is from Iran. In their grocery store, this married couple is accustomed to focusing on one task at a time rather than multitasking. Philippe and Sasha work on a system of polychronic time. (((FALSE)))
Which of the following is NOT a good piece of advice for those who travel for business? (((Technology-based meetings conducted through videoconferencing are often as effective as expensive, time-consuming domestic or business travel.)))
Globalization is the trend (((of the world economy toward becoming a more interdependent system.)))
Joe, who is from Michigan, is transferring to the Paris office. As one who has studied the effects of globalization on business, Joe understands that it crucial to (((understand the local culture.)))
Great systems of _____ are the hallmark of great civilizations. (((communications)))
According to Rosabeth Moss Kantor of the Harvard Business School, which three events of the 1980s paved the way for the global economy? (((The Berlin Wall came down, Asian countries opened their economies to foreign investors, and there was a worldwide trend of governments deregulating their economies.)))
Don, who owns an investment firm with customers worldwide, has witnessed how dangerous global economic interdependency can be. During a lunch meeting he mentioned to a customer, "In my opinion, ______________ have been two negative effects of global economic interdependency for the United States." (((other countries stealing U.S. technology and the loss of well-paying jobs in the United States)))
Technology philosopher Nicholas Negroponte proposed that the global market driven by electronic information ((("forces things to get bigger and smaller at the same time.")))
Tom, who owns a successful business with two locations and a few international clients, was approached by a large organization about dramatically expanding his company. Tom later told his wife that he is happy with his success, but he wants to stay small because if he decides to add new products, small companies (((can get started more easily and maneuver faster.)))
Hannah, an engineer for an oil company, is interested in working overseas at this stage of her life because she knows that foreign work experience demonstrates ____ to potential employers. (((independence, resourcefulness, and entrepreneurship)))
Tops Burger, a _____________ based in Amarillo, Texas, has restaurants in 25 countries. (((multinational corporation)))
Shoes International is a global nonprofit organization that works with schools and various companies to donate shoes to poor children throughout the world. Shoes International is an example of a (((multinational organization.)))
Ethnocentric managers believe that their native country, culture, language, and behavior (((are superior to all others.)))
_____ is another word for ethnocentrism. (((Parochialism)))
George, a regional manager for a global corporation, is meeting with several upset managers from the Rome plant about Steve, who is originally from Florida but moved to Italy to manage the Rome facility. Phillipe, the assistant manager, tells George, "We all feel that Steve sees things only one way—his—and very seldom considers our perspective on things." Steve is employing a ______ management style. (((parochial)))
Maria, the vice president of sales for an international organization, believes that employees in her foreign offices understand best how to handle the personnel and practices in their offices. So when the corporate HR manager approached Maria about reprimanding Robert, a native Parisian who is the Paris office manager, for not doing things according to corporate policy, Maria backed Robert and said, "I believe that the home office should leave Robert alone because he best understands the local culture." Maria is a(n) ___ manager. (((polycentric)))
Many companies are interested in expanding globally in order to (((gain access to supplies, new markets, and lower labor costs.)))
______ are Mexican manufacturing plants with low labor costs that give special privileges to their American owners in return for employing Mexican citizens. (((Maquiladoras)))
Quality Air, a company that builds airplanes, typically orders parts for its airplanes from other companies. _______ is the practice Quality Air is using. (((Outsourcing)))
Dominique owns an international grocery store, the World Food Market, where customers can purchase foods and canned goods from other countries. World Food Market is an example of a company that (((imports.)))
Countertrading is ___ goods for goods. (((bartering)))
Technology Experts is a U.S. company that specializes in IT research. It has formed a joint venture with Processor Inc., a Canadian organization that has a reputation for producing quality PCs. The two companies will work together to market laptops in China, sharing the risks and rewards of starting the new enterprise together. Technology Experts and Processor Inc. are partners in a(n) (((strategic alliance.)))
Donald is the owner of The Party Hat, a U.S. event-planning company. He plans to open an event-planning company, Events & Adventures, in London. Events & Adventures will be totally owned and controlled by The Party Hat, which makes Events & Adventures a ____ of The Party Hat. (((wholly-owned subsidiary)))
Free trade is the (((movement of products between countries without political or economic obstruction.)))
The United States uses government regulations, such as tariffs, embargoes, and import quotas, to limit the import of goods and services and to protect U.S. industries against foreign competition. ______ is the use of these regulations. (((Trade protectionism)))
A tax on automobiles imported into the United States that raises prices on imported vehicles to make the price of cars produced in the United States more competitive is a(n) ______; a tax on all oil imported into the United States, which is implemented to raise money for the U.S. government, is a(n) (((protective tariff; revenue tariff)))
Prior to 1990, the United States imposed a tax, known as a(n) ________, on automobiles imported from Mexico. (((tariff)))
An import quota is a(n) (((trade barrier.)))
The United States limits the amount of cheese purchased from other countries; this is a(n) ____ policy . (((import quota)))
Shrimpers in South Louisiana have accused China of selling shrimp in the United States at a lower price than U.S. suppliers. The goal of the Chinese, they argue, is to drive down the price of U.S. shrimp. The U.S. shrimpers are accusing the Chinese of (((dumping.)))
________ is prevented by quotas. (((Dumping)))
At one time, the United States used a(n) ________ to prohibit the import of all products made in Cuba. (((embargo)))
The WTO (((monitors and enforces trade agreements.)))
The agreements of the World Trade Organization are based on (((GATT.)))
The World Bank (((provides low-interest loans for improvements in developing nations.)))
Recent protests against World Bank operations involve complaints that the World Bank (1) supports countries that permit sweatshop labor or restrict religious freedom, and (2) (((finances projects that could damage the ecosystem.)))
The IMF has (((helped support some weaker European countries during the recent financial crisis.)))
NAFTA and the EU are both (((trading blocs.)))
According to the watchdog group Public Citizen, NAFTA is responsible for (((the loss of 845,000 U.S. jobs.)))
_______ is a trading bloc consisting of 28 trading partners in Europe. (((The European Union)))
______ is the group of 21 Pacific Rim countries whose purpose is to improve economic and political ties, and to reduce tariffs and other trade barriers across the region. (((APEC)))
ASEAN is a(n) (((trading bloc consisting of 10 countries in Asia.)))
George owns an international grocery store in New Jersey, the Global Food Bazaar. He has benefited highly from the trading bloc that allows him to purchase food products from Central American nations at a nice discount. With many of his customers from Costa Rica, the Dominican Republic and Guatemala, he is particularly grateful for ______, a trading bloc. (((CAFTA-DR)))
The United States granted most favored nation trading status to China, which means that (((China receives favorable trading treatment, such as the reduction of import duties.)))
During global culture training for a group of employees who are being transferred to Egypt, Samira, the HR training manager, remarked that "in the Arab world, men often hold hands, which is meant to express (((solidarity and friendship.")))
solidarity and friendship." (((the shared set of beliefs, values, knowledge, and patterns of behavior common to its people.)))
Susan, from California, arrived in Saudi Arabia and immediately noticed that the way Saudi men treat women is vastly different than the way men in the United States treat women. Susan was feeling uncomfortable and disoriented because she was experiencing (((culture shock.)))
______ in a low-context culture. (((Shared meanings are derived primarily from written and spoken words)))
The GLOBE project is an ongoing cross-cultural investigation of nine cultural dimensions involved in ___ processes. (((leadership and organizational)))
According to the GLOBE project, in-group collectivism refers to (((the extent to which people should take pride in being members of their family, circle of close friends, and work organization.)))
According to the findings of the GLOBE project, the assertiveness dimension represents the extent to which a society values (((confrontation and competitiveness.)))
When communicating across cultures, the best strategy regarding language use is to (((learn the local language.)))
Monochronic time is a (((preference for doing one thing at a time.)))
The _____ is the law that makes it illegal for employees of U.S. companies to make "questionable" or "dubious" contributions to political decision makers in foreign nations. (((Foreign Corrupt Practices Act)))
Which of the following is NOT one of the BRICS countries? (((Indonesia)))
Which statement is NOT true of the BRICS countries? (((The BRICS countries have joined together to form a trading bloc.)))
A business model outlines what customer needs the company will satisfy, the operations of the business, the company components and functions, and the projected revenue and expenses. (((TRUE)))
When using MBO, managers should meet about once a year, either informally or formally, to review progress on meeting objectives. (((FALSE)))
Every five years, the upper-level managers of ABC Technology meet to develop a new large-scale action plan that will set the direction for the company for the next five years. In that meeting the managers work on developing a strategy. (((TRUE)))
When Holiday Inn Express stated that the organization strives to "be the primary source of hospitality in the communities" it serves by providing its customers "the best possible experience and by being good neighbors," it is stating its code of ethics. (((FALSE)))
Alison, the director of marketing for a global company, is planning the contributions she and her department will make to the organization over the next 24 months. This type of planning is called operational planning. (((FALSE)))
Goals are arranged in a hierarchy known as a means-end chain because in the chain of management, the accomplishment of high-level goals is the means leading to the accomplishment of low-level goals or ends. (((FALSE)))
The production plant for Ace Manufacturing has been located in the same place for over 100 years. It has just completed its new factory, which includes its new corporate offices. The planning required for moving Ace into a new facility is a single-use plan. (((TRUE)))
Greg, owner of Comfortable Home Heating and Air, has been having problems with employee morale. After investigating, he found that some of his department managers were using management by objectives, but the accounting and sales managers were not utilizing MBO in their departments. For the MBO program to be successful, Greg must be sure that it is implements throughout his entire organization. (((TRUE)))
The planning/control cycle has two control steps: (1) control the direction by comparing results with the plan and (2) control the direction by increasing organization diversity and synergy. (((FALSE)))
In the planning/control cycle, managers take corrective action by correcting deviations in the plan being carried out or by improving future plans. (((TRUE)))
Mr. Smith, CEO of an auto dealership, has been working on the two planning steps of the planning/control cycle with his managers. To do this, Mr. Smith and his managers need to make their plan and then carry out the plan. (((TRUE)))
Setting a very challenging goal for yourself, such as "I will double the number of hours I study next semester so that I can increase my GPA by at least half a point," can keep you motivated and focused on achievement. This type of goal, which involves pushing yourself beyond what you mind might think is safe is known as a(n) (((stretch goal.)))
Research suggests that when writing out a plan to achieve your biggest goals, it is important to (((make a concrete plan and break goals into manageable bites.)))
While sitting at her desk, Ann is setting goals and deciding how to achieve them. She is (((planning.)))
______ can help workers focus on critical problems, choices, and opportunities. (((Planning and strategic management)))
Deon is opening a car wash, and he has approached SCORE, an SBA organization that assists small-business owners in setting up their companies. As a new business owner, the SCORE counselors would probably advise Deon to (((write a business plan.)))
Prior to opening her new restaurant, Nia is determining what items to offer on the menu, the anticipated profits and expenses, the tasks of various employees, and how those jobs will be handled. Nia is creating her (((code of ethics.)))
________ is the primary reason for an organization to adopt planning and strategic management. (((Developing MBO)))
_______ is the primary reason that entrepreneurs create business plans. (((To get financing and think through important details)))
The first three steps in planning his organization are (in the correct order) (((writing a mission statement, creating a vision statement, and strategic planning)))
The planning process begins with two attributes: a mission statement and a vision statement. What questions are answered with each of these? (((What is our reason for being?; What do we want to become?)))
What is our reason for being?; What do we want to become? (((purpose of an organization.)))
The ______ are strategic, tactical, and operational. (((three levels of planning)))
At the strategic level of planning, managers must pay attention to the environment outside the organization, be future oriented, and (((deal with uncertain and competitive conditions.)))
Operational planning must be completed by first-line managers every ____, and tactical planning is done by middle management every (((1 to 52 weeks; 6 to 24 months.)))
The upper-level managers of Synergy Technology are meeting for the week to look at the long-term company goals and overall direction of the organization. The CEO has expressed her concern over the economy and has told her managers to look closely at the environment outside the organization before making decisions and to be future oriented. In this meeting, top managers of Synergy are (((doing strategic planning.)))
_______ is planning done by middle managers to determine what contributions their departments or similar work units can make with their given resources during the next 6 to 24 months. (((Tactical planning)))
______ is planning done by first-line managers to determine how to accomplish specific tasks with available resources within the next 1 to 52 weeks. (((Operational planning)))
Suppose you manage an upscale restaurant in New York City. __________ would involve writing employee schedules and a list of things to do for the chef and other kitchen staff. (((Operational planning)))
Strategic planning should communicate not only general goals about growth and profits but also (((ways to achieve them)))
Dave owns several computer repair shops across the Midwest, and he has set a goal for his company of cutting costs in all his locations over the next three years. Dave has set a(n (((strategic goal.)))
____ are goals set by and for first-line managers and are concerned with short-term matters associated with realizing tactical goals. (((Operational goals)))
A ______ is a specific commitment to achieve a measurable result within a stated period of time. (((goal)))
_____ can be another term used for a goal. ((( objective)))
Susan is a front-line supervisor in a grocery retailer. She desires to decrease the time she spends working on doing her daily closing work by 45 minutes. This objective set by Susan and for her is an example of a(n) (((operational goal.)))
This month, Alice, the branch manager of Quality Computer Sales, has heard several complaints from customers that Quality employees do not return calls. Today a good customer told Alice that "my phone calls and e-mails are returned after maybe a week or so." Hearing enough, Alice mandated that "all customer phone calls and e-mails must be returned within 24 hours." This new directive by Alice is an example of a(n) (((policy.)))
Fast Delivery trains its truck loaders how to set the packages in the delivery vehicles, so that when delivery drivers are pulling packages off their trucks, they are organized in a specific order and with the label facing forward to reduce errors and save time. A(n) _______ is being implemented when directing that the trucks be loaded in this specific manner. (((procedure)))
A ______ is a standing plan that outlines the response to particular problems or circumstances. (((procedure)))
A ______ is a standing plan that designates specific required action. (((rule)))
The department chair of the IT department issued a directive that "Absolutely no food or drink are allowed in any computer lab." She issued this directive because several computers and keyboards in the computer labs were recently damaged by spilled drinks and food. This is a(n) ___________ issued by the department chair. (((rule)))
_____ are plans developed for activities that are not likely to be repeated in the future, such as plans for programs or projects. (((single-use plans)))
A _____ is a single-use plan encompassing a variety of projects or activities. (((program)))
The Human Resources department of French Quarter Seafood conducted a training workshop on setting and achieving organization goals. Alex, the store manager of the New Orleans location, has been working with his managers to set objectives that are clearly defined, focused on achieving specific results, and can be accomplished with a given amount of resources. Which aspect of SMART goals is missing? (((Goals should specify the target dates or deadline dates when they are to be attained.)))
Which of the following goals satisfies the SMART criteria? ((("In the next quarter, our Houston sales rep will increase sales by 10%.")))
The purpose of MBO is to (((motivate subordinates.)))
Research on MBO shows that people are impelled to set more difficult goals when (((both management and subordinates participate.)))
For MBO to be successful, top management must be committed to the program, MBO must be applied organizationwide, and (((objectives must cascade.)))
MBO works by objectives moving through the organization; that is, top managers set general organizational objectives, which are translated into divisional objectives, which are translated into departmental objectives. The hierarchy ends in individual objectives set by each employee. This is an example of MBO working as objectives ______ through the organization. (((cascade down)))
When utilizing SMART goals or MBO, deadlines can help you keep your eye on the ______ while simultaneously paying attention to ______. (((big picture; the necessary details)))
To ensure that plans are moving in the right direction, managers can use the (((planning/control cycle.)))
The planning/control cycle has two planning steps. They are (((1) make the plan, then (2) carry out the plan.)))
Alicia, a district manager for a global retailer, is traveling to different stores in her district to discuss with store managers how to increase profitability for the rest of the fiscal year. What is Alicia doing? (((controlling the direction of the planning/control cycle)))
John, the owner of a lawn care service, likes to meet with his employees six months after their yearly evaluation to discuss how they are progressing on suggestions on and plans to improve their yearly job evaluation. At this point, by comparing the six-month results with the past evaluation, John is in the ______ steps of the planning/control cycle. (((control)))
The planning/control cycle has two control steps: (1) control the direction by comparing results with the plan and (2) control the direction by taking corrective action in two ways—namely, by (((correcting deviations in the plan being carried out or improving future plans.)))
correcting deviations in the plan being carried out or improving future plans. (((It outlines how goals are going to be met.)))
Which of the following companies is NOT one of the "frightful five" companies that dominate the Internet economy? (((Instagram)))
. You have an idea for a company that sources fruits from local farms and makes fresh juices on a daily basis. You want to start a subscription-based service in which households within a 100-mile radius subscribe to your plan and receive two gallons of freshly squeezed juice (for example, cherry juice, apple juice, lemonade) twice a week. As you think about starting your business, you ask yourself: How rare is the service I am offering? How valuable is it? Can I organize the company to maximize my advantages in the marketplace? After asking these questions, review and identify which question from VRIO did you forget to ask? (((Is the resource or capability costly for other firms to imitate?)))
VRIO is a framework for analyzing a resource or capability to determine its competitive strategic potential by answering four questions. VRIO stands for (((value, rarity, imitability, and organization.)))
Which of the following is an example of a mission statement? ((("to offer the lowest prices on new hardcover books, anywhere in the world")))
A short-term usually spans ______ while a long-term goal typically spans (((12 months; 1 to 5 years.)))
You receive a memo that reads as follows: “In recent months, our customer evaluations have dropped, and our sales have decreased by 10 percent. To restore our business, we will do the following: (1) We will contact all of our customers and offer a 10 percent discount on their next purchase. (2) For the customers who spend more than $1,000 per year, we will invite them to join our Insiders’ Club, which gives them discounted tickets to concerts and sporting events. (3) Effective immediately, we will begin staffing our call center 24 hours a day, 7 days a week. (4) Customer-service personnel will be given more discretion in helping customers resolve problems, with minimal interference from supervisors.” This memo is a(n) (((action plan.)))
Which of the following is intended for one-time use and is not likely to be repeated in the future? (((a program)))
Which of the following is an example of a rule? ((("An employee who comes to work intoxicated will be terminated immediately.")))
While preparing a self-assessment of your job performance the previous year, you are required to list three goals for the coming year. You list the following as one of your goals: “I will triple sales in my territory by the end of the next fiscal year.” Based on the SMART criteria, what is the problem with this goal? (((It is not attainable.)))
Management by objectives (MBO) and goal cascading will not work without an effective (((goal-setting process.)))
___ is the overall goal of MBO. (((Motivating employees)))
Cascading goals being _____ and "cascade" to (((at the top of the organization; individual employees.)))
Which sequence of goals is consistent with "cascading" goals that begin at the top of the organization? (((strategic goals > divisional goals > department goals > individual goals)))
Which of the following is a tactical goal? (((By the end of this fiscal year, we will achieve a 10% cost reduction in our office expenses.)))
A and B Office Supply, a small family-owned company, sells high-priced desks, some as expensive as $10,000, to executives in its area. Very few companies have chosen to market this product, and A and B has enjoyed record profits over the last 25 years. A and B Office Supply is an example of a company that would typically not choose to utilize strategic planning. (((TRUE)))
The strategic-management process is often touched off by a crisis. (((TRUE)))
The first three steps in the strategic-management process are establishing the mission and the values statement, assessing the current reality, and formulating the grand strategy. (((TRUE)))
When Pablo and his partner wrote the mission statement for their fitness center, P&P Fitness, it did not include descriptions of their customers or the products and services they offer, because those descriptions belong in a vision statement. (((FALSE)))
Brilliante Pens, a 125-year-old pen manufacturer, markets very high-quality pens, many of which cost more than $1,000 each, to executives globally, always maintaining its reputation of superior value in its narrow market. Brilliante is using a cost-focus strategy. (((FALSE)))
For over 20 years, ABC Manufacturing produced only one product, a car part for Moraine Assembly (the Moraine, Ohio, General Motors truck factory). Because its single-product business is so closely tied to Moraine Assembly, ABC is a vulnerable company. (((TRUE)))
In a single-product strategy, a company makes and sells only one product within its market. An example of a businessperson following a single-product strategy is a farmer who grows and sells only corn. (((TRUE)))
Execution consists of questioning, analysis, and follow-through to mesh strategy with reality, align people with goals, and achieve promised results (((TRUE)))
Because all work ultimately entails some human interaction, effort, or involvement, Bossidy and Charan believe that focusing on organization synergy is the most important process in strategy execution. (((FALSE)))
_____ focuses on developing a comprehensive program for long-term success. (((Strategic planning)))
The best ways to avoid faddish management ideas are to (((understand your own core values, and use this knowledge to create or accept the core values of your company.)))
To understand your own values, you should (((articulate them in writing and test them through daily decision making.)))
The three key principles of ______ are the creation of a unique and valuable position, trade-offs in competing, and creating a "fit" among activities. (((strategic positioning)))
According to Harvard Business School professor Michael Porter, strategic positioning means _____ to achieve sustainable competitive advantage. (((distinctive positioning)))
What are the three sources of a unique and valuable strategic position? (((few needs, many customers; broad needs, few customers; broad needs, many customers)))
Gig companies, such as Amazon, Google, and Apple, are no longer content simply to enhance part of your life. Their new strategy is to build a device, sell it to consumers, and then (((sell them the content to play on their device.)))
______ requires a manger to visit an earlier step in the strategic-management process in order to revise actions if necessary. (((The feedback loop)))
In order, the steps of the strategic-management process are (((establish the mission, vision, and values statements; assess the current reality; formulate the grand strategy; implement the strategy; and maintain strategic control.)))
. "Who are our customers? What are our major products or services? In what geographic areas do we compete?" A good _____ will answer these questions. (((mission statement)))
Is it appropriate for the organization and for the times? Does it set standards of excellence and reflect high ideals? Is it ambitious? A good _______ will answer these questions. (((vision statement)))
The growth, stability, and defensive strategies are common (((grand strategies.)))
Access Office Equipment has shifted to sales and service of laptops and PCs, where it has the potential to triple the number of its customers. The company is no longer offering repairs on older types of office equipment because the demand for service on this equipment is low, and profits in that part of the business have dropped significantly. Access Office Equipment is implementing a _____ strategy. (((growth)))
Organic Foods International has decided to increase its market share by hiring a marketing rep to visit businesses in the area and invite their employees to shop at Organic Foods and attend monthly health events that take place at Organic Foods stores. Each of the individual stores has hired two new employees to handle the anticipated increase in customer traffic. Organic Foods is using a _____ strategy. (((growth)))
Delux Technology has a reputation of reliability and a winning customer service, qualities that helped to build this highly respected name brand over the last 15 years. Speaking at a recent business conference, Steve, the CEO of Delux, told his audience, "We have built our reputation by changing little over the last several years, but consistently helping customers with great, caring service and a reliable product." Which type of strategy does Delux Technology use? (((stability strategy)))
_________ is another term for a defensive strategy. (((A retrenchment strategy)))
Rashid, the CEO of Top Productions, said to his new vice president of accounting, "In the past I had resistance to new ideas by employees who felt that our plans threatened their influence or their jobs. So when you tell your collections department that we just hired a collection agency to handle bad debt, you may have to sell your collections manager and his supervisors on using the agency. You also have to emphasize the fact that nobody is going to be laid off." Selling middle and supervisory managers on changes to overcome their resistance is often a necessary part of (((strategy implementation.)))
___________ is(are) included in strategic control. (((Monitoring the execution of strategy and making adjustments, if necessary,)))
Bryan Barry suggests that to keep a strategic plan on track, organizations should (((engage people, keep it simple, stay focused, and keep moving.)))
Pedro, CEO of a successful IT company, is constantly reading press releases, ads, and news articles about his competition. He regularly checks information about new competitive products and visits trade shows to study his competition. Pedro is involved in (((competitive intelligence.)))
A SWOT analysis is (((a search for the strengths, weaknesses, opportunities, and threats affecting an organization.)))
In a SWOT analysis, organizational strengths are (((skills and capabilities that give a company advantages in executing its strategies.)))
Organizational opportunities in a SWOT analysis are (((environmental factors that the organization may exploit for a competitive advantage.)))
A forecast is a(n) (((projection for the future.)))
Contingency planning is also known as (((scenario planning and scenario analysis.)))
The basic assumption that the picture of the present can be projected into the future is the basis of a (((trend analysis.)))
________ is the process by which a company compares its performance with that of high-performing organizations. (((Benchmarking)))
When organizations endeavor (1) to keep their costs (and hence the prices of their products or services) below those of competitors and (2) to target a wide market, they are utilizing a _____ strategy. (((cost-leadership)))
Organic Market sells high-quality products and grocery items that are unique to its area; in addition it offers delivery for customers who are unable to leave their home and a number of regular workshops regarding healthy eating. By offering these types of products and services, Organic Market is pursuing a _____ strategy. (((differentiation)))
Mountain Rescue Incorporated sells emergency safety and rescue products to ski patrols and rescue workers at prices that are below those of its competitors, which offer a larger line of more expensive products and focus on broader markets. Mountain Rescue Incorporated is pursuing a ____ strategy. (((cost-focus)))
The Golden Bed in Dayton, Ohio, sells high-quality, unique bedding that is a real value to upscale homes in the area. No other company in its local area markets bedding of such quality and value. By offering this type of product, The Golden Bed is utilizing a _____ strategy. (((focused-differentiation)))
Motherboard Technology makes and sells only one product, a high-quality processor for mainframe computers, within its market—Silicon Valley, California. Which strategy is Motherboard Technology using? (((single-product)))
Quality Services is an organization that operates several companies that market food products, restaurant equipment, and paper and plastic products, and it even has a division that counsels restaurant owners, helping them launch new restaurants successfully. Quality Services is pursuing a _____ strategy. (((diversification)))
When an organization operates several businesses to spread the risk, such as a landscaping company that offers not only landscaping services but also lawn care, snow removal, and other services, that business is utilizing a ____ strategy. (((diversification)))
In the 1970s, Atari singlehandedly created a market for home video games, offering a console and cartridges that contained games like Space Invaders, Centipede, and Asteroids. At the time, Atari effectively implemented a ______ strategy. (((blue ocean)))
The BCG matrix is a means of evaluating strategic business units on the basis of their (((business growth rates and share of the market.)))
In the BCG matrix, _______ are organizations that have slow growth but high market share, with income that often finances stars and question marks. (((cash cows)))
In the BCG matrix, _____ are companies that have high growth and high market share, and are definite keepers. (((stars)))
Global CEOs desire ___________ even more than profit growth, stimulating innovation, customer loyalty, and finding qualified employees. (((excellence in execution)))
Successful controlling within an organization, according to Bossidy and Charan, demands that managers build a foundation of controlling using ___________, the three core processes. (((people, strategy, and operations)))
What is the assessment of the external environment? What are the critical issues facing the business? Can the business execute the strategy? According to Bossidy and Charan, a ______ should address these some of these questions. (((strong strategic plan)))
A ____ statement should describe what the organization stands for, its core priorities, the values its employees embody, and what its products contribute to the world. (((values)))
Decision making is the process of identifying and choosing alternative courses of action. (((TRUE)))
When evaluating the alternatives of a decision, managers need to base decisions strictly on cost, quality, and feasibility. (((FALSE)))
The rational model assumes that managers have complete information, are able to make an unemotional analysis, and are able to make the best decision for the organization. (((TRUE)))
Nonrational models of decision making assume that managers have complete information and are able to make the best decision for the organization. (((FALSE)))
With satisficing, managers look for alternatives until they find one that is satisfactory, not optimal. (((TRUE)))
Managers with a low tolerance for ambiguity and an orientation toward task and technical concerns when making decisions have an analytical decision-making style. (((FALSE)))
Anne, the electronics department manager at a local Target store, has a good working relationship with her employees. She believes in an open-door policy, and she encourages the open exchange of opinions in her department. Anne utilizes the directive style of management. (((FALSE)))
A decision tree, which is the hierarchy or chain of command used by many organizations when discussing alternatives, must be followed when getting decisions approved. (((FALSE)))
When faced with a decision, Myles asks himself "Why not just take the easiest way out?" This approach, known as relaxed change, is a form of evidence-based decision making. (((FALSE)))
Danielle, the IT department chair at a community college, has been having problems with her students not coming to class, so their grades are suffering. If Danielle were to use defensive avoidance to respond to this situation, she would likely respond by procrastinating, passing the buck, or denying the risk of any negative consequences. (((TRUE)))
With an escalation of commitment bias, decision makers increase their commitment to a project despite negative information about it. (((TRUE)))
The larger and more diverse the group, the higher the quality of the decision. (((FALSE)))
__________ is the tendency for people with strong prior beliefs, when confronted with a choice, to make their decisions based on their beliefs even if their beliefs are false. (((Prior-hypothesis bias)))
A ____ is a choice made from among available alternatives. (((decision)))
Alex is the R&D manager with Digital Copier Inc. The company is having a problem with one of its most popular copiers jamming. The vice president of marketing told Alex that "we need to quickly discover how to turn this problem into a desirable situation. You need to determine the underlying cause of the problem and perhaps even make improvements to the copier." When Alex analyzes the underlying causes of the paper-jam problem, which function is he performing? (((diagnosis)))
Female investors make trades much less often than men because they (((do more research, and they tend to base their investment decisions on considerations other than just numbers.)))
In the third step of rational decision making (evaluating alternatives and selecting a solution), you need to evaluate each alternative not only according to cost and quality but also according to which of the following questions? (((Is it ethical, feasible, and effective?)))
José, the sales manager for a software development firm, is working on the problem of increasing sales by using the rational model of decision making. In the first step he identified the problem of his employees needing more training. In the second step he thought of alternative solutions; and in the third step he evaluated alternatives and selected a solution. José needs to _____ in the fourth and final step. (((implement and evaluate the training program chosen)))
The rational model of decision making is also called the ____ model. (((classical)))
_____ are difficulties that inhibit the achievement of goals. For example, the lack of parking may be the main reason why a new restaurant is failing. (((Problems)))
Patricia is the owner of a floral shop. A successful global candle company approached her about selling its products in her shop. Patricia knows this product and company can help her exceed her existing goals. Selling candles is a(n) _____ for Patricia. (((opportunity)))
With the community interested in eating healthy, Sue Ellen, a restaurant owner, is considering adding more vegetarian dishes to her menu. This decision is uncertain and risky, and she finally decides to go with her gut feelings. Sue Ellen is exhibiting (((nonrational decision making.)))
_____ is the idea that proposes that the ability of decision makers to be rational is limited by numerous constraints, such as complexity, time, cognitive capacity, values, skills, habits, and unconscious reflexes. (((Bounded rationality)))
Susanne, the CEO of a national IT manufacturer, was approached by Simple Phones, a new company that is marketing a new type of phone, to partner with the company on a project. The results of the partnership are uncertain because the company (Simple Phones) and its technology are new. Susanne is uncertain what to do because she is limited by numerous constraints, such as the uncertainty and complexity of the technology, the management success of Simple Phones, and time (because other companies are interested in the partnership). Susanne is experiencing (((bounded rationality.)))
Paula, the controller of Tasty Pizza, is purchasing several new delivery vehicles. Paula has numerous work responsibilities, so she has limited time to shop for cars. Because of the time constraints, she cannot make an extensive search for the best alternative, so she looks for cars until she finds a model that is satisfactory. Paula is following the _____ model. (((satisficing)))
Chad and his partner, John, have decided to update their computer network, although they have no expertise in this area. During a meeting with John, Chad commented that "Our decision is limited by numerous constraints, such as our understanding of the complexity of technology, time and money, imperfect information, and our conflicting goals." These limitations are hindrances to (((rational decision making.)))
Sal has been in the landscaping business for over 40 years. When Jaime, his business partner, suggested adding snow removal as an extra service, Sal responded with "My gut feeling is to say yes because of my experience in the past and what I have seen other companies do over the years, both successfully and not." Sal is acting on _____ in his response to Jaime. (((intuition)))
Which of the following is an example of the "curse of knowledge"? (((A writer with a large vocabulary writers a novel using many words that the average reader cannot understand.)))
Which of the following is not a hindrance to perfectly rational decision making? (((intuition)))
Olivia, the owner of The Town Gazette, a small city newspaper, started her business two years ago, believing that there was still enough demand for her product. However, because people are busy and because so much news is now available online, she has seen the demand for her paper drop steadily. Olivia made a decision to change the original direction of the company and focus more on an Internet news service. Olivia is relying on _______ by demonstrating that managers need to think and act as if their company is an unfinished prototype, and by seeing her business as outsiders do. (((evidence-based decision making)))
Style Mart uses time-series forecasts, such as past holiday sales and the strength of store sales this year, to predict future sales based on patterns of historical data. Style Mart is using (((analytics.)))
______ is a data-mining technique used to predict future behavior and anticipate the consequences of change. (((Predictive modeling)))
Which of the following is NOT recognized as a key implementation principle of evidence-based management? (((Treat your organization as a closed system.)))
_____ is the process of studying large amounts of data of a variety of types to uncover hidden patterns, unknown correlations, and other useful information. (((Big Data analytics)))
Debbie likes working in the banking industry because it has a lot of structure and certainty, and the daily work is routine. Debbie probably (((has a low tolerance for ambiguity.)))
Dave is a salesperson who takes a long time to make decisions. He loves sales because he responds well to the pressure he faces in the many new or uncertain situations he encounters as a salesperson. Like most successful salespeople, he is high in his tolerance for ambiguity. Dave likely has a(n) _____ decision-making style. (((analytical)))
Manuel, a server in a popular family restaurant, is the most popular employee with both customers and employees, and as a result, he usually earns the most in tips. He is always ready to lend a hand when asked, and with his warm personality and supportive nature, people confide in him. Manuel does have a problem telling people no, and he has a tendency to avoid conflict because he is concerned about hurting others or getting people mad. Manuel likely has a(n) ______ decision-making style. (((behavioral)))
A decision tree is (((a graph of decisions and their possible consequences.)))
BP and Halliburton executives made the decision not to invest in the oil well blowout preventer that experts believe would have prevented the Deepwater Horizon oil rig explosion and fire off the coast of Louisiana. The executives made their decision based on their belief that their decision would have no great negative consequences. Their decision is an example of (((relaxed avoidance.)))
Ace Manufacturing has high turnover due to issues with the corporate office. Trevor, the plant manager, is frustrated and cannot find a solution to stop it. At first he offered raises to many of the good employees who were leaving, but after a while, Paul started blaming those who were leaving, saying, "They are poor-quality employees and they are to blame for leaving." Trevor is engaged in (((defensive avoidance.)))
Orlando, the manager of a camera store, believes that his store may be closed by corporate in the near future, so he cannot sleep well at night and he is dealing with severe anxiety and irritability, and he has been ill several times this month. Several times Orlando has missed important details in his job, and he almost got in an automobile accident this morning. Orlando is experiencing (((panic.)))
How high priority is this situation? How believable is the information about the situation? How quickly must I act on the information about the situation? Managers can use these questions to make decisions about (respectively) (((importance, credibility, and urgency.)))
Readily available information may not present a complete picture of a situation due to (((availability bias.)))
When Marcos, the sales manager, is deciding on who will train the new sales trainee, he tries not to look at just the recent sales numbers but also checks past sales reports to determine who is the most productive salesperson. By using this method he is not affected by the ______ bias. (((availability)))
When people seek information to support their point of view and discount data that do not, they are subject to ______ bias. For example, Joe uses the fact that his commute is now shorter to confirm his belief that there are fewer cars on the road. However, Joe is not accounting for the fact that he recently took over the graveyard shift, and that traffic is always lighter at night and in the wee hours of the morning. (((confirmation)))
Malik and his managers spent a large sum of money on the new training program, and they feel that there has been little improvement as a result of the investment. The training is scheduled to continue for two more months, and Malik feels that the company has already spent too much money on the training to simply abandon it. Malik is experiencing (((sunk-cost bias.)))
Felipe and Meg are selling their home. They listed their house three months ago at an extremely high selling price, a price they randomly chose. They do not want to reduce the price to reflect what the marketplace shows their home is really worth. Felipe and Meg are participants in _____ bias. (((anchoring and adjustment)))
Republic advertisements about Representative Joe Smith, the Democratic candidate, portray him as lazy, dishonest, and poor at his job. At the same time, the Democratic ads for Joe Smith describe him as a caring, honest, hard worker. The sponsors of these advertisements know that ads can affect whom voters choose on Election Day. The tendency of decision makers (the voters in this example) to be influenced by how a situation or problem is presented to them is an example of (((framing bias.)))
_____ causes people to view events as being more predictable than they really are. (((Hindsight bias)))
The advantages of _______ are having a greater pool of knowledge, gaining different perspectives, gaining intellectual stimulation, having a better understanding of decision rationale, and having a deeper commitment to the decision. (((group decision making)))
Goal displacement, satisficing, and groupthink are (((the disadvantages of group decision making)))
Satisficing is the tendency of (((a group to settle on a decision that is "good enough.")))
In regard to decision making, which of the following statements reflects what research has found? (((Groups make better decisions than most individuals acting alone.)))
With regard to decision making, it is accurate to say that groups (((have higher decision-making accuracy when group members know a good deal about the relevant issues.)))
Roberta, the computer department manager at IT Mart, is considering whether to use a group to develop a plan to help employees sell extended warranties on technology sales. Roberta should consider having the group make this decision if (((it will increase acceptance of the plan.)))
____ is a technique used to help groups generate multiple ideas and alternatives for solving problems. (((Brainstorming)))
The Delphi technique is a group process that uses physically dispersed experts who fill out questionnaires to (((anonymously generate ideas.)))
Don, a new manager, commented to a friend, "I am excited about different aspects of my new job. The company computers have entered the field of decision making. The network not only collects information more quickly, but the system also reduces the roadblocks when I need a group consensus." Don is referring to (((Don, a new manager, commented to a friend, "I am excited about different aspects of my new job. The company computers have entered the field of decision making. The network not only collects information more quickly, but the system also reduces the roadblocks when I need a group consensus." Don is referring to)))
A(n) _______ is a computer-based information system that provides a flexible tool for analysis and helps managers focus on the future. (((decision support system)))
Which of the following is not a guideline that can lead to effective brainstorming? (((Eliminate ideas that will be too costly.)))
Productivity expert Odette Pollar recommends that managers delegate (1) emergencies and (2) confidential matters, and (3) personnel-related matters. (((FALSE)))
An organizational structure is a set of shared, taken-for-granted implicit assumptions that a group holds and that determines how it perceives, thinks about, and reacts to its environments. (((FALSE)))
A company with a market culture has a strong internal focus, and it concentrates on developing the business and marketing plans, as well as helping employees market their product(s). (((FALSE)))
Dave, a graphic designer with Development Advertising, enjoys working for the company because of the flexibility of its management and its culture, which encourages risk taking, innovation, and creativity. Dave gets quick answers from his boss, which allows him and Development Advertising to quickly respond to changes in the marketplace. Development Advertising is an example of an adhocracy culture. (((TRUE)))
Thrifty Bank has an external focus, concentrating on strategic planning, risk taking, and flexibility over stability. Thrifty Bank has a hierarchy culture. (((FALSE)))
The AFL-CIO is an example of a for-profit organization, and the March of Dimes is an example of a nonprofit organization. (((FALSE)))
The arrangement of having discrete parts of a task done by different people is known as division of labor. (((TRUE)))
Fine Oil uses an organizational structure that includes both functional divisions (such as marketing and HR) and a divisional chain of command (such as the chemical and petroleum products divisions). For example, Glen, a salesperson for Fine Oil, reports to both the marketing manager and the petroleum products manager as he starts to develop the Shell Oil account. Fine Oil uses a matrix structure. (((TRUE)))
Lena works as an order-taker at Fast Burger, a fast-food restaurant. She does not cook food, or even package the final order, but she does input the order and takes payment. Lena reports directly to the shift manager, and she seldom interacts with the general manager. Fast Burger is an example of a mechanistic organization. (((TRUE)))
Routine tasks and paperwork, as well as jobs that help your subordinates grow, are (((tasks that a manager should delegate, according to Odette Pollar.)))
To do more in a day, ____," according to productivity expert Odette Pollar. (((you must do less—not do everything faster)))
According to the competing values framework, the four types of ______ are clan, adhocracy, market, and hierarchy. (((organizational cultures)))
. Insurance company Acuity has a clan culture; employees have generous perks and are empowered to participate in a way in the company that is fun. The end results are profitability and an enviably low 2% turnover rate. A clan culture has a(n) (((internal focus and values flexibility.)))
Juan, the owner of Quality Catering, is driven by competition. He is very focused on meeting deadlines and quality, and on delivering the results that customers want. He pushes his managers to continually exceed their productivity goals, and he stresses that profits take precedence over employee development and satisfaction. Juan feels "there is not enough time for training." However, his employees are regularly rewarded for their success in meeting company goals. Quality Catering has a(n) ____ culture. (((market)))
Phillip, owner of Technology Sales, said, "We have to focus on our customers and how can we beat the competition, so we have to be flexible." He encourages his R&D department to develop innovative products and focuses on being quick to respond to market changes. Technology Sales has a(n) (((adhocracy culture.)))
_______ are the rituals and ceremonies of a company, as well as the manner of dress, awards, myths and stories told about a company. (((Observable artifacts of organizational culture)))
Espoused values are _____. For example, Hewlett-Packard stresses the "HP Way," a collegial, egalitarian culture that gave as much authority and job security to employees as possible. (((explicitly stated values and norms preferred by an organization)))
Symbols, stories, heroes, and rites and rituals are ways in which ____ is (are) most often transmitted to employees. (((culture)))
Which of the following is an accurate conclusion based on a recent meta-analysis of more than 38,000 organizational units and 616,000 individuals? (((Employees have more positive work attitudes when working in organizations with clan cultures.)))
Hewlett-Packard founders David Packard and William Hewlett created a close-knit organizational culture that gave a lot of responsibility to employees and fostered innovation within the company. ______ are individual responsibility and the importance of innovation. (((Espoused values)))
Making formal statements, engaging in rites and rituals, utilizing employee training and coaching, demonstrating how a leader reacts to a crises, being a role model, and giving rewards, promotions, and bonuses are some of the teaching methods that organizations can utilize to (((effect corporate change.)))
Organizations that are formed to offer services to clients and not make a profit are ____, whereas, ________ are those organizations that are formed to make money, or profits, by offering products or services. (((nonprofit organizations; for-profit organizations)))
Dr. Gomez is retiring as a doctor in private practice. He wants to start a(n) __________, Doctor Help, that will provide low-income individuals in the United States with medical assistance at no cost. (((nonprofit organization)))
Examples of ______ are the AFL-CIO (a union) and the National Federation of Independent Business. Both are voluntary groups with the purpose of advancing member interests (((mutual-benefit organizations)))
________ is(are) another way of describing a vertical hierarchy. (((A chain of command)))
Having realistic expectations and thinking about the kind of manager you want to be, not forgetting to manage upward and sideways as well as downward, getting guidance from other managers, and resisting isolation is good advice for those who are (((transitioning upward in an organization.)))
________ in an organization is the division of labor. For example, in a publishing company, there are people who acquire manuscripts (sponsoring editors), people who edit manuscripts (developmental editors), and people who turn manuscripts into printed books (production editors). (((The arrangement of having discrete parts of a task done by different people)))
__________ is also known as division of labor. (((Work specialization)))
A hierarchy of authority is ____. Most military organizations are known for having a strong hierarchy of authority. (((a control mechanism for making sure the right people do the right things at the right time)))
Tomás, a sales associate in the furniture department of the Larkspur Department Store, reports to Carolina, the furniture department manager. This morning, Carolina asked Tomás to change the price tags on the desk chairs. While working on this task, David, the computer department manager, told Tomás that he wanted him to unload some computers. Tomás is confused about what to do because under the principle of unity of command, (((employees should report to no more than one manager.)))
_____ are the two kinds of spans of control. (((Narrow and wide)))
Maria, the CEO of EnviroSystems, has only three people reporting to her: the vice president of marketing, vice president of HR, and vice president of accounting. Maria has a (((narrow span of control.)))
During weekly conference calls, Mary, a restaurant manager for a national chain of restaurants, and the other managers in her district explain to their district manager the reasons for different decisions; they also explain why certain costs and sales were higher or lower than they were the previous week. _______ is the explanation of their decisions and work results. (((Accountability)))
. ________ enables a manager to make decisions, give orders, and utilize resources. For example, the manager of a local pizzeria has the right to hire and fire drivers based on their punctuality, performance, and customer-service ratings. (((Authority)))
Ralph, the manager of a busy hardware store, knows that he needs to entrust many of his tasks to managers and other employees. _____ is this process of assigning tasks to lower-level managers and employees. Ralph follows the 70% rule, which states that he should assign tasks to employees who can perform each task at least 70% as well as Ralph can. (((Delegation)))
_____ is the process of assigning managerial authority and responsibility to lower-level managers and employees. (((Delegation)))
During a meeting, Tammy, a branch manager for USA Bank, pointed to the corporate organization chart on the wall. Tammy remarked that "These people provide advice, recommendations, and research for us, and they are indicated with a dotted line. Laura (our CEO) and the vice presidents of our organization are up here, indicated on the organization chart by a solid line vertical line." _______ are indicated on the organization chart by a solid line ____, and ______ are indicated by a dotted line. (((Line managers; staff personnel)))
Astrid, a customer service representative with NorthTel Wireless Services, was asked by one of her customers if NorthTel would be interested in joining the chamber of commerce to meet potential customers and increase its contacts in the local business community. Astrid believes this is a very good idea and approached her manager, DeShawn, about becoming a chamber member. DeShawn said to Astrid, "Because the cost of membership is over $500 and you will have to leave the office to attend meetings, I will have to get approval from management above me." NorthTel is an example of an organization with (((centralized authority.)))
With ______, important decisions are made by middle-level and supervisory-level managers. (((decentralized authority)))
Organizational design is concerned with an organization developing (((optimal structures of accountability and responsibility to execute its strategies.)))
______ occurs when people are hired or promoted, or denied hiring or promotion, for reasons not relevant to the job. (((Discrimination)))
Tech Geeks, a sole proprietorship that uses contract labor, has all of its authority centralized in a single person (the owner), a flat hierarchy, few rules, and low work specialization. Tech Geeks is an organization with a (((simple structure.)))
The structure of Sandal Mart consists of people with similar specialties put together in formal groups, such as the marketing, accounting, and human resource departments. Sandal Mart has a _____ structure. (((functional)))
Which type of workplace discrimination occurs when an organization uses an employment practice that results in unfavorable outcomes to a protected class? (((adverse impact)))
Time Warner has different divisions for magazines, movies, recordings, cable television, and so on. The Warner Bros. part of the empire alone has divisions spanning movies and television, a broadcast network, retail stores, theaters, amusement parks, and music. Time Warner is an example of an organization with ____ divisions (((. product)))
If a firm gives all international assignments to people without disabilities, assuming they will therefore not require special accommodations, which type of workplace discrimination has occurred? (((disparate treatment)))
IT Technology has designed its corporate structure with divisions based on their location. For example, there are the Southern and Northern Divisions, and globally there are the European Division and the Asian Division. IT Technology is an example of a structure with (((geographic divisions.)))
For years, a mathematics professor experienced abusive behavior at a community college where he taught. It began with a group of managers spreading rumors and false accusations that threatened his job. The abuse was emotionally draining and took time and focus away from his job. The professor was the victim of (((bullying.)))
Ford Motor Co. has separate divisions for passenger cars, for large trucks, and for farm products. Ford is an example of an organization with (((customer divisions.)))
Which of the following statements about sexual harassment is true? (((A person who feels he or she must acquiesce to a sexual proposition to keep his or her job is facing quid pro quo harassment.)))
Organizations using a matrix structure (((contain two command structures, in which some people report to two bosses.)))
Maria works in a manufacturing facility, Zap Auto Parts. Throughout the building there were posters and calendars of a sexually explicit nature, making Maria very uncomfortable. Zap Auto Parts (((has a hostile work environment.)))
Quality Paper Products utilizes a structure that includes functional divisions, such as accounting and production, and a divisional chain of command, such as the home products and office divisions. For example, Mariana, a salesperson, reported to both her sales manager and the office products manager when she first started working on the city of Springfield account. Quality Paper Products has a (((matrix structure.)))
_______________ is the process of locating and attracting qualified applicants for jobs open in the organization. (((Recruiting)))
The Safety Committee, a temporary team at the Annapolis Tool & Die Corporation, was established to make the factory floor a safer place. Members of the committee still work in their departments full-time, but they meet twice a month for a few hours as a committee. The Safety Committee is an example of a ____ within an organization. (((horizontal design)))
With a hollow structure, the organization (((. has a central core of key functions and outsources others to vendors who are less expensive or faster.)))
Which of the following is the most effective source of new employees? (((employee referrals)))
With a modular structure, a firm _____. An example of the modular structure is the massive 787 Dreamliner project, in which Boeing contracted with many suppliers, each responsible for one component or assembly, which were then integrated to make the aircraft. (((assembles portions of product provided by outside contractors)))
Compared to internal recruiting, which of the following is true of external recruiting? (((The process is more expensive and takes longer.)))
When Jason became one of three final candidates for a managerial position with a large pharmaceutical company, the hiring manager scheduled a special meeting with him. The two talked about the stressful deadlines and heavy travel required of the position, as well as the compensation and benefits. Jason appreciated that the hiring manager took the time to provide a(n) (((Applicants with weak credit scores are more likely to be unqualified and dishonest employees.)))
Raul, a salesperson for the Lovely Landscapes landscaping company, is working with a longtime customer who is interested in either cutting back on the services or getting a better price due to the tight economy. Raul tells his manager, "I want to take care of the customer with the best value. So what do you think is the best package to offer in this situation?" Raul and his manager are trying to utilize the (((contingency design.)))
A(n) ______________ is susceptible to legal attack because some questions may infringe on non-job-related matters such as privacy, diversity, or disability. (((unstructured interview)))
During a(n) _______________, Megan asked Jeff to tell her about the actions he took to overcome some sort of adversity. (((behavioral description interview)))
Phone Tech, a global phone company, frequently needs to respond quickly to fast-changing consumer tastes with its smartphones. Therefore, Phone Tech has fewer rules and procedures than most organizations, and networks of employees are encouraged to cooperate and respond quickly to unexpected tasks and changes in technology. Phone Tech is utilizing a(n) ____ structure. (((organic)))
According to the law, which of the following is considered an employment test? (((any procedure used in the employment selection decision process)))
The more subunits into which an organization breaks down, the (((more highly differentiated it becomes.)))
The Myers-Briggs Type Indicator is a(n) ______ test. (((personality)))
Specialists work together to achieve ________ in a highly integrated organization. (((a common goal)))
When Jacqueline interviewed for a position as a computer programmer, she was asked to take a test on a particular programming language. Jacqueline took a(n) ______ test. (((performance)))
The scientists, nurses, and doctors who work for Medical Technology, an organization that researches the cure for several diseases, work together for a common goal of ending these illnesses. Medical Technology provides an example of (((integration.)))
_____ is the degree to which a test measures the same thing consistently. (((Reliability)))
A formal chain of command, the standardization of rules and procedures, and the use of cross-functional teams and computer networks so that there is frequent communication and coordination of the parts are the means for achieving (((a common goal through integration.)))
Which of the following is NOT one of the five steps in the training process? (((attendance)))
Human resource (HR) management consists of the activities managers perform to plan for, attract, develop, and retain an effective workforce. (((TRUE)))
Haley, an HR manager for Accurate Tool and Die, had a meeting with the Vice President of Safety to determine if the new safety training program appears to be effective in preventing and reducing the number of accidents on the shop floor. In which step of the training process is Haley engaged? (((evaluation)))
A job specification describes the minimum qualifications a person must have to perform the job successfully. For example, a job specification for an editor may require all editors to have flawless grammar skills. (((TRUE)))
Which of the following training and development methods is most appropriate when people just need to learn facts such as work rules or legal matters? (((videotapes, workbooks, and lectures)))
The 2010 health care reform legislation requires employers with less than 50 employees to provide health insurance. (((FALSE)))
Sabra, an HR manager, is designing a training class for those working on the new cross-functional teams within her company. This class is aimed at improving group decision making and interpersonal relations. What method of delivery should Sabra choose? (((role-playing, practice, and discussion)))
The Family and Medical Leave Act requires employers to provide 12 weeks of unpaid leave for medical and family reasons, including for childbirth, adoption, or family emergency. (((TRUE)))
Which of the following is a typical off-the-job training method? (((e-learning)))
The Equal Pay Act requires an extension of health insurance benefits after termination (((FALSE)))
___________ refers to educating technical and operational employees in how to better do their current job. (((Training)))
Performance appraisal consists of (((assessing performance and providing feedback.)))
The most effective sources of new employee sources are job postings on websites. (((FALSE)))
MBO is consistent with a(n) ______ performance appraisal. (((objective)))
An example of a behavioral-description interview question is "What was the best idea you ever sold to a supervisor, teacher, peer, or subordinate?" (((TRUE)))
_____ are subject to validity problems due to evaluator bias. (((Trait appraisals)))
A company that evaluates its long-haul truck drivers based on miles driven over the past 12 months is using subjective appraisal (((FALSE)))
Trait appraisals (((trait)))
The type of appraisal in which employees are evaluated by their managers, peers, and subordinates is called the 360-degree assessment. (((TRUE)))
The type of performance appraisal that judges specific, observable aspects of performance like being on time for work is a(n) ______ appraisal. (((behavioral)))
Benefits are additional nonmonetary forms of compensation designed to enrich the lives of all employees in the organization. Common benefits include medical insurance, vacation days, and credit unions, all paid for in whole or in part by the company. (((TRUE)))
A feedback technique in which an employee is appraised by her boss, peers, subordinates, and possibly clients is called a __________ appraisal. (((360-degree)))
The great majority of union workers are in the public sector. (((TRUE)))
Using 360-degree feedback appraisals makes it more difficult for managers to (((unfairly favor or punish particular employees.)))
Mediation is the process in which a neutral third party, an arbitrator, listens to both parties in a dispute and makes a decision that the parties have agreed will be binding on them. (((FALSE)))
__________ is a performance review system in which all employees within a business unit are ranked against one another. (((Forced ranking)))
Which of the following is NOT one of the soft skills employers often find lacking in many Millennial applicants? (((familiarity with social media)))
____________ includes the activities managers perform to plan for, attract, develop, and retain an effective workforce. (((Human resource management)))
Which of the following is NOT part of the strategic human resource management process? (((Negotiate employment contracts.)))
The point of the performance appraisal is to (((stimulate better job performance.)))
What is the final step in the strategic human resource management process? (((Perform appraisals of people)))
The best type of performance appraisal involves (((giving employees continuous, real-time coaching, feedback, and solutions.)))
________ capital is the economic or productive potential of employee knowledge and actions (((Human)))
Which of the following is the best statement of performance feedback? (((Your last report contained seven errors which I think you could improve.")))
_______ capital is the economic or productive potential of strong, trusting, and cooperative relationships. This type of capital can help you land a job; a national survey of recruiters revealed that 74% had found the highest-quality applicants through employee referrals. (((Social)))
Which of the following statements is NOT true regarding performance appraisals and feedback? (((Almost all workers believe that performance appraisals are objective and fair.)))
Which of the following is NOT a type of compensation? (((promotions)))
A specialist at UPS who rides with couriers to learn about delivery time measurements and potential service problems is conducting a job (((analysis.)))
Which of the following is NOT an example of an incentive? (((education reimbursements)))
Michele had been working as an executive assistant to the president for nearly 25 years, so when she retired, no one had a good idea of all that her job entailed. Before Michele retired, Sharon sat with her for two weeks to observe her duties and ask her to explain all the functions she performs as executive assistant to the president. Sharon was performing a(n) (((job analysis.)))
___________ are generous severance packages, negotiated by executives, which they receive if the company is taken over by another company. (((Golden parachutes)))
A ______ summarizes what the holder of the job does and how and why he or she does it. For example, this document might state, "In this job, the worker delivers pizzas within a 10-mile radius of the pizzeria, quickly and promptly, while obeying all traffic laws and driving responsibly." (((job description)))
Which of the following is NOT a characteristic of a successful incentive pay plan? (((complex and comprehensive rules)))
In managing employees under an incentive pay plan, a manager should (((regularly communicate with employees about the plan.)))
Which of the following is NOT a principal reason for transferring an employee? (((The employee is suspected of illegal behavior.)))
The ______________ enforce(s) procedures allowing employees to vote to have a union and for collective bargaining. (((National Labor Relations Board)))
Being dismissed "for cause" means that (((an employee is being fired for poor job performance or unacceptable behaviors.)))
Which piece of legislation first established the U.S. federal minimum wage? (((Fair Labor Standards Act of 1938)))
The risk of "wrongful termination" lawsuits is reduced with careful (((documentation.)))
The Equal Employment Opportunity Commission was established by _________________. (((the Title VII of the Civil Rights Act)))
______ means that anyone can be dismissed at any time for any reason at all, or for no reason. (((Employment at will)))
Which of the following helps guarantee nonhazardous working conditions for employees? (((Occupational Safety and Health Act)))
The three types of dismissals are ______, downsizings, and layoffs. (((firings)))
The ____________ is a law that requires an extension of health insurance benefits after termination. (((Consolidated Omnibus Budget Reconciliation Act (COBRA))))
The ________________ requires employers with more than 50 employees to provide health insurance. (((Patient Protection and Affordable Care Act)))
The ___________________ is the part of the labor-management agreement that states that employees who receive union benefits must join the union, or at least pay dues to it. (((union security clause)))
Allie has a degenerative eye disease that is causing her to gradually lose her eyesight. When Allie asked her employer, TrueBlue Fashions, to purchase a low-vision computer for her use at work, she was fired. TrueBlue Fashions has violated the (((Americans with Disabilities Act (ADA).)))
In 2011, when automakers began to create new jobs, new union hires were offered about half the pay ($14 an hour) that autoworkers were getting before ($28). This is an example of (((two-tier wage contracts.)))
A binding decision may be sought from ________ if a grievance procedure is not sufficient to solve a labor-management dispute (((an arbitrator)))
Beginning with __________, a body of law has grown that requires organizations to provide employees with nonhazardous working conditions. (((the Occupational Safety and Health Act (OSHA) of 1970)))
The marketplace is becoming less homogeneous and moving toward more niche products. (((TRUE)))
Demassification refers to customers becoming part of mass markets that reflect mass behavior and mass values. (((FALSE)))
Reactive change involves making carefully thought-out changes in anticipation of possible or expected problems or opportunities. (((FALSE)))
B corporations are ones whose stocks are not listed on NASDAQ or as part of the Dow Jones Industrial Average. (((FALSE)))
An example of innovative change is a department store deciding to adopt a new practice used by competitors by staying open 24 hours a day and requiring employees to work flexible schedules, a change that employees are likely to see as moderately threatening. (((TRUE)))
Adaptive change is the least threatening type of change and is therefore least likely to create resistance. (((TRUE)))
Corporate restructurings threaten to eliminate jobs and generally trigger strong resistance. (((TRUE)))
In OD, single interventions have been found to work better than multiple interventions. (((FALSE)))
Innovation is mostly the product of hard work and dedication rather than a "eureka" moment of epiphany. (((TRUE)))
A process innovation is a change in the appearance or the performance of a product or a service or the creation of a new one. (((FALSE)))
An organizational culture that celebrates failure helps foster innovation. (((TRUE)))
Which of the following is an effective way to deal with change and innovation? (((Have the courage to follow your ideas.)))
Which of the following is not a supertrend shaping the future of business? (((information becoming a competitive advantage)))
Which of the following is an example of a proactive change? (((Ciara explores improvements in bonus structures with her staff and begins to implement them despite the fact that her employees are generally content.)))
Which of the following is an example of a force for change originating outside the organization? (((social and political pressures)))
The increasing diversity of the American workforce, and the fact that people aged 18–34 are more likely to be living with their parents than with a spouse or partner, are both examples of a(n) ______ force for change. (((demographic)))
The invention of a machine to make plastic corks for wine bottles has severely affected companies that produce traditional cork. The invention of the machine for making plastic corks is an example of a(n) ______ advancement. (((technological)))
In the Internet Age, retailers like Amazon and Apple are not constrained by physical shelf space and can offer consumers a much wider variety of products; yet small sales, one or two rather than millions of items at a time, can produce big profits. This development is an example of (((the marketplace becoming more segmented and moving toward more niche products.)))
Which of the following is an inside force for change? (((low productivity and turnover)))
Social media (LinkedIn, Facebook) can make the search for new employees more efficient. The rise of social media is an example of a(n) __________ advancement or force. (((technological)))
Which of the following is not an inside force that indicates organizational change might be needed? (((increased competition)))
The human resource manager at Helping Hands Inc. has just calculated that the employee absenteeism rate for 2016 is 15 percent higher than it was in 2015, and the turnover rate increased 18 percent over the same time period. Which of the following is NOT an option she should consider for improving the situation? (((Wait until 2017 and see what the turnover rate is at the end of that year.)))
Sparkling Clean, Inc. is considering implementing a system that will pay its cleaning workers based on the number of completed residential jobs, coupled with satisfactory ratings on random inspections to ensure quality. This system is new to the organization, and it is an example of __________ change. (((innovative)))
Northstar Insurance is about to begin using a program that will change the way its adjusters settle insurance claims. Adjusters will be able to complete an adjustment and issue a check right at the scene of the accident. Although employees are uncertain that the new system will work, other insurance agencies have been successful with this approach. At Northstar Insurance, the new program represents ________ change. (((innovative)))
________ is an emotional/behavioral response to real or imagined threats to an established work routine. (((Resistance to change)))
Which of the following is not a factor that affects the level of resistance to change? (((the gender of the change agent)))
Employees are likely to see an adaptive change as (((least threatening.)))
At Toys 4 Tiny Tots, employees know that during the December shopping season they are often required to work different schedules, weekends, and overtime. The December work schedules at Toys 4 Tiny Tots is an example of a(n) ______ change because workers have experienced it in the past. (They experience it every December.) ((( adaptive)))
___________ change introduces a new practice to an organization but one that is not new to the industry. (((Innovative)))
An innovative change involves ______ complexity, cost, and uncertainty. (((moderate)))
Teachers Credit Union has decided that tellers must rotate through a new weekend shift on Saturday afternoons because several of its financial competitors have recently begun to offer additional hours to customers. Teachers Credit Union is implementing a(n) ______ change (((innovative)))
A(n) _____ change involves introducing a practice that is new to the industry, such as using drones (rather than delivery trucks) to deliver products to customers. (((radically innovative)))
Links Cable Network has decided to offer a one-hour appointment window for customers needing installation or repair of its service, which will require the company to have several technicians on call. Links hopes this practice will give it an advantage over the competition, none of which have adopted such a practice. Links Cable Network is introducing a(n) ______ change (((radically innovative)))
Which of the following is not a leading reason that employees resist change? (((lack of personal ethics)))
_______ is the process by which a company compares its performance with that of high-performing organizations. (((Benchmarking)))
_____________ is the set of techniques used for implementing planned change to make people and organizations more effective. (((Organizational development)))
A _______ often puts organizational development into practice. (((change agent)))
Which of the following is not one of the primary uses of OD? (((improving recruitment)))
In which one of the following situations would OD techniques be the most helpful? (((dealing with employee stress that comes from revitalizing an organization or adapting to a merger)))
As an OD consultant, Sharon is designing a survey of employee attitudes to be given to workers at SW Emergency Clinic, a provider of emergency-care services in a low-income neighborhood. In collecting this information, Sharon is conducting the ______ stage of OD. (((diagnosis)))
What shall we do about the problem? is the question that would most likely be asked during the ______ stage of OD. (((treatment or intervention)))
Omar, an OD consultant, is working with members of a cross-functional team to build cohesiveness and practice skills to function better as a team. Omar is conducting the ______ stage of the OD process. (((treatment, or intervention,)))
A year after she assisted Thomas Engineering in making changes to an employee incentive plan, Kara, an OD consultant, is visiting the company to determine if the changes were helpful. Kara will compare sales and turnover data from the last three years to the current year. Kara is in the ______ stage of the OD process. (((evaluation)))
Which of the following circumstances is less unlikely to increase the success of OD? (((single intervention)))
A(n) ______ innovation is a change in the way a product or service is conceived, manufactured, or disseminated. (((process)))
The management department at a local university began posting all assignments and other class materials to a course management website instead of creating a packet or printed materials for students to purchase each term. The management department is implementing a ______ innovation. (((process)))
Which of the following jobs are least likely to be taken over by automation? (((knowledge-work jobs that require creativity and problem solving)))
Suppose a soft-drink company, Loco Cola, is conducting a force-field analysis to determine which forces could facilitate a proposed change (the introduction of a new lemon-lime soda) and which forces could act against it. Which of the following would Loco Cola consider a counterthruster? (((Loco Cola has never been able to gain any significant market share outside of the Midwest.)))
Personality test data should be supplemented with information from reference checks, personal interviews, ability tests, and job performance records when hiring. (((TRUE)))
Integrity tests are ineffective because dishonest people are able to fake conscientiousness, even on a paper-and-pencil test. (((FALSE)))
Those who have the Type A behavior pattern are involved in a chronic, determined struggle to accomplish more in less time. (((TRUE)))
Internals likely would prefer and respond more productively to incentives such as merit pay or sales commissions. (((TRUE)))
Individuals with an external locus of control tend to display greater work motivation, which leads to higher salaries. (((FALSE)))
Lee has hired two new employees for her team, Jim and Judy. Jim is outgoing and attractive, while Judy is very bright but seems quiet and unsure. Lee immediately expects Jim to outperform Judy at the job. Lee is likely experiencing the halo effect. (((TRUE)))
After a recent exam, students earning Ds blamed factors such as bad luck, unclear lectures, and unfair testing for their poor performance. These students are engaged in self-serving bias. (((TRUE)))
One of the ways to create a Pygmalion effect is to encourage employees to visualize failing at executing tasks and use their fear of failure to motivate them. (((FALSE)))
An example of a self-fulfilling prophecy is a server who expects some poorly dressed customers to be stingy tippers, who therefore gives them poor service, and so gets the result he expected—a much lower tip than usual. (((TRUE)))
Investors who are more likely to buy a stock if they see something about it in the news or if it has a high one-day return provide an example of the recency effect. (((TRUE)))
The glass ceiling is a concept that applies to women, but not to minorities. (((FALSE)))
Roberto finds his job description and the criteria for promotion vague, an example of role ambiguity. (((TRUE)))
Workplace stress diminishes all of the following except (((job turnover.)))
______ consists of stable psychological traits and behavioral attributes that give a person his or her identity. (((Personality)))
Which of the following is not one of the Big Five personality dimensions? (((tolerance for ambiguity)))
______ is the personality dimension that describes how intellectual, imaginative, curious, and broad-minded a person is. (((Openness to experience)))
______ is the personality dimension that describes how achievement-oriented and persistent a person is. (((Conscientiousness)))
Matt is frequently nervous, tense, and worried, both at work and at home. He likely scores low on which of the Big Five personality dimensions? (((emotional stability)))
Julio really enjoys mingling at work functions, both to network for new contacts and simply to share stories with other interesting people. Julio probably scores high in (((extroversion.)))
Ana Maria believes that she controls her own destiny. She is a rather shy person who is extremely dependable and responsible, and her coworkers respect her greatly. Ana Maria has a(n) ______ locus of control and would rate high on the _______ dimension of personality. (((internal, conscientiousness)))
Paolo is always coming up with new ideas and seeking funding to explore them. He sees opportunities in the marketplace and wants to be the first person to come up with new products and services that customers want. Paolo, a person who is apt to take initiative and persevere to influence the environment, has a(n) ______ personality. (((proactive)))
If you have a(n) ____________, you believe you control your own destiny. (((internal locus of control)))
People with ______ exhibit less anxiety, greater motivation, and stronger expectations that effort leads to performance (((an internal locus of control)))
Employees with a(n) ______ locus of control will probably resist close managerial supervision and should probably be placed in jobs requiring high initiative and lower compliance. (((internal)))
__________ is the extent to which people feel secure and unworried. (((Emotional stability)))
Brad supervises several employees who seem to have low self-efficacy. To improve this situation, he should do all the following except (((avoid rewarding small successes.)))
_____________ is the extent to which people like or dislike themselves. (((Self-esteem)))
People with low self-esteem ______ than those with higher self-esteem. (((are more dependent on others)))
Those high in this emotional intelligence trait tend to be empathetic. (((social awareness)))
Which of the following is not a trait of emotional intelligence? (((self-sufficiency)))
_______________ is the ability to control your emotions and act with honesty and integrity in reliable and adaptable ways. (((Self-management)))
The interdisciplinary field of organizational ______ tries to explain and predict workplace behavior to help managers better lead and motivate others. (((behavior)))
The three components of ______ are affective, cognitive, and behavioral. (((attitudes)))
The ______ component of an attitude consists of the beliefs and knowledge one has about a situation. (((cognitive)))
The statement "Drew is slow to return phone calls" reflects the ______ component of an attitude. (((cognitive)))
Because people are uncomfortable with inconsistency between their attitudes and behaviors, they will seek to reduce (((cognitive dissonance.)))
Under which of the following circumstances would the desire to reduce cognitive dissonance be greatest? (((when large amounts of money are on the line)))
Which of the following is not one of the main ways to reduce cognitive dissonance? (((Complete an emotional intelligence assessment.)))
According to Festinger, how people deal with cognitive dissonance depends on three factors. Which are the correct three factors? (((importance, control, and rewards)))
Which of the following is not a step in the perceptual process? (((causal attribution)))
Which of the following is a distortion in perception? (((cognitive dissonance)))
________ is the tendency to attribute to an individual the characteristics one believes are typical of the group to which that individual belongs. (((Stereotyping)))
Ted was hiring a new financial analyst, and he had several good candidates. He was leaning toward hiring Akiko, a Japanese American woman, because he thinks Asians are better at math. Ted is exhibiting which distortion in perception? (((stereotyping)))
______ occurs when we form an impression of an individual based on a single trait. (((The halo effect)))
______ is the tendency to remember recent information better than earlier information. (((The recency effect)))
A server expects a group of poorly dressed customers to be stingy tippers and gives them poor service, so he gets the result that he expects, a very small tip. This is an example of (((a self-fulfilling prophecy.)))
________________ is the extent to which employees have positive or negative feelings about various aspects of their work. (((Job satisfaction)))
Job satisfaction results in stronger ______ and lower levels of ______. (((organizational commitment; perceived stress)))
____ is the extent to which an employee identifies with an organization and is committed to its goals. (((Organizational commitment)))
________ programs help employees integrate and transition to a new job. (((Onboarding)))
________ behaviors are employee behaviors that exceed the work-role requirements for the job. (((Organizational citizenship)))
Melissa impressed her boss by making many suggestions for department improvements and by putting in many weekend hours to do extra tasks to develop the new procedures. Melissa is exhibiting (((organizational citizenship behavior.)))
Absenteeism, drug and alcohol abuse, and disciplinary problems are examples of (((counterproductive work behaviors)))
______ represent(s) all the ways people are unlike and alike. (((Diversity)))
Which of the following is not one of the four layers of the diversity wheel? (((management status)))
______ is at the center of the diversity wheel. (((Personality)))
Organizational dimensions, the outer layer of the diversity wheel, include all of the following except (((ethnicity.)))
Which of the following dimensions is not an example of an external or secondary dimension on the diversity wheel? (((seniority)))
Which of the following dimensions is an example of an organizational dimension on the diversity wheel? (((union affiliation)))
Which of the following diversity issues is true in the U.S. workforce? (((By 2060, the white population is projected to change from 77.5 percent in 2014 to 68.5 percent, African Americans from 13.2 percent to 14.3 percent, Asians from 5.4 percent to 9.3 percent, Hispanics or Latinos from 17.4 percent to 28.6 percent, and American Indian/Alaskan Native from 1.2 percent to 1.5 percent)))
Which of the following is a common reason that some women face a glass ceiling? (((lack of mentors)))
Investors who are more likely to buy a stock if they see something about it in the news or if it has a high one-day return are experiencing (((the recency effect.)))
______ is the tension people feel when they are facing or enduring extraordinary demands, constraints, or opportunities and are uncertain about their ability to handle them effectively. (((Stress)))
An example of _______ would be when a manager expects employees to perform badly, they probably will, and when the manger expects them to perform well, they probably will. (((a self-fulfilling prophecy, or the Pygmalion effect,)))
You have important clients in town who want to have dinner with you, but your mother has a plumbing leak and has asked you to come over and shut off the water until she can get it fixed tomorrow. You are experiencing (((role conflict.)))
Which of the following is not a symptom of negative stress? (((slurred speech)))
______ is a state of emotional, mental, and even physical exhaustion, expressed as listlessness, indifference, or frustration. (((Burnout)))
Which of the following is the most common drug of abuse? (((alcohol)))
. A(n) ______ program includes plans to help employees cope with stress, burnout, substance abuse, health problems, and family issues that influence job performance. (((employee assistance)))
Which of the following is not a suggested strategy for reducing unhealthy stressors in organizations? (((develop a more formal structure with close, authoritative supervision)))
Employees at KarKare, Inc., believe that the company is fair to all employees, regardless of their age, gender, race, ethnicity, sexual orientation, or disability. It is accurate to say that KarKare has a positive (((diversity climate.)))
General Electric paid employees who were smokers up to $750 to quit and stay off cigarettes. This program was three times as successful as a different GE "quit smoking" program in which employees received no paid incentives. The $750 in the first program is an example of an intrinsic reward. (((FALSE)))
Those with a high need for achievement may be the least efficient managers because they may resist making decisions that make others resent them. (((FALSE)))
According to equity theory, employees who feel they are being underrewarded are more likely to support organizational change, more apt to cooperate in group settings, and less apt to turn to arbitration and the courts to remedy real or imagined wrongs. (((FALSE)))
To be most motivating, goals should be specific and challenging but achievable. (((TRUE)))
Simplified, repetitive jobs lead to job dissatisfaction, poor mental health, and a low sense of accomplishment and personal growth (((TRUE)))
The task significance for a craftsperson who goes through all the steps to build a handmade acoustic guitar is higher than it is for an assembly-line worker who just installs windshields on cars. (((FALSE)))
To use positive reinforcement, give praise to employees not for showing up for work on time (an expected part of any job) but for showing up early. (((TRUE)))
Most workers rate monetary benefits higher than a caring boss. (((FALSE)))
Goal-setting theory suggests that behavior tends to be repeated if it has positive consequences and tends not to be repeated if it has negative consequences. (((FALSE)))
A manager should administer punishment in public in front of employees because the embarrassment enhances the punishment and its effectiveness. (((FALSE)))
New research shows that the cubicle is not a motivational environment, stifling creativity and morale for many workers. (((TRUE)))
Intrinsic motivation is not as long lasting as extrinsic motivation, and intrinsic motivation has a less positive impact on task performance (((FALSE)))
______ is the set of psychological processes that arouse and direct goal-directed behavior. (((Motivation)))
Which of the following is not part of a simple model of motivation? (((punishment)))
Content perspectives, also known as need-based perspectives, include all except which one of the following theories? (((equity theory)))
physiological, safety, love, esteem, and self-actualization. (((esteem)))
To achieve psychological growth, according to the self-determination theory, people need to satisfy the three innate needs: _______, autonomy, and competence. (((relatedness)))
According to the acquired needs theory, the desire to influence others is part of the need for ((( power)))
Sharon has excelled as an auditor for a large financial consulting firm. She loves building mutually beneficial relationships and is extremely well liked by her clients. Sharon likely has a (((high need for affiliation.)))
Sue is very aggressive at her job, and her coworkers complain that she often manipulates them and her boss in order to get ahead. She attempts to direct everything and everyone, regardless of the consequences to others or even the organization itself. Sue is expressing what type of need for power? (((personal)))
According to Herzberg, which of the following is an example of a motivating factor? (((the work itself)))
According to Herzberg, which of the following is an example of a hygiene factor? (((interpersonal relationships)))
______________ is a model of motivation that explains how people strive for fairness and justice in social exchanges or give-and-take relationships. (((Equity theory)))
Clint is a branch manager for a large freight company. He has noticed low morale lately, perhaps because of the cramped quarters, stricter policies, and lack of raises this year. According to Herzberg, Clint should first concentrate on (((hygiene factors)))
_______ perspectives are theories that try to understand the thought processes by which people decide how to act. (((Process)))
Which of the following is a process perspective on motivation? (((goal-setting theory)))
In equity theory, employees are motivated to (((resolve feelings of injustice and see fairness in the rewards they expect for task performance.)))
When workers perceive they are being treated fairly on the job, they are most likely to (((support organizational change.)))
The theory under which people make the choice that promises them the greatest reward if they think they can get it is (((expectancy theory.)))
After struggling with the accounting training, Peter is unsure whether he can complete the end-of-year financial reporting with no errors in the time allotted. In this case, Peter is low on the ______ element of expectancy theory. (((expectancy)))
______ is the expectation that successful performance of the task will lead to the desired outcome. (((Instrumentality)))
The HR manager told Jim that the company pays the total health insurance costs for a family of four. As a single man, this benefit did not seem especially important to him right now. Here, Jim is low on the ______ element of the expectancy theory. (((valence)))
When using goal-setting theory to motivate employees, managers should (((help workers understand and accept the goals.)))
Goal-setting theory suggests that employees can be motivated by goals that have all of the following characteristics except (((little or no feedback.)))
According to goal-setting theory, goal setting helps motivate you by doing all the following except (((taking attention away from real work.)))
Which of the following is the best statement of a goal? (((Reduce production errors by 10 percent by the end of the quarter.)))
__________ is the process of increasing the number of tasks in a job to increase variety and motivation. (((Job enlargement)))
Research shows that simplified jobs lead to all the following except (((increased productivity.)))
Job ______ is the opposite of ______. (((enlargement; scientific management)))
______ is the process of building into a job such motivating factors as recognition and achievement. (((Job enrichment)))
Of the following, which is not a core job characteristic? (((teamwork)))
_______ is the extent to which a job allows an employee to make decisions about scheduling different tasks and deciding how to perform them. (((Autonomy)))
Victoria runs a flower shop. She recently made some changes so that one employee answers phones and completes the order and payment process, while another creates arrangements, and a third packages and delivers floral arrangements. Previously, each employee was responsible for the order from the phone call through the delivery. Victoria has made changes to (((task identity.)))
Which of the following is NOT a contingency factor in the job characteristics model? (((autonomy)))
The motivating potential score (MPS) is calculated as part of using (((the job characteristics model.)))
____________ is the theory that attempts to explain behavior change by suggesting that behavior with positive consequences tends to be repeated while behavior with negative consequences tends not to be repeated. (((Reinforcement theory)))
_______ is the use of desirable consequences to strengthen a particular behavior. (((Positive reinforcement)))
______ is the process of strengthening a behavior by withdrawing something negative. (((Negative reinforcement)))
_____ is the weakening of behavior by ignoring it or making sure it is not reinforced. (((Extinction)))
It has been two years since any employee has received a bonus at Tech Outlet, and the possibility of getting one no longer seems to motivate employees. In this case, management has inadvertently applied (((0)))
In using reinforcement, a manager should ((( clearly communicate the desired behavior.)))
When using punishment, a manager should (((do it in conjunction with positive reinforcement.)))
Which of the following is not an advisable criterion for an effective incentive plan? (((The rewards require record-setting performance.)))
A sales commission is an example of a ______ compensation plan. (((pay-for-performance)))
José has a summer job hand-dyeing shirts that will be sold on the boardwalk. He is paid $5.00 per shirt. José is being paid on a ______ compensation plan. (((pay-for-performance)))
A Scanlon plan is a type of ______ compensation plan. (((gainsharing)))
Because the production division at Heavenly Gates Inc. is $10,000 below budget due to process improvements, this year 30 percent of the savings will be returned to the division to spend as desired, as a form of (((gainsharing)))
Which of the following is not a practice used to create a flexible workplace? (((gainsharing)))
A group is defined as two or more freely interacting individuals who share norms, share goals, and have a common identity. (((TRUE)))
Common commitment is the essence of a team, and what differentiates it from a group. (((TRUE)))
Twenty-six new employees from Global Technologies Inc. are attending orientation, receiving information about company policies, and filling out various forms. These employees comprise a group. (((TRUE)))
A self-managed team is designed to include members from different areas within an organization, such as finance, operations, and sales. (((FALSE)))
The majority of workplace learning is informal. (((TRUE)))
An example of a maintenance role is an initiator, someone who suggests new goals or ideas. (((FALSE)))
Norms are a typically explicit and are often discussed openly. (((FALSE)))
Teams often enforce norms to help individuals avoid embarrassing situations. (((TRUE)))
Conflict is a process in which one party perceives that its interests are being opposed or negatively affected by another. (((TRUE)))
A moderate level of conflict can induce creativity and initiative, thereby raising performance. (((TRUE)))
Stephen and Tyler went to the same college, and they have never liked each other. Stephen thinks Tyler is a loudmouth, while Tyler thinks Stephen is arrogant and self-impressed. If Stephen and Tyler end up working for the same company, they are likely to experience a multicultural conflict. (((FALSE)))
When parties to a conflict have deeply rooted, opposing value systems, collaboration is inappropriate for resolving the conflict. (((TRUE)))
Which of the following is the best way to manage virtual teams? (((Check in regularly, but resist the urge to micromanage.)))
Check in regularly, but resist the urge to micromanage. (((meet regularly, face-to-face.)))
Which of the following is not a benefit of teamwork in an organization? (((increased stress)))
Two or more freely interacting individuals who share norms, share goals, and have a common identity form a (((group)))
A _______ is a small group of people with complementary skills who are committed to a common purpose, performance goals, and approach for which they hold themselves mutually accountable (((team)))
A(n) ________ group is created to do something productive for the organization and is headed by a leader. For example, this group may be created to develop a new line of skin-care products specifically for people over the age of 60. (((formal)))
Which of the following is true of informal groups? (((Informal groups can advance or undercut the plans of formal groups.)))
In all but the worst weather, Beth and three coworkers met each day at 12:15 to walk the wilderness trail behind their office building. The four co-workers are an example of a(n) (((informal group.)))
A team composed of people from different departments who are pursuing a common objective is called a _____. For example, ToysRUs may bring together employees from manufacturing, design, sales, marketing, and market research to brainstorm new board games for adults. (((cross-functional team)))
Rita is on a team that consists of members who work together over time and distance via electronic media to combine effort and achieve common goals. Rita is on a(n) (((virtual team.)))
At Allen and Thomas Architects, a group of designers are developing the first drawings for a proposed multiuse office development in an older section of a large city that is being revitalized. What type of work team is this group? (((project team)))
Self-managed teams are groups of workers who have been given ______ for their task domains. (((administrative oversight)))
Administrative oversight given to self-managed teams does not typically include (((hiring and firing.)))
When using a self-managed team, a manager should (((delegate administrative oversight of work scheduling and customer interaction.)))
Which of the following is not one of the five stages of team development? (((reforming)))
The question the group is asking during the forming stage of group development is ((("Why are we here?")))
During the forming stage of a group or team, the leader should (((allow people to socialize and become acquainted.)))
_________, the second stage of team development, is characterized by the emergence of individual personalities and roles and conflicts within the group. (((Storming)))
During the ______ stage of team development, close relationships develop as unity and harmony emerge. (((norming)))
The question that individuals ask during the norming stage of group development is ((("What do the others expect me to do?")))
The stage of team development during which a group sets guidelines about issues like attendance and punctuality is the ______ stage. (((norming)))
Groups that make it through storming generally do so because (((a respected member other than the leader challenges the group to resolve power struggles.)))
The principal by-product of the norming stage of team development is (((group cohesiveness)))
______ is a "we feeling" that binds group members together. (((Group cohesiveness)))
helping the team identify group goals and values. (((concentrate on solving problems and completing tasks.)))
During the performing stage of team development, the group answers the question ((("Can we do the job properly?")))
Which of the following is not considered an essential consideration in building a group into an effective team? ((( unanimity)))
Individuals are said to be collaborating when (((they are sharing information and coordinating efforts to achieve a collective outcome.)))
Trust is based on _______, which in turn is based on past acts of integrity and follow-through on promises. (((credibility)))
Members of a team develop their ______ based on the expectations of the team, of the organization, and of themselves (((roles)))
Jamal points out during the meeting that the group has fallen a half hour behind schedule according to the agenda, and should get back to the important work at hand. He is performing a ______ role. ((( task)))
Which of the following is not a reason why group or team members typically enforce norms? (((to create a written document of behavioral guidelines for new employees)))
Dysfunctional conflict is sometimes called ______ conflict. (((negative)))
Which of the following is not characteristic of workgroups with excessive conflict? (((creativity)))
Which of the following is a manifestation of excessive conflict in the workplace? (((violence and workplace aggression)))
Workplace performance is maximized when (((conflict is at a moderate level.)))
________ conflict is defined as interpersonal opposition based on personal dislike, disagreement, or differing styles. For example, Sasha and Alexandra, who both work as salespeople at a local boutique, may simply not like each other. (((Personality)))
Maneka is on a team with Larson, and they are often in conflict. Maneka likes to begin her work with careful planning and she gets started immediately. Larson, on the other hand, likes trying out several ideas, and tends to be working frantically at the last minute. Their conflict most likely stems from (((clash of personality, values, and attitudes)))
Which of the following is a source of intergroup conflict in the workplace? (((inconsistent goals or reward systems)))
Ken, whose primary job is supervising a small production group, is not getting cooperation from all members on the cross-functional team he leads. In particular, Bethany, a senior marketing manager, seems to resist his direction and tries to influence team members to go in another direction. The source of conflict in this case may be (((status differences.)))
____________ is ordering an outcome, when a manager relies on his or her formal authority and power to resolve a conflict, but the needs of the other party are largely ignored. (((Forcing)))
_____ is the conflict-handling style in which a person neglects his or her own concerns and allows the concerns of another to prevail. (((Accommodating)))
When issues causing conflict are trivial, or when emotions are high and cooling off would be helpful, which conflict-handling style is preferable? (((avoiding)))
Larry proposed the purchase of a new Epson copier to his boss. Amir said that he has always used and liked Xerox copiers, but when he thinks about it later, he realizes he does not really feel strongly about the brand of copy machine used in the office. Amir should probably adopt a(n) ______ conflict-handling style when trying to resolve this matter with Larry. (((accommodating)))
Sharon told her team that she is moving forward with a change to the bonus structure, despite vocal objections from several team members. Sharon is using the ______ conflict-handling style. (((dominating)))
______ is the conflict-handling style in which both parties give up something to gain something. (((Compromising)))
The disadvantage of the ______ conflict-handling style is that it is very time-consuming. (((collaborating)))
In which of the following conditions should constructive conflict be stimulated? (((The group seems to be afflicted with inertia and apathy.)))
______ is the process of having two people or groups play opposing roles in a debate to better understand a proposal. (((The dialectic method)))
When Cisco Systems told executives they would gain or lose 30 percent of their bonuses based on how well they worked with peers, and in three years had record profits, the company (((improved workplace cohesiveness.)))
Leadership is the ability to influence employees to voluntarily pursue organizational goals. (((TRUE)))
Leaders manage and managers lead, but the two activities are not synonymous. (((TRUE)))
Employees who are having computer problems at TravelWorld go to Amy rather than the IT department because she is efficient and considerate about helping out and is extremely knowledgeable. Amy has expert power. (((TRUE)))
Emily has a charismatic personality, and finds it easy to get others to agree with her. Emily has referent power. (((TRUE)))
Narcissistic leaders tend to bring out the best work behaviors in others (((FALSE)))
The primary purpose of task-oriented leadership behaviors is to ensure that people, equipment, and other resources are used efficiently to accomplish a mission. (((TRUE)))
As a manager, when you provide rewards or reprimand your subordinates, you are practicing transactional leadership. (((TRUE)))
In the contingency leadership model, the most important component of situational control is position power (((FALSE)))
Adaptive, flexible organizational cultures are less likely to foster transformational leadership than bureaucratic ones. (((FALSE)))
Female leaders tend to use transformational leadership more than male leaders do. (((TRUE)))
Steve Jobs is an example of a transformational leader; he had a vision of developing an "insanely great" desktop computer. (((TRUE)))
Passive leaders inspire trust in their followers because they express their integrity by being consistent, single-minded, and persistent in pursuit of their goal. (((FALSE)))
Leaders benefit most from followers who are helpers (most compliant) rather than rebels (least compliant (((TRUE)))
The three dimensions of situational control are leader-member relations, the task structure, and (((position power.)))
Leadership is the ability to ______ employees to voluntarily pursue organizational goals. (((influence)))
Which of the following is not one of the five sources of power? (((appearance)))
The power that managers have resulting from their formal positions within organizations is called ______ power (((legitimate)))
Anar congratulated her staff when the team received an industry award for their project, and also sent a companywide e-mail announcing it. Here, Anar is using her ______ power. (((reward)))
Karl was put on probation for a string of customer complaints about his poor service and professionalism. His supervisor was using ______ power. (((coercive)))
Trina is outgoing, warm, and truly inspirational when she talks about where she wants to lead her division. Her employees seem eager to do anything she asks. Because of her personality and not her position, Trina has ______ power. (((referent)))
________, which include acting friendly or making someone feel good or important before making a request, are influence tactics. (((Ingratiating tactics)))
Using threats or intimidation to persuade someone is which influence tactic? (((pressure)))
The ______ approach to leadership attempts to identify distinctive characteristics that account for the effectiveness of leaders. ((( trait)))
Organizations may apply trait theory by (((using personality assessments.)))
Which of the following would be considered a negative interpersonal trait often found in leaders? (((Machiavellianism)))
Which of the following is characteristic of men in the workplace, compared with women? (((using a more autocratic style)))
______ leadership approaches attempt to determine the unique behaviors used by effective leaders. (((Behavioral)))
Which of the following is not a behavioral leadership approach? ((( trait)))
Research indicates that women tend to display more _____, while men tend to display more ______. (((relationship leadership; task leadership)))
James manages his employees by carefully monitoring their production, comparing what they do to predicted schedules and desired budgets. James can best be described as a ______ leader. (((task-oriented)))
Increasing employee psychological empowerment requires four kinds of behaviors, including all of the following except (((inspiration.)))
The approach to leadership that suggests that effective leadership behavior depends on the situation at hand is the ______ approach. (((contingency)))
One of three dimensions of situational control in the contingency leadership model, ______ is the extent to which tasks are routine, unambiguous, and easily understood. (((task structure)))
Erin, a department manager, is struggling trying to lead a cross-functional team. Most members are other department managers at her level, and they see no reason to do what she asks. Here, Erin has (((weak position power.)))
According to the contingency model, a relationship-oriented leadership style works best in ______ situations. (((moderate-control)))
Sammie leads a task force charged with restructuring the order-processing system in the organization. The task force is composed of her peers over whom she has little control, and some of whom do not seem to respect her. According to the contingency model, Sammie should adopt a(n) ___________ leadership style. (((task-oriented)))
Do my subordinates accept me as a leader? This question reflects which of the following dimensions of situational control? (((leader-member relations)))
Kim supervises a group of customer service representatives. Kim is respected and well liked by her staff, and she has worked hard to make sure each of them knows how to do their job well. She is responsible for all staffing and reward decisions in her department. Kim has _____ in the contingency model. (((high situational control)))
In the contingency model, if your leadership orientation does not match the situation in your workplace, Fiedler recommends that you (((move to a more suitable situation.)))
Which of the following is not an employee characteristic of relevance in path-goal theory? (((position power)))
James leads a training and development team. He expects excellence, sets challenging goals, and expresses confidence that all team members are capable of doing their part to meet and exceed the goals. According to revised path-goal theory, James is using a(n) ______ leadership style. (((achievement-oriented)))
Revised path-goal theory indicates to managers that they must consider (((modifying their leadership style to fit employee and task characteristics)))
According to the concept of full-range leadership, leadership behaviors vary along a range from ______ leadership at one extreme to transformational leadership at the other. (((laissez-faire)))
Transformational leadership is positively associated with all the following except (((less commitment to organizational change.)))
Which of the following is a characteristic of transformational leaders? (((They encourage people to do exceptional things with trust, commitment, and loyalty to produce significant results and change.)))
Charismatic leadership is now considered part of ______ leadership. (((transformational)))
Employees at Employees Credit Union were really impressed with Ryan, the new CEO. He was the first person who had fully developed a vision for the company about where it could go, and had energized everyone to try to get there. Ryan is a ______ leader. (((transformational)))
Which of the following is not positively associated with transformational leadership, according to research? (((less group cohesion)))
The ______ model of leadership emphasizes that leaders have different sorts of relationships with different employees. (((leader-member exchange (LMX))))
In the leader-member exchange model, the relationship between leader and follower that is characterized by mutual trust, respect, and liking is known as a(n) ________ exchange. (((in-group)))
A _______ leader is one who focuses on providing increased service to others. (((servant)))
Starbucks CEO Howard Schultz has made sure his employees have health insurance and work in a positive environment. He could best be described as which type of leader? (((servant)))
Amy, the manager of a branch location of a large travel firm, treated company resources as if they were her own and encouraged continued development and training of her employees. She cared about the staff deeply and even organized international volunteering activities to promote their growth. Amy could best be described as which type of leader? (((servant)))
Research shows that followers seek and admire leaders who create feelings of (((significance.)))
Leaders especially want ________ who are productive, reliable, honest, cooperative, proactive, and flexible. (((followers)))
Do my subordinates perform unambiguous, easily understood tasks? This question reflects the ______ dimension of situational control. (((task structure)))
Noise is any disturbance that interferes with the transmission of a message, and it can occur in any medium. (((TRUE)))
Face-to-face communication is the richest form of communication because the receiver observes multiple cues in the message. (((TRUE)))
If you must tell an employee that he is facing a sexual harassment investigation, you should use a rich medium of communication. (((TRUE)))
An order-taker who has a thick accent, and someone who speaks too fast at a fast-food drive-through, are examples of communication barriers. (((TRUE)))
When a supervisor is afraid to voice an idea because she fears criticism, the result is a receiver barrier. (((FALSE)))
One of the ways to improve communication is to avoid animation. (((FALSE)))
Formal channels of communication are typically faster than the grapevine. (((FALSE)))
In general, one long meeting is preferable to several shorter meetings. (((FALSE)))
When using text messages for important workplace communications, use abbreviations and emojis to streamline the communication. (((FALSE)))
Generally people retain about 75 percent of what they hear. (((FALSE)))
An audience can typically remember about 10 points from a speech. (((FALSE)))
Which of the following statements is not true about communication in the workplace? (((Emails can cut through the misunderstandings that result from misinterpreted phone calls.)))
Communication is (((the transfer of information and understanding from one person to another.)))
You are a(n) ________ communicator when your intended message is sent accurately in the least amount of time. (((efficient)))
Which of the following is not an element of the communication model? (((decoder)))
______________ is the process of translating a message into understandable symbols or language. (((Encoding)))
Which of the following is a step in the communication process? (((The message is transmitted through a medium.)))
A(n) ______ form of media allows receivers to observe multiple cues, such as body language and tone of voice, and allows senders to get feedback. Face-to-face communication is an example of this form of media. (((rich)))
Of the following, which medium is the richest? (((videoconference)))
Of the following, which medium is the least rich? (((flier on a bulletin board)))
A manager needs to inform employees that the quarterly project management training will start on May 1, about two months from now. Generally, which of the following should the manager use in this situation? (((e-mail)))
Executives at Breakers Engineering want to let middle managers know that the company is going to be acquired by its largest competitor. Assuming the following paths of communication are possible, how should executives communicate this information? (((Hold a face-to-face meeting.)))
Claire wanted to tell her employees about the seminar, but she put the flier in a stack of papers and forgot about it until after the deadline, so none of her employees were able to sign up for the seminar. What type of barrier has occurred in this situation? (((sender barrier)))
Lai worked on a global team for an American company, and all her work had to be completed in her second language, English. Sometimes her teammates misinterpreted her meaning. Lai has unintentionally created of a(n) ______ barrier to communication. (((encoding)))
Debbie works part time in a coffee shop. Some of the equipment is so noisy it prevents her from correctly hearing what the customer orders. The noisy equipment is an example of a(n) ______ barrier to communication. (((physical)))
Tracie, working at a research facility in Washington, DC, needs to communicate with Juan, who is in Mexico City, about a project they are working on. Although Juan has perfect command of English, they are still facing a(n) ______ barrier to communication. (((physical)))
____ is a state of reduced attention that is expressed in behavior that is rigid or thoughtless. This state is natural because human beings simply cannot keep up with all the stimuli they receive. (((Mindlessness)))
______ is terminology that is specific to a particular profession or group. For example, statisticians talk about t-tests, chi-square tests, confidence intervals, and normal distributions. (((Jargon)))
Which of the following is not a personal barrier to communication? (((obsolete technology)))
Tyrone was recently promoted to manager. Although he used to take advice from his peers, he seems no longer willing to listen to any input from them or his boss, and he is determined to run the project how he thinks best. Which personal barrier to communication is Tyrone exhibiting? (((an oversized ego)))
Which of the following is not a way in which nonverbal communication is expressed? (((writing a thank-you note)))
Most people from Western cultures tend to ______ when conveying bad news or negative feedback. (((avoid eye contact)))
Which of the following is a "do" when attempting to improve communication? (((Smile)))
Compared to women, men tend to (((use emoticons to express sarcasm and banter.)))
According to Deborah Tannen, men are more likely than women to (((boast about achievements.)))
Channels of communications that follow the chain of command and are considered as official are _____ communication channels (((formal)))
Which of the following indicates how official communications should be routed? (((organizational chart)))
A manager who is instructing his employee about work tasks is communicating (((vertically.)))
The more management levels through which a message passes, the (((more distorted the message may become.)))
The dean at Corbin Business College disseminated a new five-year strategic plan to department chairs. This is an example of ________ communication (((downward)))
Diane, a marketing manager, shares some of the market research her department has analyzed with the research and development group. Diane is communicating (((horizontally.)))
At the end of the fiscal year, Dilinger Investments releases an earnings report to clients. By sending the earnings reports to its clients, Dilinger Investments is communicating (((externally.)))
______ communication channels develop outside the organizational structure and do not follow the chain of command. (((Informal)))
Gossip and rumor in an organization are part of the _____, which itself is a type of ______ communication channel. (((grapevine; informal)))
The grapevine is (((more active when official communication is lacking.)))
The key to protecting digital communication systems against fraud, hackers, identity theft, and other threats is (((using prevention techniques like strong passwords.)))
When thieves use your name and good credit rating to get cash or buy things, they are engaging in (((identity theft.)))
Which of the following is not true of e-mail? (((The number of e-mail users worldwide has peaked and begun to decline.)))
Which of the following is appropriate for workplace e-mail? (((maintaining work-life balance by not sending e-mails before or after work hours)))
maintaining work-life balance by not sending e-mails before or after work hours (((the Internet and social media.)))
Generally, typical listeners retain _____ of what they hear. (((20-50 percent)))
Active ______ is the process of actively decoding and interpreting verbal messages, which requires full attention and processing. (((listening)))
Which of the following is not a suggestion that may help you be a good listener? (((mentally compose your response)))
Active listening requires all of the following except (((focusing on the delivery rather than content.)))
Which of the following is a tip for writing more effectively for business purposes? (((start by stating your purpose and what you expect of the reader)))
The introduction to a business speech should (((get right to the point)))
Which of the following is not appropriate to include in the conclusion of a speech? (((details you forgot from earlier)))
Which of the following is the best advice for face-to-face communication? (((Begin with good news, and lead up to bad news.)))
Imagine that you work for a life insurance company. The company is offering a new type of life insurance policy for senior citizens (people over age 65) and retirees. Keeping generational differences in mind, which of the following is the best medium for communicating with potential customers/clients? (((telephone calls)))
Which of the following is an example of a barrier in cross-cultural communication? (((The "OK" hand symbol means "all right" or fine in the United States, but is considered an insult in parts of Europe.)))
Among people age 18–24, the most popular social network is _______; among people aged 35–44, the most popular social network is ________. (((Snapchat; LinkedIn)))
Which of the following statements is not true of social media? (((Social media rarely reduces productivity in the workplace.)))
Which term refers to the use of the Internet at work for personal use? ((( cyberloafing)))
Which of the following is not an element of an effective social media policy? (((Encourage employees to post honest, bold, and even unpopular decisions.)))
Which of the following is not one of the top five qualities that employers are looking for most in college graduates? (((creativity)))
Which of the following is an example of controlling communication? (((You must complete the audit by Friday of this week.)))
Peer relationship is about: Sponsorship – opening doors, Coaching – teaching ‘the ropes’ and Protection – providing support. ((False ))
Knowledge is the Natural abilities you are good at.((FALSE))
Mode is the middle number.((FALSE))
The first step in creating your personal marketing plan is to conduct a career audit.((TRUE))
Line graphs are a way of presenting proportional data in the form of a circle((FALSE))
When giving feedback to others you should start with the negative points first then with the postive after.((FALSE))
Quantitative data refers to intangible qualities or features.((FALSE))
A multiple-cause diagram will not enable you to show the causes and the ways in which they are connected.((FALSE))
Bar charts are designed to show frequency distribution((TRUE))
Median is the average of numbers.((FALSE))
Interviews have a good response rate((TRUE))
In Brainstorming there are two important principles that apply, one of them is to limit generation of ideas that looks good only and not to waste time on others.((FALSE))
If your negotiation seems to be failing because someone else is being difficult you can threaten the other person.((FALSE))
"Per cent" simply means "out of 100"((TRUE))
Sampling is helpful to us because we can investigate a sample of a larger population in order to learn just about the sample.((False))
Force-field diagrams shows the inputs to a system or to an operation and the outputs from it.((FALSE))
Skilled negotiators demonstrate the following positive behavior: Argument dilution and Behavior chains.((FALSE))
In giving feedback place the feedback in the context of relationship((TRUE))
Mentoring is a time-honored practice in which a relatively inexperienced person is helped by a more experienced one.((TRUE))
When giving feedback you should listen actively and ask for precision.((FALSE))
Limits to assertiveness: take no account of structural and political imbalances in organization or society at large.((TRUE))
Observing can be potentially intrusive and ethically problematic.((TRUE))
Primary data is data that is already exist.((FALSE))
Surveys & questionnaires problem is that it often lack the richness and subtlety it is possible to uncover other methods.((TRUE))
Brainstorming is an approach to performance evaluation. ((FALSE))
Peer relationships can support our development needs, can bring some unique advantages and offer mutuality.((TRUE))
Qualitative data describes measurable or countable features of whatever has been investigated.((FALSE))
Observing is a technique in which a sample of the population is asked questions about the issues the investigator is interested in.((FALSE))
A system map shows the boundary of the system and the different subsystems inside the boundary.((TRUE))
Assertive behavior means standing up for yourself, but in ways which respect the rights of others.((TRUE))
Skilled negotiators demonstrate the following positive behavior:Test understanding and summarize, Flag or signal, and Ask lots of questions.((TRUE))
Talents are abilities or expertise that you have acquired by training or experience((FALSE))
One way of considering career development is to think of it in terms of a personal marketing plan.((TRUE))
Input-output diagrams are an explicit prompt for exploring the restraining forces.((FALSE))
Limits to assertiveness: Encourage people to say more and frank((FALSE))
Reasons for using active listening is for stating your own views, disagreeing with other’s views, making a request, and refusing a request.((FALSE))
If your negotiation seems to be failing because someone else is being difficult you have a choice of not reacting.((TRUE))
Skilled negotiators avoid the following negative behavior: Defense/attack spirals and Irritators((TRUE))
Active Listening is a behavior that means standing up for yourself, but in ways which respect the rights of others.((FALSE))
Transactional marketing is oriented towards single purchase while Relationship Marketing is oriented towards repeat sales.((TRUE))
The 5 functions of management defined by Fayol are:((Planning, Organizing, Commanding, Coordinating and Controlling))
A number of tools used by organization to communicate with their audiences. Together these tools are called:((The marketing communication mix))
Which of the following defines best the break-even:((The point at which total costs and total revenues are equal))
The psychological contract refers to the set of expectations of the employment relationship formally stated in a written contract.((FALSE))
It is essential for every business to consider some external forces before they can take decisions. The tool commonly used to evaluate different external factors which impact an organization is:((STEEP model))
The product life cycle stages are 4 clearly defined phases, each with its own characteristics. Stages include: introduction, growth, maturity and decline.((TRUE))
Branding means creating an identity for a product((TRUE))
Which type of leader influences the others through the exchange of valued rewards?((Transactional))
Quantitative aspects: Some aspects of performance can be measured as quantitative targets like:((Timescales, Deadlines, Amounts produced, Costs, Resource usage))
The first step in customer decision making process is need recognition((TRUE))
Segmentation is a process of breaking down a total market into smaller, more distinct segments that have a similar characteristics((TRUE))
Employee empowerment is the process of motivating the employee by granting him a financial compensation that meets his expectations((FALSE))
What is a transformational leader?((A leader who inspires and motivates the workers to new levels by offering them a vision of a better future))
Which among the following are Fayol management processes?((Forecasting and planning, organizing, leading, coordinating, controlling))
Asset are:((Is something which the organization owns and which has a market value))
SMART model means:((Specific, Measurable, Agreed, Realistic, Timed))
Reasons for involvement & empowerment: ((All of the above))
Net worth is the value of the assets plus the value of the liabilities as recorded in the books of accounts.((FALSE))
Asset is something which the organization owns and which has a market value.((TRUE))
Psychological contract represents the mutual beliefs, perceptions, and informal obligations between an employer and an employee((TRUE))
AIUAPR model includes: Awareness, Interest, Understanding, Attitude, Purchase, Repeat purchase.((TRUE))
Mentoring is a powerful personal development and empowerment tool, it is a helpful relationship based upon mutual trust and respect.((TRUE))
Branding is very important for a business, because:((It goes beyond a logo or graphic element, it increases the value of the company and makes acquiring new customers easier))
Physiological needs are those things that are required to sustain biological life: food, water, air, shelter, clothing, and sex.((TRUE))
The aim of the selection interview is to ascertain whether the candidate is interested in the job and competent to do it((TRUE))
Performance appraisal is a method by which the job performance of competitor is evaluated.((FALSE))
Liability is a debt that the organization owes to another person or organization.((TRUE))
Employee empowerment is a strategy and philosophy that enables top management to make decisions about their jobs.((FALSE))
The following are the different types of costs:((Fixed, variable, direct, indirect))
Human resources are concerned with all management decisions and actions that affect the relationship between an organization and its members.((TRUE))
The formula of the break-even point is the following:((Fixed costs / contribution per unit sold))
Internal staffing and promotion decisions are part of HR duties.((TRUE))
When a product starts to shrink due to market saturation and because the customers are switching to a different type of product, the product has then reached the:((Decline stage of its life-cycle))
Pricing must consider : (( All of the above ))
Diversification is one of the four alternative growth strategies in the Ansoff Matrix. A diversification strategy achieves growth by developing new products for completely new markets. (( TRUE ))
Performance appraisal are a part of career development and consist of regular reviews of employee performance with in organization (( TRUE ))
internal staffing and promotion decisions are part of HR duties (( TRUE ))
The product life cycle stages are 4 clearly defined phases, each with its own characteristics. Stages include: introduction, growth, maturity and decline. (( TRUE ))
The psychological contract represents the mutual beliefs, perceptions, and informal obligations between an employer and an employee. (( TRUE ))
strategic planning helps a business set its (( missions ))
A SWOT analysis is used to (( identify new market ))
raw materials used to produce products are (( Variable expenses. ))
which of the following statements is true of venture capital firms (( They prefer to back small firms with opportunities for fast growth. ))
_____ are those capital requirements that are of a relatively permanent nature and are necessary for the functioning of a business. (( fixed assets ))
Peer relationship is about: Sponsorship – opening doors, Coaching – teaching ‘the ropes’ and Protection – providing support. ((False))
During a manager's meeting, Paula, a district manager, discussed how to handle sensitive employee issues and indicated correctly that management is really just an art. (((FALSE)))
Ali, a CEO of a large IT organization, understands that many challenges are coming in his industry; therefore, he tells his upper-level management that it's helpful to look at theoretical perspectives of management to help make predictions and develop principles to guide future company strategies and actions. (((TRUE)))
When faced with a business problem, Sally, a young grocery store owner, discusses business problems with Don, the assistant manager and an employee for over 30 years. Sally believes Don "has seen it all" and knows there are few really new ideas; plus the business has done many simple things, such as offering friendly service, to create and maintain a competitive edge. Sally's management style is best described as (((evidence-based.)))
Susan, a movie theater manager, recently implemented a policy stating that workers who call in sick on Friday or Saturday nights will have their free movie privileges revoked for one month. In observing her workers, she has noticed that some of them feel that the new policy is not fair, and they have not been working as hard as they did in the past. What are Susan's employees doing? (((soldiering)))
Hugo Munsterberg believed that psychologists can contribute to industry by _____. For example, Munsterberg might suggest that a company implement a monthly bonus plan that rewards workers for achieving the goals set by their managers. (((devising management strategies to influence employees to follow management's interests)))
Mary Parker Follett anticipated some of today's workplace trends, such as (((self-managed teams and worker empowerment.)))
Maslow's hierarchy of human needs included ((( physiological, safety, love, esteem, and self-actualization.)))
Paul English of Kayak.com uses new employees as an excuse for altering an existing open-office seating arrangement. English's action is an application of ____ to management. (((the behavioral science approach)))
Gabriel, the CEO of a large global production company, is excited about the introduction of statistics and computer simulations in the design stage of the company's products. Gabriel is using (((quantitative techniques.)))
Operations management focuses on managing the production and delivery of an organization's products or services (((more effectively.)))
A computer manufacturer is seeking to cut costs by designing an inventory system that reduces the number of finished products in stock due to overproduction and to set in place a production schedule that better matches customers' orders. These plans are an example of ____ in action. (((operations management)))
The Jones family had a fire that destroyed their home. Various departments of a restoration company, from the initial cleanup crew to the assistance with the Jones family moving back in, worked together to achieve this goal. All of the company's interrelated parts working together to accomplish a goal (in this case, the restoration of the Jones home) is an example of a(n) (((system.)))
A local farmer grows and sells tomatoes and beans to the local grocers. Look at the farmer's business as a system. In which category do the tomatoes, beans, squash, profits, and losses fall? (((output)))
Donna, a chef in a fine restaurant, utilizes top-of-the-line equipment as she prepares meals for customers. Donna and her equipment are a(n) ______ in the restaurant's system. (((input)))
When a line cook in a restaurant cooks uses raw meat to cook a hamburger that becomes part of the restaurant's Super Burger Special (available only on Saturday nights during football season), the cook is taking part in a (((transformation process.)))
For years ABC Copier, a copier machine supplier, enjoyed strong sales and a huge share of the copier market, far ahead of its nearest competitor. However, over the years, ABC seldom asked customers for feedback. So when some of ABC's competition responded to customers' needs and started offering copiers with new features (such as wireless printing from laptop and tablet computers), ABC lost much of its market share. ABC's system is best described as (((closed)))
Dominique, an executive chef in a large hotel, recently attended a training conference sponsored by several top professionals in his field, where he learned numerous ways his restaurant and hotel can better serve customers. Dominique is excited that management asked him to present this information to the hotel staff so that the restaurant and hotel can make needed improvements. Dominique's hotel is a(n) _____ organization. (((learning)))
The task environment of Top-Notch Sewing consists of just two groups, customers and stockholders, who give the employees and management of Top-Notch the daily tasks that the company's employees will handle. (((FALSE)))
Pleasant Resort, a successful 100-year-old hotel and resort, entered into a co-marketing arrangement with A-Plus Hotels, a five-star hotel chain. Both companies benefit from the new relationship. Pleasant is now able to market its facility and services through A-Plus's marketing team, and A-Plus Hotels now offers access to an upscale resort that it can recommend to its customers who are looking for a resort experience. A-Plus Hotels and Pleasant Resort can be considered strategic allies. (((TRUE)))
Paul, the vice president of marketing in an international sales organization, is considering opening an office in a new country. He is looking at the current economic, political, legal, and technological global forces in that country that can affect the success of his company's expansion into the new area. These forces that Paul is studying are international forces. (((TRUE)))
Just before introduction of a new revolutionary laptop, the Top205 by Top Computers, Greg, the CEO of Top, was informed that a problem with a part inside the Top205 may cause reliability issues. Greg knows that correcting the problem will delay the introduction of the product, and be expensive due to possible order cancelations and added advertising costs. But he also knows that allowing the new flawed computer into the market will affect Top Computers' reputation. Greg's decision whether to delay the introduction of the Top205 or proceed as planned is (((TRUE)))
Alonzo, the president of his high school's senior class and honor society, has been very busy and did not study for an important history test. Although Alonzo is very confident, he realizes that he probably will not do well on the test. He has talked himself into cheating because he believes that cheating will help him maintain his strong GPA, which he will need to be accepted by a good college. He tells his friend, "I don't usually cheat, but I really have to do it." Which of the following might explain Alonzo's behavior? (((motivated blindness)))
Don, the owner of a bait and tackle shop, attended an entrepreneurship workshop that discussed the triple bottom line, which measures an organization's ____ performance. (((social, environmental, and financial)))
What mechanism is used to provide a systematic assessment of an organization's performance in implementing socially responsible programs, often based on predefined goals? (((social audit)))
An organization's internal stakeholders consist of (((the board of directors, employees, and owners.)))
The marketing director for Zap Games, a video game company, has informed his employees that he feels the company needs to improve its relationship with the distributors of the company's products because the distributors are part of the ______ environment for Zap Games. He believes that a stronger relationship with distributors will lead to more market share, and higher profits, for both Zap Games and its distributors (((task)))
Don was recently promoted, and his salary increase includes retirement investment options, so he has decided to take part in an ESOP. This means that Don (((will be buying or receiving shares of his company's stock.)))
Amy, a successful banker and educator, has decided to retire, but she is very interested in staying involved with a company "that cares." So when asked to serve on its board of directors, she gladly accepted the invitation. As a member of the board, Amy will be involved with the organization's (((overall strategic goals and approval of major decisions.)))
ABC Brothers' cleaning products and equipment are sold to consumers and commercial cleaning companies in Best Supply's janitorial supply and equipment store. ABC relies heavily on Best Supply to help sell its goods and services to customers. Best Supply is a(n) ____ for ABC Brothers' products. (((distributor)))
Joe, a lobbyist who represents the oil industry, is meeting with several members of Congress to try to exert political influence by contributing funds to the lawmakers' election campaigns. Joe also is involved in a letter-writing campaign to promote his cause. Joe is working on behalf of a (((special-interest group.)))
The mass media are part of an organization's (((task environment.)))
Tom and his managers are discussing the unemployment, inflation, and interest-rate trends that might affect their chain of coffee shops over the next 12 months and the projected growth in the areas where the stores are located. The managers are studying the ______ forces in their organization's general environment. (((economic)))
_____ forces lead to new developments in methods for transforming resources into goods or services. For example, a new technique called hydraulic fracturing (or fracking) has been developed to harvest natural gas trapped below the earth's surface. (((Technological)))
Changes in the economic, political, legal, and technological global system that may affect an organization are ______ forces. For example, stresses within the European Union (including the United Kingdom's recent decision to exit the EU) may have global ramifications. (((international)))
Joe, the owner of ABC Electronics, just discovered that his trusted friend Paul, his accountant for over 30 years, has been mishandling the company books and stealing from the company bank account. Joe must decide whether to ignore his friend's actions and avoid bad publicity for ABC Electronics, or to have Paul arrested for embezzlement. Joe is facing (((an ethical dilemma.)))
. ______ are the relatively permanent and deeply held underlying beliefs and attitudes that help determine a person's behavior. These beliefs may include the idea that employees should be promoted solely on the basis of merit, that no bribes will be paid to foreign officials to obtain business licenses, and that companies should offer extended maternity leave upon the employee's request. (((Values)))
Consider the following situation: Local livestock farmers could allow the runoff of manure nutrients into a stream that feeds a local lake because polluting the stream helps farmers in the short term. However, farmers will act ethically in the short run to avoid harming others in the long run because doing so is also in the farmer's best long-term interests. This type of thinking reflects the _____ approach to deciding ethical dilemmas. (((individual)))
With recent reports of identity theft, Mr. Jones, the CEO of a construction company, is concerned about his employees' privacy, and because of recent accidents on the job, he is also worried about the safety of his workers. Mr. Jones's concerns with privacy and health and safety are key elements in the _____ approach to deciding ethical dilemmas. (((moral rights)))
Greg, the president of an IT company, is friends with Jack, the CEO of HyperTech, a company that develops and manufactures computer hardware. Jack tells Greg that HyperTech is about to announce the creation of a revolutionary new computer processor that will quadruple processing capacity and cause HyperTech's sales and profits to skyrocket. Greg then purchases a large number of shares of HyperTech company stock before the news of the new computer processor is made public. In which illegal activity has Greg engaged? (((insider trading)))
Jane, a successful store manager, leads her employees by encouraging them and showing that she genuinely believes in them. She also encourages group and team efforts. Jane manages at Kohlberg's _____ level of personal moral development. (((conventional)))
Don has managed several restaurants for over 50 years and is close to retirement. With his maturity and business experience, his decisions are sound, based on solid, proven values. Don tends to be an independent thinker who focuses on empowering his employees. Don manages at Kohlberg's ______ level of personal moral development. (((postconventional)))
The employees of Mom's Pizza, a local pizza restaurant, feel that the owners and managers really try to follow the ethical policies outlined in their company's code of ethics. Pete, a cook, said that "Just today, Joe, our manager, called our supplier to let them know we received an extra case of tomatoes that we did not pay for. And they do this type of thing almost every day." Mom's Pizza maintains a(n) (((ethical climate.)))
A(n) ____ is a formal written set of ethical standards guiding an organization's actions. (((code of ethics)))
Phillip said to a close friend, "I am fed up with my company's continual disregard for the environment. They secretly dispose of hazardous chemicals into a nearby stream constantly. I don't know how they get away with it! I am going to call a reporter at the local newspaper and reveal what my company is doing." Phillip's decision to tell somebody about his company's actions is an example of (((whistle-blowing.)))
As the CEO of a company that produces products for schools, Hannah believes that her company needs not only to produce a profit but also to do things that benefit society. _______ guides Hannah's beliefs. (((Social responsibility)))
The company that Don owns, the Ardmore General Store, is a family-owned company that has been in business for more than 100 years. Don wants to give back to the people of the community to acknowledge their role in the store's success. He decides to donate a significant portion of the store's profits to a charity every year. Don's decision is an example of ___ in action. (((corporate social responsibility)))
Mr. Smythe, a wealthy businessman who made his fortune in the oil industry, understands the importance of a college education, so he has donated over $10 million over the last several years to colleges and universities. Mr. Smythe's actions are an example of (((philanthropy.)))
On an organization's board of directors, (((inside directors may be members of the firm and outside directors are supposed to be elected from outside the firm.)))
With regard to boards of directors, and in particular their oversight of the CEO, the board of directors' _______ is the biggest complaint (((lack of independence from the CEO)))
Sally has seen such great interest in her scented candles that she has decided to start her own small business selling them. Sally's company can use the Internet and the World Wide Web to operate globally, helping her get a global business started more easily because she can put products on a website and sell worldwide. So, in a sense, the Internet wipes out the former advantages of distribution and scope that large companies used to have. (((TRUE)))
In 1999, the top exporting nation was the United States, but in 2015 the world's top exporting nation was China. (((FALSE)))
A greenfield venture is a strategic alliance with individuals and organizations in developing nations; the venture's goals are to assist, educate, and share the risks and rewards of starting a new nonprofit organization in the developing country. (((FALSE)))
After a morning of global cultural training, Marie, the vice president of marketing, is having lunch with Ann, who will be managing the company's new office in South Korea. Marie tells Ann, "There are cultural differences between the United States and South Korea that will affect the success of our office. For example, I suggest giving a couple of cans of Spam to your larger customers, because Spam is considered elegant in South Korea." Marie's advice to Ann is good because it takes into account the many kinds of cultural differences between the United States and other countries. (((TRUE)))
Last week Paul, CEO of Quality Furniture in South Carolina, traveled to Europe to visit customers. While overseas, Paul checked his e-mail daily and showed his company's website to customers, explaining how the website will help them place orders and receive merchandise more quickly. After visiting the last customer Friday morning, Paul was able to return to the corporate office in South Carolina to meet with his board of directors that night. _______ is the "shrinking" of time and space with air travel and electronic media. (((the global village)))
John, a construction manager, has been having problems finding quality employees in some of the company's foreign offices. While speaking with his attorney, Alisha, he mentioned that "in our locations overseas, many of our good employees have been quitting, and as you know, we have numerous lawsuits over personnel policies." Alisha explained that historically, problems with employee turnover and lawsuits are a result of ____ policies. (((ethnocentric)))
Roberto, the vice president of marketing, is talking to Francis, a sales manager in a foreign office, about several local situations that Francis's reps are facing in their territory. Roberto tells Francis that he believes there are differences and similarities between home and foreign personnel and practices, and that Francis should use whatever techniques are most effective. Roberto is a(n) ____ manager. (((geocentric)))
Alice's Pasta Kitchen makes quality pasta and sells its products to customers in other countries. Alice's Pasta Kitchen is ____ its products. (((exporting)))
Using ________, Don's Carpet installed carpet at no cost for City Office Supply, in exchange for $1,000 of office supplies. (((countertrading)))
Which of the following is an example of licensing? (((When a company allows a foreign company to pay it a fee to make or distribute the first company's product or service.)))
Luxere Hotels International, an American company, sells the rights to other hospitality companies globally to open hotels with the Luxere name for a fee and a share of the profit, in return for using Luxere's brand name and a package of materials and services. Luxere is engaged in (((franchising.)))
Sometimes a country's laws forbid foreigners from owning a business in the nation. In the presence of such laws, a(n) _____ is one way for an American company to have a presence in that foreign country. (((joint venture)))
When planning a trip to France, Jim and his wife, both Americans, were concerned about how much they could afford to spend in Europe because sometimes the U.S. dollar will buy more goods and sometimes it will buy less, based on changing economic conditions. The ____ rate is the rate at which the currency of one area or country can be exchanged for the currency of another's (((exchange)))
Tom and his fiancée Susan decided to have their wedding on a beach in Acapulco, Mexico. Local businesses provided most of the services for the wedding, such as the limousine and the catering services. Throughout the weekend of the wedding, the couple found that workers and limos arrived late; so did the priest who was performing the wedding. In response to the couple's frustration, the Mexican workers remarked, "Ustedes tienen que ser flexibles," which in English means "You have to be flexible with time." This attitude regarding time is (((polychronic.)))
.______ is a government's seizure of a domestic or foreign company's assets. (((Expropriation)))
Greg, a geologist from Kansas who works for ExxonMobil, has decided to transfer to one of ExxonMobil's offices in Saudi Arabia. Greg is a(n) ____ while living in Saudi Arabia, (((expatriate)))
Which of the following statements is FALSE? (((The United States was the world's largest economy in 2014.)))
Spark, Inc., which manufactures the fireworks that municipal governments buy to put on their annual fireworks shows on the Fourth of July, moved most of its manufacturing operations from the United States to China a decade ago. Now, Spark is moving production back to the United States because the company's managers are uncomfortable with many Chinese business practices. Spark, Inc. engages in __________ when it moves its operations back to the United States. (((reshoring)))
Roberto works for a company that highly values performance-oriented traits, such as achieving a certain level of sales and increasing market share by a certain percentage. In analyzing this aspect of Roberto's company, Geert Hofstede would examine the company's _______ dimension. (((masculinity/femininity)))
The upper-level managers of Deluxe Coffee are meeting to discuss the company's long-range goals and vision. Like any organization, Deluxe Coffee should adopt planning and strategic management for just two reasons: (1) to provide direction and momentum and (2) to encourage retention of high-value employees. (((FALSE)))
Paula is the store manager for a national grocery retailer, and she received the yearly tactical plan from Greg, her district manager. Paula is meeting with her department managers to plan how to implement the specific tasks outlined in Greg's plan for the next year. Paula is engaged in operational planning. (((TRUE)))
Jerome's Italian Pizza is in a very competitive industry, and its upper management believes that it can achieve higher profits by focusing on a strategy of lowering costs and prices, providing quicker delivery, and providing good customer service. "Increase profits by 15% a year for each of the next five years" is an example of a strategic goal. (((TRUE)))
When utilizing MBO, the manager and employee jointly set objectives for the employee, the manager develops action plans, the manager and employee periodically review the employee's performance, and the manager makes performance appraisals and rewards the employee according to the results. (((TRUE)))
With increasing competition in the restaurant business, Sal, owner of Mom's Pizza, is dealing with growing uncertainty. He is meeting with his managers to develop a future course of action to meet company goals. This process is known as (((planning.)))
The profits of ABC Grocery have dropped significantly over the last several months. After investigating, the owner realizes that many of the store's departments are moving in the wrong direction. The owner realizes that these departments need stronger and closer management, but she also realizes that she has not provided her managers with enough direction regarding her expectations for the store's goals and the employees' performance. This example shows how an organization can progress in the wrong direction as a result of (((poor planning.)))
An organization's _______ expresses the need the firm will fill, the operations of the business, its components and functions, and the expected revenues and expenses. (((business model)))
Clothing maker Patagonia states that its purpose as an organization is to "build the best product, cause no unnecessary harm, [and] use business to inspire and implement solutions to the environmental crisis." This is the company's (((mission statement.)))
Elite Coffee and Bakery, a café that offers a relaxing environment to enjoy quality coffees, teas, soups, and other menu items, has a poster in the front of its restaurants that reads, "We will always provide a quality product to help our customers better meet the challenges of their day and always take the time to offer caring service one person at a time." This statement is Elite's (((mission statement.)))
During a management meeting, Dave, the CEO of Top Office Equipment, reminded his management team of where the company wants to go as an organization: "Top Office Equipment is the expert whom customers should always call regarding all their office equipment needs because Top Office strives to understand customer needs and provide the right answers for them." This statement is Top Office Equipment's (((vision statement.)))
One Fortune article states that "vision should describe what's happening to the world you compete in and what you want to do about it" and (((it should guide decisions.")))
A company's strategic planning may have to be done _____ due to frequently changing competition and technology. (((closer to every one or two years than every five years)))
The upper management of a large national retail grocery store has passed along the new goals of improving customer service to the district managers. At this point, the district managers need to determine how the stores in their district will achieve top management's goal with the given resources during the next 6 to 24 months. The district managers are developing ______ planning. (((tactical)))
The top management of Tasty Foods, a food distribution company, has set strategic goals of increasing organizational market share and also decreasing corporate costs over the next three years. Greg, a division manager for Tasty Foods, has looked at his resources, and he has decided how his division can contribute to the two strategic goals set by upper management: (1) by partnering with another company and (2) by hiring a procurement manager to negotiate lower prices from vendors. Greg's next step is to roll out his ____ to his staff (((tactical goals)))
Warby Parker, a manufacturer of fashionable prescription eyewear, notes on its website, "Warby Parker was founded with a rebellious spirit and a loft objective: to offer designer eyewear at a revolutionary price, while leading the way for socially-conscious business." This excerpt from the company's website states Warby Parker's (((vision statement.)))
The management of Quality Carpet Cleaning finalized the company's action plan for increasing its market share over the next few years. The management then set up a 12-month plan that defines how the organization will conduct its business based on the action plan, including clear sales targets for each week. The second plan is an example of a(n) _____ plan. (((operating)))
Paula, the store manager, asked Tom to organize the prize giveaways for the store's grand reopening open house. Tom is organizing a(n) (((project)))
Billy told his boss, Gwen, that he is going to start training for the upcoming marathon to be healthier and to have more energy for work. When Gwen asked him about his running history, Billy said, "I really have never run before and don't exercise much. I just know I need to do something." Gwen told Billy, "You should start slowly, maybe setting smaller, realistic exercise goals that are achievable." Gwen is helping Billy set a (((SMART goal.)))
With setting SMART goals, you should choose only a few goals on which to focus. These goals should be results-oriented and (((support the organization's vision.)))
____ is the process for motivating employee performance in which the manager and employee jointly set objectives for the employee, the manager develops action plans, the manager and employee periodically review the employee's performance, and the manager makes a performance appraisal and rewards the employee according to the results achieved. (((MBO)))
André, the plant manager, is interested in increasing the facility's productivity by utilizing MBO so that his managers and their employees are more focused on objectives. This month André asked his managers to concentrate on the two first steps of MBO, which are (((to jointly set objectives with their employees and to have managers develop action plans.)))
Steve, a hotel general manager, has had great success using MBO. Hannah, his district manager, has found that Steve's employees are highly motivated. Because Steve _________, Hannah is able to discern that he is using MBO. (((rewards his employees with compliments, raises, and other benefits as they meet their goals)))
You are reading a business document that outlines your company's long-term goals and direction. These goals include "Reduce energy consumption in our offices by 20%," "Grow out market share in China from 3% to 10%," "Enter a strategic alliance with a partner in Australia to begin selling our products in Sydney and Brisbane," and "Equip all sales representatives with an app that allows them to provide an immediate quote when meeting with customers." What type of document are you reading? (((a strategic plan)))
Based on the VRIO framework, which of the following start-up companies has the least potential to have a competitive advantage, to be profitable, and to grow? (((a cat-sitting service, where you take care of people's cats when they are on vacation; you decide to advertise your services on Craigslist and the local newspaper)))
The Whole Foods chain of supermarkets has stated, "With great courage, integrity, and love, we embrace our responsibility to co-create a world where each of us, our communities, and our planet can flourish. All the while, celebrating the sheer love and joy of food." This proclamation is Whole Foods' (((values statement)))
Which of the following is NOT a step in the process of management by objectives (MBO)? (((Managers determine each employee's strengths and weaknesses.)))
As a manager, you meet with each of your team members quarterly to review their progress toward their written and agreed-upon goals. One of your employees, Claire, had a goal of increasing sales in her territory by 10% in the first quarter of the year. However, when you review Claire’s sales numbers, you see that her sales have actually gone down. You know that Claire is a hard worker and has an excellent relationship with her customers. When you sit down to discuss Claire’s performance, she explains what is happening in her territory. A major automobile manufacturer, which once provided 25,000 jobs, has moved its operations to Mexico and laid off all 25,000 of its workers. As a result, the economy in Claire’s region has become quite depressed, and the residents are living off their savings. Taking the steps of the planning/control cycle into account, what is your best course of action with Claire? (((Recognize the economic reality of Claire's territory and change her goals to make them ambitious but realistic.)))
Donna's Restaurant is a popular café that specializes in home-cooked meals, friendly service, and a menu that contains vegan and vegetarian dishes, (menu items that no other restaurant in the area offers). Donna's Restaurant is engaging in strategic positioning by offering the unique menu items of vegan and vegetarian dishes. (((TRUE)))
Frank is interested in rewriting the vision statement for his antique shops, and he wants his employees and his business to grow. Therefore, the vision for Frank's stores should be positive and inspiring, and it should stretch the company and his employees to achieve objectives that they believe are not possible. (((TRUE)))
The vision statement is an organization's purpose or reason for being, and a company's mission statement is its long-term goal of what it wants to become. (((FALSE)))
Competitive intelligence means gaining information about one's competitors' activities so that you can anticipate their moves and react appropriately. For example, managers gain competitive intelligence by reading business publications, in order to understand competitors' business plans, goals, and strategies. (((TRUE)))
The managers of a small bakery, Butter & Batter, decided to do a SWOT analysis to study the strengths and weaknesses within the organization and the external environment. A key benefit of conducting a SWOT analysis is being able to better formulate strategies in pursuit of the firm's mission. (((TRUE)))
Organizational threats are an environmental factor that can hinder an organization's ability to achieve a competitive advantage. For example, managers may realize that the government is about to place restrictions on exports, which will limit the company's ability to sell its products overseas (which will in turn greatly reduce its profits). (((TRUE)))
Instructor Services is a technology company that offers many IT services in highly populated southern Ohio. The company's services and products include computer training, support, monitoring, repair, network design, virus removal, and software upgrades. It even sells refurbished computers. The source of Instructor Services' strategic position is (((broad needs and many customers.)))
With small businesses in very competitive industries, small differences in performance may affect that company's survival. In this case, it is worth the effort for the company's managers to implement (((strategic planning.)))
A local karate studio offers many services: karate lessons to young children (under age 10), judo lessons to preteens, kickboxing courses for teenagers and adults, and self-defense courses for older people. We can say that the studio has achieved _____ because the studio's activities interact and reinforce one another. (((fit)))
The second step in the strategic-management process is a(n) ________, where managers look at where the organization stands, and then determine what is working and what could be different to maximize efficiency and effectiveness in achieving the organization's mission. (((current reality assessment)))
An organization's vision statement needs to describe (((An organization's vision statement needs to describe)))
Over the course of 40 years, Sal grew his company to six package shipping stores. With his retirement approaching and the increased competition, he decided to reduce the number of locations to two. Sal's reduction of effort represents a (((defensive strategy.)))
Strategy formulation is the process of (((choosing among different strategies and altering them to best fit the organization's needs.)))
George, a chef and owner of L'Auberge, a popular restaurant, is always visiting his competitors to observe how they are doing things in their restaurants. He told one of his managers, "I eat dinner at a lot of restaurants because I want to know what is going on. I am always concerned that one of our competitors will surprise us with a new service or menu item, like ours but better." In which activity is George engaging? (((competitive intelligence)))
Susana keeps a close eye on her company's internal and external environment to discover possible opportunities for new products and to discern possible threats from the competition. In which activity is Susana engaged? (((environmental scanning)))
Many customers shop at Organic Foods because of the employees' extensive product knowledge. In a SWOT analysis, the employees' high levels of product knowledge are an example of the company's (((strengths.)))
Don knows that one of the reasons people do not return to his electronics store is because of the slow service. How would a SWOT analysis classify the slow service at Don's electronic store? (((as a weakness)))
Christopher's Restaurant and Catering serves delicious vegetarian and vegan dishes. So when the local community became interested in eating a healthier diet, Christopher's benefited. In a SWOT analysis, the changing community attitudes are an example of a(n) _____ for Christopher's Restaurant. (((opportunity)))
The owners of Smith's Yard Mart, a family-owned garden center in a rural community, are concerned over the news that a large retailer with a garden center is building a new store on the other side of town. According to a SWOT analysis, this new retailer is a(n) ____ to Smith's. (((threat)))
Sales this year at Donna's Pawn Shop have been high, and based on several factors, Donna projects next year's sales to also be good. However, even with her forecast of continued strong sales, Donna and her business partner need to develop a plan in case sales drop unexpectedly. ________ is the type of planning for alternative future conditions. (((Contingency planning)))
Michael Porter proposed that business-level strategies originate with the primary competitive forces in the firm's environment; these forces are (((the threats of new entrants and substitute products or services, the bargaining power of suppliers and buyers, and competitive rivalries.)))
Tom and his family have developed a successful business selling a liquid spray fertilizer to farmers. The fertilizer consists of rich, organic, composted material. Recently NuBreed Seed, a national seed company, has been marketing a powdered chemical fertilizer to its customers; NuBreed's fertilizer is less expensive than Tom's product. NuBreed's efforts are an example of the ___ in Porter's model for industry analysis. (((threats of substitute products and services)))
Porter's competitive strategies of cost-leadership and differentiation focus on ____ markets, while the cost-focus and focused-differentiation strategies focus on ____ markets. (((wide; narrow)))
A company's overall ability to execute is a function of effectively executing according to three processes, but Bossidy and Charan believe that the _____ process is the most important. (((people)))
Which of the following questions is NOT useful for evaluating a company's values statement? (((Does it set standards of excellence and reflect high ideals?)))
Tom is in the marketing department with HomeMade Soups. He was given the responsibility of determining if customers liked the new spicy chicken noodle soup. According to HomeMade Soups' sales data, over 4,000 customers purchased the new soup last month. If Tom surveys 25 of those customers, he has selected an appropriate sample size for decision making. (((FALSE)))
Luis, the sales manager of IT Sales, mentioned to his assistant, Marla, "I just got off the phone with one of our best customers and he told me about quite a few customer service issues, and I'm glad he took the time to call. You know, most people will not call a company when they have a problem; they just tell friends, family, and coworkers." This statement about who customers call when complaining about faulty customer service is accurate. (((TRUE)))
Last year, Eduardo's home of 50 years, which contained so many family, school, and career memories, was destroyed in a fire. It is normal for this event to have a negative impact on his life and decision-making abilities for at least three years (((FALSE)))
George, a longtime hotel manager for Holiday Inn, was meeting with Sally, a new front-desk manager, and telling her about his recent decision not to fire a front desk employee about whom several guests had complained. George discussed the four stages of the decision process. He said, "I first identified the problem, and then I gathered alternative solutions. I evaluated each choice and selected a solution. I then implemented my decision to not fire the employee but instead to provide him with additional training. Finally, I followed up with an evaluation of the solution I'd chosen." George is using the ______ process. (((rational decision-making model)))
The model of decision making that explains how managers should make decisions, assuming managers will make logical decisions that will be the optimum in furthering the organization's best interests, is known as the ____. For example, a manager who uses this model may be personally opposed to outsourcing jobs overseas, but she nonetheless decides to outsource customer-service operations to India because doing so is in the company's best interests. (((rational decision-making model)))
Despite your best intentions, it's hard to bring the best evidence to bear on your decisions. Why? (((There's too much evidence; and there's not enough good evidence and the side effects outweigh the cure.)))
ABC Medical Technology, a fast-growing global research firm, has accumulated so much client information that conventional database management systems cannot handle its customers' needs. Therefore, ABC purchased very sophisticated analysis software and supercomputing-level hardware. For the company's work with hospitals and university medical schools, ABC has been noted in The Wall Street Journal as a "real pioneer in innovation and productivity." ABC Medical Technology is tapping the power of (((Big Data.)))
A person's ________ reflects the combination of how he or she perceives and responds to information. (((decision-making style)))
Marie is a small-business owner who loves to take risks. She also enjoys going out and meeting customers and potential clients—the social aspect of her job. Marie relies on intuition and discussions with others to acquire information. As an interior decorator, Marie utilizes her imaginative strengths, taking a broad perspective to problem solving, and she likes to consider many options and future possibilities. Marie's decision-making style is best described as (((conceptual.)))
Sam was recently promoted to the production manager of a busy shoe factory. He was chosen because prior to his promotion he had proven himself to be very efficient, and as a department manager he had a logical and practical approach to solving problems. Sam has a reputation for being decisive and getting things done. Most of the factory's employees like Sam, but they feel his style is autocratic, because he likes to do things by the book. Sam has a(n) ______ style. (((directive)))
Scott has a bachelor's degree in ethics and human resources, and he has extensive experience working with employees and managers regarding ethics, especially in the area of ethical dilemmas. In his current position he has assisted extensively in determining if the company's decisions are both ethical and lawful. Which position in the company does Scott's background make him ideally suited for? (((ethics officer)))
Is the proposed action legal? If yes, does the proposed action maximize shareholder value? If yes, is the proposed action ethical? If no, would it be ethical to take the proposed action? These four questions, which managers of all organizations should ask when confronted with a decision on an action, form the basis of (((Bagley's ethical decision tree)))
Patty, the CEO of an oil drilling company, and her top management team recently discovered that their facilities are damaging an Asian beach and the local wildlife. They understand that they need to decide if temporarily closing or not closing the facility is unethical. To help them make their decision, they consider the following four questions: "Is not temporarily closing the facility legal? If yes, does this proposed action maximize shareholder value? If yes, is not temporarily closing the facility ethical? If no, would it be ethical to take the proposed action?" Patty and her top managers are using ____ to help them make their decision. (((Bagley's ethical decision tree)))
Dwight, the general manager of a hotel, knows that one of his housekeeping employees has a serious substance-abuse issue. Dwight knows that because of the seriousness of the issue he needs to talk with the employee. However, his assistant manager asks Dwight if he would like her to deal with it. Dwight knows this is not the best option, but he agrees, knowing it is the easy way out. Dwight's decision is an example of (((relaxed change.)))
With defensive avoidance, a manager "acts" by _______ when he or she can't find a good solution to a problem (((procrastinating, passing the buck, or denying the risk of any negative consequences)))
Gregory was talking with Kareem, his assistant manager, saying, "When I make a decision on which employees will do a project, I use three simple factors, which I call my decision-making 'rules of thumb.' I consider their attitude and knowledge, and how hard they work." Gregory's rules of thumb, _________, is what he uses when making a decision. (((heuristics)))
Phil, the store manager, recently had to fire Sue, an employee who graduated from North College. He felt that Sue was not well trained at her school. So when Steven, also a graduate from North College, applied for Sue's old position, he did not want to consider hiring him, feeling Steven also probably did not receive the proper training. Phil was affected by _____ bias. (((representativeness)))
____ occurs when people's subjective confidence in their decision making is greater than their objective accuracy. For example, Sara thinks she is a superb writer and editor, but her reports are always filled with typos, grammatical errors, and sentence fragments. (((Overconfidence bias)))
Peter, the owner of a local real estate agency, is even more committed to making the used copier, which he recently purchased, work even though it keeps jamming, and his employees have been telling him about other problems they have been having with the copier. The ______ affects Peter's decision making. (((escalation of commitment bias)))
George, Donna, and Maria are on the five-member Benefits Task Force that is researching options for the new benefits package. Over lunch, George suggests that going with the Members' Choice HMO is the best course of action, and Donna and Maria agree with George for the sake of unanimity, without ever researching and accurately assessing the decision. George, Donna, and Maria are engaged in (((groupthink.)))
The Safety Committee at Office Supply Mart was formed to help make the store a safer place, and some of the committee's suggestions are going to be costly. During the committee meeting, the vice president of accounting announced that company profits were much lower than expected, and she suggested that profits are more important than safety. The profit goal is now outweighing the committee's safety goal, which is an example of (((goal displacement.)))
How do you know that a group in which you're taking part is suffering from groupthink? (((Peer pressure makes you agree with the ideas of the other people in your group.)))
A company's organizational structure is its formal and informal marketing systems of goods, services, ideas, and customer relationships. (((FALSE)))
Changing organizational culture is essentially a teaching process, a process in which members instruct each other about the organization's preferred values, beliefs, expectations, and behaviors. (((TRUE)))
When Angela, a new nurse was hired at Mercy Hospital, the HR manager gave her a handout containing a chart that showed a family-tree-like pattern of boxes and lines. This document, which shows the formal lines of authority and the organization's official positions and work specializations, at the hospital is called an organization chart. (((TRUE)))
Tom's Hardware has a simple organizational structure. There are no levels of middle management, because all employees report to the owner, Tom, or to his assistant managers. Tom's Hardware has a thin organizational structure. (((FALSE)))
Two advantages of decentralization are that (1) managers are encouraged to solve their own problems, and (2) decisions are made more quickly, increasing the organization's flexibility and efficiency (((TRUE)))
While figuring out how to save money for a bride who is having financial problems, Paula, the catering manager of Oh Happy Day, asked her assistant, "What do you think is the best menu to offer under these particular circumstances? How can we offer the bride and groom's guests a great meal within their budget?" Paula and her assistant are using the contingency approach. (((TRUE)))
Adam, the owner of Adam's Roofing, was talking to a visitor in his office, telling her that "We are a fun-loving group that believes in teamwork and a family atmosphere at work, which significantly affects our work outcomes. Plus, we do things together outside of work. This is the 'social glue' that binds the members of our company together." Adam was referring to his company's (((organizational culture.)))
Jean-Paul, the store manager at an H&M outlet, was speaking with Patty, his new assistant manager, about the store's system of job relationships; he also explained whom she will report to and who will report to her. Jean-Paul said, "This structure is one of the things that motivates our employees to work together to achieve our company's goals." Jean-Paul was telling Patty about the store's (((organizational structure.)))
. Online retailer Amazon relies heavily on a hierarchy culture to manage its vast and complex shipping processes. ___________ are part of Amazon's hierarchy culture. (((An internal focus and values stability and control)))
Employees and customers are treated like a family at Bayou Seafood Café. Paul and Betsey, the owners, work to encourage unity to increase their employees' job satisfaction and commitment. At their café, Paul and Betsey have built a(n) (((clan culture.)))
Elly and Sylvia, owners of Gaia Organic Foods, do not have a formal policy about dress code and work procedures because they see their company as a family place that is enjoyable to work for. Elly and Sylvia believe that if you treat employees like family, the company will do well and grow. _____ are the core beliefs that represent the core values of Gaia Organic Foods' culture. (((Basic assumptions)))
IKEA employees are inspired to work hard by an anecdote from their Swedish founder, Invar Kamprad, who told how he was berated by his father for failing repeatedly to get out of bed to milk the cows on his family's farm. Then one day he got an alarm clock. "‘Now by jiminy, I'm going to start a new life,' he determined, setting the alarm for twenty to six and removing the ‘off button.'" Invar Kamprad is an example of a(n) (((hero.)))
Rites and rituals are ____. For example, employees of New Belgium Brewery in Fort Collins, Colorado, get a free brewery-hopping trip to Belgium after being employed by the company for five years. (((the activities and ceremonies that celebrate important occasions and accomplishments in the organization's life)))
Danilo has been working for Metropolitan Taxi for three months. He expected that his job would be to drive around the city, pick up fares, and drop them off at their desired locations, but in his first few days on the job he realized that the job entails much more. He must keep his taxi cab clean and neat, and some days he will be stationed at the shop and go to people's houses to pick them up and bring them to the local airport. Within a few months, he has learned that taxi drivers often express frustration with the people they call "civilians"—that is, non–taxi drivers, whose driving skills they complain about incessantly. In his first few months on the job, ________ describes what Danilo is undergoing. (((organizational socialization)))
Della, the new CEO of Sky Advertising, has been with the firm for over 25 years. She was picked by the board to turn the 85-year-old agency around, because it had lost its edge in the Internet age. To infuse new life and energy into the agency, Della wants to bring back some old ideas that previously worked at Sky. She plans on having managers and veteran employees instruct each other about the organization's values, beliefs, and expectations; telling stories about some of the company legendary ad campaigns; coming up with a slogan that summarizes Sky's abilities in a simple and memorable phrase; and having quarterly ceremonies where creativity and innovation are rewarded. The things that Della wants to do are all examples of (((embedding culture.)))
According to Chester I. Barnard's classic definition, an organization is a (((system of consciously coordinated activities or forces of two or more people)))
_______ are voluntary collectives with the purpose of advancing their members' interests. One example of such a collective is the Teamsters Union. (((Mutual-benefit organizations)))
Alana, the children's department manager at Shoe Mart, has eight employees in her department, also known as her _________. All of eight of these employees report directly to her. (((span of control)))
Jim was recently promoted to the position of dining room supervisor at The Crab Shack, and he is now responsible for making decisions on numerous matters in the dining room and giving orders to the serving staff. At the same time, Jim has the obligation to perform the many new tasks assigned to him as a supervisor, such as overseeing the closing work and the cleaning of the dining room. Jim's obligations are known as his (((responsibility.)))
Patricia, the manager of Prime Health Club, was telling her new trainer that "I have many decisions to make in a day, and as the club manager, during the course of a week, I have to give orders to most of the 35 employees. Then there is my budget and the club resources, which I oversee." Patricia's _______ gives her the right to make decisions, give orders, and utilize resources. (((authority)))
Artem, the president of Instructor Services, told one of his new hires that "Managers at this company are encouraged to solve their own problems rather than buck the decision to a higher level, and decisions are made more quickly, which increases our organization's flexibility and efficiency." The key benefits of _______ are the advantages that Artem related to his new hires. (((decentralized authority)))
After being referred by a friend, Hasina attended a job interview. She didn't feel it went well, but she was surprised when her friend told her the boss didn't like that Hasina had worn her hijab, a traditional headscarf worn by Muslim women. The boss told a coworker after the interview, "Our customers prefer working with Christians." Hasina is a victim of (((discrimination.)))
________ entails job posting, which means placing information about job vacancies and qualifications in places where employees can see them such as on bulletin boards or the company's intranet. (((Internal recruiting)))
Sally, Greg, Juan, and Amar are working on a project for a customer that is aimed at cutting the client's electrical costs. The four members of this workgroup are located throughout the Midwest, and they are utilizing the phone, e-mail, and collaborative computing to complete this project. This workgroup is an example of a (((virtual organization.)))
Peter, the owner of Happy Burger, is involved in the company's daily decisions and has established procedures for every task. The company rules are clearly specified in the handbook, and Peter also believes in close supervision of all employees. Happy Burger is a(n) _______ organization. (((mechanistic)))
Which of the following is a good suggestion for managers when conducting interviews? (((After the interview, write a short report with a quantitative score of the candidate's qualifications.)))
According to Lawrence and Lorsch, the stability of an organization's environment determines the degree of (((differentiation or integration that is appropriate.)))
Tiffany is an architect for the building of a large office complex downtown. Frequently a group of steel workers whistle at her as she climbs a ladder or interacts with staff. The men's actions create a hostile work environment. (((TRUE)))
As a sales manager, Danyce is conducting performance appraisals for her team members. To do so, she tracks each person's sales calls, amount of sales, and revenues on a quarterly basis. Danyce is using a(n) ______ system of appraisal. (((objective)))
Explaining the organization's mission and operations, the job routine and the organization's work rules and benefits should be an integral part of employee orientation. (((TRUE)))
Every year, 10% of GE's managers are assigned the bottom grade during the annual review cycle, and if they don't improve, they are asked to leave the company. GE uses a forced ranking performance review system. (((TRUE)))
Dora is conducting a performance appraisal for Sue, one of her employees. The company's performance appraisal form asks her to rate Sue's performance on various items like "Submits reports on time with minimal errors" on a scale from 1 to 5. Dora's is using a ______ appraisal system. (((behaviorally anchored rating scale (BARS))))
Cody was surprised to learn that not everyone would receive a bonus this year. Instead, management planned to rank all of the employees in Cody's division and award bonuses only to the top 20% in terms of sales. Cody's company is using a(n) ______ performance review system. (((forced ranking)))
________________ is the purpose of the strategic human resource process. (((Getting optimal work performance to help realize the company's mission and vision)))
In considering the staffing an organization might need in the future, human resource managers should (((understand the organization's vision and strategy in order to hire personnel to support them.)))
A manager must consider fairness, __________, and others' resentments in considering a promotion of an employee (((nondiscrimination)))
A(n) ___________ is a report listing your organization's employees by name, education, training, languages, and other important information. (((human resource inventory)))
To ________, Ricco, an HR manager, used reports from the U.S. Bureau of Labor Statistics and the U.S. Census Bureau for information about his industry's labor pool in his geographic area. (((recruit from outside the organization)))
_______ are organizations of employees formed to protect and advance their members' interests by bargaining with management over job-related issues. (((Labor unions)))
A California software firm, MacroSoft, was growing rapidly and hiring frequently, but it primarily focused on campus recruiting at two California universities, UC Berkeley and Stanford. When one 45-year-old candidate did not get an interview after a phone screening, he went to the company's website and noted all of the young people in charge. Which law allows him to file a discrimination lawsuit against MacroSoft? (((Age Discrimination in Employment)))
Bethany had a starring role in a film that was shot in her hometown of Baton Rouge, Louisiana. Though the Screen Actors Guild, an actors' union, exists in Louisiana, Bethany was not required to join it because of the state's (((right-to-work)))
Typically the ________ clause in a union contract is tied to the U.S. Bureau of Labor Statistics' consumer price index (CPI). (((cost-of-living adjustment (COLA))))
Walt Disney Company's investment of $1 billion into a wearable technology that it hopes will revolutionize the way visitors spend money at Walt Disney World, featuring a website ("My Disney Experience") and data-collecting wristbands ("MagicBands") that interact with scanners throughout the 40-square-mile theme park, is an example of proactive change. (((TRUE)))
During annual inventory week, a department store may ask its employees to work 12 hours a day instead of the usual 8. During tax-preparation time, the store's accounting department may work similar hours. Although accounting employees are in a different department from stockroom and sales employees, it's reasonable to expect that the accounting employees wouldn't be terribly upset by the temporary change in hours because they've seen it in effect elsewhere in the store. This is an example of innovative change. (((FALSE)))
Procter & Gamble modified a liquid detergent to make it available as a concentrated powder in a pouch. P&G's new product is an example of a core innovation. (((TRUE)))
Keurig created a new category of coffee/tea preparation by offering one-cup-at-a-time pod-style brewing with its "K-cups." Keurig's product is an example of a transformational innovation. (((TRUE)))
________ change is made in response to arising problems or opportunities. BP's response to an explosion on a drilling platform in the Gulf of Mexico is an example of this kind of change. (((reactive)))
Walmart's implementation of RFID, a type of technology that allows it to improve inventory tracking, is an example of a(n) ______ change because Walmart already had a very effective technology tracking system in place before it implemented RFID. (((proactive)))
Firms such as General Motors and McDonald's actively collect information about customer preferences and try to address them in their new products. General Motors and McDonald's are actively trying to adapt to (((shareholder, customer, and market changes)))
Joan, a new manager, must enforce sales report deadlines, but her team is struggling. She creates a new system to streamline the process and helps everyone understand why the deadlines are important. On which of the following internal forces is Joan trying to have an impact? (((manager's behavior)))
Studies exploring nurses' perceptions about using PDAs in their daily patient practice found initial resistance, with some nurses concerned about the cost and short technological life cycle of these devices. As part of the __________ stage of change, the nurses' managers try to instill in them the motivation to change, encouraging them to let go of attitudes and behaviors that are resistant to innovation. (((unfreezing)))
Appliance America's customer complaints have increased because its drivers often get lost when attempting deliveries, which means that the deliveries arrived late. The drivers have resisted using GPS systems, claiming they know their territories. But managers are now encouraging them to look for the best solution to the increased level of customer complaints. Appliance America is in the ________ stage of Lewin's change model. (((unfreezing)))
The two cofounders of Network Appliance, a data-storage firm in Sunnyvale, California, were feuding with each other because one founder couldn't stick to his decisions, which drove the other founder crazy. A(n) ____________ began working with the warring executives in separate sessions to solve the problem. (((organizational behavior specialist)))
As owner and senior accountant at a tax preparation firm, Mark is changing the work schedule of all employees, including managers and accountants, for the months of March and April, the company's busiest time. He is requiring everyone to work on weekends, just as he has during the tax-preparation season for the last ten years. Mark should expect that his employees will be (((hardly upset or not upset at all.)))
Lewin's change model consists of (((three stages: unfreezing, changing, and refreezing.)))
During Lewin's changing stage, managers should (((give employees new information, perspectives, and models for behavior.)))
During Lewin's refreezing stage, managers should (((encourage and reinforce the desired change in the employees.)))
After Mary's Gifts and Cards surveyed its customers, it discovered that its customers preferred the company's competition more than 70 percent of the time. The most frequently cited reason was customer service. During which stage of Lewin's change model should Mary provide her employees with this information about the problems with customer service? (((unfreezing)))
When it comes to learning from failure in the innovation process, former Procter & Gamble CEO A. G. Lafley suggests that the key is to (((fail early, fail cheaply, and don't make the same mistake twice)))
Which of the following factors most reduces an organization's ability to learn from failure? (((employees who blame others for failure)))
Which of the following is one of Scott Berkun's seeds of innovation for organizations? (((philanthropy)))
A technological innovation that improves the performance and speed of a computer's operating system would be considered a(n) ______ innovation (((product)))
A managerial innovation that improves the efficiency of a company's cross-functional teams would be considered a ______ innovation. (((process)))
Personal Care Products recently introduced a new acai berry shampoo. Rinn, director of new product development, has just reviewed the dismal results. As it appears that his team adequately researched the product's acceptance by consumers prior to its introduction, he is confused by the outcome. But to keep his team open to innovation, Rinn should (((foster an innovative culture and climate that permits experimentation, risk taking, and failure)))
Which of the following is not a way to encourage and foster innovation? (((withholding raises and promotions when innovation attempts don't work out)))
According to the systems model of change, which of the following is not an input? (((the organization's products or services)))
A smoker who claims that the habit is not as dangerous as antismoking messages suggest, saying "My grandmother smokes and she's in her 80s," is attempting to increase cognitive dissonance. (((FALSE)))
Your supervisor says the company needs you to stay late to meet an important deadline, but your family expects you to be present for your child's birthday party. You are experiencing role overload. (((FALSE)))
Carolina's manager notices that she exhibits an internal locus of control when she speaks about her work. Her manager should (((provide incentives such as merit pay or sales commissions.)))
Boring, tedious jobs generally reduce people's perceptions of their (((self-efficacy.)))
If a manager knows one of her employees has low self-esteem and wishes to enhance it, she should (((reinforce the employee's positive attributes and skills.)))
People high in ______ are responsive to others' social and interpersonal cues. (((emotional intelligence)))
________ are abstract ideals that guide one's thinking and behavior across all situations. (((Values)))
The statement "I really don't like that Ivan got so angry in that meeting" reflects the ______ component of an attitude. (((affective)))
The statement "I won't give Roberto such a tight deadline again" reflects the ______ component of an attitude. (((behavioral or intentional)))
_____________ is the process of interpreting and understanding one's environment. (((Perception)))
____ is the tendency to attribute another person's behavior to his or her personal characteristics rather than to the situation the person is in. (((Fundamental attribution bias)))
Because she beat her goal this year by nearly 30 percent, Lori has been telling everyone in the sales department of her incredible skill as a salesperson. But last year when she didn't even reach her goal, she said her failure was due to the poor economy. Lori provides an example of (((self-serving bias.)))
Carol really doesn't like her new boss and is not happy with the new tasks she's been assigned and the long hours she's been working. Still, she truly believes in what the company is trying to accomplish. Carol has (((low job satisfaction.)))
A person's overall satisfaction with work depends on how he or she feels about several components. Which of the following is not one of these components? (((life outside of work)))
The Americans with Disabilities Act requires employers to (((reasonably accommodate an individual's disability.)))
Jake has a master's degree in psychology, but he is working at a sunglasses cart in the mall. He is currently experiencing (((underemployment.)))
______ is the belief that one's native country, culture, language, abilities, or behavior is superior to those of another culture. (((Ethnocentrism)))
When others' expectations exceed one's ability, ______ occurs. (((role overload)))
Motivation cannot be directly observed in another's behavior; it must be inferred from one's behavior. (((TRUE)))
To motivate employees, employers need to primarily focus on fulfilling the top level of Maslow's hierarchy of needs, self-actualization. (((FALSE)))
John's manager just told his team about this year's contest, the winner of which will receive an all-expense paid trip to Bangkok. John does not like to travel to places where he can't speak the language, so he is not very enthusiastic. This reward has a high valence for John. (((FALSE)))
Ricardo's sales manager just informed his sales team that all sales in January will earn an extra 5 percent commission. The team gets right to work, being motivated by a(n) (((extrinsic reward)))
Paula's management professor just told her class that the final exam is optional for students like Paula who currently have an A. Paula decides to take the exam anyway because she likes the subject and wants to master the material. Paula is motivated to take the exam by a(n) (((intrinsic reward.)))
Maslow's levels of needs, in order from lowest (most basic) to highest level, are (((physiological, safety, love, esteem, and self-actualization.)))
After three shuttle accidents in three months, one of which resulted in a critical injury to a driver, Citywide Shuttle drivers received additional training, and the company retrofitted all shuttles with new braking systems. Which of Maslow's needs are addressed by Citywide Shuttle's actions? (((safety)))
An accounting firm reimburses employees for tuition and fees if they complete job-related coursework with a B or better. The accounting firm's tuition-reimbursement policy helps its employees meet their _______ needs. (((self-actualization)))
For managers, the importance of Maslow's contribution is that he showed that workers (((have needs beyond that of just earning a paycheck.)))
Using self-determination theory to motivate employees at Cloud9, a data storage company, the vice chair makes 10–20 phone calls a day to thank special employees "caught doing something right." The vice chair is fulfilling her employees' ______ needs. (((competence)))
Even when he started his first job, Will was not content to be just one of the employees. His boss noticed that he often coached his coworkers about ways to improve their work, even when it wasn't his job. Will probably has a (((high need for power)))
McClelland's need for achievement corresponds most closely to (((McClelland's need for achievement corresponds most closely to)))
According to Herzberg's two-factor theory, in the zone between the motivating factors and the hygiene factors, employees are (((neither satisfied nor dissatisfied.)))
According to Herzberg's two-factor theory, only ______ factors can make employees satisfied with their jobs. (((motivating)))
According to Herzberg's theory, the first thing managers of employees who dislike their jobs should do is to (((make sure pay levels, policies, and working conditions are reasonable.)))
Randy complained to his boss, Maryann, that he received the same bonus this quarter as everyone else, despite the longer hours he had worked and his higher level of experience, production, and efficiency. If Maryann can't change the bonus, she should expect that Randy might respond in any of the following ways (((ignoring his feeling of resentment and trying harder in next quarter.)))
At his review last year, Bryan was promised a 20 percent raise if he met his production goals. Raises were included in today's paychecks, and although Bryan has met all of his goals, he received only a cost-of-living raise. In the future, Bryan's ______ will probably be (((instrumentality; low)))
______ involves division of an organization's work among its employees and applies motivational theories to jobs to increase satisfaction and performance. (((Job design)))
Michele's job as an accounting assistant was recently modified to include reconciling bank accounts and making deposits, two tasks previously done by the accounting manager. This increase in responsibility would best be described as (((job enrichment.)))
A technician who is responsible for keeping an airport's control tower's electronic equipment in working order has higher ____________ than a person wiping down cars in a carwash. (((task significance)))
As a maintenance person for the air force, Alex services aircraft engines, which protects the lives and safety of military personnel and increases their ability to conduct missions. Alex's job has a high level of (((task significance.)))
A supervisor told a salesperson who had not made any calls to clients and therefore did not make quota, "Well, if this continues in the next 30 days, you'll probably be let go." The supervisor provides an example of _____ by presenting the likely negative outcome to the salesperson. (((punishment)))
Ted's manager required him to give up his company car because he had missed sales goals for four consecutive quarters. Ted's manager used (((punishment.)))
Nancy's employer distributes checks at the end of each quarter, representing an equitable portion of 5 percent of the company's pretax profits for the previous period. These checks represent (((profit sharing.)))
Daniel teaches sixth grade at a local elementary school. He significantly increased his salary by earning a master's degree in education. Daniel's increased salary is an example of (((pay for knowledge)))
During the storming stage of Tuckman's five-stage model, the leader of a team should allow members the empowerment they need to concentrate on solving problems and completing the assigned task. (((FALSE)))
The best way to manage virtual teams is to focus on what's accomplished, not whether an employee is working from her patio or at 10 p.m. (((TRUE)))
Devil's advocacy is the process of assigning someone to play the role of critic to voice possible objections to a proposal. (((TRUE)))
Molly organized several teachers to discuss the school's interior painting scheduled for the summer. They looked at several brands and heard a presentation by a designer who then helped them choose a color palette to recommend to school administrators. In this instance, the teachers made up a(n) (((formal group.)))
The group development stage in which individuals test the leader's policies and assumptions as they try to determine how they fit into the power structure is the ______ stage. (((storming)))
Depak's team was not making much progress on defining a new production process. Nicole was being very uncooperative. She did not agree with Depak's direction, and she has not completed her tasks. This team is in what stage of group development? (((storming)))
Because hard feelings about group leadership and assignments have passed, the members of Robin's group now seem to be relating much better. At the meeting tomorrow, she should take advantage of this moment by (((helping the team identify group goals and values.)))
Levi led a team that has just finished up a very challenging research project that will assist management in developing long-range plans. Despite the stress of the past few months, most participants seem sad it's over. Now Levi should (((have an awards ceremony.)))
________ is defined as reciprocal faith in others' intentions and behaviors (((Trust)))
A(n) ______ role is behavior that concentrates on getting the team's work done. For example, one team member's role may be to generate daily sales information at the end of each working day. (((task)))
Someone at a team meeting who says, "Let's hear from those who oppose this plan," is performing a ______ role. (((maintenance)))
Tension was apparent as the management team discussed changes to promotion requirements. But then Chris made a joke about Jake's white-knuckled grip on his pen, and the laughter seemed to lighten the mood. Chris was acting in a ______ role. (((maintenance)))
The Nordstrom department store chain emphasizes the great lengths to which it goes in customer service. By enforcing its norms for exceptional customer service, Nordstrom is seeking (((to emphasize the organization's important values and identity.)))
Paige missed another deadline and her boss, Linda, is very upset. She will have to explain to the client again why the project is behind. Linda thinks she may say something she'll regret if she talks to Paige about this now, so she decides to wait a while. Linda is using the ______ conflict-handling style. (((avoiding)))
______ is designed to elicit different opinions without inciting people's personal feelings. (((Programmed conflict)))
Rebecca asked Gavin, one of her team members, to purposefully think of and voice criticisms as the group discussed a popular idea to open a branch office in another state. This is an example of the use of (((devil's advocacy.)))
When Men's Wearhouse fired a salesperson who wasn't sharing walk-in customer traffic, and total clothing sales volume among all salespeople increased significantly, the company (((reduced destructive internal competition.)))
One's power comes as part of one's job; it is the right to perform or command. (((FALSE)))
Reward power results from managers' authority to punish their employees. (((FALSE)))
______ power results from one's specialized knowledge. (((Expert)))
______ power derives from one's personal attraction. (((Referent)))
Lamar brought his proposal to Lindsay before the meeting saying, "I'm sure you have some ideas on this, and we could try to get them incorporated upfront." Which influence tactic is Lamar using? (((consultation)))
When Nelson saw Betty in the stockroom stuffing her purse with expensive printer cartridges, Betty was quick to say, "We've been friends since first grade, so I'm sure you won't say anything about this." Betty was using a(n) _____ on Nelson. (((personal appeal)))
Regina apologized to Clay for an e-mail that upset him. She said she had chosen an inconsiderate way of stating her idea, and that she'd be happy to discuss it further. Regina is (((expressing consideration behavior.)))
Fiedler's contingency leadership model determines if a leader's style is (((task-oriented or relationship-oriented.)))
____ is the model that requires a manager to assess her own leadership orientation and the level of her situational control. (((Fiedler's contingency model)))
_____ is the questionnaire used in Fiedler's model to determine leadership orientation. (((Least preferred coworker (LPC) scale)))
In Fiedler's contingency leadership model, ________ is the amount of influence a leader has in his or her immediate work environment. (((situational control)))
David's peers were surprised when he received a promotion and suddenly became their supervisor. They thought he was rather unreliable and weren't sure he was up to the task. According to the contingency model, David has (((poor leader-member relations.)))
Jane leads a task force developing specifications for a new customer database to be used by several departments. Jane is an IT supervisor, but most of the other task force members are directors of other departments. At the first meeting, a few of them asked questions that she couldn't answer. According to the contingency model, Jane's situational control is likely (((low.)))
Marcel supervises a group of paralegals serving the firm's lawyers. He gets along well with his employees, and he has created detailed procedures for all the types of legal document they encounter. Marcel hires and fires; he also gives work assignments, performance appraisals, and promotions. The optimal leadership style for Marcel is _____ according to the contingency model. (((task-oriented)))
According to Fiedler's contingency model, ______ situation control favors a leader who is ______-oriented. (((moderate; relationship)))
____________ is the model in which an effective leader makes desirable rewards available, clarifies how employees can achieve objectives, and provides them support in doing so. (((House's path-goal model)))
According to House's revised path-goal theory, a leader's style should vary depending on (((employee characteristics and environmental factors.)))
The leadership styles of ________ originally included supportive and achievement-oriented and two others, but was more recently revised to include a total of eight styles. (((House's path-goal model)))
Judith supervises a call center department that receives stress-producing calls from unhappy customers. Turnover has increased by 33 percent over the last four months. Judith is understanding and patient with her staff, and tells them she knows what they're going through since it wasn't so long ago that she took those calls. According to revised path-goal theory, Judith is using a(n) ______ leadership style. (((supportive)))
Pat is something of a cheerleader around his team. He shares his vision and expresses his confidence in his team's ability to achieve his vision. He is quick to compliment and acknowledge team members' accomplishments, and he is enthusiastic about their successes. According to revised path-goal theory, Pat is using a(n) ______ leadership style. (((value-based)))
House's revision of his theory puts more emphasis on the need for leaders to foster (((intrinsic motivation through empowerment.)))
Lynn had many spelling and grammar mistakes in her report, some of which made the content difficult to understand. Lynn's report is an example of poor encoding. (((TRUE)))
The sun was shining directly into Matt's eyes, so he looked away and missed the coach's demonstration of a defensive technique. In this case, the sun acts as feedback in the communication process. (((FALSE)))
The danger of using a lean medium for nonroutine matters is that it results in information oversimplification; it doesn't provide enough of the information the receiver needs and wants. (((TRUE)))
Jyl, a supervisor, told her employees, "The store will close Monday for inventory. All employees are expected to participate." Jyl's communication to her employees is an example of upward communication (((FALSE)))
When the band came out on stage and the lead singer shouted, "Hello, Dallas!" the crowd roared with applause. The crowd's response is an example of (((. feedback.)))
Which of the following is not an example of noise in the communication process in a typical classroom? (((a student answering a professor's question)))
Which of the following is the most appropriate medium for a manager advising employees of a revision to the company's bonus structure? (((face-to-face meeting)))
Roger was talking to a coworker in a meeting, and he wasn't listening when his supervisor announced work assignments. This situation is an example of what type of barrier to communication? (((receiver barrier)))
Which of the following is a "don't" when attempting to improve communication? (((Close your eyes.)))
Which of the following statements is true? (((Men tend to withdraw and isolate themselves when problem solving, whereas women seek out others for support and can interpret men's withdrawing as lack of caring.)))
________ is a person's characteristic speaking patterns, such as the use of pacing, pausing, questions, and stories. (((A linguistics style)))
Doug gave some people street directions, but because they only nodded their heads and didn't repeat the directions back to him, he doesn't really know whether the directions were understood. What type of barrier is present here? (((feedback)))
Which of the following is an example of empathetic communication? (((I understand your frustration; this deadline is intense, and we're running on reduced staff.")))
Ryan is sitting in a seminar where a speaker is explaining the company's retirement plan options. Ryan is only 24 years old, and he thinks retirement is so far away that he doesn't think he needs to pay attention to the speaker. He starts texting his friends and updating his Facebook page; he even slumps in his chair. He has completely tuned out of the presentation. Ryan's listening style in this case is best described as (((detached.)))
Maslow's hierarchy of needs includes all EXCEPT which of the following?((cognition))
In Maslow's hierarchy of needs, food, water and sleep are considered _____ motives.((physiological))
it is very important for the company to analyze the competition exerted on it by different actors. for this, the company uses: (( Porter's five forces model ))
During a manager's meeting, Paula, a district manager, discussed how to handle sensitive employee issues and indicated correctly that management is really just an art.((FALSE))
In Maslow's hierarchy of needs, food, water and sleep are considered _____ motives. ((physiological))
Maslow's hierarchy of needs includes all EXCEPT which of the following (( cognition ))
During a manager’s meeting, Paula, a district manager, discussed how to handle sensitive employee issues and indicated correctly that management is really just an art. (((FALSE)))
Ali, a CEO of a large IT organization, understands that many challenges are coming in his industry; therefore, he tells his upper-level management that it’s helpful to look at theoretical perspectives of management to help make predictions and develop principles to guide future company strategies and actions. (((TRUE)))
When faced with a business problem, Sally, a young grocery store owner, discusses business problems with Don, the assistant manager and an employee for over 30 years. Sally believes Don "has seen it all" and knows there are few really new ideas; plus the business has done many simple things, such as offering friendly service, to create and maintain a competitive edge. Sally’s management style is best described as (((evidence-based.)))
Susan, a movie theater manager, recently implemented a policy stating that workers who call in sick on Friday or Saturday nights will have their free movie privileges revoked for one month. In observing her workers, she has noticed that some of them feel that the new policy is not fair, and they have not been working as hard as they did in the past. What are Susan’s employees doing? (((soldiering)))
Hugo Munsterberg believed that psychologists can contribute to industry by _____. For example, Munsterberg might suggest that a company implement a monthly bonus plan that rewards workers for achieving the goals set by their managers. (((devising management strategies to influence employees to follow management’s interests)))
Mary Parker Follett anticipated some of today’s workplace trends, such as (((self-managed teams and worker empowerment.)))
Maslow’s hierarchy of human needs included ((( physiological, safety, love, esteem, and self-actualization.)))
Paul English of Kayak.com uses new employees as an excuse for altering an existing open-office seating arrangement. English’s action is an application of ____ to management. (((the behavioral science approach)))
Gabriel, the CEO of a large global production company, is excited about the introduction of statistics and computer simulations in the design stage of the company’s products. Gabriel is using (((quantitative techniques.)))
Operations management focuses on managing the production and delivery of an organization’s products or services (((more effectively.)))
A computer manufacturer is seeking to cut costs by designing an inventory system that reduces the number of finished products in stock due to overproduction and to set in place a production schedule that better matches customers’ orders. These plans are an example of ____ in action. (((operations management)))
The Jones family had a fire that destroyed their home. Various departments of a restoration company, from the initial cleanup crew to the assistance with the Jones family moving back in, worked together to achieve this goal. All of the company’s interrelated parts working together to accomplish a goal (in this case, the restoration of the Jones home) is an example of a(n) (((system.)))
A local farmer grows and sells tomatoes and beans to the local grocers. Look at the farmer’s business as a system. In which category do the tomatoes, beans, squash, profits, and losses fall? (((output)))
Donna, a chef in a fine restaurant, utilizes top-of-the-line equipment as she prepares meals for customers. Donna and her equipment are a(n) ______ in the restaurant’s system. (((input)))
When a line cook in a restaurant cooks uses raw meat to cook a hamburger that becomes part of the restaurant’s Super Burger Special (available only on Saturday nights during football season), the cook is taking part in a (((transformation process.)))
For years ABC Copier, a copier machine supplier, enjoyed strong sales and a huge share of the copier market, far ahead of its nearest competitor. However, over the years, ABC seldom asked customers for feedback. So when some of ABC’s competition responded to customers’ needs and started offering copiers with new features (such as wireless printing from laptop and tablet computers), ABC lost much of its market share. ABC’s system is best described as (((closed)))
Dominique, an executive chef in a large hotel, recently attended a training conference sponsored by several top professionals in his field, where he learned numerous ways his restaurant and hotel can better serve customers. Dominique is excited that management asked him to present this information to the hotel staff so that the restaurant and hotel can make needed improvements. Dominique’s hotel is a(n) _____ organization. (((learning)))
The task environment of Top-Notch Sewing consists of just two groups, customers and stockholders, who give the employees and management of Top-Notch the daily tasks that the company’s employees will handle. (((FALSE)))
Pleasant Resort, a successful 100-year-old hotel and resort, entered into a co-marketing arrangement with A-Plus Hotels, a five-star hotel chain. Both companies benefit from the new relationship. Pleasant is now able to market its facility and services through A-Plus’s marketing team, and A-Plus Hotels now offers access to an upscale resort that it can recommend to its customers who are looking for a resort experience. A-Plus Hotels and Pleasant Resort can be considered strategic allies. (((TRUE)))
Paul, the vice president of marketing in an international sales organization, is considering opening an office in a new country. He is looking at the current economic, political, legal, and technological global forces in that country that can affect the success of his company’s expansion into the new area. These forces that Paul is studying are international forces. (((TRUE)))
Just before introduction of a new revolutionary laptop, the Top205 by Top Computers, Greg, the CEO of Top, was informed that a problem with a part inside the Top205 may cause reliability issues. Greg knows that correcting the problem will delay the introduction of the product, and be expensive due to possible order cancelations and added advertising costs. But he also knows that allowing the new flawed computer into the market will affect Top Computers’ reputation. Greg’s decision whether to delay the introduction of the Top205 or proceed as planned is (((TRUE)))
Alonzo, the president of his high school’s senior class and honor society, has been very busy and did not study for an important history test. Although Alonzo is very confident, he realizes that he probably will not do well on the test. He has talked himself into cheating because he believes that cheating will help him maintain his strong GPA, which he will need to be accepted by a good college. He tells his friend, "I don’t usually cheat, but I really have to do it." Which of the following might explain Alonzo’s behavior? (((motivated blindness)))
Don, the owner of a bait and tackle shop, attended an entrepreneurship workshop that discussed the triple bottom line, which measures an organization’s ____ performance. (((social, environmental, and financial)))
What mechanism is used to provide a systematic assessment of an organization’s performance in implementing socially responsible programs, often based on predefined goals? (((social audit)))
An organization’s internal stakeholders consist of (((the board of directors, employees, and owners.)))
The marketing director for Zap Games, a video game company, has informed his employees that he feels the company needs to improve its relationship with the distributors of the company’s products because the distributors are part of the ______ environment for Zap Games. He believes that a stronger relationship with distributors will lead to more market share, and higher profits, for both Zap Games and its distributors (((task)))
Don was recently promoted, and his salary increase includes retirement investment options, so he has decided to take part in an ESOP. This means that Don (((will be buying or receiving shares of his company’s stock.)))
Amy, a successful banker and educator, has decided to retire, but she is very interested in staying involved with a company "that cares." So when asked to serve on its board of directors, she gladly accepted the invitation. As a member of the board, Amy will be involved with the organization’s (((overall strategic goals and approval of major decisions.)))
ABC Brothers’ cleaning products and equipment are sold to consumers and commercial cleaning companies in Best Supply’s janitorial supply and equipment store. ABC relies heavily on Best Supply to help sell its goods and services to customers. Best Supply is a(n) ____ for ABC Brothers’ products. (((distributor)))
Joe, a lobbyist who represents the oil industry, is meeting with several members of Congress to try to exert political influence by contributing funds to the lawmakers’ election campaigns. Joe also is involved in a letter-writing campaign to promote his cause. Joe is working on behalf of a (((special-interest group.)))
The mass media are part of an organization’s (((task environment.)))
Tom and his managers are discussing the unemployment, inflation, and interest-rate trends that might affect their chain of coffee shops over the next 12 months and the projected growth in the areas where the stores are located. The managers are studying the ______ forces in their organization’s general environment. (((economic)))
_____ forces lead to new developments in methods for transforming resources into goods or services. For example, a new technique called hydraulic fracturing (or fracking) has been developed to harvest natural gas trapped below the earth’s surface. (((Technological)))
Changes in the economic, political, legal, and technological global system that may affect an organization are ______ forces. For example, stresses within the European Union (including the United Kingdom’s recent decision to exit the EU) may have global ramifications. (((international)))
Joe, the owner of ABC Electronics, just discovered that his trusted friend Paul, his accountant for over 30 years, has been mishandling the company books and stealing from the company bank account. Joe must decide whether to ignore his friend’s actions and avoid bad publicity for ABC Electronics, or to have Paul arrested for embezzlement. Joe is facing (((an ethical dilemma.)))
. ______ are the relatively permanent and deeply held underlying beliefs and attitudes that help determine a person’s behavior. These beliefs may include the idea that employees should be promoted solely on the basis of merit, that no bribes will be paid to foreign officials to obtain business licenses, and that companies should offer extended maternity leave upon the employee’s request. (((Values)))
Consider the following situation: Local livestock farmers could allow the runoff of manure nutrients into a stream that feeds a local lake because polluting the stream helps farmers in the short term. However, farmers will act ethically in the short run to avoid harming others in the long run because doing so is also in the farmer’s best long-term interests. This type of thinking reflects the _____ approach to deciding ethical dilemmas. (((individual)))
With recent reports of identity theft, Mr. Jones, the CEO of a construction company, is concerned about his employees’ privacy, and because of recent accidents on the job, he is also worried about the safety of his workers. Mr. Jones’s concerns with privacy and health and safety are key elements in the _____ approach to deciding ethical dilemmas. (((moral rights)))
Greg, the president of an IT company, is friends with Jack, the CEO of HyperTech, a company that develops and manufactures computer hardware. Jack tells Greg that HyperTech is about to announce the creation of a revolutionary new computer processor that will quadruple processing capacity and cause HyperTech’s sales and profits to skyrocket. Greg then purchases a large number of shares of HyperTech company stock before the news of the new computer processor is made public. In which illegal activity has Greg engaged? (((insider trading)))
Jane, a successful store manager, leads her employees by encouraging them and showing that she genuinely believes in them. She also encourages group and team efforts. Jane manages at Kohlberg’s _____ level of personal moral development. (((conventional)))
Don has managed several restaurants for over 50 years and is close to retirement. With his maturity and business experience, his decisions are sound, based on solid, proven values. Don tends to be an independent thinker who focuses on empowering his employees. Don manages at Kohlberg’s ______ level of personal moral development. (((postconventional)))
The employees of Mom’s Pizza, a local pizza restaurant, feel that the owners and managers really try to follow the ethical policies outlined in their company’s code of ethics. Pete, a cook, said that "Just today, Joe, our manager, called our supplier to let them know we received an extra case of tomatoes that we did not pay for. And they do this type of thing almost every day." Mom’s Pizza maintains a(n) (((ethical climate.)))
A(n) ____ is a formal written set of ethical standards guiding an organization’s actions. (((code of ethics)))
Phillip said to a close friend, "I am fed up with my company’s continual disregard for the environment. They secretly dispose of hazardous chemicals into a nearby stream constantly. I don’t know how they get away with it! I am going to call a reporter at the local newspaper and reveal what my company is doing." Phillip’s decision to tell somebody about his company’s actions is an example of (((whistle-blowing.)))
As the CEO of a company that produces products for schools, Hannah believes that her company needs not only to produce a profit but also to do things that benefit society. _______ guides Hannah’s beliefs. (((Social responsibility)))
The company that Don owns, the Ardmore General Store, is a family-owned company that has been in business for more than 100 years. Don wants to give back to the people of the community to acknowledge their role in the store’s success. He decides to donate a significant portion of the store’s profits to a charity every year. Don’s decision is an example of ___ in action. (((corporate social responsibility)))
Mr. Smythe, a wealthy businessman who made his fortune in the oil industry, understands the importance of a college education, so he has donated over $10 million over the last several years to colleges and universities. Mr. Smythe’s actions are an example of (((philanthropy.)))
On an organization’s board of directors, (((inside directors may be members of the firm and outside directors are supposed to be elected from outside the firm.)))
With regard to boards of directors, and in particular their oversight of the CEO, the board of directors’ _______ is the biggest complaint (((lack of independence from the CEO)))
Sally has seen such great interest in her scented candles that she has decided to start her own small business selling them. Sally’s company can use the Internet and the World Wide Web to operate globally, helping her get a global business started more easily because she can put products on a website and sell worldwide. So, in a sense, the Internet wipes out the former advantages of distribution and scope that large companies used to have. (((TRUE)))
In 1999, the top exporting nation was the United States, but in 2015 the world’s top exporting nation was China. (((FALSE)))
A greenfield venture is a strategic alliance with individuals and organizations in developing nations; the venture’s goals are to assist, educate, and share the risks and rewards of starting a new nonprofit organization in the developing country. (((FALSE)))
After a morning of global cultural training, Marie, the vice president of marketing, is having lunch with Ann, who will be managing the company’s new office in South Korea. Marie tells Ann, "There are cultural differences between the United States and South Korea that will affect the success of our office. For example, I suggest giving a couple of cans of Spam to your larger customers, because Spam is considered elegant in South Korea." Marie’s advice to Ann is good because it takes into account the many kinds of cultural differences between the United States and other countries. (((TRUE)))
Last week Paul, CEO of Quality Furniture in South Carolina, traveled to Europe to visit customers. While overseas, Paul checked his e-mail daily and showed his company’s website to customers, explaining how the website will help them place orders and receive merchandise more quickly. After visiting the last customer Friday morning, Paul was able to return to the corporate office in South Carolina to meet with his board of directors that night. _______ is the "shrinking" of time and space with air travel and electronic media. (((the global village)))
John, a construction manager, has been having problems finding quality employees in some of the company’s foreign offices. While speaking with his attorney, Alisha, he mentioned that "in our locations overseas, many of our good employees have been quitting, and as you know, we have numerous lawsuits over personnel policies." Alisha explained that historically, problems with employee turnover and lawsuits are a result of ____ policies. (((ethnocentric)))
Roberto, the vice president of marketing, is talking to Francis, a sales manager in a foreign office, about several local situations that Francis’s reps are facing in their territory. Roberto tells Francis that he believes there are differences and similarities between home and foreign personnel and practices, and that Francis should use whatever techniques are most effective. Roberto is a(n) ____ manager. (((geocentric)))
Alice’s Pasta Kitchen makes quality pasta and sells its products to customers in other countries. Alice’s Pasta Kitchen is ____ its products. (((exporting)))
Using ________, Don’s Carpet installed carpet at no cost for City Office Supply, in exchange for $1,000 of office supplies. (((countertrading)))
Which of the following is an example of licensing? (((When a company allows a foreign company to pay it a fee to make or distribute the first company’s product or service.)))
Luxere Hotels International, an American company, sells the rights to other hospitality companies globally to open hotels with the Luxere name for a fee and a share of the profit, in return for using Luxere’s brand name and a package of materials and services. Luxere is engaged in (((franchising.)))
Sometimes a country’s laws forbid foreigners from owning a business in the nation. In the presence of such laws, a(n) _____ is one way for an American company to have a presence in that foreign country. (((joint venture)))
When planning a trip to France, Jim and his wife, both Americans, were concerned about how much they could afford to spend in Europe because sometimes the U.S. dollar will buy more goods and sometimes it will buy less, based on changing economic conditions. The ____ rate is the rate at which the currency of one area or country can be exchanged for the currency of another’s (((exchange)))
Tom and his fiancée Susan decided to have their wedding on a beach in Acapulco, Mexico. Local businesses provided most of the services for the wedding, such as the limousine and the catering services. Throughout the weekend of the wedding, the couple found that workers and limos arrived late; so did the priest who was performing the wedding. In response to the couple’s frustration, the Mexican workers remarked, "Ustedes tienen que ser flexibles," which in English means "You have to be flexible with time." This attitude regarding time is (((polychronic.)))
.______ is a government’s seizure of a domestic or foreign company’s assets. (((Expropriation)))
Greg, a geologist from Kansas who works for ExxonMobil, has decided to transfer to one of ExxonMobil’s offices in Saudi Arabia. Greg is a(n) ____ while living in Saudi Arabia, (((expatriate)))
Which of the following statements is FALSE? (((The United States was the world’s largest economy in 2014.)))
Spark, Inc., which manufactures the fireworks that municipal governments buy to put on their annual fireworks shows on the Fourth of July, moved most of its manufacturing operations from the United States to China a decade ago. Now, Spark is moving production back to the United States because the company’s managers are uncomfortable with many Chinese business practices. Spark, Inc. engages in __________ when it moves its operations back to the United States. (((reshoring)))
Roberto works for a company that highly values performance-oriented traits, such as achieving a certain level of sales and increasing market share by a certain percentage. In analyzing this aspect of Roberto’s company, Geert Hofstede would examine the company’s _______ dimension. (((masculinity/femininity)))
The upper-level managers of Deluxe Coffee are meeting to discuss the company’s long-range goals and vision. Like any organization, Deluxe Coffee should adopt planning and strategic management for just two reasons: (1) to provide direction and momentum and (2) to encourage retention of high-value employees. (((FALSE)))
Paula is the store manager for a national grocery retailer, and she received the yearly tactical plan from Greg, her district manager. Paula is meeting with her department managers to plan how to implement the specific tasks outlined in Greg’s plan for the next year. Paula is engaged in operational planning. (((TRUE)))
Jerome’s Italian Pizza is in a very competitive industry, and its upper management believes that it can achieve higher profits by focusing on a strategy of lowering costs and prices, providing quicker delivery, and providing good customer service. "Increase profits by 15% a year for each of the next five years" is an example of a strategic goal. (((TRUE)))
When utilizing MBO, the manager and employee jointly set objectives for the employee, the manager develops action plans, the manager and employee periodically review the employee’s performance, and the manager makes performance appraisals and rewards the employee according to the results. (((TRUE)))
With increasing competition in the restaurant business, Sal, owner of Mom’s Pizza, is dealing with growing uncertainty. He is meeting with his managers to develop a future course of action to meet company goals. This process is known as (((planning.)))
The profits of ABC Grocery have dropped significantly over the last several months. After investigating, the owner realizes that many of the store’s departments are moving in the wrong direction. The owner realizes that these departments need stronger and closer management, but she also realizes that she has not provided her managers with enough direction regarding her expectations for the store’s goals and the employees’ performance. This example shows how an organization can progress in the wrong direction as a result of (((poor planning.)))
An organization’s _______ expresses the need the firm will fill, the operations of the business, its components and functions, and the expected revenues and expenses. (((business model)))
Clothing maker Patagonia states that its purpose as an organization is to "build the best product, cause no unnecessary harm, [and] use business to inspire and implement solutions to the environmental crisis." This is the company’s (((mission statement.)))
Elite Coffee and Bakery, a café that offers a relaxing environment to enjoy quality coffees, teas, soups, and other menu items, has a poster in the front of its restaurants that reads, "We will always provide a quality product to help our customers better meet the challenges of their day and always take the time to offer caring service one person at a time." This statement is Elite’s (((mission statement.)))
During a management meeting, Dave, the CEO of Top Office Equipment, reminded his management team of where the company wants to go as an organization: "Top Office Equipment is the expert whom customers should always call regarding all their office equipment needs because Top Office strives to understand customer needs and provide the right answers for them." This statement is Top Office Equipment’s (((vision statement.)))
One Fortune article states that "vision should describe what’s happening to the world you compete in and what you want to do about it" and (((it should guide decisions.")))
A company’s strategic planning may have to be done _____ due to frequently changing competition and technology. (((closer to every one or two years than every five years)))
The upper management of a large national retail grocery store has passed along the new goals of improving customer service to the district managers. At this point, the district managers need to determine how the stores in their district will achieve top management’s goal with the given resources during the next 6 to 24 months. The district managers are developing ______ planning. (((tactical)))
The top management of Tasty Foods, a food distribution company, has set strategic goals of increasing organizational market share and also decreasing corporate costs over the next three years. Greg, a division manager for Tasty Foods, has looked at his resources, and he has decided how his division can contribute to the two strategic goals set by upper management: (1) by partnering with another company and (2) by hiring a procurement manager to negotiate lower prices from vendors. Greg’s next step is to roll out his ____ to his staff (((tactical goals)))
Warby Parker, a manufacturer of fashionable prescription eyewear, notes on its website, "Warby Parker was founded with a rebellious spirit and a loft objective: to offer designer eyewear at a revolutionary price, while leading the way for socially-conscious business." This excerpt from the company’s website states Warby Parker’s (((vision statement.)))
The management of Quality Carpet Cleaning finalized the company’s action plan for increasing its market share over the next few years. The management then set up a 12-month plan that defines how the organization will conduct its business based on the action plan, including clear sales targets for each week. The second plan is an example of a(n) _____ plan. (((operating)))
Paula, the store manager, asked Tom to organize the prize giveaways for the store’s grand reopening open house. Tom is organizing a(n) (((project)))
Billy told his boss, Gwen, that he is going to start training for the upcoming marathon to be healthier and to have more energy for work. When Gwen asked him about his running history, Billy said, "I really have never run before and don’t exercise much. I just know I need to do something." Gwen told Billy, "You should start slowly, maybe setting smaller, realistic exercise goals that are achievable." Gwen is helping Billy set a (((SMART goal.)))
With setting SMART goals, you should choose only a few goals on which to focus. These goals should be results-oriented and (((support the organization’s vision.)))
____ is the process for motivating employee performance in which the manager and employee jointly set objectives for the employee, the manager develops action plans, the manager and employee periodically review the employee’s performance, and the manager makes a performance appraisal and rewards the employee according to the results achieved. (((MBO)))
André, the plant manager, is interested in increasing the facility’s productivity by utilizing MBO so that his managers and their employees are more focused on objectives. This month André asked his managers to concentrate on the two first steps of MBO, which are (((to jointly set objectives with their employees and to have managers develop action plans.)))
Steve, a hotel general manager, has had great success using MBO. Hannah, his district manager, has found that Steve’s employees are highly motivated. Because Steve _________, Hannah is able to discern that he is using MBO. (((rewards his employees with compliments, raises, and other benefits as they meet their goals)))
You are reading a business document that outlines your company’s long-term goals and direction. These goals include "Reduce energy consumption in our offices by 20%," "Grow out market share in China from 3% to 10%," "Enter a strategic alliance with a partner in Australia to begin selling our products in Sydney and Brisbane," and "Equip all sales representatives with an app that allows them to provide an immediate quote when meeting with customers." What type of document are you reading? (((a strategic plan)))
Based on the VRIO framework, which of the following start-up companies has the least potential to have a competitive advantage, to be profitable, and to grow? (((a cat-sitting service, where you take care of people’s cats when they are on vacation; you decide to advertise your services on Craigslist and the local newspaper)))
The Whole Foods chain of supermarkets has stated, "With great courage, integrity, and love, we embrace our responsibility to co-create a world where each of us, our communities, and our planet can flourish. All the while, celebrating the sheer love and joy of food." This proclamation is Whole Foods’ (((values statement)))
Which of the following is NOT a step in the process of management by objectives (MBO)? (((Managers determine each employee’s strengths and weaknesses.)))
As a manager, you meet with each of your team members quarterly to review their progress toward their written and agreed-upon goals. One of your employees, Claire, had a goal of increasing sales in her territory by 10% in the first quarter of the year. However, when you review Claire’s sales numbers, you see that her sales have actually gone down. You know that Claire is a hard worker and has an excellent relationship with her customers. When you sit down to discuss Claire’s performance, she explains what is happening in her territory. A major automobile manufacturer, which once provided 25,000 jobs, has moved its operations to Mexico and laid off all 25,000 of its workers. As a result, the economy in Claire’s region has become quite depressed, and the residents are living off their savings. Taking the steps of the planning/control cycle into account, what is your best course of action with Claire? (((Recognize the economic reality of Claire’s territory and change her goals to make them ambitious but realistic.)))
Donna’s Restaurant is a popular café that specializes in home-cooked meals, friendly service, and a menu that contains vegan and vegetarian dishes, (menu items that no other restaurant in the area offers). Donna’s Restaurant is engaging in strategic positioning by offering the unique menu items of vegan and vegetarian dishes. (((TRUE)))
Frank is interested in rewriting the vision statement for his antique shops, and he wants his employees and his business to grow. Therefore, the vision for Frank’s stores should be positive and inspiring, and it should stretch the company and his employees to achieve objectives that they believe are not possible. (((TRUE)))
The vision statement is an organization’s purpose or reason for being, and a company’s mission statement is its long-term goal of what it wants to become. (((FALSE)))
Competitive intelligence means gaining information about one’s competitors’ activities so that you can anticipate their moves and react appropriately. For example, managers gain competitive intelligence by reading business publications, in order to understand competitors’ business plans, goals, and strategies. (((TRUE)))
The managers of a small bakery, Butter & Batter, decided to do a SWOT analysis to study the strengths and weaknesses within the organization and the external environment. A key benefit of conducting a SWOT analysis is being able to better formulate strategies in pursuit of the firm’s mission. (((TRUE)))
Organizational threats are an environmental factor that can hinder an organization’s ability to achieve a competitive advantage. For example, managers may realize that the government is about to place restrictions on exports, which will limit the company’s ability to sell its products overseas (which will in turn greatly reduce its profits). (((TRUE)))
Instructor Services is a technology company that offers many IT services in highly populated southern Ohio. The company’s services and products include computer training, support, monitoring, repair, network design, virus removal, and software upgrades. It even sells refurbished computers. The source of Instructor Services’ strategic position is (((broad needs and many customers.)))
With small businesses in very competitive industries, small differences in performance may affect that company’s survival. In this case, it is worth the effort for the company’s managers to implement (((strategic planning.)))
A local karate studio offers many services: karate lessons to young children (under age 10), judo lessons to preteens, kickboxing courses for teenagers and adults, and self-defense courses for older people. We can say that the studio has achieved _____ because the studio’s activities interact and reinforce one another. (((fit)))
The second step in the strategic-management process is a(n) ________, where managers look at where the organization stands, and then determine what is working and what could be different to maximize efficiency and effectiveness in achieving the organization’s mission. (((current reality assessment)))
An organization’s vision statement needs to describe (((An organization’s vision statement needs to describe)))
Over the course of 40 years, Sal grew his company to six package shipping stores. With his retirement approaching and the increased competition, he decided to reduce the number of locations to two. Sal’s reduction of effort represents a (((defensive strategy.)))
Strategy formulation is the process of (((choosing among different strategies and altering them to best fit the organization’s needs.)))
George, a chef and owner of L’Auberge, a popular restaurant, is always visiting his competitors to observe how they are doing things in their restaurants. He told one of his managers, "I eat dinner at a lot of restaurants because I want to know what is going on. I am always concerned that one of our competitors will surprise us with a new service or menu item, like ours but better." In which activity is George engaging? (((competitive intelligence)))
Susana keeps a close eye on her company’s internal and external environment to discover possible opportunities for new products and to discern possible threats from the competition. In which activity is Susana engaged? (((environmental scanning)))
Many customers shop at Organic Foods because of the employees’ extensive product knowledge. In a SWOT analysis, the employees’ high levels of product knowledge are an example of the company’s (((strengths.)))
Don knows that one of the reasons people do not return to his electronics store is because of the slow service. How would a SWOT analysis classify the slow service at Don’s electronic store? (((as a weakness)))
Christopher’s Restaurant and Catering serves delicious vegetarian and vegan dishes. So when the local community became interested in eating a healthier diet, Christopher’s benefited. In a SWOT analysis, the changing community attitudes are an example of a(n) _____ for Christopher’s Restaurant. (((opportunity)))
The owners of Smith’s Yard Mart, a family-owned garden center in a rural community, are concerned over the news that a large retailer with a garden center is building a new store on the other side of town. According to a SWOT analysis, this new retailer is a(n) ____ to Smith’s. (((threat)))
Sales this year at Donna’s Pawn Shop have been high, and based on several factors, Donna projects next year’s sales to also be good. However, even with her forecast of continued strong sales, Donna and her business partner need to develop a plan in case sales drop unexpectedly. ________ is the type of planning for alternative future conditions. (((Contingency planning)))
Michael Porter proposed that business-level strategies originate with the primary competitive forces in the firm’s environment; these forces are (((the threats of new entrants and substitute products or services, the bargaining power of suppliers and buyers, and competitive rivalries.)))
Tom and his family have developed a successful business selling a liquid spray fertilizer to farmers. The fertilizer consists of rich, organic, composted material. Recently NuBreed Seed, a national seed company, has been marketing a powdered chemical fertilizer to its customers; NuBreed’s fertilizer is less expensive than Tom’s product. NuBreed’s efforts are an example of the ___ in Porter’s model for industry analysis. (((threats of substitute products and services)))
Porter’s competitive strategies of cost-leadership and differentiation focus on ____ markets, while the cost-focus and focused-differentiation strategies focus on ____ markets. (((wide; narrow)))
A company’s overall ability to execute is a function of effectively executing according to three processes, but Bossidy and Charan believe that the _____ process is the most important. (((people)))
Which of the following questions is NOT useful for evaluating a company’s values statement? (((Does it set standards of excellence and reflect high ideals?)))
Tom is in the marketing department with HomeMade Soups. He was given the responsibility of determining if customers liked the new spicy chicken noodle soup. According to HomeMade Soups’ sales data, over 4,000 customers purchased the new soup last month. If Tom surveys 25 of those customers, he has selected an appropriate sample size for decision making. (((FALSE)))
Luis, the sales manager of IT Sales, mentioned to his assistant, Marla, "I just got off the phone with one of our best customers and he told me about quite a few customer service issues, and I’m glad he took the time to call. You know, most people will not call a company when they have a problem; they just tell friends, family, and coworkers." This statement about who customers call when complaining about faulty customer service is accurate. (((TRUE)))
Last year, Eduardo’s home of 50 years, which contained so many family, school, and career memories, was destroyed in a fire. It is normal for this event to have a negative impact on his life and decision-making abilities for at least three years (((FALSE)))
George, a longtime hotel manager for Holiday Inn, was meeting with Sally, a new front-desk manager, and telling her about his recent decision not to fire a front desk employee about whom several guests had complained. George discussed the four stages of the decision process. He said, "I first identified the problem, and then I gathered alternative solutions. I evaluated each choice and selected a solution. I then implemented my decision to not fire the employee but instead to provide him with additional training. Finally, I followed up with an evaluation of the solution I’d chosen." George is using the ______ process. (((rational decision-making model)))
The model of decision making that explains how managers should make decisions, assuming managers will make logical decisions that will be the optimum in furthering the organization’s best interests, is known as the ____. For example, a manager who uses this model may be personally opposed to outsourcing jobs overseas, but she nonetheless decides to outsource customer-service operations to India because doing so is in the company’s best interests. (((rational decision-making model)))
Despite your best intentions, it’s hard to bring the best evidence to bear on your decisions. Why? (((There’s too much evidence; and there’s not enough good evidence and the side effects outweigh the cure.)))
ABC Medical Technology, a fast-growing global research firm, has accumulated so much client information that conventional database management systems cannot handle its customers’ needs. Therefore, ABC purchased very sophisticated analysis software and supercomputing-level hardware. For the company’s work with hospitals and university medical schools, ABC has been noted in The Wall Street Journal as a "real pioneer in innovation and productivity." ABC Medical Technology is tapping the power of (((Big Data.)))
A person’s ________ reflects the combination of how he or she perceives and responds to information. (((decision-making style)))
Marie is a small-business owner who loves to take risks. She also enjoys going out and meeting customers and potential clients—the social aspect of her job. Marie relies on intuition and discussions with others to acquire information. As an interior decorator, Marie utilizes her imaginative strengths, taking a broad perspective to problem solving, and she likes to consider many options and future possibilities. Marie’s decision-making style is best described as (((conceptual.)))
Sam was recently promoted to the production manager of a busy shoe factory. He was chosen because prior to his promotion he had proven himself to be very efficient, and as a department manager he had a logical and practical approach to solving problems. Sam has a reputation for being decisive and getting things done. Most of the factory’s employees like Sam, but they feel his style is autocratic, because he likes to do things by the book. Sam has a(n) ______ style. (((directive)))
Scott has a bachelor’s degree in ethics and human resources, and he has extensive experience working with employees and managers regarding ethics, especially in the area of ethical dilemmas. In his current position he has assisted extensively in determining if the company’s decisions are both ethical and lawful. Which position in the company does Scott’s background make him ideally suited for? (((ethics officer)))
Is the proposed action legal? If yes, does the proposed action maximize shareholder value? If yes, is the proposed action ethical? If no, would it be ethical to take the proposed action? These four questions, which managers of all organizations should ask when confronted with a decision on an action, form the basis of (((Bagley’s ethical decision tree)))
Patty, the CEO of an oil drilling company, and her top management team recently discovered that their facilities are damaging an Asian beach and the local wildlife. They understand that they need to decide if temporarily closing or not closing the facility is unethical. To help them make their decision, they consider the following four questions: "Is not temporarily closing the facility legal? If yes, does this proposed action maximize shareholder value? If yes, is not temporarily closing the facility ethical? If no, would it be ethical to take the proposed action?" Patty and her top managers are using ____ to help them make their decision. (((Bagley’s ethical decision tree)))
Dwight, the general manager of a hotel, knows that one of his housekeeping employees has a serious substance-abuse issue. Dwight knows that because of the seriousness of the issue he needs to talk with the employee. However, his assistant manager asks Dwight if he would like her to deal with it. Dwight knows this is not the best option, but he agrees, knowing it is the easy way out. Dwight’s decision is an example of (((relaxed change.)))
With defensive avoidance, a manager "acts" by _______ when he or she can’t find a good solution to a problem (((procrastinating, passing the buck, or denying the risk of any negative consequences)))
Gregory was talking with Kareem, his assistant manager, saying, "When I make a decision on which employees will do a project, I use three simple factors, which I call my decision-making ’rules of thumb.’ I consider their attitude and knowledge, and how hard they work." Gregory’s rules of thumb, _________, is what he uses when making a decision. (((heuristics)))
Phil, the store manager, recently had to fire Sue, an employee who graduated from North College. He felt that Sue was not well trained at her school. So when Steven, also a graduate from North College, applied for Sue’s old position, he did not want to consider hiring him, feeling Steven also probably did not receive the proper training. Phil was affected by _____ bias. (((representativeness)))
____ occurs when people’s subjective confidence in their decision making is greater than their objective accuracy. For example, Sara thinks she is a superb writer and editor, but her reports are always filled with typos, grammatical errors, and sentence fragments. (((Overconfidence bias)))
Peter, the owner of a local real estate agency, is even more committed to making the used copier, which he recently purchased, work even though it keeps jamming, and his employees have been telling him about other problems they have been having with the copier. The ______ affects Peter’s decision making. (((escalation of commitment bias)))
George, Donna, and Maria are on the five-member Benefits Task Force that is researching options for the new benefits package. Over lunch, George suggests that going with the Members’ Choice HMO is the best course of action, and Donna and Maria agree with George for the sake of unanimity, without ever researching and accurately assessing the decision. George, Donna, and Maria are engaged in (((groupthink.)))
The Safety Committee at Office Supply Mart was formed to help make the store a safer place, and some of the committee’s suggestions are going to be costly. During the committee meeting, the vice president of accounting announced that company profits were much lower than expected, and she suggested that profits are more important than safety. The profit goal is now outweighing the committee’s safety goal, which is an example of (((goal displacement.)))
How do you know that a group in which you’re taking part is suffering from groupthink? (((Peer pressure makes you agree with the ideas of the other people in your group.)))
A company’s organizational structure is its formal and informal marketing systems of goods, services, ideas, and customer relationships. (((FALSE)))
Changing organizational culture is essentially a teaching process, a process in which members instruct each other about the organization’s preferred values, beliefs, expectations, and behaviors. (((TRUE)))
When Angela, a new nurse was hired at Mercy Hospital, the HR manager gave her a handout containing a chart that showed a family-tree-like pattern of boxes and lines. This document, which shows the formal lines of authority and the organization’s official positions and work specializations, at the hospital is called an organization chart. (((TRUE)))
Tom’s Hardware has a simple organizational structure. There are no levels of middle management, because all employees report to the owner, Tom, or to his assistant managers. Tom’s Hardware has a thin organizational structure. (((FALSE)))
Two advantages of decentralization are that (1) managers are encouraged to solve their own problems, and (2) decisions are made more quickly, increasing the organization’s flexibility and efficiency (((TRUE)))
While figuring out how to save money for a bride who is having financial problems, Paula, the catering manager of Oh Happy Day, asked her assistant, "What do you think is the best menu to offer under these particular circumstances? How can we offer the bride and groom’s guests a great meal within their budget?" Paula and her assistant are using the contingency approach. (((TRUE)))
Adam, the owner of Adam’s Roofing, was talking to a visitor in his office, telling her that "We are a fun-loving group that believes in teamwork and a family atmosphere at work, which significantly affects our work outcomes. Plus, we do things together outside of work. This is the ’social glue’ that binds the members of our company together." Adam was referring to his company’s (((organizational culture.)))
Jean-Paul, the store manager at an H&M outlet, was speaking with Patty, his new assistant manager, about the store’s system of job relationships; he also explained whom she will report to and who will report to her. Jean-Paul said, "This structure is one of the things that motivates our employees to work together to achieve our company’s goals." Jean-Paul was telling Patty about the store’s (((organizational structure.)))
. Online retailer Amazon relies heavily on a hierarchy culture to manage its vast and complex shipping processes. ___________ are part of Amazon’s hierarchy culture. (((An internal focus and values stability and control)))
Employees and customers are treated like a family at Bayou Seafood Café. Paul and Betsey, the owners, work to encourage unity to increase their employees’ job satisfaction and commitment. At their café, Paul and Betsey have built a(n) (((clan culture.)))
Elly and Sylvia, owners of Gaia Organic Foods, do not have a formal policy about dress code and work procedures because they see their company as a family place that is enjoyable to work for. Elly and Sylvia believe that if you treat employees like family, the company will do well and grow. _____ are the core beliefs that represent the core values of Gaia Organic Foods’ culture. (((Basic assumptions)))
IKEA employees are inspired to work hard by an anecdote from their Swedish founder, Invar Kamprad, who told how he was berated by his father for failing repeatedly to get out of bed to milk the cows on his family’s farm. Then one day he got an alarm clock. "‘Now by jiminy, I’m going to start a new life,’ he determined, setting the alarm for twenty to six and removing the ‘off button.’" Invar Kamprad is an example of a(n) (((hero.)))
Rites and rituals are ____. For example, employees of New Belgium Brewery in Fort Collins, Colorado, get a free brewery-hopping trip to Belgium after being employed by the company for five years. (((the activities and ceremonies that celebrate important occasions and accomplishments in the organization’s life)))
Danilo has been working for Metropolitan Taxi for three months. He expected that his job would be to drive around the city, pick up fares, and drop them off at their desired locations, but in his first few days on the job he realized that the job entails much more. He must keep his taxi cab clean and neat, and some days he will be stationed at the shop and go to people’s houses to pick them up and bring them to the local airport. Within a few months, he has learned that taxi drivers often express frustration with the people they call "civilians"—that is, non–taxi drivers, whose driving skills they complain about incessantly. In his first few months on the job, ________ describes what Danilo is undergoing. (((organizational socialization)))
Della, the new CEO of Sky Advertising, has been with the firm for over 25 years. She was picked by the board to turn the 85-year-old agency around, because it had lost its edge in the Internet age. To infuse new life and energy into the agency, Della wants to bring back some old ideas that previously worked at Sky. She plans on having managers and veteran employees instruct each other about the organization’s values, beliefs, and expectations; telling stories about some of the company legendary ad campaigns; coming up with a slogan that summarizes Sky’s abilities in a simple and memorable phrase; and having quarterly ceremonies where creativity and innovation are rewarded. The things that Della wants to do are all examples of (((embedding culture.)))
According to Chester I. Barnard’s classic definition, an organization is a (((system of consciously coordinated activities or forces of two or more people)))
_______ are voluntary collectives with the purpose of advancing their members’ interests. One example of such a collective is the Teamsters Union. (((Mutual-benefit organizations)))
Alana, the children’s department manager at Shoe Mart, has eight employees in her department, also known as her _________. All of eight of these employees report directly to her. (((span of control)))
Jim was recently promoted to the position of dining room supervisor at The Crab Shack, and he is now responsible for making decisions on numerous matters in the dining room and giving orders to the serving staff. At the same time, Jim has the obligation to perform the many new tasks assigned to him as a supervisor, such as overseeing the closing work and the cleaning of the dining room. Jim’s obligations are known as his (((responsibility.)))
Patricia, the manager of Prime Health Club, was telling her new trainer that "I have many decisions to make in a day, and as the club manager, during the course of a week, I have to give orders to most of the 35 employees. Then there is my budget and the club resources, which I oversee." Patricia’s _______ gives her the right to make decisions, give orders, and utilize resources. (((authority)))
Artem, the president of Instructor Services, told one of his new hires that "Managers at this company are encouraged to solve their own problems rather than buck the decision to a higher level, and decisions are made more quickly, which increases our organization’s flexibility and efficiency." The key benefits of _______ are the advantages that Artem related to his new hires. (((decentralized authority)))
After being referred by a friend, Hasina attended a job interview. She didn’t feel it went well, but she was surprised when her friend told her the boss didn’t like that Hasina had worn her hijab, a traditional headscarf worn by Muslim women. The boss told a coworker after the interview, "Our customers prefer working with Christians." Hasina is a victim of (((discrimination.)))
________ entails job posting, which means placing information about job vacancies and qualifications in places where employees can see them such as on bulletin boards or the company’s intranet. (((Internal recruiting)))
Sally, Greg, Juan, and Amar are working on a project for a customer that is aimed at cutting the client’s electrical costs. The four members of this workgroup are located throughout the Midwest, and they are utilizing the phone, e-mail, and collaborative computing to complete this project. This workgroup is an example of a (((virtual organization.)))
Peter, the owner of Happy Burger, is involved in the company’s daily decisions and has established procedures for every task. The company rules are clearly specified in the handbook, and Peter also believes in close supervision of all employees. Happy Burger is a(n) _______ organization. (((mechanistic)))
Which of the following is a good suggestion for managers when conducting interviews? (((After the interview, write a short report with a quantitative score of the candidate’s qualifications.)))
According to Lawrence and Lorsch, the stability of an organization’s environment determines the degree of (((differentiation or integration that is appropriate.)))
Tiffany is an architect for the building of a large office complex downtown. Frequently a group of steel workers whistle at her as she climbs a ladder or interacts with staff. The men’s actions create a hostile work environment. (((TRUE)))
As a sales manager, Danyce is conducting performance appraisals for her team members. To do so, she tracks each person’s sales calls, amount of sales, and revenues on a quarterly basis. Danyce is using a(n) ______ system of appraisal. (((objective)))
Explaining the organization’s mission and operations, the job routine and the organization’s work rules and benefits should be an integral part of employee orientation. (((TRUE)))
Every year, 10% of GE’s managers are assigned the bottom grade during the annual review cycle, and if they don’t improve, they are asked to leave the company. GE uses a forced ranking performance review system. (((TRUE)))
Dora is conducting a performance appraisal for Sue, one of her employees. The company’s performance appraisal form asks her to rate Sue’s performance on various items like "Submits reports on time with minimal errors" on a scale from 1 to 5. Dora’s is using a ______ appraisal system. (((behaviorally anchored rating scale (BARS))))
Cody was surprised to learn that not everyone would receive a bonus this year. Instead, management planned to rank all of the employees in Cody’s division and award bonuses only to the top 20% in terms of sales. Cody’s company is using a(n) ______ performance review system. (((forced ranking)))
________________ is the purpose of the strategic human resource process. (((Getting optimal work performance to help realize the company’s mission and vision)))
In considering the staffing an organization might need in the future, human resource managers should (((understand the organization’s vision and strategy in order to hire personnel to support them.)))
A manager must consider fairness, __________, and others’ resentments in considering a promotion of an employee (((nondiscrimination)))
A(n) ___________ is a report listing your organization’s employees by name, education, training, languages, and other important information. (((human resource inventory)))
To ________, Ricco, an HR manager, used reports from the U.S. Bureau of Labor Statistics and the U.S. Census Bureau for information about his industry’s labor pool in his geographic area. (((recruit from outside the organization)))
_______ are organizations of employees formed to protect and advance their members’ interests by bargaining with management over job-related issues. (((Labor unions)))
A California software firm, MacroSoft, was growing rapidly and hiring frequently, but it primarily focused on campus recruiting at two California universities, UC Berkeley and Stanford. When one 45-year-old candidate did not get an interview after a phone screening, he went to the company’s website and noted all of the young people in charge. Which law allows him to file a discrimination lawsuit against MacroSoft? (((Age Discrimination in Employment)))
Bethany had a starring role in a film that was shot in her hometown of Baton Rouge, Louisiana. Though the Screen Actors Guild, an actors’ union, exists in Louisiana, Bethany was not required to join it because of the state’s (((right-to-work)))
Typically the ________ clause in a union contract is tied to the U.S. Bureau of Labor Statistics’ consumer price index (CPI). (((cost-of-living adjustment (COLA))))
Walt Disney Company’s investment of $1 billion into a wearable technology that it hopes will revolutionize the way visitors spend money at Walt Disney World, featuring a website ("My Disney Experience") and data-collecting wristbands ("MagicBands") that interact with scanners throughout the 40-square-mile theme park, is an example of proactive change. (((TRUE)))
During annual inventory week, a department store may ask its employees to work 12 hours a day instead of the usual 8. During tax-preparation time, the store’s accounting department may work similar hours. Although accounting employees are in a different department from stockroom and sales employees, it’s reasonable to expect that the accounting employees wouldn’t be terribly upset by the temporary change in hours because they’ve seen it in effect elsewhere in the store. This is an example of innovative change. (((FALSE)))
Procter & Gamble modified a liquid detergent to make it available as a concentrated powder in a pouch. P&G’s new product is an example of a core innovation. (((TRUE)))
Keurig created a new category of coffee/tea preparation by offering one-cup-at-a-time pod-style brewing with its "K-cups." Keurig’s product is an example of a transformational innovation. (((TRUE)))
________ change is made in response to arising problems or opportunities. BP’s response to an explosion on a drilling platform in the Gulf of Mexico is an example of this kind of change. (((reactive)))
Walmart’s implementation of RFID, a type of technology that allows it to improve inventory tracking, is an example of a(n) ______ change because Walmart already had a very effective technology tracking system in place before it implemented RFID. (((proactive)))
Firms such as General Motors and McDonald’s actively collect information about customer preferences and try to address them in their new products. General Motors and McDonald’s are actively trying to adapt to (((shareholder, customer, and market changes)))
Joan, a new manager, must enforce sales report deadlines, but her team is struggling. She creates a new system to streamline the process and helps everyone understand why the deadlines are important. On which of the following internal forces is Joan trying to have an impact? (((manager’s behavior)))
Studies exploring nurses’ perceptions about using PDAs in their daily patient practice found initial resistance, with some nurses concerned about the cost and short technological life cycle of these devices. As part of the __________ stage of change, the nurses’ managers try to instill in them the motivation to change, encouraging them to let go of attitudes and behaviors that are resistant to innovation. (((unfreezing)))
Appliance America’s customer complaints have increased because its drivers often get lost when attempting deliveries, which means that the deliveries arrived late. The drivers have resisted using GPS systems, claiming they know their territories. But managers are now encouraging them to look for the best solution to the increased level of customer complaints. Appliance America is in the ________ stage of Lewin’s change model. (((unfreezing)))
The two cofounders of Network Appliance, a data-storage firm in Sunnyvale, California, were feuding with each other because one founder couldn’t stick to his decisions, which drove the other founder crazy. A(n) ____________ began working with the warring executives in separate sessions to solve the problem. (((organizational behavior specialist)))
As owner and senior accountant at a tax preparation firm, Mark is changing the work schedule of all employees, including managers and accountants, for the months of March and April, the company’s busiest time. He is requiring everyone to work on weekends, just as he has during the tax-preparation season for the last ten years. Mark should expect that his employees will be (((hardly upset or not upset at all.)))
Lewin’s change model consists of (((three stages: unfreezing, changing, and refreezing.)))
During Lewin’s changing stage, managers should (((give employees new information, perspectives, and models for behavior.)))
During Lewin’s refreezing stage, managers should (((encourage and reinforce the desired change in the employees.)))
After Mary’s Gifts and Cards surveyed its customers, it discovered that its customers preferred the company’s competition more than 70 percent of the time. The most frequently cited reason was customer service. During which stage of Lewin’s change model should Mary provide her employees with this information about the problems with customer service? (((unfreezing)))
When it comes to learning from failure in the innovation process, former Procter & Gamble CEO A. G. Lafley suggests that the key is to (((fail early, fail cheaply, and don’t make the same mistake twice)))
Which of the following factors most reduces an organization’s ability to learn from failure? (((employees who blame others for failure)))
Which of the following is one of Scott Berkun’s seeds of innovation for organizations? (((philanthropy)))
A technological innovation that improves the performance and speed of a computer’s operating system would be considered a(n) ______ innovation (((product)))
A managerial innovation that improves the efficiency of a company’s cross-functional teams would be considered a ______ innovation. (((process)))
Personal Care Products recently introduced a new acai berry shampoo. Rinn, director of new product development, has just reviewed the dismal results. As it appears that his team adequately researched the product’s acceptance by consumers prior to its introduction, he is confused by the outcome. But to keep his team open to innovation, Rinn should (((foster an innovative culture and climate that permits experimentation, risk taking, and failure)))
Which of the following is not a way to encourage and foster innovation? (((withholding raises and promotions when innovation attempts don’t work out)))
According to the systems model of change, which of the following is not an input? (((the organization’s products or services)))
A smoker who claims that the habit is not as dangerous as antismoking messages suggest, saying "My grandmother smokes and she’s in her 80s," is attempting to increase cognitive dissonance. (((FALSE)))
Your supervisor says the company needs you to stay late to meet an important deadline, but your family expects you to be present for your child’s birthday party. You are experiencing role overload. (((FALSE)))
Carolina’s manager notices that she exhibits an internal locus of control when she speaks about her work. Her manager should (((provide incentives such as merit pay or sales commissions.)))
Boring, tedious jobs generally reduce people’s perceptions of their (((self-efficacy.)))
If a manager knows one of her employees has low self-esteem and wishes to enhance it, she should (((reinforce the employee’s positive attributes and skills.)))
People high in ______ are responsive to others’ social and interpersonal cues. (((emotional intelligence)))
________ are abstract ideals that guide one’s thinking and behavior across all situations. (((Values)))
The statement "I really don’t like that Ivan got so angry in that meeting" reflects the ______ component of an attitude. (((affective)))
The statement "I won’t give Roberto such a tight deadline again" reflects the ______ component of an attitude. (((behavioral or intentional)))
_____________ is the process of interpreting and understanding one’s environment. (((Perception)))
____ is the tendency to attribute another person’s behavior to his or her personal characteristics rather than to the situation the person is in. (((Fundamental attribution bias)))
Because she beat her goal this year by nearly 30 percent, Lori has been telling everyone in the sales department of her incredible skill as a salesperson. But last year when she didn’t even reach her goal, she said her failure was due to the poor economy. Lori provides an example of (((self-serving bias.)))
Carol really doesn’t like her new boss and is not happy with the new tasks she’s been assigned and the long hours she’s been working. Still, she truly believes in what the company is trying to accomplish. Carol has (((low job satisfaction.)))
A person’s overall satisfaction with work depends on how he or she feels about several components. Which of the following is not one of these components? (((life outside of work)))
The Americans with Disabilities Act requires employers to (((reasonably accommodate an individual’s disability.)))
Jake has a master’s degree in psychology, but he is working at a sunglasses cart in the mall. He is currently experiencing (((underemployment.)))
______ is the belief that one’s native country, culture, language, abilities, or behavior is superior to those of another culture. (((Ethnocentrism)))
When others’ expectations exceed one’s ability, ______ occurs. (((role overload)))
Motivation cannot be directly observed in another’s behavior; it must be inferred from one’s behavior. (((TRUE)))
To motivate employees, employers need to primarily focus on fulfilling the top level of Maslow’s hierarchy of needs, self-actualization. (((FALSE)))
John’s manager just told his team about this year’s contest, the winner of which will receive an all-expense paid trip to Bangkok. John does not like to travel to places where he can’t speak the language, so he is not very enthusiastic. This reward has a high valence for John. (((FALSE)))
Ricardo’s sales manager just informed his sales team that all sales in January will earn an extra 5 percent commission. The team gets right to work, being motivated by a(n) (((extrinsic reward)))
Paula’s management professor just told her class that the final exam is optional for students like Paula who currently have an A. Paula decides to take the exam anyway because she likes the subject and wants to master the material. Paula is motivated to take the exam by a(n) (((intrinsic reward.)))
Maslow’s levels of needs, in order from lowest (most basic) to highest level, are (((physiological, safety, love, esteem, and self-actualization.)))
After three shuttle accidents in three months, one of which resulted in a critical injury to a driver, Citywide Shuttle drivers received additional training, and the company retrofitted all shuttles with new braking systems. Which of Maslow’s needs are addressed by Citywide Shuttle’s actions? (((safety)))
An accounting firm reimburses employees for tuition and fees if they complete job-related coursework with a B or better. The accounting firm’s tuition-reimbursement policy helps its employees meet their _______ needs. (((self-actualization)))
For managers, the importance of Maslow’s contribution is that he showed that workers (((have needs beyond that of just earning a paycheck.)))
Using self-determination theory to motivate employees at Cloud9, a data storage company, the vice chair makes 10–20 phone calls a day to thank special employees "caught doing something right." The vice chair is fulfilling her employees’ ______ needs. (((competence)))
Even when he started his first job, Will was not content to be just one of the employees. His boss noticed that he often coached his coworkers about ways to improve their work, even when it wasn’t his job. Will probably has a (((high need for power)))
McClelland’s need for achievement corresponds most closely to (((McClelland’s need for achievement corresponds most closely to)))
According to Herzberg’s two-factor theory, in the zone between the motivating factors and the hygiene factors, employees are (((neither satisfied nor dissatisfied.)))
According to Herzberg’s two-factor theory, only ______ factors can make employees satisfied with their jobs. (((motivating)))
According to Herzberg’s theory, the first thing managers of employees who dislike their jobs should do is to (((make sure pay levels, policies, and working conditions are reasonable.)))
Randy complained to his boss, Maryann, that he received the same bonus this quarter as everyone else, despite the longer hours he had worked and his higher level of experience, production, and efficiency. If Maryann can’t change the bonus, she should expect that Randy might respond in any of the following ways (((ignoring his feeling of resentment and trying harder in next quarter.)))
At his review last year, Bryan was promised a 20 percent raise if he met his production goals. Raises were included in today’s paychecks, and although Bryan has met all of his goals, he received only a cost-of-living raise. In the future, Bryan’s ______ will probably be (((instrumentality; low)))
______ involves division of an organization’s work among its employees and applies motivational theories to jobs to increase satisfaction and performance. (((Job design)))
Michele’s job as an accounting assistant was recently modified to include reconciling bank accounts and making deposits, two tasks previously done by the accounting manager. This increase in responsibility would best be described as (((job enrichment.)))
A technician who is responsible for keeping an airport’s control tower’s electronic equipment in working order has higher ____________ than a person wiping down cars in a carwash. (((task significance)))
As a maintenance person for the air force, Alex services aircraft engines, which protects the lives and safety of military personnel and increases their ability to conduct missions. Alex’s job has a high level of (((task significance.)))
A supervisor told a salesperson who had not made any calls to clients and therefore did not make quota, "Well, if this continues in the next 30 days, you’ll probably be let go." The supervisor provides an example of _____ by presenting the likely negative outcome to the salesperson. (((punishment)))
Ted’s manager required him to give up his company car because he had missed sales goals for four consecutive quarters. Ted’s manager used (((punishment.)))
Nancy’s employer distributes checks at the end of each quarter, representing an equitable portion of 5 percent of the company’s pretax profits for the previous period. These checks represent (((profit sharing.)))
Daniel teaches sixth grade at a local elementary school. He significantly increased his salary by earning a master’s degree in education. Daniel’s increased salary is an example of (((pay for knowledge)))
During the storming stage of Tuckman’s five-stage model, the leader of a team should allow members the empowerment they need to concentrate on solving problems and completing the assigned task. (((FALSE)))
The best way to manage virtual teams is to focus on what’s accomplished, not whether an employee is working from her patio or at 10 p.m. (((TRUE)))
Devil’s advocacy is the process of assigning someone to play the role of critic to voice possible objections to a proposal. (((TRUE)))
Molly organized several teachers to discuss the school’s interior painting scheduled for the summer. They looked at several brands and heard a presentation by a designer who then helped them choose a color palette to recommend to school administrators. In this instance, the teachers made up a(n) (((formal group.)))
The group development stage in which individuals test the leader’s policies and assumptions as they try to determine how they fit into the power structure is the ______ stage. (((storming)))
Depak’s team was not making much progress on defining a new production process. Nicole was being very uncooperative. She did not agree with Depak’s direction, and she has not completed her tasks. This team is in what stage of group development? (((storming)))
Because hard feelings about group leadership and assignments have passed, the members of Robin’s group now seem to be relating much better. At the meeting tomorrow, she should take advantage of this moment by (((helping the team identify group goals and values.)))
Levi led a team that has just finished up a very challenging research project that will assist management in developing long-range plans. Despite the stress of the past few months, most participants seem sad it’s over. Now Levi should (((have an awards ceremony.)))
________ is defined as reciprocal faith in others’ intentions and behaviors (((Trust)))
A(n) ______ role is behavior that concentrates on getting the team’s work done. For example, one team member’s role may be to generate daily sales information at the end of each working day. (((task)))
Someone at a team meeting who says, "Let’s hear from those who oppose this plan," is performing a ______ role. (((maintenance)))
Tension was apparent as the management team discussed changes to promotion requirements. But then Chris made a joke about Jake’s white-knuckled grip on his pen, and the laughter seemed to lighten the mood. Chris was acting in a ______ role. (((maintenance)))
The Nordstrom department store chain emphasizes the great lengths to which it goes in customer service. By enforcing its norms for exceptional customer service, Nordstrom is seeking (((to emphasize the organization’s important values and identity.)))
Paige missed another deadline and her boss, Linda, is very upset. She will have to explain to the client again why the project is behind. Linda thinks she may say something she’ll regret if she talks to Paige about this now, so she decides to wait a while. Linda is using the ______ conflict-handling style. (((avoiding)))
______ is designed to elicit different opinions without inciting people’s personal feelings. (((Programmed conflict)))
Rebecca asked Gavin, one of her team members, to purposefully think of and voice criticisms as the group discussed a popular idea to open a branch office in another state. This is an example of the use of (((devil’s advocacy.)))
When Men’s Wearhouse fired a salesperson who wasn’t sharing walk-in customer traffic, and total clothing sales volume among all salespeople increased significantly, the company (((reduced destructive internal competition.)))
One’s power comes as part of one’s job; it is the right to perform or command. (((FALSE)))
Reward power results from managers’ authority to punish their employees. (((FALSE)))
______ power results from one’s specialized knowledge. (((Expert)))
______ power derives from one’s personal attraction. (((Referent)))
Lamar brought his proposal to Lindsay before the meeting saying, "I’m sure you have some ideas on this, and we could try to get them incorporated upfront." Which influence tactic is Lamar using? (((consultation)))
When Nelson saw Betty in the stockroom stuffing her purse with expensive printer cartridges, Betty was quick to say, "We’ve been friends since first grade, so I’m sure you won’t say anything about this." Betty was using a(n) _____ on Nelson. (((personal appeal)))
Regina apologized to Clay for an e-mail that upset him. She said she had chosen an inconsiderate way of stating her idea, and that she’d be happy to discuss it further. Regina is (((expressing consideration behavior.)))
Fiedler’s contingency leadership model determines if a leader’s style is (((task-oriented or relationship-oriented.)))
____ is the model that requires a manager to assess her own leadership orientation and the level of her situational control. (((Fiedler’s contingency model)))
_____ is the questionnaire used in Fiedler’s model to determine leadership orientation. (((Least preferred coworker (LPC) scale)))
In Fiedler’s contingency leadership model, ________ is the amount of influence a leader has in his or her immediate work environment. (((situational control)))
David’s peers were surprised when he received a promotion and suddenly became their supervisor. They thought he was rather unreliable and weren’t sure he was up to the task. According to the contingency model, David has (((poor leader-member relations.)))
Jane leads a task force developing specifications for a new customer database to be used by several departments. Jane is an IT supervisor, but most of the other task force members are directors of other departments. At the first meeting, a few of them asked questions that she couldn’t answer. According to the contingency model, Jane’s situational control is likely (((low.)))
Marcel supervises a group of paralegals serving the firm’s lawyers. He gets along well with his employees, and he has created detailed procedures for all the types of legal document they encounter. Marcel hires and fires; he also gives work assignments, performance appraisals, and promotions. The optimal leadership style for Marcel is _____ according to the contingency model. (((task-oriented)))
According to Fiedler’s contingency model, ______ situation control favors a leader who is ______-oriented. (((moderate; relationship)))
____________ is the model in which an effective leader makes desirable rewards available, clarifies how employees can achieve objectives, and provides them support in doing so. (((House’s path-goal model)))
According to House’s revised path-goal theory, a leader’s style should vary depending on (((employee characteristics and environmental factors.)))
The leadership styles of ________ originally included supportive and achievement-oriented and two others, but was more recently revised to include a total of eight styles. (((House’s path-goal model)))
Judith supervises a call center department that receives stress-producing calls from unhappy customers. Turnover has increased by 33 percent over the last four months. Judith is understanding and patient with her staff, and tells them she knows what they’re going through since it wasn’t so long ago that she took those calls. According to revised path-goal theory, Judith is using a(n) ______ leadership style. (((supportive)))
Pat is something of a cheerleader around his team. He shares his vision and expresses his confidence in his team’s ability to achieve his vision. He is quick to compliment and acknowledge team members’ accomplishments, and he is enthusiastic about their successes. According to revised path-goal theory, Pat is using a(n) ______ leadership style. (((value-based)))
House’s revision of his theory puts more emphasis on the need for leaders to foster (((intrinsic motivation through empowerment.)))
Lynn had many spelling and grammar mistakes in her report, some of which made the content difficult to understand. Lynn’s report is an example of poor encoding. (((TRUE)))
The sun was shining directly into Matt’s eyes, so he looked away and missed the coach’s demonstration of a defensive technique. In this case, the sun acts as feedback in the communication process. (((FALSE)))
The danger of using a lean medium for nonroutine matters is that it results in information oversimplification; it doesn’t provide enough of the information the receiver needs and wants. (((TRUE)))
Jyl, a supervisor, told her employees, "The store will close Monday for inventory. All employees are expected to participate." Jyl’s communication to her employees is an example of upward communication (((FALSE)))
When the band came out on stage and the lead singer shouted, "Hello, Dallas!" the crowd roared with applause. The crowd’s response is an example of (((. feedback.)))
Which of the following is not an example of noise in the communication process in a typical classroom? (((a student answering a professor’s question)))
Which of the following is the most appropriate medium for a manager advising employees of a revision to the company’s bonus structure? (((face-to-face meeting)))
Roger was talking to a coworker in a meeting, and he wasn’t listening when his supervisor announced work assignments. This situation is an example of what type of barrier to communication? (((receiver barrier)))
Which of the following is a "don’t" when attempting to improve communication? (((Close your eyes.)))
Which of the following statements is true? (((Men tend to withdraw and isolate themselves when problem solving, whereas women seek out others for support and can interpret men’s withdrawing as lack of caring.)))
________ is a person’s characteristic speaking patterns, such as the use of pacing, pausing, questions, and stories. (((A linguistics style)))
Doug gave some people street directions, but because they only nodded their heads and didn’t repeat the directions back to him, he doesn’t really know whether the directions were understood. What type of barrier is present here? (((feedback)))
Which of the following is an example of empathetic communication? (((I understand your frustration; this deadline is intense, and we’re running on reduced staff.")))
Ryan is sitting in a seminar where a speaker is explaining the company’s retirement plan options. Ryan is only 24 years old, and he thinks retirement is so far away that he doesn’t think he needs to pay attention to the speaker. He starts texting his friends and updating his Facebook page; he even slumps in his chair. He has completely tuned out of the presentation. Ryan’s listening style in this case is best described as (((detached.)))
Maslow’s hierarchy of needs includes all EXCEPT which of the following?((cognition))
In Maslow’s hierarchy of needs, food, water and sleep are considered _____ motives.((physiological))
it is very important for the company to analyze the competition exerted on it by different actors. for this, the company uses: (( Porter’s five forces model ))
During a manager’s meeting, Paula, a district manager, discussed how to handle sensitive employee issues and indicated correctly that management is really just an art.((FALSE))
In Maslow’s hierarchy of needs, food, water and sleep are considered _____ motives. ((physiological))
Maslow’s hierarchy of needs includes all EXCEPT which of the following (( cognition ))
What is true of wholesalers? (((Wholesalers are extremely important because of the marketing activities they perform.)))
Many service providers are considered retailers because they (((provide their services directly to consumers)))
A merger occurs when (((two companies combine to form a new company)))
A(n) ________ is a partnership established for a specific project or for a limited time. (((joint venture)))
An entrepreneur has been primarily associated with the willingness to((take risks.))
What is one of the difficulties faced by small business owners?((worries about employee problems))
What is a difference between high technology businesses and other small businesses?((High technology businesses require greater capital and have higher initial startup costs thanother small businesses))
What is true of a capitalist economic system?((Prices of goods and services are determined by supply and demand))
The quantity of goods and services that consumers are willing to buy at different prices at a specific time is referred to as((demand.))
The quantity of products that businesses are willing to sell at different prices at a specific time is referred to as((supply.))
What is a defining characteristic of public corporations?((Their stock can be bought, sold, or traded by anyone))
Which business provides a service, but is neither owned by the government nor focuses on earning profits?((a nonprofit corporation))
What is a true statement about the board members of a corporation?((They have a duty of care and loyalty to oversee the management of the firm.))
Preferred stockholders of a corporation((have a claim to profits before other stockholders do.))
Reasons for using active listening is for stating your own views, disagreeing with other’s views, making a request, and refusing a request. ((False))
Choosing the appropriate channel to communicate helps businesses reach their target audience. ((( TRUE )))
Businesses need to build goodwill only with external clients and stakeholders. ((( FALSE )))
Brendan is giving a presentation to clients on a new line of products. To reduce the distance between himself and his audience, he should talk about the audience’s feelings toward the company and its products. ((( FALSE )))
To build goodwill with content, arrange the information to meet the audience’s needs rather than your own. ((( TRUE )))
While conveying bad news messages, being straightforward always damages the speaker’s credibility. ((( FALSE )))
Positive emphasis can be created by putting the negative information in the middle of the message and presenting it concisely. ((( TRUE )))
When writing to people you do not know well outside your organization, it is good to address them by their first names to build a professional relationship. ((( FALSE )))
When referring to people with disabilities or conditions, it is preferable to name the person first, rather than identify them by their condition. ((( TRUE )))
When you produce a business document with photographs, the completed documents should show exactly 50 percent men and 50 percent women. ((( FALSE )))
Phone calls are an appropriate platform for business communication when tone is an important part of the message. ((( TRUE )))
Which statement about goodwill in business communication is true? ((( Goodwill helps to create less staff turnover. )))
Marketers at Annie’s Organic Dairy decide to tweet about the fresh peach ice cream flavor available now for a limited time. In this example, Twitter represents what element of communication? ((( sender )))
What process refers to the computer-assisted examination of text to understand its meaning? ((( channel selection )))
Selecting an oral or visual communication channel makes it easier to ((( deliver complex data. )))
What doesyou-attitude primarily focus on? ((( emphasizing the audience's point of view )))
Revisions foryou-attitude often_________blank because the revision is more specific and has more information. ((( increase the length of sentences )))
Wordiness in communication generally refers to ((( having more words in a sentence than the meaning requires. )))
Quinn works as a customer service representative at Quality Autos. She writes a letter to a customer who has recently become a member of the lifetime service club. Which introductory statement should Quinn use to best illustrateyou-attitude? ((( You are now a lifetime service club member at Quality Autos! )))
Which statement about enhancing customer relationships through communication is true? ((( "I" suggests that you are concerned about personal issues. )))
While applying you-attitude, passive verbs should ideally be used while ((( reporting bad news or limitations. )))
Tony is the owner of a sporting goods store. He interviews 10 people for the post of store manager and eventually selects Anya. Tony emails Anya to let her know that she has been selected for the post. Which statement willbest convey this information throughyou-attitude? ((( You have been selected for the post of store manager. )))
Which statement best exemplifies the use of impersonal expressions? ((( Incentive plans for all employees have been canceled. )))
What strategy is suggested for organizing information that builds goodwill? ((( Use headings and lists so readers can find key points quickly. )))
Which communication strategy best demonstrates ayou-attitude for an international audience? ((( Include phases such as “we all believe.” )))
What is a suggested technique for creating positive emphasis in a message? ((( justifying the negative information in the message by giving a reason )))
Which examplebest illustrates a sentence with a hidden negative? ((( Please be patient; the service outage will be resolved as soon as possible. )))
Which statement about creating positive emphasis in a message is true? ((( Linking a negative element to a benefit is good only when the audience acknowledges it as a benefit. )))
While adding positive emphasis to a message, negatives should be placed in the _________blank to deemphasize the negative. ((( middle of the message )))
Jamal is a team leader at a direct mail advertising company. He was asked to write a progress report on a client’s recent campaign. Although Jamal’s team did not deliver all the mailers on time, he plans to communicate this to his boss using positive emphasis. Which statement should Jamal use to deliver this information with a positive tone? ((( We have completed more than half of the deliveries and will complete the rest by the end of the week. )))
What is a negative word with a negative connotation? ((( deny )))
Which example uses double negatives? ((( Never fail to reach your sales targets. )))
Which example contains a hidden negative? ((( We hope the products you ordered have been delivered to you. )))
What is an effective way to check for positive emphasis? ((( Recognize that it should not be so overdone that it has become insincere. )))
What was a result of the research conducted by Martin Seligman for the life insurance industry? ((( It showed that optimistic salespeople sold more insurance than pessimistic salespeople. )))
Emilee is a pharmaceutical sales rep. She receives an email from her boss stating, "All salespeople must send their sales report by the end of the day." Emilee is not happy about how this statement is phrased. The most likely reason for her unhappiness is that the way the statement is phrased is ((( impolite. )))
What is a desirable tone for business writing? ((( confident )))
Celia is the customer relationship manager at a designer clothing store. She is drafting a letter to Teresa Parker, a new client. Celia wants to inform the client that she will receive a 40 percent discount on purchases during her birthday month. Which salutation should Celia use to address this client? ((( Dear Ms. Parker )))
Which of the following examples illustrates the lowest level of politeness in a message? ((( Submit your campaign report by Friday at the latest. )))
Which statement about bias-free language is true? ((( It can help avoid lawsuits. )))
What is the most acceptable gender-neutral group term to refer to individuals with Central and Latin American backgrounds? ((( Latino )))
What is an example of a nonsexist job title? ((( flight attendant )))
Which job title contains gendered language? ((( fireman )))
In official communication, when addressing a woman you don't know well, it is better to ((( use "Ms." as the courtesy title. )))
Regarding nonsexist language, which statement about courtesy titles for women is true? ((( If a woman types her name in a certain way, use the title she designates to herself to address her. )))
When a person writing a message knows neither the name nor the gender of the reader, the message writer should ((( use a general group to which the reader belongs. )))
Raegan is writing an email to the hiring manager of a publishing company, explaining why she would make a good candidate for an open internship position. Raegan does not know the name or the gender of the manager. Which salutation is most appropriate for Raegan to use in her letter? ((( Dear Hiring Manager )))
What strategy is most effective when trying to eliminate gendered generic pronouns? ((( using plural nouns or pronouns )))
When talking about people with disabilities, use language that ((( names the person first. )))
To make language nonracist and nonageist, the best approach is to ((( refer to a group by the term it prefers. )))
Lee, the owner of a financial consulting firm, plans to announce several renovations to make the company offices more accommodating to employees with visual, hearing, mobility, or cognitive impairments. Which announcement made by Lee is bias-free? ((( We are installing new accommodations for all employees with disabilities. )))
When relying on electronic messaging platforms, what is important to keep in mind? ((( Informality is always preferred. )))
Phone calls are a good choice for business communication under what circumstances? ((( The matter is not time sensitive. )))
What technique is considered best practice for sending effective business emails? ((( Incorporate all caps, underlining and emoticons to increase readability. )))
Elaina is responsible for maintaining PartyTime Rental’s Facebook page. She reads a lengthy complaint from a customer who says her party was ruined because her order was delivered late and contained the wrong color table linens. Elaina plans to contact the delivery team and the set-up crew to find out what actually happened. Meanwhile, what action should Elaina take? ((( Respond by posting about the thousands of satisfied customers that PartyTime has served. )))
Twitter is likely a good channel for which business communication situation? ((( Sharing positive stories about company products in use. )))
The writing skills used most often by all types of workers include email and presentations with visuals, such as PowerPoint. ((( TRUE )))
Writing skills have decreased in importance in today’s business environment as most matters can be handled successfully with automated form letters. ((( FALSE )))
The costs of poor communication include lost goodwill. ((( TRUE )))
A document’s poor organization or writing style can make it difficult for the reader to understand the message. ((( TRUE )))
The best way to keep an audience involved in a presentation is to make them guess what the presenter wants to convey. ((( FALSE )))
Josh answers a customer phone call which involves a billing complaint. He then follows up by sending the customer a claims adjustment. This is an example of internal communication. ((( FALSE )))
An annual report prepared for stockholders may have more than one communication purpose. ((( TRUE )))
Larissa, a director of human resources, is writing an email to employees informing them about changes to the company benefits package. She will best engage her audience by emphasizing how the changes will improve the company’s profitability. ((( FALSE )))
Which form is an example of verbal communication? ((( tweets )))
Duncan, a web developer, recently gave a presentation on online discourse and its ethical implications. The presentation was a huge success, mainly because Duncan maintained the audience’s attention through interesting nonverbal communication. What is a form of nonverbal communication that Duncan might have used in his presentation? ((( graphics )))
Which statement about communication skills at entry-level jobs is true? ((( Communication skills are required for persuading others and adapting information to particular audiences. )))
Which type of skills consistently rank first among the qualities that employers look for in college graduates? ((( communication )))
According to the survey conducted by the National Commission on Writing, __________ is a writing responsibility that all employees have. ((( emailing )))
According to the survey conducted by the National Commission on Writing, __________ functions are the least likely to be outsourced by organizations. ((( communication )))
What does research indicate is a clear benefit to educated workers considered among the top in writing skills, compared to workers considered among the bottom in writing skills? ((( earn more on average )))
Which statement is true about writing skills required at work? ((( Almost every entry-level professional requires good communication skills. )))
What is an example of an internal audience for written communication at work? ((( subordinates )))
What is an example of an external audience for written communication at work? ((( a potential employee )))
Aziz works at Architectural Consulting in Boston. He recently gave a presentation on "The Principles of Sustainable Design" to the members of his team, the sales team, and the marketing team. He also showed the presentation to his superiors before making a few changes and presenting it to the firm’s customers. The external audience for Aziz’s presentation includes ((( the customers. )))
Fuyuko, a marketing planner for Glamour Time Cosmetics, needs to give a presentation to her superiors about the launch plan for a new line of organic lipsticks. Once she receives approval, she will share details of the launch with distributors, clients, and retailers. She will also then present the plan to the press to spread word about the new lipstick line. Who among the following would be included in Fuyuko’s internal audience? ((( superiors )))
Santos, a production manager at a small manufacturing company, has written a report for the organizational director on the types of equipment upgrades needed to accommodate a new line of products. Along with the report, Santos has also sent a memo explaining why he forwarded the document to his boss. The accompanying document is called a ((( transmittal. )))
Which document is used for performance appraisals, salaries, and hiring? ((( job description )))
Tracee, an analyst at a major print media company, is preparing a document on the problems faced by the industry during a recent period. In the report, she explains that increased competition from more agile digital competitors and declining advertising revenues pose major challenges for the future. Which document is Tracee preparing in this scenario? ((( quarterly report )))
Chloe, a communications manager at an insurance company, needs to evaluate her team members' work for the last financial year. She plans to prepare a report on the evaluation of objectives they helped achieve. In this scenario, Chloe is preparing a(n) __________, a type of __________ document. ((( performance appraisal; internal )))
A memo of congratulations is written to ((( employees who have won awards or been promoted. )))
A claims adjustment granting a customer request to be given credit for defective goods is a document written for what primary purpose? ((( to inform )))
Which statement is true regarding the purposes of organizational writing? ((( Most organizational messages have multiple purposes. )))
Which description is a characteristic of good writing? ((( It follows widely accepted practices. )))
Which statement about good business and administrative communication is true? ((( It represents messages that are free from errors in spelling and grammar. )))
Which statement about conventions in business writing is true? ((( They are widely accepted practices. )))
Which strategy is a key to using conventions effectively? ((( remembering that conventions always need to fit the current situation )))
Which strategy will improve business communication by saving the audience’s time? ((( using forecasting statements to organize a message )))
When you convey information in which the audience’s reaction will be neutral, this is considered a(n) ((( informative message. )))
Bradley just accepted a new job at a competing firm, which pays much better than his current sales position at a small family-owned business. He composes a formal letter of resignation, which he plans to give to the company owner tomorrow morning. In this scenario, the company owner represents what type of audience? ((( primary )))
Which message recipient has the power to stop your message and control whether it reaches the intended audience? ((( gatekeeper )))
What is a good strategy for communicating in writing with multiple types of audiences? ((( Keep the core message consistent. )))
Melanie must contact all customers who have purchased the Happy Time Baby Swing. The recent model has a defective support bracket, which requires immediate replacement to ensure safe usage. What strategy is most appropriate for Melanie in analyzing this rhetorical situation? ((( Analyze her audience as members of a group. )))
What is an example of a demographic audience characteristic? ((( age )))
Which audience characteristic will likely be most important in messages designed to persuade? ((( values )))
Randall, the plant manager, must inform the plant’s 500 employees about pending layoffs. While he knows the audience will be hostile, Randall hopes to persuade them that this is the only way to prevent a total shutdown. In this situation, what is a suggested communication strategy for Randall? ((( Begin his message with areas of common ground shared with the audience. )))
As a business communicator, what information is least likely to be an important consideration in formulating your message? ((( the speaker’s psychographic profile )))
Which is one of the five questions that should be asked when analyzing business communication problems? ((( What information must your message include? )))
Which action should be followed to effectively analyze business communication problems? ((( ensuring that benefits are well adapted to the needs of the audience )))
During a department manager’s meeting, Devon proposes consolidating two existing departments into one. What strategy will be most effective in making his proposal more convincing to his audience? ((( Phrase benefits of his proposal in you-attitude. )))
What is an effective technique for overcoming potential obstacles in a persuasive message? ((( Show how the audience will benefit when the action is complete. )))
What action describes a way to ensure ethical business communication? ((( using design and graphics to make content understandable )))
An informative message has the primary purpose of de-emphasizing any negative elements. ((( FALSE )))
The length of an informative message depends upon your communication purposes, among other considerations. ((( TRUE )))
An informative message must reference audience benefits to be effective. ((( FALSE )))
Adding an informational hook such as a coupon can help standard informational messages build goodwill for your organization. ((( TRUE )))
Receiving too much information from a company is appreciated by most consumers. ((( FALSE )))
A good strategy for organizing an informative or positive message is to begin with the good news. ((( TRUE )))
It is important to highlight information in a subject line if the information is neutral or negative. ((( FALSE )))
A way to ensure that an email gets read is to use important information in the subject line. ((( TRUE )))
A goodwill ending to an email should include your regards to the reader’s family. ((( FALSE )))
To summarize a document, it is best to begin with some supporting evidence or the details of a message. ((( FALSE )))
What is an example of a positive message? ((( offering details to employees about the insurance benefits that a company offers )))
Which statement about informative messages is true? ((( They can eliminate future messages on the same subject. )))
Which example would be classified as an informative message? ((( distributing the minutes of a meeting to all members of the meeting )))
What is a primary purpose of an informative message? ((( to have the receiver view the information positively )))
Cyndee is a customer relations manager at Westport Bank. She needs to write to a customer to let him know that his application for an increase in his credit limit has been approved. In her message, she includes brief information about the company's history and testimonials from other valued customers. What is the secondary purpose of including this additional information? ((( building a good image of her organization )))
Delores must send in her smartphone to the manufacturer for repair. When it arrives at the customer service department, she receives an email confirming its receipt and stating that the repair is covered under her warranty. What is the primary purpose of the email that Delores receives? ((( to provide information and good news to Delores )))
Almost all informative and positive messages should focus on ((( benefits to the audience. )))
What is an effective way to develop audience benefits in informative and positive messages? ((( employ you-attitude )))
Informative and positive messages do not need benefits when ((( the benefits make the audience seem selfish. )))
It is not necessary to include benefits in positive and informative messages when ((( presenting factual information only. )))
Which statement about using stories in informative messages is true? ((( They enable people to put facts in context. )))
Which statement about information overload is true? ((( It makes people block or filter out emails they should read. )))
Positive and informative messages should emphasize benefits when ((( presenting benefits that may not be obvious. )))
Why is it essential to include benefits in a message about a policy change? ((( It helps ensure that recipients will understand and support the change. )))
Using a story to frame benefits in a business message is a strategy employed to ((( put facts into context. )))
Janna owns a specialty spice store. She emails her customers regularly to tell them about new product offerings and gift ideas. What other information would most entice customers to open these emails and build goodwill for Janna’s business? ((( winter soup recipes )))
Jeep encourages owners to post videos to its “My Jeep Story” social media sites and show their vehicles in beautiful landscapes around the globe. One company-generated post shows Jeep donating to the USO in honor of this customer group. The company is using what strategy to communicate the benefits of owning a Jeep? ((( telling a story )))
Which statement about using humor in business communications is true? ((( It can help increase the chances that a message is read or heard. )))
Which strategy will make a response to a customer complaint more positive? ((( Show that you are listening and want to resolve the problem. )))
What should senders keep in mind when expressing gratitude in an informative message? ((( It needs to be specific. )))
When using a basic pattern of organization in an informative message, you should ((( understand the rationale behind the pattern. )))
Generally speaking, an informative and positive message should begin with ((( the most important information or good news. )))
How should negative information be presented in informative messages? ((( It should be stated as positively as possible. )))
Which strategy should be used when creating subject lines to help ensure that an email is read? ((( Name drop in the subject line to make an appropriate connection. )))
What is the best subject line for a neutral or positive message from a supervisor to their subordinates? ((( Points to Be Discussed in the Next Meeting )))
Malik is a publicity director for a transnational consumer electronics consortium. He has produced the company’s annual report for 2022 and now intends to distribute it to the principal corporate stakeholders. What is the most appropriate subject line for an email message with the report attached? ((( 2022 Annual Report )))
What is a good strategy for ending a lengthy but routine email? ((( Summarize the basic point. )))
A goodwill ending for an informative business message should focus on ((( the business relationship between parties. )))
Which statement about a goodwill ending in positive and informative messages is true? ((( It should show you see the reader as an individual. )))
What is the best example of a goodwill ending in a positive business email? ((( I am looking forward to receiving more orders from you. )))
The last paragraph of a message written to one specific person should ((( fit only that individual specifically. )))
As the bank’s branch manager, Johan is writing to a client about a business expansion loan for her small company, the Gooey Gourmet. What is thebest example of a goodwill ending for this informative message? ((( I look forward to seeing the expansion of the Gooey Gourmet. )))
The purpose of a transmittal message is to ((( explain what you are sending to someone. )))
What should be the first element appearing in a transmittal message? ((( description of what is being sent )))
Which statement about a transmittal message is true? ((( It should include information that would help the reader understand the document. )))
Which practice should be used when summarizing a message? ((( Start with the main points and then move on to explaining supporting material. )))
Charlene’s boss asks her to summarize an internal meeting that some team members had to miss. Which element is least important for inclusion in Charlene’s report? ((( the length of the meeting )))
Austin is a real estate agent who must submit two competing offers to his client. In addition to the official documents, Austin includes a message telling his clients what he is sending, how to compare them, and the legal deadlines for their response. This additional information sent by Austin is a type of ((( transmittal message. )))
When summarizing a document for your colleagues or supervisor, it is best to begin with ((( the main point. )))
Reports that recommend action are persuasive messages. ((( TRUE )))
Common errors that weaken arguments in a message are known as logical fallacies. ((( TRUE )))
When consumers decide to buy products or services, they are guided exclusively by logic; emotional appeal has no place in purchasing decisions. ((( FALSE )))
When you know that the audience does not feel any resistance to doing as you ask, you should use a problem-solving pattern as a persuasive strategy. ((( FALSE )))
Direct requests do not need to include audience benefits to be effective. ((( TRUE )))
In complicated direct requests, avoid anticipating possible responses because they will confuse the audience. ((( FALSE )))
Common grounds in a message to a larger audience is based on a respect for and sensitivity to the audience’s position. ((( TRUE )))
Help keep an audience from ignoring a deadline by showing that acting now will save time or money. ((( TRUE )))
A primary purpose of a fund-raising appeal is to build a good image of the organization. ((( FALSE )))
What is the primary purpose of a persuasive message? ((( to have the audience act or change beliefs )))
What is a secondary purpose of a persuasive message? ((( to build a good image of the communicator )))
On what should emotional appeal primarily depend? ((( values of the audience )))
Shaunna and her friends are debating over the kind of music that is most popular amongst their generation. Shaunna says that most of her friends love country music, so country must be the most popular. Which common logical fallacy has she committed? ((( hasty generalization )))
Credibility isbest described as the ((( audience's response to you as the source of the message. )))
Which strategy will cause you to be persuasive? ((( Show the audience that your proposal meets their needs. )))
The reasons or logic that a writer or speaker offers in a message is referred to as the ((( argument. )))
Annette works for an environmental organization and recently submitted a report on ways to curb climate. However, her report stated that the only way to curb climate change is by reducing energy consumption. Therefore, energy consumers are faced with two choices: to reduce energy consumption and curb climate change or not to reduce this consumption and increase climate change. Her manager gave her feedback on the report saying that there are several ways to tackle climate change. Which logical fallacy is illustrated in Annette’s report? ((( appeal to popularity )))
Deja recently gave a presentation to her company's board of directors about trade relations with Mexico. The presentation was visually appealing because it incorporated many images and videos. However, the board found her logic of quoting celebrity Selena Gomez’s opinion on trade relations with Mexico to be weak and irrational. Which logical fallacy did Deja commit? ((( appeal to authority )))
What is the best way to build one's credibility when you are not expert, high profile or well-known? ((( by being factual and reliable )))
As a long-time employee, Duncan has deep knowledge of the company’s capabilities and products. His recommendations are always based on solid evidence and experience. During a recent meeting with a new client, Duncan is asked to present the proposal and answer all the client’s questions, even though his supervisor is also present. Duncan’s value in this meeting is an example of ((( credibility. )))
Evelyn is an advertising executive for a clothing manufacturer. She is working on a presentation which introduces a new line of spring clothing. She knows that her presentation needs to include important statistics and figures to support her arguments. However, she intends to convince her audience, purchasers for a retail store that features “green” products, primarily by using emotional appeal. Which statement can best help Evelyn emotionally connect with this audience? ((( "We will use eco-friendly raw materials to help conserve the environment." )))
Which statement is true regarding persuasion and culture? ((( Organizational culture shapes which persuasive strategies will be most effective. )))
What does Daniel Pink's research indicate about motivators used at workplaces? ((( Carrot motivators decrease innovation and turn creative work into drudgery. )))
The most appropriate situation for a direct request pattern to be used is when ((( an audience is likely to agree easily to a request. )))
A problem-solving pattern should be used in a persuasive message when ((( logic is more important than emotion in the decision. )))
In which situation is a direct request pattern most likely to be successfully used? ((( when the audience may not read the entire message )))
In which situation is a sales pattern most likely to be used? ((( when the audience may resist doing as you ask )))
Which statement is true of threats? ((( They produce tension in a workplace. )))
Hanami is working on a wildlife sanctuary campaign and needs to collate progress reports from the fund-raising and communication departments. She has yet to receive the communication report from the communications manager, who is working on multiple projects and is currently facing a time crunch. Which statement provides the most effective direct request asking for the progress report? ((( "Please send the progress report for the communication part of the campaign by March 20." )))
When you expect no objections from an audience, you should use a(n) ((( direct persuasive request. )))
Which guideline should be followed when making a direct request? ((( Simply ask for what is needed in the message. )))
In written direct requests, it is recommended to ((( put the request in the subject line. )))
Arjun wants to obtain a newer version of a computer program he uses occasionally at the office. Considering this, what is the most direct way he can make this request to his supervisor? ((( Please provide the latest version of this computer program for my use. )))
In problem-solving persuasive messages, when you are explaining the solution to a problem, ((( start with a solution that the audience favors and show why it won't work before presenting your solution. )))
What strategy should be used in subject lines for problem-solving messages? ((( using a benefit that is likely to help the audience )))
Axel is concerned about an intersection near his business that has been the cause of a few minor accidents in the neighborhood, including one that injured an employee. Axel decides to ask the city’s Traffic Control Committee to place a stop sign at the intersection, but he is aware that they are not very receptive to inputs from local businesses. Which subject line for his message is most appropriate for this audience? ((( "Proposal to improve safety at intersection number 12" )))
Renata works as an assistant editor at a publishing house in Manhattan. Since she resides 25 miles away from her workplace, she spends a lot of time traveling. As a result, she barely finds time to spend with her children. She intends to talk to her manager to ask if she could work from home a few days each week. What would be effective common ground for Renata to propose in solving this problem? ((( increasing productivity )))
Cadie must approve all travel reimbursement claims for everyone in the department. She is having difficulty with too many late reimbursement requests, which further delays the issuing of checks by the Accounting Department. In her communication to the staff, which message offers her audience a reason to respond to her request promptly? ((( "Send in expense reports by the 25th of each month to ensure your prompt reimbursement." )))
Which strategy most likely helps build emotional appeal? ((( stories )))
An orphanage recently held a campaign under the banner "Give today; help our tomorrow." The campaign was organized through posters and social media. Each day a page was dedicated to one of the children and described the life of the child and that child's hobbies, likes, and dislikes. The page concluded with a plea to the audience to help sustain the child's needs. Which strategy did the organization use in their message? ((( emotional appeal )))
What is an example of an emotional appeal using psychological description? ((( "Imagine relaxing on your recliner and the aroma of Ramirez Dark Roast coffee rising from your cup." )))
Which guideline should be followed to create the right tone in a persuasive message? ((( Provide a reason for a request, which improves the tone. )))
Which point should be kept in mind while writing performance appraisals? ((( They should protect the organization and motivate the employee. )))
To be most effective, performance reviews should be ((( regular. )))
What is a form of direct marketing? ((( telemarketing )))
A good opener for a sales or fund-raising message will ((( provide a reasonable transition to the body of the message. )))
"Anything the mind can conceive and believe, it can achieve." ‒Napoleon Hill. What kind of an opener to a message is this? ((( quotation )))
Which component of a message provides the logical and emotional links that move an audience from a first flicker of interest to the action that is wanted? ((( body )))
In a fund-raising message, the action close of the message must ((( make the action sound easy for the audience to perform. )))
In a fund-raising message, the action close of the message must ((( make the action sound easy for the audience to perform. )))
What describes a common type of P.S. (postscript) used in a direct mail letter or an email? ((( It restates points made in a message. )))
Amelie works for Earth Wise, a nonprofit organization. She is composing a letter to raise funds from supporters for a campaign on climate change. In the body of the message, she writes, "According to the Stanley Climate Reports, climate change will leave a billion people homeless by 2030." What does this statement in the body of the message offer logical proof for? ((( The problem of climate change deserves attention. )))
How should emotional appeal be used effectively in fund-raising messages? ((( using sensory details to help people connect to a cause )))
A persona in a message refers to the ((( character who allegedly writes the letter to make a message interesting. )))
What strategy is suggested for the most successful face-to-face persuasion? ((( Identify mutual interests. )))
What does you-attitude primarily focus on? ((( emphasizing the audience's point of view )))
Many service providers are considered retailers because they (((provide their services directly to consumers)))
A merger occurs when (((two companies combine to form a new company)))
A(n) ________ is a partnership established for a specific project or for a limited time. (((joint venture)))
An entrepreneur has been primarily associated with the willingness to((take risks.))
What is one of the difficulties faced by small business owners?((worries about employee problems))
What is a difference between high technology businesses and other small businesses?((High technology businesses require greater capital and have higher initial startup costs thanother small businesses))
What is true of a capitalist economic system?((Prices of goods and services are determined by supply and demand))
The quantity of goods and services that consumers are willing to buy at different prices at a specific time is referred to as((demand.))
The quantity of products that businesses are willing to sell at different prices at a specific time is referred to as((supply.))
What is a defining characteristic of public corporations?((Their stock can be bought, sold, or traded by anyone))
Which business provides a service, but is neither owned by the government nor focuses on earning profits?((a nonprofit corporation))
What is a true statement about the board members of a corporation?((They have a duty of care and loyalty to oversee the management of the firm.))
Preferred stockholders of a corporation((have a claim to profits before other stockholders do.))
Visual communication and document design have little effect on how readers understand the information being presented.(((FALSE)))
The same design conventions work well with all audiences.(((FALSE)))
According to visual communications expert Charles Kostelnick, document creators should consider four levels of design: the individual letters and words, the blocks of text, the graphics, and the document as a whole.(((TRUE)))
The style of a font shapes the reader’s response to a document as much as the size of the font.(((TRUE)))
The design process should be planned and implemented only after one has completed writing a document.(((FALSE)))
Accessible design is preferable, but not required.(((FALSE)))
A document that looks appealing always works well with an audience.(((FALSE)))
The information on the back of a brochure should be as interesting and engaging as the information on the front.(((TRUE)))
Designers should avoid placing a FAQ (Frequently Asked Questions) feature on a website’s home page, as it delays visitors in accessing the site’s primary content.(((FALSE)))
The majority of website visitors will read or scan a page for 10 seconds or less.(((TRUE)))
The design of a document conveys a specific image of the(((document’s creator.)))
Regarding business communications, the design of a document should primarily reflect and reinforce a company’s(((image or brand.)))
Which statement about design conventions is true?(((They provide a design language and shape audience expectations)))
What is a common design convention associated with online shopping sites?(((a shopping cart or basket to add items for purchase)))
Anders is an employee at a web design company. His current project is about designing an online shopping site for a client. For the content, Anders chose the Verdana and Calibri fonts and boldfaced the details regarding terms and conditions and membership fees. In addition, he used the color blue to emphasize details on discounts offered and gift vouchers available. According to Charles Kostelnick's four levels of design, which level of design applies to Anders’ choices on this project?(((intra)))
Olena is creating a brochure for an office tour. She plans to use headings like "Day 1" to differentiate events planned on different days and bullets to list the plan for each day. According to Charles Kostelnick's levels of design, Olena's choices belong to the __________ of design.(((inter-level)))
Dallas, who runs a small lifestyle store in Michigan, recently designed a website to publicize her products and increase her customer base. While designing the site, she used snapshots of her products and photos of local celebrities who visited her store. In addition, she used a bar chart to show that her eco-friendly products are better than other items available in the market. According to Charles Kostelnick's four levels of design, Dallas' choices belong to the __________ of design.(((extra-level)))
Marissa is currently preparing a report on "The Devaluation of the Venezuelan Bolivar." She understands that she has a lot of content and decides to use a large page size of 841 × 1189 mm. She also places the page numbers on the bottom right corner where she feels they will be easily noticeable. Given these facts, we can conclude that Marissa is concentrating on Charles Kostelnick's __________ of design.(((supra-level)))
Every news release and environmental report issued by the Sierra Club includes its green and black logo in the upper left-hand corner of every page. This element is an example of which level of design?(((extra-level)))
Design standards focusing on font choices and their acceptable sizes as well as the use of color to emphasize key words are concerned with which level of design?(((intra)))
Jess recently finished writing an assignment for an end-of-year project. If she used a sans serif font for her headings, which one did she choose?(((Arial)))
Which instructions regarding font choices should be followed while designing a document?(((Twelve-point type should be used for letters, memos, emails, and reports.)))
What statement is true regarding the use of sans serif fonts in documents?(((They should be substituted for fixed-pitch fonts.)))
Lynnie needs to write a report for a class assignment, and she has already finished planning and gathering all the necessary information for the report. Which font should Lynnie choose if she wants all letters to take an equal amount of space?(((Courier)))
Patricia is a sales manager at a bank. She recently received an email from a team member, Ross, requesting leave after the adoption of his child. Though the email was well-worded, she felt as though Ross was shouting at her. What is most likely the reason for Patricia's reaction?(((important sentences in all capitals)))
What is the best guideline for creating white space in a document?(((use a mix of paragraph lengths.)))
Gretta is writing a report on "Impacts of Deforestation in the Amazon" for an English-speaking audience. She will emphasize global warming as the most critical impact of this policy, and wants to place this information in the most important part of the page. In which section of her report should Gretta place this information?(((bottom left)))
Full justification should be used when(((one wants to increase the formality of a document.)))
Readers of English tend to view pages in a Z pattern. This makes which two sections of the page the most important?(((upper left quadrant; bottom right quadrant)))
Mathai has designed a brochure and is not happy with the alignment of the graphics and the text. She feels that the headings, graphics, and the text are not unified. What is the best way to align them throughout the document?(((use a grid.)))
Which of guideline should be followed when using colors and decorative devices in documents?(((analyze an audience before making color choices because the meanings of colors may)))
Color has different connotations among cultures; for example, North American viewers often associate the color __________ with a warning.(((red)))
Repeating design elements in your document has what effect?(((creates a unified look)))
What are aspects of the rhetorical situation that should be analyzed prior to researching or drafting content?(((purpose, medium, audience)))
Blair works at a Tri-State Tires. Her manager asks her to design a promotional item to increase sales of the company’s new flagship line of all-weather tires. After some thought, Blair decides to create a brochure featuring several pictures of vehicles using the tires in a variety of extreme weather situations, since photographs are more likely to excite customers than complex charts and statistics. Which aspect of the rhetorical situation has Blair analyzed?(((purpose)))
From a design perspective, when drafting text for a business communication, it is important to consider how the text is tethered to(((accompanying graphic content.)))
Olive is designing a brochure for an upcoming career fair that she hopes will help attract a more diverse set of job candidates to her company, which currently employs only 20% women and 10% people of color. When selecting visuals for the brochure, which strategy is the most appropriate?(((use photos featuring many women and people of color to maintain authenticity while reflecting the company’s hiring goals.)))
The design of a document should be used to control and optimize(((the audience’s experience.)))
Which term refers to the act of making a document inviting and usable for all users, without treating people who need special accommodations differently from other users?(((inclusivity)))
What is the best example of inclusive design?(((using appropriate contrast and legible fonts to make images and text easier to read)))
Which design reflects accessibility?(((using high contrast backgrounds in advertising to make products stand out)))
According to usability expert Jakob Nielsen, how many people need to test a document to reveal 85% of its flaws?(((5)))
Who is most likely to be able to identify design flaws in a document?(((people who are most likely to have trouble reading the document)))
Cam has just finished writing a draft of the instructions that will be included with the new robot vacuum produced by his company. Which process is likely to reveal the most flaws in the document’s design?(((observe and interview a diverse group of users while they read the document.)))
The best way to capture an audience’s attention with a brochure is to(((speak to their needs.)))
Rianne is designing a brochure to encourage people to consider her town as a destination for their next vacation. What should have the most influence over where she places textual and visual elements?(((the audience’s viewing tendencies)))
According to usability consultant Jakob Nielsen, the audience’s tendency to view webpages in an F-shaped pattern means that(((the top of a page must contain the most important information.)))
What is an effective way to ensure that a reader stays long enough on a home page?(((make clear what readers will get if they click on a certain link.)))
Charles designed a web page with large blocks of text, small graphics, minimum animation, and a light background. He had also included links that change color when visited and a search feature at the top of the page. When he presented the web page to his clients, they found a major problem in the web page. Which element is most likely the problem with this web page?(((large blocks of text)))
Zoe designed a web page. To make it usable by people with vision impairments, she placed navigational links, a search box, and a link to a text-only version of the site in the upper left-hand corner. She also arranged the navigational links alphabetically and provided alternative text for all images and animations. She used a light background color for the site and three different fonts. Finally, she used phrases like "Click here" for hyperlinks. When people with vision impairments used this website, there was a common problem experienced by them. Which element is most likely the cause of the problem faced by these users?(((use of phrases like "Click here" for hypertext links)))
When designing a website, it is a good design strategy to(((provide visual variety through bullets and indentations.)))
A website that was created with mobile-first design will(((provide a functional experience across all types of devices.)))
Which type of website is best suited for adaptive design?(((a government tax website containing reports, contact information, and a payment portal)))
Ronnie is a student with visual impairment who regularly accesses his university website. Which step should the designer of this website undertake to make it useful for viewers with visual impairment like Ronnie?(((provide alternative text or "alt tag" for all images and applets.)))
Jake has finished designing a website for the airline where he works. He now needs to test its usability. According to usability consultant Jakob Nielsen, which step should Jake take?(((observe and interview people using the site at various stages of design.)))
Because visuals can be easily added by computer, including them in your documents is always a good idea(((FALSE)))
A visual allows important information to be emphasized only if the visual appears at the beginning or at the end of a document(((FALSE)))
Jonathan is preparing an article on a new species of plant. He includes several photographs of the leaves, which are less than a centimeter long. To make an effective visual, he should also include a small object, such as a penny or paperclip, so that the reader can gauge the scale of the plant(((TRUE)))
You are creating a table with 20 rows of data. A good way to help readers line up items accurately is to double-space after every row in the table(((FALSE)))
A Gantt chart included in a progress report will indicate the work which has been completed, tasks in progress, and work yet to be done.(((TRUE)))
When time is a variable on a line graph, it should be placed on the vertical axis.(((FALSE)))
Renata is comparing the sales volume in her region to the sales volumes in seven other regions across the country. She should choose a bar graph for her presentation because audiences find it easy to compare items in them.(((TRUE)))
Infographics are being used more frequently in business applications because they rely on numbers to tell the audience a story.(((FALSE)))
Visual information is best organized from left to right.(((FALSE)))
Nadine is creating a written presentation for a client who is new to the printing industry. In the presentation, she supports her text with visuals, letting them speak for themselves, and providing little explanation of the material. This technique will help Nadine make a more positive impact on the client.(((FALSE)))
In a final presentation or document, visuals and data displays should be used to(((emphasize material that might be skipped if it were buried in a paragraph.)))
Brady, a senior marketing manager, is preparing a presentation for the board of directors of his company. The presentation depicts sales patterns for the company's top products in different geographical areas. Brady includes graphs, charts, and tables to present the data. However, the evening before the presentation, he learns that the board considers visuals and data displays a waste of time. In this scenario, what should Brady do to ensure that his presentation is well-received?(((He should use only the important and necessary visuals because his purpose and the information call for them.)))
The number of visuals needed in a presentation is most dependent upon which factor?(((The rhetorical situation)))
Images refers to what specific type of visuals?(((Photographs, drawings, and maps)))
Anshul wants to persuade the school board that the high school cafeteria needs immediate repair after a severe storm. What type of visual will be most persuasive and realistic?(((Photographs showing water damage and mold)))
The safety team’s presentation will discuss the interconnecting levels of the mine, and how each level must have two or more exits in case of accidents. What is the best visual to illustrate this concept, and eliminate unnecessary details for the audience?(((Drawing)))
Mimiko creates a visual for her presentation to an environmental conservation company. The visual demonstrates how various companies' pollution systems are destroying undersea life. She wants to ensure that her visual contains enough detail to make her point clear so that the firm takes action. In this scenario, which visual is Mimiko most likely to use to display her data?(((Drawing)))
To make map displays more accurate and ethical, you should(((consider how to reduce distortion.)))
_________blank are numbers or words arrayed in rows and columns.(((Tables)))
When you want an audience to be able to identify exact values in a set of data, you should use a(((table.)))
For the visual presentation of data to be effective, a table should(((place items that readers are to compare in columns rather than in rows.)))
When you want to emphasize specific numbers to your reader, use a(n)(((table.)))
What do charts use to convey information to the viewer?(((Spatial relationships)))
Nilsson’s presentation describes the administrative structure within his firm. He plans to show the CEO at the top, 5 division vice presidents who report to her, plus 30 department heads scattered among the divisions. To illustrate various administrative relationships, which visual should he use?(((Organizational chart)))
Which visual should you use to show a projected timeline for a project?(((Gantt chart)))
What is recommended for creating an effective Gantt chart?(((Specify critical activities)))
Gantt charts include the variables of sequence of tasks, duration of tasks, and(((overlap of tasks.)))
Five companies contributed different amounts to a local charity. Which visual wouldbest illustrate the comparative contribution of each company to the total amount contributed?(((Pie graph)))
What is a suggested guideline for constructing bar graphs?(((Put the bars close together to make comparison easy.)))
Which statement about designing pie graphs as visuals or data displays is true?(((They show segments of the whole.)))
When you want the audience to compare items or see a distribution or correlation, use a(((bar graph.)))
In which scenario would a bar graph be most effective?(((to show a company's growth in net sales over time)))
_________blank allow you to compare either several aspects of each item or several items over time(((Grouped bar graphs)))
_________blank identify positive and negative values, or winners and losers.(((Deviation bar graphs)))
Which statement about bar graphs as visuals or data displays is true?(((Bar graphs can be used to show correlations and to compare items over time.)))
What is a good practice when finding stories to tell through visuals?(((Simplify the data on a selected topic and convert numbers to easy-to-understand units.)))
Ted is summarizing his thesis using bar graphs. When creating them, he is careful about ordering the bars in chronological order, labeling both axes, and making the bars different colors when they refer to different items. Because he has long labels, he uses horizontal bars. He also places some labels inside the bars and some outside for clarity. In this scenario, which action taken by Ted is most likely to reduce the quality of his bar graphs?(((Placing some labels inside the bars and some outside)))
Wilson, a sales manager, conducted a competition to see which of his team members exceeded sales quotas in the previous month. To decide the winner, he collects his data and creates a visual. Which visual will best help him identify a winner?(((Deviation bar graph)))
Alba is collating the results of a survey she conducted. The purpose of the survey was to identify the number of people across four regions who felt strongly about recycling, those who did not feel strongly, and those who were indifferent to recycling. Now, she wants to depict the values as percentages as well as whole numbers. Which visual is likely to be most useful for Alba?(((Grouped bar graph)))
Hailey constructs a line graph to depict the increasing productivity of her three team members over the past year. She puts time on the vertical axis and productivity on the horizontal axis. She labels both the axes and gives the graph a title. She does not use 3-D perspectives because she feels they will make the values difficult to read. When she presents her graph to her manager, he is dissatisfied because he finds it difficult to read. What is the most likely reason for his dissatisfaction with the graph?(((She has depicted time on the vertical axis, not the horizontal axis.)))
Shayna is preparing a bar graph to depict the varying amounts of iron ore found in different geographic regions of the country. She adds a title to the graph and labels both the axes. All the labels are present inside the bars, and the width of the bars varies according to the value each represents. She ensures that the bars are placed close enough together to make comparison easy. Which correction to Shayna’s bar graph will help improve its quality?(((The width of all the bars should be the same.)))
Hamlin, a sales manager, must present the annual sales report for his product at next month's senior management meeting. Before creating the graphs for the report, he is making sure that his figures are accurate and that no information is missing from key locations. Which guideline for creating effective graphs is Hamlin following in this scenario?(((Check the reliability of the data.)))
Which statement is true regarding the use of scatterplot graphs?(((They show relationships between two sets of numerical data.)))
What distinguishes hybrid visuals such as dynamic data displays from other types?(((They combine visual elements with interactivity.)))
Gayle, the managing partner at Premium Properties, will be speaking to new staff about the daily activities of successful agents. She creates a visual with images of agents showing properties, meeting with clients, taking photos, posting online, and checking paperwork. She also includes a line graph correlating home showings with sales, and a bar graph using house icons to indicate this year’s sales completed at, above, and below original asking price. To reach her audience effectively, what visual strategy is Gayle using?(((She is using a visual cluster.)))
What is the advantage to users of dynamic data displays?(((They can be adapted to personal needs or interests.)))
Bertram needs a dramatic visual to convey qualitative and quantitative data from two related research studies. He needs to make a big impact on an audience that will not be concerned with specific numbers and minute details as well as tell the story quickly. Which kind of graphic is the best choice for his presentation?(((Infographic)))
Zahra works for firm that analyzes global rainfall and drought patterns. Her manager asks her to develop a graphic for their internal website that will be easy to update and will allow analysts all over the world to view a variety of data based on their needs. What kind of visual should Zahra add to the website?(((Dynamic data display)))
What is a common convention of visuals and data displays?(((A title telling the story that the visual or data display shows)))
Which statement about colors and their common connotations in the United States is true?(((Red is used to suggest danger.)))
Truncated graphs are most acceptable when(((audiences know the basic data set well.)))
What reflects an important ethical convention when designing visuals?(((Starting the y-axis of graphs at zero)))
Keeping in mind cultural differences between audiences, the most effective visual is likely to be one that(((organizes information according to the reading customs of the audience.)))
When integrating visuals and data displays into your text, a fuller explanation of a visual is necessary when(((the material is new to the audience.)))
When integrating visuals into text, what is the best strategy for discussing visuals?(((Refer to tables and charts, but not maps or graphs.)))
Formal preparation for job hunting should begin about one or two months prior to graduation from your university.(((FALSE)))
The secret to successful networking is reciprocity, which means building a two-way relationship.(((TRUE)))
Résumés should not be more than one page long because readers find them too long to skim.(((FALSE)))
It is ethical to omit a low GPA on your résumé.(((TRUE)))
A chronological résumé emphasizes degrees, job titles, and dates.(((TRUE)))
If you were laid off from your previous job, it is important to explain the layoff in your application materials.(((FALSE)))
If you are sending your résumé in the text of an email, start all lines at the right margin.(((FALSE)))
Before sending out an application letter, Maija ensures that she checks the company website to learn the name and address of the specific individual who will be receiving it. This is an effective technique to use while writing a job application letter.(((TRUE)))
A good strategy for a professional-sounding job application letter is to use an informal style that avoids the use of technical jargon. (( False ))
Ashanti has one full year to go before she graduates and begins interviewing. Which step is most helpful for Ashanti toward her formal preparation for job hunting?(((Get a co-op job or internship in her field of interest.)))
Finn, a marketing assistant, does not like his current job because it offers a very limited scope for growth. He is looking for a new job in the same field. What would be the most professional way for Finn to look for a new job?(((He should use the time outside of work hours for his job search.)))
Which statement is true regarding how employers hire new employees?(((Employers look for employees in the exact opposite way from the way most people look for jobs.)))
Which statement is true of successful job searches?(((They build relationships through networking.)))
Which statement is true of online job fairs?(((They are characterized by a wide geographic range.)))
What should job seekers be aware of regarding networking?(((Good networkers look for ways to reciprocate.)))
A specialized use of the Internet for job searching is_________blank, which refers to marketing yourself to potential employers.(((personal branding)))
Which online tool allows you to include useful information beyond your résumé and access a powerful search engine designed to connect you to other professionals?(((LinkedIn)))
What statement is accurate about job candidates and their social media profiles?(((Remove embarrassing personal information from all social media platforms.)))
What describes a way that employers typically use résumés?(((Employers use résumés to decide whom to interview.)))
What is an appropriate length for a résumé of a recent college graduate?(((1–2 pages)))
Ulla is now working as a receptionist but is applying for a junior assistant position at her firm. Her current duties are drafting and printing letters, reorganizing the filing system, ordering office supplies, interviewing new clients, and answering phones. What strategy should she use in organizing her résumé?(((Emphasize duties such as interviewing new clients which are most relevant to the new position.)))
What functions do verbs and gerunds generally serve in a résumé?(((They create a more dynamic image of you than nouns do.)))
Which element can make your résumé look busy?(((using multiple fonts)))
Which statement is true of honesty on résumés?(((One of the most frequent inaccuracies on a résumé is incorrect dates of employment.)))
According to research, which key words or phrases are employers most likely to look for in the résumés of recent college graduates?(((ability to work in a team)))
What types of information are employers allowed to gather during a background check?(((criminal records)))
A chronological résumé is more appropriate than a skills résumé when(((your education and experience are a logical preparation for the position for which you are applying.)))
When is it recommended to use a skills résumé?(((when your education and experience are not the usual route to the position for which you’re applying.)))
Which element is considered an essential in a résumé?(((experience)))
Which listings should always stand as separate categories on a résumé, even if you have only one item under each head?(((Education and Experience)))
What should you keep in mind when designating experience on your chronological résumé?(((Focus on results rather than duties.)))
What is recommended to job candidates regarding making their personal website accessible to prospective employers?(((Ensure that both its content and writing looks professional.)))
What is the best way to deal with a situation when all your work experience is in your family's business?(((List the company you worked for and list a client who can talk about your work.)))
What is a recommended way to explain a previous layoff to a prospective employer?(((giving a neutral explanation about the layoff)))
When emailing your résumé to a prospective employer, what is considered a good practice?(((Write a simple subject line that includes your name and the word "résumé.")))
Yei is preparing to apply for a new job, and the recruiter has requested that she send her résumé via email. Before sending the résumé, she should(((email the résumé to herself to see how it will look.)))
When sending a résumé as an attachment to an email, what should you mention in the body of the email?(((the types of files included in the attachment)))
If you are sending your résumé in the text of an email, you should(((eliminate decorative elements.)))
What is a difference between a solicited letter and a prospecting letter?(((A solicited letter helps you apply to an advertised opening; a prospecting letter helps you tap into the hidden job market.)))
Lorenzo is a recent college graduate. He is writing a job application letter in response to an advertisement in a job portal. What should Lorenzo do while organizing his letter?(((He should relate his achievements to the work he would be doing in the new job.)))
What should be included in the first paragraph of a prospecting job application letter?(((a statement that will catch the reader's attention)))
Clement is a recent college graduate. He is eager to impress with his job application letter and wants it to stand out among all the other letters the employers receive. What could appropriately separate him from other applicants applying for the same position?(((his knowledge of the company)))
The closing sentence of an application letter reads, "I want an interview with you." Which statement is most likely to be the best revision for this sentence?(((I look forward to talking to you about how my qualifications could meet the company's needs.)))
Kaori is applying for a financial analyst position recently posted at a nearby firm. Her qualifications are a good fit for the requirements of the position. Which sentence is the best way to end the letter which will accompany her résumé?(((I look forward to discussing how I can contribute to your strong financial team.)))
What is a suggested way to demonstrate in a job application letter that you are more qualified than other candidates?(((Stress your related accomplishments and experiences.)))
What is an important similarity between résumés and job application letters?(((Both must be error-free and show evidence of your communication skills.)))
A sentence in a job application letter reads, "You can call my professor to verify whether the recommendation is valid." Which statement is most likely to be thebest revision for this sentence to ensure positive emphasis?(((Professor Patel can provide more specific information about my coursework and qualifications.)))
Which example in a job application letterbest demonstrates you-attitude?(((My experience in sales has taught me to accurately predict customer demand.)))
Which statement about you-attitude in a job application letter is true?(((It must show the employer what you can do for them, not what they can do for you.)))
Which sentence best illustrates you-attitude?(((My coursework in project management has increased my ability to improve team efficiency.)))
What is recommended in terms of paragraph length for a job application letter?(((If a paragraph covers two or more subjects, it should be divided into two or more paragraphs.)))
Which statement about the length of a typical job application letter is true?(((A letter should consist of at least three paragraphs.)))
Which statement about following up after sending a job application letter is true?(((You can follow up with the employer once if you hear nothing after two or three weeks.)))
Letters, emails, and computer printouts of numerical data are examples of formal reports.(((FALSE)))
A good report problem in business can have a hypothetical audience.(((FALSE)))
In preparing a report, write the purpose statement before researching the specific alternatives your report will discuss.(((TRUE)))
Information crucial to your proof should appear in the body of your report.(((TRUE)))
Processing information before presenting it to the audience is considered a manipulation of the data and should be completely avoided.(((FALSE)))
In chronological progress reports, it is recommended to summarize progress in terms of goals and the original schedule, using measurable statements.(((TRUE)))
The conclusions and recommendations section of a report presents new information that was excluded from the rest of the report.(((FALSE)))
What is the best example of a formal report?(((an analysis of a company’s issue with a title page, a transmittal, and a table of contents)))
What type of report collects data for the reader, but does not interpret or analyze it?(((information)))
Martin, a production manager at a manufacturing facility, prepares a report that evaluates an action proposed by the management committee of his company. He illustrates through evidence that the proposed action will not provide the expected productivity gains. In this case, Martin is preparing a(n)_________blank report.(((feasibility)))
Vincenzo, a manager of a language translation firm, must explain and substantiate the need for the purchase of the latest AI machine translation software. To do so, he needs to write a(n)_________blank report.(((justification)))
Jessie heads a project team which is upgrading technology used by local first responders. The project has been funded by a federal grant and has specific procedures and deadlines. To qualify for the second phase of funding, Jessie must analyze the work done so far and indicate how her team is advancing to the deadline. What type of report is she preparing?(((progress report)))
Which type of report documents the causes of a failure or possible products that are not economically or technically feasible under current condition?(((closure)))
Which criteria does a good report problem in business or administration meet?(((The audience for the report is able to implement the recommended action.)))
What is true of good report problems?(((They are important enough to be worth solving.)))
Dominic’s firm is losing money on a new line of footwear, even though it bears the name of a famous basketball star. His boss asks him to report on this issue and recommend action. Before he can begin to explore potential solutions, what is the first step that Dominic should take?(((Define the problem accurately.)))
A good purpose statement describes the organizational problem or conflict as well as the(((report’s rhetorical purpose.)))
Which element of a report helps clarify the organizational problem, the specific technical questions to be answered, and the rhetorical purpose of the report?(((purpose statement)))
Min is preparing a report titled "The Rate of Urban Development in Ten Districts." After analyzing the data collected, she prepares a draft of the table of contents. What is she most likely to include in the appendix of her report?(((technical data that will be carefully looked over by specialists)))
What should most heavily influence how much information you include in a report?(((what the audience needs to understand the problem and make a decision)))
Harlen must submit a quarterly report in which he makes recommendations to his company’s decision makers on which products have potential for growth. He also has a lot of supporting technical data that will be analyzed by only a few interested engineers. Where should he include the supporting data?(((appendix)))
Enzo is preparing a report for his firm’s executives that will present the benefits and drawbacks of using electronic media over print media. They will use this information to plan their marketing strategy for the coming year. Which pattern of organizing information willbest serve his purpose?(((pro-and-con)))
Which pattern of organizing data works well when the solutions the audience is likely to favor will not work?(((elimination of alternatives)))
Grady, a branch manager of a cruise company in Florida, needs to prepare a report on the recent challenges faced by the accounts, marketing, and reservation departments of his branch. In the report, he also needs to include different ways of resolving the issues faced by each department. Which pattern of organizing data would best help Grady present his report?(((functional)))
Kian’s firm manufactures cars in three countries, although all the automobiles are sold in the U.S. He is asked to compare costs for labor, materials, and distribution in the three facilities, but is not required to recommend improvements. What organization(((geographic)))
The European Union’s Commission on the Environment is researching the impact of shipping on air and water pollution throughout the continent. This research will not result in any further action or recommendation, but summarizes the problem and its ramifications for a broad audience. This is an example of a(n)_________blank report.(((informative)))
A(n)_________blank report evaluates two or more alternatives before suggesting that one among them be chosen, and usually consists of the decision to be made, a listing of the alternatives, and an explanation of the criteria.(((recommendation)))
Sadie, a campaign manager at Classroom Connection, needs to purchase computers for distribution in a few villages. This action will enable her to achieve one of the milestones of the literacy campaign she is working on. What kind of report does she need to prepare to substantiate the need for placing an order for 30 computers?(((justification)))
While writing a report, which strategy is recommended for using headings?(((Use a different font for headings and subheadings to distinguish them from the body of the text.)))
Identify the correct statement about presenting information effectively in reports.(((Ensure that your headings are parallel with each other.)))
What statement is true about progress reports?(((They reassure the funding agency or employer that you are working toward a solution.)))
In a progress report, what is the best strategy for referencing problems encountered?(((Focus on solutions, not the problems themselves)))
Including a Gantt chart in a progress report enables the audience to see(((tasks and their expected duration.)))
Livia is writing a progress report. In the report, she proposes a different approach from the one that was previously approved, while also remaining open to the idea of continuing with the original plan. What is Livia’s progress report trying to accomplish?(((to float trial balloons)))
Li Mei is writing a progress report on the development of a new inventory system for a client. She and her team faced a technical glitch last week, which has since been solved. What is thebest way to present this information to the client?((("We overcame a major technical glitch last week. We are now on track to finish the work by the due date.")))
Rona has completed writing a chronological progress report on marketing a newly launched line of clothing. Which statement would best summarize her report?((("The project is 25 percent complete and going according to our planned work schedule.")))
Linda is writing a report for her company, National Care, with a recommendation to organize a fund-raising campaign. The campaign will try to provide aid to people whose homes have been destroyed by a recent catastrophic flood. What is an appropriate title for her report?(((Campaign to Help Flood Victims)))
In which section of a report should the release date be included?(((title page)))
Jamila is writing a report on a campaign for children's rights that she had organized. She is currently working on a section of the report that mentions the person who authorized the campaign and the intended purpose of the campaign. The section also highlights areas of future additional research. This section is intended to help orient the reader to the report and build a good image of the report and the writer. Which section of the report is Jamila most likely to be working on?(((memo of transmittal)))
Which element isnot necessary to include in a letter or memo of transmittal?(((the history of the problem discussed)))
What is used to present the logic skeleton of a report and to give the most useful information in the shortest space?(((summary abstracts)))
Regarding the different sections of a report, what allows a reader to evaluate the report on appropriate grounds by identifying how broad an area the report surveys.?(((scope statement)))
Noreen has been asked by her manager to create a report that outlines the factors to be considered if they decide to relocate their business. While creating the report, she outlines the relative importance of the cost of office space to the availability of skilled workers in the introduction section of the report. In which part of the introduction should Noreen include this information?(((criteria)))
What section of a report lists suggested action items that will solve the problem?(((recommendations)))
What section of a report concisely describes topics the report discusses and indicates the depth of the discussion?(((descriptive abstract)))
What does the background section of a report mainly cover?(((information detailing the history of the problem)))
What statement is true regarding white papers?(((They are written to educate the audience.)))
Tavis, a commercial real estate agent, is writing a report titled “U.S. Senior Housing and Investor Insights” for an industry publication. While he will not send it to his clients directly, he will include a link to the article on his website. In the article Tavis discusses the new challenges found in this market sector and how investment strategies should adapt. He includes significant quantifiable data to support his conclusions. What kind of report is Tavis writing?(((white paper report)))
List price × trade discount rate = amount of trade discount. ((( TRUE )))
Trade discounts represent increases to the purchase price. ((( FALSE )))
The trade discount results if the customer pays the bill within the discount period. ((( FALSE )))
Amounts of trade discounts are not fixed. ((( TRUE )))
Trade discounts sometimes are taken on freight. ((( FALSE )))
Net price times rate of trade discount equals amount of trade discount. ((( FALSE )))
Net price plus amount of trade discount equals list price. ((( TRUE )))
The portion could represent the amount of trade discount in the portion formula. ((( TRUE )))
The complement of 25% is 52%. ((( FALSE )))
A radio with a net price of $700 and a trade discount rate of 30% has a list price of $1,000. ((( TRUE )))
A chain discount is a series of three or more successive discounts. ((( FALSE )))
Chain discounts sometimes may be added together. ((( FALSE )))
The net price equivalent rate helps one calculate the actual cost to the seller. ((( FALSE )))
The complement of 10% is 90%, or in decimal 0.90. ((( TRUE )))
The net price equivalent rate should be rounded to the nearest hundredth. ((( FALSE )))
The single equivalent discount rate is found by taking the net price equivalent rate and adding it to 1. ((( FALSE )))
A single equivalent discount rate of 0.45975 means the buyer saves approximately 46 cents per $1 of list price. ((( TRUE )))
Net price times single equivalent discount rate equals amount of trade discount. ((( FALSE )))
The net price equivalent rate of 10/20/5 is 0.684. ((( TRUE )))
The single equivalent discount rate of 9/5/4 is 0.17008. ((( TRUE )))
Trade discounts must be paid within the cash discount period. ((( FALSE )))
A cash discount is a reduction from the gross amount of the invoice if payment is made within the discount period. ((( TRUE )))
A discount period is always greater than the credit period. ((( FALSE )))
Cash discounts are not taken on freight or returned goods. ((( TRUE )))
FOB shipping point means the seller pays cost of freight. ((( FALSE )))
Sometimes the seller will prepay the freight if it is FOB destination. ((( FALSE )))
Knuckles on your hand represent months with 30 days. ((( FALSE )))
50 days from July 9 is August 28. ((( TRUE )))
Feb. 15 is 71 days from Nov. 29. ((( FALSE )))
A cash discount sometimes is taken on the amount of trade discount. ((( FALSE )))
2/10, n/30 is an example of the ordinary dating method. ((( TRUE )))
2/10, n/30 means the credit period is 30 days from the end of the discount period. ((( FALSE )))
3/10, 1/15, n/30 means that a 3% cash discount is available from day 11 to day 15. ((( FALSE )))
4/10, n/30, ROG means a 4% discount can be taken within 10 days before the receipt of goods. ((( FALSE )))
2/10, EOM means that the cash discount period ends on the tenth day of the month that follows the sale, as long as the invoice is dated 25th or earlier in the month. ((( TRUE )))
2/10, EOM means the credit period ends 20 days after the end of the discount period. ((( TRUE )))
Goods sold after the 25th of the month with terms of 2/10, EOM results in the discount period being extended 10 days longer. ((( FALSE )))
Those making partial payments are not allowed to take a cash discount. ((( FALSE )))
The actual credit one receives in making a partial payment is calculated by taking the partial payment and dividing by (1 + the discount rate). ((( FALSE )))
Amount of trade discount equals: ((( List price × trade discount rate )))
Which of the following is usually not true about customer catalogs? ((( Completely updated once a year with a new revised catalog )))
Amount of trade discount is represented by the: ((( Portion )))
The complement of 35% is: ((( 65% )))
If the net price of a stove is $900 and the trade discount rate is 40%, the list price is: ((( $1,500 )))
The net price equivalent rate of 9/15/18 is: ((( 0.63427 )))
The single equivalent discount rate of the trade discount 5/4/1 is: ((( 0.90288 )))
The credit period is: ((( None of these )))
Cash discounts are: ((( Never calculated on freight and returns )))
FOB shipping point means: ((( Buyer pays the cost of freight )))
Which month has 30 days? ((( June )))
200 days from March 3 is: ((( September 19 )))
60 days from Nov. 19 is: ((( Jan. 18 )))
2/10, n/30 is an example of: ((( Ordinary dating )))
With terms of 2/10, n/30: ((( Credit period is 30 days from date of invoice )))
2/10, n/30, ROG means the cash discount period ends: ((( 10 days after the goods are received )))
An invoice dated March 6 with terms of 1/10, EOM results in the end of the discount period on: ((( April 10 )))
Roland Corporation buys stoves from a wholesaler. The list price of a stove is $900, with a trade discount of 30%. The net price is: ((( $630 )))
A local RadioShack store wants to buy a new line of plasma TVs. Manufacturer A offers chain discounts of 18/12, and Manufacturer B offers terms of 17/13. Which single equivalent discount rate is the best deal? ((( 0.2784: Manufacturer A )))
An invoice dated 5/19/xx received by Jones Supply indicated a balance of $17,000. This balance included a freight charge of $1,000. Terms of the bill were 5/10, 4/30, n/60. Assuming Jones pays off the bill on June 4, he pays: ((( None of these )))
Black & Decker Manufacturing sold a set of saws to True Value Hardware. The list price was $3,800. Black & Decker offered a chain discount of 8/3/1. The net price of the saws is: ((( $3,357.21 )))
A manufacturer of skis offered chain discounts of 10/5/4 to many of its customers. Joe Jones ordered skis that had a total list price of $149,500. The amount of the trade discount is: ((( $26,790.40 )))
A bedroom set lists for $12,000 and carries a trade discount of 30%. Freight (FOB shipping point) of $150 is not part of list price. Calculate the delivered price of the bedroom set including the freight. Assume a cash discount of 2%. ((( $8,382 )))
An invoice dated March 12 in the amount of $50,000 is received by Reliance Corporation on March 16. Cash discount terms on the invoice are 3/10, n/30. On March 16 Reliance mails a check in the amount of $8,000 as partial payment on the invoice. The outstanding balance owed on the invoice is: ((( $41,752.58 )))
The complement of 17% is: ((( 83% )))
The net price equivalent rate of 3/7/13 is: ((( 0.784827 )))
Brenda purchased a total of five computers for $3,300. She received a 20% trade discount. What did Brenda pay for the computers? ((( $2,640 )))
Walgreens is offered a chain discount on cosmetics from Libby Corp. of 8/5/2; Roma Company offers a chain discount of 7/6/4. What is the highest discount offered? ((( 0.160768 )))
Brooks Brothers bought a suit that has a list price of $1,219, with a chain discount of 7/4/2. What is the net price of the suit? ((( $1,066.56 )))
"Complete the following:Item List Price Trade Discount Net Price
Television $900 30% ?" ((( $630 )))
"Complete the following: Item List Price Trade Discount Net Price
Smartphone ? 40% $600" ((( $1,000 )))
"Find amount of (A) net price and (B) trade discount Item List Price Chain Discount Net Price (A) Trade Discount (B)
Dishwasher $1400 13/10/4 ? ?
" ((( A. $1,052.35; B. $347.65 )))
Which option yields the highest single equivalent discount? Option A Option B 6/8/12 7/9/10 ((( Option A )))
Complete: Item List Price Chain Discount Net Price Equivalent Rate Single Equivalent Discount Rate Amount of Trade Discount Net Price Rug $1,400 18/12 (A) (B) (C) (D) ((( "A. 7216
B. 2784
C. $389.76
D. $1,010.24" )))
Calculate (A) amount to be credited and (B) balance outstanding: Invoice Amount Terms Invoice Date Payment Amount Date Paid Amount to be credited Balance Outstanding $3,000 2/10, n/30 July 5th $600 July 14th (A) (B) ((( A. $612.24; B. $2,387.76 )))
Calculate (A) cash discount and (B) net amount paid: Gross Invoice (including freight) Freight Date of Invoice Terms Date of Payment Cash Discount Net Amount Paid $1,800 $50 8/11 2/10, 1/15, n/30 8/22 (A) (B) ((( A. $17.50; B. $1,782.50 )))
An invoice for office furniture for $1,210.88 with terms of 2/10, n/60 is dated October 3. (A) What is the last date on which the cash discount may be taken? (B) If paid within discount period, how much is the cash discount? (C) If the discount is missed, on what day does the credit period end? ((( A. Oct.13; B. $24.22; C. Dec. 2 )))
Jones Company received an invoice for $1,450 dated August 28, terms 2/10 EOM. (A) What is the last date of the discount and credit periods? (B) If paid on October 3, how much would Jones pay? ((( (A) Oct. 10, Oct. 30; (B) $1,421.00 )))
Calculate the net price based on the information given below. Item List Price Single Trade Discount Net Price Dell Laptop $1,200 30% ? ((( $840 )))
Calculate the list price based on the information given below.Item List Price Single Trade Discount Net Price Camera ? 40% $699 ((( Divide $699 by 60%. )))
Calculate the net price and amount of trade discount (use net price equivalent rate as well as single equivalent discount rate): Item List Price Chain Discount Net Price Trade Discount Troy-Bilt lawn mower $499 7/9 ? ? ((( Net Price: $422.30; Trade Discount: $76.70 )))
Calculate the End of Discount Period and End of Credit Period.Date of Invoice Terms End of Discount Period End of Credit Period August 9th 2/10, n/30 ? ? ((( Aug. 19; Sept 8 )))
Calculate the End of Discount Period and End of Credit Period. Date of Invoice Terms Date Goods Received End of Discount Period End of Credit Period June 19 3/10, n/30 ROG July 8 ? ? ((( July 18; August 7 )))
"Calculate the End of Discount Period and End of Credit Period. Date of nvoice Terms End of Discount Period End of Credit Period Nov 12 2/10 EOM ? ?
" ((( Dec. 10; Dec. 30 )))
Calculate the End of Discount Period and End of Credit Period. Date of nvoice Terms End of Discount Period End of Credit Period Nov 28 2/10 EOM ? ? ((( Jan. 10; Jan. 30 )))
Brooks Dept. Store bought a stereo from a wholesaler with a $1,200 list price and a 28% trade discount. What is the trade discount amount? What is the net price of the stereo? ((( A- Trade discount $336 \ B- Net price $864))
Net price $864
" )))
Jones of San Diego sold Long of Baton Rouge a video system with a $6,000 list price. Sale terms were 2/10, n/30 FOB San Diego. Jones agreed to prepay the $35 freight. Assuming Long pays the invoice within the discount period, what does Long pay Jones? ((( $5,915 )))
Calculate the net price based on the information given below. Item List Price Chain Discount Net Price Sofa $1250 10/5/5/4 ? ((( $974.70 )))
Calculate the list price based on the information given below. Item List Price Chain Discount Net Price Grill ? 3/5 258.02 ((( $280 )))
Which option yields the highest single equivalent discount? Option A Option B 7/10/17 9/11/16 ((( Option B yields the higher discount, of .319684 )))
Calculate the Net Price Equivalent Rate in Decimals, Single Equivalent Rate in decimals, Amount of Trade Discount and Net Price based on the information given below. Item List Price Chain Discount Net Price Equivalent Rate Single Equivalent Discount Rate Amount of Trade Discount Net Price Ring $500 15/20 (A) (B) (C) (D) ((( "A. 68
B. 32
C. $160
D. $340
" )))
Invoice: $1,000 Terms: 3/10, n/30 Invoice date: May 5 Payment amount: $800 Date paid: May 9 A) Amount credited: ________ B) Balance outstanding: ________ ((( A. $824.74; B. $175.26 )))
Invoice: $600 Terms: 2/10, 1/20, n/30 Invoice date: May 5 Payment amount: $400 Date paid: May 17 A) Amount credited: ________ B) Balance outstanding: ________ ((( A. $404.04; B. $195.96 )))
"Gross invoice (includes freight): $500 Freight: $100 Date of invoice: 9/10
Terms: 3/10, 1/15, n/30 Date of payment: 9/22 Discount: (A) Net Price: (B)
" ((( "A. $4.00
B. $496
" )))
Gross invoice (includes freight): $1,000 Freight: $100 Date of invoice: 8/11 Terms: 2/10, 1/15, n/30 Date of payment: 8/30 Discount: (A) Net Price: (B) ((( "A. $0
B. $1,000
" )))
Calculate the Last Day of Discount Period and End of Credit Period based on the information given below. Date of Invoice Date Goods Received Terms Last Day of Discount Period End of Credit Period Oct 7 2/10, n/30 (A) (B) June 12 2/10, EOM (C) (D) July 29 2/10, EOM (E) (F) March 8 July 2 2/10, n/30, ROG (G) (H) ((( A. Oct. 17 B. Nov. 6 C. July 10 D. July 30 E. Sept. 10 F. Sept. 30 G. July 12 H. Aug. 1 )))
Calculate the Last Day of Discount Period and End of Credit Period based on the information given below. Date of Invoice Date Goods Received Terms Last Day of Discount Period End of Credit Period September 5 3/10, n/30 (A) (B) May 4 3/10, EOM (C) (D) April 26 3/10, EOM (E) (F) January 1 March 3 3/10, n/30, ROG (G) (H) ((( """A. Sept. 15 B. Oct. 5 C. June 14 D. June 24 E. June 10 F. June 24 G. March 13 H. April 2 ""
" )))
A dining room set lists for $1,500 and carries a trade discount of 40%. Freight charges are $150. Calculate the final price of the dining room set. ((( $1,050 )))
Marvin Gate bought some fencing from a wholesaler for $6,000. The wholesaler offered a trade discount of 35%. What was the original price? (Round to the nearest cent.) ((( $9,230.77 )))
Buffy Rich wants to buy a new line of stereos for her shop. Manufacturer A offers trade discounts of 20/14, and Manufacturer B offers trade discounts of 25/7. Assuming both manufacturers have the same list price, which manufacturer should Buffy buy from? ((( Buffy should buy from Manufacturer A. )))
Jones Manufacturing sent Blue Company an invoice for equipment with a list price of $10,000. The invoice is dated July 27 with terms of 2/10, EOM. Blue Company is entitled to a 40% trade discount. If the invoice is paid on September 8, how much does Blue pay? ((( $5,880 )))
" ((( .736281 )))
A Timex watch with a 30% trade discount costs $39.99. What is the list price? ((( $57.13 )))
Ashley Furniture received an invoice for $4,850, dated May 19. Terms were 2/15, net 30 days. How much will Ashley pay if the invoice was electronically paid on June 2? ((( $4,753 )))
Given a list price of $4,400 and a chain discount of 2/3, find the A. Trade Discount, and B. Net Price ((( "A. $217.36
B. $4,182.64
" )))
A local Ford dealership purchased a new Ford Focus from the manufacturer that lists for $22,178. Ford Motor Company offers the dealership an 18% trade discount. Shipping costs totaled $795.00. What did the local dealership pay for the Focus? ((( $18,980.96 )))
Gross profit is net sales minus the cost of bringing merchandise into the store. ((( TRUE )))
Operating expenses are the unusual expense of doing business. ((( FALSE )))
Gross profit plus operating expenses equals net income. ((( FALSE )))
Markup represents an amount needed to cover operating expenses. ((( TRUE )))
Selling price = cost − markup. ((( FALSE )))
When markups are based on cost, the selling price is 100%. ((( FALSE )))
The amount of markup is represented as the portion only when markups are based on cost. ((( FALSE )))
The selling price can be calculated if the cost and the percent markup on cost are given. ((( TRUE )))
Actual cost is equal to the cost times the markup percent on cost plus 1. ((( FALSE )))
Cost is equal to the selling price divided by (1 + markup percent on cost) when markup is based on cost. ((( TRUE )))
When markup is based on selling price, the cost is 100%. ((( FALSE )))
When markup is based on selling price, the cost is the base. ((( FALSE )))
Percent markup on the selling price is equal to the amount of markup divided by the selling price. ((( TRUE )))
If the selling price and percent markup on selling price are given, the actual cost can be calculated. ((( TRUE )))
Selling price times 1 minus markup percent on selling price will equal the cost if markup is based on selling price. ((( TRUE )))
Dollar markup divided by the selling price equals percent markup on cost. ((( FALSE )))
Percent markup on selling price can be converted to percent markup on cost by a formula. ((( TRUE )))
Dollar markdowns represent price increases to the original selling price. ((( FALSE )))
The markdown percent is the amount of markdown divided by the new sale price. ((( FALSE )))
A final selling price may be the result of a series of markdowns (and possibly some markups). ((( TRUE )))
To place a price on perishable items, there is no need to calculate the total cost as well as total selling price of the items. ((( FALSE )))
Contribution margin is selling price plus unit cost. ((( FALSE )))
Breakeven point is fixed cost divided by contribution margin. ((( TRUE )))
Net income is calculated as: ((( Net sales - costs - operating expenses )))
Markup is: ((( Selling price - cost )))
When markup is based on cost: ((( Cost is 100% )))
An Apple iPod sells for $299, which is marked up 40% of the selling price. The cost of the iPod is: ((( $179.40 )))
Gap sells jeans that cost $21.00 and sell for $29.95. The percent of markup based on cost is: ((( 42.62% )))
(1 + markup percent on cost) × cost equals: ((( Selling price )))
If percent markup on cost and selling price are known, one is able to compute the: ((( Cost )))
When markups are based on the selling price, the: ((( Selling price is 100% )))
Markdowns may be caused by: ((( Style changes )))
The markdown percent is calculated by: ((( Amount of markdown divided by original selling price )))
Setting a price on perishable items does not include: ((( Calculating a selling price per day )))
A computer sells for $995, which is marked up 35% of the selling price. The cost of the computer is: ((( None of these )))
Red Jeans Inc. sells jeans that cost $16.55 for a selling price of $35.99. The percent of markup based on cost is: ((( None of these )))
A local True Value Hardware Store marks its goods up 38% on cost. If a snow blower cost True Value $400, the selling price would be: ((( $552.00 )))
Jay King, owner of a local Bed and Bath store, knows that his customers will pay at most $299 for an air mattress. Assuming Jay wants a 40% markup on the selling price, the most he could pay the manufacturer for the air mattress is: ((( $179.40 )))
Ski Market sells snowboards. Ski Market knows that the most people will pay for the snowboards is $129.99. Ski Market is convinced that it needs a 45% markup based on cost. The most that Ski Market can pay to its supplier for the snowboards is: ((( None of these )))
At the end of the summer, Walgreens advertised blow-up pools for 66% off the regular price. Jeff Jones saw a pool with a regular price of $49.99. The dollar markdown is: ((( $32.99 )))
A local Dot Dress Shop is selling a suit for $99. Because of changing styles, the first markdown was 8% and second markdown was 25%. The suit still did not sell, so a final markdown of 10% was taken. The sale price is currently: ((( $61.48 )))
Contribution margin is: A) Net sales ((( Revenues minus variable cost )))
Lester Co. produces toy kites. It has a fixed cost of $62,150. If the selling price per unit is $9.50 and the variable cost per unit is $6.25, the breakeven point is: ((( None of these )))
Jackie Smith, a customer of Roger Blank, will pay only $190 for a tennis racket. Assuming Roger works on a 60% markup on the selling price, the most Roger will pay the manufacturer is: ((( $76 )))
A video game sells at Arnolds for $14.99. Arnolds marks the game up at 40% of the selling price. The cost of the video game to Arnold is: ((( $8.99 )))
A wooden duck with a regular selling price of $125.99 is marked down to $79.99. The percent of markdown is: ((( 36.51% )))
An Apple iPad sells for $699.00 on eBay. The markup is 30% on cost. What is the total cost of the iPad sold on eBay? ((( $537.69 )))
Assume markup is based on cost. Complete: Cost % of Markup Amount of Markup Selling Price $500 30% A B ((( A. $150 ; B. $650 )))
Assume markup is based on cost. Solve for the actual cost (round to nearest cent): Selling price $600 Markup % on cost 35% Actual cost? ((( $444.44 )))
Assume markup is based on cost. If the cost of a table is $62 and it sells for $110, what is the percent of markup based on cost? Round to the nearest hundredth percent. ((( 77.42% )))
Assume markup is based on selling price. Complete: Selling Price % of Markup Dollar Markup Cost $750 30% A B ((( "A. $225
B. $525
" )))
Assume markup is based on selling price. Solve for the selling price (round to nearest cent): 25% markup on selling price Actual cost $650 Selling price? ((( $866.67 )))
Complete (round answers to nearest cent): Item Total Qty Bought Unit Cost Total Cost Markup % on Cost Total Sales % That Will Spoil Selling Price Per Unit Brownies 40 $ .70 A 40% B 10% C ((( A. $28; B. $39.20; C. 39.20 / (40 - 4) = 1.09 )))
Calculate the final selling price to the nearest cent (round each calculation to nearest cent as needed): Original Selling Price 1st Markdown 2nd Markdown Markup Final Markdown $6,000 9% 13% 4% 9% ((( $4,495.59 )))
Convert a 40% markup percent on cost to markup percent on selling price. (Round to nearest hundredth percent.) ((( 28.57% )))
Breck Hardware purchased from Black & Decker 10 Dustbusters for $24.95 each. What should Breck charge its customers for each Dustbuster if it has a 39.5% markup on cost? (Round to the nearest cent.) ((( $34.81 )))
In downtown Boston, a bakery produces 200 bagels a day at a cost of $.20 each. It is expected that 25% of the bagels will spoil before being sold. Assuming the bakery expects to make a 40% markup on its cost, what should the selling price of each bagel be? ((( $.37 )))
Tri-City sells a microwave for $550. Tri-City marks up the microwave 70% on cost. What are the cost and markup of the microwave? ((( Cost = $323.53; Dollar markup = $226.47 )))
Munroe Corporation sells Nautilus equipment for $399.95. Munroe marks up the equipment 30% on the selling price. What did the equipment cost Munroe? Round to the nearest cent. ((( $279.97 )))
" )))
Pat Maninan, a customer of Brown Co., will pay $400 for a new kitchen table. Brown has a 55% markup on the selling price. What is the most Brown can pay for this kitchen table? ((( $180 )))
Calculate the dollar markup and selling price. Cost % of Markup on Cost Dollar Markup Selling Price $400 20% A B ((( A. $80; B. $480 )))
Calculate the dollar markup and selling price. Cost % of Markup on Cost Dollar Markup Selling Price $1,200 30% A B ((( A. $360; B. $1,560 )))
Selling price $500 Markup % on cost 40% Actual cost? ((( $357.14 )))
Selling price $700 Markup % on cost 30% Actual cost? ((( $538.46 )))
Selling price $700 Markup % on cost 30% Actual cost? ((( $538.46 )))
A. If the cost of a TV is $120 and it sells for $200, what is the percent of markup based on cost? Round to the nearest hundredth percent. B. Check your answer. ((( A. 66.67% B. Check works out. )))
Calculate the dollar markup and cost. Selling Price % Markup on Selling Price Dollar Markup Cost $800 25% A B ((( A. $200; B. $600 )))
Calculate the dollar markup and cost. Selling Price % Markup on Selling Price Dollar Markup Cost $900 30% A B ((( A. $270; B. $630 )))
26% Markup on Selling Price Actual Cost = $700 Selling Price = ? ((( $945.95 )))
"28% markup on selling price Actual cost $900 Selling price?
" ((( $1,250 )))
Calculate the total cost, total sales, and selling price per unit. Item Total Quantitty Bought Unit Cost Total Cost Markup % on Cost Total Sales % Spoilage Selling Price per Unit Cost Brownies 50 $ .65 A 35% B 20% C ((( A. $32.50; B. $43.88; C. $1.10 )))
Calculate the total cost, total sales, and selling price per unit. Item Total Qty Bought Unit Cost Total Cost Markup % on Cost Total Sales % That Will Spoil Selling Price Per Unit Eclairs 30 $ .90 A 45% B 30% C ((( A. $27; B. $39.15; C. $1.86 )))
Calculate the final selling price. Original Selling Price 1st Markdown 2nd Markdown Markup Final Markdown Final Selling Price $4,000 10% 15% 6% 8% ? ((( $2,984.11 )))
Calculate the final selling price. Original Selling Price 1st Markdown 2nd Markdown Markup Final Markdown Final Selling Price $7,000 15% 10% 8% 8% --- ((( $5,320.73 )))
Convert a 40% markup percent on cost to markup percent on selling price. (Round to the nearest hundredth percent.) ((( 28.57% )))
Convert a 30% markup percent on cost to markup percent on selling price. (Round to the nearest hundredth percent.) ((( 23.08% )))
A toaster is marked up $10 and sells for $45.00. Find the cost and percent markup if the markup is based on cost. (Round to the nearest hundredth percent.) ((( Cost = $35; Percent Markup = 28.57% )))
Blue Jeans Inc. sells jeans that cost $15.99 for a selling price of $42.95. What is the percent of markup based on cost? (Round to the nearest hundredth percent.) ((( 168.61% )))
Find the cost and markup of a stove if it sells for $400 and is marked up 100% of the cost. ((( "Cost = $200; Markup = $200
" )))
Al Shelf knows his goods are marked up 36% on cost. If a TV cost Al $280, what would the selling price be? ((( $380.80 )))
A video game sells at Arnolds for $199.95. Arnolds marks the game up at 30% of the selling price. What was the cost of the video game to Arnolds? ((( $139.97 )))
Jane Corporation produces fudge bars. Total fixed cost is $55,500. Each package of fudge bars sells for $4.95 with a variable unit cost of $3.10. What is the breakeven point for Jane Corporation? ((( 30,000 fudge bars )))
Evelyn Smith, a customer of Bill Blank, will pay only $125 for a tea set. Assuming Bill works on a 40% markup on the selling price, what is the most Bill could pay the manufacturer for this tea set? ((( $75 )))
A wooden duck with a regular selling price of $109.45 is marked down to $69.50. What is the percent of markdown based on the regular price? (Round to the nearest tenth percent.) ((( 36.5% )))
Convert a 36% markup on selling price to the equivalent percent markup on cost. ((( 56.3% )))
Best Buy sells Kindle E-readers for $79.99 that cost the store $16.09. What is the percent markup on cost? (Round to the nearest tenth.) ((( 397.1% )))
Hannecke Hardware sells a wheelbarrow for $69.99. Hannecke marks up the wheelbarrow 40% on the selling price. What did the wheelbarrow cost Hannecke? ((( $41.99 )))
The Fair Labor Standards Act entitles employees to time and a half over 36 1/2 hours. ((( FALSE )))
Gross pay = hours worked × rate per hour. ((( TRUE )))
Overtime is always calculated at time and a half the regular rate. ((( FALSE )))
Being paid biweekly means the salary is paid twice a month. ((( FALSE )))
Being paid biweekly is the same as being paid semimonthly. ((( FALSE )))
Straight piece rate pay provides a worker an incentive to produce more since pay is based on the number of units produced. ((( TRUE )))
Differential pay means that there is one fixed rate per unit that is produced. ((( FALSE )))
A draw is an advance that need not be repaid if the commission does not exceed the draw. ((( FALSE )))
In the portion formula, the commission a worker receives would be the base. ((( FALSE )))
A variable commission scale means that for different levels of net sales there are different commission rates. ((( TRUE )))
Net pay - deductions = gross pay. ((( FALSE )))
FICA stands for federal withholding tax. ((( FALSE )))
The Federal Insurance Contributions Act funds the federal income tax program. ((( FALSE )))
A calendar year is July 1 to June 30. ((( FALSE )))
The number of dependents one claims does not affect the amount of Social Security and Medicare tax paid. ((( TRUE )))
A married person will pay in more Social Security and Medicare tax than a single person. ((( FALSE )))
A self-employed person has at least three employees. ((( FALSE )))
Exempt wages means not taxable wages. ((( TRUE )))
Circular E from the Internal Revenue Service contains many tax tables. ((( TRUE )))
There is a maximum amount of Medicare tax an employee can contribute in a calendar year. ((( FALSE )))
The base amount for calculating Social Security taxes never changes from year to year. ((( FALSE )))
The W-4 is used strictly to calculate Social Security and Medicare taxes. ((( FALSE )))
The percentage method can be used to determine the amount of FIT. ((( TRUE )))
A quarter is 13 weeks. ((( TRUE )))
The taxable earnings column of a payroll register shows the actual tax. ((( FALSE )))
The FICA tax now requires separate reporting for Social Security and Medicare. ((( TRUE )))
Payroll registers cannot record cumulative earnings. ((( FALSE )))
FUTA tax is computed by the wage-bracket table. ((( FALSE )))
SUTA tax is usually paid quarterly. ((( TRUE )))
SUTA tax has no maximum amount that can be paid in a calendar year. ((( TRUE )))
Employers usually pay employees: ((( All of these )))
If you are paid twice a month, you are being paid: ((( Semimonthly )))
John Flynn works at Stutz Toyota and is paid a(n) ________ against his earned commissions. ((( Draw )))
Differential pay schedule is based on: ((( Different levels of performance )))
Social Security and Medicare tax: ((( Have different rates )))
The more allowances claimed on Form W-4: ((( The less money deducted for FIT )))
The percentage method aids in calculating: ((( FIT )))
A merit rating system for SUTA means: ((( Rate varies with employment record )))
The taxable earnings column of a payroll register records: ((( What wages will be taxed )))
FICA tax base: ((( Changes yearly )))
FUTA tax is paid: ((( By the employer )))
Lee Wong is a sales clerk at Sears. She is paid $8.00 per hour plus a commission of 4% on all sales. Assuming Lee works 39 hours and has sales of $4,000, her gross pay is: ((( $472 )))
Leo Corporation pays its employees on a graduated commission scale: 6% on the first $40,000 in sales, 7% on sales from $40,000 to $80,000, and 9% on sales greater than $80,000. Bernie Kaminsky had sales of $105,000. His commission is: ((( $7,450 )))
Ron Swift earned $1,500 last week. He is married, is paid biweekly, and claims two exemptions. Using the percentage method (use tables in the handbook), his income tax is: ((( $73.76 )))
To date, Jay Ward has cumulative earnings of $118,000. This week he is paid $3,000. The total amount of Social Security tax for this week is (assume a rate of 6.2% on $128,400 for Social Security and 1.45% for Medicare): ((( $186.00 )))
Jill Hartman earns $750 per week plus 3% of sales in excess of $6,500. If Jill sells $25,000 in the first week, her earnings are: ((( $1,305 )))
Lana Powell has cumulative earnings of $116,000 at the end of September. In the first week in October she earns $2,000. The amount deducted for Social Security and Medicare from her check is (assume Social Security rate of 6.2% on $118,500 and Medicare of 1.45%): ((( $153.00 )))
Percentage method tables can show: ((( All of these )))
Jasper works at Panera Company for $11.25 per hour plus a commission of 2% of her sales. Assuming Jasper worked 26 hours last week and had sales of $2,610, her gross pay is: ((( $344.70 )))
Sandi works for Elizabeth Arden Cosmetics and earns $500 per week salary plus 4% commission on sales over $2,000. If Sandi sold $2,733 last week, what was her salary? ((( None of these )))
David is paid on a graduated commission scale at Nooter Company. He receives 2% commission on the first $20,000, 6% on sales from $20,000 to $70,000, 8.5% commission on sales from $70,000 to $100,000, and 10% commission on sales over $100,000. David had sales of $82,000. His commission is: ((( $4,420 )))
Complete the following weekly payroll: Assume: (1) FIT calculated by percentage method; (2) Social Security 6.2% on $128,400; and Medicare 1.45%.Cumulative balance before this payroll is below maximum as related to cumulative earnings in calculating Social Security. Name Marital Status Exemptions Claimed Gross FIT Social Security Medicare Net Pay Joe Riense M 3 $1,400 A B C D ((( "A. $105.31
B. $86.80
C. $20.30
D. $1,187.59
" )))
Joyce Meadow pays her three workers $160, $470, and $800, respectively, per week. Calculate what Joyce will pay at the end of the first quarter for (A) state unemployment and (B) federal unemployment. Assume a state rate of 5.4%. Federal rate is .6%. Base is $7,000. ((( "A. $820.26
B. 91.14
" )))
Jeff Jones earns $1,200 per week. He is married and claims four withholding allowances. The Social Security rate is 6.2% on $128,400, and Medicare is 1.45%. Each paycheck, his employer also deducts $42.50 for health insurance. What is his net pay? (Calculate FIT by percentage method.) ((( $993.96 )))
John Rose, owner of Rose Corporation, has three employees who earn $500, $600, and $700, respectively. After 12 weeks how much has John contributed for state and federal unemployment? Assume a state rate of 5.4% and a federal rate of .6%. ((( SUTA = $1,080; FUTA = $120 )))
Bill Burton earns $750 per week plus 5% of sales in excess of $7,500. If Bill sells $34,000, how much are his weekly earnings? ((( $2,075 )))
Aley Company pays all its employees a graduated commission scale: 3% on the first $30,000 sales, 5% on sales from $30,001 to $110,000, and 7% on sales more than $110,000. Jim Jones, an employee of Aley, had $130,000 in sales. What commission did Jim earn? ((( $6,300.00 )))
Molly Flynn, an accountant at Coor Company, earned $127,400 from January to June. In July, Molly earned $4,600. Assume a tax rate of 6.2% for Social Security on the first $128,400 and a Medicare rate of 1.45%. How much is the tax for Social Security and Medicare? ((( $62 Social Security; $66.70 Medicare )))
Paula Ryan pays her two employees $410 and $650 per week. Assume a state unemployment rate of 5.4% and a federal rate of .6%. What state and federal unemployment tax will Paula pay at the end of Quarter 1 and Quarter 2? (( (( State unemployment Quarter1: 665.82 \ Federal unemployment Quarter1: 73.98 \ State unemployment Quarter2: 90.18 \ State unemployment Quarter2: 10.02 ))
Calculate the gross pay of Alice Flynn, a corn picker, assuming she picked 4,150 ears of corn:Corn Picked Rate per ear 1 - 2,000 $.04 per ear 2,001 - 3,000 .05 per ear Over 3,000 .09 per ear ((( $233.50 )))
Calculate the gross pay of Alice Flynn, an apple picker, assuming she picked 6,250 apples: Apples Picked Rate per apple 1 - 2,500 $.04 2,501 - 3,500 .06 3,501 - 4,000 .08 Over 4,000 .10 ((( $425 )))
Calculate the net pay. The balance before this weekly payroll is $90 below maximum as related to cumulative earnings in calculating Social Security. Assume a tax rate of 6.2% for Social Security on $128,400 and 1.45% for Medicare:Name Marital Status Exemptions Claimed Gross FIT Soc Sec Medicare Net Pay B Burns M 2 $900 A B C D ((( A) $54.89 B) SS = $5.58; C) Medicare = $13.05; D) Net pay = $826.48 )))
Jill is $200 from reaching maximum base as related to cumulative earnings for Social Security. She is paid weekly: Marital Status Exemptions Claimed Gross FIT Soc. Sec. Medicare Net Pay Jill Rye M 3 $950 A B C D ((( A-51.31 \ B-12.40 \ C-13.78 \ D-872.51 )))
Robin Small pays her three workers $180, $450, and $615, respectively, per week. Calculate what Robin will pay at the end of the first quarter for (A) state unemployment and (B) federal unemployment. Assume a state rate of 5.8%. Federal rate is .6%. Base is $7,000. ((( SUTA = $881.02; FUTA = $91.14 )))
Rodney Ranger is a shoe salesman for More Co. He is paid $8.60 per hour plus a commission of 3% on all sales. For the week, assuming Rodney works 30 hours and has sales of $2,400, what is his gross pay? ((( $330 )))
Rolf Company pays its employees on a graduated commission scale: 1% on the first $25,000 sales, 3% on sales from $25,001 to $95,000, and 5% on sales greater than $95,000. If John Jones, an employee of Rolf, had sales of $100,000, what commission did John earn? ((( $2,600 )))
Robert Bryan, Jr., earns $400 per week plus 3% of sales in excess of $7,000. If Robert sells $21,000, how much are his weekly earnings? ((( $820 )))
Jim Dashinger is a salesman who receives a $100 draw per week. He receives a 12 1/2% commission on all sales. Sales for Jim for the month were $28,000. What did Jim receive after taking the draw into consideration? (Assume a five-week month.) ((( $3,000 )))
Janice Tax, an accountant for Flee Corp., earned $122,700 from January to June. In July she earned $8,000. Assuming a FICA tax rate of 6.2% for Social Security on $128,400 and 1.45% for Medicare, what is the FICA amount for the month? ((( $1,189.40 )))
Jim Long earned $1,900 biweekly. He claims two exemptions and is married. As his accountant, calculate his federal income tax deduction. Use the percentage method. (Use the tables in the handbook to answer this question.) ((( $121.76 )))
Caroline earns $290.25 per week plus 4% commission on all sales. If Caroline had sales of $11,500 for each week, what was her total pay for the month assuming a four-week month? ((( $3,001 )))
Sonny Siebert earned $485,000 a year as a rookie pitcher for the Cleveland Indians. Assuming a tax rate of 6.2% for Social Security up to $128,400 and Medicare tax of 1.45%, how much did Sonny pay in FICA taxes as a rookie pitcher? ((( $14,993.30 )))
Calculate the gross pay for Ryan Campbell, an orange picker, based on the following differential scale. Ryan picked 3,250 oranges during this pay period. 1-1,000 $.10 1,000 - 2,000 $.13 2,000 - 4,000 $.17 Over 4,000 $.25 ((( $442.50 )))
Dan Zabrowski works according to the following schedule for a given week, and is paid overtime at 1.5 times. Find his total hours, and his gross pay for the week. Employee M T W TH F S Total Hrs Rate per Hr Gross Pay Dan Z. 10.5 10.5 0 10.5 0 10.5 A $15.80 B ((( A. 42 hrs; B. $679.40 )))
Jenny earns $700 per week, plus 6% of sales in excess of $5,700. If Jenny sold $15,791 last week, what were her weekly earnings? ((( $1,305.46 )))
The interest is the amount of money borrowed. ((( FALSE )))
The time of a loan could be expressed in months, years, or days. ((( TRUE )))
The amount a bank charges for the use of money is called interest. ((( TRUE )))
Simple Interest = principal × rate. ((( FALSE )))
18 months is the same as 1.5 years. ((( TRUE )))
The exact interest method represents time as the exact number of days divided by 365. ((( TRUE )))
The federal government likes to use ordinary interest. ((( FALSE )))
July 10 to March 15, given no leap year, is 119 days. ((( FALSE )))
Ordinary interest is never used by banks. ((( FALSE )))
Ordinary interest results in a slightly higher rate of interest than exact interest. ((( TRUE )))
Principal is equal to rate divided by interest times time. ((( FALSE )))
Rate is equal to interest divided by the principal times time. ((( TRUE )))
To convert time in days, it is necessary to multiply the time in years times 360 or 365. ((( TRUE )))
The U.S. Rule is a method that allows the borrower to receive proper interest credit when a debt is paid off in more than one payment before the maturity date. ((( TRUE )))
In the U.S. Rule, the first step is to calculate interest on the total life of the loan. ((( FALSE )))
In the U.S. Rule, the partial payment first covers the interest and the remainder reduces the principal. ((( TRUE )))
Ordinary interest is required by all banks. ((( FALSE )))
Interest is the cost of borrowing. ((( TRUE )))
In calculating interest in the U.S. Rule from the last partial payment, the interest is subtracted from the adjusted balance. ((( FALSE )))
Interest is equal to: (((Principal × rate × time )))
Simple interest usually represents a loan of: ((( One year or less )))
Federal Reserve banks as well as the federal government like to calculate simple interest based on: ((( Exact interest )))
A note dated August 18 and due on March 9, given no leap year, runs for exactly: ((( 203 days )))
A $40,000 loan at 4% dated June 10 is due to be paid on October 11. The amount of interest is (assume ordinary interest): ((( $546.67 )))
Interest on $5,255 at 12% for 30 days (use ordinary interest) is: ((( $52.55 )))
Given interest of $11,900 at 6% for 50 days (ordinary interest), one can calculate the principal as: ((( $1,428,000.00 )))
Interest of $1,632 with principal of $16,000 for 306 days (ordinary interest) results in a rate of: ((( 12% )))
Which of the following is not true of the U.S. Rule? ((( None of these )))
The U.S. Rule: ((( Allows borrowers to receive interest credit )))
At maturity, using the U.S. Rule, the interest calculated from the last partial payment is: ((( Added to adjusted balance )))
The number of days between Aug. 9 and Jan. 3 is: ((( 147 )))
A note dated Dec. 13 and due July 5 (assume no leap year) runs for exactly: ((( 204 days )))
Janet Home went to Citizen Bank. She borrowed $7,000 at a rate of 8%. The date of the loan was September 20. Janet hoped to repay the loan on January 20. Assuming the loan is based on ordinary interest, Janet will pay back how much interest on January 20? ((( $189.78 )))
Joan Roe borrowed $85,000 at a rate of 11 3/4%. The date of the loan was July 8. Joan is to repay the loan on Sept. 14. Assuming the loan is based on exact interest, the interest Joan will pay on Sept. 14 is: ((( $1,860.68 )))
Joe Flynn visits his local bank to see how long it will take for $1,200 to amount to $2,100 at a simple interest rate of 7%. The time is (round time in years to nearest tenth): ((( 10.7 years )))
Joyce took out a loan for $21,900 at 12% on March 18, 2018, which will be due on January 9, 2019. Using ordinary interest, Joyce will pay back on Jan. 9 a total amount of: ((( $24,068.10 )))
Sandra Gloy borrowed $5,000 on a 120-day 5% note. Sandra paid $500 toward the note on day 40. On day 90 she paid an additional $500. Using the U.S. Rule, her adjusted balance after the first payment is: ((( $4,527.78 )))
With interest of $1,832.00 and a principal of $16,000 for 206 days, using the ordinary interest method, the rate is: ((( 20% )))
The number of days between May 20 and November 22 is: ((( 186 )))
Sue Gastineau borrowed $17,000 from Regions Bank at a rate of 5.5% to open her lingerie shop. The date of the loan was March 5. Sue hoped to repay the loan on September 19. Assuming the loan is based on ordinary interest, Sue will pay back how much in interest expense? ((( $514.25 )))
On May 17, Jane took out a loan for $33,000 at 6% to open her law practice office. The loan will mature the following year on January 16. Using the ordinary interest method, what is the maturity value due on January 16? ((( $34,342 )))
Round to nearest cent: Principal Interest Rate Time Simple Interest Total Amount Owed $32,000 7% 3 yrs A B ((( "A. $6,720
B. $38,720
" )))
Round to nearest cent: Principal Interest Rate Time Simple Interest Total Amount Owed $18,000 9% 4 mos A B ((( "A. $540
B. $18,540
" )))
Use ordinary interest: Principal Interest Rate Date Borrowed Date Repaid Time Simple Interest Amount Paid Back $9,000 11% Apr 20 Aug 8 A B C ((( "A. 110 days
B. $302.50
C. $9,302.50
" )))
Use exact interest. Round to nearest cent:Principal Interest Rate Date Borrowed Date Repaid Time Simple Interest mount Paid Back $15,000 12% May 12 Sept 16 A B C ((( "A. 127 Days
B. $626.30
C. $15,626.30
" )))
Find the adjusted balance (principal) using the U.S. Rule (360 day) after the first payment. Rate Total Time for Note Note Amount Partial Payment on Day 30 Partial Payment on Day 90 11% 120 days $7,000 $900 $1,200 ((( $6,164.17 )))
Round to nearest cent or hundredth percent as needed: Principal Rate Time Simple Interest ? 11% 2 mos $1,250 ((( $68,181.94 )))
Interest is $405 on a principal balance of $5,000; assuming a 7 month loan, what is the rate? Round to the nearest cent or hundredth percent as needed. ((( 13.89% )))
On May 19, Bette Santoro borrowed $3,000 from Resse Bank at a rate of 12½%. The loan is to be repaid on October 8. Assuming the loan is based on exact interest, what is the total interest cost to Bette? ((( $145.89 )))
Jones of Boston borrowed $40,000, on a 90-day 10% note. After 60 days, Jones made an initial payment of $6,000. On day 80, Jones made an additional payment of $7,000. Assuming the U.S. Rule, what is the adjusted balance after the second payment? Use 360 days. ((( $27,859.26 )))
Amy Koy met Pat Quinn on Sept. 8 at Queen Bank. After talking with Pat, Amy decided she would like to consider a $9,000 loan at 10 1/2% to be repaid on Feb. 17 of the next year on exact interest. Calculate the amount that Amy would pay at maturity under this assumption. Round all answers to the nearest cent. ((( $9,419.42. )))
Find A and B in the table below. Principal Interest Rate Time Simple Interest Total Amount Owed $25,000 10% 2 yrs A B ((( A. $5,000; B. $30,000 )))
Find A and B in the table below. Principal Interest Rate Time Simple Interest Total Amount Owed $16,000 8% 3 mos A B ((( A. $320 B. $16,320 )))
Find A and B in the table below. Principal Interest Rate Time Simple Interest Total Amount Owed $18,000 9% 3 yrs A B ((( A. $4,860 B. $22,860 )))
Find A and B in the table below. Principal Interest Rate Time Simple Interest Total Amount Owed $22,000 12% 8 mos A B ((( A. $1,760 B. $23,760 )))
Use ordinary interest: Principal Interest Rate Date Borrowed Date Repaid Time Simple Interest Amount Paid Back $8,000 12% May 5 Aug 10 A B C ((( A. 97 Days; B. $258.67; C. $8,258.67 )))
Use ordinary interest: Principal Interest Rate Date Borrowed Date Repaid Time Simple Interest Amount Paid Back $12,000 9% Apr 5 Aug 9 A B C ((( A. 126 days; B. $378; C. $12,378 )))
Use exact interest: Principal Interest Rate Date Borrowed Date Repaid Time Simple Interest Amount Paid Back $12,000 14% Jun 8 Oct 7 A B C ((( A. 121 days; B. $556.93; C. $12,556.93 )))
Use exact interest: Principal Interest Rate Date Borrowed Date Repaid Time Simple Interest Amount Paid Back $24,000 12% May 4 July 9 A B C ((( A. 66 days; B. $520.77; C. $24,520.77 )))
Given: a 12% 90-day $4,000 note. Find the adjusted balance (principal) using the U.S. Rule (360 days) after the first $800 payment on the 40th day. ((( $3,253.33 )))
Given: an 11% 120-day $9,000 note. Find the adjusted balance (principal) using the U.S. Rule (360 days) after the first payment on the 65th day of $1,000. ((( $8,178.75 )))
Solve: Principal Rate Time Simple Interest ? 12% 2 mos $960 ((( $48,000 )))
Solve: Principal Rate Time Simple Interest $6,000 ? 6 mos $330 ((( 11% )))
Solve: Principal Rate Time (in years) Simple Interest $5,000 5% ? $800 ((( 3.2 years )))
Solve: Principal Rate Time (in days using ordinary interest) Simple Interest $60,000 12% ? $3,600 ((( 180 days )))
Abby borrowed $3,000 at 12 3/4% on Sept. 10. The loan is due on Jan. 29. Assuming the loan is based on ordinary interest, how much will Abby pay on Jan. 29? ((( $3,149.81 )))
Louis Carroll visits his local bank to see how long it will take for $1,000 to amount to $1,900 at a simple interest rate of 12 1/2%. A. How long will it take in years? B. How long will it take in months? ((( A. 7.2 years; B. 86.4 months )))
Alice took out a loan for $19,500 at 13 1/2% on Nov 4, 2018 which will be due on January 14, 2019. Using ordinary interest, what will be the interest cost and what amount will Alice pay back on January 14, 2019? ((( $519.19; $20,019.19 )))
Bruce Seem took out the same loan as Alice in the preceding problem, but his terms were exact interest. What is the difference in interest cost and what will Bruce pay back on January 14, 2019? Who had better terms and why? ((( $7.11; $20,012.08; Bruce had better terms because his interest owed is less. )))
Ben Young borrowed $5,000 on April 19 from Reliance Bank at a rate of 6.75%. Ben must repay the loan on December 16 of the same year. Assuming the loan is based on exact interest, what is the total interest cost? ((( I = $222.84 )))
Calculate the following: Principal Rate Time (in years) Simple Interest $55,000 6.25% ? $5,156.25 ((( 1.5 years )))
A percent is the result of a number being expressed as a fraction of 100. ((( TRUE )))
40% is 40 parts out of 1,000. ((( FALSE )))
To convert a decimal to a percent, the decimal point is moved two places to the left. ((( FALSE )))
0.83 in percent equals 83%. ((( TRUE )))
0.532 in percent equals 5.32%. ((( FALSE )))
0.009 in percent equals .9%. ((( TRUE )))
0.6% means 6/10 of 10%. ((( FALSE )))
0.5% means 5/10 of 1%. ((( TRUE )))
Numbers greater than 1% in decimal will have at least two zeros in front of the number. ((( FALSE )))
8/10 of 1% is equal to 0.008. ((( TRUE )))
To convert a decimal to percent, divide the number by 100 and add the % sign. ((( FALSE )))
To convert a percent to a decimal, drop the percent sign and move the decimal point two places to the left. ((( TRUE )))
72.4% in decimal equals 0.0724. ((( FALSE )))
8 3/4% in decimal equals 0.00875. ((( FALSE )))
6 1/2% in decimal equals 0.065. ((( TRUE )))
1/4 in percent is equal to 250%. ((( FALSE )))
3/20 in percent is equal to 15%. ((( TRUE )))
To convert a percent to a fraction, attach the percent sign and divide by 100. ((( FALSE )))
33% converted to a fraction is 33/100. ((( TRUE )))
1/7% converted to a fraction is 1/700. ((( TRUE )))
139% converted to a fraction is 1 39/1,000. ((( FALSE )))
12 1/2% is equal in fraction to 3/8. ((( FALSE )))
5 1/3% is equal in fraction to 4/75. ((( TRUE )))
The base of the portion formula represents the whole or 100%. ((( TRUE )))
The rate in the portion formula is always expressed with a percent symbol. ((( FALSE )))
The rate in the portion formula could be expressed as a percent, decimal, or fraction. ((( TRUE )))
The portion is sometimes expressed with a percent symbol. ((( FALSE )))
The portion can never be larger than the base. ((( FALSE )))
The portion is equal to the base divided by the rate. ((( FALSE )))
A rate of 40% could be expressed as 2/5. ((( TRUE )))
In many problems the base is found before the word "of." ((( FALSE )))
The rate can be found by dividing the base into the portion. ((( TRUE )))
The base is calculated by dividing the portion into the rate. ((( FALSE )))
To calculate a percent increase, the portion is the missing element. ((( FALSE )))
A percent decrease can never be more than 50%. ((( FALSE )))
To convert a decimal to a percent you: ((( Move the decimal point two places to the right and attach a percent symbol. )))
To convert a percent to a decimal: ((( Drop percent symbol and move decimal point two places to left. )))
To convert a percent to a fraction, which of the following is not done? ((( Divide by 1/100. )))
The rate in the portion formula is equal to:In the percent increase or decrease problems, the portion is: ((( Portion divided by base. )))
In the percent increase or decrease problems, the portion is: ((( Amount of increase or decrease. )))
0.65 converted to percent is: ((( 65%. )))
102% converted to decimal is: ((( 1.02. )))
0.0066% converted to decimal is: ((( 0.000066. )))
93/100 converted to percent equals: ((( 93%. )))
170% converted to a fraction equals: ((( 17/10. )))
9 3/4% converted to a fraction equals: ((( 39/400. )))
160% of 1,825 is: ((( 2920.00 )))
750 is what percent of 900? (Round to nearest tenth of a percent.) ((( 83.3%. )))
1,440 is 120% of ________: ((( 1200.00 )))
An LCD flat screen TV at Best Buy increased in price from $900 to $1,200. What was the percent of increase? ((( 33 1/3%. )))
The price of a hairpiece increased from $500 to $550. What was the percent of increase? ((( 10%. )))
The price of an iPod dropped from $299.99 to $180.55. What was the percent decrease in price? (Round to nearest hundredth percent.) ((( 39.81%. )))
Pete Moran bought a Dell laptop computer priced at $699. He put down 30%. The amount of the down payment is: ((( $209.70. )))
Lee Wong receives an annual salary of $65,000 from CVS Pharmacy. Today his boss informs him that he will be getting a $3,000 raise. The percent increase rounded to the nearest tenth of a percent is: ((( 4.6%. )))
Barnes & Noble bookstore ordered 800 art books. On verifying the order, only 160 books were actually received. The percent of the order missing was: ((( 80%. )))
Bill Space gets a hit 3 out of each 10 times at bat. The percent of the times he fails to get a hit is: ((( 70%. )))
Jim Debt was reviewing the total accounts receivable. This month he received $80,000 from credit customers. This represented 40% of all receivables due. The total amount of accounts receivable due is: ((( $200,000. )))
Joe Company in 2017 had sales of $800,000. In 2018 sales were up 75%. Calculate the sales for 2018. ((( $1,400,000. )))
Granite College has 1,200 female students. This represents 40% of the total student body. The number of male students at Granite is: ((( None of these. )))
Lee, the manager of Lexington soccer team, was trying to figure the percent of games the team lost this season. Their record was 30 wins and 10 losses. What percent of the soccer games did they lose? ((( 25%. )))
Mel Company has a net income, before taxes, of $95,000. The treasurer of the company estimates 45% of net income will have to be paid for federal and state taxes. The tax for both federal and state is: ((( $42,750. )))
0.616 converted to a percent is: ((( 61.6%. )))
1,620 is 120% of ________: ((( 13,500. )))
Ford Motor Company sells the Fusion Hybrid SE for $26,340. The Fusion Titanium sells for $30,490. What is the percentage increase in price for the Titanium over the Fusion SE? (Round to tenth.) ((( 15.8%. )))
Jamie earned $25,000 as an early childhood teacher in the Lindbergh School District. She was promoted to a sixth grade teaching position that included an 80% raise. What will she earn as a new sixth grade teacher? ((( $45,000. )))
"Convert to percent: .005
" ((( .5% )))
Convert to percent: 7.853 ((( 785.3% )))
Convert to decimal: 1/5% ((( .002 )))
Convert to decimal: 5 3/4% ((( .0575 )))
Convert to percent (to nearest tenth of a percent): 5/6 ((( 83.3% )))
Convert to percent: 4/5 ((( 0.80 )))
Convert to fraction (round to lowest terms as needed): 1/8% ((( 1/800 )))
Convert to fraction (round to lowest terms as needed): 81 1/4%. ((( 13/16 )))
Solve for portion: 12% of 600 ((( 72.00 )))
Solve for portion: 40% of 12,000 ((( 4800.00 )))
"Solve for rate (round to nearest tenth percent): 66 is ________% of 900
" ((( 7.3% )))
Solve for base (round to nearest tenth): 67 is 30% of ________ ((( 223.3 )))
The price of a hamburger increased from $1.10 to $1.59. What percent did the rice increase? (Round to nearest hundredth or hundredth percent as needed.) ((( 44.55% )))
Juan Sevez buys a new computer priced at $650. He makes a down payment of 15%. How much of the purchase is not paid for? (Round to nearest hundredth or hundredth percent as needed.) ((( $552.50 )))
Of 3,600 students surveyed, 40% eat cold pizza for breakfast. How many students do not eat cold pizza for breakfast? (Round to nearest hundredth or hundredth percent as needed.) ((( 2160.00 )))
Natalie Five puts down $800 on a car. Natalie still owes 90% of the purchase price. What was the price she bought the car for? (Round to nearest hundredth or hundredth percent as needed.) ((( $8,000 )))
Convert the following decimal to percent: .193 ((( 19.3% )))
Convert the following decimal to percent: 9 ((( 9.00 )))
Convert the following decimal to percent: 15.82 ((( 1,582% )))
Convert the following decimal to percent: .007 ((( .7% )))
Convert the following percent to a decimal: 12% ((( .12% )))
Convert the following percent to a decimal: 2.48% ((( .0248 )))
Convert the following percent to a decimal: 400% ((( 4.0 )))
Convert the following percent to a decimal: 1/4% ((( .0025 )))
Convert the following fraction to percent. (Round to the nearest tenth percent): 1/7 ((( 14.3% )))
Convert the following fraction to percent. (Round to the nearest tenth percent): 12/7 ((( 171.4% )))
US Airways has a net Income before taxes of $60,000. It is expected that 35% of the net income will go to federal and state taxes. How much will US Airways have left? ((( $39,000 )))
North Conway Ski Shop ordered 300 pairs of skis. When the shop received the order, 60 pairs of skis were missing. What percent of the order did the shop receive? ((( 80% )))
Joy Lindman, who works for GE, receives an annual salary of $85,000. Today she was informed by her boss that she will be receiving a $3,000 raise. What percent of her old salary is the $3,000 raise? (Round to nearest hundredth percent.) ((( 3.53% )))
The price of a Delta Airlines ticket from New York to Orlando increased to $450. This is an 8% increase. What was the old fare to the nearest cent? ((( $416.67 )))
Convert .194 to a percent: ((( 19.4% )))
9.15% in decimal is equal to: ((( .0915 )))
Convert .895 to a percent: ((( 89.5% )))
Convert 12/13 to a percent (round to hundredth percent): ((( 92.31% )))
Convert 24/15 to a percent: ((( 160% )))
Convert 18.54% to a fraction (reduce): ((( 927/5,000 )))
80% of what number is 50? ((( 62.5 )))
900 is what percent of 3,600? ((( 0.25 )))
22% of 80 is what? ((( 17.6 )))
42% of 900 is what? ((( 378.00 )))
200 is what percent of 1,000? ((( 0.20 )))
60% of what number is 180? ((( 300.00 )))
Complete: PRODUCT Cost in 2017 Cost in 2018 Amount of $ Inc/Dec Percent Change (round to hundredths) Textbook $21.50 $25.60 A B ((( A. $4.10; B. 19.07% )))
Complete: PRODUCT Cost in 2017 Cost in 2018 Amount of $ Inc/Dec Percent hange (round to hundredths) Time $15.50 $18.25 A B" ((( A. $2.75; B. 17.74%. )))
150 is 60% of ________? ((( 250.00 )))
21 is 6% of ________? ((( 350.00 )))
800 is 5 1/4% of ________ (round to nearest tenth)? ((( 15,238.1 )))
1,200 is 55 1/4% of ________? ((( 2,171.95 )))
Out of 3,000 students surveyed, 600 preferred cheese pizza over mushroom. What percent of students preferred cheese pizza? ((( 0.20 )))
Out of 5,000 students surveyed, 1,000 preferred diet drinks over sugared sodas. What percent preferred nondiet drinks? ((( 0.80 )))
At Y University, it is estimated that 42% of all students are from out of state. The current enrollment is 1,400. What is the number of in-state students? ((( 8.12 )))
Earl Miller receives an annual salary of $80,000. Today, his boss, Jay, informs him he will be getting a 4% raise. What will his salary be after the raise? ((( $83,200 )))
The cost of a video game increased in price from $20 to $39. What was the percent of the increase? ((( 0.95 )))
The price of a calculator decreased from $60 to $15. What was the percent of the decrease? ((( 0.75 )))
The sales tax in Massachusetts is 6.25%. Joanne bought a wood stove with a sales tax of $15. What was the cost of the wood stove before tax? ((( $240 )))
Amy Gridley earns a gross pay of $500. If payroll deductions are 28%, what is her take-home pay? ((( 360.00 )))
At the end of the season, the Marblehead Indians had 18 wins and 4 losses. What percent of the games did the team win? (Round to the nearest hundredth percent.) ((( 81.82% )))
At Acorn College the tuition has increased 20% for each of the last two years. If tuition this year is $17,280, what was the tuition two years ago? (Round all answers to nearest dollar.) ((( $12,000 )))
In 2018 North Shore Community College had a total student body that was 5% more than in 2017, which was 5% more than in 2016. The enrollment in 2018 was 4,200. How many students attended the college in 2017? How many students attended the college in 2016? (Round answer to nearest student.) ((( 4000; 3810 )))
Convert to a decimal: 1/6% (round decimal to the nearest ten thousandths): ((( .0017 )))
22 is 72% of ________ (round to the tenth): ((( 30.6 )))
Merck Company has downsized by firing 22,386 employees because of prolonged recessionary conditions. This represents 18% of its global workforce. What was the total global workforce before the layoffs? ((( 124,367 )))
Ann earns $3,900 per month as a gym teacher in California. Her state, local, and federal taxes represent 53% of her monthly gross income. What amount does Ann take home after taxes are deducted from her salary? ((( $1,833 )))
An equation represents a math expression of equality. ((( TRUE )))
4 + 3 = 9 is an equation of equality. ((( FALSE )))
Variables can only be numbers. ((( FALSE )))
If there is no number in front of a letter, it is really the number 1. ((( TRUE )))
AB or A(B) means A divided by B. ((( FALSE )))
C/D means C divided by D. ((( TRUE )))
Whatever you do to one side of an equation, you must do to the other. ((( TRUE )))
A formula is not an equation. ((( FALSE )))
Variables and constants are terms of mathematical expressions. ((( TRUE )))
B + 8 = 9; B = 17. ((( FALSE )))
B - 11 = 40; B = 29. ((( FALSE )))
When an equation uses addition, the variable can be solved by the opposite process: subtraction. ((( TRUE )))
B/30 = 9; B = 270. ((( TRUE )))
B/5 + 3 = 15; B = 16. ((( FALSE )))
20(C - 9) means the 20 is multiplied only times the C to clear the parentheses. ((( FALSE )))
2(V - 8) = 90; V = 53. ((( TRUE )))
6H + 2H - 20 = 44; H = 8. ((( TRUE )))
In word problems, the word "of" indicates an equal sign. ((( FALSE )))
More than one unknown in a word problem can be represented in terms of the same variable assigned to the first unknown. ((( TRUE )))
There is only one set way to solve all word problems. ((( FALSE )))
If there are two unknowns, the variable is always assigned to the larger value. ((( FALSE )))
1/4B − 10 = 7; B = 67. ((( FALSE )))
5(Z + 60) = 900; Z = 120. ((( TRUE )))
A letter or a symbol that represents one or more numbers is a: ((( Variable )))
An equation is: ((( A math expression of equality )))
A times C is represented by: ((( AC )))
C + 100 = 55; C equals: ((( -45 )))
Q + 92 = 128; Q equals: ((( 36.00 )))
8G = 96; G equals: ((( 12.00 )))
B/8 = 90; B equals: ((( None of these )))
N/8 + 6 = 58; N equals: ((( 416.00 )))
2,000(A - 5) = 110,000; A equals: ((( 60.00 )))
What number decreased by 630 equals 1,510? ((( 2140.00 )))
What number increased by 3,080 equals 5,082? ((( 2002.00 )))
Jane sells 8 times as many Volvos as Melissa. If the difference in their sales is 35, how many cars did Jane sell? ((( 40.00 )))
Staples has a discount on flash drives and boxes of computer paper. It offers flash drives for $6 each and boxes of computer paper for $40 each box. If total sales on these discount items is $4200 for the week, and customers bought 5 times as many flash drives as boxes of computer paper, how many boxes of computer paper were sold? ((( 60.00 )))
A local Walmart sells sweatpants ($7) and jackets ($14). If total sales were $6,160 and customers bought 8 times as many sweatpants as jackets, what would be the number of jackets sold? ((( 88.00 )))
Coffee costs $12 per case, and tea costs $8 per case. If an order comes in for a total of 250 cases for $2,400, what was the specific number of cases of tea? (Hint: Let C = cases of coffee.) ((( 150.00 )))
Dolls cost $140 per carton, and trucks cost $430 per carton. If an order comes in for a total of 100 cartons for $28,500, what was the number of cartons of dolls? (Hint: Let T = cartons of dolls.) ((( 50.00 )))
Belts cost $160 per carton, and shoes cost $465 per carton. If an order comes in for a total of 88 cartons for $19,265, what is the number of cartons of belts? (Hint: Let S = cartons of belts) ((( 71.00 )))
Marvin Co. sells pens ($6) and flashlights ($10). If total sales were $624 and customers bought 7 times as many pens as flashlights, what would be the number of pens sold? ((( 84.00 )))
Jill Co. sells thermometers ($2) and hot water bottles ($6). If total sales were $312 and customers bought 10 times as many thermometers as hot water bottles, what would be the number of hot water bottles sold? ((( 12.00 )))
In Ryan Corporation, the first shift produced 5 1/2 times as many light bulbs as the second shift. If the total light bulbs produced were 16,250, how many light bulbs were produced on shift 1? ((( 13750.00 )))
In Johnson Corporation, the first shift produced 8 1/2 times as many light bulbs as the second shift. If the total light bulbs produced were 24,700, how many light bulbs were produced on shift 2? ((( 2600.00 )))
7G = 112; G equals: ((( 16.00 )))
P + 77 = 101; P equals: ((( 24.00 )))
Scholastic Books sells 4 times as many textbooks as Keller Publishing; together they sold 7,775 textbooks. How many did Scholastic sell? ((( 6220.00 )))
Solve the following: ((( 77.00 )))
Solve for the unknown, showing all work. (If necessary, round to the nearest hundredth.) A + 80 = 165 ((( 85.00 )))
Solve for the unknown, showing all work. (If necessary, round to the nearest hundredth.) 1,105 = B + 82 ((( 1023.00 )))
Solve for the unknown, showing all work. (If necessary, round to the nearest hundredth.) Q/40=20 ((( Q = 800 )))
Solve for the unknown, showing all work. (If necessary, round to the nearest hundredth.) 6B - 80 = 160 ((( B = 40 )))
Solve for the unknown, showing all work. (If necessary, round to the nearest hundredth.) 7 (A - 4) = 21 ((( A = 7 )))
Solve for the unknown, showing all work. (If necessary, round to the nearest undredth.) 8A - 40 = 4A + 60 ((( A = 25 )))
Over the past month, 900 trucks were sold. Abe sold 4 times as many as Joe. How many did each sell? A. Number of trucks Abe sold. B. Number of trucks Joe sold. ((( "A. 720
B. 180
" )))
A furniture company produced 5 times as many beds on shift 2 as it did on shift 1. If a otal of 240 beds were produced, how many were produced on each shift? A. Number produced on Shift 1. B. Number produced on Shift 2. ((( "A. 40
B. 200
" )))
1/9 of all sales were for cash. If cash sales were $59,000, what were the total sales? ((( 531000.00 )))
One day Marika Company produced 6 times as many wood stoves as Barry Company. If the difference in their production is 15, how many stoves did Barry produce that day? ((( 3.00 )))
Runyon Company sells T-shirts ($5) and shorts ($6). If total sales were $1,040 and people bought 4 times as many T-shirts as shorts, what would be the number of T-shirts and shorts sold? A. Number of T-shirts sold. B. Number of shorts sold. ((( "A. 160
B. 40
" )))
Erasers cost $6 per carton, and pencils cost $7 per carton. If an order comes in for a total of 16 cartons for $100, what was the number of cartons of each bought? Hint: Let erasers = (16 - p) A. Number of pencil cartons. B. Number of eraser cartons. ((( "A. 4
B. 12" )))
American Airlines reduced its round trip fare from Boston to New York by $45. The sale price was $139. What was the original price? ((( 184.00 )))
If Sears Auto Center sells six times as many batteries as Firestone and their difference in sales is 250,000 batteries, how many batteries did each sell? ((( Sears: 300,000; Firestone: 50,000 )))
Jim Murray and Al Sming sold a total of 600 camcorders. Jim sold 5 times as many amcorders as Al. How many did each sell? ((( # Jim sells: 500; # Al sells: 100 )))
Peter Mabet sold a total of 400 oatmeal ($1.50) and wheat ($2) breads during the Labor Day weekend. How many of each did Peter sell if total sales were $700? Check your answer. (Let Wheat = W.) ((( Number of wheat breads: 200; Number of oatmeal breads: 200 )))
A + 20 = 55 ((( 35.00 )))
1,080 = B + 30 ((( 1050 )))
7B - 70 = 140 ((( B = 30 )))
8 (A - 5) = 70 ((( 13.75 )))
10A - 20 = 4A + 80 (answer to closest hundredth). ((( 16.67 )))
B + 40 = 90 ((( 50.00 )))
2,015 = Q + 90 ((( 1925 )))
5A - 10 = 90 ((( 20.00 )))
10 (C - 7) = 30 ((( 10.00 )))
20A - 40 = 8A + 160 (answer to nearest hundredth). ((( 16.67 )))
Over the past month, 700 trucks were sold. Abe sold 6 times as many as Joe. How many did each sell? A. Number Abe sold. B. Number Joe sold ((( "A. Abe sold 600
B. Joe sold 100
" )))
A furniture company produces 4 times as many beds on shift 2 as on shift 1. If a total of 1,000 beds were produced, how many were produced on each shift? A. Number produced on Shift 1. B. Number produced on Shift 2. ((( "A. Shift 1 produced 200
B. Shift 2 produced 800
" )))
1/7 of all sales were for cash. If cash sales were $28,000, what were the total sales? ((( "Total Sales
28000
" )))
Runyon Co. sells T-shirts ($4) and shorts ($5). If total sales were $850 and people bought 3 times as many T-shirts as shorts, what would be the number of T-shirts and shorts sold? A. Number of T-shirts. B. Number of shorts. ((( "A. 150 T-shirts
B. 50 shorts
" )))
Erasers cost $5 per carton, and pencils cost $7 per carton. If an order comes in for a total of 15 cartons for $85, what number of cartons of each was bought? (Hint: Let cartons of pencils = P.) ((( "A. 5 Cartons of pencils - B. 10 Cartons of erasers ))
B. 10 Cartons of erasers
" )))
The Rand Co. produces 11 times as many rings on shift 1 as on shift 2. If a total of 12,000 rings were produced, how many were produced on each shift? A. Number produced on shift 1. B. Number produced on shift 2. ((( A. 11,000 B. 1,000 )))
Over the past month, 900 telephones were serviced. If Jill services 8 times as many phones as Rose, how many did each service? A. Number Jill serviced. B. Number Rose serviced ((( A. Jill serviced 800 B. Rose serviced 100 )))
1/5 of all sales were for cash. If cash sales were $4,000, what were total sales? ((( 20000.00 )))
The Office Supply Store sells index card packs ($2) and boxes of pencils ($7). If total sales were $184 and customers bought 8 times as many index card packs as boxes of pencils, what would be the number of each sold? A. Total number of index card packs; B. Total number boxes of pencils ((( A. 64 index card packs B. 8 Boxes of pencils )))
Pens cost $6 per carton, and elastics cost $4 per carton. If an order comes in for a total of 12 cartons for $60, what was the specific number of cartons of pens as well as elastics? (Hint: Let P = Pens) A. Number of cartons of pens; B. Number of cartons of elastics. ((( A. 6 cartons of pens B. 6 cartons of elastics )))
Art Neuner and Paul Kosponato sell trucks for Blue Auto. Over the past year, they sold 240 trucks. Assuming Paul sells 7 times as many as Art, how many trucks did each sell? A. Number of trucks Art sold; B. Number of trucks Paul sold. ((( A. 30 B. 210 )))
/8 of all wood stoves are sold for cash. If cash sales were $2,200 for the week, what was the total of all sales? (Let S = sales.) ((( 17600.00 )))
"Wrenches cost $50 per carton, and hammers cost $150 per carton. If an order comes in for a total of 60 cartons for $5,000, what was the number of cartons of wrenches as well as hammers? (Hint: Let Wrenches = W).
A. Total number of wrenches; B. Total number of hammers.
" ((( "A. 40 cartons
B. 20 cartons
" )))
The cost of oil is 6 times the cost of wood. The total bill for oil and wood is $700. What was the expense for each? A. Cost of oil; B. Cost of wood ((( "A. $600
B. $100
" )))
Mick and Mack went to a supermarket and bought boxes of soap detergent. Mack could buy only four boxes, which was as much as Mick. How many boxes did Mick buy? ((( 20 boxes )))
Jarvis Co. sells thermometers ($3) and hot water bottles ($7). If total sales were $760 and customers bought 4 times as many thermometers as hot water bottles, what would be the number of each sold? (Let B = number of hot water bottles.) A. Number of Thermometers; B. Number of Bottles. ((( "A. 160
B. 40
" )))
At Miduale Electronics, the first shift produced 6 times as many items as the second shift. If total production was 4,900, what was the output for each shift? A. Output from first shift; B. Output from second shift. ((( "A. 4,200 - B. 700)
B. 700
" )))
Melons cost $20 per crate, and apples cost $8 per crate. An order comes in for a total of 48 crates for $420. What was the number of crates of melons and apples? (Let crates of melons = M.) A. Number of crates of melons; B. Number of crates of apples. ((( "A. 3 crates
B. 45 crates
" )))
JCP sold a pair of pants that was reduced by $12.00. The selling price was $48.00. What was the original price? ((( 60.00 )))
Momentum Electronics produces 4 times as many E-readers on shift 1 as it does on shift 2. If a total of 85 E-readers were produced, how many were made on shift 1? ((( 68.00 )))
Lucy and Ayu sold a total of 1,080 iPads. Lucy sold 7 times as many as Ayu. How many did each sell? A. Number Lucy sold; B. Number Ayu sold. ((( "A. 945
B. 135
" )))
Clare Frances sells air fresheners for automobiles for $6.00 and powdered air cans for $4.00. Total sales were $196. Customers bought 4 times as many air fresheners as powdered air cans. How many of each was sold? What was the total dollar value sold for each? A. Number air fresheners; B. Number powdered air cans ((( "A. 28
B. 7
" )))
The number to the right of the decimal is the hundredths place. ((( FALSE )))
A mixed decimal does not contain a whole number. ((( FALSE )))
.3, .30, and .300 have the same value. ((( TRUE )))
A proper fraction is converted to a decimal equivalent by dividing the denominator into the numerator. ((( TRUE )))
The rules of rounding are completely different for decimals compared to whole numbers. ((( FALSE )))
In division, the decimal is never moved in the divisor. ((( FALSE )))
When multiplying decimals, you should disregard the decimals and multiply the whole numbers. In the product, the total of the decimal places from the multiplier and multiplicand is inserted starting at the left and counting the number of places to the right. ((( FALSE )))
0.04 is 4 tenths. ((( FALSE )))
5,000 Mexican pesos equal $630.00 in U.S. currency. (Use Handbook.) ((( FALSE )))
1/15 results in a repeating decimal. ((( TRUE )))
0.000009 is 9 hundred thousandths. ((( FALSE )))
Stock quotations are expressed in decimals. ((( TRUE )))
$600 in U.S. currency is equal to $660 in Canadian currency. (Use Handbook) ((( FALSE )))
0.42 is equal to 42/100. ((( TRUE )))
0.06 3/4 is equal to 675/10,000. ((( TRUE )))
5/6 in decimal form is 0.875. ((( FALSE )))
0.46 rounded to the nearest tenth is 0.5. ((( TRUE )))
75.9914 rounded to the nearest hundredth is 75.99. ((( TRUE )))
0.14382 rounded to the nearest thousandth is 0.144. ((( TRUE )))
In decimal form nine-tenths is .9. ((( TRUE )))
In decimal form nine thousand four hundred eighty-one ten thousandths is 0.09481. ((( FALSE )))
0.817 converted to a fraction is 817/1,000. ((( TRUE )))
4 2/7 converted to a decimal and rounded to the nearest thousandth is 4.285. ((( FALSE )))
1.4 + 0.14 + 20.001 equals 21.451. ((( FALSE )))
5.11 times 6.5 equals 33.215. ((( TRUE )))
275 divided by 1.5 equals 183.33 rounded to the nearest thousandth. ((( FALSE )))
6,690 divided by 10,000 equals 0.6690. ((( TRUE )))
29.41 minus 0.008 equals 29.402. ((( TRUE )))
6.33 times 7.41 = 46.9053. ((( TRUE )))
The decimal equivalent of 1/7 is .143 to the nearest thousandth. ((( TRUE )))
A. J. Ryan bought 500 shares of Google at $1200.55 per share. Assuming no commission, what did Ryan spend? ((( 600275.00 )))
Pumpkins at a local farm sell for $.49 per pound. Jim Ring spent $73.50. How many pounds of pumpkins were purchased? ((( 150.00 )))
Jeff Jones has contracted for an oil delivery price of $1.95 per gallon. His tank holds 210 gallons. If Jeff has six fill-ups during the winter, what will be his total cost? ((( 2457.00 )))
North Shore Community College reimburses faculty members $.535 cents per mile to go to a workshop. Professor Wales submitted her travel log for a total of 650.11 miles. What reimbursement can Professor Wales expect? (Round to the nearest cent.) ((( 347.81 )))
The Weather Channel reported that normal snowfall is 138.44 inches for Lexington County. This winter the following snowfall occurred:How much was the snowfall below normal? ((( 60.682 inches )))
Dave Adams bought season tickets for the Boston Celtics. The cost was $2,410.20 for 30 home games. What is the average price of the tickets per game? Round to the nearest cent. ((( 80.34 )))
Printed pencils cost $0.33 1/3 each for an order of 18,000 pencils. On Monday Jim Company placed an order for 18,000 pencils. What is the exact cost of the pencils for Jim Co.? Be sure to use the fractional equivalent in your calculation. ((( 6000.00 )))
If Jan sells 2,500 glow sticks on the Fourth of July at $1.99 each, what will her total profit be if each stick cost her $0.88? ((( 2775.00 )))
Joyce went to Lowes to buy carpeting. She will need 88.4 sq. yards for downstairs, 15.1 sq. yards for the halls, and 177.18 sq. yards for the upstairs bedrooms. She chose a commercial carpet that costs $18.11 per sq. yard. Joyce ordered foam padding at $3.25 per sq. yard. The installers quoted Joyce a labor cost of $4.25 per sq. yard for installation. What will the total job cost Joyce? ((( 7188.21 )))
Abby is touring China for four days. The car rental is $38.15 per day plus 12 1/2 cents per mile. If Abby drives 380.5 miles, how much does she owe the rental company? ((( 200.16 )))
B. Brady signed a new basketball contract for the year for $1,005,000. Assuming a schedule of 13 games, what are his earnings per game? ((( 77307.69 )))
Nancy Billows bought 150 shares of NBC Universal stock at $22.885 per share. Her broker charged her $22.05 for the brokerage commission. What is the average cost per share (include brokerage fee) to the nearest cent? ((( 23.03 )))
Melvin Small typed 485 business letters. The cost of writing these letters was estimated to be $3,455.25. What was the average cost of a letter to the nearest cent? ((( 7.12 )))
Jill traveled to Canada and bought a new lamp for $460 in Canadian currency. In U.S. dollars (to the nearest cent) this is equivalent to: (Use Handbook) ((( 365.47 )))
Jim sells college textbooks. To date Jim has sold $141,002 worth of textbooks. His sales quota is $194,159. Tomorrow he expects to sell $15,005 worth of textbooks to B College. His last stop will be Munroe College. If he sells what he expects to at B College, what must he sell at Munroe to meet his quota? ((( 38152.00 )))
Jen bought 10,000 shares of a stock valued at $32.83 per share. How much did she pay in total? ((( 328300.00 )))
Best Buy sold 638 iPads during the month of May for a total, including tax, of $296,963.48; what was the average cost to the consumer? ((( 465.46 )))
The cost to educate a child in grade school is $8,917.45 per school year. What is the cost to educate 1000 children? ((( 8917450.00 )))
Convert to proper fraction or mixed number (do not reduce to lowest terms): .8 ((( 8/10 )))
Convert to proper fraction or mixed number (do not reduce to lowest terms): 5.27 ((( 5 27/100 )))
Convert to proper fraction or mixed number (do not reduce to lowest terms): .851 ((( 851/1,000 )))
Convert fraction to decimal (or mixed decimals and round to nearest hundredth): 5/7 ((( .71 )))
Convert fraction to decimal (or mixed decimals and round to nearest hundredth): 1/7 ((( .14 )))
Convert fraction to decimal (or mixed decimal and round to nearest hundredth): 13 6/7 ((( 13.86 )))
Subtract: 9.1581 - 2.85 ((( 6.3081 )))
Multiply (round solution to nearest hundredth): 9.158 × 14.382 ((( 131.71 )))
Divide (round to nearest tenth): 118,000 ÷ 3.95 ((( 29,873.4 )))
Complete by shortcut method: 43.81 × 1,000 ((( 43,810 (move decimal three places to the right) )))
Complete by shortcut method: 11,896.413 × 1,000 ((( 11,896,413 )))
Complete by shortcut method:3,069.44 ÷ 1,000 ((( 3.06944 )))
Jim Smith has two options in buying an iPad. He could pay cash of $225.45 or $30 down and 15 monthly payments of $18.75. If he pays cash, how much does he save? ((( 85.80 )))
Rene Roller bought a new computer for $1,499. What would the same computer cost in Canada? (Round to the nearest cent.) Use table in Handbook. ((( $1499 × $$1.2587 = $1,886.79 )))
Mel Doane is taking his office staff out to lunch. He has left on his credit card a spending limit of $99.50. If the total meal includes two fruit cups, two onion soups, two veal, two roast beef, three coffees, and one soft drink, will Mel be able to use the card? (Disregard tip plus tax.) If so, how much more charging will Mel be allowed before hitting his credit limit? ((( Yes. $40.75 )))
Total the following: Six hundred sixty-eight and eight hundred one thousandths Twelve and forty-nine hundredths Three and four thousandths Fifty-one hundredths Three hundred ten and four tenths ((( 995.205 )))
Convert decimal fraction to decimal: 8/100 ((( .08 )))
Convert decimal fraction to decimal: 19/100 ((( .19 )))
Convert decimal fraction to decimal: 16/1,000 ((( .016 )))
Convert to proper fraction or mixed number (do not reduce to lowest terms): .6 ((( 6/10 )))
Convert to proper fraction or mixed number (do not reduce to lowest terms): 3.88 ((( 3 88/100 )))
Convert to proper fraction or mixed number (do not reduce to lowest terms): .823 ((( 823/1,000 )))
"Convert fraction to decimal (or mixed decimal) and round to nearest hundredth as needed: 4/7
" ((( .57 )))
Convert fraction to decimal (or mixed decimal) and round to nearest hundredth as needed: 2/9 ((( .22 )))
Convert fraction to decimal (or mixed decimal) and round to nearest hundredth as needed: 4 6/7 ((( 4.86 )))
Convert fraction to decimal (or mixed decimal) and round to nearest tenth as needed: 1/3 ((( .3 (.33) )))
"Rearrange and add: 3.5, 10.86, 7.482, 150.0413, 89.1
" ((( 260.9833 )))
Subtract and round to nearest tenth: 13.685 - 4.12 ((( 9.6 )))
Multiply (round solution to nearest hundredth): 7.541 × 12.943 ((( 97.60 )))
Divide (round to nearest tenth): 338,182 ÷ 6.94 ((( 48,729.4 )))
Complete by the shortcut method: 68.325 × 1,000 ((( 68,325 )))
Complete by the shortcut method: 3,002,118.395 × 100 ((( 300,211,839.5 )))
Professor Burns attended a computer seminar at IBM. The college reimburses Professor Burns at $.41 per mile. Professor Burns traveled 520.4 miles. What will the college pay Professor Burns? (Round to nearest cent.) ((( 213.36 )))
Convert to proper fraction or mixed number (do not reduce to lowest terms): .7 ((( 7/10 )))
Convert to proper fraction or mixed numbers (do not reduce to lowest terms): 4.31 ((( 4 31/100 )))
Convert to proper fraction or mixed numbers (do not reduce to lowest terms): .721 ((( 721/1,000 )))
Convert to proper fraction or mixed numbers (do not reduce to lowest terms): .621 ((( 621/1,000 )))
Convert to proper fraction or mixed numbers (do not reduce to lowest terms): 6.31 ((( 6 31/100 )))
Convert to proper fraction or mixed numbers (do not reduce to lowest terms): .9 ((( 9/10 )))
Convert fraction to decimal (or mixed decimal) and round to nearest hundredth: 4/9 ((( .44 )))
Convert fraction to decimal (or mixed decimal) and round to nearest hundredth: 1/8 ((( .13 )))
Convert fraction to decimal (or mixed decimal) and round to nearest hundredth: 14 5/7 ((( 14.71 )))
Convert fraction to decimal (or mixed decimal) and round to nearest hundredth: 3/7 ((( .43 )))
Convert fraction to decimal (or mixed decimal) and round to nearest hundredth: 2/5 ((( .40 )))
Subtract: 8.1432 - 3.17 ((( 4.9732 )))
Subtract: 12.19432 - 10.005 ((( 2.18932 )))
Multiply (round solution to nearest hundredth): 8.143 × 13.281 ((( 108.15 )))
Multiply (round solution to nearest hundredth): 4.325 × 11.183 ((( 48.37 )))
"Divide (round to nearest tenth): 115,000 ÷ 2.85
" ((( 40,350.9 )))
"Divide (round to nearest tenth): 182,185 ÷ 3.24
" ((( 56,229.9 )))
"Complete by shortcut method: 32.61 ÷ 100
" ((( .3261 )))
Complete by shortcut method: 10,591 ÷ 1,000 ((( 10.591 )))
Complete by shortcut method: 2,059.32 × 1,000 ((( 2,059,320 )))
Complete by shortcut method: 41.82 × 100 ((( 4,182 )))
Complete by shortcut method: 8,194.335 × 1,000 ((( 8,194,335 )))
Complete by shortcut method: 18,915.32 ÷ 1,000 ((( 18.91532 )))
Write the decimal form for the following: Four tenths ((( .4 )))
Write the decimal form for the following: Three ten thousandths ((( .0003 )))
Write the decimal form for the following: One hundred twenty thousandths ((( .120 )))
Write the decimal form for the following: Nine tenths ((( .9 )))
Write the decimal form for the following: Twenty hundredths ((( .20 )))
Write the decimal form for the following: Twelve ten thousandths ((( .0012 )))
Jan Smith bought a new food processor for $49.88. Jan gave the salesperson a $100 bill. What change did Jan receive? ((( $50.12 )))
The state reimburses Bill 55 cents per mile. Bill submitted his travel log for a total of 520.9 miles. What reimbursement can he expect? (Round to nearest cent.) ((( $286.50 )))
Mary is touring Washington, D.C., for three days. The car rental is $19.25 per day plus 21 ½ cents per mile. If Mary drives 362.95 miles, how much does she owe the rental company? ((( $135.78 )))
Alice subscribes to the symphony. The cost is $295.55 for 32 performances. What is the average price per performance? If Alice sells you four tickets, what will they cost you? Use the rounded average cost per ticket in your calculation. (Round answer to nearest cent.) ((( $295.55 ÷ 32 = $9.24 average price $9.24 × 4 = $36.96 cost of 4 tickets )))
Could you help John decide which product is cheaper per ounce? ((( The 79 cent canned fruit is cheaper per ounce. )))
If Bill sells 491,010 kites at $2.95 each, what will his total profit be if each kite cost him $1.98? ((( 476279.70 )))
The cost of a plant is $25.95 for the florist. He purchases 300 of these hanging plants. What is the total cost to the florist? ((( 7785.00 )))
Larry Hess bought a camera in Japan for 33,000 yen. What is the price in U.S. dollars? (Round to the nearest cent.) (Use Handbook) ((( 33,000 yen × 0.009087 = $299.87 )))
Rene Rodrigues vacationed in Mexico and spent 9,200 pesos. What would this be in U.S. dollars? (Round to the nearest cent.) (Use Handbook) ((( $515.20 (9,200 × .0560) )))
The Lange family vacationed in Mexico and spent 12,500 pesos. What would this be in U.S. dollars? (Round to the nearest cent.) (Use Handbook) ((( $700.00 = 12,500 × .0560 )))
Convert to proper fraction: .7191 ((( 7191/10,000 )))
Solve (do not round): 5.25 × 4.617 ((( 24.23925 )))
Jamie wants to buy a new Ford Escape priced at $20,256. If she finances the car with no down payment, her monthly payment will be $455.76 for 48 months. If she pays cash, how much interest expense will she save? ((( $1620.48 )))
A proper fraction is when the numerator is greater than the denominator. ((( FALSE )))
The writing of a whole number and a proper fraction is an improper fraction. ((( FALSE )))
4/5 is a proper fraction. ((( TRUE )))
When a mixed number is converted to an improper fraction, the new numerator is placed over the old denominator. ((( TRUE )))
The greatest common divisor can be zero. ((( FALSE )))
Inspection as well as the step approach could be used to find the least common denominator. ((( TRUE )))
In the step approach the last divisor used is the greatest common divisor. ((( TRUE )))
Fractions should never be reduced to their lowest terms. ((( FALSE )))
The greatest common divisor and the least common denominator are really the same. ((( FALSE )))
The least common denominator of fractions can be found by observation or by the use of prime numbers. ((( TRUE )))
4 is a prime number. ((( FALSE )))
2, 5, 7, 11, and 13 are all examples of prime numbers. ((( TRUE )))
Cancellation is a technique to reduce fractions to the lowest terms. ((( TRUE )))
The reciprocal is not used in dividing fractions. ((( FALSE )))
Raising a fraction to higher terms does change the value of the fraction. ((( FALSE )))
A mixed number is a whole number and a proper fraction. ((( TRUE )))
1 4/5 is an example of a(n): ((( Mixed number )))
13/2 converted to a mixed number is: ((( 6.50 )))
The greatest common divisor of 20/30 is: ((( 10.00 )))
The first step in using the step approach to finding the greatest common divisor is to: ((( Divide the numerator into the denominator )))
The first step in converting 30/50 = ?/200 to higher terms is to: ((( Divide 200 by 50 )))
4/5 + 6/5 equals: ((( 2.00 )))
In adding 4/5 + 18/100 the least common denominator is: ((( 100.00 )))
Which of the following is not a prime number? ((( 24.00 )))
The LCD of 6/20, 9/5, 7/50, and 3/4 is: ((( 100.00 )))
The cancellation method: ((( Is an alternative method to reducing fractions to the lowest terms )))
The reciprocal is used: ((( In dividing fractions )))
Which step is not included in the step approach to calculating the greatest common divisor? ((( Divide larger number into smaller number )))
To find LCD by prime numbers you should: ((( Take denominators and arrange in a row )))
A trip to Portland, Oregon, from Boston will take 7 3/4 hours. Assuming we are two-thirds of the way there, how much longer in hours will the trip take? ((( 2.58 )))
Joe Jackson worked 8 hours on Monday, 4 1/4 hours on Tuesday, 6 1/8 hours on Wednesday, 7 1/4 hours on Thursday, and 8 1/8 hours on Friday. Calculate the total number of hours Joe worked for the week. ((( 33.75 )))
Cartons of humidifiers are stocked in 25,500 sq. ft. of warehouse space at Home Depot. If each carton requires 4 1/4 sq. ft. of space, how many cartons can be stored in this space? ((( 6000.00 )))
At a local Subway, Jill Jones owns 1/4 of the company and Roger Moore owns 1/8. Bill Moore owns the rest. What part is owned by Bill? ((( 5/8 )))
The price of a new Apple iPod has increased by 1/4. If the original price of the Apple was $200, what is the price today? ((( 250.00 )))
The price of a Panasonic 3D flat screen television decreased by 1/5. If the original price was $1,500, what is the price today? ((( 1200.00 )))
Lisa Wolf has 20 1/8 days of vacation per year at Walmart. To date she has taken 4 1/2 days in January, 3 1/4 days in February, and 4 1/8 days in March. How much more vacation time is Lisa entitled to? ((( 8.25 )))
A machine at Staples photocopies 12 1/4 pages per minute. If the machine runs 700 minutes, how many pages will be photocopied? ((( 8575.00 )))
Jeff Jones is paid $80 per day at his job at J.C. Penney. Jeff became ill on Monday and had to leave after 2/5 of a day. What did he earn on Monday? (Assume no work, no pay.) ((( 32.00 )))
The price of a baseball ticket at Yankee Stadium increased by 2 1/4 over the last three years. If the original price of a ticket was $60, what is the price of the ticket today? ((( 135.00 )))
Alice Hall, who loves to cook, makes an apple cake (serves six) for her family. The recipe calls for 2 1/2 pounds of apples, 2 1/4 cups of flour, 1/5 cup of margarine, 1 1/4 cups of sugar, and 4 eggs. Since guests are coming, she would like to make this cake so it will serve 24. How many pounds of apples should she use? ((( 10.00 )))
In a recent local taste contest testing Coke against Pepsi, it was found that 3/5 of all people surveyed preferred the taste of Coke. If 7,500 people were in the survey, how many chose Pepsi? ((( 3000.00 )))
The price of a $200,000 home listed by REMAX was reduced by 1/20. What is the new price? ((( None of these )))
Jane Ring cut a 6-ft. Subway sandwich into 1 1/2-ft. sandwiches. How many sandwiches can be cut from the 6-ft. sub? ((( 4.00 )))
An American Airlines trip from Boston to Los Angeles takes 8 1/2 hours. Assuming we are 1/4 of the way, how long has the trip taken so far? ((( 2.13 )))
The greatest common divisor of 60/216 is: ((( 12 )))
The LCD for 3/10, 20/25, and 18/75 is: ((( 150.00 )))
Jordan traveled 6/7 of an estimated 1,800-mile trip. How many miles remain in her trip? ((( 257.00 )))
The average cost of a ticket to the 2016 World Series Game 1 was $1200. The average cost in 2017 increased by 1/5. What was the cost in 2017? ((( 1440.00 )))
"Indicate type of fraction: 3 4/7
" ((( Mixed Number )))
"Indicate type of fraction: 6/7
" ((( Proper fraction )))
Indicate type of fraction: 10/9 ((( Improper fraction )))
Convert to a mixed number: 89/6 ((( 14.83 )))
Convert to an improper fraction: 14 1/8 ((( 113/8 )))
"Given 18/66, A. Find greatest common divisor (use the step approach or the observation method) and B. Convert to owest terms.
" ((( A. 6; B. 3/11 )))
Convert to higher terms: 8/9 = 96/? ((( 108.00 )))
Add (reduce to lowest terms): 6/15 + 2/15 ((( 8/15 )))
Add (Reduce to lowest terms): 1/7 + 5/14 ((( 7/14 = 1/2 )))
Find LCD by using prime numbers (show work): 1/8 + 1/4 + 1/3 + 1/6 ((( 24.00 )))
Subtract (reduce to lowest terms if necessary): 13 1/7 - 5 5/21 ((( 7.90 )))
Multiply (cancel as needed): 11 3/8 × 6 6/7 ((( 78.00 )))
On a plane trip to Hawaii, the baggage weight projected was 2,182 1/4 lbs. The actual weight of all bags totaled 2,095 1/3 lbs. By how much was the projected weight overstated? ((( 86 11/12 lbs. )))
Acme Track Incorporated received 360 pairs of Nike running shoes. Each pair sells for $58. Acme found 1/9 of the pairs to be defective and returned them. Assuming each pair cost Acme $26, what profit did Acme make assuming all non-defective sneakers were sold? (profit = sales - cost) ((( 10240.00 )))
Indicate type of fraction: 3 ¾ ((( Mixed number )))
Indicate type of fraction: 5/6 ((( Proper fraction )))
Indicate type of fraction: 10/9 ((( Improper fraction )))
Convert to a mixed number: 113/6 ((( 18.83 )))
Convert to an improper fraction: 9 1/8 ((( 73/8 )))
Calculate greatest common divisor by step approach and reduce to lowest terms: 180/440 ((( 9/22 )))
Convert to higher terms: 7/19 = ?/114 ((( 42.00 )))
Find LCD by using prime numbers (show work): 1/2 + 1/6 + 1/8 + 1/4 ((( 24.00 )))
5/9 ÷ 5 = ((( 1/9 )))
At Flynn Manufacturing, 30 1/4 rolls of tape are made each hour on a new high-speed machine. If the machine runs 12 hours, how many rolls of tape will be produced? ((( 363 rolls of tape )))
At Kentucky Fried Chicken, a survey showed 2/3 of all people preferred skinless chicken over the regular chicken. If 2,400 people responded to the survey, how many preferred regular chicken? ((( 800 )))
At United Airlines, Pete Roy worked 8 3/4 hours on Monday, 4 1/2 hours on Tuesday, 9 1/4 hours on Wednesday, 10 1/2 hours on Thursday, and 7 hours on Friday. How many total hours did Pete work during the week? ((( 40.00 )))
The Boston Red Sox announced that the price of their $50 bleacher seats will increase next year by 1/5. What will be the new ticket price? ((( 60.00 )))
Indicate the type of fraction: 3 1/8 ((( Mixed number )))
Indicate the type of fraction : 6/7 ((( Proper fraction )))
Indicate the type of fraction: 12/11 ((( Improper fraction )))
Indicate the type of fraction: 5/6 ((( Proper fraction )))
Indicate the type of fraction: 15/14 ((( Improper fraction )))
Indicate the type of fraction: 12 9/20 ((( Mixed number )))
Convert to a mixed number: 88/7 ((( 12.57 )))
Convert to a mixed number: 77/3 ((( 25.67 )))
Convert to an improper fraction: 12 1/7 ((( 85/7 )))
Given 90/320, A. Find greatest common divisor (use the step approach or the observation method) and B. Convert to lowest terms. ((( A. 10 B. 9/32 )))
Convert to an improper fraction: 11 1/9 ((( 100/9 )))
Given 12/96, A. Find greatest common divisor (use the step approach or the observation method) and B. Convert to lowest terms. ((( A. 12 B. 1/8 )))
8/9 = 72/? ((( 72/81 )))
3/4 = 36/? ((( 36/48 )))
Add (reduce to lowest terms): 4/15 + 1/15 ((( 1/3 )))
Add (reduce to lowest terms): 2/7 + 3/14 ((( 1/2 )))
Add (reduce to lowest terms): 3/7 + 1/21 ((( 10/21 )))
Add (reduce to lowest terms): 4/7 + 13/14 ((( 1.50 )))
Find LCD by using prime numbers (show work): 1/2 + 1/5 + 1/4 + 1/20 ((( 20.00 )))
Find LCD by using prime numbers (show work): 1/3 + 1/4 + 1/6 + 1/8 ((( 24.00 )))
Subtract (reduce to lowest terms if necessary): 12 1/8 - 92/3 ((( 2 11/24 )))
Subtract (reduce to lowest terms if necessary): 14 1/4 - 3 3/4 ((( 10 1/2 )))
Multiply (cancel as needed and express final answer as a mixed number): 12 3/8 , 7 1/6 ((( 88.69 )))
Louis Carroll worked 8 1/2 hours on Monday, 2 3/4 hours on Tuesday, 7 1/2 hours on Wednesday, 7 1/4 hours on Thursday, and 8 hours on Friday. Calculate the total number of hours John worked for the week. ((( 34 hours )))
Al, Ronda, and Rony enter into a partnership. Al owns 1/4 of the company, and Ronda owns 1/8. Calculate the part that is owned by Rony. ((( 5/8 )))
Hilton Hotels announced a price decrease of 1/10 from its $290 weekend package. What is the new weekend package rate? ((( 261.00 )))
Bill Murray has 16 3/4 days of vacation per year. To date, he has taken 1 3/4 days in January, 4 2/3 days in February, and 2 1/6 days in March. How much more vacation time is Bill entitled to? ((( 8 1/6 days )))
A trip to New York from Boston will take 4 1/2 hours. Assuming we are two-thirds of the way there, how much longer will the trip take? ((( 1 1/2 hours )))
The price of a new car increased by 2/3 over the last five years. If the original price of the car was $12,000, what is the price today? ((( 20000.00 )))
Mel Corp. produces 18¼ widgets each hour. If the machine runs 16 hours, how many widgets will be produced? ((( 292.00 )))
Cans of soup are stocked in 1,250 sq. ft. of warehouse space. If each can requires 2 1/2 sq. ft. of space, how many cans of soup can be stored in this space? ((( 500 cans )))
In a recent taste testing survey, it was found that of all people surveyed preferred the taste of "A" chicken over "B" chicken. If 3,500 people were in the survey, how many favored "A"? How many favored "B"? ((( A. 2,500; B. 1,000 )))
Convert to an improper fraction: 16 3/8 ((( 131/8 )))
Indicate what type of fraction: 12 4/7 ((( Mixed number )))
Complete: 10 1/8 divided by 3/8 ((( 27.00 )))
At Truman Middle School, 2/3 of the girls surveyed preferred playing capture the flag at gym. If 600 girls responded to the survey, how many preferred playing something else? ((( 200.00 )))
Reduce the following to the lowest terms: 162/567 ((( 2/7 )))
Sellers' catalogs list only the net price. ((( FALSE )))
A net price equivalent rate of 0.74225 means that for each $10 the buyer's cost will be approximately 74 cents. ((( FALSE )))
A local college bookstore paid a net price of $12,500 for textbooks for the coming semester. The publisher offered a trade discount of 20%. The publisher's original list price was: ((( $15,625 )))
Joanne's Dress Shop received an invoice dated July 25 for $1,400, with terms of 2/10, 1/15, n/60. On August 8, Joanne's Dress Shop sent a partial payment of $750. The actual amount that should be credited is: ((( $757.58 )))
J.C. Penney of Boston sold office equipment for $12,000 to Lee's of San Diego. Terms of the sale are 3/10, n/30 FOB Boston. J.C. Penney has agreed to prepay freight $300. Assuming Lee's pays within the discount period, how much will Lee's pay J.C. Penney? ((( $11,940 )))
Mel's furniture received an invoice dated September 27 for five bedroom sets at $3,000 each. The invoice indicated a trade discount of 5/8/3. The seller of the furniture prepaid the freight of $200. Terms were 2/10 EOM. Assuming Mel pays on November 2, what amount would be paid? (Be sure to include the freight cost.) ((( $12,662.37 )))
Pete's Appliance received a bill dated July 15 for $26,000 with terms of 2/10, 1/15, n/60. On July 27 Pete's sent in a partial payment of $6,000. What is the amount credited as well as the balance due? ((( $6,060.61 amount credited; $19,939.39 balance due )))
Jones Advertising Agency received a $3,000 invoice dated June 8. Terms were 2/10, 1/15, n/60. On June 22 Jones Advertising sent a $1,600 partial payment. What credit should Jones Advertising receive? What is Jones's outstanding balance? Round to the nearest cent. ((( $1,616.16; $1,383.84 )))
Morgan Company received from Lee Company an invoice dated September 27. Terms were 2/10 EOM. List price on the invoice was $5,000 (freight not included). Morgan receives a 9/7 chain discount. Freight charges are Morgan's responsibility, but Lee agreed to prepay the $150 freight charge. Morgan pays the invoice on November 9. What does Morgan pay Lee? ((( $4,296.87 )))
Bill's TV buys a television from a wholesaler with a list price of $600 and a trade discount of 32%. What is the amount of the trade discount and what is the net price of the television? ((( $192; $408 )))
The Blue Manufacturer sells radios to Bob's Discount Outlet. Radios carry a trade discount of 45%. If the price of a radio is $90, what does Bob pay? ((( $49.50 )))
If a manufacturer's list price is $800 and Bill's Outlet buys the goods for $650, what is the trade discount percent? (Round answer to the nearest hundredth percent.) ((( 18.75% )))
Al's Supply sold a chain saw to Mark's Lumber. The list price was $1,250. Al's Supply offered a chain discount of 6/3/1. What was the net price of the chain saw? ((( $1,128.35 )))
A manufacturer of ice skates offered a chain discount of 7/5/1. Bobby's Sport Shop ordered 30 pairs of skates that had a list price of $925. What was the amount of the trade discount? ((( $115.93 )))
Ed's Print Shop received an invoice dated May 10 for $2,500 with terms 3/10, 1/15, n/60. On May 22, Ed's Print Shop sent a partial payment of $2,000. What is the actual amount that should be credited, and what is Ed's Print Shop's outstanding balance? ((( Amount credited: $2,020.20; Outstanding balance: $479.80 )))
"Bloomberg's is offered the following chain discount(s); what is the lowest net price equivalent rate it will pay? Company A Company B 7/9/13 8/6/10
Bill's Furrier marks up mink coats $3,000. This represents a 50% markup on cost. What is the cost of the coats? ((( $6,000 )))
J.C. Penney sells a Timex watch for $139.99 that cost $89.97. J.C. Penney's percent of markup based on the selling price is: ((( 35.73% )))
Zale's bought a tea set for $1,400. Zale's wants to markup the set 55% of the selling price. The selling price of the tea set should be: ((( $3,111.11 )))
Mr. Small, the store manager for Jay's Appliance, is having a difficult time placing a selling price on a refrigerator that cost $410. Mr. Small knows his boss would like to have a 45% markup based on cost. The selling price should be: ((( $594.50 )))
Belle's Bake Shop makes croissants that cost $1.75 each. Past experience shows that 10% of the croissants will spoil and have to be discarded. Assuming Belle wants a 45% markup based on cost and produces 300 croissants, each croissant should sell for: ((( $2.82 )))
A local Dunkin' Donuts makes blueberry muffins that cost $.69 each. Past experience shows that 15% of the muffins will spoil and have to be discarded. Assuming that this shop wants a 30% markup based on cost and produces 200 muffins, each muffin should sell for: ((( $1.06 )))
Fathers' Day suits were advertised for 35% off the regular price. A suit regularly sells for $210. The amount of the markdown is: ((( $73.50 )))
Camille Keegan sells lamps for $105.55 that cost her $75.00. Camille's percent of markup based on the selling price is: ((( 28.94% )))
Kohl's sells watches that cost $6.95 for a selling price of $39.99. What is the percent markup on cost? (Round to the nearest tenth percent.) ((( 475.4% )))
Macy's Department Stores markup men's cologne 63% on cost for an 8-ounce bottle. A bottle of cologne costs Macy's $23.00. What is the selling price for the 8-ounce bottle? ((( $37.49 )))
Johnny Mac's Sporting Goods bought a baseball glove from Rawlings Sporting Goods for $66.00. They want to markup the glove 70% on selling price. What should Johnny's sell the glove for? ((( $220.00 )))
Straub's Bakery makes 200 Danish cakes that cost $2.70 each. Straub's needs a 66% markup on cost and normally discards 10% of what it makes. At what price should Straub's sell the Danish cakes? ((( $4.98 )))
Sullivan's Handbags marks up its bags at 45% of the selling price. Pat Sullivan saw a bag at a trade show that she would sell to her customers for $85.00. What is the most she could pay for the bag and still retain the 45% markup of the selling price? ((( $46.75 )))
Pete's Deli sells corned beef at $2.55 per pound. It pays its wholesaler $1.95 per pound. What is the markup rate based on A) cost and B) selling price? (Round to nearest tenth percent.) ((( A = 30.8%; B = 23.5% )))
Bill Angel marks up his goods 38% on cost. A Nikon camera cost Bill $410. What is Bill's selling price? ((( $565.80 )))
Talbot's sells ladies sport coats for $99.99 that cost $65.50. What is the percent markup on cost? Round to nearest hundredth percent. ((( 52.66% )))
Bill's Hardware marks up snow blowers $130 and sells them for $485. Markup is on cost. What are the cost and percent markup to the nearest hundredth percent? ((( $355; 36.62% )))
Jingle Corporation produces toy footballs. Each football sells for $9.95 with a variable unit cost of $7.10. Assuming a fixed cost of $11,400 what is Jingle's breakeven point? (((4,000))) toy footballs
French Co. marks up its goods 45% on cost. What is French's equivalent markup on selling price? Round to the nearest hundredth percent. ((( 31.03% )))
Lucy's Bakeshop makes cupcakes that cost $.95 each. Lucy knows that 20% of the cupcakes will spoil. Assume Lucy wants a 35% markup on cost and produces 60 cupcakes. What should Lucy charge for each cupcake? Round to the nearest cent. ((( $1.60 )))
Louis's Hat Shop bought a hat for $95. Louis wants to markup the hat by 55% of the selling price. What should the selling price of the hat be? ((( $211.11 )))
Jorge Keegan sells lamps for $92.10 that cost him $62.00. What is Jorge's percent of markup based on the selling price? (Round to nearest hundredth percent.) ((( 32.68% )))
Mary's Bakeshop makes triple chocolate brownies that cost $1.50 each. Past experience shows that 20% of the brownies will spoil. Assuming Mary wants a 40% markup based on cost and produces 200 brownies, what should each brownie sell for? ((( $2.63 )))
Ace Hardware purchased 15 shovels from Skill Company for $7.00 each. Ace's markup is 33% on selling price. What should Ace charge for the shovels? (Round to hundredths.) ((( $10.45 )))
Lonnie Ianazzo, owner of Anthony's Jewelers in Tulsa, Oklahoma, marks up diamond earrings 83% on cost. The earrings cost Anthony's Jewelers $120 each. At what price should Anthony's sell the earrings? ((( $219.60 )))
An override can never help increase a manager's total pay. ((( FALSE )))
An employee's commission is always based on total dollar sales. ((( FALSE )))
The employer will match the employee's Social Security and Medicare contribution. ((( TRUE )))
The higher the number of allowances claimed on Form W-4, the less in taxes will be taken out of one's paycheck. ((( TRUE )))
Jim Ross is an automobile salesman who receives a salary of $500 per week plus a commission of 3% on all sales. During a four-week period he sold $42,000 worth of cars. What were Jim's average weekly earnings? ((( None of these )))
Bill Burne's earnings are subject to deductions for Social Security, Medicare, and FIT. To date his cumulative earnings are $110,000. This week Bill earns $1,200. Bill, who is married, is paid weekly and claims three exemptions. Assume the Social Security rate is 6.2% on $128,400 and 1.45% for Medicare. His net pay for the week is (use percentage table in handbook to calculate FIT): ((( $1,026.89 )))
Jim Smith is a salesman who receives a $1,100 draw per week. He receives a 12% commission on all sales. Sales for Jim were $205,000 for the month. Assuming a four-week month, Jim's final commission after the draw for the month is: ((( $20,200 )))
Mindy makes boots for Belleville Boot Factory. She is paid on the following differential pay scale: 1–50 = $ 1.65 51–150 = $ 3.30 151–200 = $ 4.95 Over 200 = $ 6.00 What is Mindy's pay if she produced 192 boots for the week? ((( None of these )))
Bob Neff, owner of an automotive dealership, pays one of his salesmen, Mike, a $1,300 draw per week plus 6% on all commission sales. Mike sold seven cars over the four-week period, totaling $186,900 for the month. Mike's commission minus the draw is: ((( $6,014 )))
Calculate Jim's gross pay (assume time and a half over 40 hours):M T W TH F S Total Hrs Rate per Hr Total Pay 10 8 0 A $9.50 B ((( A. 46 hours; B. $465.50 )))
Calculate the gross pay for Ryan O'Neal, an apple picker, assuming he picked 7,200 apples:Number apples Rate per apple 1 - 2,000 .07 2,001 - 3,000 .08 Over 3,000 .12 ((( $724 )))
Ron Smith has cumulative earnings of $110,100. This week he is paid $2,000. Assuming the Social Security rate is 6.2% on $128,400 and 1.45% for Medicare, what is the amount of Social Security and Medicare taken out of Ron's earnings? ((( Social Security: $124; Medicare: $29.00 )))
Alvin Shore sells jewelry at Ross Jewelers. Ross pays Alvin $10.50 per hour plus a 6% commission on all sales. Assume Alvin works 36 hours and had sales of $4,900. What is Alvin's gross pay? ((( $672 )))
Andrew Buckner earns $2,580 biweekly. He is married and claims three exemptions. What is Andrew's federal income tax? Use the percentage method (Use the tables in the handbook to answer this question). ((( $184.20 )))
Mike Goldsmith is paid $16.40 per hour. Last week he worked 40 hours, and this week 60 hours. Calculate Mike's pay for each week (time and a half over 40 hours): ((( $656; $1,148 )))
The Banker's Rule is the same as the exact interest method. ((( FALSE )))
The U.S. Rule is seldom used in today's workplace. ((( FALSE )))
The amount charged for the use of a bank's money is called: ((( Interest )))
Jill Ley took out a loan for $60,000 to pay for her child's education. The loan would be repaid at the end of eight years in one payment with interest of 6%. The total amount Jill has to pay back at the end of the loan is: ((( $88,800 )))
Matty Kaminsky owns a new Volvo. His June monthly interest is $400. The rate is 8 ½%. Matty's principal balance at the beginning of June is (use 360 days): ((( $56,470.59 )))
Janet took out a loan of $50,000 from Bank of America at 8% on March 19, 2015, which is due on July 8, 2016. Using exact interest, the amount of Janet's interest cost is: ((( None of these )))
Jim Murphy borrowed $30,000 on a 120-day 14% note. Jim paid $5,000 toward the note on day 95. On day 105 he paid an additional $6,000. Using the U.S. Rule, Jim's adjusted balance after the first payment is: ((( $26,108.33 )))
Desiree Brown borrowed $50,000 on a 90-day 8% note. Desiree paid $3,000 toward the note on day 40. On day 60 she paid an additional $4,000. Using the U.S. Rule, Desiree's adjusted balance after the first payment is: ((( $47,444.44 )))
Banks and other financial institutions sometimes calculate simple interest based on: ((( Banker's rule, ordinary interest )))
Ron Tagney owns his own truck. His June interest is $210. The current rate of interest is 11%. Assuming a 360-day year, what was Ron's principal balance at the beginning of June? (Round to nearest cent.) ((( $22,909.26 )))
Round all answers to the nearest cent. Woody's Café's real estate tax of $1,110.85 was due on November 1, 2017. Due to financial problems, Woody was unable to pay his café's real estate tax bill until January 15, 2018. The penalty for late payment is 8 1/4% ordinary interest. (A) What is the penalty Woody will have to pay and (B) what will Woody pay on January 15? ((( A. $19.09 B. $1,129.94 )))
Round all answers to the nearest cent. Angel Hall borrowed $82,000 for her granddaughter's college education. She must repay the loan at the end of nine years with 9¼% interest. What is the maturity value Angel must repay? ((( $150,265 )))
Lou Valdez is buying a truck. His monthly interest is $155 at 10 1/4 %. What is Lou's principal balance after the beginning of November? Use 360 days. Round all answers to the nearest cent. DO NOT round the denominator in your calculation. ((( $18,146.48 )))
Jane Smith took out a loan for $40,000 to pay for her child's education. The loan would be repaid at the end of eight years in one payment with interest of 12%. What is the total amount Jane has to pay back at the end of the loan? ((( $78,400 )))
Molly Joy owns her own car. Her June monthly interest was $205. The rate is 13 1/2%. Find out what Joy's principal balance is at the beginning of June. Use 360 days. (Do not round denominator in calculation.) ((( $18,222.22 )))
Rochelle Destin bought a new Buick Enclave. Her monthly January interest was $294.00. The current rate of interest is 9%. Assuming a 360-day year, what was Rochelle's balance at the beginning of January? ((( $39,200 )))
Joe and Kathy Graczak borrowed $132,000 for their son's four-year college education. They must repay the loan at the end of 10 years. With 7.6% on parent PLUS loan, what is the maturity value Joe and Kathy must repay? ((( $232,320 )))
Tom borrowed $150,000 for his son Jeff's law school tuition at the University of Mississippi. Tom received a rate of 3.75% for 5 years. Using 360 days in a year, what is the maturity value of the loan? ((( $178,125 )))
Joyce Flynn, an insurance agent, earned in her first week on the job a commission of $1,800. Her commission percent is 30%. Joyce's total sales for the week were: ((( $6,000. )))
The sales tax in Bill's state is 7%. Bill bought a Ford Mustang having a sales tax of $1928.22. What was the cost of the car? ((( $27,546. )))
At the Boston Children's Museum, it was estimated that 38% of all visitors are from in state. On Saturday, 8,000 people attended the museum. What is the number of out-of-state people in attendance? ((( 4,960. )))
In Professor Shannon's economics class there are 12 male students. Females represent 60% of the class total. How many students does Professor Shannon have in his economics class? ((( 30. )))
In 2018, the cost of electricity at Pete's Diner increased by 25%. In 2017, the cost to Pete was $410.50. What is the total cost of electricity to Pete in 2018? (Round to nearest hundredth or hundredth percent as needed.) ((( $513.13 )))
IUB Corporation projected a year end net income of $80,000. This net income represented 15% of projected annual sales. What are IUB's projected annual sales? (Round to nearest dollar.) ((( $533,333 )))
Ray Jene earns $900 a week at a Publix supermarket. Ray's payroll deductions are 28%. What is Ray's take-home pay? ((( $648 )))
Joyce Clark is reviewing the accounts receivable of Moe's Hardware. Credit customers paid $80,000 this month. This represents 30% of all receivables due. What are Moe's total accounts receivable? (Round to nearest dollar.) ((( $266,667 )))
Bonnie's Bookstore ordered 200 business math texts. Only 50 have arrived to date. What percent of the order is missing? ((( 0.75 )))
Alvin's Bookstore ordered 300 marketing texts. To date 30 texts are still missing. What percent has been received? ((( 0.90 )))
Bill Blum bought a dozen hamburgers at Von's Drive-In and was charged a meal tax of $.80. The tax rate is 5%. How much was the cost of the dinner before tax? ((( $16.00 )))
Mel's Hardware ordered 80 rakes. When the order was received, 20 rakes were missing. What percent of the order was received? ((( 0.75 )))
Mel Rickter is reviewing the total accounts receivable. This month, he received $45,000 from credit customers. This represented 25% of all receivables due. What is the total amount of Mel Rickter's accounts receivable? ((( $180,000 )))
Dierberg's meat department annual sales per store were $485,000. Sales for next year are expected to rise by 32%. What are the forecasted sales for the meat department next year? ((( $640,200 )))
4C stands for twice Cathy's age. ((( FALSE )))
B − 6 represents Bob's age six years ago. ((( TRUE )))
Jerry's age 3 years ago can be expressed as: ((( A - 3 )))
+31 - (-16) equals: ((( 47.00 )))
One-ninth of all sales at a local Subway are for cash. If cash sales for the week were $690, what were Subway's total sales? ((( 6210.00 )))
One-eleventh of all sales at Joe's Diner are for cash. If cash sales for the week were $380, what were Joe's total sales? ((( 4180.00 )))
Lane is 10 times Mel's age. If the difference in their age is 27, how old is Mel? ((( 3 )))
Rick and Ranger sell Toyotas for Tom's Auto. Over the past year they sold 360 cars. Assuming Rick sells 5 times as many as Ranger, how many Toyotas did Ranger sell? ((( 60.00 )))
Mike and Bill sell cars for Jangle's Auto. Over the past year they sold 640 cars. Assuming Mike sells 3 times as many as Bill, how many cars did Mike sell? ((( 480.00 )))
At Shaw's Market, apples cost $10 per case and bananas cost $6 per case. If an order comes in for a total of 300 cases for $2,000, what was the specific number of cases of apples? (Hint: Let A = cases of apples.) ((( 50.00 )))
Joe Berry and Jane Rose received a total of $180,000 from a deceased relative's estate. They decided to put away $80,000 in a trust for their child and divide the remainder into 3/4 for Jane and 1/4 for Joe. How much will Jane receive? ((( 75000.00 )))
Al Resse and Marci Rey received a total of $210,000 from a deceased relative's estate. They decided to put away $50,000 in a trust for their child and divide the remainder into 1/4 for Marci and for 3/4 Al. How much will Al receive? ((( 120000.00 )))
Kohl's sells casual shirts for $9 and polo shirts for $18.00. Total sales were $1,080, and customers bought 4 times as many casual shirts as polo shirts. How many casual shirts did Kohl's sell? ((( 80.00 )))
Ryan Small budgets 1/8 of his weekly salary for food. Ryan's food bill is $80. What is his weekly salary? ((( 640.00 )))
Brian Smith sells sets of movie posters ($12) and baseball cards ($8) at a local flea market. Last Sunday, Brian's total sales were $840. People bought 4 times as many movie posters as baseball cards. How many of each did Brian sell? Check your answer. ((( 15 cards; 60 posters )))
Marika is 8 times Barry's age. If the difference in their age is 7 years, how old is Marika? ((( 8 years old )))
Louis is 6 times Mary's age. If the difference in their ages is 20, how old is Louis? ((( 24 years )))
Andy Foll is 6 times as old as Mel Kaas. Andy is 15 years older than Mel. What is Mel's age? ((( 3.00 )))
Jane weighs 12 pounds less than Shelley. Assuming Jane weighs 120 pounds, find Shelley's weight. ((( 132 pounds )))
Cecil budgets of his weekly salary for comic books. Cecil's weekly comic book bill is $30.00. What is his weekly salary? ((( 180.00 )))
Roast beef at Albertson's was $4.225 per pound. At Stop and Shop it was $3.881 per pound. How much heaper is Stop and Shop per pound? ((( 0.34 )))
Lester Moy bought two new Michelin Tires for his car at $170.55 per tire. He was also charged $4.95 per tire for mounting, $2.65 per tire for valves, and $3.99 per tire for balancing. What was Lester's final bill? ((( 364.28 )))
The average pay of employees is $399.44 per week at a local True Value Hardware store. Jay Miller earns $411.11 per week. How much is Jay's pay over the average? ((( 11.67 )))
Mike Roland traveled 10,850 miles. His Ford truck averaged 16 miles per gallon. Assuming a gallon of gas cost $2.10, what was Mike's gasoline cost for the trip? ((( 1424.06 )))
Last week Alice Roe earned wages of $459.82. Payroll deductions included Social Security for $33.19 and federal income tax of $82.91. What is Alice's net pay? ((( 343.72 )))
Last year, Norm's December electric bill was $149.32. This year, his bill for the same month is $194.31. How much higher is his bill this year than last year? ((( 44.99 )))
Facebook's initial IPO was $38 per share. Arianna purchased 616 shares. Assuming a cost of $7.00 to purchase the 616 shares, what was Arianna's total cost? ((( 23415.00 )))
In 2017, America's Healthcare cost $14,009.33 per family. Assuming a forecasted cost of $21,065.25 in 2018, how much more will a family pay in 2018? ((( 7055.92 )))
To grow grass in Mel Smith's yard, a landscaper estimates that he will need 55 pounds of grass seed. Each pound will cost Mel $1.83. The landscaper will also charge Mel six hours of labor at $12.50 per hour. If Mel pays early, he will be entitled to a cash discount of $3.50. Assuming Mel takes advantage of the discount, how much does he pay the landscaper? ((( 172.15 )))
The average weekly pay at Wendy's is $412.88. Bill Long earns $485.11. How much is Bill over the average? ((( $72.73 )))
Pete Smith bought two new car tires from Firestone for $89.95 per tire. Firestone charged Pete $3.25 per tire for mounting, $2.60 per tire for valves, and $3.80 per tire for balancing. What is Pete's final bill? ((( $199.20 )))
The average pay of employees is $298.60 per week. John Morse earns $372.80 per week. How much is John's pay over the average? ((( $74.20 )))
The price of deli cuts was $2.255 per pound at Bill's Market. Bob's Market advertises the same deli cuts for $1.999. How much cheaper is Bob's Market per pound? (Round answer to nearest cent.) ((( $ .256 or $ .26 cheaper )))
Jeff has an internship in Connecticut this summer for 56 days. The company has offered him the following: $163.88 per day for hotel and $30.00 per day stipend. How much will Jeff's internship cost the company? ((( $10,857.28 )))
Reducing a fraction to the lowest terms does not change the fraction's value. ((( TRUE )))
Shelley Tilton bought 1 3/4 lbs of sliced roast beef, 8 1/2 lbs of sliced ham, and 3/4 lb of coleslaw at Albertson's Market. What was the total weight of her purchases? ((( 11 lbs )))
Matt Kaminsky bought a Volvo that is 3 3/4 times as expensive as the car his parents bought. If his parents paid $15,000 for theirs, what is the cost of Matt's car? ((( 56250.00 )))
Mia Wong bought a new Bose radio for $280. Bill, a friend of Mia's, can afford to pay only 3/4 as much as Mia. What is the most Bill could pay for the radio? ((( 210.00 )))
The price of a Swatch watch increased 1 3/4 times from the price last year. If this year's price is $175, what was last year's price? ((( 100.00 )))
Mary Cecil buys 3 pizzas for her son's birthday party. Each pizza has 8 servings. Sixteen people eat pizza at the party (assume each has 1 serving). What part of the pizza remains uneaten? ((( 1/3 )))
The average number of students for Professor Shannon's finance class was 20. During the fall semester there was an increase of 3/5 in students. How many students are registered for his class in the fall? ((( 32.00 )))
At Louis's grocery, each case of Cheerios takes up 3 1/2 square feet. If Louis sets aside 6,930 square feet, how many cases of Cheerios can Louis store? ((( 1980.00 )))
Last year's sales at Mel's Cinema totaled $144,600. This year's sales should increase by 1/3. How much should sales increase by, and what will sales be in the new year? ((( $48,200; $192,800 )))
John Rone bought a home that is 2 1/2 times as expensive as the home his parents bought. If his parents paid $80,000 for theirs, what is the cost of John's home? ((( 200 000 )))
Sellers’ catalogs list only the net price. ((( FALSE )))
A net price equivalent rate of 0.74225 means that for each $10 the buyer’s cost will be approximately 74 cents. ((( FALSE )))
A local college bookstore paid a net price of $12,500 for textbooks for the coming semester. The publisher offered a trade discount of 20%. The publisher’s original list price was: ((( $15,625 )))
Joanne’s Dress Shop received an invoice dated July 25 for $1,400, with terms of 2/10, 1/15, n/60. On August 8, Joanne’s Dress Shop sent a partial payment of $750. The actual amount that should be credited is: ((( $757.58 )))
J.C. Penney of Boston sold office equipment for $12,000 to Lee’s of San Diego. Terms of the sale are 3/10, n/30 FOB Boston. J.C. Penney has agreed to prepay freight $300. Assuming Lee’s pays within the discount period, how much will Lee’s pay J.C. Penney? ((( $11,940 )))
Mel’s furniture received an invoice dated September 27 for five bedroom sets at $3,000 each. The invoice indicated a trade discount of 5/8/3. The seller of the furniture prepaid the freight of $200. Terms were 2/10 EOM. Assuming Mel pays on November 2, what amount would be paid? (Be sure to include the freight cost.) ((( $12,662.37 )))
Pete’s Appliance received a bill dated July 15 for $26,000 with terms of 2/10, 1/15, n/60. On July 27 Pete’s sent in a partial payment of $6,000. What is the amount credited as well as the balance due? ((( $6,060.61 amount credited; $19,939.39 balance due )))
Jones Advertising Agency received a $3,000 invoice dated June 8. Terms were 2/10, 1/15, n/60. On June 22 Jones Advertising sent a $1,600 partial payment. What credit should Jones Advertising receive? What is Jones’s outstanding balance? Round to the nearest cent. ((( $1,616.16; $1,383.84 )))
Morgan Company received from Lee Company an invoice dated September 27. Terms were 2/10 EOM. List price on the invoice was $5,000 (freight not included). Morgan receives a 9/7 chain discount. Freight charges are Morgan’s responsibility, but Lee agreed to prepay the $150 freight charge. Morgan pays the invoice on November 9. What does Morgan pay Lee? ((( $4,296.87 )))
Bill’s TV buys a television from a wholesaler with a list price of $600 and a trade discount of 32%. What is the amount of the trade discount and what is the net price of the television? ((( $192; $408 )))
The Blue Manufacturer sells radios to Bob’s Discount Outlet. Radios carry a trade discount of 45%. If the price of a radio is $90, what does Bob pay? ((( $49.50 )))
If a manufacturer’s list price is $800 and Bill’s Outlet buys the goods for $650, what is the trade discount percent? (Round answer to the nearest hundredth percent.) ((( 18.75% )))
Al’s Supply sold a chain saw to Mark’s Lumber. The list price was $1,250. Al’s Supply offered a chain discount of 6/3/1. What was the net price of the chain saw? ((( $1,128.35 )))
A manufacturer of ice skates offered a chain discount of 7/5/1. Bobby’s Sport Shop ordered 30 pairs of skates that had a list price of $925. What was the amount of the trade discount? ((( $115.93 )))
Ed’s Print Shop received an invoice dated May 10 for $2,500 with terms 3/10, 1/15, n/60. On May 22, Ed’s Print Shop sent a partial payment of $2,000. What is the actual amount that should be credited, and what is Ed’s Print Shop’s outstanding balance? ((( Amount credited: $2,020.20; Outstanding balance: $479.80 )))
French Company marks up its goods 45% on cost. What is French's equivalent markup on selling price? Round to the nearest hundredth percent. ((( 31.03% )))
Lucy’s Bakeshop makes cupcakes that cost $.95 each. Lucy knows that 20% of the cupcakes will spoil. Assume Lucy wants a 35% markup on cost and produces 60 cupcakes. What should Lucy charge for each cupcake? Round to the nearest cent. ((( $1.60 )))
Louis’s Hat Shop bought a hat for $95. Louis wants to markup the hat by 55% of the selling price. What should the selling price of the hat be? ((( $211.11 )))
Jorge Keegan sells lamps for $92.10 that cost him $62.00. What is Jorge’s percent of markup based on the selling price? (Round to nearest hundredth percent.) ((( 32.68% )))
Mary’s Bakeshop makes triple chocolate brownies that cost $1.50 each. Past experience shows that 20% of the brownies will spoil. Assuming Mary wants a 40% markup based on cost and produces 200 brownies, what should each brownie sell for? ((( $2.63 )))
Ace Hardware purchased 15 shovels from Skill Company for $7.00 each. Ace’s markup is 33% on selling price. What should Ace charge for the shovels? (Round to hundredths.) ((( $10.45 )))
Lonnie Ianazzo, owner of Anthony’s Jewelers in Tulsa, Oklahoma, marks up diamond earrings 83% on cost. The earrings cost Anthony’s Jewelers $120 each. At what price should Anthony’s sell the earrings? ((( $219.60 )))
An override can never help increase a manager’s total pay. ((( FALSE )))
An employee’s commission is always based on total dollar sales. ((( FALSE )))
The employer will match the employee’s Social Security and Medicare contribution. ((( TRUE )))
The higher the number of allowances claimed on Form W-4, the less in taxes will be taken out of one’s paycheck. ((( TRUE )))
Jim Ross is an automobile salesman who receives a salary of $500 per week plus a commission of 3% on all sales. During a four-week period he sold $42,000 worth of cars. What were Jim’s average weekly earnings? ((( None of these )))
Bill Burne’s earnings are subject to deductions for Social Security, Medicare, and FIT. To date his cumulative earnings are $110,000. This week Bill earns $1,200. Bill, who is married, is paid weekly and claims three exemptions. Assume the Social Security rate is 6.2% on $128,400 and 1.45% for Medicare. His net pay for the week is (use percentage table in handbook to calculate FIT): ((( $1,026.89 )))
Jim Smith is a salesman who receives a $1,100 draw per week. He receives a 12% commission on all sales. Sales for Jim were $205,000 for the month. Assuming a four-week month, Jim’s final commission after the draw for the month is: ((( $20,200 )))
Mindy makes boots for Belleville Boot Factory. She is paid on the following differential pay scale: 1–50 = $ 1.65 51–150 = $ 3.30 151–200 = $ 4.95 Over 200 = $ 6.00 What is Mindy’s pay if she produced 192 boots for the week? ((( None of these )))
Bob Neff, owner of an automotive dealership, pays one of his salesmen, Mike, a $1,300 draw per week plus 6% on all commission sales. Mike sold seven cars over the four-week period, totaling $186,900 for the month. Mike’s commission minus the draw is: ((( $6,014 )))
Calculate Jim’s gross pay (assume time and a half over 40 hours):M T W TH F S Total Hrs Rate per Hr Total Pay 10 8 0 A $9.50 B ((( A. 46 hours; B. $465.50 )))
Calculate the gross pay for Ryan O’Neal, an apple picker, assuming he picked 7,200 apples:Number apples Rate per apple 1 - 2,000 .07 2,001 - 3,000 .08 Over 3,000 .12 ((( $724 )))
Ron Smith has cumulative earnings of $110,100. This week he is paid $2,000. Assuming the Social Security rate is 6.2% on $128,400 and 1.45% for Medicare, what is the amount of Social Security and Medicare taken out of Ron’s earnings? ((( Social Security: $124; Medicare: $29.00 )))
Alvin sells jewelry at Ross Jewelers. Ross pays Alvin $10.50 per hour plus a 6% commission on all sales. Assume Alvin works 36 hours and had sales of $4,900. What is Alvin's gross pay? (( 672 ))
Andrew Buckner earns $2,580 biweekly. He is married and claims three exemptions. What is Andrew’s federal income tax? Use the percentage method (Use the tables in the handbook to answer this question). ((( $184.20 )))
Mike Goldsmith is paid $16.40 per hour. Last week he worked 40 hours, and this week 60 hours. Calculate Mike’s pay for each week (time and a half over 40 hours): ((( $656; $1,148 )))
The Banker’s Rule is the same as the exact interest method. ((( FALSE )))
The U.S. Rule is seldom used in today’s workplace. ((( FALSE )))
The amount charged for the use of a bank’s money is called: ((( Interest )))
Jill Ley took out a loan for $60,000 to pay for her child’s education. The loan would be repaid at the end of eight years in one payment with interest of 6%. The total amount Jill has to pay back at the end of the loan is: ((( $88,800 )))
Matty Kaminsky owns a new Volvo. His June monthly interest is $400. The rate is 8 ½%. Matty’s principal balance at the beginning of June is (use 360 days): ((( $56,470.59 )))
Janet took out a loan of $50,000 from Bank of America at 8% on March 19, 2015, which is due on July 8, 2016. Using exact interest, the amount of Janet’s interest cost is: ((( None of these )))
Jim Murphy borrowed $30,000 on a 120-day 14% note. Jim paid $5,000 toward the note on day 95. On day 105 he paid an additional $6,000. Using the U.S. Rule, Jim’s adjusted balance after the first payment is: ((( $26,108.33 )))
Desiree Brown borrowed $50,000 on a 90-day 8% note. Desiree paid $3,000 toward the note on day 40. On day 60 she paid an additional $4,000. Using the U.S. Rule, Desiree’s adjusted balance after the first payment is: ((( $47,444.44 )))
Banks and other financial institutions sometimes calculate simple interest based on: ((( Banker’s rule, ordinary interest )))
Ron Tagney owns his own truck. His June interest is $210. The current rate of interest is 11%. Assuming a 360-day year, what was Ron’s principal balance at the beginning of June? (Round to nearest cent.) ((( $22,909.26 )))
Round all answers to the nearest cent. Woody’s Café’s real estate tax of $1,110.85 was due on November 1, 2017. Due to financial problems, Woody was unable to pay his café’s real estate tax bill until January 15, 2018. The penalty for late payment is 8 1/4% ordinary interest. (A) What is the penalty Woody will have to pay and (B) what will Woody pay on January 15? ((( A. $19.09 B. $1,129.94 )))
Round all answers to the nearest cent. Angel Hall borrowed $82,000 for her granddaughter’s college education. She must repay the loan at the end of nine years with 9¼% interest. What is the maturity value Angel must repay? ((( $150,265 )))
Lou Valdez is buying a truck. His monthly interest is $155 at 10 1/4 %. What is Lou’s principal balance after the beginning of November? Use 360 days. Round all answers to the nearest cent. DO NOT round the denominator in your calculation. ((( $18,146.48 )))
Jane Smith took out a loan for $40,000 to pay for her child’s education. The loan would be repaid at the end of eight years in one payment with interest of 12%. What is the total amount Jane has to pay back at the end of the loan? ((( $78,400 )))
Molly Joy owns her own car. Her June monthly interest was $205. The rate is 13 1/2%. Find out what Joy’s principal balance is at the beginning of June. Use 360 days. (Do not round denominator in calculation.) ((( $18,222.22 )))
Rochelle Destin bought a new Buick Enclave. Her monthly January interest was $294.00. The current rate of interest is 9%. Assuming a 360-day year, what was Rochelle’s balance at the beginning of January? ((( $39,200 )))
Joe and Kathy Graczak borrowed $132,000 for their son’s four-year college education. They must repay the loan at the end of 10 years. With 7.6% on parent PLUS loan, what is the maturity value Joe and Kathy must repay? ((( $232,320 )))
Tom borrowed $150,000 for his son Jeff’s law school tuition at the University of Mississippi. Tom received a rate of 3.75% for 5 years. Using 360 days in a year, what is the maturity value of the loan? ((( $178,125 )))
Joyce Flynn, an insurance agent, earned in her first week on the job a commission of $1,800. Her commission percent is 30%. Joyce’s total sales for the week were: ((( $6,000. )))
The sales tax in Bill’s state is 7%. Bill bought a Ford Mustang having a sales tax of $1928.22. What was the cost of the car? ((( $27,546. )))
At the Boston Children’s Museum, it was estimated that 38% of all visitors are from in state. On Saturday, 8,000 people attended the museum. What is the number of out-of-state people in attendance? ((( 4,960. )))
In Professor Shannon’s economics class there are 12 male students. Females represent 60% of the class total. How many students does Professor Shannon have in his economics class? ((( 30. )))
In 2018, the cost of electricity at Pete’s Diner increased by 25%. In 2017, the cost to Pete was $410.50. What is the total cost of electricity to Pete in 2018? (Round to nearest hundredth or hundredth percent as needed.) ((( $513.13 )))
IUB Corporation projected a year end net income of $80,000. This net income represented 15% of projected annual sales. What are IUB’s projected annual sales? (Round to nearest dollar.) ((( $533,333 )))
Ray Jene earns $900 a week at a Publix supermarket. Ray’s payroll deductions are 28%. What is Ray’s take-home pay? ((( $648 )))
Joyce Clark is reviewing the accounts receivable of Moe’s Hardware. Credit customers paid $80,000 this month. This represents 30% of all receivables due. What are Moe’s total accounts receivable? (Round to nearest dollar.) ((( $266,667 )))
Bonnie’s Bookstore ordered 200 business math texts. Only 50 have arrived to date. What percent of the order is missing? ((( 0.75 )))
Alvin’s Bookstore ordered 300 marketing texts. To date 30 texts are still missing. What percent has been received? ((( 0.90 )))
Bill Blum bought a dozen hamburgers at Von’s Drive-In and was charged a meal tax of $.80. The tax rate is 5%. How much was the cost of the dinner before tax? ((( $16.00 )))
Mel’s Hardware ordered 80 rakes. When the order was received, 20 rakes were missing. What percent of the order was received? ((( 0.75 )))
Mel Rickter is reviewing the total accounts receivable. This month, he received $45,000 from credit customers. This represented 25% of all receivables due. What is the total amount of Mel Rickter’s accounts receivable? ((( $180,000 )))
Dierberg’s meat department annual sales per store were $485,000. Sales for next year are expected to rise by 32%. What are the forecasted sales for the meat department next year? ((( $640,200 )))
4C stands for twice Cathy’s age. ((( FALSE )))
B − 6 represents Bob’s age six years ago. ((( TRUE )))
Jerry’s age 3 years ago can be expressed as: ((( A - 3 )))
’+31 - (-16) equals: ((( 47.00 )))
One-ninth of all sales at a local Subway are for cash. If cash sales for the week were $690, what were Subway’s total sales? ((( 6210.00 )))
One-eleventh of all sales at Joe’s Diner are for cash. If cash sales for the week were $380, what were Joe’s total sales? ((( 4180.00 )))
Lane is 10 times Mel’s age. If the difference in their age is 27, how old is Mel? ((( 3.00 )))
Rick and Ranger sell Toyotas for Tom’s Auto. Over the past year they sold 360 cars. Assuming Rick sells 5 times as many as Ranger, how many Toyotas did Ranger sell? ((( 60.00 )))
Mike and Bill sell cars for Jangle’s Auto. Over the past year they sold 640 cars. Assuming Mike sells 3 times as many as Bill, how many cars did Mike sell? ((( 480.00 )))
At Shaw’s Market, apples cost $10 per case and bananas cost $6 per case. If an order comes in for a total of 300 cases for $2,000, what was the specific number of cases of apples? (Hint: Let A = cases of apples.) ((( 50.00 )))
Joe Berry and Jane Rose received a total of $180,000 from a deceased relative’s estate. They decided to put away $80,000 in a trust for their child and divide the remainder into 3/4 for Jane and 1/4 for Joe. How much will Jane receive? ((( 75000.00 )))
Al Resse and Marci Rey received a total of $210,000 from a deceased relative’s estate. They decided to put away $50,000 in a trust for their child and divide the remainder into 1/4 for Marci and for 3/4 Al. How much will Al receive? ((( 120000.00 )))
Kohl’s sells casual shirts for $9 and polo shirts for $18.00. Total sales were $1,080, and customers bought 4 times as many casual shirts as polo shirts. How many casual shirts did Kohl’s sell? ((( 80.00 )))
Ryan Small budgets 1/8 of his weekly salary for food. Ryan’s food bill is $80. What is his weekly salary? ((( 640.00 )))
Brian Smith sells sets of movie posters ($12) and baseball cards ($8) at a local flea market. Last Sunday, Brian’s total sales were $840. People bought 4 times as many movie posters as baseball cards. How many of each did Brian sell? Check your answer. ((( 15 cards; 60 posters )))
Marika is 8 times Barry’s age. If the difference in their age is 7 years, how old is Marika? ((( 8 years old )))
Louis is 6 times Mary’s age. If the difference in their ages is 20, how old is Louis? ((( 24 years )))
Andy Foll is 6 times as old as Mel Kaas. Andy is 15 years older than Mel. What is Mel’s age? ((( 3.00 )))
Jane weighs 12 pounds less than Shelley. Assuming Jane weighs 120 pounds, find Shelley’s weight. ((( 132 pounds )))
Cecil budgets of his weekly salary for comic books. Cecil’s weekly comic book bill is $30.00. What is his weekly salary? ((( 180.00 )))
Roast beef at Albertson’s was $4.225 per pound. At Stop and Shop it was $3.881 per pound. How much heaper is Stop and Shop per pound? ((( 0.34 )))
Lester Moy bought two new Michelin Tires for his car at $170.55 per tire. He was also charged $4.95 per tire for mounting, $2.65 per tire for valves, and $3.99 per tire for balancing. What was Lester’s final bill? ((( 364.28 )))
The average pay of employees is $399.44 per week at a local True Value Hardware store. Jay Miller earns $411.11 per week. How much is Jay’s pay over the average? ((( 11.67 )))
Mike Roland traveled 10,850 miles. His Ford truck averaged 16 miles per gallon. Assuming a gallon of gas cost $2.10, what was Mike’s gasoline cost for the trip? ((( 1424.06 )))
Last week Alice Roe earned wages of $459.82. Payroll deductions included Social Security for $33.19 and federal income tax of $82.91. What is Alice’s net pay? ((( 343.72 )))
Last year, Norm’s December electric bill was $149.32. This year, his bill for the same month is $194.31. How much higher is his bill this year than last year? ((( 44.99 )))
Facebook’s initial IPO was $38 per share. Arianna purchased 616 shares. Assuming a cost of $7.00 to purchase the 616 shares, what was Arianna’s total cost? ((( 23415.00 )))
In 2017, America’s Healthcare cost $14,009.33 per family. Assuming a forecasted cost of $21,065.25 in 2018, how much more will a family pay in 2018? ((( 7055.92 )))
To grow grass in Mel Smith’s yard, a landscaper estimates that he will need 55 pounds of grass seed. Each pound will cost Mel $1.83. The landscaper will also charge Mel six hours of labor at $12.50 per hour. If Mel pays early, he will be entitled to a cash discount of $3.50. Assuming Mel takes advantage of the discount, how much does he pay the landscaper? ((( 172.15 )))
The average weekly pay at Wendy’s is $412.88. Bill Long earns $485.11. How much is Bill over the average? ((( $72.73 )))
Pete Smith bought two new car tires from Firestone for $89.95 per tire. Firestone charged Pete $3.25 per tire for mounting, $2.60 per tire for valves, and $3.80 per tire for balancing. What is Pete’s final bill? ((( $199.20 )))
The average pay of employees is $298.60 per week. John Morse earns $372.80 per week. How much is John’s pay over the average? ((( $74.20 )))
The price of deli cuts was $2.255 per pound at Bill’s Market. Bob’s Market advertises the same deli cuts for $1.999. How much cheaper is Bob’s Market per pound? (Round answer to nearest cent.) ((( $ .256 or $ .26 cheaper )))
Jeff has an internship in Connecticut this summer for 56 days. The company has offered him the following: $163.88 per day for hotel and $30.00 per day stipend. How much will Jeff’s internship cost the company? ((( $10,857.28 )))
Reducing a fraction to the lowest terms does not change the fraction’s value. ((( TRUE )))
Shelley Tilton bought 1 3/4 lbs of sliced roast beef, 8 1/2 lbs of sliced ham, and 3/4 lb of coleslaw at Albertson’s Market. What was the total weight of her purchases? ((( 11 lbs )))
Matt Kaminsky bought a Volvo that is 3 3/4 times as expensive as the car his parents bought. If his parents paid $15,000 for theirs, what is the cost of Matt’s car? ((( 56250.00 )))
Mia Wong bought a new Bose radio for $280. Bill, a friend of Mia’s, can afford to pay only 3/4 as much as Mia. What is the most Bill could pay for the radio? ((( 210.00 )))
The price of a Swatch watch increased 1 3/4 times from the price last year. If this year’s price is $175, what was last year’s price? ((( 100.00 )))
Mary Cecil buys 3 pizzas for her son’s birthday party. Each pizza has 8 servings. Sixteen people eat pizza at the party (assume each has 1 serving). What part of the pizza remains uneaten? ((( 1/3 )))
The average number of students for Professor Shannon’s finance class was 20. During the fall semester there was an increase of 3/5 in students. How many students are registered for his class in the fall? ((( 32.00 )))
At Louis’s grocery, each case of Cheerios takes up 3 1/2 square feet. If Louis sets aside 6,930 square feet, how many cases of Cheerios can Louis store? ((( 1980.00 )))
Last year’s sales at Mel’s Cinema totaled $144,600. This year’s sales should increase by 1/3. How much should sales increase by, and what will sales be in the new year? ((( $48,200; $192,800 )))
John Rone bought a home that is 2 1/2 times as expensive as the home his parents bought. If his parents paid $80,000 for theirs, what is the cost of John’s home? ((( 200 000 )))
What is true of wholesalers? (((Wholesalers are extremely important because of the marketing activities they perform.)))
Many service providers are considered retailers because they (((provide their services directly to consumers)))
A merger occurs when (((two companies combine to form a new company)))
A(n) ________ is a partnership established for a specific project or for a limited time. (((joint venture)))
An entrepreneur has been primarily associated with the willingness to((take risks.))
What is one of the difficulties faced by small business owners?((worries about employee problems))
What is a difference between high technology businesses and other small businesses?((High technology businesses require greater capital and have higher initial startup costs thanother small businesses))
What is true of a capitalist economic system?((Prices of goods and services are determined by supply and demand))
The quantity of goods and services that consumers are willing to buy at different prices at a specific time is referred to as((demand.))
The quantity of products that businesses are willing to sell at different prices at a specific time is referred to as((supply.))
What is a defining characteristic of public corporations?((Their stock can be bought, sold, or traded by anyone))
Which business provides a service, but is neither owned by the government nor focuses on earning profits?((a nonprofit corporation))
What is a true statement about the board members of a corporation?((They have a duty of care and loyalty to oversee the management of the firm.))
Preferred stockholders of a corporation((have a claim to profits before other stockholders do.))
10 (C − 7) = 30 ((( 10)))
What percent of 150 is 60? (((40)))
2,000(A − 5) = 110,000; A equals: (((60)))
Jim Ross is an automobile salesman who receives a salary of $500 per week plus a commission of 3% on all sales. During a four-week period he sold $42,000 worth of cars. What were Jim's average weekly earnings? ((None of these))
"Bloomberg’s is offered the following chain discount(s); what is the lowest net price equivalent rate it will pay? Company A Company B 7/9/13 8/6/10
Bill’s Furrier marks up mink coats $3,000. This represents a 50% markup on cost. What is the cost of the coats? ((( $6,000 )))
J.C. Penney sells a Timex watch for $139.99 that cost $89.97. J.C. Penney’s percent of markup based on the selling price is: ((( 35.73% )))
Zale’s bought a tea set for $1,400. Zale’s wants to markup the set 55% of the selling price. The selling price of the tea set should be: ((( $3,111.11 )))
Mr. Small, the store manager for Jay’s Appliance, is having a difficult time placing a selling price on a refrigerator that cost $410. Mr. Small knows his boss would like to have a 45% markup based on cost. The selling price should be: ((( $594.50 )))
Belle’s Bake Shop makes croissants that cost $1.75 each. Past experience shows that 10% of the croissants will spoil and have to be discarded. Assuming Belle wants a 45% markup based on cost and produces 300 croissants, each croissant should sell for: ((( $2.82 )))
A local Dunkin’ Donuts makes blueberry muffins that cost $.69 each. Past experience shows that 15% of the muffins will spoil and have to be discarded. Assuming that this shop wants a 30% markup based on cost and produces 200 muffins, each muffin should sell for: ((( $1.06 )))as the home his parents bought
Fathers’ Day suits were advertised for 35% off the regular price. A suit regularly sells for $210. The amount of the markdown is: ((( $73.50 )))
Camille Keegan sells lamps for $105.55 that cost her $75.00. Camille’s percent of markup based on the selling price is: ((( 28.94% )))
Kohl’s sells watches that cost $6.95 for a selling price of $39.99. What is the percent markup on cost? (Round to the nearest tenth percent.) ((( 475.4% )))
Macy’s Department Stores markup men’s cologne 63% on cost for an 8-ounce bottle. A bottle of cologne costs Macy’s $23.00. What is the selling price for the 8-ounce bottle? ((( $37.49 )))
Johnny Mac’s Sporting Goods bought a baseball glove from Rawlings Sporting Goods for $66.00. They want to markup the glove 70% on selling price. What should Johnny’s sell the glove for? ((( $220.00 )))
Straub’s Bakery makes 200 Danish cakes that cost $2.70 each. Straub’s needs a 66% markup on cost and normally discards 10% of what it makes. At what price should Straub’s sell the Danish cakes? ((( $4.98 )))
Sullivan’s Handbags marks up its bags at 45% of the selling price. Pat Sullivan saw a bag at a trade show that she would sell to her customers for $85.00. What is the most she could pay for the bag and still retain the 45% markup of the selling price? ((( $46.75 )))
Pete’s Deli sells corned beef at $2.55 per pound. It pays its wholesaler $1.95 per pound. What is the markup rate based on A) cost and B) selling price? (Round to nearest tenth percent.) ((( A = 30.8%; B = 23.5% )))
Bill Angel marks up his goods 38% on cost. A Nikon camera cost Bill $410. What is Bill’s selling price? ((( $565.80 )))
Talbot’s sells ladies sport coats for $99.99 that cost $65.50. What is the percent markup on cost? Round to nearest hundredth percent. ((( 52.66% )))
Bill’s Hardware marks up snow blowers $130 and sells them for $485. Markup is on cost. What are the cost and percent markup to the nearest hundredth percent? ((( $355; 36.62% )))
Jingle Corporation produces toy footballs. Each football sells for $9.95 with a variable unit cost of $7.10. Assuming a fixed cost of $11,400 what is Jingle’s breakeven point? ((( 4,000 toy footballs"
Elora is printing pages of her term paper. It costs 0.15 for each black and white page, and 0.84 for each color page. Elora’s paper will contain two color pages, and 32 black and white pages. What is the total cost? ((6.48))
Complete by the shortcut method: 41.82 × 100 ((4182))
20(C − 9) means 20 is multiplied only times the C to clear the parentheses. ((False))
Abby takes home $4,100 biweekly, after taking out 32% for payroll deductions. What is Abby’s biweekly salary, before deductions. ((6029.41))
6H + H − 20 = 44 − H; H = 8. ((True))
The state reimburses Cho 55 cents per mile. Cho submitted a travel log for a total of 520.9 miles. What reimbursement can Cho expect? (Round to nearest cent.) ((286.50))
Subtract (reduce to lowest terms): 15/16 - 17/20 (( 7/80))
Complete by the shortcut method: 18,915.32 ÷ 1,000 (( 18.91532 ))
Total the following: Six hundred sixty-eight and eight hundred one thousandths Twelve and forty-nine hundredths Three and four thousandths Fifty-one hundredths ((995.205))
Tanis has contracted for an oil delivery price of $3.95 per gallon. His tank holds 210 gallons. If Tanis has six fill-ups during the winter, what will be his total cost? ((4977))
8 (A − 5) = 70 (((14)))
Convert the following decimal to a percent: 15.82 ((1582))
Madison earns $750 per week plus 3% of sales in excess of $6,500. If Madison sells $25,000 in the first week, her earnings are: (($1,305))
Aley Company pays all its employees a graduated commission scale: 3% on the first $30,000 sales, 5% on sales from $30,001 to $110,000, and 7% on sales more than $110,000. Wyatt, an employee of Aley, had $130,000 in sales. What commission did Wyatt earn? (( $6,300.00 ))
Interest on $6,500 at 3% for 17 months is: (( 276.25 ))
If the net price of a laptop is $900 and the trade discount rate is 40%, the list price is: (( 1,500))
Blue Jeans Incorporated sells jeans that cost $15.99 for a selling price of $42.95. What is the percent of markup based on cost? (( 168.61))
A note dated August 18 and due on March 9, given no leap year, runs for exactly: (( 203 Days )
Brooks Department Store bought a stereo from a wholesaler with a $1,200 list price and a 28% trade discount. (( A-336 / B-864 ))
Neo, a customer of Asher Tennis, will pay only $190 for a tennis racket. Assuming Asher Tennis works on a 60% markup on the selling price, the most Asher Tennis will pay the manufacturer is: (( 76$ ))
With an interest of $24.31 and a principal of $3,500 for 100 days, using the ordinary interest method, the rate is: ((2.5%))
LM Corporation pays its employees on a graduated commission scale: 6% on the first $40,000 in sales, 7% on sales from $40,000 to $80,000, and 9% on sales greater than $80,000. Banisha had sales of $105,000. Banisha’s commission is: (( $7,450))
A note dated December 13 and due July 5 (assume no leap year) runs for exactly: (( 204 Days ))
With an interest of $1,832.00 and a principal of $16,000 for 206 days, using the ordinary interest method, the rate is: (( 20% ))
Sonny earned $485,000 a year as a rookie pitcher for the Cleveland Indians. Assuming a tax rate of 6.2% for Social Security up to $142,800 and Medicare tax of 1.45%, how much did Sonny pay in FICA taxes as a rookie pitcher? (( 14379.50))
The percentage method table is used to calculate _________ (( FIT (Federal Income Tax) ))
On May 19, Sansa borrowed $3,000 from a bank at a rate of 12½%. The loan is to be repaid on October 8. Assuming the loan is based on exact interest, what is the total interest cost to Sansa? (( 145.89 ))
Sashi works for Elizabeth Arden Cosmetics and earns $500 per week salary plus 4% commission on sales over $2,000. If Sashi sold $2,733 last week, what was her salary? (( 529.32 ))
Jonas borrowed $8,000 on a 60-day 10% note. Jonas paid $4,000 toward the note on day 40. On day 50, he paid an additional $3,000. Using the U.S. Rule, Jonas's adjusted balance after the first payment is: ((4088.89 ))
Rodney is a shoe salesman for More Company. He is paid $12.60 per hour plus a commission of 3% on all sales. For the week, assuming Rodney works 30 hours and has sales of $2,400, what is his gross pay? (( 450 ))
Pauli pays her two employees $410 and $650 per week. Assume a state unemployment rate of 5.4% and a federal rate of 0.6%. What state and federal unemployment tax will Pauli pay at the end of Quarter 1 and Quarter 2? (( State unemployment Quarter1: 665.82 \ Federal unemployment Quarter1: 73.98 \ State unemployment Quarter2: 90.18 \ State unemployment Quarter2: 10.02 ))
Lana has cumulative earnings of $116,000 at the end of September. In the first week in October, she earns $2,000. The amount deducted for Social Security and Medicare from her check is (assume Social Security rate of 6.2% on $142,800 and Medicare of 1.45%): (( 153.00 ))
Jasper works at Panera Company for $17.25 per hour plus a commission of 2% of sales. Assuming Jasper worked 26 hours last week and had sales of $2,610, Jasper’s gross pay is: ((500.70))
Aniyah borrowed $82,000, on a 120-day 4% note. After 50 days, Aniya made an initial payment of $40,000. On day 90, Aniyah made an additional payment of $25,000. Assuming the U.S. Rule, what is the adjusted balance after the second payment? Use exact days. (( 18927.29))
1.26 times a year ((Biweekly)) ||| 2.May be used in calculating Social Security and Medicare ((Payroll register)) ||| 3.Pay rate based on schedule of units completed ((Differential pay)) ||| 4.Aids in determining amount of FIT withheld ((W-4)) ||| 5.52 times a year ((Weekly)) ||| 6.Minimum wage standard ((Fair Labor Standards Act)) ||| 7.Broken down into Social Security and Medicare ((FICA)) ||| 8.Aids unemployment program ((FUTA)) 9.Income tax ((FIT)) ||| 10.Wages before deductions ((Gross pay)) ||| 11.Wages subject to tax ((Taxable earnings)) 12.Gross pay minus deductions ((Net pay)) ||| 13.Commission to manager ((Override)) ||| 14.Method to calculate FIT ((Percentage method)) ||| 15.24 times a year ((Semimonthly)) ||| 16.Affected by company's employment record ((SUTA)) ||| 17.Wages based on percent of the value of goods sold ((Straight commission)) ||| 18.Different levels result in different commission scales ((Variable commission scale)) ||| 19.Alternative method to percentage method ((Wage bracket))
Convert fraction to decimal (or mixed decimal) and round to nearest hundredth as needed: 4 6/7 (( 4.86 ))
Subtract (reduce to lowest terms): 15/16 - 17/20 (( 7/80))
Lulu grocery planned a big sale on apples and received 750 crates from the wholesale market. Lulu will bag these apples in plastic. Each plastic bag holds 1/9 of a crate. If Lulu has no loss to perishables, how many bags of apples can be prepared? (( 6750))
Huy traveled 10,850 miles. His Ford truck averaged 16 miles per gallon. Assuming a gallon of gas cost $3.10, what was Mike's gasoline cost for the trip? ((( 2102.19 )))
North Shore Community College reimburses faculty members $0.535 cents per mile to go to a workshop. Professor Wales submitted a travel log for a total of 650.11 miles. What reimbursement can Professor Wales expect? Note: Round to the nearest cent. ((347.81))
At Kentucky Fried Chicken, a survey showed 2/3 of all people preferred skinless chicken over the regular chicken If 2,400 people responded to the survey, how many preferred regular chicken? ((800))
At United Airlines, Terrell worked 8 3/4 , hours on Monday, 4 1/2 hours on Tuesday,9 1/4 hours on Wednesday 10 1/2 hours on Thursday, and 7 hours on Friday, How many total hours did Pete work during the week? ((40))
Elora is printing pages of her term paper, It costs 0.15 for each black and white page, and 0.84 for each color page, Elora’s paper will contain two color pages, and 32 black and white pages, What is the total cost? (($6.48))
Lulu Hypermarket planned a big sale on apples and received 750 crates from the wholesale market, LULU will bag these apples in plastic. Each plastic bag holds 1/9 of a crate, If LULU has no loss to perishables, how many bags of apples can be prepared? ((6750))
Mick and Mack went to a supermarket and bought boxes of soap detergent, Mack could buy only four boxes, which was 1/5 as much as Mick, How many boxes did Mick buy? ((20))
Subtract (reduce to lowest terms if necessary): 14 ¼ - 3 ¾ ((10 ½))
Subtract (reduce to lowest terms),15/16-17/20 ((7/80))
The average pay of employees is $898.60 per week, Achack earns $772.80 per week, How much is Achack's pay under the average? Note: Round your answer to 2 decimal places. ((125.8))
The price of a Panasonic 3D flat screen television decreased by 1/5 , If the original price was $1,500, what is the price today? (($1,200))
A frequency distribution is a grouping of quantitative data into overlapping classes showing the number of observations in each class.(((FALSE))
A frequency table for qualitative data has class limits.(((FALSE))
To summarize the gender of students attending a college, the number of classes in a frequency table depends on the number of students.(((FALSE))
In frequency distributions, classes are mutually exclusive if each individual, object, or measurement is included in only one category.(((TRUE))
In a bar chart, the horizontal axis is usually labeled with the values of a qualitative variable.(((TRUE))
In a bar chart, the heights of the bars represent the frequencies in each class.(((TRUE))
The midpoint of a class is halfway between the lower and upper limits.(((TRUE))
A class interval can be determined by subtracting the lower limit of a class from the lower limit of the next higher class.(((TRUE))
To convert a frequency distribution to a relative frequency distribution, divide each class frequency by the sum of the class frequencies.(((TRUE))
To convert a frequency distribution to a relative frequency distribution, divide each class frequency by the number of classes.(((FALSE))
A pie chart is similar to a relative frequency distribution.(((TRUE))
A pie chart shows the relative frequency in each class.(((TRUE))
To construct a pie chart, relative class frequencies are used to graph the "slices" of the pie.(((TRUE))
A cumulative frequency distribution is used when we want to determine how many observations lie above or below certain values.(((TRUE))
A frequency polygon is a very useful graphic technique when comparing two or more distributions.(((TRUE))
Monthly commissions of first-year insurance brokers are $1,270, $1,310, $1,680, $1,380, $1,410, $1,570, $1,180, and $1,420. These figures are referred to as(((raw data.))
A small sample of computer operators shows monthly incomes of $1,950, $1,775, $2,060, $1,840, $1,795, $1,890, $1,925, and $1,810. What are these ungrouped numbers called?(((Raw data))
When data are collected using a quantitative, ratio variable, what is true about a frequency distribution that summarizes the data?(((Upper and lower class limits must be calculated.))
When data are collected using a qualitative, nominal variable, what is true about a frequency table that summarizes the data?(((A pie chart can be used to summarize the data.))
A student was interested in the cigarette-smoking habits of college students and collected data from an unbiased random sample of students. The data are summarized in the following table: What is wrong with this frequency table?(((The classes are not mutually exclusive.))
A student was interested in the cigarette-smoking habits of college students and collected data from an unbiased random sample of students. The data are summarized in the following table: What type of chart best represents the frequency table?(((Bar chart))
A student was interested in the cigarette-smoking habits of college students and collected data from an unbiased random sample of students. The data are summarized in the following table: What type of chart best represents the relative class frequencies?(((Pie chart))
When a class interval is expressed as 100 up to 200,(((observations with values of 200 are excluded from the class.))
For a relative frequency distribution, relative frequency is computed as(((the class frequency divided by the number of observations.))
The relative frequency for a class represents the(((percentage of observations in the class.))
A group of 100 students was surveyed about their interest in a new International Studies program. Interest was measured in terms of high, medium, or low. In the study, 30 students responded high interest, 40 students responded medium interest, and 30 students responded low interest. What is the relative frequency of students with high interest?(((0.30))
A group of 100 students were surveyed about their interest in a new Economics major. Interest was measured in terms of high, medium, or low. In the study, 30 students responded high interest, 50 students responded medium interest, and 20 students responded low interest. What is the best way to illustrate the relative frequency of student interest?(((Use a pie chart.))
The monthly salaries of a sample of 100 employees were rounded to the nearest $10. They ranged from a low of $1,040 to a high of $1,720. If we want to condense the data into seven classes, what is the most convenient class interval?((($100 ))
What is the following table called?(((Frequency distribution))
For the following distribution of heights, what are the limits for the class with the greatest frequency? (((65 and up to 70))
In a frequency distribution, the number of observations in a class is called the class(((frequency.))
Why are unequal class intervals sometimes used in a frequency distribution?(((To avoid a large number of classes with very small frequencies.))
Refer to the following distribution of commissions: What is the relative frequency for salespeople who earn from $1,600 up to $1,800?(((0.20 OR 0.22))
Refer to the following distribution of commissions: To plot a cumulative frequency distribution, the first coordinate would be(((X = 600, Y = 0.))
Refer to the following distribution of commissions: What is the relative frequency of salespeople who earn $1,600 or more?(((30.8%))
Refer to the following distribution of commissions: For the preceding distribution, what is the midpoint of the class with the greatest frequency?(((1,500))
Refer to the following distribution of commissions: What is the class interval?(((200))
Refer to the following wage breakdown for a garment factory: What is the class interval for the preceding table of wages?((($3 ))
Refer to the following wage breakdown for a garment factory: What is the class midpoint for the class with the greatest frequency?((($8.50 ))
Refer to the following wage breakdown for a garment factory: What are the class limits for the class with the smallest frequency?(((13 and up to 16))
Refer to the following distribution of ages: For this distribution of ages, what is the relative class frequency for the lowest class?(((0.20))
Refer to the following distribution of ages:What is the class interval?(((10))
Refer to the following distribution of ages: What is the class midpoint of the highest class?(((65))
Refer to the following information from a frequency distribution for heights of college women recorded to the nearest inch: the first two class midpoints are 62.5" and 65.5". What is the class interval?(((3"))
Refer to the following information from a frequency distribution for heights of college women recorded to the nearest inch: the first two class midpoints are 62.5" and 65.5". What are the class limits for the lowest class?(((61 and up to 64))
Refer to the following information from a frequency distribution for heights of college women recorded to the nearest inch: the first two class midpoints are 62.5" and 65.5". What are the class limits for the third class?(((67 and up to 70))
Refer to the following distribution: What is the relative class frequency for the $25 up to $35 class?(((0.04))
Refer to the following distribution: What is the class midpoint for the $45 up to $55 class?(((50))
Refer to the following distribution: What are the class limits for the class with the highest frequency?(((55 up to 65))
Refer to the following frequency distribution of days absent during a calendar year by employees of a manufacturing company: How many employees were absent for 3 up to 6 days?(((31))
Refer to the following frequency distribution of days absent during a calendar year by employees of a manufacturing company: How many employees were absent fewer than six days?(((91))
Refer to the following frequency distribution of days absent during a calendar year by employees of a manufacturing company: How many employees were absent six or more days?(((22))
Refer to the following frequency distribution of days absent during a calendar year by employees of a manufacturing company: How many employees were absent for 6 up to 12 days?(((20))
Refer to the following breakdown of responses to a survey of room service in a hotel:What is the class interval for this frequency table?(((None apply))
Refer to the following breakdown of responses to a survey of room service in a hotel: What is the class with the greatest frequency?(((Highly satisfied))
Refer to the following breakdown of responses to a survey of room service in a hotel: What percentage of the responses indicated that customers were satisfied?(((33%))
Refer to the following breakdown of responses to a survey of room service in a hotel: What type of chart should be used to describe the frequency table?(((A bar chart))
Refer to the following breakdown of responses to a survey of room service in a hotel: What type of chart should be used to show relative class frequencies?(((A pie chart))
Refer to the following breakdown of responses to a survey of "Are you concerned about being tracked while connected to the Internet?" What is the class interval for the preceding frequency table?(((None apply))
Refer to the following breakdown of responses to a survey of "Are you concerned about being tracked while connected to the Internet?" What is the class with the greatest frequency?(((Very concerned))
Refer to the following breakdown of responses to a survey of "Are you concerned about being tracked while connected to the Internet?" What percentage of the responses indicated that users were somewhat concerned?(((20%))
Refer to the following breakdown of responses to a survey of "Are you concerned about being tracked while connected to the Internet?" What type of chart should be used to describe the frequency table?(((A bar chart))
Refer to the following breakdown of responses to a survey of "Are you concerned about being tracked while connected to the Internet?" What type of chart should be used to show relative class frequencies?(((A pie chart))
Refer to the following breakdown of responses to a survey of "How confident are you that you saved enough to retire?" What is the class interval for the preceding frequency table?(((None apply))
Refer to the following breakdown of responses to a survey of "How confident are you that you saved enough to retire?" What is the class with the greatest frequency?(((Somewhat confident))
Refer to the following breakdown of responses to a survey of "How confident are you that you saved enough to retire?" What percentage of the responses indicated that users were very confident?(((21%))
Refer to the following breakdown of responses to a survey of "How confident are you that you saved enough to retire?" What type of chart should be used to describe the frequency table?(((A bar chart))
Refer to the following breakdown of responses to a survey of "How confident are you that you saved enough to retire?" What type of chart should be used to show relative class frequencies?(((A pie chart))
A pie chart shows the(((relative frequencies of a qualitative variable.))
A table summarizing a set of data showing the fraction of the total number of items in several classes is a(((relative frequency table.))
In order to convert class frequency to relative class frequency, we(((divide the frequency of the class by the sample size. ))
In constructing a frequency distribution, the approximate class interval is computed as((((maximum value - minimum value)/(number of classes).))
A population is a collection of all individuals, objects, or measurements of interest.(((TRUE))
Statistics are used as a basis for making decisions.(((TRUE))
A listing of 100 family annual incomes is an example of statistics.(((FALSE))
The average number of passengers on commercial flights between Chicago and New York City is an example of a statistic.(((TRUE))
Statistics is used to report the summary results of market surveys.(((TRUE))
A sample is a portion or part of the population of interest.(((TRUE))
To infer something about a population, we usually take a sample from the population.(((TRUE))
Descriptive statistics are used to find out something about a population based on a sample.(((FALSE))
There are four levels of measurement: qualitative, quantitative, discrete, and continuous.(((FALSE))
The ordinal level of measurement is considered the "lowest" level of measurement.(((FALSE))
A store asks shoppers for their zip codes to identify market areas. Zip codes are an example of ratio data.(((FALSE))
An ordinal level of measurement implies some sort of ranking.(((TRUE))
Data measured on a nominal scale can only be classified into categories.(((TRUE))
The terms descriptive statistics and inferential statistics can be used interchangeably.(((FALSE))
A marketing research agency was hired to test a new DVD player. Consumers rated it outstanding, very good, fair, or poor. The level of measurement for this experiment is ordinal.(((TRUE))
The Union of Electrical Workers of America with 9,128 members polled 362 members about a new wage package that will be submitted to management. The population is the 362 members.(((FALSE))
"The CIA World Factbook cited these numbers for the United States:
• The birthrate is 13.66 births per 1,000 of the population.
• The average life expectancy for females is 81.17 years.
• Approximately 316.7 million persons reside in the United States.
Each of these numbers is referred to as a statistic.
"(((TRUE))
If we select 100 persons from 25,000 registered voters and question them about candidates and issues, the 100 persons are referred to as the population.(((FALSE))
Statistics is defined as a body of techniques used to facilitate the collection, organization, presentation, analysis, and interpretation of information for the purpose of making better decisions.(((TRUE))
Categorizing voters as Democrats, Republicans, and Independents is an example of interval level measurement.(((FALSE))
The order in which runners finish in a race would be an example of continuous data.(((FALSE))
Based on a sample of 3,000 people, the civilian unemployment rate in the United States was 5.5%. 5.5% is referred to as a statistic.(((TRUE))
The principal difference between the interval and ratio scale is that the ratio scale has a meaningful zero point.(((TRUE))
The branch of mathematics used to facilitate the collection, organization, presentation, analysis, and interpretation of numerical information is referred to as statistics.(((TRUE))
The number of children in a family is a discrete variable.(((TRUE))
The main purpose of descriptive statistics is to(((summarize data in a useful and informative manner.))
Which of the following is an example of a continuous variable?(((Tons of concrete to complete a parking garage))
The incomes of 50 loan applicants are obtained. Which level of measurement is income?(((Ratio))
When TV advertisements report that "2 out of 3 dentists surveyed indicated they would recommend Brand X toothpaste to their patients," an informed consumer may question the conclusion because(((the advertisement does not include the total number of dentists surveyed.))
A bank asks customers to evaluate its drive-through service as good, average, or poor. Which level of measurement is this classification?(((Ordinal))
A portion or part of a population is called a(((sample.))
If Gallup, Harris, and other pollsters asked people to indicate their political party affiliations as Democrat, Republican, or independent, the data gathered would be an example of which scale of measurement?(((Nominal))
The members of each basketball team wear numbers on their jerseys. What scale of measurement are these numbers considered?(((Nominal))
A marketing class of 50 students evaluated the instructor using the following scale: superior, good, average, poor, or inferior. The descriptive summary showed the following survey results: 2% superior, 8% good, 45% average, 45% poor, and 0% inferior.(((Most students rated the instructor as poor or average.))
A survey includes a question about marital status that has the following responses: single, married, divorced, separated, or widowed. What is the scale of measurement for this question?(((Nominal))
Respondents were asked, "Do you now earn more than or less than you did five years ago?" What is this level of measurement?(((Ordinal))
Which word is NOT part of the definition of descriptive statistics?(((Predicting))
The reported unemployment is 5.5% of the population. What measurement scale is used to measure unemployment?(((Ratio))
The Equal Employment Opportunity Act requires employers to classify their employees by gender and national origin. Which level of measurement is this?(((Nominal))
What level of measurement is the Centigrade temperature scale?(((Interval))
What type of variable is the number of gallons of gasoline pumped by a filling station during a day?(((Continuous))
The performance of personal and business investments is measured as a percentage called "return on investment." What type of variable is "return on investment"?(((Continuous))
What type of variable is the number of robberies reported in your city?(((Quantitative))
What type of variable is the number of auto accidents reported in a given month?(((Discrete))
The names of the positions in a corporation, such as chief operating officer or controller, are examples of what type of variable?(((Qualitative))
What type of variable is "pounds of popcorn" served at a movie theater?(((Continuous))
The final rankings of the top 20 NCAA college basketball teams are an example of which level of measurement?(((Ordinal))
Your height and weight are examples of which level of measurement?(((Ratio))
Shoe style is an example of what level of measurement?(((Nominal))
The general process of gathering, organizing, summarizing, analyzing, and interpreting data is called(((statistics.))
The Nielsen Ratings break down the number of people watching a particular television show by age. What level of measurement is age?(((Ratio))
An example of a qualitative variable is(((color of ink in a pen.))
Which one of the following is NOT an example of discrete data?(((Number of miles between New York City and Chicago))
A group of women tried five brands of fingernail polish and ranked them according to preference. What level of measurement is this?(((Ordinal))
A university wishes to conduct a student survey. In one of the questions students are asked to mark their gender as either male or female. Gender is an example of the(((nominal scale.))
Income is a variable often used in business and economics. Income is an example of a variable that uses the(((ratio scale.))
When statisticians analyze sample data in order to draw conclusions about the characteristics of a population, this is referred to as(((statistical inference.))
The length of a bridge, measured in meters, is an example of(((quantitative data.))
A value that is typical or representative of the data is referred to as a measure of central location.(((TRUE))
The arithmetic mean is the sum of the quantitative observations divided by the total number of observations(((TRUE))
For a set of data arranged or sorted in numerical order, the value of the observation in the center is called the weighted mean.(((FALSE))
A set of o1rdinal-, interval-, or ratio-level data may have only one mode.(((FALSE))
The mode is the value of the observation that appears most frequently.(((TRUE))
Extremely high or low scores affect the value of the median.(((FALSE))
The sum of the deviations from the mean for the set of numbers 4, 9, and 5 will equal zero.(((TRUE))
In a sample of 10 people, 3 persons earn $8 an hour, 6 earn $9 an hour, and 1 earns $12 an hour. The weighted mean hourly wage is $9.(((TRUE))
For any distribution, there is an equal number of values above and below the mean.(((FALSE))
Variation describes the degree of dispersion in the data.(((TRUE))
The variance is the mean of the sum of the squared deviations between each observation and the median.(((FALSE))
The standard deviation is the positive square root of the variance.(((TRUE))
In a company, the standard deviation of the ages of female employees is 6 years and the standard deviation of the ages of male employees is 10 years. These statistics indicate that the dispersion of age is greater for females than for males.(((FALSE))
According to the Empirical rule, about 95% of the observations lie within plus and minus 2.00 standard deviations.(((TRUE))
The geometric mean is the nth root of the product of n observations.(((TRUE))
The sum of the deviations of each data value from this measure of central location will always be zero.(((Mean))
For any data set, which measures of central location have only one value?(((Mean and median))
Which measures of central location are not affected by extremely small or extremely large values?(((Mode and median))
What is the relationship among the mean, median, and mode in a symmetric distribution?(((They are all equal.))
For a data set, half of the observations are always greater than the ________.(((median))
What is the lowest level of measurement to which a median can be computed?(((Ordinal))
For a data set with an odd number of observations that have been sorted from smallest to largest values, where is the median located?((((n + 1)/2))
Which one of the following is referred to as the population mean?(((µ))
On a finance exam, 15 accounting majors had an average grade of 90. On the same exam, 7 marketing majors averaged 85, and 10 finance majors averaged 93. What is the weighted mean for all 32 students taking the exam?(((89.84))
A survey item asked students to indicate their class in college: freshman, sophomore, junior, or senior. Which measure(s) of central location would be appropriate for the data generated by that questionnaire item?(((Mode and median))
What is the median of 26, 30, 24, 32, 32, 31, 27, and 29?(((29.5))
The net incomes (in $millions) of a sample of steel fabricators are $86, $67, $86, and $85. What is the modal net income?((($86 ))
"A stockbroker placed the following order for a customer:
$ 50 shares of Kaiser Aluminum at $104 a share $ 100 shares of GTE at $25.25 a share $ 20 shares of Boston Edison at $9.125 a share What is the weighted arithmetic mean price per share? "((($46.51 ))
"During the past six months, a purchasing agent placed the following three orders for coal:
Tons of Coal 1,200 3,000 500
Price Per Ton $ 28.50 $ 87.25 $ 88.00
What is the weighted arithmetic mean price per ton?"((($72.33 ))
A sample of single persons receiving Social Security payments revealed these monthly benefits: $826, $699, $1,087, $880, $839, and $965. How many observations are below the median?(((3))
Over the last six months, the following numbers of absences have been reported: 6, 0, 10, 14, 8, and 0. What is the median number of monthly absences?(((7))
Assume a student received the following grades for the semester: History, B; Statistics, A; Spanish, C; and English, C. History and English are 5 credit-hour courses, Statistics a 4 credit-hour course, and Spanish is a 3 credit-hour course. If 4 grade points are assigned for an A, 3 for a B, and 2 for a C, what is the weighted mean grade for the semester?(((2.76))
A sample of the paramedical fees charged by clinics revealed these amounts: $55, $49, $50, $45, $52, and $55. What is the median charge?((($51.00 ))
The times (in minutes) that several underwriters took to review applications for similar insurance coverage are 50, 230, 52, and 57. What is the median length of time required to review an application?(((54.5))
"A bottling company offers three kinds of delivery service: instant, same day, and within five days. The profit per delivery varies according to the kind of delivery. The profit for an instant delivery is less than the other kinds because the driver has to go directly to a grocery store with a small load and return to the bottling plant. To find out what effect each type of delivery has on the profit picture, the company summarized the data in the following table based on deliveries for the previous quarter.
What is the weighted mean profit per delivery?
"((($97 ))
Sometimes, a data set has two values that have the highest and equal frequencies. In this case, the distribution of the data can best be described as ________.(((bimodal (having two modes)))
A disadvantage of using an arithmetic mean to summarize a set of data is that ________.(((it can be biased by one or two extremely small or large values))
The mean, as a measure of central location, would be inappropriate for which one of the following?(((Marital status of college students at a particular university))
What is a disadvantage of the range as a measure of dispersion?(((It is based on only two observations.))
The sum of the differences between sample observations and the sample mean is equal to ________.(((zero))
If the variance of the "number of daily parking tickets" issued is 100, the standard deviation is defined as the ________.(((square root of the variance of the "number of daily parking tickets"))
What is the relationship between the variance and the standard deviation?(((Variance is the square of the standard deviation.))
For a sample of similar-sized all-electric homes, the March electric bills were (to the nearest dollar): $212, $191, $176, $129, $106, $92, $108, $109, $103, $121, $175, and $194. What is the range?((($120 ))
The following are the weekly amounts of welfare payments made by the federal government to a sample of six families: $139, $136, $130, $136, $147, and $136. What is the range?((($17 ))
The monthly amounts spent for food by families of four receiving food stamps approximates a symmetrical, normal distribution. The sample mean is $150 and the standard deviation is $20. Using the Empirical rule, about 95% of the monthly food expenditures are between what two amounts?((($110 and $190))
The ages of all the patients in the isolation ward of the hospital are 38, 26, 13, 41, and 22. What is the population variance?(((106.8))
A sample of small bottles and their contents has the following weights (in grams): 4, 2, 5, 4, 5, 2, and 6. What is the sample variance of bottle weight?(((2.33))
The distribution of a sample of the outside diameters of PVC pipes approximates a symmetrical, bell-shaped distribution. The arithmetic mean is 14.0 inches, and the standard deviation is 0.1 inches. About 68% of the outside diameters lie between what two amounts?(((13.9 and 14.1 inches))
The sample variance of hourly wages was 10. What is the sample standard deviation?((($3.16 ))
Based on the Empirical rule, what percent of the observations will lie between plus or minus two standard deviations from the mean?(((95%))
A sample of wires coming off the production line was tested for tensile strength. The statistical results (in PSI) were the following:According to the Empirical rule, the middle 95% of the wires tested had a tensile strength between approximately what two values?(((420 and 580))
In the calculation of the arithmetic mean for grouped data, which value is used to represent all the values in a particular class?(((The class midpoint))
The net annual sales of a sample of small retail clothing stores were organized into the following relative frequency distribution.What is the mean net sales (in $ millions)?(((Mean cannot be computed.))
Consider two populations with the same mean. Since they have the same mean, then(((none of these is correct.))
The sample mean(((is found by adding all data values and dividing them by n.))
Which measure of central location is used to determine an average annual percent increase?(((Geometric mean))
The U.S. Federal Aviation Administration reported that passenger revenues on international flights increased from $528 million in 1986 to $5,100 million in 2009. What is the geometric mean annual percent increase in international passenger revenues?(((10.4))
The Investment Research Institute reported in its Mutual Fund Fact Book that the number of mutual funds increased from 5,725 in 1999 to 7,977 in 2009. What is the geometric mean annual percent increase in the number of funds?(((3.37))
Production of passenger cars in Japan increased from 3.94 million in 1999 to 6.74 million in 2009. What is the geometric mean annual percent increase?(((5.5))
The number of students at a local university increased from 2,500 students to 5,000 students in 10 years. Based on a geometric mean, the university grew at an average percentage rate of(((7.2 percent per year.))
The variance of a sample of 121 observations equals 441. The standard deviation of the sample equals(((21))
When computing the arithmetic mean, the smallest value in the data set(((can be any value.))
A dot plot is an easy way to represent the relationship between two variables.(((FALSE))
A dot plot is useful for quickly graphing frequencies in a small data set.(((TRUE))
A dot plot shows the symmetry of a distribution.(((TRUE))
A dot plot is useful for showing individual observations.(((TRUE))
A dot plot is useful for showing the range of the data.(((TRUE))
Quartiles divide a distribution into four equal parts.(((TRUE))
Quartiles divide a distribution into 10 equal parts.(((FALSE))
Percentiles divide a distribution into 100 equal parts.(((TRUE))
A student scored in the 85th percentile on a standardized test. This means that the student scored lower than 85% of all students who took the test.(((FALSE))
The 50th percentile of a distribution is the same as the distribution mean.(((FALSE))
A box plot graphically shows the 10th and 90th percentiles.(((FALSE))
The "box" in a box plot shows the interquartile range.(((TRUE))
A box plot shows the skewness of a distribution.(((TRUE))
The coefficient of skewness is the standard deviation divided by the mean.(((FALSE))
If a distribution is negatively skewed, the distribution is not symmetrical and the long tail is to the left.(((TRUE))
A scatter diagram of sales versus production is labeled with sales on the Y-axis and production on the X-axis.(((TRUE))
A scatter diagram of sales versus production may be constructed by plotting the minimum, first quartile, median, third quartile, and maximum values of each variable.(((FALSE))
A scatter diagram is used to illustrate a relationship between gender and the preference for Coke or Pepsi.(((FALSE))
A relationship between two nominal variables is summarized by a contingency table.(((TRUE))
A stem-and-leaf diagram shows the actual data values.(((TRUE))
In a stem-and-leaf display, the leaf represents a class of a frequency distribution.(((FALSE))
In a stem-and-leaf display, the leaf represents the members of a class in a frequency distribution.(((TRUE))
A dot plot shows ________.(((the general shape of a distribution))
A dot plot is best applied when ________.(((a single variable is summarized))
A dot plot is best applied for a data set with ________.(((50 observations))
A dot plot can be used to show ________.(((the distribution for a quantitative variable))
The following graph is a ________.(((Dot plot))
The test scores for a class of 147 students are computed. What is the location of the test score associated with the third quartile?(((111))
Quartiles divide a distribution into ________.(((4 equal parts))
In a distribution, the second quartile corresponds with the __________.(((median))
Percentiles divide a distribution into ________.(((100 equal parts))
To locate the percentile for a given observation in a data set, the data must be ________.(((sorted and listed from the minimum to the maximum values))
If a student places in the 99th percentile on an exam, she performed better than 99% of all students who completed the exam. Her performance is similar to a statement based on a ________.(((cumulative frequency distribution))
In the following set of data: (1, 3, 5, 6, 7, 9, 100), what are the first, second, and third quartiles?(((3, 6, and 9))
The following graph is a ________.(((box plot))
What statistics are needed to draw a box plot?(((The minimum, maximum, median, first and third quartiles))
A box plot shows ________.(((the relative symmetry of a distribution for a set of data))
What does the interquartile range describe?(((The range of the middle 50% of the observations))
The interquartile range is graphically presented in a ________.(((box plot))
Outliers are clearly presented in a ________.(((box plot))
Using the following statistics to describe a distribution of data, what is the interquartile range? Minimum = 10 Q1 = 25 Median = 50 Q3 = 75 Maximum = 95 (((50)))
Minimum = 10
Q1 = 25
Median = 50
Q3 = 75
Maximum = 95
"(((50))
If Pearson’s coefficient of skewness is equal to zero, the shape of the distribution is ________.(((symmetric))
The range of the coefficient of skewness is ________.(((both positive and negative values))
The following graph is ________.(((positively skewed))
What is the value of the Pearson's coefficient of skewness for a distribution with a mean of 17, a median of 12, and a standard deviation of 6?(((+2.5))
A sample of experienced typists revealed that their mean typing speed is 87 words per minute and the median typing speed is 73 words per minute. The standard deviation of typing speed is 16.9 words per minute. What is the Pearson coefficient of skewness?(((+2.5))
The following graph is a ________.(((scatter diagram))
The following graph illustrates ________.(((a positive or direct relationship))
In a scatter diagram, we describe the relationship between ________.(((two variables measured at the interval or ratio level))
In a contingency table, we describe the relationship between ________.(((two variables measured at the ordinal or nominal level))
A contingency table would be used to summarize data such as ________.(((company employees by gender and organizational title))
A stem-and-leaf display includes the following row: 3 | 0 1 3 5 7 9. Assume that the data are rounded to the nearest whole number. Which of the following statements is true?(((The maximum value in the class is 39.))
The following table is a ________.(((stem-and-leaf display))
A stem-and-leaf display includes the following row: 5 | 10 11 31 52 79 98. Assume that the data are rounded to the nearest whole number.(((The class interval is 100.))
In the following table,(((the range is 33.))
In the following table,(((there are 4 observations in the second class.))
The median of a sample will always equal the ________.(((50th percentile))
The 75th percentile is referred to as the ________.(((third quartile))
The 67th percentile is ________.(((the value below which 67% of the observations occur))
The eighth decile ________.(((is the same as the 80th percentile))
Which of the following is NOT a measure of dispersion?(((The 50th percentile))
What is the difference between a histogram and a bar chart? ((A histogram is used to display quantitative data and a bar chart is used to display qualitative data.))
The probability of rolling a 3 or 2 on a single die is an example of conditional probability. ((( FALSE )))
The probability of rolling a 3 or 2 on a single die is an example of mutually exclusive events. ((( TRUE )))
To apply the special rule of addition, the events must be mutually exclusive. ((( TRUE )))
A joint probability measures the likelihood that two or more events will happen concurrently. ((( TRUE )))
The joint probability of two independent events, A and B, is computed as P(A and B) = P(A) × P(B). ((( TRUE )))
The joint probability of two events, A and B, that are NOT independent is computed as ((( TRUE )))
A coin is tossed four times. The joint probability that all four tosses will result in a head is 1/4 or 0.25. ((( FALSE )))
If there are "m" ways of doing one thing, and "n" ways of doing another thing, the multiplication formula states that there are (m) × (n) ways of doing both. ((( TRUE )))
A combination of a set of objects is defined by the order of the objects. ((( FALSE )))
The complement rule states that the probability of an event not occurring is equal to 1 minus the probability of its occurrence. ((( TRUE )))
If two events are mutually exclusive, then P(A and B) = P(A) × P(B). ((( FALSE )))
An illustration of an experiment is turning the ignition key of an automobile as it comes off the assembly line to determine whether or not the engine will start. ((( TRUE )))
The National Center for Health Statistics reported that of every 883 deaths in recent years, 24 resulted from an automobile accident, 182 from cancer, and 333 from heart disease. What is the probability that a particular death is due to an automobile accident? ((( 24/883 or 0.027 )))
If two events A and B are mutually exclusive, what does the special rule of addition state? ((( P(A or B) = P(A) + P(B) )))
What does the complement rule state? ((( P(A) = 1 – P(not A) )))
Which approach to probability is exemplified by the following formula? Probability of an event :No of time event occurred in the post/Total number of observationsThe empirical approach ((( The empirical approach )))
"A study of 200 computer service firms revealed these incomes after taxes: Income After Taxes Number of Firms
Under $1 million 102
$1 million up to $20 million 61
$20 million or more 37 What is the probability that a particular firm selected has $1 million or more in income after taxes?" ((( 0.49 )))
A firm offers routine physical examinations as part of a health service program for its employees. The exams showed that 8% of the employees needed corrective shoes, 15% needed major dental work, and 3% needed both corrective shoes and major dental work. What is the probability that an employee selected at random will need either corrective shoes or major dental work? ((( 0.20 )))
A survey of top executives revealed that 35% of them regularly read Time magazine, 20% read Newsweek, and 40% read U.S. News & World Report. A total of 10% read both Time and U.S. News & World Report. What is the probability that a particular top executive reads either Time or U.S. News & World Report regularly? ((( 0.65 )))
A study by the National Park Service revealed that 50% of the vacationers going to the Rocky Mountain region visit Yellowstone Park, 40% visit the Grand Tetons, and 35% visit both. What is the probability that a vacationer will visit at least one of these magnificent attractions? ((( 0.55 )))
A tire manufacturer advertises, "The median life of our new all-season radial tire is 50,000 miles. An immediate adjustment will be made on any tire that does not last 50,000 miles." You purchased four of these tires. What is the probability that all four tires will wear out before traveling 50,000 miles? ((( 1/16 or 0.0625 )))
A sales representative calls on four hospitals in Westchester County. It is immaterial what order he calls on them. How many ways can he organize his calls? ((( 24 )))
There are 10 AAA batteries in a box and 3 are defective. Two batteries are selected without replacement. What is the probability of selecting a defective battery followed by another defective battery? ((( 1/120 or about 0.0083 )))
Giorgio offers the person who purchases an 8-ounce bottle of Allure two free gifts, chosen from the following: an umbrella, a 1-ounce bottle of Midnight, a feminine shaving kit, a raincoat, or a pair of rain boots. If you purchased Allure, what is the probability you randomly select an umbrella and a shaving kit in that order? ((( 0.05 )))
A board of directors consists of eight men and four women. A four-member search committee is randomly chosen to recommend a new company president. What is the probability that all four members of the search committee will be women? ((( 1/495 or 0.002 )))
A lamp manufacturer designed five lamp bases and four lampshades that could be used together. How many different arrangements of base and shade can be offered? ((( 20 )))
A gumball machine has just been filled with 50 black, 150 white, 100 red, and 100 yellow gumballs that have been thoroughly mixed. Sue and Jim each purchase one gumball. What is the likelihood that both Sue and Jim will get red gumballs? ((( 0.062 )))
What does 6!2!/4!3! Equal? ((( 10 )))
In a management trainee program, 80% of the trainees are female, while 20% are male. Ninety percent of the females attended college; 78% of the males attended college. A management trainee is selected at random. What is the probability that the person selected is a female who did NOT attend college? ((( 0.08 )))
In a management trainee program, 80% of the trainees are female, while 20% are male. Ninety percent of the females attended college; 78% of the males attended college. A management trainee is selected at random. What is the probability that the person selected is a female who attended college? ((( 0.72 )))
In a management trainee program, 80% of the trainees are female, while 20% are male. Ninety percent of the females attended college; 78% of the males attended college. A management trainee is selected at random. What is the probability that the person selected is a male who did NOT attend college? ((( 0.044 )))
In a management trainee program, 80% of the trainees are female, while 20% are male. Ninety percent of the females attended college; 78% of the males attended college. A management trainee is selected at random. What is the probability that the person selected is a male who did NOT attend college? ((( P (male) P (did not attend college | male) )))
In a management trainee program, 80% of the trainees are female, while 20% are male. Ninety percent of the females attended college; 78% of the males attended college. A management trainee is selected at random. What is the probability that the person selected is a female who attended college? ((( P (female) P (did attend college | female) )))
A supplier delivers an order for 20 electric toothbrushes to a store. By accident, three of the electric toothbrushes are defective. What is the probability that the first two electric toothbrushes sold are defective? ((( 6/380 or 0.01579 )))
An electronics firm sells four models of stereo receivers, three amplifiers, and six speaker brands. When the four types of components are sold together, they form a "system." How many different systems can the electronics firm offer? ((( 72 )))
The numbers 0 through 9 are used in code groups of four to identify an item of clothing. Code 1083 might identify a blue blouse, size medium. The code group 2031 might identify a pair of pants, size 18, and so on. Repetitions of numbers are not permitted—in other words, the same number cannot be used more than once in a total sequence. As examples, 2,256, 2,562, or 5,559 would not be permitted. How many different code groups can be designed? ((( 5,040 )))
How many permutations of the three letters C, D, and E are possible? ((( 6 )))
You are assigned to design color codes for different parts. Three colors are used to code on each part. Once a combination of three colors is used—such as green, yellow, and red—these three colors cannot be rearranged to use as a code for another part. If there are 35 combinations, how many colors are available? ((( 7 )))
A developer of a new subdivision wants to build homes that are all different. There are three different interior plans that can be combined with any of five different home exteriors. How many different homes can be built? ((( 15 )))
Six basic colors are used in decorating a new condominium. They are applied to a unit in groups of four colors. One unit might have gold as the principal color, blue as a complementary color, red as the accent color, and touches of white. Another unit might have blue as the principal color, white as the complementary color, gold as the accent color, and touches of red. If repetitions are permitted, how many different units can be decorated? ((( 1,296 )))
The ABCD football association is considering a Super Ten Football Conference. The top 10 football teams in the country, based on past records, would be members of the Super Ten Conference. Each team would play every other team in the conference during the season and the team winning the most games would be declared the national champion. How many games would the conference commissioner have to schedule each year? (Remember, Oklahoma versus Michigan is the same as Michigan versus Oklahoma.) ((( 45 )))
A rug manufacturer has decided to use seven compatible colors in her rugs. However, in weaving a rug, only five spindles can be used. In advertising, the rug manufacturer wants to indicate the number of different color groupings for sale. How many color groupings using the seven colors taken five at a time are there? (This assumes that five different colors will go into each rug—in other words, there are no repetitions of color.) ((( 21 )))
The first card selected from a standard 52-card deck was a king. If it is returned to the deck, what is the probability that a king will be drawn on the second selection? ((( 1/13 or 0.077 )))
The first card selected from a standard 52-card deck was a king. If it is NOT returned to the deck, what is the probability that a king will be drawn on the second selection? ((( 3/51 or 0.0588 )))
Which approach to probability assumes that events are equally likely? ((( Classical )))
An experiment may have ____________. ((( one or more outcomes )))
When are two experimental outcomes mutually exclusive? ((( If one outcome occurs, then the other cannot. )))
Probabilities are important information when ________. ((( using inferential statistics )))
The result of a particular experiment is called a(n) ___________. ((( outcome )))
The probability of two or more events occurring concurrently is called a(n) ________. ((( joint probability )))
The probability of a particular event occurring, given that another event has occurred, is known as a(n) ________. ((( conditional probability )))
A graphical method used to calculate joint and conditional probabilities is ________. ((( a tree diagram )))
When an experiment is conducted "without replacement," ________. ((( events are dependent )))
If two events are independent, then their joint probability is computed with ________. ((( the special rule of multiplication )))
When applying the special rule of addition for mutually exclusive events, the joint probability is ________. ((( 0 )))
A group of employees of Unique Services will be surveyed about a new pension plan. In-depth interviews with each employee selected in the sample will be conducted. The employees are classified as follows: Classification Event Number of Employees Supervisors A 120 Maintenance B 50 Production C 1,460 Management D 302 Secretarial E 68 What is the probability that the first person selected is classified as a maintenance employee? ((( 0.025 )))
A group of employees of Unique Services will be surveyed about a new pension plan. In-depth interviews with each employee selected in the sample will be conducted. The employees are classified as follows: Classification Event Number of Employees Supervisors A 120 Maintenance B 50 Production C 1,460 Management D 302 Secretarial E 68 What is the probability that the first person selected is either in maintenance or in secretarial? ((( 0.059 )))
A group of employees of Unique Services will be surveyed about a new pension plan. In-depth interviews with each employee selected in the sample will be conducted. The employees are classified as follows: Classification Event Number of Employees Supervisors A 120 Maintenance B 50 Production C 1,460 Management D 302 Secretarial E 68 What is the probability that the first person selected is either in management or in supervision? ((( 0.21 )))
Each salesperson in a large department store chain is rated on their sales ability and their potential for advancement. The data for the 500 sampled salespeople are summarized in the following table. Potential for Advancement Fair Good Excellent Sales ability Below average 16 12 22 Average 45 60 45 Above average 93 72 135 What is the probability that a salesperson selected at random has above-average sales ability and has excellent potential for advancement? ((( 0.27 )))
Each salesperson in a large department store chain is rated on his or her sales ability and potential for advancement. The data for the 500 sampled salespeople are summarized in the following table. Potential for Advancement Fair Good Excellent Sales ability Below average 16 12 22 Average 45 60 45 Above average 93 72 135 What is the probability that a salesperson selected at random will have average sales ability and good potential for advancement? ((( 0.12 )))
Each salesperson in a large department store chain is rated on his or her sales ability and potential for advancement. The data for the 500 sampled salespeople are summarized in the following table. Potential for Advancement Fair Good Excellent Sales ability Below average 16 12 22 Average 45 60 45 Above average 93 72 135 What is the probability that a salesperson selected at random will have below-average sales ability and fair potential for advancement? ((( 0.032 )))
Each salesperson in a large department store chain is rated on his or her sales ability and potential for advancement. The data for the 500 sampled salespeople are summarized in the following table. Potential for Advancement Fair Good Excellent Sales ability Below average 16 12 22 Average 45 60 45 Above average 93 72 135 What is the probability that a salesperson selected at random will have an excellent potential for advancement given he or she also has above-average sales ability? ((( 0.45 )))
Each salesperson in a large department store chain is rated on his or her sales ability and potential for advancement. The data for the 500 sampled salespeople are summarized in the following table. Potential for Advancement Fair Good Excellent Sales ability Below average 16 12 22 Average 45 60 45 Above average 93 72 135 What is the probability that a salesperson selected at random will have an excellent potential for advancement given he or she also has average sales ability? ((( 0.30 )))
An automatic machine inserts mixed vegetables into a plastic bag. Past experience revealed that some packages were underweight and some were overweight, but most of them had satisfactory weight. Weight % of Total Underweight 2.5 Satisfactory 90.0 Overweight 7.5 What is the probability of selecting three packages that are overweight? ((( 0.0004219 )))
An automatic machine inserts mixed vegetables into a plastic bag. Past experience revealed that some packages were underweight and some were overweight, but most of them had satisfactory weight. Weight % of Total Underweight 2.5 Satisfactory 90.0 Overweight 7.5 What is the probability of selecting three packages that are satisfactory? ((( 0.729 )))
In a finance class, the final grade is based on three tests. Historically, the instructor tells the class that the joint probability of scoring As on the first two tests is 0.5. A student assigns a probability of 0.9 that she will get an A on the first test. What is the probability that the student will score an A on the second test given that she scored an A on the first test? ((( 0.56 )))
If A and B are mutually exclusive events with P(A) = 0.2 and P(B) = 0.6, then P(A or B) = ________. ((( 0.80 )))
If P(A) = 0.62, P(B) = 0.47, and P(A or B) = 0.88, then P(A and B) = ________. ((( 0.2100 )))
A method of assigning probabilities based upon judgment, opinions, and available information is referred to as the ________. ((( subjective method )))
Your favorite soccer team has two remaining matches to complete the season. The possible outcomes of a soccer match are win, lose, or tie. What is the possible number of outcomes for the season? ((( 9 )))
A random variable represents the outcome of an experiment. ((( TRUE )))
The probability of a particular outcome must always be between 0.0 and 1.0 inclusive. ((( TRUE )))
The random variable for a Poisson probability distribution can assume an infinite number of values. ((( TRUE )))
A probability distribution is a mutually exclusive and collectively exhaustive listing of experimental outcomes that can occur by chance, and their corresponding probabilities. ((( TRUE )))
To construct a binomial probability distribution, the mean must be known. ((( FALSE )))
To construct a binomial distribution, it is necessary to know the total number of trials and the probability of success on each trial. ((( TRUE )))
The mean of a probability distribution is called its expected value. ((( TRUE )))
The variance of a probability distribution is based on the sum of squared differences from the mean multiplied by the probability of X. ((( TRUE )))
The variance measures the skewness of a probability distribution. ((( FALSE )))
The mean of a binomial distribution is the product of n and π. ((( TRUE )))
The variance of a binomial distribution is found by nπ(1 - π). ((( TRUE )))
In a Poisson distribution, the probability of success may vary from trial to trial. ((( FALSE )))
The binomial probability distribution is always negatively skewed. ((( FALSE )))
When sampling is done without replacement from a finite population and the outcomes are measured as either a success or failure, the hypergeometric distribution should be applied. ((( TRUE )))
As a general rule of thumb, if the items selected for a sample are not replaced and the sample size is less than 5% of the population, the binomial distribution can be used to approximate the hypergeometric distribution. ((( TRUE )))
If the variance is 3.6 grams, what is the standard deviation? ((( 1.897 )))
A total of 60% of the customers of a fast-food chain order a hamburger, french fries, and a drink. If a random sample of 15 cash register receipts is selected, what is the probability that 10 or more will show that the above three food items were ordered? ((( 0.403 )))
Judging from recent experience, 5% of the computer keyboards produced by an automatic, high-speed machine are defective. If six keyboards are randomly selected, what is the probability that none of the keyboards are defective? ((( 0.735 )))
"The probability distribution for the number of automobiles lined up at a Lakeside Olds dealer at opening time (7:30 a.m.) for service is:
Number Probability
1 0.05
2 0.30
3 0.40
4 0.2
On a typical day, how many automobiles should Lakeside Olds expect to be lined up at opening time?
" ((( 2.85 )))
On a very hot summer day, 5% of the production employees at Midland States Steel are absent from work. The production employees are randomly selected for a special in-depth study on absenteeism. What is the probability of randomly selecting 10 production employees on a hot summer day and finding that none of them are absent? ((( 0.599 )))
Sweetwater & Associates write weekend trip insurance at a very nominal charge. Records show that the probability that a motorist will have an accident during the weekend and file a claim is 0.0005. Suppose they wrote 400 policies for the coming weekend, what is the probability that exactly two claims will be filed? ((( 0.0614 )))
A listing of all possible outcomes of an experiment and their corresponding probabilities of occurrence is called a ____________. ((( probability distribution )))
Which one of the following is NOT a condition of the binomial distribution? ((( Sampling at least 10 trials )))
Which is true for a binomial distribution? ((( The probability of success remains the same from trial to trial. )))
Which shape describes a Poisson distribution? ((( Positively skewed )))
Sponsors of a local charity decided to attract wealthy patrons to its $500-a-plate dinner by allowing each patron to buy a set of 20 tickets for the gaming tables. The chance of winning a prize for each of the 20 plays is 50–50. If you bought 20 tickets, what is the chance of winning 15 or more prizes? ((( 0.021 )))
What kind of distributions are the binomial and Poisson probability distributions? ((( Discrete )))
Which of the following is correct about a probability distribution? ((( All of these answers are correct. )))
Data show that the weight of an offensive linesman may be any weight between 200 and 350 pounds. The distribution of weight is based on a ________. ((( Continuous random variable )))
Carlson Jewelers permits the return of their diamond wedding rings, provided the return occurs within two weeks of the purchase date. Their records reveal that 10% of the diamond wedding rings are returned. Five different customers buy a wedding ring. What is the probability that none of the customers return a ring? ((( 0.590 )))
In a large metropolitan area, past records revealed that 30% of all the high school graduates go to college. From 20 graduates selected at random, what is the probability that exactly 8 will go to college? ((( 0.114 )))
Chances are 50–50 that a newborn baby will be a girl. For families with five children, what is the probability that all the children are girls? ((( 0.031 )))
A new car was put into production. It involved many assembly tasks. Each car was inspected at the end of the assembly line and the number of defects per unit was recorded. For the first 100 cars produced, there were 40 defective cars. Some of the cars had no defects, a few had one defect, and so on. The distribution of defects followed a Poisson distribution. Based on the first 100 cars produced, about how many out of every 1,000 cars assembled should have one or more defects? ((( About 330 )))
The production department has installed a new spray machine to paint automobile doors. As is common with most spray guns, unsightly blemishes often appear because of improper mixtures or other problems. A worker counted the number of blemishes on each door. Most doors had no blemishes; a few had one; a very few had two; and so on. The average number was 0.5 per door. The distribution of blemishes followed the Poisson distribution. Out of 10,000 doors painted, about how many would have no blemishes? ((( About 6,065 )))
A manufacturer of headache medicine claims it is 70% effective within a few minutes. That is, out of every 100 users, 70 get relief within a few minutes. A group of 12 patients are given the medicine. If the claim is true, what is the probability that eight have relief within a few minutes? ((( 0.231 )))
A true/false test consists of six questions. If you guess the answer to each question, what is the probability of getting all six questions correct? ((( 0.016 )))
A farmer who grows genetically engineered corn is experiencing trouble with corn borers. A random check of 5,000 ears revealed the following: Many of the ears contained no borers. Some ears had one borer. A few had two borers, and so on. The distribution of the number of borers per ear approximated the Poisson distribution. The farmer counted 3,500 borers in the 5,000 ears. What is the probability that an ear of corn selected at random will contain no borers? ((( 0.4966 )))
A tennis match requires that a player win three of five sets to win the match. If a player wins the first two sets, what is the probability that the player wins the match, assuming that each player is equally likely to win each set? ((( 0.875 )))
A machine shop has 100 drill presses and other machines in constant use. The probability that a machine will become inoperative during a given day is 0.002. During some days, no machines are inoperative, but on other days, one, two, three, or more are broken down. What is the probability that fewer than two machines will be inoperative during a particular day? ((( 0.9824 )))
A coin is tossed three times. The following table summarizes the experiment. What is the following table called? Number of Heads Probability of the Outcome 0 1/8 = 0.125 1 3/8 = 0.375 2 3/8 = 0.375 3 1/8 = 0.125 ((( Probability distribution )))
What is the only variable in the Poisson probability formula? ((( x )))
Which of the following is NOT a characteristic of a binomial probability distribution? ((( The number of trials is limited to two. )))
What must you know to develop a binomial probability distribution? ((( The probability of success and the number of trials )))
To apply a Poisson probability distribution, the mean can be computed as ________. ((( nπ )))
In a Poisson distribution, the variance is equal to ________. ((( nπ )))
A statistics professor receives an average of five e-mail messages per day from students. Assume the number of messages approximates a Poisson distribution. What is the probability that on a randomly selected day she will have no messages? ((( 0.0067 )))
A statistics professor receives an average of five e-mail messages per day from students. Assume the number of messages approximates a Poisson distribution. What is the probability that on a randomly selected day she will have five messages? ((( 0.1755 )))
A statistics professor receives an average of five e-mail messages per day from students. Assume the number of messages approximates a Poisson distribution. What is the probability that on a randomly selected day she will have two messages? ((( 0.0842 )))
A company is studying the number of monthly absences among its 125 employees. The following probability distribution shows the likelihood that people were absent 0, 1, 2, 3, 4, or 5 days last month. Number of Days Absent Probability 0 0.60 1 0.20 2 0.12 3 0.04 4 0.04 5 0.00 What is the mean number of days absent? ((( 0.72 )))
A company is studying the number of monthly absences among its 125 employees. The following probability distribution shows the likelihood that people were absent 0, 1, 2, 3, 4, or 5 days last month. Number of Days Absent Probability 0 0.60 1 0.20 2 0.12 3 0.04 4 0.04 5 0.00 What is the variance of the number of days absent? ((( 1.1616 )))
A company is studying the number of monthly absences among its 125 employees. The following probability distribution shows the likelihood that people were absent 0, 1, 2, 3, 4, or 5 days last month. Number of Days Absent Probability 0 0.60 1 0.20 2 0.12 3 0.04 4 0.04 5 0.00 Given the probability distribution, which of the following predictions is correct? ((( There is a 0.12 probability that an employee will be absent two days per month. )))
A company is studying the number of monthly absences among its 125 employees. The following probability distribution shows the likelihood that people were absent 0, 1, 2, 3, 4, or 5 days last month. Number of Days Absent Probability 0 0.60 1 0.20 2 0.12 3 0.04 4 0.04 5 0.00 What is the mode of the distribution? ((( 0 days )))
"A company is studying the number of monthly absences among its 125 employees. The following probability distribution shows the likelihood that people were absent 0, 1, 2, 3, 4, or 5 days last month.
Number of Days Absent Probability
0 0.60
1 0.20
2 0.12
3 0.04
4 0.04
5 0.00
What is the standard deviation of the number of days absent?
" ((( 1.0778 )))
For the following distribution: X P(x) 0 0.130 1 0.346 2 0.346 3 0.154 4 0.026 What is the mean of the distribution? ((( 1.604 )))
For the following distribution: X P(x) 0 0.130 1 0.346 2 0.346 3 0.154 4 0.026 What is the variance of the distribution? ((( 0.964 )))
For the following distribution: X P(x) 0 0.027 1 0.189 2 0.441 3 0.343 What is the mean of the distribution? ((( 2.1 )))
For the following distribution: x P(x) 0 0.027 1 0.189 2 0.441 3 0.343 What is the variance of the distribution? ((( 0.63 )))
For the following distribution: x P(x) 0 0.900 1 0.09 2 0.007 3 0.003 What is the mean of the distribution? ((( 0.113 )))
For the following distribution: X P(x) 0 0.900 1 0.09 2 0.007 3 0.003 What is the variance of the distribution? ((( 0.132 )))
The following is a binomial probability distribution with n = 3 and π = 0.20. X P(x) 0 0.512 1 0.384 2 0.096 3 0.008 The mean of the distribution is ________. ((( 0.60 )))
The following is a binomial probability distribution with n = 3 and π = 0.20. X P(x) 0 0.512 1 0.384 2 0.096 3 0.008 The variance of the distribution is ________. ((( 0.48 )))
The following is a Poisson probability distribution with µ = 0.1. x P(x) 0 0.9048 1 0.0905 2 0.0045 3 0.0002 The mean of the distribution is ________. ((( 0.1 )))
The following is a Poisson probability distribution with µ = 0.1. x P(x) 0 0.9048 1 0.0905 2 0.0045 3 0.0002 The variance of the distribution is ________. ((( 0.1 )))
For a binomial distribution, the mean is 0.6 and n = 2. What is π for this distribution? ((( 0.3 )))
For a binomial distribution, the mean is 4.0 and n = 8. What is π for this distribution? ((( 0.5 )))
The marketing department of a nationally known cereal maker plans to conduct a national survey to find out whether or not consumers of flake cereals can distinguish one of their favorite flake cereals. To test the questionnaire and procedure to be used, eight persons were asked to cooperate in an experiment. Five very small bowls of flake cereals were placed in front of a person. The bowls were labeled A, B, C, D, and E. The person was informed that only one bowl contained his or her favorite flake cereal. Suppose that the eight persons in the experiment were unable to identify their favorite cereal and just guessed which bowl it was in. What is the probability that none of the eight guessed correctly? ((( 0.168 )))
An insurance agent has appointments with four prospective clients tomorrow. From past experience the agent knows that the probability of making a sale on any appointment is one out of five. Using the rules of probability, what is the likelihood that the agent will sell a policy to three of the four prospective clients? ((( 0.026 )))
In which of the following discrete distributions does the probability of a success vary from one trial to the next? ((( Hypergeometric )))
Which of the following is a requirement for use of the hypergeometric distribution? ((( Only two possible outcomes )))
Affirmative action commitments by many organizations have led to an increase in the number of women in executive positions. Satellite Office Systems has vacancies for two executives that it will fill from among four women and six men. What is the probability that no woman is selected? ((( 1/3 )))
Affirmative action commitments by many organizations have led to an increase in the number of women in executive positions. Satellite Office Systems has vacancies for two executives that it will fill from among four women and six men. What is the probability that at least one woman is selected? ((( 2/3 )))
A type of probability distribution that shows the probability of x successes in n trials, where the probability of success remains the same from trial to trial, is referred to as a ________. ((( binomial probability distribution )))
A measure of the long-run average value of a random variable used to represent the central location of a probability distribution is called a(n) ________. ((( expected value )))
An experiment consists of making 80 telephone calls in order to sell a particular insurance policy. The random variable in this experiment is a ________. ((( discrete random variable )))
The mean or expected value for a binomial probability distribution is ________. ((( μ = nπ )))
For any uniform probability distribution, the mean and standard deviation can be computed based on the maximum and minimum values of the random variable. ((( TRUE )))
For a uniformly distributed random variable, x, P(x) is constant. ((( TRUE )))
For a uniformly distributed random variable, x, P(x) = 1/(b - a). ((( TRUE )))
When referring to the normal probability distribution, there is not just one; there is a "family" of normal probability distributions. ((( TRUE )))
A group of normal distributions can have equal arithmetic means but different standard deviations. ((( TRUE )))
Some normal probability distributions are positively skewed. ((( FALSE )))
Within plus and minus one standard deviation of the mean, the area under any normal curve is about 68%. ((( TRUE )))
The standard normal distribution is a special normal distribution with a mean of 0 and a standard deviation of 1. ((( TRUE )))
The z-scores for X values greater than the mean are negative. ((( FALSE )))
Nonstop Airlines determined that the mean number of passengers per flight is 152 with a standard deviation of 10 passengers. Practically all flights have between 142 and 162 passengers. ((( FALSE )))
The number of different standard normal distributions is unlimited. ((( FALSE )))
The normal distribution can be used to approximate a binomial distribution when nπ is less than 5. ((( FALSE )))
A "continuity correction factor" is used to compute probabilities when using the normal distribution to approximate the binomial distribution. ((( TRUE )))
For the exponential distribution, the mean and standard deviation are equal. ((( TRUE )))
The exponential distribution is a probability distribution for a random variable measured as a rate of an event occurring. ((( TRUE )))
The shape of any uniform probability distribution is ________. ((( rectangular )))
The mean of any uniform probability distribution is ________. ((( (a + b)/2 )))
The standard deviation of any uniform probability distribution is ________. ((( √(b-a)2/12 )))
The upper and lower limits of a uniform probability distribution are ________. ((( the maximum and minimum values of the random variable )))
The time to fly between New York City and Chicago is uniformly distributed with a minimum of 120 minutes and a maximum of 150 minutes. What is the probability that a flight is less than 135 minutes? ((( 0.50 )))
The time to fly between New York City and Chicago is uniformly distributed with a minimum of 120 minutes and a maximum of 150 minutes. What is the probability that a flight is more than 140 minutes? ((( 0.333 )))
The time to fly between New York City and Chicago is uniformly distributed with a minimum of 120 minutes and a maximum of 150 minutes. What is the probability that a flight is between 125 and 140 minutes? ((( 0.50 )))
What is an important similarity between the uniform and normal probability distributions? ((( The mean and median are equal. )))
Which of the following is true regarding the normal distribution? ((( The mean, median, and mode are all equal. )))
For the normal distribution, the mean plus and minus two standard deviations will include about what percent of the observations? ((( 95% )))
For a standard normal distribution, what is the probability that z is greater than 1.75? ((( 0.0401 )))
What is the area under the normal curve between z = 0.0 and z = 1.79? ((( 0.4633 )))
What is the area under the normal curve between z = -1.0 and z = -2.0? ((( 0.1359 )))
What is the probability that z is between 0.0 and 2.0? ((( 0.4772 )))
The mean amount spent by a family of four on food is $500 per month with a standard deviation of $75. Assuming that the food costs are normally distributed, what is the probability that a family spends less than $410 per month? ((( 0.1151 )))
For a normal distribution, what is the likelihood (expressed as a percentage) that a random variable is within plus and minus two standard deviations of the mean? ((( 95.44% )))
The mean score of a college entrance test is 500; the standard deviation is 75. The scores are normally distributed. What percent of the students scored below 320? ((( About 0.82% )))
The weekly incomes of a large group of executives are normally distributed with a mean of $2,000 and a standard deviation of $100. What is the z-score for an income of $2,100? ((( +1.000 )))
A new extended-life light bulb has an average life of 750 hours, with a standard deviation of 50 hours. If the life of these light bulbs approximates a normal distribution, about what percent of the distribution will be between 600 hours and 900 hours? ((( 99.74% )))
An accelerated life test on a large number of type-D alkaline batteries revealed that the mean life for a particular use before they failed is 19.0 hours. The distribution of the lives approximated a normal distribution. The standard deviation of the distribution was 1.2 hours. About 95.44% of the batteries failed between what two values? ((( 16.6 and 21.4 )))
The mean of a normal distribution is 400 pounds. The standard deviation is 10 pounds. What is the probability of a weight between 415 pounds and the mean of 400 pounds? ((( 0.4332 )))
The distribution of the annual incomes of a group of middle management employees approximated a normal distribution with a mean of $37,200 and a standard deviation of $800. About 68% of the incomes lie between what two incomes? ((( $36,400 and $38,000 )))
Which of the following is true in a normal distribution? ((( The mean divides the distribution into two equal areas. )))
Tables of normal distribution probabilities are found in many statistics books. These probabilities are calculated from a normal distribution with ________. ((( a mean of 0 and a standard deviation of 1 )))
Two normal distributions are compared. One has a mean of 10 and a standard deviation of 10. The second normal distribution has a mean of 10 and a standard deviation of 2. Which of the following is true? ((( The dispersions of the distributions are different. )))
A normally distributed random variable has ________. ((( any value between -∞ and + ∞ )))
The total area of a normal probability distribution is _______. ((( 1.00 )))
In an illustration of a normal probability distribution, a shaded area represents _______. ((( a probability )))
The standard normal probability distribution is unique because it has ________. ((( a mean of 0 and a standard deviation of 1 )))
The weekly mean income of a group of executives is $1,000 and the standard deviation of this group is $100. The distribution is normal. What percent of the executives have an income of $925 or less? ((( About 23% )))
The weight of cans of fruit is normally distributed with a mean of 1,000 grams and a standard deviation of 50 grams. What percent of the cans weigh 860 grams or less? ((( 0.0026 )))
What is a normal distribution with a mean of 0 and a standard deviation of 1 called? ((( Standard normal distribution )))
The seasonal output of a new experimental strain of pepper plants was carefully weighed. The mean weight per plant is 15.0 pounds, and the standard deviation of the normally distributed weights is 1.75 pounds. Of the 200 plants in the experiment, how many produced peppers weighing between 13 and 16 pounds? ((( 118 )))
Ball-Bearings, Inc. produces ball bearings automatically on a Kronar BBX machine. For one of the ball bearings, the mean diameter is set at 20.00 mm (millimeters). The standard deviation of the production over a long period of time was computed to be 0.150 mm. What percent of the ball bearings will have diameters of 20.27 mm or more? ((( 3.59% )))
A national manufacturer of unattached garages discovered that the distribution of the time for two construction workers to erect the Red Barn model is normally distributed with a mean of 32 hours and a standard deviation of 2 hours. What percent of the garages take between 32 and 34 hours to erect? ((( 34.13% )))
A large manufacturing firm tests job applicants. Test scores are normally distributed with a mean of 500 and a standard deviation of 50. Management is considering placing a new hire in an upper-level management position if the person scores in the upper sixth percent of the distribution. What is the lowest score a new hire must earn to qualify for a responsible position? ((( 578 )))
An analysis of the grades on the first test in History 101 revealed that they approximate a normal curve with a mean of 75 and a standard deviation of 8. The instructor wants to award the grade of A to the upper 10% of the test grades. To the nearest percent, what is the dividing point between an A and a B grade? ((( 85 )))
The annual commissions per salesperson employed by a retailer of mobile communication devices averaged $40,000, with a standard deviation of $5,000. What percent of the salespersons earn between $32,000 and $42,000? ((( 60.06% )))
The mean of a normal probability distribution is 500 and the standard deviation is 10. About 95% of the observations lie between what two values? ((( 480 and 520 )))
A cola-dispensing machine is set to dispense a mean of 2.02 liters into a container labeled 2 liters. Actual quantities dispensed vary, and the amounts are normally distributed with a standard deviation of 0.015 liters. What is the probability a container will have less than 2 liters? ((( 0.0918 )))
The employees of Cartwright Manufacturing are awarded efficiency ratings. The distribution of the ratings approximates a normal distribution. The mean is 400, the standard deviation is 50. What is the area under the normal curve between 400 and 482? ((( 0.4495 )))
Customers of the Key Refining Company charge an average of $70 per month. The distribution of amounts charged is approximately normal, with a standard deviation of $10. What is the probability of selecting a credit card customer at random and finding the customer charged between $70 and $83? ((( 0.4032 )))
Management is considering a bonus system to increase production. One suggestion is to pay a bonus on the highest 5% of production based on past experience. Past records indicate that an average of 4,000 units of a small assembly is produced during a week. The distribution of the weekly production is approximately normally distributed with a standard deviation of 60 units. If the bonus is paid on the upper 5% of production, the bonus will be paid on how many units or more? ((( 4,099 )))
The average score of 100 students taking a statistics final was 70, with a standard deviation of 7. Assuming a normal distribution, what is the probability that a student scored 90 or higher? ((( 0.0021 )))
The average score of 100 students taking a statistics final was 70 with a standard deviation of 7. Assuming a normal distribution, what is the probability that a student scored greater than 65? ((( 0.7611 )))
The average score of 100 students taking a statistics final was 70, with a standard deviation of 7. Assuming a normal distribution, what test score separates the top 5% of the students from the lower 95% of the students? ((( 81.55 )))
The average score of 100 students taking a statistics final was 70 with a standard deviation of 7. Assuming a normal distribution, what test score separates the top 25% of the students from the lower 75% of students? ((( 74.69 )))
A binomial distribution has 100 trials (n = 100) with a probability of success of 0.25 (π = 0.25). To apply the normal distribution to approximate the binomial, what are the mean and standard deviation? ((( µ = 25 and σ = 4.33 )))
A binomial distribution has 50 trials (n = 50) with a probability of success of 0.50 (π = 0.50). To use the normal distribution to approximate the binomial, what are the mean and standard deviation? ((( µ = 25 and σ = 3.5355 )))
A binomial distribution has 100 trials (n = 100) with a probability of success of 0.25 (Ï€ = 0.25). We would like to find the probability of 75 or more successes using the normal distribution to approximate the binomial. Applying the continuity correction factor, what value would be used to calculate a z-score? ((( 74.5 )))
A binomial distribution has 100 trials (n = 100) with a probability of success of 0.25 (Ï€ = 0.25). We would like to find the probability of 34 or more successes using the normal distribution to approximate the binomial. Applying the continuity correction factor, what z-score should be used? ((( 1.963 )))
A binomial distribution has 50 trials (n = 50) with a probability of success of 0.50 (Ï€ = 0.50). We would like to find the probability of 34 or more successes using the normal distribution to approximate the binomial. Applying the continuity correction factor, what z-score should be used? ((( 2.404 )))
If an average of 12 customers are served per hour, then one customer arrives every ________. ((( 5 minutes )))
If an average of 60 customers are served per hour, then one customer arrives every ________. ((( 1 minute )))
If an average of 12 12 customers are served per hour, what is the probability that the next customer will arrive in 6 minutes or less? ((( 0.70 )))
If an average of 12 customers are served per hour, what is the probability that the next customer will arrive in 3 minutes or less? ((( 0.45 )))
The center of a normal probability distribution is ________. ((( the mean of the distribution )))
The z-value for a standard normal distribution ________. ((( can be either positive or negative )))
Consider a continuous random variable x, which is uniformly distributed between 65 and 85. The probability of x taking on a value between 75 to 90 is ________. ((( 0.50 )))
Consider a standard normal random variable z. What is the value of z if the area to the right of z is 0.2643? ((( 0.63 )))
When data are collected using a qualitative, nominal variable (e.g., male or female), what is true about a frequency table that summarizes the data?(((The number of classes corresponds to the number of a variable's values.))
A student was studying the political party preferences of a university's student population. The survey instrument asked students to identify themselves as a Democrat or a Republican. This question is flawed because(((the categories are not exhaustive.))
A student was studying the political party preferences of a university's student population. The survey instrument asked students to identify their political preferences—for example, Democrat, Republican, Libertarian, or another party. The best way to illustrate the frequencies for each political preference is a(((bar chart.))
A student was studying the political party preferences of a university's student population. The survey instrument asked students to identify their political preferences—for example, Democrat, Republican, Libertarian, or another party. The best way to illustrate the relative frequency distribution is a(((pie chart.))
The number of employees less than the upper limit of each class at Lloyd's Fast Food Emporium is shown in the following table: What is it called?(((A cumulative frequency distribution))
Here is a sample distribution of hourly earnings in Paul's Cookie Factory: The limits of the class with the smallest frequency are((($12.00 and up to $15.00.))
What level of measurement is a person's "favorite sport"?(((Nominal))
For any data set, Chebyshev's theorem estimates the proportion of observations that occurs within k standard deviations of the mean, where k is greater than 1.0.(((TRUE))
For the most recent seven years, the U.S. Department of Education reported the following number of bachelor's degrees awarded in computer science: 4,033; 5,652; 6,407; 7,201; 8,719; 11,154; 15,121. What is the annual arithmetic mean number of degrees awarded?(((About 8,327))
A question in a market survey asks for a respondent's favorite car color. Which measure of central location should be used to summarize this question?(((Mode))
A purchasing agent for a trucking company is shopping for replacement tires for their trucks from two suppliers. The suppliers' prices are the same. However, Supplier A's tires have an average life of 60,000 miles with a standard deviation of 10,000 miles. Supplier B's tires have an average life of 60,000 miles with a standard deviation of 2,000 miles.Which of the following statements is true?(((The life of Supplier B's tire is more predictable than the life of Supplier A's tires.))
According to Chebyshev's theorem, at least what percent of the observations lie within plus and minus 1.75 standard deviations of the mean?(((67%))
For the past week, a company's common stock closed with the following prices: $61.5, $62, $61.25, $60.875, and $61.5. What was the price range?((($1.125))
A sample of assistant professors on the business faculty at state-supported institutions in Ohio revealed the mean income to be $72,000 for nine months, with a standard deviation of $3,000. Using Chebyshev's theorem, what proportion of the faculty earns more than $66,000, but less than $78,000?(((At least 75%))
A population consists of all the weights of all defensive tackles on a university's football team. They are Johnson, 204 pounds; Patrick, 215 pounds; Junior, 207 pounds; Kendron, 212 pounds; Nicko, 214 pounds; and Cochran, 208 pounds. What is the population standard deviation (in pounds)?(((About 4))
Each person who applies for an assembly job at Robert's Electronics is given a mechanical aptitude test. One part of the test involves assembling a plug-in unit based on numbered instructions. A sample of the length of time it took 42 persons to assemble the unit was organized into the following frequency distribution.What is the standard deviation (in minutes)?(((3.89))
Pearson's coefficient of skewness is a measure of a distribution's symmetry.(((TRUE))
A large oil company is studying the number of gallons of gasoline purchased per customer at self-service pumps. The mean number of gallons is 10.0, with a standard deviation of 3.0 gallons. The median is 10.75 gallons. What is Pearson's coefficient of skewness in this instance?(((-0.75))
What is the possible range of values for Pearson's coefficient of skewness?(((-3 and +3))
An individual can assign a subjective probability to an event based on the individual's knowledge about the event. ((( TRUE )))
Bayes' theorem is a method to revise the probability of an event given additional information. ((( TRUE )))
Bayes' theorem is used to calculate a subjective probability. ((( FALSE )))
When an event's probability depends on the occurrence of another event, the probability is a(n) ________. ((( conditional probability )))
The process used to calculate the revised probability of an event given additional information can be obtained through ________. ((( Bayes' theorem )))
A study of interior designers' opinions with respect to the most desirable primary color for executive offices showed the following: Primary Color Number of Opinions Red 92 Orange 86 Yellow 46 Green 91 Blue 37 Indigo 46 Violet 2 What is the probability that a designer does NOT prefer red? ((( 0.77 )))
A study of interior designers' opinions with respect to the most desirable primary color for executive offices showed the following: Primary Color Number of Opinions Red 92 Orange 86 Yellow 46 Green 91 Blue 37 Indigo 46 Violet 2 What is the probability that a designer does NOT prefer yellow? ((( 0.885 )))
A study of interior designers' opinions with respect to the most desirable primary color for executive offices showed the following: Primary Color Number of Opinions Red 92 Orange 86 Yellow 46 Green 91 Blue 37 Indigo 46 Violet 2 What is the probability that a designer does NOT prefer blue? ((( 0.9075 )))
Using the terminology of Bayes' theorem, a posterior probability can also be defined as a ((( revised probability )))
David's gasoline station offers 4 cents off per gallon if the customer pays in cash and does not use a credit card. Past evidence indicates that 40% of all customers pay in cash. During a one-hour period, 15 customers buy gasoline at this station. What is the probability that at least 10 pay in cash? ((( 0.033 )))
David's gasoline station offers 4 cents off per gallon if the customer pays in cash and does not use a credit card. Past evidence indicates that 40% of all customers pay in cash. During a one-hour period, 15 customers buy gasoline at this station. What is the probability that all 15 pay in cash? ((( 0.0 )))
David's gasoline station offers 4 cents off per gallon if the customer pays in cash and does not use a credit card. Past evidence indicates that 40% of all customers pay in cash. During a one-hour period, 15 customers buy gasoline at this station. What is the probability that more than 8 and less than 12 customers pay in cash? ((( 0.092 )))
David's gasoline station offers 4 cents off per gallon if the customer pays in cash and does not use a credit card. Past evidence indicates that 40% of all customers pay in cash. During a one-hour period, 15 customers buy gasoline at this station. This situation is an example of what type of discrete probability distribution? ((( Binomial probability distribution )))
The time to fly between New York City and Chicago is uniformly distributed with a minimum of 120 minutes and a maximum of 150 minutes. What is the distribution's mean? ((( 135 minutes )))
The time to fly between New York City and Chicago is uniformly distributed with a minimum of 120 minutes and a maximum of 150 minutes. What is the distribution's standard deviation? ((( 8.66 minutes )))
Which of the following is a characteristic of the normal probability distribution? ((( It's bell-shaped. )))
A study of a company's practice regarding the payment of invoices revealed that an invoice was paid an average of 20 days after it was received. The standard deviation equaled five days. Assuming that the distribution is normal, what percent of the invoices were paid within 15 days of receipt? ((( 15.87% )))
Suppose a tire manufacturer wants to set a mileage guarantee on its new XB 70 tire. Tests revealed that the tire's mileage is normally distributed with a mean of 47,900 miles and a standard deviation of 2,050 miles. The manufacturer wants to set the guaranteed mileage so that no more than 5% of the tires will have to be replaced. What guaranteed mileage should the manufacturer announce? ((( 44,518 )))
The average starting salary of individuals with a master's degree in statistics is normally distributed with a mean of $48,000 and a standard deviation of $6,000. What is the probability that a randomly selected individual with a master's in statistics will get a starting salary of at least $55,000? ((( 0.121 )))
When data are collected using a qualitative, nominal variable (e.g., male or female), what is true about a frequency table that summarizes the data?(((The number of classes corresponds to the number of a variable’s values.))
A student was studying the political party preferences of a university’s student population. The survey instrument asked students to identify themselves as a Democrat or a Republican. This question is flawed because(((the categories are not exhaustive.))
A student was studying the political party preferences of a university’s student population. The survey instrument asked students to identify their political preferences—for example, Democrat, Republican, Libertarian, or another party. The best way to illustrate the frequencies for each political preference is a(((bar chart.))
A student was studying the political party preferences of a university’s student population. The survey instrument asked students to identify their political preferences—for example, Democrat, Republican, Libertarian, or another party. The best way to illustrate the relative frequency distribution is a(((pie chart.))
The number of employees less than the upper limit of each class at Lloyd’s Fast Food Emporium is shown in the following table: What is it called?(((A cumulative frequency distribution))
Here is a sample distribution of hourly earnings in Paul’s Cookie Factory: The limits of the class with the smallest frequency are((($12.00 and up to $15.00.))
What level of measurement is a person’s "favorite sport"?(((Nominal))
For any data set, Chebyshev’s theorem estimates the proportion of observations that occurs within k standard deviations of the mean, where k is greater than 1.0.(((TRUE))
For the most recent seven years, the U.S. Department of Education reported the following number of bachelor’s degrees awarded in computer science: 4,033; 5,652; 6,407; 7,201; 8,719; 11,154; 15,121. What is the annual arithmetic mean number of degrees awarded?(((About 8,327))
A question in a market survey asks for a respondent’s favorite car color. Which measure of central location should be used to summarize this question?(((Mode))
A purchasing agent for a trucking company is shopping for replacement tires for their trucks from two suppliers. The suppliers’ prices are the same. However, Supplier A’s tires have an average life of 60,000 miles with a standard deviation of 10,000 miles. Supplier B’s tires have an average life of 60,000 miles with a standard deviation of 2,000 miles.Which of the following statements is true?(((The life of Supplier B’s tire is more predictable than the life of Supplier A’s tires.))
According to Chebyshev’s theorem, at least what percent of the observations lie within plus and minus 1.75 standard deviations of the mean?(((67%))
For the past week, a company’s common stock closed with the following prices: $61.5, $62, $61.25, $60.875, and $61.5. What was the price range?((($1.125))
A sample of assistant professors on the business faculty at state-supported institutions in Ohio revealed the mean income to be $72,000 for nine months, with a standard deviation of $3,000. Using Chebyshev’s theorem, what proportion of the faculty earns more than $66,000, but less than $78,000?(((At least 75%))
A population consists of all the weights of all defensive tackles on a university’s football team. They are Johnson, 204 pounds; Patrick, 215 pounds; Junior, 207 pounds; Kendron, 212 pounds; Nicko, 214 pounds; and Cochran, 208 pounds. What is the population standard deviation (in pounds)?(((About 4))
Each person who applies for an assembly job at Robert’s Electronics is given a mechanical aptitude test. One part of the test involves assembling a plug-in unit based on numbered instructions. A sample of the length of time it took 42 persons to assemble the unit was organized into the following frequency distribution.What is the standard deviation (in minutes)?(((3.89))
Pearson’s coefficient of skewness is a measure of a distribution’s symmetry.(((TRUE))
A large oil company is studying the number of gallons of gasoline purchased per customer at self-service pumps. The mean number of gallons is 10.0, with a standard deviation of 3.0 gallons. The median is 10.75 gallons. What is Pearson’s coefficient of skewness in this instance?(((-0.75))
What is the possible range of values for Pearson’s coefficient of skewness?(((-3 and +3))
An individual can assign a subjective probability to an event based on the individual’s knowledge about the event. ((( TRUE )))
Bayes’ theorem is a method to revise the probability of an event given additional information. ((( TRUE )))
Bayes’ theorem is used to calculate a subjective probability. ((( FALSE )))
When an event’s probability depends on the occurrence of another event, the probability is a(n) ________. ((( conditional probability )))
The process used to calculate the revised probability of an event given additional information can be obtained through ________. ((( Bayes’ theorem )))
A study of interior designers’ opinions with respect to the most desirable primary color for executive offices showed the following: Primary Color Number of Opinions Red 92 Orange 86 Yellow 46 Green 91 Blue 37 Indigo 46 Violet 2 What is the probability that a designer does NOT prefer red? ((( 0.77 )))
A study of interior designers’ opinions with respect to the most desirable primary color for executive offices showed the following: Primary Color Number of Opinions Red 92 Orange 86 Yellow 46 Green 91 Blue 37 Indigo 46 Violet 2 What is the probability that a designer does NOT prefer yellow? ((( 0.885 )))
A study of interior designers’ opinions with respect to the most desirable primary color for executive offices showed the following: Primary Color Number of Opinions Red 92 Orange 86 Yellow 46 Green 91 Blue 37 Indigo 46 Violet 2 What is the probability that a designer does NOT prefer blue? ((( 0.9075 )))
Using the terminology of Bayes’ theorem, a posterior probability can also be defined as a ((( revised probability )))
David’s gasoline station offers 4 cents off per gallon if the customer pays in cash and does not use a credit card. Past evidence indicates that 40% of all customers pay in cash. During a one-hour period, 15 customers buy gasoline at this station. What is the probability that at least 10 pay in cash? ((( 0.033 )))
David’s gasoline station offers 4 cents off per gallon if the customer pays in cash and does not use a credit card. Past evidence indicates that 40% of all customers pay in cash. During a one-hour period, 15 customers buy gasoline at this station. What is the probability that all 15 pay in cash? ((( 0.0 )))
David’s gasoline station offers 4 cents off per gallon if the customer pays in cash and does not use a credit card. Past evidence indicates that 40% of all customers pay in cash. During a one-hour period, 15 customers buy gasoline at this station. What is the probability that more than 8 and less than 12 customers pay in cash? ((( 0.092 )))
David’s gasoline station offers 4 cents off per gallon if the customer pays in cash and does not use a credit card. Past evidence indicates that 40% of all customers pay in cash. During a one-hour period, 15 customers buy gasoline at this station. This situation is an example of what type of discrete probability distribution? ((( Binomial probability distribution )))
The time to fly between New York City and Chicago is uniformly distributed with a minimum of 120 minutes and a maximum of 150 minutes. What is the distribution’s mean? ((( 135 minutes )))
The time to fly between New York City and Chicago is uniformly distributed with a minimum of 120 minutes and a maximum of 150 minutes. What is the distribution’s standard deviation? ((( 8.66 minutes )))
Which of the following is a characteristic of the normal probability distribution? ((( It’s bell-shaped. )))
A study of a company’s practice regarding the payment of invoices revealed that an invoice was paid an average of 20 days after it was received. The standard deviation equaled five days. Assuming that the distribution is normal, what percent of the invoices were paid within 15 days of receipt? ((( 15.87% )))
Suppose a tire manufacturer wants to set a mileage guarantee on its new XB 70 tire. Tests revealed that the tire’s mileage is normally distributed with a mean of 47,900 miles and a standard deviation of 2,050 miles. The manufacturer wants to set the guaranteed mileage so that no more than 5% of the tires will have to be replaced. What guaranteed mileage should the manufacturer announce? ((( 44,518 )))
The average starting salary of individuals with a master’s degree in statistics is normally distributed with a mean of $48,000 and a standard deviation of $6,000. What is the probability that a randomly selected individual with a master’s in statistics will get a starting salary of at least $55,000? ((( 0.121 )))
What is true of wholesalers? (((Wholesalers are extremely important because of the marketing activities they perform.)))
Many service providers are considered retailers because they (((provide their services directly to consumers)))
A merger occurs when (((two companies combine to form a new company)))
A(n) ________ is a partnership established for a specific project or for a limited time. (((joint venture)))
An entrepreneur has been primarily associated with the willingness to((take risks.))
What is one of the difficulties faced by small business owners?((worries about employee problems))
What is a difference between high technology businesses and other small businesses?((High technology businesses require greater capital and have higher initial startup costs thanother small businesses))
What is true of a capitalist economic system?((Prices of goods and services are determined by supply and demand))
The quantity of goods and services that consumers are willing to buy at different prices at a specific time is referred to as((demand.))
The quantity of products that businesses are willing to sell at different prices at a specific time is referred to as((supply.))
What is a defining characteristic of public corporations?((Their stock can be bought, sold, or traded by anyone))
Which business provides a service, but is neither owned by the government nor focuses on earning profits?((a nonprofit corporation))
What is a true statement about the board members of a corporation?((They have a duty of care and loyalty to oversee the management of the firm.))
Preferred stockholders of a corporation((have a claim to profits before other stockholders do.))
The reported unemployment is 5.5% of the population. What level of measurement is used to measure unemployment? ((Ratio))
What type of chart would best represent the data from this frequency table? ((Bar Chart))
The interquartile range is graphically presented in a ___________.(((box plot))
In the following set of data: (4, 5, 2, 1, 7, 61, 100), what are the first, second, and third quartiles?(((2, 5, and 61))
A student was interested in the cigarette-smoking habits of college students and collected data from an unbiased random sample of students. The data are summarized in the following table: What type of chart best represents the date from this frequency table?(((Bar chart))
If the variance of the "number of daily parking tickets issued is 100" the standard deviation is defined as the ____________. ((square root of the variance of the "number of daily parking tickers"))
The weight of cans of fruit is normally distributed with a mean of 1,000 grams and a standard deviation of 50 grams. What percent of the cans weigh 860 grams or less? ((( 0.26% )))
For the following distribution: X P(x) 0 0.130 1 0.346 2 0.346 3 0.154 4 0.024 What is the mean of the distribution? ((( 1.596 )))
Tables of normal distribution probabilities are found in many statistics books. These probabilities are calculated from a standardized normal distribution with ___________. ((a mean of 0 and a standard deviation of 1))
Compute the mean of the following population values: 6, 3, 5, 7, 6. (Round your answer to 1 decimal place.) ((5.4))
A sample of the paramedical fees charged by clinics revealed these amounts: $11, $44, $32, $42, $45, and $35. What is the median charge? (( $38.50))
The test scores for a class of 203 students are computed. What is the location of the test score associated with the third quartile? ((153))
A survey of top executives revealed that 33% of them regularly read Time magazine, 21% read Newsweek, and 41% read U.S. News & World Report. A total of 12% read both Time and U.S. News & World Report. What is the probability that a particular top executive reads either Time or U.S. News & World Report regularly? ((0.62))
A survey of top executives revealed that 27% of them regularly read Time magazine, 23% read Newsweek, and 38% read U.S. News & World Report. A total of 10% read both Time and U.S. News & World Report. What is the probability that a particular top executive reads either Time or U.S. News & World Report regularly? ((0.55))
Compute the mean of the following population values: 5, 7, 6, 5, 5 (Round your answer to 1 decimal place.) (( 5.6 ))
If the variance of the “number of daily parking tickets” issued is 100, the standard deviation is defined as the _______ ((square root of the variance of the "number of daily parking tickets))
A group of 100 students was surveyed about their interest in a new International Studies program, Interest was measured in terms of high, medium, or low. In the study, 30 students responded high interest, 40 students responded medium interest, and 30 students responded low interest. What is the relative frequency of students with high interest? ((0.30))
Compute the mean of the following population values: 5, 7, 6, 5, 5 (Round your answer to 1 decimal place.) ((5.6))
On a finance exam, 15 accounting majors had an average grade of 90. On the same exam, 7 marketing majors averaged 85, and 10 finance majors averaged 93, What is the weighted mean for all 32 students taking the exam? ((89.84))
The incomes of 50 loan applicants are obtained, Which level of measurement is income? ((Ratio ))
The members of each basketball team wear numbers on their jerseys, What scale of measurement are these numbers considered? ((Nominal))
The net incomes (in $millions) of a sample of steel fabricators are $86, $67, $86, and $85 What is the modal net income? ((86))
When you take your clothes to the dry cleaner to be cleaned or go to the spa to get a massage, you are not receiving a product because you have not received anything tangible in return. ((FALSE))
Economics refers to all activities concerned with obtaining money and using it effectively. ((FALSE))
The focus of all marketing activities is the successful transformation of resources into goods and services. ((FALSE))
All economic systems must address three important issues, one of which is how the goods and services will be produced. ((TRUE))
________ is the primary goal of business. ((Profit))
Which of the following is an example of an intangible product? ((a music concert))
Amit hired an attorney to help him draft a purchase and sale agreement for his new home. The home is an example of a product that is a(n) ((tangible item.))
Which of the following is an example of a tangible product? ((an automobile))
Which statement is true about businesses and profits? ((Earning profits contributes to society by creating resources that support social institutions and government.))
Which of the following is NOT an accurate statement regarding nonprofit organizations? ((They have the fundamental goal of earning profits.))
Master Motors is working to find a way to produce the SUVs and trucks its customers want while meeting environmental requirements. This is an effort to please multiple ((stakeholders))
Among the major activities of business, which of the following is an element of management? ((acquire resources))
Talia is the owner of a small business. She has an HR person, an operations manager, a marketing manager, and a sales manager on staff. Who is responsible for the financial decisions that arise in the course of business? ((Talia, the owner))
What is true regarding the responsibilities of the people involved in a business? ((The primary responsibility of providing financial resources for the operation of the business lies with the business owners.))
Tanveer has responsibility for the functions of organizing, staffing, planning, and controlling activities in a large company. These functions are most closely associated with ((managers))
Allie, the manager of a new pet grooming company, makes sure her grooming salon always has plenty of treats and toys for the animals as well as beverages and snacks for their owners to ensure a quality customer service experience. This is an example of the ________ element of management. ((operations))
________ gather information and conduct research to determine what customers want. They also plan and develop products and make decisions about how much to charge for their products and when and where to make them available. ((Marketers))
The focus of all marketing activity is to ((satisfy the needs and wants of consumers.))
Advertising, personal selling, coupons, and sweepstakes are elements included in the ________ aspect of marketing activities. ((promotion))
Fabulous Fitness has a new exercise video that it hopes will increase its sales. It is offering coupons and a sweepstakes to get people to buy the video. These are examples of ((sales promotion.))
Decisions regarding product adoption, development, branding, and positioning fall under which element of the marketing mix? ((product))
When a business fails or does not make a profit, ________ have the most to lose in terms of finances. ((owners))
Leon is an accountant for a small company. As such, he participates primarily in ________ activities. ((financial))
The study of how resources are distributed for the production of goods and services within a social system is referred to as ((economics.))
Quentin owns a company that makes metal and wooden signs. In order to acquire the different kinds of wood and steel he needs to make his product, Quentin needs ((capital))
The factors of production are the ((land, labor, capital, and enterprise used to produce goods and services))
Which of the following is a natural resource? ((minerals))
Which of the following is an intangible resource? ((company reputation))
________ describe(s) how a particular society distributes its resources to produce goods and services. ((An economic system))
Who subscribes to the ideals of a communist economy as proposed by Karl Marx? ((Felix believes that in a society, people should contribute according to their ability and receive benefits according to what they need.))
Which of the following is considered a command economy? ((communism))
Which of the following is a characteristic of communist economies in reality? ((critical shortages of consumer goods))
Which communist country was the first to make strong economic gains by adopting capitalist approaches to business? ((China))
How are profits handled in communist systems? ((Excess income goes to the government.))
What is true of product availability in a communist system? ((Consumers have a limited choice of both goods and services.))
In which economic system do most people work for government-owned industries or farms? ((communist))
What is true in a socialist economy? ((Competition is restricted in basic industries.))
________ is an economic system in which the government owns and operates basic industries such as utilities, health care, and banking. ((Socialism))
Which statement is true regarding a socialist system of economy? ((Central planning determines what basic goods and services are produced.))
Chi Ling lives in a country where income and social services are distributed equally among its citizens. However, the citizens must pay high taxes and the country often experiences high levels of unemployment. Chi Ling lives in a(n) ________ nation. ((socialist))
Egalitarianism refers to the ((equal distribution of income and social services))
What is true of a free-market system? ((All economic decisions are made without government intervention.))
What is true of a capitalist economic system? ((Prices of goods and services are determined by supply and demand.))
________ is often called the father of capitalism. ((Adam Smith))
How does modified capitalism differ from pure capitalism? ((In modified capitalism, the government intervenes and regulates business to some extent.))
A county encourages individuals to own and operate the majority of businesses, but its government passes laws to enforce fair business practices such as labor laws, antitrust legislation, and laws against deceptive advertising. This is an example of ((modified capitalism.))
Which country is an example of an economic system based on capitalism? ((Japan))
All of the following are examples of mixed economies EXCEPT ((in socialist France, the French government owns the postal service industry.))
The quantity of goods and services that consumers are willing to buy at different prices at a specific time is referred to as ((demand))
The ________ is the price at which the number of products that businesses are willing to supply equals the number of products consumers are willing to purchase at a specific point in time. ((equilibrium price))
The quantity of products that businesses are willing to sell at different prices at a specific time is referred to as ((supply))
Generally, when a retailer announces a discount sale on all its products, what happens? ((Consumers will demand more products.))
Kristy is a member of a warehouse club. She likes shopping there because she can buy items like toilet paper and groceries in bulk to save money. This relates to which element of supply and demand? ((demand))
Which statement describes the force that drives the distribution of resources (goods and services, labor, and money) in a free-enterprise economy? ((Prices for goods and services vary according to the changes in supply and demand.))
Which market is most likely to be a monopoly? ((electricity supply))
According to Adam Smith, competition ((fosters efficiency.))
Using the supply and demand curves shown in the graph that follows, what is the equilibrium price for soft drinks? ((55 cents))
Jeff is an entrepreneur who decided to start an online bakery. Since all his business will be done online and he can bake in his home kitchen, he is saving building and other brick-and-mortar costs that allow him to pass these savings on to his customers. Reducing prices to customers is an element of ((competition))
When different businesses promote similar products that differ in some characteristics but satisfy the same consumer need, the competitive environment is most likely to be ((monopolistic competition.))
________ is an example of a product of monopolistic competition. ((Aspirin))
Trevor owns a local farm stand. Although he sells fruit and vegetables, like the grocery store chains in the area, he has some power over the price he charges because he can offer consumers features they want. For instance, his fruit and vegetables are organic and locally grown. This is an example of ((monopolistic competition.))
Which of the following exists when there are many small businesses selling one standardized product, such as agricultural commodities like wheat, corn, and cotton? ((pure competition))
The market for corn in Mexico has a large number of sellers and buyers, and there is no difference in the products sold by each seller. As a result, prices are determined by the ((forces of supply and demand.))
What competitive environment exists when there is only one business providing a product in a given market? ((a monopoly))
What is true regarding an oligopoly? ((Prices charged by different firms stay fairly close because a price cut or increase by one company will trigger a similar response from another company.))
Following a bad harvest, the prices of agricultural commodities in Brazil have increased dramatically, as have the prices of products manufactured from agricultural commodities. Government intervention has failed to arrest the rising prices. Based on this information, Brazil is experiencing ((inflation.))
Economic expansion occurs when ((an economy is growing and people are spending more money.))
Ramona is living on a fixed income since her retirement. She is worried that the economy is expanding too rapidly, which could result in ((inflation.))
What is true of economic expansion? ((The standard of living rises because more people are employed and have money to spend.))
What is a characteristic of economic depression? ((Business output is sharply reduced.))
What is associated with a decline in spending? ((economic contraction))
Contractions of the economy lead to ((recession.))
Increased unemployment can reduce consumer demand for goods and services, leading to ((deflation.))
A country experiences a rapid expansion of its economy, which resulted in prices doubling every 12 hours. This is an example of ((hyperinflation.))
A budget deficit occurs when a nation ((spends more than it takes in from taxes.))
If a country balances the money it spends for social, defense, and other programs with the amount of money it takes in from taxes, this country ((will not have a budget deficit.))
________ measures changes in the prices of goods and services purchased for consumption by typical urban households. ((Consumer price index))
________ indicates the income level of "average" Americans. It is useful in determining how much "average" consumers spend and how much money Americans are earning. ((Per capita income))
All of these statements are true of the U.S. national debt EXCEPT ((the recently enacted tax cuts in the United States are projected to decrease the national debt significantly over the next 10 years.))
In Norway, most citizens are able to afford basic necessities and some degree of comfort. This means Norway has a(n) ((high standard of living.))
What is true of an open economy, like that of the United States? ((Open economies are major players in international trade.))
What would hinder business activity as it relates to an open economy? ((too much regulation of business activities))
What are the two primary factors economists consider when evaluating the American economy? ((growth in GDP and jobs))
Before the Industrial Revolution, the economy of North America under the colonists was characterized by ((farming, and the production of agricultural goods.))
What change occurred in the United States during the Industrial Revolution? ((Farmers began to move to cities to find jobs in factories.))
Who set up the first American textile factory after he memorized the plans for an English factory and emigrated to the United States? ((Samuel Slater))
In the history of the American economy, the period following the Industrial Revolution was known for an increasing emphasis on ((manufacturing.))
When the assembly line was applied to more industries, America evolved into a(n) ((marketing economy.))
What is a characteristic of the marketing economy phase of the American economy? ((Expensive goods such as cars and appliances could be purchased on a time-payment plan.))
All of the following contributed to the evolution of the service economy in America EXCEPT ((the desire of women to be stay-at-home-moms.))
As the service economy in America continues to rapidly grow, advances in technology are contributing to new service products based on technology and digital media that provide smart phones, social networking, and e-commerce. This suggests that the country may be moving toward being a ________ economy. ((digital))
An entrepreneur has been primarily associated with the willingness to ((take risks.))
All of the following are true of entrepreneurs EXCEPT ((many of the companies started by early entrepreneurs have failed to stay in operation.))
Which of the following emerged in the United States as a result of industrialization? ((increase in the variety of goods available))
What does the American government do to counteract economic contraction? ((It reduces interest rates.))
What is useful in fighting inflation? ((raising interest rates))
What is true of ethics and social responsibility in business? ((Ethical conduct and corporate social responsibility are linked to increased profits.))
Several employees in a well-known corporation, Truck Corporation, were arrested for intentionally selling defective truck parts to their customers. Scandals like this ((undermine public confidence in corporations.))
All of the following are true of workplace diversity EXCEPT it ((decreases the financial performance of companies.))
The acceptability of behavior of a business rests solely with the business itself. ((FALSE))
An ethical issue is an identifiable problem, situation, or opportunity that requires a person or organization to choose from among several actions that may be evaluated as ethical or unethical. ((TRUE))
The concept of social responsibility is universally accepted. ((FALSE))
A business must choose to be either profitable or socially responsible; they cannot be both. ((FALSE))
The principles and standards that determine acceptable conduct in business organizations are referred to as ((business ethics.))
Which statement is true regarding business ethics and social responsibility? ((Companies can be both profitable and socially responsible.))
Premier Products is well-received by the community in which it operates. Every year Premier hosts a charity fundraiser to support local charities; its employees regularly volunteer within the community and the company is committed to using sustainable manufacturing processes. The actions taken by Premier Products best exemplify ((social responsibility.))
Which law reformed the financial industry and offered consumers protection against complex and/or deceptive financial products? ((the Dodd-Frank Act))
Recently, ________ became the number-one consumer complaint with the Federal Trade Commission. ((identity theft))
One of the most difficult things for a business to restore after an ethics scandal is/are ((trust.))
Which statement is true of how the mass media reports on the ethical conduct of businesses? ((The mass media focuses more on misconduct than good ethical conduct.))
Which statement is true of business ethics? ((Ethical issues are not limited to for-profit organizations.))
What is a key step in evaluating ethical decisions in business? ((learning how to recognize and resolve ethical issues))
________ are payments, gifts, or special favors intended to influence the outcome of a decision. ((Bribes))
Joylene and Kurt are competing for a promotion. Hoping to sway her manager to promote her over Kurt, Joylene gives her manager an expensive bottle of wine. This is an example of ((bribery.))
All of the following are common areas of misconduct observed in the workplace EXCEPT ((separation of personal financial interests from business dealings.))
What is true of misusing company time? ((Time theft costs can be difficult to measure but are estimated to cost companies hundreds of billions of dollars annually.))
Abusive behavior is difficult to assess and manage because ((people have very diverse cultures and lifestyles.))
Felix speaks fluent Spanish, and English is his second language. When communicating with his colleagues, Amanda and Joe, he uses words that are normal in his language, but they consider them profane. What does this exhibit? ((Abusive behavior is difficult to assess and manage.))
What is associated with a hostile workplace when a person or group is targeted and is threatened, harassed, belittled, verbally abused, or overly criticized? ((bullying))
What is true of bullying? ((Although sexual harassment has legal recourse, bullying has little legal recourse at this time.))
Within the concept of abusive behavior, ________ should be a consideration. ((intent))
________ is the buying or selling of stocks by insiders who possess material that is still not public. ((Insider trading))
Physical threats, ignoring someone, yelling, and the use of profanity fall under which type of organizational misconduct? ((abusive behavior))
Spending an excessive amount of time on personal e-mails, submitting personal expenses on company expense reports, and using the company copier for personal use are all examples of ((the misuse of company resources.))
When Rhea used her company credit card to buy $50,000 worth of new furniture for her house, she was fired. Which type of misconduct does this exemplify? ((misuse of company resources))
Which organization has developed a Corruption Perceptions Index that ranks countries based on their level of corruption? ((Transparency International))
Which country is included in the list of least corrupt countries? ((Denmark))
Lindsay, a manager, makes decisions that benefit her financially at the expense of her firm. This is an example of which ethical issue in business? ((conflict of interest))
The fact that businesspersons are expected not to harm customers, clients, and competitors knowingly through deception, misrepresentation, coercion, or discrimination is part of the practice of ((fairness and honesty.))
The European Commission investigated Google to determine whether it promoted its own search results over those of other search engines in spite of their relevance. The aspect of fairness at issue here is ((competition.))
All of the following are associated with dishonesty EXCEPT ((competition.))
In the realm of business ethics, when automobile companies fail to issue recalls in a timely manner, this is an issue related to ((communications.))
People in the entertainment industry claim that requiring warning labels on movies and videogames violates their ________ right. ((First Amendment))
The National Business Ethics Survey found that employees who feel pressured to compromise ethical standards view ________ as the greatest source of such pressure. ((top and middle managers))
Who is engaged in plagiarism? ((Nathan came up with a new training method, but his boss Bill claimed the idea as his own.))
The warning on cigarette packages about the health implications of smoking is an example of ((communications.))
Which behavior is an example of ethical consideration within the purview of business relationships? ((keeping company secrets))
If a manager pressures a subordinate to engage in activities that he or she may otherwise view as unethical, it would be an ethical issue related to ((business relationships.))
Which of the following indicates that a person has begun the process of resolving an ethical issue? ((recognizing the ethical issue and openly discussing it with others))
Which of the following is NOT a question to consider in determining whether an action is ethical? ((Is this activity customary across all industries in your country?))
The three key factors that influence ethical decisions in an organization are: individual moral standards, the influence of managers and co-workers, and ((the opportunity to engage in misconduct.))
What is true of professional codes of ethics? ((They do not have to be too detailed.))
All of the following are true of the causes of ethical conflict EXCEPT ((codes of conduct increase ethical conflict for employees.))
A set of formalized rules and standards that describes what a company expects of its employees is called a ((code of ethics.))
Codes of ethics, policies on ethics, and ethics training programs advance ethical behavior because they ((prescribe which activities are acceptable and which are not.))
According to the National Business Ethics Survey, employees in organizations that have written codes of conduct and ethics training, ethics offices or hotlines, and systems for reporting are more likely to ((report misconduct when they observe it.))
Codes of ethics foster ethical behavior by ((limiting the opportunity to behave unethically by providing punishments for violations of the rules and standards.))
Kellen just took a job as a financial advisor with a large firm. He wants to make sure that all of his business dealings are ethical. Which of the following will give him guidance on any gray or ambiguous area he may face at his new job? ((a code of ethics))
What is a reason why a code of ethics is important? ((It alerts employees about important issues and risks to address.))
What is true of the Dodd-Frank Act passed by the U.S. Congress in 2010? ((It encourages whistleblowers to provide information about corporate misconduct through monetary rewards.))
All of the following are true of current trends in ethics programs EXCEPT ((organizations are moving toward legally based ethical initiatives.))
What is one of the most common factors that executives give for why turnover increases? ((a lack of transparency among company leaders))
Being profitable relates to the ________ dimension of social responsibility. ((economic))
________ is the extent to which businesses meet the legal, ethical, economic, and voluntary responsibilities placed on them by their various stakeholders. ((Corporate citizenship))
Which of the following involves the activities and organizational processes adopted by businesses to meet their social responsibilities? ((corporate citizenship))
A company committed to corporate citizenship might be engaged in ((sustainable manufacturing processes.))
What is a dimension of social responsibility? ((ethical))
Being a "good corporate citizen" is an example of the ________ dimension of social responsibility. ((voluntary))
Philanthropic contributions made by a business to a charitable organization represent the ________ dimension of social responsibility. ((voluntary))
What is an argument that supports social responsibility? ((As members of society, businesses and their employees should support society through taxes and contributions to social causes.))
Studies have found a direct relationship between social responsibility and ________ in business. (( profitability))
Businesses must first be responsible to their ________, who are primarily concerned with earning a profit. ((owners))
What has research shown regarding the relationship between social responsibility and profitability? ((Social responsibility is linked to improved business performance.))
What is a major hindrance of any firm trying to increase profits? ((a lack of employee commitment))
Laws regarding workplace safety are enforced by the ((Occupational Safety and Health Administration))
A major social responsibility for businesses is providing ________ for all employees. ((equal opportunities))
________ is defined as the activities that individuals, groups, and organizations undertake to protect their rights as customers. ((Consumerism))
Employees expect businesses to ((keep them informed of what is happening in the company.))
Which item are companies offering to improve employee relations? ((extended parental leave))
The right to ________ means that a business must not knowingly sell anything that could result in personal injury or harm to consumers. ((safety))
The right to choose ensures that ((consumers have access to a variety of goods and services at competitive prices.))
The right to ________ assures consumers that their interests will receive full and sympathetic consideration when the government formulates policy. ((be heard))
The right to ________ gives consumers the freedom to review all details about the products they wish to buy. ((be informed))
________ emerged as a major issue in the 20th century in the face of increasing evidence that pollution, uncontrolled use of natural resources, and population growth were putting increasing pressure on the long-term sustainability of these resources. ((Environmental protection))
________ involves the interaction among nature and individuals, organizations, and business strategies and includes the assessment and improvement of business strategies, economic sectors, work practices, technologies, and lifestyles, so that they maintain the health of the natural environment. ((Sustainability))
Pollution of water and soil from activities such as oil and gas drilling is primarily related to ((sustainability issues.))
________ refers to attaching a positive environmental association on an unsuitable product, service, or practice. ((Greenwashing))
All of the following are ways businesses are working to resolve environmental concerns EXCEPT ((creating a positive association for unsuitable products, services, or practices.))
The fact that environmental responsibility requires trade-offs means that it ((imposes costs on both business and the public.))
The most common way that businesses exercise community responsibility is through ((donations to charitable organizations.))
Many businesses, recognizing that employees lack basic work skills, are becoming more concerned about the quality of ________ in the United States. ((education))
While unemployment is often seen as an economic issue, it carries ethical implications when ((high unemployment is coupled with a large gap between rich and poor.))
A survey of employers conducted in Indiana revealed that ________ percent reported leaving positions unfilled because the applicants were not qualified. ((39))
Which organization funds programs to train the hard-core unemployed so that they can find jobs and support themselves? ((National Alliance of Businessmen))
A sole proprietor has unlimited liability in meeting the debts of his or her business. ((TRUE))
Limited partners do not participate in the management of the business. ((TRUE))
The assets and liabilities of a corporation are separate from its owners. ((TRUE))
An S corporation is taxed just like any other corporation. ((FALSE))
A strategic alliance occurs when one company purchases another company by buying most of its stock. ((FALSE))
Most tender offers of corporate raiders are hostile. ((FALSE))
Which form of business ownership is the easiest to establish? ((sole proprietorships))
Which form of business organization is the most common in the United States? ((sole proprietorships))
Marco is an independent contractor who prepares architectural plans for building contractors; however, he is not an employee of any of the contractors for which he works. Marco is a(n) ((sole proprietor.))
Which statement characterizes a sole proprietorship? ((A sole proprietorship would most likely focus on services rather than on the manufacture of goods.))
What is an advantage of a sole proprietorship? ((It is easy and inexpensive to form.))
Carmen owns a small gift shop and is solely responsible for its debts and liabilities. She is also entitled to all the profits the shop makes after paying off taxes and other expenses. Her business is an example of a(n) ((sole proprietorship.))
Nathan is a sole proprietor. How is the income earned in his business taxed? ((Nathan pays one tax that includes both the business and his personal income.))
Ritesh wants to open his own bowling alley that will also serve alcoholic beverages and food. What will Ritesh have to do to start his business? ((He will need to fill out paperwork to obtain the proper licenses and permits.))
Darlene runs an antique store in her local community. She started the business with her own money and solely manages the day-to-day operations of the business. What is true in this scenario? ((Darlene can easily dissolve the business whenever she wants.))
Which form of business ownership has the most freedom from government regulation? ((sole proprietorship))
In sole proprietorships, owners ((are limited in the sources of funding available to them.))
What is a disadvantage of sole proprietorships? ((lack of continuity))
Which form of business ownership is directly linked to the life of its owner and his or her ability to work? ((a sole proprietorship))
________ can be considered as both an advantage and a disadvantage of a sole proprietorship. ((Taxation))
When it comes to borrowing funds, a sole proprietor ((oftentimes must pledge a car, a house, other real estate, or other personal assets to guarantee a loan.))
When Dr. Hamilton, a local physical therapist, dies, her children sell her physical therapy practice to a young, new therapist who just moved to the area. What advantage will this new therapist gain by buying a business rather than starting a new one? ((some of the customers will stay with the business))
Why is it difficult for sole proprietorships to hire qualified employees? ((It is difficult for sole proprietorships to match the wages and benefits offered by large corporations.))
A ________ is an association of two or more persons who carry on as co-owners of a business for profit. ((partnership))
All of the following are keys to a successful partnership EXCEPT partners must ((keep profit sharing equal regardless of contributions.))
What are the two basic types of partnership? ((general and limited))
Smith, Taylor, and Jones is a law firm specializing in medical malpractice. Each owner has unlimited liability for the debts of the business and all owners share in the management of the firm. This law firm is an example of a ((general partnership.))
A ________ has at least one general partner, who assumes unlimited liability, and at least one limited partner, whose liability is limited to his or her investment in the business. ((limited partnership))
In a limited partnership, ((at least one partner has unlimited liability.))
When are limited partnerships generally used? ((when a project is risky and the chance of loss is great))
GasSource, a national provider of natural gas, is a partnership that is traded on securities exchanges. This allows it to have the tax benefits of a limited partnership but the liquidity of a corporation. GasSource is an example of a(n) ((master limited partnership.))
The legal documents that identify the basic agreements between partners are called ((articles of partnership.))
Articles of partnership usually cover issues, including ((provisions for leaving the partnership.))
Keesha, Mandy, and Isabel are starting their own pet grooming business, and they are drafting articles of partnership. What item should they be sure to include in their articles of partnership? ((what to do if one of the partners becomes disabled))
It is easier to raise funds in partnerships than in sole proprietorships because ((several partners mean greater earning power and credit.))
The decision-making process in a partnership tends to be faster when ((there are two partners and both of them are involved in the day-to-day activities of the business.))
In the partnership form of ownership, ((the business will face fewer regulatory controls than a corporation.))
Avantika Chang and Haba Shah, both financial planners, start a financial consulting firm together. Both of them have unlimited liability for the debts of the business, and the management of the firm is equally shared between them. This law firm is an example of a(n) ((general pfewer regulatory controlsartnership.))
What is an advantage of a partnership over a corporation? ((fewer regulatory controls))
Lawyers, Inc. is a large law practice with partners that specialize in real estate closings, estate planning, and divorce proceedings. Since each partner specializes in a different area of law, clients would be likely to ((view the service as higher quality than that provided by one person.))
What is a disadvantage of a partnership? ((difficulty in selling the partnership interest))
How is taxation handled in partnerships? ((Partners pay taxes at the income tax rate for individuals.))
Partnerships are quasi-taxable organizations. This means that they ((do not pay taxes when submitting the partnership tax return to the Internal Revenue Service.))
The profits earned by a partnership are distributed ((in the proportions specified in the articles of partnership.))
What is an advantage of partnerships over sole proprietorships? ((In partnerships, the owners have access to more funds than in sole proprietorships.))
What is a disadvantage of partnerships over sole proprietorships? ((In partnerships, profits have to be shared, whereas in sole proprietorships all profits belong exclusively to the owner.))
Molly wants to start a new business in collaboration with her husband and sister-in-law. The features she is looking for in the business are as follows: least possible regulatory controls, faster decision making, and ease of organization. She also wants a business in which she will not be forced to pay off any business debts from her personal assets. Which type of business ownership is best suited to her needs? ((limited partnership))
Isaac starts a consulting firm with his friend Arnie. They decide to equally share the profits and have unlimited liability for the debts of their business. Such unlimited liability can be a distinct disadvantage for Isaac if he has ((more financial resources than Arnie.))
Profits of a corporation that are distributed in the form of cash payments to stockholders are called ((dividends))
A ________ is a legal entity, created by the state, whose assets and liabilities are separate from its owners. ((corporation))
________ are typically owned by many individuals and organizations who own shares of the business, called stock. (( Corporations))
Pipes R Us, a small plumbing company, decides to incorporate. It wants to have greater flexibility than traditional corporations. What type of corporation would work best for this company? ((S corporation))
When a business decides to incorporate, it will do so according to the laws of the ________ in which it incorporates. ((state))
If a business wants to incorporate, its incorporators must file legal documents with the appropriate state office. What are these legal documents called? ((articles of incorporation))
________ are legal documents that the state issues to companies based on information the company provides in the articles of incorporation. ((Corporate charters))
A corporation doing business outside the state in which it is chartered is known as a(n) ((foreign corporation))
A corporation doing business in the state in which it is chartered is known as a(n) ((domestic corporation.))
The Food Palace, Inc. is a large grocery chain that was incorporated in the United States. Over the years, the corporation has grown, and it has now opened locations in five other nations. In those nations, The Food Palace is considered to be a(n) ((alien corporation.))
Businesses set up as private corporations ((are owned and managed by just one or a few people.))
What is a defining characteristic of public corporations? ((Their stock can be bought, sold, or traded by anyone.))
Mountain Outfitters is a large company that manufactures durable outdoor clothing. The company has stock that anyone can buy, sell, or trade, but since it is a large company, its stockholders are far removed from its management. What type of business is this? ((a public corporation))
Organizations such as the National Aeronautics and Space Administration (NASA) and the U.S. Postal Service that are owned by the federal, state, or local governments are known as ((quasi-public corporations.))
A private corporation sells its stock to raise the money it needs for global expansion. As a result, anyone can now buy, sell, or trade the stocks of the company. This process of "going public" is referred to as ((an initial public offering.))
Which business provides a service, but is neither owned by the government nor focuses on earning profits? ((a nonprofit corporation))
Books to Grow, a corporation owned and managed by volunteers, supplies books and educational materials to impoverished children in Latin America. Its focus is not on earning a profit but on serving the community. Books to Grow is an example of a(n) ((nonprofit corporation.))
All of the following are true of 501(c)(3) organizations EXCEPT ((they must answer to their shareholders.))
What is a true statement about the board members of a corporation? ((They have a duty of care and loyalty to oversee the management of the firm.))
Geraldine is a professor at a local college who serves on the board of directors for a leading academic publisher. She is a valued member of the board because she brings her unique perspective from the field, and she is qualified, knowledgeable, and independent of the company. Geraldine is a(n) ((outside director.))
A difference between common stockholders and preferred stockholders within a corporation is that common stockholders ((receive dividends after preferred stockholders.))
Preferred stockholders of a corporation (( have a claim to profits before other stockholders do.))
What is a proxy? ((a written authorization assigning voting privileges to someone else))
The right of common stockholders to have the opportunity to purchase new shares of stock is called a ((preemptive right.))
Keith wants to buy stock in a large pharmaceutical company. Since he lives in the city where the company is headquartered, he wants to attend its annual meetings and vote for the board of directors. He also wants to maintain a 5 percent ownership of the company, so having priority in purchasing new shares on the marketplace is very important to him. What type of stock should Keith purchase? ((common stock))
The form of business ownership that has the ability to raise capital most easily is a ((public corporation.))
What is an advantage that corporations provide as a form of business ownership? ((It is easy to transfer ownership in corporations.))
A disadvantage of the corporate form of business ownership is that the ((formation of a corporation can be costly and it faces double taxation.))
An advantage of corporations over sole proprietorships is that ((unlike sole proprietorships, the existence of corporations is unaffected by the death or withdrawal of any of the stockholders.))
An advantage of a corporation over a partnership is that ((as a legal entity, a corporation can enter into contracts without as much difficulty as a partnership.))
Sasha works for a large corporation, and sometimes, she finds it difficult to see how she fits into the corporate picture. However, recently, her company decides to establish an employee stock ownership plan. Which is a benefit of this type of plan? ((It can boost employee productivity.))
A(n) ________ is a partnership established for a specific project or for a limited time. ((joint venture))
A characteristic of joint ventures is that they ((are popular in situations that call for large investments.))
A(n) ________ is a form of business ownership that is taxed as though it were a partnership; it is very popular among entrepreneurs and represents almost half of all corporate filings. ((S corporation))
A characteristic of S corporations is that ((their shareholders can shift income and appreciation to others.))
In an S corporation, net profits and losses of the corporation pass to the owners. What effect does this have? ((It eliminates double taxation.))
Which form of business ownership provides limited liability but is taxed like a partnership? ((a limited liability company))
The "limited" part of a limited liability company refers to the ((limited liability of the business.))
A difference between limited liability companies (LLCs) and S corporations is that ((LLCs are very simple to run, whereas S corporations are not.))
A similarity between limited liability companies and S corporations is that both ((offer limited liability to owners.))
An organization composed of individuals or small businesses that have banded together to reap the benefits of belonging to a larger organization is known as a(n) ((cooperative.))
The Dairy Farmers of America is an association of dairy farmers. Its agenda is not to make profits but to help its members market their products and to offer farmers a fair voice in national policymaking. Thus, The Dairy Farmers of America is an example of a(n) ((cooperative.))
A merger occurs when ((two companies combine to form a new company.))
Sweet Scents, a candle company, purchases Nature Candle Products, its direct competitor, by buying most of its stock. In this scenario, Sweet Scents is involved in a(n) ((acquisition.))
Subs and More merges with its supplier, Top Shelf Ingredients, in an effort to have a constant supply of sauce for its sub sandwiches. This is an example of a ((vertical merger.))
Feed & Grow and Boost Products, two fertilizer companies, merge to reduce the number of corporations competing within the fertilizer industry. This merger is an example of a ((horizontal merger.))
When two companies of different industries join together, it is an example of a ((conglomerate merger.))
When a company issues a poison pill to try to head off a hostile takeover attempt, the company will ((allow stockholders to buy shares of stock at prices lower than the market value.))
A corporate raider is trying to take over the Cliff Company. In an attempt to avoid this hostile takeover, The Cliff Company is requiring a large majority of stockholders to approve the takeover. Which method of evading a takeover attempt does this describe? ((a shark repellant))
When a group of investors borrows money from banks and other institutions to acquire a company and uses the assets of the purchased company to guarantee repayment of the loan, a ________ is taking place. ((leveraged buyout))
Entrepreneurship is the process of creating and managing a business to achieve desired objectives. ((TRUE))
Small businesses, by their very nature, are small and therefore do not contribute significantly to job creation. ((FALSE))
Manufacturing businesses are attractive to entrepreneurs because of the low start-up costs required to establish a manufacturing business. ((FALSE))
Trevor lives in Florida during the winter months. While in Florida, he rents out his main home in Colorado to people traveling there for the skiing season. Trevor is participating in the sharing economy. ((TRUE))
A drawback of franchising is the need for franchisees to strictly adhere to standardized operations. ((TRUE))
Marissa is a fitness expert who starts her own personal training company. She conducts fitness classes for women and provides personal training sessions for clients. Marissa can be regarded as a(n) ((entrepreneur.))
A person who is involved in the process of creating and managing a business to achieve his or her desired objectives is engaged in ((entrepreneurship.))
Randy owns a small, local catering business. He has been hired for company picnics, backyard barbecues, and family celebrations. As word of mouth has spread about his delicious food, good service, and fair prices, he has started getting more business. To handle this, he has hired three employees to help cook and serve food at these events. What kind of entrepreneur is Randy? ((a microentrepreneur))
What is the purpose of the Small Business Administration? ((It offers managerial and financial assistance to small businesses.))
What type of business has been growing faster than other classifiable firms and represents almost 30 percent of all small businesses? ((minority-owned businesses))
Which type of business has created a majority of net new jobs annually? ((small businesses))
Baubles and Bangles, an independent jewelry store, acquires jewelry and accessories from producers and wholesalers and sells them to consumers. Baubles and Bangles is a ((retailer.))
If Yvette wants to start her own business, what might persuade her to open a retail operation? ((Retailing requires low initial financing.))
Clarissa makes one-of-a-kind ceramic vases. To avoid the costs of opening a store, she sells these pieces on Etsy. This is an example of ((direct marketing.))
A clothing store located in a shopping mall wants to give customers an opportunity to place orders. What is one form of direct marketing the store can use to provide this service? ((It can send catalogs to its customers.))
While Justin attends college, he is working part-time for a cutlery company. He goes door-to-door and offers consumers face-to-face presentations in their homes. This is an example of ((direct selling.))
All of the following are true of direct selling EXCEPT ((it requires buying a lot of inventory.))
Which scenario best illustrates a small business operating in a retail industry as opposed to wholesaling its products? ((The Case Place imports cell phone covers and sells them directly to customers through kiosks at the mall.))
What is true of wholesalers? ((Wholesalers are extremely important because of the marketing activities they perform.))
Many service providers are considered retailers because they ((provide their services directly to consumers.))
Walt owns a business in which he offers standardized test preparation classes and individual tutoring for students. This is an example of a ((service business.))
In the manufacturing sector, small businesses sometimes have an advantage over large firms because small businesses ((can customize products to meet specific customer needs and wants.))
What is a difference between high technology businesses and other small businesses? ((High technology businesses require greater capital and have higher initial startup costs than other small businesses.))
Menami and Michael recently had their first child. As two working parents, they struggled to find good, economical childcare. As a result, they developed their own software to link parents with licensed, reasonably priced childcare providers. This is an example of the ((sharing economy.))
Bob and Tom live and work in Texas, but they love vacationing in the Bahamas. After several years, they decide to buy a vacation home in the Bahamas. When they are not there, they rent the property to other people for $1,200 per week. This is an example of ((the sharing economy.))
A challenge associated with the sharing economy is ((whether workers are viewed as independent contractors or employees.))
As an advantage of being a small business owner, independence means ((you get to be your own boss.))
Seamus has worked for a large corporation for years, and he feels like he just does not fit in the "corporate mold" anymore. He decides to start his own business because he wants the freedom to choose with whom he works and when and where he works. What advantage of small business ownership seems the most prevalent in this example? (( independence))
A small businesses wanting to minimize its operating costs might do all of the following EXCEPT ((hire and maintain a large finance department.))
Take One Inc. is a small apparel store started by an aspiring designer. The store needs to compete against larger, well-established multinational brands. What strategy will most help Take One Inc. avoid competition from larger firms? ((Focus on and target small market niches or product needs.))
Which description is the BEST example of a market niche? ((elderly people, residing in assisted living centers, who suffer from macular degeneration))
What is true regarding the reputation of small businesses? ((Because of their capacity to focus on narrow niches, small businesses can develop enviable reputations for quality and service.))
What is one of the difficulties faced by small business owners? ((worries about employee problems))
Mark owns his own small bakery. In addition to being the owner, he also serves as the manager, sales force, head baker, shipping and receiving clerk, bookkeeper, and custodian. Mark is the first person to arrive at the bakery every morning and the last to leave every night. What is the most applicable challenge Mark faces? ((a high stress level))
If a small business has to cope with growth, it requires the owner to ((give up a certain amount of direct authority.))
Chai Ling decides to open a Vietnamese restaurant in her neighborhood. She will be solely responsible for the management of the restaurant. Which problem is she likely to face while running the business? ((She may find it difficult to deal with the stress of rent increases or employee turnover.))
A business that lacks funds to operate normally faces a problem known as ((undercapitalization.))
Kaden and Elizabeth run a small farm stand on their property, selling vegetables, fruits, and plants to support themselves. During a particularly bad growing season, they try to get financing from a small rural bank in their community but they do not get it. What is the most likely reason for this? ((The small rural bank lacked the necessary financial expertise to counter the risks involved with small-business loans.))
Elaine has a great idea for starting a new business, but she is not sure if she has the knowledge or experience to run a growing business effectively. This relates to which cause of small business failure? ((managerial incompetence))
Leon starts a business using the savings from his previous job. He plans to run his business on the revenue generated from sales. However, after a few months, he finds it difficult to pay his staff, rent, and other expenses. Seasonal sales and inability to secure sufficient credit from local banks make it difficult for Leon to operate the business normally. Which cause of small-business failure does this scenario best illustrate? ((undercapitalization))
________ is considered to be the shortest path to failure in business. ((Undercapitalization))
Cruz is a very talented computer programmer who starts his own small software development firm. He finances the firm from his savings. Initially, he is able to get many projects from customers and completes them successfully. However, as his business grows, he fails to make good decisions in hiring new employees, billing the clients, and negotiating prices. He eventually ends up liquidating the business. Which cause of small-business failure does this scenario best illustrate? ((managerial inexperience))
Why do some small businesses choose to stay small and are not interested in wide-scale growth? ((Escalating costs with growth can threaten the success of the business.))
Amber starts a small firm that manufactures clothing for leading fashion brands in the United States. What would likely be a principal immediate threat to this firm? ((rising inflation))
It has often been said that the greatest impediment to the success of a small business is ((the entrepreneur.))
Manuel wants to start his own small business. All of the following are things he must to do to start a business EXCEPT (() hire employees to work for the business.))
A precise statement of the rationale for a business and a step-by-step explanation of how it will achieve its goals is known as a(n) ((business plan.))
Which statement is true of a business plan? ((A business plan should establish a strategy for acquiring sufficient funds to keep a business going.))
Arnie approaches a bank to get a loan for a business he plans to launch later that year. He explains his business idea and its capital requirements to the bank manager. The manager asks Arnie to provide a formal document that includes a rationale for the business, an explanation of how it will achieve its goals, an analysis of the competition, and estimates of income and expenses, among other information. What document does the bank want from Arnie? ((a business plan))
In the process of starting a business, along with developing a business plan, the entrepreneur needs to ((decide an appropriate legal form of business ownership.))
To make profits from a small business, the owner must first provide or obtain ________ to start the business and keep it running smoothly. ((capital))
Jayla wants to open her own business making cakes. She has $10,000 of her own money and receives another $20,000 from family and friends. This combined $30,000 that Jayla can use toward her new business is referred to as ((capital.))
Steven wants to start his own tour company. What will be the most important source of funds for this new business? ((Steven, the owner))
What is a source of equity financing? ((selling personal assets to raise funds))
Nathan sells a piece of land he inherited from his grandparents and starts a business with the proceeds. Which source of funding is this? ((equity financing))
Roland wants to start a small editing company. He has enough funds to start the business but not enough to run it effectively. What is one thing he can do to provide working capital to his company? ((He can refrain from drawing a full salary.))
Sandy works in a factory that manufactures decorative lighting for office spaces, hotels, and designer homes. She wants to start her own business because she believes that the market for decorative lighting has a great growth potential. However, she does not want to depend on anyone else for procuring the initial capital and wants to raise the amount herself. In this scenario, which method could Sandy adopt to raise funds through equity financing? ((Sandy could invest her savings in the new business.))
Liam and Rachel are business partners. They initially started the business with their savings, but now the company plans to expand its operations, and the required amount of capital cannot be raised through savings or by reinvesting profits. Thus, the partners decide to sell stock in their business to family members, friends, and employees. Which source of capital are Liam and Rachel using for the expansion of their business? ((equity financing))
Persons or organizations that agree to provide some funds for a new business in exchange for ownership interest or stock are called ((venture capitalists.))
Mark funds small businesses that he believes have the potential to grow large. When these companies are still in their initial stages and need investment, he buys their stocks at a low price and later sells them at higher prices when they are successful. Thus, Mark is a(n) ((venture capitalist.))
Obtaining money from venture capitalists to start a new business venture is an example of ((equity financing.))
Which entity is the main supplier of external financing to small businesses? ((banks))
Securing a mortgage from a bank for a new business venture is an example of ((debt financing.))
Yolanda requests financing from the bank for her small hair salon. The bank requires Yolanda to put up a financial interest in the property or fixtures of the business to guarantee payment of the debt. What is this financial interest called? ((collateral))
If a small business fails to repay a bank loan, what will the lending institution do to recover its loss? ((claim and sell the collateral or mortgage))
Banks and other financial institutions can grant a small business a ________, which is an agreement by which a financial institution promises to lend a business a predetermined sum on demand. ((line of credit))
Small businesses may obtain funding from their suppliers in the form of a ________, which means that suppliers allow the business to take possession of the needed goods and services and pay for them at a later date or in installments. ((trade credit))
Tunde, an entrepreneur, starts a consulting firm with very limited funds. In order to conduct business, he purchases a few laptops and other office equipment from a local electronics wholesaler. The wholesaler understands his financial situation and allows him to pay for the goods in installments. In this scenario, the type of funding obtained by Tunde can be regarded as a ((trade credit.))
Nina recently opened a spice shop. She did not have enough savings and, hence, applied for a loan to obtain the additional funds required to start the shop. She had to provide her house as collateral to obtain the loan. The type of financing obtained by Nina can be regarded as a ((mortgage.))
Which statement best describes bartering? ((It is the practice of small businesses trading their own products for the goods and services offered by other businesses.))
Tabitha borrows some capital for her business at a low rate of interest from a community organization called Women Entrepreneurs Unite. Which form of funding a new small business does this scenario best illustrate? ((debt financing))
Frank, the owner of a local hardware store, decides to stock extra snow blowers and shovels this year, since his city is predicted to have an unusually snowy winter. Lacking sufficient capital to purchase the extra inventory, he borrows a predetermined sum of money from his bank. Which source of funding does this scenario best illustrate? ((line of credit))
Juanita wants to run her own bowling alley, but she does not want to start it from scratch. She decides to buy an existing business. What is one advantage of doing this? ((It provides a built-in network of customers, suppliers, and distributors.))
Since Sanjana decides to use her savings to purchase the rights to own and operate a Subway outlet rather than starting her own restaurant, she is most likely to be a(n) ((franchisee.))
Cole started the first outlet of Powerhouse, a youth athletic training facility in Chicago. The business expanded over time, and he became the owner of a chain of facilities in Illinois. Though the business had potential to expand outside Illinois, Cole did not want to take the risk or responsibility. Hence, he decided to sell the license to own and operate facilities under the brand name Powerhouse to independent owners. In return, he charged them an initial fee and a small share in the monthly profits. In this scenario, Cole is a(n) ((franchiser.))
In the business approach known as franchising, ((a franchisee often receives building specifications and designs from the franchiser.))
Which statement describes an advantage of franchising instead of building an independent business from scratch? ((A franchise outlet often reaches the break-even point faster than an independent business would.))
All of the following are advantages of franchising EXCEPT ((more freedom in purchasing goods.))
Jill has always dreamed of owning and managing a hotel. Instead of starting her own independent hotel, she decides to buy a franchise from Hilton. What is one thing she should keep in mind about becoming a franchisee? (())
What is one commonly reported disadvantage of franchising? ((restrictions on purchasing))
Claire opened her own Starbucks franchise in her community. She is pleased with how quickly she was able to open her franchise and that she was able to establish a good customer base from the start because of name recognition. However, since she started this business to be her own boss, Claire is likely to ((get frustrated with the restrictions of the franchise.))
All of the following are programs and resources a small business owner can use to improve his or her ability to compete with other businesses EXCEPT ((classified files from his competitors.))
Jessie owns a small data processing company. In setting up her business, she asks a few other small-business owners she knows for advice on dealing with employees and government regulation, improving processes, and solving problems. What does this scenario exemplify? ((networking))
Which statement accurately describes baby boomers? ((They are not actively pursued by small businesses.))
Millennials number around 83 million and represent a huge business opportunity in the United States. What is a defining feature of millennials? ((This cohort is concerned with advancement, recognition, and improved capabilities.))
Rosemary was born in 1952. She is a wealthy North Carolina native. As she ages, Rosemary spends more of her money on travel, financial planning, and health care. Rosemary is a(n) ((baby boomer.))
What is an implication of technological advances to small businesses? ((It provides new opportunities for small businesses to expand their operations abroad.))
Which group represents over 17 percent of those living in the United States and is largely employed in the retail industry? ((immigrants))
What is true of home-based businesses? ((Approximately 52 percent of all small businesses are based out of the home.))
What affect has the economic turbulence of recent years had on small businesses? ((It has provided both opportunities and threats for small businesses.))
A company is struggling in the marketplace and decides to reduce layers of management, personnel, and work tasks in order to remain competitive. This company is ((downsizing))
What is one purpose for downsizing? ((to gain the advantages of small businesses))
Bette works for a large company that is trying to function like a smaller company. What might the company do to achieve this goal? ((The company can make its operating units function more like independent small businesses.))
Which statement accurately describes intrapreneurs? ((Individuals in large firms who take responsibility for the development of innovations within the organizations are intrapreneurs.))
Brett works for a large sporting goods corporation. He champions a new football helmet with advanced concussion protection. Brett is a(n) ((intrapreneur.))
Javier is an employee at a company that provides information technology solutions to other firms. Recognizing his potential to innovate, the top management of the company gives him the complete liberty to develop a new smartphone application using company resources. In this scenario, Javier best illustrates a(n) ((intrapreneur.))
Any organization requiring the coordination of resources requires management. ((TRUE))
For any company, owners and shareholders are the only sources of primary funding. ((FALSE))
Directing involves forecasting events and determining the best course of action from a set of options or choices. ((FALSE))
The first step in the decision-making process is developing options directly. ((FALSE))
Grace manages a beach rental kiosk. Her goal is to run this business in such a way that she achieves the desired result, making a profit and providing her customers with the items they need to enjoy their vacation. Thus, her primary managerial concern is ((effectiveness.))
If a manager is concerned about doing the work with the least cost and waste possible, then his or her primary managerial concern is ((efficiency.))
Hiring people to carry out the work of an organization is known as ((staffing.))
Bradley is a manager at a company that has made the difficult decision to downsize. After his company eliminated 50 employees, what must Bradley do to effectively manage his remaining employees and help them through the situation? ((promote optimism and positive thinking))
The Electronics Warehouse wants to make sure its products are always available to customers. Since the company sells many electronic products, including computers, televisions, printers, and gaming equipment, it will need many diverse ________ to maximize efficiencies and provide creative solutions to help the company reduce expenses and reach its objectives. ((suppliers))
When a manager makes sure there is adequate funding to pay for essential activities, he or she is managing ________ resources. ((financial))
To earn a profit, managers must acquire, coordinate, and control all of the following resources EXCEPT ((competitive products.))
Ross is one of the senior managers of a large company. He is part of a team that determines the objectives of the company based on future trends in the industry and decides how to accomplish them. Ross is involved in the management function of ((planning.))
Which management function is crucial because it lays the foundation for all other management functions? ((planning))
Before an organization can plan a course of action, it must first ((determine what it wants to achieve.))
Which question does the mission statement of an organization seek to answer? ((What business are we in?))
Which of these basic questions will be answered by a well-developed mission statement? ((Who are our customers?))
Chip owns a scuba shop in Australia. His business is doing well, catering to local divers and tourists who want to learn to dive. He has developed a strong organizational culture, a good marketing plan, and a coherent business strategy. What serves as the foundation for developing these strengths? ((a mission statement that delivers a clear answer to the five basic questions))
Trevor is setting goals for his sales team. What should Trevor keep in mind when developing his goal? ((He should make sure any goal states exactly what is to be achieved.))
What should be kept in mind when setting company goals? ((A goal has three key components: an attribute sought, a target to be achieved, and a time frame in which the goal is to be achieved.))
The principal difference between objectives and goals is that ((objectives are measurable, while goals need not be measurable.))
A company objective related to ________ is generally stated in terms of percentage of sales increase and market share, with the goal of increasing those figures. ((competitive advantage))
Growth objectives essentially relate to ((adapting and releasing new products in the market.))
Get Fit Gym wants to make the best use of its resources. As a result, the company hires college students to work at the front desk, so personal trainers can focus on helping members in the gym and teaching fitness classes. What type of objective is the most relevant to this example? ((efficiency))
________ provide direction for all managerial decisions and establish criteria for evaluating performance within organizations. ((Objectives))
________ plans establish the long-range objectives and the overall course of action by which a firm fulfills its mission. ((Strategic))
Strategic plans generally cover a time frame of ((one year or longer.))
The top management of Hipster Clothing decides to set up an online store to cater to the changing business environment and reach out to the Hispanic market. The management also identifies that the success of the company will depend on its ability to create and develop new products and processes in the next two years. This is an example of ((strategic planning.))
All of the following are true of strategic plans EXCEPT ((can be developed by any manager in a firm.))
Tactical plans usually cover a period of ((one year or less.))
What is a difference between strategic and tactical plans? ((Strategic plans are long term, while tactical plans are short term.))
What type of plan helps keep the organization on the course established in the strategic plan? ((tactical plans))
Easy Does It, a home improvement store, has a strategic plan to open a new store next year. As a result, it develops a plan to determine what needs to be accomplished in order to get the new store up and running. This is a ________ plan. ((tactical))
________ plans are very short-term plans that specify what actions individuals, work groups, or departments need to accomplish in a month, a week, or even a day. ((Operational))
What is an example of an operational plan? ((A work group at a large manufacturing plant is assigned a weekly production quota to ensure there are sufficient products available to elevate market share and ultimately help the firm be number one in its product category.))
TopToBottom.com, a leading online retailer, uses the services of Privacy Pals Inc., a small startup that helps companies manage and keep their electronic data safe. This helps TopToBottom.com keep a backup of its proprietary information in case of virus attacks or any sort of damage to property. This is an example of ((contingency planning.))
What is true regarding crisis management? (() Many companies have crisis management teams to deal specifically with problems, permitting other managers to continue to focus on their regular duties.))
Dividing work into small units and assigning it to specific individuals is a task related to the management function of ((organizing.))
Vanessa works for a large cable company and is the manager for its service and repair team. Each day Vanessa reviews the list of job orders and locations of each job. She divides the job orders among her repair team so the jobs are completed in the most efficient manner possible based on where each job is located. What function of management does this represent? ((organizing))
All of the following are reasons organizing is important EXCEPT ((eliminates synergy.))
Mitch owns a large moving company. Over the years, the market has changed, and Mitch has had to adapt how he structures his resources and activities to continue to accomplish his business objectives efficiently and effectively. What does this scenario demonstrate? ((organizing occurs continuously))
Providing incentives to employees to motivate them toward achieving organizational objectives relates to the management function of ((directing.))
Emma manages workflow at a magazine. She sets and implements the deadlines and encourages the writers and editors to do their work. What managerial function is Emma fulfilling? ((directing))
Frieda is a sales manager for a uniform supply company. Her favorite part of the day is when she gets to interact one-on-one with her employees motivating them, providing leadership, and rewarding them for doing their jobs well. Frieda enjoys the ________ function of management. ((directing))
________ is the process of evaluating and correcting activities to keep an organization on course. ((Controlling))
When Tammy informs her supervisor that the furniture assemblers at their manufacturing plant are using 20 percent more raw materials in the current month than in preceding months, she is involved in the management function of ((controlling.))
For an organization, the first step in the management function of controlling is to ((measure the actual performance.))
Jaclyn manages a fast food restaurant. One day, while sitting in the restaurant taking her break, she hears a group of customers complaining about how slow the service has been getting and that they may have to stop eating there. What is the most effective control action Jaclyn can take to resolve the problem? ((She can watch her employees work to determine the cause of the problem, and then respond appropriately.))
In an organization, decisions regarding adding new products, acquiring companies, and moving into foreign markets would most typically be made by ((the top management.))
Isra manages a small coffee house. In fact, this business is so small that Isra is the only manager. What level manager is Isra? ((Isra assumes the responsibilities of all three levels.))
What is true of top management compensation? ((Many firms are trying to align CEO compensation with performance.))
Kylene is a manager at a large technology company. Since her company wants to serve consumers from fast-growing demographic groups such as Hispanics, African Americans, and Asian Americans, she wants to hire employees from those groups to help make decisions regarding issues related to consumer diversity. All of the following are rules she should follow in diversity recruiting EXCEPT ((limit hiring to individuals from the chosen demographic groups.))
Who is responsible for tactical and operational planning in an organization? ((middle managers))
The specific operations of an organization, plant, division, or department would most typically involve the ________ of the company. ((middle management))
What has been a recent trend with regard to the different levels of management within an organization? ((The ranks of middle managers have been shrinking as more and more companies downsize to be more productive.))
Sven manages the computer department at his company. He helps the organization implement its general guidelines by managing the specific operations related to his department. Sven is a ((middle manager.))
Positions such as foremen, supervisors, and office service managers are a part of the ________ of a company. ((first-line management))
Oliver is a manager at a clothing store. He spends most of his time in the store with his employees, making sure they work their scheduled hours, watching them interact with customers, and ensuring all the daily tasks are completed sufficiently. Oliver is a ((first-line manager.))
Roger and his team members are primarily responsible for procuring the funds needed for the successful operation of the organization and investing that money to pursue organizational goals. Thus, Roger is a ((financial manager.))
David is a bank manager. He manages the activities of an entire branch without specializing in any one particular function. Thus, David is a(n) ((administrative manager.))
Production and operations managers are concerned with ((transforming resources into goods and services))
A marketing manager is responsible for ((planning, pricing, and promoting products and overseeing their distribution))
Tracy is a manager at a law firm. Part of her role is conducting performance reviews for the paralegals in the office. By doing so, she is fulfilling the managerial role of a ((leader.))
Lloyd is the manager of the marketing department for a large firm. His department was given 3 new computer systems. Lloyd must determine which of his employees should get the new equipment. By doing so, Lloyd is acting as a ((resource allocator))
Daniel is a first-line manager at a heating and cooling company. Because Daniel has worked in the field for over 20 years, he is able to train employees, answer questions, provide guidance, and solve problems. These abilities most closely describe his ((technical expertise.))
The ________ style of leadership can be a powerful motivator because it demonstrates a great deal of trust and confidence in an employee and allows employees to meet their work demands with little or no interference. ((free-rein))
Which statement is true of authentic leaders? ((Authentic leaders are identified by the way in which they conduct themselves with stakeholders.))
Which statement is true of employee empowerment? ((Employee empowerment does not mean that managers are not needed.))
Chippers, a new bar and restaurant, encourages its employees and managers to participate in creating specialty drinks and menu items. This is an example of ((participative decision making.))
Which is the first step in decision making? ((recognizing and defining the decision situation))
The managers at Hydro Inc. have recognized declining sales on their water purifiers and must decide what to do to improve it. What should be their first step? ((defining the situation))
When selecting the best option among a series of possible decisions, ((it may be possible to use a combination of several options.))
When analyzing options in the decision-making process, managers must consider the appropriateness and ________ of each option. ((practicality))
Chrissy wants to develop a list of possible courses of action to draw more customers to her shop, so she calls a meeting of her sales staff. Together, they discuss the issue and suggest ideas to solve the problem. This is called ((brainstorming.))
Which statement best describes the importance of monitoring the consequences of decisions? ((The consequences of decisions may not be apparent quickly enough without monitoring.))
A(n) ________ is like a calendar, containing both specific and vague items, which helps a manager figure out what must be done and how to get it done to meet the objectives set by the organization. ((agenda))
The process of building relationships and sharing information with colleagues who can help a manager achieve the items on his or her agenda is referred to as ((networking.))
Liz manages a large floor refinishing company. She spends a lot of time communicating with a variety of people and participating in activities that do not seem to have much to do with goals of her company but are crucial for getting the job done. What is this called? ((networking))
All of the following are challenges managers confront in the business world today EXCEPT ((the stagnant nature of the workforce.))
The more complex organizations become, the less they need to develop formal structures to function efficiently. ((FALSE))
A wide span of management works best when the level of interaction between superiors and subordinates is low. ((TRUE))
Multidivisional structure is the simplest organizational structure that is based on direct lines of authority extending from the top executive to the lowest level employees of an organization. ((FALSE))
Horizontal communication involves the traditional flow of information from upper organizational levels downward. ((FALSE))
What is true of organizational culture? ((Organizational culture is more commonly expressed informally))
An organizational structure is ((the arrangement or relationship of positions within an organization.))
Marvin started a small restaurant business. At first, he did everything himself, buying the ingredients, preparing and serving the food, creating the menu and setting the prices, and handling the finances of the restaurant. As his business grows, Marvin may have to do all of the following EXCEPT ((keep doing everything himself.))
What is a primary function of an organizational chart? ((It is a visual display of the organizational structure.))
What does an organizational chart display? ((chain of command))
Grayson owns a local smoothie store. His business started small, but over the years, word of mouth has spread, bringing more people in to try his delicious homemade smoothies. Since business is booming, Grayson decides to expand his business and open three additional stores in surrounding towns. What does growth like this require in terms of structure? ((Growth requires developing a formal structure to function efficiently.))
________ is the division of labor into small, specific tasks and the assignment of employees to do a single task. ((Specialization))
What is the rationale for specialization? (( efficiency))
Motor Parts, a large auto parts store, did an experiment to see whether it was faster to have one person work on each car repair or have a team of people work together on a repair. It found that each employee, working independently, could do three to five auto repairs per day. However, when one person removed the old part, another person prepared the new part, and a third person installed the new part, they were able to complete up to twenty repairs per day. This scenario demonstrates that ((people can perform more efficiently if they master just one task rather than all tasks.))
What is an advantage of job specialization? ((It makes training employees easier.))
Which is a disadvantage of job specialization? ((It can lead to dissatisfaction and boredom among employees if overly done.))
Adam Smith, an 18th-century economist, illustrated improvements in efficiency in manufacturing units through the application of ((specialization.))
________ is the grouping of jobs into working units usually called units, groups, or divisions. ((Departmentalization))
Sam owns a large manufacturing company that supplies food and beverages to grocery stores. His company uses more than one departmentalization plan to enhance productivity. Most likely, his company would have all of the following departments EXCEPT a ________ department. ((fire))
Small businesses commonly employ ________ departmentalization. ((functional))
A company that has departments for marketing, finance, human resources, and production is organized by ((function.))
If the type of departmentalization used by an organization tends to emphasize departmental units rather than the organization as a whole and decision making is slow, then these would be weaknesses indicative of ________ departmentalization. ((functional))
World Travel Tours is organized into two departments: one for frequent travelers and one for occasional travelers. This illustrates ________ departmentalization. ((customer))
In comparing customer departmentalization and geographical departmentalization, it can be said that ((neither focuses on the organization as a whole.))
Pierce Products has a computer electronics division, telecommunications equipment division, and kitchen appliances division. Pierce Products uses ________ departmentalization to structure its organization. (( product))
Which statement represents an advantage of product departmentalization? ((It simplifies decision making and helps coordinate all activities related to a product.))
Ski and Surf, a large distributor of skiing and diving equipment in the United States, is organized into four business units: (1) Mountain States Ski, (2) West Coast Surf, (3) South Central States, and (4) Central States. What two types of departmentalization are illustrated in this example? ((product and geographical))
Multinational corporations often use a geographical approach to departmentalization because ((of vast differences between different regions.))
Blue Spruce Products groups employees based on whether they handle the sales accounts of wholesale customers, regular consumers, first-time buyers, or one-time buyers. Which method of departmentalization is the company using? ((customer departmentalization))
Master Mopeds Inc. has slightly different versions of the same moped for the American, European, and Asian markets. Its employees are grouped into divisions based on the market in which they have been assigned. This is an example of ________ departmentalization. (( geographical))
Alarm Solutions Inc., a large theft protection company, has different representatives to assist business clients and individual clients. This is an example of ________ departmentalization. ((customer))
Which statement represents a disadvantage of geographical departmentalization? ((It results in duplication of tasks.))
Which concept deals with giving employees the power to make commitments and use resources to accomplish the assigned tasks? ((delegation of authority))
________ is the principle that employees who accept an assignment and the authority to carry it out are answerable to a superior for the outcome. ((Accountability))
Soumya, the president of a higher-education publishing firm, delegates responsibility for all editorial activities to Sarah, the executive editor. Sarah accepts this responsibility and has the authority to obtain all relevant information, make decisions about what to publish, and delegate any and all activities to her subordinates. Sarah delegates all the editorial activities to the managing editors of each academic discipline. In turn, these managers assign specific projects to their development editors. Who is ultimately accountable to Soumya for all the editorial activities? ((Sarah))
Kent is a salesman at a large water products distributor where all the decisions are made by the top levels of management. However, his managers give him the responsibility to carry out his daily and routine sales activities. What type of organization does Kevin work for? ((centralized))
A drawback of centralization is that it ((may take a long time for an organization as a whole to implement decisions and respond to changes and problems on a regional scale.))
Caleb is a manager at a small diner in a tourist town. There are many competing restaurants in the area, so the restaurant owners depend on Caleb and his employees to pay attention to what customers order most often to determine what the daily and weekly specials should be. Caleb has the authority to make changes to the menu and offer promotions to attract more customers. What type of organization does Caleb manage? ((decentralized))
In a(n) ________ organization, the decision-making authority is delegated as far down the chain of command as possible. ((decentralized))
Decentralization works best in companies ((that operate in complex, unpredictable environments.))
Span of management refers to the ((number of subordinates reporting to a manager.))
A wide span of management is appropriate when ((subordinates are highly competent.))
A narrow span of management is appropriate when ((superiors and subordinates are not in close proximity.))
A narrow span of management would typically be found in? ((firms with centralized authority.))
Which statement accurately describes the difference between tall and flat organizations? ((Tall organizations have a narrow span of management, whereas flat organizations have a wide span of management.))
The more layers in an organization, ((the slower the communication.))
An organizational structure that is based on direct lines of authority from the top executive to the lowest level of employees is called a ________ structure. ((line))
An advantage of line structure in organizations is that it ((enables managers to make decisions quickly.))
A disadvantage of line structures is that they ((require managers to possess a wide range of knowledge and skills.))
A line-and-staff structure ((allows line managers to focus on their expertise in the business operation.))
A disadvantage of line-and-staff structures is that they ((create ambiguous lines of communication.))
An advantage of multidivisional structures is that they ((allow divisional and department managers to specialize.))
A disadvantage of a multidivisional structure is that it ((inevitably creates work duplication.))
Several years ago, Russ started his own heating and cooling business. As his business grew, Russ noticed that his traditional structure was becoming difficult to coordinate, making communication difficult and decision making slow. He decided to restructure his business, organizing his departments into six larger groups: (1) heating repairs, (2) heating sales, (3) heating installation, (4) air conditioning repairs, (5) air conditioning sales, and (6) air conditioning installation. What type of structure has Russ adopted? (() multidivisional))
At Infrastructure Innovations, specialists from different departments are deployed together to form a project team and work on a single project, such as building a bridge or restoring a road. As a result, employees have to report to their functional managers as well the concerned project managers. Thus, Infrastructure Innovations is using a ________ structure. ((matrix))
What is a basis for using a matrix structure within an organization? ((projects))
A ________ is also called a project management structure. ((matrix structure))
Matrix organizational structures ((have employees reporting to two managers.))
An advantage of a matrix structure is that it ((provides flexibility, enhanced cooperation, and creativity.))
A disadvantage of matrix structures is that they ((are generally expensive and quite complex.))
Compared to work teams, work groups emphasize ((individual accountability.))
A permanent, formal group that performs a specific task is a ((committee.))
A firm has an independent, permanent work group that develops and revises codes of ethics, suggests ways the organization can contribute toward the environment, and regularly reviews other specific issues and concerns within the organization. This is an example of a ((committee.))
A temporary group of employees, usually chosen for their expertise, that is responsible for bringing about a particular change is a(n) ((task force.))
Which statement about task forces is true? ((They may occasionally be composed of individuals from outside an organization.))
Once a month, the sales managers of Black Corporation meet to discuss sales performance, production issues, and delivery schedules. Since the sales managers work in different parts of the country, they conduct their meetings through a videoconference and follow up with e-mails. This is an example of a ((virtual team.))
After a large staffing firm reorganized and laid off 50 employees, it faced lawsuits alleging that the layoffs targeted minorities and women. The firm assembled a group to conduct a two-year study to address the allegations and provide recommendations for improving company reorganization practices. What type of group does this scenario describe? ((a task force))
A special type of project team that focuses on devising, designing, and implementing a new product is a ((product-development team))
To meet the competition in the mobile phone industry, Trestor Electronics Inc. creates a new project team to conceive, design, and implement a smart phone that will have some breakthrough features. The team includes specialists from various divisions, such as from computers, software, and television. This project team is an example of a ((product-development team.))
Which type of team consists of fairly small groups of workers brought together to address specific quality, service, or productivity problems within an organization? ((quality-assurance teams))
Access Motors released a new car model a year ago, and it has received customer complaints about the flimsy plastic used around the lights and in the interior consoles of the car. To address this problem, Access Motors forms a quality circle. What is a benefit of forming this type of group? ((It will shift the organization to a more participative culture.))
Accurate Awnings is experiencing production delays resulting in a backlog of customer orders, so the firm assembles a team to look into the issue. The team is given full authority to implement a new process for streamlining production without having to get management approval. In this scenario, the team is likely to be ((self-directed.))
What is true of self-directed work teams? ((They are designed to give employees a feeling of ownership of a whole job.))
All of the following are alternatives to face-to-face communication EXCEPT ((traditional meetings.))
Which problem has been created by increased access to the Internet at work? ((Employees may abuse company e-mail and Internet access.))
What is upward communication? ((communication that flows from lower to higher levels of an organization))
Noreen is a supervisor at a fast food restaurant. Every week, she submits a progress report to Micah, the restaurant manager, to let him know how the restaurant is doing and to bring any employee performance concerns and customer feedback to her attention. What type of communication does this scenario describe? ((upward communication))
Hans is a manager of a bookstore. At the end of each year, he gives his employees performance reviews to let them know what they are doing well, what they need to improve on, and their annual raise amount. This is a form of ________ communication. ((downward))
Downward communication typically conveys ((assignments of tasks and responsibilities.))
Melvin and Darryl are both managers at a sporting goods store. Melvin is a manager in the finance department and Darryl is a lower-level manager in the shoe department. When these managers communicate, it is known as ________ communication. ((diagonal))
Lisa and Gin are friends who work at the same company but in different departments. They often go out for lunch and drinks after work, and sometimes, they send each other personal e-mails and instant messages at work. Such friendships comprise the ________ of a firm. ((informal organization))
Teresa is a manager of a real estate company. She is aware that her company has an active grapevine because she has heard her employees gossiping in the lunch room and around the office. All of the following are things she can do to use the grapevine to her advantage EXCEPT ((confronting her employees about gossip.))
The exchange of information among colleagues and peers on the same organizational level is referred to as ________ communication. ((horizontal))
Diagonal communication ((occurs when individuals from different units and departments within an organization communicate.))
Diverse Electronics Inc. wants to ensure that its products best suit the needs of its customers, so the head of the marketing department holds a meeting with the managers of the production department. This illustrates ________ communication. ((diagonal))
What do employees list as their top complaint, related to communication, in the workplace? ((Their managers fail to listen to their concerns.))
Which statement is true about e-mail communication? ((Using work e-mail accounts to send personal information may be against company policy.))
What motivates employees to perform on the job is the focus of human relations. ((TRUE))
The Hawthorne studies found that money was the primary motivator for employees. ((FALSE))
Theory Z is more participative and encourages lifelong employee commitment more than traditional management approaches. ((TRUE))
Rewarding appropriate behavior tends to be more effective to modify behavior in the long run than punishments for unacceptable behavior. ((TRUE))
The field of human relations has become increasingly important over the years as businesses strive to understand how to do all of the following EXCEPT ((boost company sales.))
Hiroaki teaches karate at a large martial arts studio. He emphasizes self-discipline and self-motivation throughout the class. At the end of class, he always gives students a piece of candy from the treasure chest as reward for good discipline. One week he was out of candy, and several students threw tantrums, showing a lack of self-discipline. What should Hiroaki do if he wants students to show self-discipline even when there is no reward? ((discuss the lifelong benefits of self-discipline))
An employee who is emotionally involved and committed to his or her work is said to be ((engaged.))
Low morale is likely to result in ((absenteeism.))
High morale contributes to ((high returns to stakeholders.))
Tasty Joe is a chain of small coffee shops that are located in college towns and mostly employ students. The company offers flexible scheduling, employee scholarship programs, and opportunities for advancement. As a result, it has loyal employees who have high levels of productivity. Based on this scenario, employees of Tasty Joe should have ((high morale.))
The personal satisfaction and enjoyment that a person feels from attaining a goal is referred to as a(n) ((intrinsic reward.))
Tatiana has always been passionate about fashion, so she went to fashion school to learn more about it and opened her own clothing boutique. She enjoys running her business and feels a deep sense of satisfaction every time she finds new styles to bring to her store. In this scenario, Tatiana is experiencing a(n) ((intrinsic reward.))
________ are benefits and/or recognition that a person receives from someone else. ((Extrinsic rewards))
What is NOT a tactic employers can use to retain good employees? ((limit communication with employees))
Hallie manages a fitness club, which employs front desk personnel, personal trainers, and fitness instructors. She wants to motivate her employees to work hard and maintain high levels of productivity. What should Hallie do to motivate her employees? ((provide growth opportunities for employees))
The birth of the study of human relations within business organizations can be traced to ((time and motion studies by Frederick W. Taylor and Frank and Lillian Gilbreth.))
According to the classical theory of motivation, what would motivate employees to work hard? ((financial incentives))
Which theory is associated with the piece-rate system? ((the classical theory of motivation))
Nolan works for a flashlight manufacturing company. He is paid a fixed amount for every unit he manufactures at the factory. He is paid higher rates for exceeding his set target. The payment system in this scenario is an example of the ________ system. ((piece-rate))
In his classical theory of motivation, Frederick Taylor believed that two things would improve worker productivity: linking pay to output and ((job specialization.))
The person primarily associated with the Hawthorne studies is ((Elton Mayo.))
The Hawthorne studies were important because they ((revealed the significance of social and psychological factors on productivity.))
What was the ultimate result of the Hawthorne studies? ((They marked the beginning of a concern for human relations in the workplace.))
According to Maslow, which need do people strive to satisfy first? ((physiological needs))
According to Maslow, humans devote all their efforts to satisfying ________ needs until they are met. Only when these needs are met can people focus their attention on satisfying the next level of needs. ((physiological))
Al works for a company that books vacation rentals. He is a competitive person, so he needs to feel like he can bring in more customers and book more rentals than the other employees. When he is able to book more rentals than his co-workers, he feels a high level of self-respect as well as respect from others. This scenario describes how Al fulfills his ________ needs. ((esteem))
Frederick Herzberg proposed ((the two-factor theory.))
A company has started using methods to give employees more responsibility and control and to involve them more in their work. The company views employees as team members and encourages them to share their ideas about how to improve its technology products. What is the benefit to using these methods? ((They motivate employees to higher levels of productivity and quality.))
Theory X and Theory Y are most closely associated with ((Douglas McGregor.))
Dmitry is a manager who believes that he can force his subordinates to work extra hours through close supervision. He instills fear among employees by threatening to cancel bonuses if deadlines are not met. Dmitry could be described as a ((Theory X manager according to McGregor.))
When it comes to managing employees, A Theory X manager is most likely to ((ignore the self-actualization needs of his employees.))
The approach that suggests that imagination, ingenuity, and creativity can help solve organizational problems, but most organizations do not make adequate use of these characteristics in their employees is ((Theory Y.))
Kelly, the sales manager at a retail store, is a supporter of micro-management tactics. She keeps a close watch on the members of her team to ensure that they do not slack at work. Kelly is most likely a manager who supports ((the Theory X approach.))
Theory Z was first described by ((William Ouchi.))
At Just Right Thermostats, managers and workers share responsibilities, managers stress employee participation in all aspects of company decision making, and employment is long term. What theory of management does Just Right Thermostats follow? ((Theory Z))
According to ________, how much people are willing to contribute to an organization depends on their assessment of the fairness of the rewards they will receive in exchange. ((equity theory))
What theory is based on the idea that employees try to maintain fairness or balance between inputs and outputs as compared to others in similar positions? (( equity theory))
How can managers avoid equity problems? ((by ensuring that rewards are distributed on the basis of performance))
Lisa learned that although she and Linda were both hired as part-time sales clerks at the same time and have similar backgrounds, Linda is paid $2 more per hour. Lisa decided to ask her boss for $2 more per hour because she thinks the pay is unfair. This scenario is related to which management theory? ((equity theory))
Taryn, Kate, and Jillian all work at a department store in the mall. They have been working there for the same amount of time, and they all have similar educational and employment backgrounds. Jillian has just discovered that Taryn and Kate both make more money than her. What is an unethical or illegal behavior Jillian may exhibit to equalize the situation? ((She may equalize the situation by stealing items from the store.))
What is an advantage associated with job rotation? ((It reduces the boredom created by specialization of tasks.))
What is a drawback of job rotation? ((It is unable to totally eliminate the problem of boredom.))
Lighthouse Inc., a publishing company, allows its employees to try out a variety of positions, ranging from editing to designing cover pages. The practice at Lighthouse Inc. best exemplifies ((job rotation.))
Which strategy adds tasks to a job instead of treating each task as a separate job? ((job enlargement))
The rationale behind a job enlargement strategy is that jobs are more satisfying when ((the number of tasks performed by an individual increases.))
Job enlargement strategies differ from job rotation strategies in that job enlargement ((increases job satisfaction more than job rotation.))
Gretchen works for a home decor store. Her normal tasks include cleaning the store fixtures, helping customers find what they are looking for, and ringing up customer purchases. One day, she notices that several of the shelves are empty, so she goes into the back room and finds merchandise to fill the shelves. When she shows her manager, Alicia, what she has done, Alicia is so impressed that she asks Gretchen to check all the shelves on a weekly basis and stock any empty shelves with new merchandise. This scenario best describes job ((enlargement.))
Job ________ is the incorporation of motivational factors such as opportunity for achievement, recognition, responsibility, and advancement into a job. ((enrichment))
Which is a likely result of job enrichment? ((Employees will have more control and authority over their job.))
A work program that allows employees to choose their starting and ending times, as long as they are at work during a specified core period is called ((flextime.))
Master Marine, a boat shop, allows employees to log in at their preferred time and log off after completing eight hours. It does not have any fixed office timings. Master Marine offers ________ to its employees. ((flextime))
What is an advantage of using a flextime schedule? ((It improves the ability to recruit and retain workers who wish to balance work and home life.))
A compressed workweek is ((a four-day or shorter period in which employees work 40 hours.))
What strategy is being used by a company that allows its employees to work full-time in less than the typical five-day work week? ((a compressed workweek))
Tanker Corp., an oil tank manufacturing company, allocates one task to an employee working from 8:00 a.m. to 12:30 p.m. and allocates the same task to be completed by another employee working from 12:30 p.m. to 5:00 p.m. This is an example of job ((sharing.))
Job ________ reflects a flexible scheduling strategy. (( sharing))
Which option is likely to be most effective for retaining employees who are trying to juggle their work duties with other responsibilities and needs? ((job sharing))
Samantha and Geri both work as receptionists at a medical office. Samantha works from 8:00 a.m. to 12:30 p.m., so she can be home when her daughter gets home from school. Geri works from 12:30 p.m. to 5:00 p.m., so she can attend graduate school classes in the morning. How does this arrangement benefit the company? ((The company has the skills of two people for one job.))
Allowing some employees to work at home part-time is ((a flexible scheduling strategy.))
Lawlor Inc., a pharmaceutical company, allows its employees to work at home a few days per week, staying connected via computers, modems, and telephones. This arrangement is called (( telecommuting))
What is an advantage of allowing employees to telecommute? ((It helps reduce overhead costs for businesses.))
Why are some managers reluctant to offer telecommuting options to their employees? ((It increases security concerns.))
Planning, as a function of human resources management, includes determining the status of current personnel when planning human resource needs for the future. ((TRUE))
The cost of hiring from external sources is less expensive than promoting from within ((FALSE))
Conferences, case studies, and web-based training are all part of on-the-job-training. ((FALSE))
When managers speak of diverse workforces, they typically mean differences in skill and knowledge. ((FALSE))
Some companies refer to human resources management as ((personnel management.))
Nadene is a human resources manager at a large legal services company. She must decide how many new employees her company will need to fill vacant positions in the near future. In this scenario, Nadene is engaging in ((planning.))
A job ________ involves determining, through observation and study, the specific tasks that comprise a job; the knowledge, skills, and abilities necessary to perform the job; and the environment in which the job will be performed. ((analysis))
Network Press, a printing business, posts an advertisement inviting applicants to pursue a role as a graphic designer. The company specifies that the job will involve illustrating content by designing aesthetically tasteful layouts. This is an example of a job ((description.))
A job ________ describes the qualifications necessary for a specific job, in terms of education, experience, personal characteristics, and physical characteristics. ((specification))
Smith, Jones, and Johnson, a law firm, is looking to hire a lawyer, so it posts the job in several legal publications. In its ad, the company requests applicants who have completed law school and passed the bar examination, have courtroom experience, and are confident and hardworking. This describes ((a job specification.))
Laser Corp. needs to hire three new employees. It wants to select people from a pool of qualified applicants. What is the process of forming this pool of applicants? ((recruitment))
Heidi manages a hair salon, and she is looking for a new head stylist to lead her team of hairdressers. She posts an ad on several job websites and uses those websites, along with LinkedIn, to look for qualified candidates. She also watches her employees work to see if any of them exhibit the skills she is looking for in a head stylist. Once she receives a few applications, finds a few qualified candidates, and determines that one of her employees might be a good fit, she sets up interviews with these potential candidates to decide who to hire. Which human resources activity does this scenario describe? ((recruiting))
When compared to the external sources of recruitment, recruiting from internal sources ((improves employee morale.))
Which is an internal source of recruitment for an organization? ((current employees))
Why do many firms have a policy of giving first consideration to their own employees? ((The cost of hiring current employees to fill job openings is inexpensive.))
Tricor Inc., a software firm, is recruiting potential employees from engineering colleges to fill its entry-level vacancies. On the basis of this information, what observation holds true? ((The management at Tricor is hiring its employees from external sources.))
A characteristic of search firms known as headhunters is that they ((look for qualified candidates who are working for other companies.))
Executive Search, an HR consultancy firm, specializes in luring qualified people away from other companies. Many companies hire Executive Search to fill their managerial or professional positions above the entry level. On the basis of this information, Executive Search is included in the category of ((headhunters.))
George is the fire chief in a small town. He is looking for a new assistant chief, and he already has three candidates to consider. He is in the process of carefully considering their applications, conducting interviews, administering performance tests, and checking their references. What is this process called? (( selection))
A publishing house is in the process of selecting a new editor from a pool of potential candidates. All of the following would be part of this selection process EXCEPT ((posting an ad for the position on publishing career websites.))
When it comes to recruiting and selecting new employees, what is the first stage of the selection process? ((the application))
Jerome has been working as a marketing manager for the last seven years. He decides to move to be closer to his family, so he is looking for a new job. When filling out an online application for a new marketing job, what information might he be asked to provide? ((his current salary))
Tabitha applies for a sales position at a china and stemware company and gets invited to an in-person interview. All of the following are questions she can expect to be asked during the interview EXCEPT ((Are you married or in a relationship?))
Which stage of the selection process may require a candidate to undergo physical examinations to determine his or her suitability for a specific job? ((testing))
Reference checking usually involves ((checking the previous work experience of a candidate.))
The ________ is a federal agency established by the Civil Rights Act of 1964 that is dedicated to increasing job opportunities for women and minorities and eliminating job discrimination based on race, religion, color, sex, national origin, or handicap. ((Equal Employment Opportunity Commission))
The Age Discrimination in Employment Act specifically focuses on hiring practices that discriminate against ((people aged 40 and older))
In the United States, wage differences are acceptable only if they are ((attributed to seniority.))
Which practice is considered to be illegal in the United States? ((forcing employees to retire on the basis of age))
Oscar owns an auto repair shop, which he has run for 40 years. Although he has hired some younger mechanics, he prefers hiring older mechanics because he finds that they ((take pride in their work.))
Orientation is the human resources function that ((includes building tours, introductions, and socialization.))
What is an advantage of the orientation process? ((It allows firms to familiarize new employees with prevalent office culture.))
Cakes and More, a popular bakery, allows its new employees to learn the processes for making its bakery goods by actually working on customer orders. The training process at Cakes and More best exemplifies ((on-the-job training.))
Health Beat, a chain of healthcare providers, sends its employees who are interested in learning about hospital management to some of the leading institutes in the nation where they attend lectures and conferences. The training process at Health Beat best exemplifies ((classroom training.))
Even though Dr. Smith has hired a qualified, educated dental assistant, he still must make sure she knows how to do the specific job tasks he requires and how to operate the dental equipment correctly. This process would take place during ((training.))
Seaside Sporting Goods Store has recently hired several new employees who have the knowledge, skills, and abilities the company needs. Why is it still important that these new employees undergo training? ((to teach them how to do their specific job tasks))
Raphael has recently started working at an investment banking firm. To get him up and running quickly, the firm pairs him with an experienced investment banker to offer him support, training, and guidance as he learns the tasks of his new job. This is an example of ((mentoring.))
A mentor can be described as ((an experienced employee who supports, trains, and guides a new employee.))
What is the primary reason that training is considered a vital function of human resources management? ((It is used to improve the skills of employees in their present positions and to prepare them for increased responsibility.))
What increasingly popular method of providing training and development may include the use of role-plays and simulations? ((experiential and involvement-oriented training exercises))
A performance appraisal that is not tied to any quantifiable measure is considered to be ((a subjective assessment.))
The use of which type of assessment criteria has triggered lawsuits by employees? ((ranking and grading measures))
Which scenario is an example of an objective assessment of employee performance? ((Blake, a car salesman, is judged by the number of cars he has sold.))
A ________ is a performance appraisal method that provides feedback from a panel that typically includes superiors, peers, and subordinates ((360-degree feedback system))
Logan is a barista at a coffee shop. His manager notices that he has been showing up late for work, getting coffee orders wrong, and being rude to customers. What should his manager do to help Logan improve his work? ((give Logan constructive criticism, so he knows what to expect and how he is viewed))
What is a side effect of employee turnover due to employees quitting or being fired? ((an increase in recruitment costs))
When an employee moves to a new job that involves more responsibility and an increase in compensation, this transition would typically be considered a(n) ((promotion.))
Which is likely to be an unacceptable reason for firing an employee? ((being a union organizer))
A well-organized human resources department strives to ((minimize losses due to separations and transfers.))
A wage/salary survey indicates ((how much compensation similar firms are paying for jobs that the firms have in common.))
Trevor and Matt both work as farmhands and perform the exact same job tasks. Although they work on different farms, when comparing their wages, they determine that Matt makes $15 per hour, while Trevor only makes $12 per hour. What is most likely the reason for this difference in wages? ((Matt is more productive than Trevor.))
Which statement is true of payment using time wages? ((It is appropriate when employees are continually interrupted.))
What is an advantage of time wages? ((They can be computed easily.))
What is a disadvantage of time wages? ((They provide no incentive to increase productivity.))
What is a major advantage of piece wages? ((They motivate employees to increase output.))
At Marvelous Motors, employees are paid a percentage of their sales in order to motivate them to sell as many cars as they can. This company incorporates ________ as an incentive system. ((commissions))
Which person is most likely compensated through a salary? ((Lester, who works as a hospital administrator.))
Alan works as a news anchor for a television network. In addition to his regular pay, at the end of each year, his company pays him and his co-workers additional compensation as a "thank you" for good work. This monetary reward would most likely be categorized as a ((bonus))
What form of compensation includes an employee stock ownership plan? ((profit sharing))
The use of employee stock ownership plans ((creates a sense of partnership between a firm and its employees.))
________ are nonfinancial forms of compensation provided to employees. ((Benefits))
Isaac works for a paper products company that provides sick leave, vacation pay, pension plans, health plans, and other forms of extra compensation. These particular items are ((fringe benefits.))
Abigail works for a large architectural firm that employs many different types of employees. As a result, her company provides a financial amount to employees so they can select the specific benefits that fit their needs. This company uses ((a cafeteria benefit plan.))
Ritesh works in a factory that produces machine parts. He feels that he deserves better pay, hours, and working conditions. Although he has talked to management several times about these items, they still have not made any improvements. He decides to join his co-workers in a(n) ________, so they can hire specialists to represent them in their dealings with management. ((labor union))
The negotiation process through which management and unions reach an agreement about compensation, working hours, and working conditions for concerned employees is referred to as ((collective bargaining.))
What is the significance of the "cost-of-living adjustment" clause in labor contracts? ((It calls for automatic wage increases during periods of inflation.))
The purpose of a labor union is to ((help its members achieve better pay, hours, and working conditions.))
Union growth has slowed in recent years because ((factories have become more automated.))
The formal, written document that spells out the relationship between the union and management for a specified period of time is called a(n) ((labor contract.))
A labor union would most likely be expected or required to accept givebacks ((during tough economic times.))
The objective of collective bargaining negotiations is for labor and management to come to an agreement on ((a labor contract.))
Which statement is true of a strike? ((It halts or disrupts the normal working of a firm.))
Andre manages a toy packaging factory. After failed negotiations between Andre and the labor union, the union members strike, partially shutting down the factory. In response, Andre shuts down the factory completely, restricting all his employees from coming to work. This situation best represents a(n) ((lockout.))
When is a firm most likely to vote for a lockout? ((when a union strike has partially shut down a plant and it seems less expensive for the plant to close completely))
Why is strikebreaking generally considered to be a last-resort measure for management? ((It does great damage to the relationship between management and labor.))
The negotiations between a union and the management representatives of a firm come to a standstill and a third party is brought in to propose solutions to help resolve the impasse. However, the solutions are not binding on the parties. This third party would most likely be a(n) ((mediator.))
A similarity between conciliators and mediators is that ((neither of them has formal power over labor or management.))
The management at Tool Works, a tool manufacturer, is involved in a dispute with the union members representing its employees. The union members want a pay raise, but the management is not ready to approve the pay increase. Failing to arrive at a compromise, the disputing parties involve a third party to resolve the issue and offer a solution that is legally binding. The third party in the given scenario best represents a(n) ((arbitrator.))
Management of a firm and labor submit to compulsory arbitration when the federal government ((requests arbitration as a means of eliminating a prolonged strike that threatens to disrupt the economy))
The management at TKL Inc., a firm manufacturing cosmetics, is involved in a dispute with union members over employee compensation. Failing to arrive at a compromise, the disputing parties involve a third party to resolve the issue and offer a solution to the problems. The solution provided by the third party is open to the approval of both parties involved and is not likely to be binding. The third party in the given scenario best represents a(n) ((mediator.))
The management at Bellows Inc. is faced with a strike threat from union members over a dispute concerning working hours for employees. The disputing parties approach a neutral third party to ensure that the discussions between them are moving forward. The third party, however, has no formal power over union representatives or over management. Which process is best exemplified in the scenario? ((conciliation))
What is a primary characteristic of diversity? ((age))
What is a secondary characteristic of diversity? (( income))
If an organization fosters and values workforce diversity, it is most likely to ((reduce conflict among employees of different cultural groups.))
Companies that do not value their diverse employees are most likely to ((experience greater prejudice.))
________ programs are legally mandated plans that try to increase job opportunities for minority groups. ((Affirmative action))
A complaint of companies regarding affirmative action plans is that they ((stifle their ability to hire the best employees.))
What factor contributed to the controversies surrounding affirmative action plans? ((increased reverse discrimination))
Historically, CompuTech, a large technology company, has employed mostly white men. More recently, it has adopted affirmative action to make up for past hiring and promotion prejudices, overcome workplace discrimination, and provide equal employment opportunities for all employees. What should CompuTech avoid doing in its quest for a more diverse workplace? ((set hiring quotas))
A marketer must understand the needs of the buyer in order to determine what products to make available. ((TRUE))
Products are either marketed to the B2B market or the B2C market, but never to both. ((FALSE))
Buying behavior includes the behavior of both consumers purchasing products for personal or household use as well as organizations buying products for business use. ((TRUE))
It is necessary for nonprofits and government institutions to market themselves to spread awareness and achieve desired outcomes. ((TRUE))
A group of activities designed to expedite transactions by creating, distributing, pricing, and promoting goods, services, and ideas is referred to as ((marketing.))
The best way marketing activities create value is by ((allowing individuals and organizations to obtain what they need and want.))
For the products launched by companies to succeed, it is important that ((all the functional areas of the business are coordinated with marketing decisions.))
One of the important functions of ________ is to provide revenue to sustain a firm. ((marketing))
Martin goes to his favorite local bakery and hands the cashier $2 for a muffin. What has just taken place? ((an exchange))
Beth and Tom are celebrating their 10-year wedding anniversary and have decided to celebrate by going on a dinner cruise. Tom goes to the website "Dinner on the Dock" to book the cruise and pays for it with his credit card. This is an example of a(n) ((exchange.))
At the heart of all business is ________, the act of giving up one thing in return for something else. ((the exchange))
What condition must be met in order for an exchange to take place? ((Buyers and sellers must be able to communicate about the "something of value" available to each.))
Alfonso and Jorge both collect baseball cards. Alfonso has a valuable and rare card that Jorge has been wanting for a long time. What must Jorge do to convince Alfonso to part with this card? ((Jorge must offer Alfonso something he values enough to give up his valuable and rare card.))
The Coffee Clutch is a small, local company that, in addition to making delicious coffee drinks, offers its customers live music, open microphone nights, free Internet access, and comfortable seating so they can enjoy their coffee with friends or while working. In the case of this company, which statement is most likely true? ((The tangible product itself may not be as important as the benefits associated with the product.))
Sarah has produced 1,000 jars of raspberry jam, using the homegrown raspberries from her field. By concentrating her efforts on advertising and publicity, she is trying to encourage people to notice and buy her product. Through her promotional activities, Sarah is primarily engaging in the marketing function of ((selling.))
Potato Hut, a popular fast-food chain, is famous for its fries and hash browns. It is one of the few fast-food chains to exclusively use the potatoes from its own farms. However, its potatoes can be harvested for only three months of the year. Which marketing function can Potato Hut employ to achieve time utility and satisfy year-round demand for its exclusive fries and hash browns? ((storing))
Billy is a packer in the meat department of a grocery store. He labels beef steaks with little or no fat as "AA" and beef steaks with more fat as "AB." When he does this, he is primarily engaging in the marketing function of ((grading.))
________ is the chance of loss associated with marketing decisions. ((Risk))
Deborah owns a rustic furniture store in New Hampshire. When customers buy her unique furniture pieces, she arranges a time for her employees to deliver the furniture to their homes using her delivery truck. This is an example of ((transporting.))
Warehouses hold some products for lengthy periods in order to ((create time utility.))
________ refers to standardizing products by dividing them into subgroups and displaying and labeling them so that consumers clearly understand their nature and quality. ((Grading))
Ben is the marketing manager of a large department store that sells a variety of household appliances, including refrigerators, washing machines, and stoves. What might Ben do to help customers finance these large, expensive items? ((arrange credit))
Through ________, marketers ascertain the need for new goods and services. ((marketing research))
Olga is shopping for a new laptop computer. All the computer brands offer laptops with standard screens and keyboards, but each brand offers different software packages, memory amounts, processor speeds, hard drive sizes, and operating systems. Olga notices that the price of laptops varies depending on the features they include. In order to choose the best laptop computer for her, Olga will most likely ((judge which type of laptop offers her the best value according to the benefits she wants and her ability to pay for those benefits.))
For greater accessibility, Bowen Tools places its products at every popular superstore. The convenience provided by the company is an example of reducing ________ costs. ((nonmonetary))
What type of cost includes anything a buyer must give up to obtain the benefits a product provides? ((customer costs))
In the context of the marketing concept, which is the first step that a business must take? ((find out what consumers want))
To gain an edge, businesses must ((continually alter, adapt, and develop products to keep pace with changing consumer needs.))
A business is least likely to survive when it ((focuses only on customer satisfaction and not on its own objectives such as boosting productivity.))
The goal of the marketing concept is ((customer satisfaction.))
Which would be an ineffective practice to follow while implementing the marketing concept? ((adopting a product orientation instead of a consumer orientation to deliver the right good or service))
In the context of the evolution of the marketing concept, the production orientation was characterized by ((new technologies that made it possible to manufacture goods with ever increasing efficiency.))
Which orientation in the evolution of the marketing concept was characterized by the supply of manufactured goods catching up with and then exceeding demand, leading businesspeople to realize they would have to promote their products to get buyers to desire them? ((sales))
During the first half of the 20th century, businesspeople viewed ________ as the primary means of increasing profits. ((sales))
The market orientation approach emphasizes ((determining what customers want first and then producing it.))
A market orientation requires organizations to ((gather and use customer information to help build long-term relationships with customers.))
Why is customer relationship management important in a market orientation? ((It can result in loyal and profitable customers.))
According to the market orientation, profits can be obtained through ((acquiring new customers.))
Because it provides a way for businesses to personalize customer relationships, ________ is a major element of any strategy to develop and manage long-term customer relationships. ((communication))
A ________ is a plan of action for developing, pricing, distributing, and promoting products that meet the needs of specific customers. ((marketing strategy))
The first step in developing a marketing strategy is ((selecting a target market.))
Kellen wants to start a new copyediting business. He identifies his target market as publishing companies that outsource their copyediting projects. Now that he has identified his target market, what must he do next? ((develop an appropriate marketing mix))
A ________ is best described as a group of people who have a need, purchasing power, and the desire and authority to spend money on goods, services, and ideas. ((market))
A ________ refers to a specific group of consumers on whose needs and wants a company focuses its marketing efforts. (( target market))
Total Auto Parts manufactures engines, brakes, mufflers, and batteries and sells them to car manufacturers. This is an example of ((business-to-business marketing.))
Home and More sells rugs, art, lamps, and other decorative pieces to homeowners, so they can express their personal styles in their homes. This is an example of ((business-to-consumer marketing.))
A company that manufactures wooden planks for construction has categorized its customers on the basis of their product requirements into three groups: construction companies, wholesalers, and government institutions. Which marketing strategy is best illustrated in this scenario? ((market segmentation))
Precision Prescriptions manufactures and sells a variety of drugs for a large market consisting of people of different genders, ages, educational backgrounds, lifestyles, geographic locations, and income levels. It assumes that all buyers have similar medical needs and wants. In this scenario, Precision Prescriptions is most likely demonstrating a ________ to marketing. ((total-market approach))
Which product is most suitable for selling through a total-market approach? ((salt and sugar))
Why does the total-market approach work best for sellers of salt, sugar, and many agricultural products? ((Everyone is a potential consumer of these products.))
What is a challenge marketers in the United States will face in the future? ((effectively addressing an increasingly racially diverse population))
In the ________ approach to market segmentation, a company develops one marketing strategy for a single market segment. ((concentration))
HS-Mobile Inc. is a mobile phone retailer that sells smartphones to high school students at more affordable prices. The company has one marketing strategy for the entire market of high school students and focuses all its efforts on this one segment. In this scenario, HS-Mobile Inc. is most likely using the ________ approach for marketing its smartphones. ((concentration))
In the ________ approach, the marketer aims its marketing efforts at two or more segments, developing a marketing strategy for each. ((multisegmentmultisegment))
Best Brands Foods, a company that manufactures and sells breakfast cereals, customizes its cereal flavors to suit different lifestyles, personal tastes, and age groups. For example, it makes organic cereals for its health-conscious customers and colorful cereals with cartoon characters for children. Which market segment approach is Best Brands Foods using in this scenario? ((multisegment approach))
In ________ marketing, all marketing efforts focus on one small, well-defined market segment that has a unique, specific set of needs. ((niche))
The Golden Group is a group of luxury hotels that caters exclusively to high-end customers who form a small part of the market and are willing to pay for luxurious amenities. The hotel has incorporated all possible luxuries to satisfy these customers. The marketing approach employed by The Golden Group best exemplifies ((niche marketing.))
If a company develops an advertising campaign exclusively for a segment of consumers with a certain income and education, which segmentation variable is the company using? ((demographic))
A skin care company develops face creams for different generations of customers—an acne treatment cream for adolescents and a wrinkle-reducing cream for older people. These face creams were developed on the basis of ________ segmentation. ((demographic))
What is a basis for the psychographic segmentation of markets? ((lifestyle))
An automobile company manufactures different cars for different market segments. It markets its SUVs for customers who live in the mountains, sedans for customers in the coastal plains, and smaller hatchback cars for those in big cities. In this scenario, the company is primarily using ________ variables of market segmentation. ((geographic))
A golf club manufacturer has separate product lines for male and female golfers. It manufactures several types of golf clubs geared to each segment. Which basis of market segmentation is this company using? ((demographic))
A beverage manufacturing company has placed its products strategically in areas where the population density is high. This is an example of ________ segmentation. (( geographic))
Guilt-Free Sweets is a bakery that produces all gluten-free baked goods to target customers who have trouble digesting gluten but still want to be able to enjoy tasty treats. What type of segmentation is this company using? ((benefit segmentation))
An oil company wants to increase customer demand for oil. What can the company do to accomplish this? ((reduce the price of its oil))
Small Treasure Gifts is deciding whether to sell its products in kiosks outside subway stations or in stores at popular malls. What element of the marketing mix does this decision most closely relate? ((distribution))
A fashion clothier is contemplating whether to sell its products in brick and mortar stores or to sell them at its online store. What element of the marketing mix does this decision most closely relate? ((distribution))
A systematic, objective process of getting information about potential customers to guide marketing decisions is referred to as ((marketing research.))
Rich runs a home improvement business. He is happy with the level of business he receives, but he wants to learn more about his customers, so he studies information compiled by the U.S. census bureau and other government agencies, databases created by marketing research firms, and sales and other internal reports. What kind of data is Rich using in this scenario? ((secondary data))
________ refers to the decision processes and actions of people who purchase and use products. ((Buying behavior))
In the context of the psychological variables of buying behavior, ________ is the process by which a person selects, organizes, and interprets information received from his or her senses. ((perception))
What is a psychological variable of buying behavior? ((attitude))
Every year, Les has bought a calendar to keep track of his appointments and events. However, this year, he bought a smartphone, which has a built in calendar, so he did not buy a new paper calendar. Which psychological variable does this scenario most closely describe? (( learning))
What would be considered to be a social variable of buying behavior? ((culture))
________ are a set of expectations for individuals based on some position they occupy. ((Social roles))
The groups with whom buyers identify and whose values or attitudes they adopt are referred to as ((reference groups.))
Micah, Felicity, and Jasmine have developed a common pattern of behavior. They all own large houses, drive fancy cars, and belong to an elite country club. Micah, Felicity, and Jasmine most likely belong to the same ((social class.))
Lucia is a Hispanic American who manages a Mexican food restaurant. She often eats the food her restaurant sells, and several times a year, she travels to Mexico to visit her family and eat authentic Mexican cuisine. Which social variable does this scenario most closely describe? ((culture))
What is a social role that a person may have? ((a student))
Which factor is least likely to be a competitive or economic force in the marketing environment? ((the laws related to employment in a country))
What would be considered as regulatory forces in a marketing environment? ((political actions of interest groups))
If a foreign trade policy of a country states that multinational companies can enter its market only by merging with a local business partner, this would represent which environmental force? ((political))
Which statement is true of the marketing environment? ((A marketing manager can influence some environmental variables.))
Why does marketing require creativity and consumer focus? ((because environmental forces can change dramatically))
Political, legal, regulatory, social, competitive, economic, and technological forces comprise the ((marketing environment.))
The terms accounting and bookkeeping are interchangeable. ((FALSE))
The fundamental accounting equation shows the relationship between profit, revenue, and market share. ((FALSE))
Nonbusiness entities typically obtain revenues through the sale of their assets. ((FALSE))
Accounting refers to the process of ((recording, measuring, and interpreting financial information.))
What is a function of the Public Company Accounting Oversight Board? ((It makes rules and policies for accounting firms and businesses.))
Nadine is a self-employed, state-certified accountant who files tax returns, prepares financial records, and audits corporate financial records. She is known as a ________ accountant. ((certified public))
Randall wants to become a certified public accountant. To accomplish this, he must be certified by the ________ in which he plans to provide accounting services. ((state))
The purpose of the Sarbanes-Oxley Act was to ((require firms to be more rigorous in their accounting and reporting practices.))
What is a requirement stated by the Sarbanes-Oxley Act? ((This act requires firms to separate their consulting and auditing businesses.))
Brad is an executive of Music Masters, a wedding and special events DJ music service. For years, he successfully hid illegal and misleading accounting practices, but eventually, he was exposed and punished with a jail sentence under the ________ Act. ((Sarbanes-Oxley))
All of the following contributed to the financial crisis in 2008 EXCEPT ((government bailouts.))
Accounting scandals at ________ led to the passage of the Sarbanes-Oxley Act. ((Worldcom and Enron))
What type of accounting involves analyzing financial documents in search of fraudulent entries or financial misconduct? ((forensic))
Jack works for the Internal Revenue Service. His main responsibility is rooting out evidence of "cooked books" at major companies. Jack is a ________ accountant. ((forensic))
Marybeth has been an accountant for many years. Since the accounting scandals in 2000, her firm added fraud-detection services, and Marybeth decided to focus on these services. Which organization could certify Marybeth, so she can focus on detecting fraud? ((the Association of Certified Fraud Examiners))
Which accountant has titles such as controller, tax accountant, and internal auditor? ((private accountants))
Avantika is a private accountant who passed a rigorous examination by the Institute of Management Accountants. After passing this examination, Avantika is a ((certified management accountant.))
Kaden works for a publishing company, recording its routine, day-to-day business transactions. His main responsibility is obtaining and recording the information his company needs to analyze its financial position. Based on this description, Kaden is most likely a(n) ((bookkeeper.))
Which statement best describes bookkeeping? ((Bookkeeping is much more mechanical than accounting.))
The principal value of a budget lies in its ((breakdown of cash inflows and outflows))
Deviations between expected operating expenses and ________ serve as a "feedback loop" to launch more detailed financial analyses in an effort to pinpoint trouble spots and opportunities. ((operating revenues))
The single most important component of an annual report is the ((signature of a certified public accountant.))
What are the fundamentals of the accounting process? ((the accounting equation and the double-entry bookkeeping system))
Ivan got a loan from a bank to start his own house contracting business. This loan is a(n) ((liability.))
Double-entry bookkeeping is a system of recording and classifying business transactions ((in separate accounts in order to balance the accounting equation.))
Carl is using the double-entry bookkeeping system to account for all the transactions during his first month in business as an electrician. All of the following could be specific asset categories for Carl EXCEPT ((supplier credit.))
The first step in the accounting cycle is to ((examine source documents.))
In the accounting cycle, after examining source documents and recording transactions in an accounting journal, what is the next step taken by a financial manager? ((posting recorded transactions))
Evelyn, a manager at a small restaurant, is engaged in conducting the accounting cycle. She begins by examining checks, credit card receipts, sales slips, and other related evidence concerning specific transactions. Which step should she follow immediately after this? ((She must record the financial transactions in a journal.))
Peeta owns a bakery. She has already gathered and examined the checks, credit card receipts, sales slips, and other related evidence concerning specific transactions. She has also recorded each financial transaction in a journal and transferred that information into a ledger. What must Peeta do at the end of the accounting period? ((She must prepare a trial balance.))
Which statement is TRUE regarding financial statements? ((Different types of organizations use different sets of accounting principles agreed upon by the public accounting profession.))
An accounting term that is interchangeable with revenue is ((sales.))
Barat owns a large chain of Indian markets. A significant part of his job is determining the amount of income left after taxes and whether the company will retain this income or pay it out in dividends to its stockholders. The income in this scenario is ________ income. ((net))
Nonbusiness entities typically obtain revenue through ((donations.))
In the context of expense accounts, salaries of executives and their staff are included under the category of ________ expenses. ((general and administrative))
Lucy has borrowed money from a bank to start her new hair salon and spa. Her direct costs of borrowing this money is known as her ________ expenses. ((interest))
The process of spreading the costs of long-lived assets such as buildings and equipment over the total number of accounting periods in which they are expected to be used is called ((depreciation.))
Depreciation, a type of expense account, is included in the ________ category. ((general and administrative))
Reliable Construction Company bought a backhoe for $100,000, which it expects to be able to use for 20 years. Rather than showing an expense of $100,000 in the first year and no expense for the crane over the next 19 years, the company can report depreciation expenses of ________ per year in each of the next 20 years because that better matches the cost of the machine to the years it is used. (($5,000))
Colin, an accountant, allocates the cost of a piece of earth-moving equipment over a specific period of time. Based on the given information, it can be concluded that Colin is implementing the process of ((depreciation.))
Net income is ((the profit or loss after all expenses have been deducted from revenue.))
A company is trying to determine its net income. To figure out this value, the company should equate the total profit or loss minus ((expenses, including taxes.))
At the end of each accounting period, the dollar amounts in all the revenue and expense accounts are moved into an account called ((retained earnings.))
If a company decides to pay dividends to its shareholders, it will decrease ((its cash and capital accounts.))
As a financial statement, the balance sheet represents ((all transactions conducted by an organization since its founding.))
Janice is preparing a balance sheet for her pet grooming company. Following long-established traditions, she lists items on her balance sheet on the basis of their ((original cost minus accumulated depreciation.))
The traditional balance sheet format placed assets on the left side with liabilities and equities on the right, but more recently a ________ format has gained wide acceptance. ((vertical))
On a balance sheet, all asset accounts are listed in descending order of ((liquidity.))
________ refers to money owed a company by its clients or customers who have promised to pay for the products at a later date. ((Accounts receivable))
Camila manages a used car dealership that allows customers to buy cars for no money down and pay in installments throughout the year. Her company builds in a bad-debts adjustment that is deducted from the accounts receivable balance to present a more realistic view of the payments likely to be received in the future for these cars. The payments the company expects to receive are called ((net receivables.))
What represents a commitment of organizational funds of at least one year? ((fixed assets))
All of the following are items that are classified as fixed assets EXCEPT ((temporary investments.))
Which of the following is an asset in the balance sheet? ((accounts receivable))
Tyler is completing a balance sheet for his home improvement company. He must include the amounts he owes to suppliers for lumber, tools, equipment, and other materials that he purchased with credit. These liabilities are called ((accounts payable.))
An account that represents all unpaid financial obligations incurred by an organization is called ((accrued expenses.))
Which category of cash flow is calculated from changes in the long-term or fixed asset accounts? ((cash from investing activities))
How does ratio analysis relate to the income statement and balance sheet? ((It brings the income statement and balance sheet into sharper focus.))
Profitability ratios measure ((how much operating income or net income a firm is able to generate relative to its assets, equity, and sales.))
Which equation will correctly calculate profit margin? ((profit margin = net income ÷ sales))
What key performance ratio do stockholders use to determine how much money they will make on their investment? ((return on equity))
A company with a low return on assets ((is not using its assets very productively.))
Return on assets and return on equity are examples of ________ ratios. (( profitability))
The Coffee House, a local coffee shop, has a low return on equity. What does this indicate? ((Immediate managerial attention is needed.))
Asset utilization ratios measure ((how well a firm uses its assets to generate each $1 of sales.))
The ________, an asset utilization ratio, indicates how many times a firm collects its accounts receivable in one year. ((receivables turnover))
What does total asset turnover represent? ((It measures how well an organization uses all of its assets in creating sales.))
________ ratios compare current (short-term) assets to current liabilities to indicate the speed with which a company can turn its assets into cash to meet debts as they fall due. ((Liquidity))
Which of the following is a debt utilization ratio? ((times interest earned ratio))
If a company is relying on borrowing and credit too extensively, this will probably be reflected in the ________ ratio. ((debt utilization))
Maya is an investor who wants to compare the performance of one manufacturing company with another on an equal, or per share, basis. What is a general finding associated with this practice? ((The more shares of stock a company issues, the less income is available for each share.))
Businesses have the right to keep and use their profits as they choose, without limitations. ((FALSE))
Finance refers to all activities concerned with obtaining money and using it effectively ((TRUE))
Owners have little responsibility to provide funds for the operation of the business. ((FALSE))
All economic systems must address three important issues, one of which is how the goods and services will be produced. ((TRUE))
_____ is the primary goal of business. ((Profit))
Which of the following is an example of an intangible product? ((a music concert))
Sue hired an attorney to help her draft a purchase and sale agreement for her new home. The home is an example of a product that is a(n) ((tangible item.))
Which of the following is an example of a tangible product? ((an automobile))
Which of the following statements is true about businesses and profits? ((Earning profits contributes to society by creating resources that support social institutions and government.))
All of the following are true of nonprofit organizations EXCEPT they ((support businesses through donations from employees.))
Auto Depot is working to find a way to produce the SUVs and trucks its customers want while meeting environmental requirements. This is an effort to please multiple ((stakeholders.))
Among the major activities of business, which of the following is an element of management? ((acquiring resources))
Mary is the owner of a small business. She has an HR person, an operations manager, a marketing manager, and a sales manager on staff. Who is responsible for the financial decisions that arise in the course of business? ((Mary))
Which of the following is true of the responsibilities of the people involved in a business? ((The primary responsibility of providing financial resources for the operation of the business lies with the business owners.))
_____ is an element of the management activity in a business. ((Operations))
Paral has responsibility for the functions of organizing, staffing, planning, and controlling activities in a large company. These functions are most closely associated with ((managers.))
Karen, the manager of a new pet grooming company, makes sure her grooming salon always has plentiful treats and toys for the animals as well as beverages and snacks for their owners to ensure a quality customer service experience. This is an example of the _______ element of management. ((operations))
_____ gather information and conduct research to determine what customers want. They also plan and develop products and make decisions about how much to charge for their products and when and where to make them available. ((Marketers))
Which of the following can be grouped under a single segment among the various elements of business activity? ((marketing and consumers))
Advertising, personal selling, coupons, and sweepstakes are forms of the _____ aspect of marketing activities. ((promotion))
Games Galore has a new video game that it hopes will increase its sales. It is offering coupons and a sweepstake to get people to buy the game. These are all examples of ((sales promotion.))
Which of the following is NOT true of the marketing mix? ((Product management involves making sure products are available in the right place at the right time.))
When a business fails or does not make a profit, _____ have the most to lose in terms of finances. ((owners))
David is an accountant for a small company. As such, he participates primarily in _______ activities. ((financial))
_____ is the study of how resources are distributed for the production of goods and services within a social system. ((Economics))
_____ are the funds used to acquire the natural and human resources needed to provide products. ((Capital resources))
Which of the following is a factor of production for a firm? ((natural resources))
Which of the following is a natural resource? ((minerals))
Which of the following is an intangible resource? ((company reputation))
A(n) _____ describes how a particular society distributes its resources to produce goods and services. ((economic system))
Joachim lives and works in a communist country. Which of the following statements is true about a communist economy? ((It is characterized by the people (through the government) owning and operating all businesses.))
Which of the following is considered a command economy? ((communism))
Which of the following is a characteristic of communist economies in reality? ((critical shortages of consumer goods))
Which of the following has become the first communist country to make strong economic gains by adopting capitalist approaches to business? ((China))
How are profits handled in communist systems? ((Excess income goes to the government.))
Which of the following is true of product availability in a communist system? ((Consumers have a limited choice of goods and services.))
In which of the following economic systems do most people work for government-owned industries or farms? ((communist))
In a socialist economy, ((competition is restricted in major industries.))
_____ is an economic system in which the government owns and operates basic industries. ((Socialism))
Which of the following is a true statement about a socialist system of economy? ((Central planning determines what basic goods and services are produced.))
In the country of Thelisia, income and social services are distributed equally among its citizens, however, they must pay high taxes and they experience high levels of unemployment. Thelisia is an example of a(n) ______ nation. ((socialist))
In a free-market system, ((all economic decisions are made without government intervention.))
In a capitalist economic system ((prices of goods and services are determined by demand and supply.))
____ is often called the father of capitalism. ((Adam Smith))
How does modified capitalism differ from pure capitalism? ((The government intervenes and regulates business to some extent))
During a recent recession, the government of Ingrid provided loans and took ownership positions in its main bank, Savings Co. What economic system does this exemplify? ((modified capitalism))
Egalitarianism refers to the ((equal distribution of income and social services.))
Which of the following countries is an example of an economic system based on capitalism? ((Japan))
All of the following are examples of mixed economies EXCEPT ((in socialist France, the French government owns the postal service industry.))
_____ relates to the number of goods and services that consumers are willing to buy at different prices at a specific time. ((Demand))
_____ is the price at which the number of products that businesses are willing to supply equals the number of products consumers are willing to purchase at a point in time. ((Equilibrium price))
_____ is the number of products that businesses are willing to sell at different prices at a specific time. ((Supply))
Generally, when a retailer announces a discount sale on all its products ((consumers will demand more products.))
Which of the following statements describes the force that drives the distribution of resources (goods and services, labor, and money) in a free-enterprise economy? ((Prices for goods and services vary according to the changes in supply and demand.))
Which of the following markets is most likely to be a monopoly? ((electricity supply))
Using the supply and demand curves shown in the graph that follows, what is the equilibrium price for soft drinks? ((55 cents))
According to Adam Smith, competition ((fosters efficiency.))
Steve is an entrepreneur who decided to start an online bakery. Since all his business will be done online and he can bake in his home kitchen, he is saving building and other brick-and-mortar costs, which allows him to pass that savings on to his customers. Reducing prices to customers is an element of ((competition))
When different businesses promote similar products that differ in some characteristics but satisfy the same consumer need, the competitive environment is most likely to be ((monopolistic competition.))
______ is an example of a product of monopolistic competition. ((Aspirin))
Kevin owns a local farm stand. Although he sells fruit and vegetables, like the grocery store chains in the area, he has some power over the price he charges because he can offer consumers features they want. For instance, his fruit and vegetables are organic and locally grown. This is an example of ((monopolistic competition.))
Which of the following exists when there are many small businesses selling one standardized product, such as agricultural commodities like wheat, corn, and cotton? ((pure competition))
The market for corn in Brazil has a large number of sellers and buyers, and there is no difference in the products sold by each seller. As a result, prices are determined by the ((forces of supply and demand.))
When there is only one business providing a product in a given market, there exists ((a monopoly))
Which of the following is true regarding an oligopoly? ((Prices charged by different firms stay fairly close because a price cut or increase by one company will trigger a similar response from another company.))
Following a bad harvest, the prices of agricultural commodities in Urbania have increased dramatically, as have the prices of products manufactured from agricultural commodities. Government intervention has failed to arrest the rising prices. Based on this information, Urbania is undergoing ((inflation.))
Economic expansion occurs when ((an economy is growing and people are spending more money))
Rachel is living on a fixed income since her retirement. She is worried that the economy is expanding too rapidly, which could result in ((inflation))
Which of the following is true of economic expansion? ((The standard of living rises because more people are employed and have money to spend.))
Which of the following is a characteristic of economic depression? ((Business output is sharply reduced.))
Which of the following is associated with a decline in spending? ((economic contraction))
Contractions of the economy lead to ((recession.))
Increased unemployment can reduce consumer demand for goods and services, leading to ((deflation))
The country of Manyland experienced a rapid expansion of its economy, which resulted in prices doubling every 12 hours. This is an example of ((hyperinflation))
The United States has a budget deficit of over $18 trillion dollars. A budget deficit occurs when a nation ((spends more than it takes in from taxes.))
If a country balances the money it spends for social, defense, and other programs with the amount of money it takes in from taxes, this country is eliminating a ((budget deficit))
_____ measures changes in the prices of goods and services purchased for consumption by typical urban households. ((Consumer price index))
_____ indicates the income level of "average" Americans. It is useful in determining how much "average" consumers spend and how much money Americans are earning. ((Per capita income))
All of the following statements are true of the U.S. national debt EXCEPT ((in 2015, the national debt reached a new low.))
In Norway, most citizens are able to afford basic necessities and some degree of comfort. This means Norway has a(n) ((high standard of living))
Which of the following is true of an open economy, like that of the United States? ((Open economies are major players in international trade and tend to grow fast))
Joe is a consumer who earns less than $50,000 per year. Joe exemplifies the ((majority of individual tax returns filed.))
To reduce its tax rates, Mootown Burgers moved its headquarters to a country with a lower tax rate. This is an example of tax ((inversion.))
Before the Industrial Revolution, the economy of North America under the colonists was characterized by ((the domestic system of manufacturing goods))
Which of the following is a change that occurred in the United States during the Industrial Revolution? ((Farmers began to move to cities to find jobs in factories))
Who set up the first American textile factory after he memorized the plans for an English factory and emigrated to the United States? ((Samuel Slater))
In the history of the American economy, the period following the Industrial Revolution was known for an increasing emphasis on ((manufacturing.))
When the assembly line was applied to more industries, America entered the ((marketing economy.))
Which of the following is a characteristic of the marketing economy phase of the American economy? ((Expensive goods such as cars and appliances could be purchased on a time-payment plan.))
Which of the following is a characteristic of a service economy? ((It is devoted to producing services that make life easier for busy consumers))
An entrepreneur has been primarily associated with the willingness to ((take risks.))
The American economic system is best described as _____ because the government regulates business to preserve competition and protect consumers and employees. ((modified capitalism))
All of the following are true of entrepreneurs EXCEPT ((many of the companies started by early entrepreneurs have failed to stay in operation.))
Which of the following emerged in the United States as a result of industrialization? ((increase in the variety of goods available))
Which of the following is true of ethics and social responsibility in business? ((Ethical conduct and corporate social responsibility are linked to increased profits))
Several employees in a well-known corporation, CarCorp, were arrested for intentionally selling defective car parts to their customers. Scandals like this ((undermine public confidence in corporations.))
All of the following are true of workplace diversity EXCEPT it ((decreases the financial performance of companies))
Which of the following is done by the American government to counteract economic contraction? ((It may reduce interest rates))
Which of the following is useful in fighting inflation? ((raising interest rates))
All business decisions cannot be judged as right or wrong, ethical or unethical. ((FALSE))
An ethical issue is an identifiable problem, situation, or opportunity that requires a person or organization to choose from among several actions that may be evaluated as ethical or unethical. ((TRUE))
Co-workers and superiors exert significant control over your choices at work through authority and example. ((TRUE))
The concept of social responsibility is universally accepted. ((FALSE))
The principles and standards that determine acceptable conduct in business organizations are referred to as ((business ethics.))
Sarah is a manager who supports ethics and compliance at her company by clearly communicating company expectations for ethical behavior to all employees. This exemplifies ((tone at the top))
Which of the following statements is true regarding business ethics and social responsibility? ((D. Companies can be both profitable and socially responsible.))
The _____ Act criminalized securities fraud and toughened penalties for corporate fraud. ((Sarbanes-Oxley))
Which of the following laws reformed the financial industry and offered consumers protection against complex and/or deceptive financial products? ((the Dodd-Frank Act))
Recently, _______ became the number-one consumer complaint with the Federal Trade Commission ((identity theft))
One of the most difficult things for a business to restore after an ethics scandal is ((trust))
Which of the following is true of how the mass media reports on the ethical conduct of businesses? ((The mass media focuses more on misconduct than good ethical conduct))
Which of the following statements is true of business ethics? ((Ethical issues are not limited to for-profit organizations))
Which of the following is a key step in evaluating ethical decisions in business? ((learning how to recognize and resolve ethical issues))
David notices that one of his colleagues arrives late and leaves early from work every day. In deciding whether or not to report this behavior, what must he do first? ((evaluate the ethics of his choice))
_____ are payments, gifts, or special favors intended to influence the outcome of a decision. ((Bribes))
Jasmine and Kevin are competing for a promotion. Hoping to sway her manager to promote her over Kevin, Jasmine gives her manager an expensive watch. This is an example of ((bribery))
All of the following are common areas of misconduct observed in the workplace EXCEPT ((separation of personal financial interests from business dealings.))
Abusive behavior is difficult to assess and manage because of ((diversity in culture and lifestyle))
Carlos speaks fluent Spanish and English is his second language. When communicating with his colleagues, Jennifer and Kim, he uses words that are normal in his language, but they consider them to be profanity. What does this exhibit? ((Abusive behavior is difficult to assess and manage))
Within the concept of abusive behavior, _____ should be a consideration. ((intent))
_____ is the buying or selling of stocks by insiders who possess material that is still not public ((Insider trading))
According to the National Business Ethics Survey, _____ is the most common ethical problem for employees. ((abusive behavior))
All of the following are examples of misusing company resources EXCEPT ((using company phones for occasional personal calls))
When Gwen used her company credit card to buy $50,000 worth of new furniture for her house, she was fired. Which type of misconduct does this exemplify? ((misuse of company resources))
Which of the following has developed a Corruption Perceptions Index? ((Transparency International))
Which of the following is true of misuse of company time? ((Time theft costs can be difficult to measure but are estimated to cost companies hundreds of billions of dollars annually.))
Which of the following countries is included in the list of least corrupt countries? ((Denmark))
Laura, a manager, makes decisions that benefit her financially at the expense of her firm. Which of the following ethical issues in business is addressed in this example? ((conflict of interest))
Which of the following is associated with a hostile workplace when a person or group is targeted and is threatened, harassed, belittled, verbally abused, or overly criticized? ((bullying))
Which of the following is true of bullying? ((Although sexual harassment has legal recourse, bullying has little legal recourse at this time.))
The fact that businesspersons are expected not to harm customers, clients, and competitors knowingly through deception, misrepresentation, coercion, or discrimination is part of the practice of ((fairness and honesty))
The European Commission investigated Google to determine whether it promoted its own search results over those of other search engines in spite of their relevance. The aspect of fairness at issue here is ((competition))
All of the following are associated with dishonesty EXCEPT ((competition))
In the realm of business ethics, when automobile companies fail to issue recalls in a timely manner, this is an issue related to ((communications))
People in the entertainment industry claim that requiring warning labels on movies and videogames violates their ________ right. ((First Amendment))
The National Business Ethics Survey found that employees who feel pressured to compromise ethical standards view _____ as the greatest source of such pressure ((top and middle managers))
Managers are responsible for ((creating a work environment that helps the organization achieve its objectives and fulfill its responsibilities.))
The warning on cigarette packages about the health implications of smoking is an example of ((communications))
Which of the following behaviors is an example of ethical consideration within the purview of business relationships? ((keeping company secrets))
Which of the following indicates that a person has begun the process of resolving an ethical issue? ((recognizing the ethical issue and openly discussing it with others))
Which of the following is NOT a question to consider in determining whether an action is ethical? ((Is this activity customary across all industries in your country?))
Ethical decisions in an organization are influenced by individual moral standards, the influence of managers and coworkers, and ((the opportunity to engage in misconduct.))
Which of the following is true of professional codes of ethics? ((They do not have to be too detailed))
All of the following are true of the causes of ethical conflict EXCEPT ((Many employees utilize the same ethical standards at work and at home.))
A set of formalized rules and standards that describes what a company expects of its employees is called a ((code of ethics))
Codes of ethics, policies on ethics, and ethics training programs advance ethical behavior because they ((prescribe which activities are acceptable and which are not))
According to the National Business Ethics Survey, employees in organizations that have written codes of conduct and ethics training, ethics offices or hotlines, and systems for reporting are more likely to ((report misconduct when they observe it))
Codes of ethics foster ethical behavior by ((limiting the opportunity to behave unethically by providing punishments for violations of the rules and standards.))
A large number of _____ cases result in retaliation against the employee, even though the government has tried to take steps to protect workers and to encourage reporting of misconduct. ((whistleblowing))
Which of the following is a reason why a code of ethics is important? ((It alerts employees about important issues and risks to address.))
Which of the following is true of the Dodd-Frank Act passed by the U.S. Congress in 2010? ((It encourages whistleblowers to provide information about corporate misconduct through monetary rewards.))
All of the following are true of current trends in ethics programs EXCEPT ((organizations are moving toward legally based ethical initiatives.))
Which of the following is one of the most common factors that executives give for why turnover increases? ((a lack of transparency among company leaders))
Being profitable relates to the _____ dimension of social responsibility. ((economic))
_____ is the extent to which businesses meet the legal, ethical, economic, and voluntary responsibilities placed on them by their various stakeholders. ((Corporate citizenship))
Which of the following involves the activities and organizational processes adopted by businesses to meet their social responsibilities? ((corporate citiWhich of the following is an example of corporate citizenship? zenship))
Which of the following is an example of corporate citizenship? ((engaging in organic farming))
Which of the following is a dimension of social responsibility? ((ethical))
Being a "good corporate citizen" is an example of the _____ dimension of social responsibility. ((voluntary))
Philanthropic contributions made by a business to a charitable organization represent the _____ dimension of social responsibility. ((voluntary))
Which of the following is an argument that supports social responsibility? ((As members of society, businesses and their employees should support society through taxes and contributions to social causes.))
Studies have found a direct relationship between social responsibility and _____ in business. ((profitability))
Businesses must first be responsible to their_____, who are primarily concerned with earning a profit. ((owners))
_____ is defined as the activities that individuals, groups, and organizations undertake to protect their rights as customers. ((Consumerism))
Which of the following is a major concern of any firm trying to increase profits? ((Lack of employee commitment))
Laws regarding workplace safety are enforced by the ((Occupational Safety and Health Administration))
The right to _____ means that a business must not knowingly sell anything that could result in personal injury or harm to consumers. ((safety))
The right to choose ensures that ((consumers have access to a variety of goods and services at competitive prices))
The right to _____ assures consumers that their interests will receive full and sympathetic consideration when the government formulates policy. ((be heard))
A major social responsibility for businesses is providing _____ for all employees ((equal opportunities))
The right to _____ gives consumers the freedom to review all details about the products they wish to buy. ((be informed))
Employees expect businesses to ((keep them informed of what is happening in the company))
All of the following are items companies are offering or considering to improve employee relations EXCEPT ((elimination of all discrimination))
_____ involves the interaction among nature and individuals, organizations, and business strategies and includes the assessment and improvement of business strategies, economic sectors, work practices, technologies, and lifestyles, so that they maintain the health of the natural environment. ((Sustainability))
Pollution of water and soil from activities such as oil and gas drilling is primarily related to ((sustainability issues))
All of the following are ways businesses are working to resolve environmental concerns EXCEPT ((creating a positive association for unsuitable products, services, or practices))
The fact that environmental responsibility requires trade-offs means that it ((imposes costs on both business and the public))
The most common way that businesses exercise community responsibility is through ((donations to charitable organizations))
_____ refers to attaching a positive environmental association on an unsuitable product, service, or practice. ((Greenwashing))
Many businesses, recognizing that employees lack basic work skills, are becoming more concerned about the quality of _____ in the United States. ((education))
_____ emerged as a major issue in the 20th century in the face of increasing evidence that pollution, uncontrolled use of natural resources, and population growth were putting increasing pressure on the long-term sustainability of these resources. ((Environmental protection))
_____ has become a significant problem since the onset of the financial crisis in 2008. ((Unemployment))
A survey of employers conducted in Indiana revealed that _____ percent reported leaving positions unfilled because the applicants were not qualified ((39))
Which of the following organizations funds programs to train the hard-core unemployed so that they can find jobs and support themselves? ((National Alliance of Businessmen))
A sole proprietor has limited liability in meeting the debts of his or her business. ((FALSE))
Limited partners do not participate in the management of the business. ((TRUE))
The assets and liabilities of a corporation are separate from its owners. ((TRUE))
An S corporation is taxed just like any other corporation. ((FALSE))
An acquisition occurs when one company purchases another company by buying most of its stock. ((TRUE))
Most tender offers of corporate raiders are hostile. ((FALSE))
Which of the following forms of business ownership is the easiest to establish? ((sole proprietorships))
Which of the following is the most common form of business organization in the United States? ((sole proprietorships))
Matt is an independent contractor who trailers horses for several local stables, however, he is not an employee of any of the stables. Matt is a(n) ((sole proprietor))
Which of the following statements is true about sole proprietorships? ((Most sole proprietorships focus on services rather than on the manufacture of goods))
Which of the following is an advantage of a sole proprietorship? ((It is easy and inexpensive to form.))
Catalina owns a small restaurant and is solely responsible for its debts and liabilities. She is also entitled to all the profits the restaurant makes after paying off taxes and other expenses. Her business is an example of a(n) ((sole proprietorship))
Sole proprietorships are typically small businesses that ((employ fewer than 50 people))
All of the following are ways technology has helped sole proprietors succeed EXCEPT ((landline telephones allow them to make free long-distance calls to customers, suppliers, and others.))
The income earned in sole proprietorships is ((taxed as personal income))
Raj wants to open his own pub in his hometown that serves local spirits and food. Which of the following will he have to do to achieve this goal? ((He will need to fill out paperwork to obtain the proper licenses and permits))
Brenda runs a bookstore in her local community. She started the business with her own money and solely manages the day-to-day operations of the business. Which of the following is true in this scenario? ((Brenda can easily dissolve the business whenever she wants.))
All of the following statements are true of the government regulation of sole proprietorships EXCEPT ((sole proprietors must follow all laws, including federal, state, and local regulations; securities laws; and employee and consumer protection regulation.))
In sole proprietorships, owners ((have access to limited sources of funds))
Which of the following is a disadvantage of sole proprietorships? ((lack of continuity))
Which of the following forms of business ownership is directly linked to the life of its owner? ((a sole proprietorship))
Which of the following forms of business ownership requires owners to perform many functions and possess diverse skills to make decisions? ((a sole proprietorship))
Salma owns a pharmacy near a hospital. She started the business with a personal loan and is wholly responsible for repaying it. Salma also needs to pay personal income tax on the profits generated from the business. Her business is an example of a(n) ((sole proprietorship))
_____ can be considered as both an advantage and a disadvantage of a sole proprietorship. ((Taxation))
Which of the following statements is true regarding the sources of funds available to a sole proprietor? ((Often, sole proprietors must pledge a car, a house, other real estate, or other personal assets to guarantee a loan))
When Dr. Taylor, a local dentist, dies, his children sell his dental office to a young, new dentist who just moved to the area. What challenge will this new dentist face in initially taking over the business? ((assuring customers that the business will continue to meet their needs))
Which of the following is a reason it is difficult for sole proprietorships to hire qualified employees? ((It is difficult for sole proprietorships to match the wages and benefits offered by large corporations.))
Which form of business ownership is least used in the United States? ((partnerships))
A _____ has been defined as "an association of two or more persons who carry on as co-owners of a business for profit." ((partnership))
All of the following are keys to a successful partnership EXCEPT partners must ((keep profit sharing equal regardless of contributions))
A ______ involves a complete sharing in the management of a business. ((general partnership))
At AgroCorp, each owner has unlimited liability for the debts of the business. It has several business units that focus on many product categories at once, which allowed the company to become a leader in category management in the agricultural market. AgroCorp is an example of a ((general partnership.))
A ______ has at least one general partner, who assumes unlimited liability, and at least one limited partner, whose liability is limited to his or her investment in the business. ((limited partnership))
When are limited partnerships generally used? ((when a project is risky and the chance of loss is great))
OilSource, a national oil provider, is a partnership that is traded on securities exchanges. This allows it to have the tax benefits of a limited partnership but the liquidity of a corporation. OilSource is an example of a(n) ((master limited partnership))
The legal documents that identify the basic agreements between partners are called ((articles of partnership))
Articles of partnership usually cover issues, including ((provisions for leaving the partnership))
Kim, Diana, and Courtney are starting their own after-school tutoring business, and they are drafting articles of partnership. All of the following are issues they should consider EXCEPT ((how to manage customer information))
It is easier to raise funds in partnerships than in sole proprietorships because ((several partners mean greater earning power and credit))
The decision-making process in a partnership tends to be faster when ((there are two partners and both of them are involved in the day-to-day activities of the business.))
Which of the following statements is true of a partnership? ((The business will face fewer regulatory controls than a corporation))
Michael Wong and Sanjay Patel, both lawyers, start a law firm together. Both of them have unlimited liability for the debts of the business, and the management of the firm is equally shared between them. This law firm is an example of a(n) ((general partnership))
Which of the following is an advantage of a partnership? ((few regulatory controls))
Insurance, Inc. is a large insurance company with partners that specialize in home, auto, and business insurance. Since each partner specializes in a different area of insurance, customers would be likely to ((view the service as higher quality than that provided by one person))
Which of the following is a disadvantage of a partnership? ((difficulty of selling the partnership interest))
Which of the following statements is true of taxation in partnerships? ((They pay taxes at the income tax rate for individuals))
In a limited partnership ((at least one partner has unlimited liability))
Partnerships are quasi-taxable organizations. This means that they ((do not pay taxes when submitting the partnership tax return to the Internal Revenue Service.))
Which of the following is true of the distribution of profits earned by a partnership? ((They are distributed in the proportions specified in the articles of partnership))
Which of the following is an advantage of partnerships over sole proprietorships? ((In partnerships, the owners have access to more funds than in sole proprietorships))
All of the following are areas that can cause friction in a partnership EXCEPT ((the diverse skills of the partners))
Which of the following is a disadvantage of partnerships over sole proprietorships? ((In partnerships, profits have to be shared, whereas in sole proprietorships all profits belong exclusively to the owner.))
Maria wants to start a new business in collaboration with her husband and brother-in-law. The features she is looking for in the business are as follows: least possible regulatory controls, faster decision making, and ease of organization. She also wants a business in which she will not be forced to pay off any business debts from her personal assets. Which type of business ownership will be best suited to her needs? ((limited partnership))
Xavier starts an investment management firm along with his friend Abdul. They decide to equally share the profits and have unlimited liability for the debts of their business. Such unlimited liability can be a distinct disadvantage for Xavier if he has ((more financial resources than Abdul.))
All of the following are ways small businesses can maintain low costs EXCEPT ((hiring and maintaining a large finance department))
Red Carpet Inc. is a small apparel store started by an aspiring designer. The store needs to compete against larger, well-established multinational brands. Which of the following strategies will most help Red Carpet Inc. avoid competition from larger firms? ((Red Carpet Inc. should focus on and target small market niches or product needs.))
Kristen is an animal lover and vegetarian. Frustrated by trying to find vegetarian substitutes for commonly needed items, she has started her own vegetarian boutique. Her store sells vegetarian food, shoes, toiletries, and many other items. Which advantage of small-business ownership is this scenario most closely related to? ((focus))
Which of the following is true regarding the reputation of small businesses? ((Because of their capacity to focus on narrow niches, small businesses can develop enviable reputations for quality and service.))
Which of the following is one of the difficulties faced by small business owners? ((worries about employee problems or competition))
Matt owns his own small bakery. In addition to being the owner, he also serves as the manager, sales force, head baker, shipping and receiving clerk, bookkeeper, and custodian. Matt is the first person to arrive at the bakery every morning and the last to leave every night. Which of the following is the most applicable challenge Matt faces? ((a high stress level))
If a small business has to cope with growth, it requires the owner to ((give up a certain amount of direct authority))
Which of the following is a reason behind the failure of small businesses? ((poor business concept))
Colin is an avid scrapbooker. He wants to start a scrapbooking store and workshop, where people can buy their supplies and scrapbook together as a community. What must he do to make sure this business will succeed? ((He must identify whether a genuine market niche exists for scrapbooking.))
Carol has decided to open a Vietnamese restaurant in her neighborhood. She will be solely responsible for the management of the restaurant. Which of the following problems is she likely to face while running the business? ((She may find it difficult to deal with the stress of rent increase))
_____ is best described as the lack of funds to operate a business normally. ((Undercapitalization))
Mark and Stephanie run a small farm stand on their property, selling vegetables, fruits, plants, and eggs to support themselves. During a particularly bad season, they tried to get financing from a small rural bank in their community. However, they were unable to obtain financing because ((the small rural bank lacked the necessary financial expertise to counter the risks involved with small-business loans.))
Jason is an artist who wants to turn his art into a small business. He had the great idea to go to other hobby-based businesses in his community and give painting lessons to their clients. First, he went to a local stable and taught its boarders how to paint portraits of their horses. Then, he went to a local dance studio and taught its patrons how to paint their dream dance costumes. Based on the buzz created by his new business, several other local businesses want to hire Jason. What is one area where he may struggle in managing his new business? ((He may lack the skills and experience to make good management decisions))
Gerald started a business using the savings from his previous job. He planned to run his business on the revenue generated from sales. However, a few months later, he found it difficult to pay his staff, rent, and other expenses. Seasonal sales and inability to secure sufficient credit from local banks made it difficult for Gerald to operate the business normally. Which of the following causes of small-business failure does this scenario best illustrate? ((undercapitalization))
Which of the following is the shortest path to failure in business? ((undercapitalization))
Penelope is a very talented software engineer who starts her own small software development firm. She finances the firm from her savings. Initially, she is able to get many projects from customers and completes them successfully. However, as her business grows, she fails to make good decisions in hiring new employees, billing the clients, and negotiating prices. She eventually ends up liquidating the business. Which of the following causes of small-business failure does this scenario best illustrate? ((managerial inexperience))
The principal immediate threats to small and mid-sized businesses include ((escalating costs))
Tiffany recently started a small firm that contract manufactures for leading fashion brands in the United States. Which of the following would likely be a principal immediate threat to this firm? ((rising inflation))
It has often been said that the greatest impediment to the success of a small business is the ((entrepreneur))
Meredith wants to start her own small business. All of the following are things she needs to do to start a business EXCEPT ((hire employees to work for the business.))
A precise statement of the rationale for a business and a step-by-step explanation of how it will achieve its goals is known as a(n) ((business plan))
Which of the following statements is true of a business plan? ((It should establish a strategy for acquiring sufficient funds to keep a business going.))
Arnold approached a bank to get a loan for a business he planned to launch later that year. He explained his business idea and its capital requirements to the bank manager. The manager asked Arnold to provide a formal document that included a rationale for the business, an explanation of how it will achieve its goals, an analysis of the competition, and estimates of income and expenses, among other information. Which of the following documents does the bank expect from Arnold, in this scenario? ((a business plan))
In the process of starting a business, the step that immediately follows the development of a business plan involves ((deciding an appropriate legal form of business ownership))
To make profits from a small business, the owner must first provide or obtain _____ to start the business and keep it running smoothly. ((capital))
Stanley has started a new zip line business in Jamaica. What is the most important source of funds for this new business? ((Stanley, the owner))
Which of the following is a source of equity financing? ((selling personal assets to raise funds))
Walter sold a piece of land he inherited from his grandparents and started a business with the proceeds. Which of the following sources has Walter used to raise funds for his business? ((equity financing))
Matt wants to start a small moving company. He has enough funds to start the business, but not enough to run it effectively. What is one thing he can do to provide working capital to his company? ((He can refrain from drawing a full salary))
Derek borrowed $50,000 from his mother for his new business venture. He promised that she would be a partner in the business and would be entitled to a significant percent of profits from the business at the end of the year. This is an example of ((equity financing))
Janice works in a factory that manufactures decorative accessories for office spaces, hotels, and designer homes. She wants to start her own business because she believes that the market for decorative products has a great growth potential. However, she does not want to depend on anyone else for procuring the initial capital and would like to raise the amount herself. In this scenario, which of the following methods could Janice adopt to raise funds through equity financing? ((Janice could invest her savings in the new business))
Jacob and Harry are business partners, and their company manufactures portable solar panels. They initially started the business with their savings. However, now the company plans to expand its operations and the required amount of capital cannot be raised through savings or by reinvesting profits. Thus, the partners have decided to sell stock in their business to family members, friends, and employees. Which of the following sources of capital have Jacob and Harry planned to use for the expansion of their business? ((equity financing))
Persons or organizations that agree to provide some funds for a new business in exchange for ownership interest or stock are called ((venture capitalists))
Ramon funds small businesses that he believes have the potential to grow large. When these companies are still in their initial stages and need investment, he buys their stocks at a low price and later sells them at higher prices when they are successful. Thus, Ramon is a(n) ((venture capitalist))
Obtaining money from venture capitalists to start a new business venture is an example of ((equity financing))
What is the main supplier of external financing to small businesses? ((banks))
Securing a mortgage from a bank for a new business venture is an example of ((debt financing))
Jordan has requested financing from the bank for his small automobile parts store. The bank requires Jordan to put up a financial interest in the property or fixtures of the business to guarantee payment of the debt. What is this financial interest called? ((collateral))
If a small business fails to repay a bank loan, what will the lending institution do to recover its loss? ((claim and sell the collateral or mortgage))
Banks and other financial institutions can grant a small business a _____, which is an agreement by which a financial institution promises to lend a business a predetermined sum on demand. ((line of credit))
Small businesses may obtain funding from their suppliers in the form of a _____, which means that suppliers allow the business to take possession of the needed goods and services and pay for them at a later date or in installments. ((trade credit))
Anisha, an entrepreneur, starts a real estate firm with very limited funds. In order to conduct business, she purchases a few laptops, LED screens, and other office equipment from a local electronics wholesaler. The wholesaler understands her financial situation and allows her to pay for the goods in installments. In this scenario, the type of funding obtained by Anisha can be regarded as a ((trade credit))
Neehara recently opened a premium bakery. She did not have enough savings and hence she applied for a loan to obtain the additional funds required to start the bakery. She had to provide her house as collateral to obtain the loan. The type of financing obtained by Neehara can be regarded as a ((mortgage))
Which of the following statements best describes bartering? ((It is the practice of small businesses trading their own products for the goods and services offered by other businesses.))
Lara, a single mother, borrowed some capital for her business from Women Progress Council at an extremely low rate of interest. Which of the following forms of funding a new small business does this scenario best illustrate? ((debt financing))
Andrew, a pharmacist, realized that he should stock extra supplies of a particular medicine to respond to an outbreak of a particular infection in the city. Lacking sufficient capital to purchase the extra inventory, he borrowed a predetermined sum of money from his bank. Which of the following sources of funding does this scenario best illustrate? ((line of credit))
Nathan started the first outlet of Dynamix Gym in New York City in 1995. The business expanded over time, and he became the owner of a chain of gyms in New York State. Though the business had potential to expand outside New York, Nathan did not want to take the risk or responsibility. Hence, he decided to sell the license to own and operate gyms under the brand name Dynamix Gym to independent owners. In return, he charged them an initial fee and a small share in the monthly profits. In this scenario, Nathan is a(n) ((franchiser.))
Which of the following statements is true of franchising? ((A franchisee often receives building specifications and designs from the franchiser))
Which of the following statements describes an advantage of franchising instead of building an independent business from scratch? ((A franchise outlet often reaches the break-even point faster than an independent business would.))
All of the following are advantages of franchising EXCEPT ((more freedom in purchasing goods.))
Stephanie has always dreamed of owning and managing a hotel. Instead of starting her own independent hotel, she decides to buy a franchise from the Marriott. What is one thing she should keep in mind about becoming a franchisee? ((She will need to buy equipment, pay for training, and obtain a mortgage or lease))
Which of the following is one of the commonly reported disadvantages of franchising? ((restrictions on purchasing))
Steven opened his own Subway franchise in his community. He is pleased with how quickly he was able to open his franchise and that he was able to establish a good customer base from the start because of name recognition. However, since he started this business to be his own boss, he is likely to ((get frustrated with the restrictions of the franchise))
_____ is a volunteer agency funded by the Small Business Administration to provide advice for owners of small firms. ((Service Corps of Retired Executives))
Chris has started his own small photography business. All of the following are programs and resources he can use to improve his ability to compete with other businesses EXCEPT ((classified files from his competitors.))
Samantha owns a small office supply company. In setting up her business, she asks a few other small-business owners she knows for advice on dealing with employees and government regulation, improving processes, and solving problems. What does this scenario exemplify? ((networking))
Which of the following statements is true of baby boomers? ((They are not actively pursued by small businesses.))
Millennials number around 83 million and represent a huge business opportunity in the United States. Which of the following is a defining feature of the millennials? ((This cohort is concerned with advancement, recognition, and improved capabilities))
Margaret was born in 1952. She is a wealthy Florida native. As she ages, Margaret spends more of her money on travel, financial planning, and health care. Margaret is a(n) ((baby boomer))
Which of the following demographic segments should both small retailers and small-service providers specializing in ethnic products target? ((immigrants))
Marco is a Latino who lives in Boston, Massachusetts. Over the years, he has noticed that more stores in his community specialize in ethnic products. This has made Boston feel more like home to him. What trend might explain this recent influx in stores that cater to the Latino population? ((In the United States, the Latino population is expected to grow by 86 percent between 2015 and 2050.))
Which of the following demographic trends has made health care and financial planning industries attractive for small businesses in the United States? ((increasing aged baby boomers who are wealthy))
Which of the following is an implication of technological advances to small businesses? ((It provides new opportunities for small businesses to expand their operations abroad.))
Which of the following is true of home-based businesses? ((Approximately 52 percent of all small businesses are based out of the home.))
Kevin makes rustic wood furniture and home accessories. During a recent time of economic turbulence, when well-funded dot-coms were failing, Kevin learned how to ((use the Internet to promote himself and sell his products online))
All of the following have provided new opportunities for small businesses EXCEPT ((the success of large information technology companies))
Downsizing is an effective way to ((gain the advantages of small businesses.))
Paula works for a large company that is trying to function like a smaller company. Which of the following is something the company can do to achieve this goal? ((The company can make its operating units function more like independent small businesses.))
Which of the following statements accurately describes intrapreneurs? ((Individuals in large firms who take responsibility for the development of innovations within the organizations are intrapreneurs.))
Jason works for a large sporting goods corporation. He champions a new hockey helmet with advanced concussion protection. Jason is a(n) ((intrapreneur))
George is an employee at a company that provides information technology solutions to other firms. Recognizing his potential to innovate, the top management of the company has given him the complete liberty to develop a new smart phone application using the resources of the company. In this scenario, George best illustrates a(n) ((intrapreneur))
Management takes place only in business settings. ((FALSE))
For any company, owners and shareholders are the only sources of primary funding. ((FALSE))
Planning involves forecasting events and determining the best course of action from a set of options or choices. ((TRUE))
The first step in the decision-making process is developing options directly ((FALSE))
Gwendolyn manages a resort boutique. Her goal is to run this business in such a way that she achieves the desired result, making a profit and providing her customers with the items they need to enjoy their vacation. Thus, her primary managerial concern is ((effectiveness))
If a manager is concerned about doing the work with the least cost and waste possible, then his or her primary managerial concern is ((efficiency))
Hiring people to carry out the work of an organization is known as ((staffing))
Nathan is a manager at a company that has made the difficult decision to downsize. After his company eliminated 100 employees, what must Nathan do to effectively manage his remaining employees and help them through the situation? ((promote optimism and positive thinking))
The Furniture Warehouse, a large furniture company, wants to make sure its products are always available to customers. Since this company sells many products, including beds, patio sets, dining room tables and chairs, couches and recliners, tables, dressers, rugs, and decorative accents, it will need many diverse _____ to maximize efficiencies and provide creative solutions to help the company reduce expenses and reach its objectives. ((suppliers))
Managers need adequate _____ resources to pay for essential activities ((financial))
To earn a profit, managers must acquire, coordinate, and control all of the following resources EXCEPT ((competitive products))
Melanie is one of the senior managers in the consumer electronics department of a large company. She is part of a team that determines the objectives of the department based on future trends in the industry and decides how to accomplish them. Melanie is involved in the management function of ((planning))
Which of the following is the first function of management? ((planning))
The plan itself specifies all of the following EXCEPT ((why the tasks should be completed.))
Before an organization can plan a course of action, it must first ((determine what it wants to achieve.))
Carrie started her own catering business. She created a website for her business, which states that her purpose is to provide her customers with delicious food at a fair price. This declares her ((mission))
Which of the following statements is true of a mission statement? ((A well-developed mission statement, no matter what the industry or size of business, will answer five basic questions.))
Which of the following questions does the mission statement of an organization seek to answer? ((What business are we in?))
Which of these basic questions will be answered by a well-developed mission statement? ((Who are our customers?))
Jamal owns a surf shop in Hawaii. His business is doing well, catering to local surfers and tourists who want to learn to surf. He has developed a strong organizational culture, a good marketing plan, and a coherent business strategy. What serves as the foundation for developing these strengths? ((a mission statement that delivers a clear answer to the five basic questions))
A(n) _____ is best described as the result that a firm wishes to achieve. ((goal))
In the context of company goals, which of the following is an effective practice? ((Company goals should be specific.))
Which of the following statements is true of the goals of a company? ((A goal has three key components: an attribute sought, a target to be achieved, and a time frame in which the goal is to be achieved.))
The principal difference between objectives and goals is that ((objectives are measurable, while goals need not be measurable.))
Which of the following objectives are generally stated in terms of percentage of sales increase and market share, with the goal of increasing those figures? ((objectives regarding competitive advantage))
Growth objectives essentially relate to ((adapting and releasing new products in the market.))
Sugar and Spice, a large candy store, has historically sold a lot of chocolate and gummy candies. However, over the years, people have become more health conscious. As a result, Sugar and Spice developed a new line of sugar-free candies that taste as good as the original, but without the guilt. What type of objective does this exemplify? ((growth))
Which of the following provides direction for all managerial decisions and establishes criteria for evaluating performance within organizations? ((objectives))
_____, a type of plan for meeting objectives, establish the long-range objectives and the overall course of action by which a firm fulfills its mission. ((Strategic plans))
How long a time period do strategic plans generally cover? ((one year or longer))
The top management of New 18 Apparels Inc. has decided to set up an online store to cater to the changing business environment and reach out to the Asian market. The management has also identified that the success of the company will depend on its ability to create and develop new products and processes in the next two years. This is an example of ((strategic planning))
All of the following are true of strategic plans EXCEPT they ((can be developed by anyone in a firm.))
Tactical plans usually cover a period of ((one year or less.))
Which of the following is a difference between strategic and tactical plans? ((Strategic plans are long term, while tactical plans are short term))
Which of the following statements is true of tactical plans? ((They allow a firm to react to changes in the environment.))
Floors and Fixtures, a home improvement store, is planning to expand and open four new stores, one each year. As a result, it develops four separate plans to determine how much to spend on each store as well as when and where to open each store. These are _____ plans. ((tactical))
Happy Trails, a trail mix store, is planning to expand its business by opening several new stores over the course of four years. Within this strategic plan, all of the following are items operational plans may specify EXCEPT the schedule for ((opening all the stores))
Which of the following is an example of an operational plan? ((A work group at a large computer plant is assigned a weekly production quota to ensure there are sufficient products available to elevate market share and ultimately help the firm be number one in its product category.))
_____ is an element in planning that deals with potential disasters such as product tampering, oil spills, fire, earthquake, computer viruses, or a reputation crisis. ((Contingency planning))
Bigbag.com, a leading online retailer, uses the services of Data Safe Inc., a small startup that helps companies manage and keep their electronic data safe. This helps Bigbag.com keep a backup of its proprietary information in case of virus attacks or any sort of damage to property. This is an example of ((contingency planning.))
Which of the following is true regarding crisis management? ((Many companies have crisis management teams to deal specifically with problems))
Dividing work into small units and assigning it to specific individuals is a task related to the management function of  ((organizing))
Which of the following statements is true of the management function of organizing? ((Organizing helps create synergy, whereby the effect of a whole system equals more than that of its parts.))
All of the following are reasons organizing is important EXCEPT it ((eliminates synergy.))
Martin owns a large moving company. Over the years, the market has changed, and Martin has had to adapt how he structures his resources and activities to continue to accomplish his business objectives efficiently and effectively. What does this scenario demonstrate? ((organizing occurs continuously))
Providing incentives to employees to motivate them toward achieving organizational objectives relates to the management function of ((directing.))
Which of the following is true of the management function of directing? ((Directing involves determining and administering appropriate rewards and recognition))
Bernadette manages workflow at a newspaper. She sets and implements the deadlines and encourages the writers and editors to do their work. What managerial function is Bernadette fulfilling? ((directing))
Christopher manages a fitness club, which hires many younger, college-aged employees. He has tried several incentive plans to motivate them to work harder, including promising raises, promotions, and bonuses. However, he has found that his employees need further motivation. He set up a meeting with a few of his employees and they explained to him that they want more than money from their jobs. Christopher can do all of the following to motivate his employees EXCEPT ((shielding his employees from having to make decisions about how the company operates.))
_____ is the process of evaluating and correcting activities to keep an organization on course. ((Controlling))
When Betsy informed her supervisor that the furniture assemblers at their manufacturing plant were using 20 percent more raw materials in the current month when compared to the preceding months, she was involved in the management function of ((controlling))
For an organization, the first step in the management function of controlling is to ((measure the actual performance))
Jill manages a fast food restaurant. One day, while sitting in the restaurant taking her break, she hears a group of customers complaining about how slow the service has been getting and that they may have to stop eating there. Which of the following is the most effective control action Jill can take to resolve the problem? ((She can watch her employees work to determine the cause of the problem, and then respond appropriately.))
In businesses, _____ include the president and other top executives, such as the chief executive officer, chief financial officer, and chief operations officer, who have overall responsibility for an organization. ((top managers))
In an organization, decisions regarding adding new products, acquiring companies, and moving into foreign markets would most typically be made by ((the top management.))
Dawn manages a small coffee house. In fact, this business is so small that Dawn is the only manager. What level manager is Dawn? ((Dawn assumes the responsibilities of all three levels.))
Which of the following is true of top management compensation? ((Many firms are trying to align CEO compensation with performance.))
Claudia is a manager at a large technology company. Since her company wants to serve consumers from fast-growing demographic groups such as Hispanics, African Americans, and Asian Americans, she wants to hire employees from those groups to help make decisions regarding issues related to consumer diversity. All of the following are rules she should follow in diversity recruiting EXCEPT ((limit hiring to individuals from the chosen demographic groups.))
_____ are responsible for tactical and operational planning that will implement the general guidelines established by top management. ((Middle managers))
The specific operations of an organization, plant, division, or department would most typically involve the _____ of the company. ((middle management))
Which of the following has been a recent trend with regard to the different levels of management within an organization? ((The ranks of middle managers have been shrinking as more and more companies downsize to be more productive.))
Stella manages the computer department at Mega Tech, Inc. She helps the organization implement its general guidelines by managing the specific operations related to her department. Stella is a ((middle manager.))
Positions such as foremen, supervisors, and office service managers are a part of the _____ of a company. ((first-line management))
Most people get their initial managerial experience as _____, those who supervise workers and the daily operations of an organization. ((first-line managers))
Brian is a manager at a clothing store. He spends most of his time in the store with his employees, making sure they work their scheduled hours, watching them interact with customers, and making sure all the daily tasks are completed sufficiently. Brian is a ((first-line manager.))
Janet works as a manager at Unicorn Infra Inc. Janet and her team members are primarily responsible for procuring the funds needed for the successful operation of the organization and investing that money to pursue organizational goals. Thus, Janet is a ((financial manager))
George is a bank manager. He manages the activities of an entire branch without specializing in any one particular function. Thus, George is a(n) ((administrative manager))
Production and operations managers are concerned with ((transforming resources into goods and services.))
Corbin is a marketing manager for a large textbook publisher. What is his primary responsibility? ((planning, pricing, and promoting products and overseeing their distribution))
Marah is a manager at a law firm. Part of her role is conducting performance reviews for the paralegals in the office. By doing so, she is fulfilling the managerial role of a ((leader))
_____ refer to the ability of an individual to think in abstract terms and to see how parts fit together to form the whole ((Conceptual skills))
Which of the following is a feature of democratic leadership? ((encouraging employees to discuss concerns and provide inputs))
The _____ style of leadership can be a powerful motivator because it demonstrates a great deal of trust and confidence in an employee and allows employees to meet their work demands with little or no interference. ((free-rein))
Which of the following is true regarding choosing the appropriate style of leadership for a company? ((An autocratic style of leadership is best for stimulating unskilled, unmotivated employees.))
Which of the following statements is true of authentic leaders? ((Authentic leaders are identified by the way in which they conduct themselves with stakeholders.))
Which of the following statements is true of employee empowerment? ((Employee empowerment does not mean that managers are not needed.))
_____ occurs when employees are provided with the ability to take on responsibilities and make decisions about their jobs. ((Employee empowerment))
Clink, a new bar and restaurant, encourages its employees and managers to participate in creating its specialty drinks and menu items. This is an example of ((participatory decision making.))
Which of the following is the first step in decision making? ((recognizing and defining the decision situation))
Managers at Sprocket Inc. have recognized declining sales on their water purifier and must decide what has to be done to improve it. Which of the following would be their next step? ((defining the situation))
When selecting the best option among a series of possible decisions ((it may be possible to use a combination of several options.))
When analyzing options in the decision-making process, managers must consider the appropriateness and _____ of each option. ((practicality))
Which of the following best describes the importance of monitoring the consequences of decisions? ((The consequences of decisions may not be apparent quickly enough without monitoring.))
A(n) _____ is like a calendar, containing both specific and vague items, that helps a manager figure out what must be done and how to get it done to meet the objectives set by an organization. ((agenda))
Which of the following refers to the process of building relationships and sharing information with colleagues who can help a manager achieve the items on his or her agenda? ((networking))
_____ is used for job networking and is gaining in popularity among the younger generation as an alternative to traditional job hunting. ((LinkedIn))
Karen manages a large carpet cleaning company. She spends a lot of time communicating with a variety of people and participating in activities that do not seem to have much to do with goals of her company. What is this called? ((networking))
All of the following are challenges managers confront in the business world today EXCEPT ((the stagnant nature of the workforce))
Every organization has an organizational culture, regardless of size, organizational type, product, or profit objective. ((TRUE))
The more complex organizations become, the less they need to develop formal structures to function efficiently. ((FALSE))
A wide span of management works best when a manager and subordinates are not in close proximity. ((FALSE))
Multidivisional structure is the simplest organizational structure that is based on direct lines of authority extending from the top executive to the lowest level employees of an organization. ((FALSE))
Horizontal communication involves the traditional flow of information from upper organizational levels downward. ((FALSE))
Which of the following is true of organizational culture? ((Organizational culture is more commonly expressed informally.))
Saddle Up, a small tack store in Massachusetts, stresses a culture of excellent customer service. To ensure customer satisfaction, the store allows customers to return most items hassle-free, even certain items that have been opened and used, such as boots, helmets, and saddles, as long as they are still in good condition. The store has a bargain section at the back of the store to sell returned items at lower prices. However, certain items, such as vet supplies, fly spray, supplements, and tack cleaning supplies, can only be returned if they are still sealed. In implementing its return policy, the store trusts its employees to ((use their best judgment.))
Organizational cultures that lack positive values may result in all of the following EXCEPT ((empowered employees.))
All of the following are results of establishing a positive organizational culture EXCEPT doing so ((can make employees unproductive and indifferent.))
Which of the following statements is true about organizational structure? ((It is the arrangement or relationship of positions within an organization.))
Marvin started a small restaurant business. At first, he did everything himself, buying the ingredients, preparing and serving the food, creating the menu and setting the prices, and handling the finances of the restaurant. As his business grows, Marvin may have to do all of the following EXCEPT ((keep doing everything himself))
Which of the following is a primary function of an organizational chart? ((It is a visual display of the organizational structure.))
Which of the following does an organizational chart display? ((chain of command))
Greg owns a local ice cream store. His business started small, but over the years, word of mouth brought more people into town to try his delicious homemade ice cream. Since business is booming, Greg has decided to expand his business and open three additional stores in surrounding towns. What does growth like this require, in terms of structure? ((Growth requires developing a formal structure to function efficiently.))
_____ is the division of labor into small, specific tasks and the assignment of employees to do a single task. ((Specialization))
What is the rationale for specialization? ((efficiency))
Auto Corp., a large auto parts store, did an experiment to see whether it was faster to have one person work on each car repair or have a team of people work together on a repair. It found that each employee, working independently, could do one to three auto repairs per day. However, when one person removed the old part, another person prepared the new part, and a third person installed the new part, they were able to complete up to twenty repairs per day. This scenario demonstrates that ((people can perform more efficiently if they master just one task rather than all tasks.))
Which of the following is an advantage of job specialization? ((It makes training employees easier.))
Which of the following is a disadvantage of job specialization? ((It can lead to dissatisfaction and boredom among employees if overly done))
18th-century economist Adam Smith illustrated improvements in efficiency in manufacturing units through the application of ((specialization))
_____ is the grouping of jobs into working units usually called units, groups, or divisions. ((Departmentalization))
Stuart owns a large manufacturing company that supplies food and beverages to grocery stores. His company uses more than one departmentalization plan to enhance productivity. His company would have all of the following departments EXCEPT a _____ department. ((fire))
Small businesses commonly employ _____ departmentalization. ((functional))
A company that has departments for marketing, finance, personnel, and production is organized by ((function))
If the type of departmentalization used by an organization tends to emphasize departmental units rather than the organization as a whole and decision making is slow, then these would be weaknesses indicative of _____ departmentalization. ((functional))
Smooth Sailing Cruises has different departments for frequent travelers and occasional travelers. This illustrates _____ departmentalization. ((customer))
Which of the following is a similarity between customer departmentalization and geographical departmentalization? ((Neither focuses on the organization as a whole.))
An electronic company has a computer electronics division, telecommunications equipment division, and kitchen appliances division. Which of the following methods of departmentalization does the company use to structure its organization? ((product departmentalization))
Which of the following is an advantage of product departmentalization? ((It simplifies decision making and helps coordinate all activities related to a product))
Spicy Dish, a large distributor of canned beans and salsa, is organized into four business units: (1) North America Salsa, (2) North America Legume, (3) Latin America, and (4) Europe and Asia. What two types of departmentalization are illustrated in this example? ((product and geographical))
Multinational corporations often use a geographical approach to departmentalization because ((of vast differences between different regions.))
Red Finch Products Inc. groups employees based on whether they are going to handle the sales account of a wholesale customer, regular consumer, first-time buyer, or a one-time buyer. Which of the following methods of departmentalization is the company using? ((customer departmentalization))
Razor Automobiles Inc. has slightly different versions of the same car for the American, European, and Asian markets. Its employees are grouped into divisions based on the market they have been assigned. This is an example of _____ departmentalization. ((geographical))
Software Solutions Inc., a large software company, has different representatives to assist business clients and individual clients. This is an example of _____ departmentalization. ((customer))
Which of the following is a disadvantage of geographical departmentalization? ((It results in duplication of tasks.))
Which of the following concepts deals with giving employees the power to make commitments and use resources to accomplish the assigned tasks? ((delegation of authority))
_____ is the obligation placed on employees through delegation to perform assigned tasks satisfactorily. ((Responsibility))
_____ is the principle that employees who accept an assignment and the authority to carry it out are answerable to a superior for the outcome. ((Accountability))
Which of the following statements is true about delegation of authority in the context of an organization? ((It does not relieve a superior of accountability for a delegated job.))
Marilyn, the president of EduPub, an educational publishing company, delegates responsibility for all editorial activities to Susan, the executive editor. Susan accepts this responsibility and has the authority to obtain all relevant information, make decisions about what to publish, and delegate any and all activities to her subordinates. She delegates all the editorial activities to the managing editors of each academic discipline. In turn, these managers assign specific projects to their development editors. Who is ultimately accountable to Marilyn for all the editorial activities? ((Susan))
The extent to which authority is delegated throughout an organization determines its ((degree of centralization))
Kevin is a salesman at a large computer company where all the decisions are made by the top levels of management. However, his managers give him the responsibility to carry out his daily and routine sales activities. What type of organization does Kevin work for? ((centralized))
Which of the following is a drawback of centralization? ((It may take a long time for an organization as a whole to implement decisions and respond to changes and problems on a regional scale.))
_____ is usually preferred when the decisions of a company are very risky and low-level managers lack decision-making skills ((Centralization))
Which of the following is true of decentralization? ((It is characteristic of organizations that operate in complex environments.))
Kristen is a manager at a small diner in a tourist town. There are many competing restaurants in the area, so the restaurant owners depend on Kristen and her employees to pay attention to what customers order most often to determine what the daily and weekly specials should be. Kristen has the authority to make changes to the menu and offer promotions to attract more patrons. What type of organization does Kristen manage? ((decentralized))
When an organization has a structure in which decision-making authority is delegated as far down the chain of command as possible, it is said to be exercising ((decentralization))
Which of the following characteristics of a company would make decentralization difficult? ((standardized processes and core products))
Span of management refers to the ((number of subordinates reporting to a manager.))
A wide span of management is appropriate when ((subordinates are highly competent.))
A narrow span of management is appropriate when ((superiors and subordinates are not in close proximity))
Which of the following statements is true about span of management? ((Wide spans of management are more common in decentralized firms, whereas narrow spans of management are typical in centralized organizations.))
Organizational layers, in the context of assigning responsibility in an organization, refer to the levels of ((management in the organization.))
Which of the following statements accurately describes the difference between tall and flat organizations? ((Tall organizations have a narrow span of management, whereas flat organizations have a wide span of management))
Which of the following statements is true about organizational layers? ((The more layers in an organization, the slower the communication.))
Auto Mart, a large car company, has store managers, district managers, regional managers, and functional managers. What type of organization does this describe? ((tall))
In the context of different forms of organizational structure, which of the following is true of line structure? ((It is the simplest organizational structure))
An organizational form that is based on direct lines of authority from the top executive to the lowest level of employees is called a _____ structure. ((line))
An advantage of line structure in organizations is that it ((enables managers to make decisions quickly.))
A disadvantage of line structures is that they ((require managers to possess a wide range of knowledge and skills.))
Which of the following statements is true about the line-and-staff structure? ((It allows line managers to focus on their expertise in the business operation.))
Which of the following is a disadvantage of line-and-staff structures? ((They create ambiguous lines of communication.))
Multidivisional structures ((organize departments into larger groups called divisions.))
Which of the following is an advantage of multidivisional structures? ((They allow divisional and department managers to specialize.))
Which of the following is a disadvantage of a multidivisional structure? ((It inevitably creates work duplication))
Several years ago, Rick started his own heating and HVAC business. As his business grew, Rick noticed that his traditional structure was becoming difficult to coordinate, making communication difficult and decision making slow. He decided to restructure his business, organizing his departments into six larger groups: (1) heating repairs, (2) heating sales, (3) heating installation, (4) air conditioning repairs, (5) air conditioning sales, and (6) air conditioning installation. What type of structure has Rick adopted? ((multidivisional))
In Red Bridge Infrastructure, specialists from different departments are deployed together to form a project team and work on a single project, such as building a bridge or restoring a road. As a result, employees have to report to their functional managers as well the concerned project managers. Thus, Red Bridge Infrastructure is using a _____ structure. ((matrix))
Which of the following is a basis for using a matrix structure within an organization? ((projects))
Which of the following organizational structures is also called a project management structure? ((matrix structure))
Which of the following is true of matrix structures? ((In matrix structures, employees are responsible to two managers.))
Which of the following is an advantage of a matrix structure? ((It provides flexibility, enhanced cooperation, and creativity.))
Which of the following is a disadvantage of matrix structures? ((They are generally expensive and quite complex.))
Matrix structures have been used in all of the following industries EXCEPT ((independent retailing.))
Compared to work teams, work groups ((have individual accountability.))
What is the purpose of work teams? ((collaboration))
Which of the following statements is true of work groups? ((They create individual work products.))
All of the following are true of virtual teams EXCEPTÂ ((virtual teams are not very common in everyday business))
A permanent, formal group that performs a specific task is a ((committee))
MB Inc. has an independent, permanent work group that develops and revises codes of ethics, suggests ways the organization can contribute toward the environment, and regularly reviews other specific issues and concerns within the organization. This is an example of a ((committee))
Which of the following is a temporary group of employees, usually chosen for their expertise, responsible for bringing about a particular change? ((task force))
Which of the following statements is true about task forces? ((They may occasionally comprise of individuals from outside an organization.))
Which of the following is an advantage of having a task force? ((Since task forces are comprised of employees from different departments, they can tackle major company issues.))
When a large accounting firm reorganized, laying off 50 employees, it faced lawsuits alleging that the layoffs targeted minorities and women. The firm developed a two-year group to address the allegations and provide recommendations for improving company reorganization practices. What type of group does this scenario describe? ((a task force))
All of the following are true of teams EXCEPT they ((function best when they have more than 10 members.))
Project teams ((run their operation and have total control of a specific work project.))
A special type of project team that focuses on devising, designing, and implementing a new product is a ((product-development team.))
To meet the competition in the mobile phone industry, Vibgyor Electronics Inc. has created a new project team to conceive, design, and implement a smart phone that will have some breakthrough features. The team includes specialists from various divisions, such as the computer division, software division, and television division. This project team is an example of a ((product-development team.))
Which of the following teams consists of fairly small groups of workers brought together to address specific quality, service, or productivity problems within an organization? ((quality-assurance teams))
Auto Giant Inc. released a new car model a year ago, and it has received customer complaints about the flimsy plastic used around the lights and in the interior consoles of the car. To address this problem, Auto Giant Inc. forms a quality circle. What is a benefit of forming this type of group? ((It will shift the organization to a more participative culture.))
A _____ is a group of employees responsible for an entire work process or segment that delivers a product to an internal or external customer. ((self-directed work team))
Which of the following is true of self-directed work teams? ((They are designed to give employees a feeling of ownership of a whole job.))
All of the following are alternatives to face-to-face communications EXCEPT ((meetings))
Which of the following is a problem created by increased access to the Internet at work? ((Employees may abuse company e-mail and Internet access.))
What is upward communication? ((communication that flows from lower to higher levels of an organization))
Katie is a supervisor at a fast food restaurant. Every week, she submits a progress report to Molly, the restaurant manager, to let her know how the restaurant is doing and to bring any employee performance concerns and customer feedback to her attention. What type of communication does this scenario describe? ((upward communication))
Which of the following is an example of downward communication? ((directions))
Hector is a manager of a bookstore. At the end of each year, he gives his employees performance reviews to let them know what they are doing well, what they need to improve, and their annual raise amount. This is a form of _____ communication. ((downward))
Downward communication typically conveys ((assignments of tasks and responsibilities.))
In an automobile company, the vice president of research and development wants to plan and coordinate the launch of a new car with the vice president of marketing. Any related communication between them will be referred to as _____ communication. ((horizontal))
Mark and Dan are both managers at a sporting goods store. Mark is a manager in the finance department and Dan is a lower-level manager from the shoe department. When these managers communicate, it is known as _____ communication. ((diagonal))
Laura and Gwen are friends who work at the same company but in different departments. They often go out for lunch and drinks after work, and sometimes, they send each other personal e-mails and instant messages at work. Such friendships comprise the _____ of a firm. ((informal organization))
Which of the following is a part of informal communication channels? ((gossip))
Claudia is a manager of a real estate company. She is aware that her company has an active grapevine because she has heard her employees gossiping in the lunch room and around the office. All of the following are things she can do to use the grapevine to her advantage EXCEPT ((confronting her employees about gossip.))
Which of the following refers to the exchange of information among colleagues and peers on the same organizational level? ((horizontal communication))
Diagonal communication ((occurs when individuals from different units and departments within an organization communicate.))
Venus Electronics Inc. wants to ensure that its products best suit the needs of its customers, so the head of the marketing department holds a meeting with the managers of the production department. This illustrates _____ communication. ((diagonal))
Which of the following do employees list as their top complaint, related to communication, in the workplace? ((Their managers fail to listen to their concerns.))
Which of the following statements is true about e-mail communication? ((Using work e-mail accounts to send personal information may be against company policy.))
Human relations is the study of the behavior of individuals and groups in organizational settings. ((TRUE))
The Hawthorne studies found that money was the primary motivator for employees. ((FALSE))
Theory Z is more participative and encourages lifelong employee commitment more than traditional management approaches. ((TRUE))
Rewarding appropriate behavior tends to be more effective to modify behavior in the long run than punishments for unacceptable behavior. ((TRUE))
The field of human relations has become increasingly important over the years as businesses strive to understand how to do all of the following EXCEPT ((boost company sales))
Which of the following statements is true of motivation? ((Needs can be motivating))
_____ is the difference between a desired state and an actual state. ((A need))
Calvin teaches karate at a large martial arts studio. After a new karate movie came out, Calvin noticed an influx in clients who want to skip the fundamental basics of karate and learn the more advanced moves featured in the movie. He is finding that working with these new clients is very challenging. Which of the following is something Calvin might do to help him succeed in reaching these new clients? ((watch the new karate movie and use some of its techniques to engage the new clients))
Graham, an employee at an architectural firm, is frustrated with his job as he is mostly involved in reproducing stock designs. He realizes the need for starting his own firm to exercise his creative streak. In the context of the motivation process, which step will immediately follow the need recognition step? ((goal-directed behavior))
Which of the following is the last step in a typical motivation process? ((satisfaction))
Low morale is likely to result in ((absenteeism))
High morale contributes to ((high returns to stakeholders))
Coffee Creations is a chain of small coffee shops that are located in college towns and mostly employ students. The company offers flexible scheduling, employee scholarship programs, and strong internal promoting, and as a result, it has loyal employees who have high levels of productivity. Based on this scenario, employees of Coffee Creation should have ((high morale.))
A(n) _____ is the personal satisfaction and enjoyment that a person feels from attaining a goal. ((intrinsic reward))
Karen has always been passionate about fashion, so she went to fashion school to learn more about it and opened her own clothing boutique. She enjoys running her business and feels a deep sense of satisfaction every time she finds new styles to bring to her store. In this scenario, Karen is experiencing a(n) ((intrinsic reward))
_____ are benefits and/or recognition that a person receives from someone else. ((Extrinsic rewards))
All of the following are ways to retain good employees EXCEPT ((micromanaging employees))
Blooming Bouquets is a florist that knows how to retain happy employees. All of the following are benefits this company might provide to its employees EXCEPT ((managers that control every part of the job))
Jamal manages a fitness club, which employs front desk personnel, personal trainers, and fitness instructors. He wants to motivate his employees to work hard and maintain high levels of productivity. Which of the following would Jamal likely do to motivate his employees? ((allow employees to make mistakes, as these become learning opportunities))
The birth of the study of human relations within business organizationscan be traced to ((time and motion studies by Frederick W. Taylor and Frank and Lillian Gilbreth))
According to the classical theory of motivation, which of the following would motivate employees to work hard? ((financial incentives))
Which of the following theories is associated with the piece-rate system? ((the classical theory of motivation))
Vixen works in a soap manufacturing company. He is paid a fixed amount for every unit he manufactures at the factory. He is paid higher rates for exceeding his set target. The payment system in this scenario is an example of the _____ system. ((piece-rate))
Welcome Home, a real estate company, encourages its realtors to partner and collaborate with each other to find appropriate homes for their clients. To generate partnership and collaboration, this company offers employees ((team incentives))
The person primarily associated with the Hawthorne studies is ((Elton Mayo))
Which of the following is labeled the Hawthorne effect? ((increase in productivity regardless of physical conditions))
The Hawthorne studies were important because they ((revealed the significance of social and psychological factors))
What was the ultimate result of the Hawthorne studies? ((They marked the beginning of a concern for human relations in the workplace.))
According to Maslow, humans devote all their efforts to satisfying _____ needs until they are met. Only when these needs are met can people focus their attention on satisfying the next level of needs. ((physiological))
Mitch works for a store that sells hot tubs and pools. He is a competitive person, so he needs to feel like he can sell hot tubs and pools better than the other store employees. When he is able to sell more hot tubs and pools that his co-workers, he feels a high level of self-respect and respect from others. This scenario describes how Mitch fulfills his _____ needs. ((esteem))
Frederick Herzberg proposed ((the two-factor theory.))
Mega Tech., a large technology company, has started using methods to give employees more responsibility and control and to involve them more in their work. The company views employees as team members and encourages them to share their ideas about how to improve its technology products. What is the benefit to using these methods? ((They motivate employees to higher levels of productivity and quality.))
Sandra manages a grocery store, which employs several baggers, cashiers, floor managers, and stockroom workers. Which of the following is something Sandra can do to ensure that her employees are satisfied with their work setting? ((She can provide adequate wages.))
Theory X and Theory Y are most closely associated with ((Douglas McGregor))
Anand is a manager who believes that he can force his subordinates to work extra hours through close supervision. He instills fear among employees by threatening to cancel bonuses if deadlines are not met. Anand could be described as a ((Theory X manager according to McGregor.))
ignore the self-actualization needs of his employees. ((Matt is a Theory X manager at an office supplies store. In managing his employees, he is most likely to ))
The approach that suggests that imagination, ingenuity, and creativity can help solve organizational problems is ((Theory Y))
Kim, the sales manager at a retail store, is a supporter of micro-management tactics. She keeps a close watch on the members of her team to ensure that they do not slack at work. Kim is most likely a manager who ((supports the Theory X approach.))
All of the following are assumptions of Theory Y EXCEPT ((the average worker has relatively little ambition.))
Theory Z was first described by ((William Ouchi.))
In the context of employee motivation theories, Theory Z includes many elements associated with the _____ approach to management. ((Japanese))
At Rustic Rooms Furniture, managers and workers share responsibilities, managers stress employee participation in all aspects of company decision making, and employment is long term. What theory of management does Rustic Rooms Furniture follow? ((Theory Z))
Theory Z and Theory Y can be seen as ((complementary))
When comparing Theory X, Theory Y, and Theory Z, which of the following statements is true? ((In regards to responsibility, Theory X is managerial, Theory Y is collaborative, and Theory Z is participative.))
"According to _____, how much people are willing to contribute to an organization depends on their assessment of the fairness of the rewards they will receive in exchange. ((equity theory))"
((equity theory))"
Which of the following statements is true regarding equity theory? ((In a fair situation, a person receives rewards proportional to his or her contribution to the organization.))
How can managers avoid equity problems? ((by ensuring that rewards are distributed on the basis of performance))
Jane learned that although she and June were both hired as part-time sales clerks at the same time and have similar backgrounds, June is paid $1 more per hour. Jane decided to stop cleaning dressing rooms on her shift because she thinks the pay is unfair. This scenario is related to which of the following management theories? ((equity theory))
Carolyn, Todd, and Bethany all work at a department store in the mall. They have been working there for the same amount of time, and they all have similar educational and employment backgrounds. Bethany has just discovered that Carolyn and Todd both make more than her. Which of the following is an unethical or illegal behavior Bethany may exhibit to equalize the situation? ((She may start stealing items from the store.))
Which of the following is stated by the expectancy theory? ((A person who wants something and has reasons to be optimistic is likely to be highly motivated.))
Which of the following best describes management by objectives? ((the need to develop goals that both managers and employees can understand and agree upon))
What is an advantage associated with job rotation? ((It reduces the boredom created by specialization of tasks.))
What is a drawback of job rotation? ((It is unable to totally eliminate the problem of boredom))
Mazer Inc., a publication house, allows its employees to try out a variety of profiles, ranging from editing to designing cover pages. The practice at Mazer best exemplifies ((job rotation))
Which of the following adds tasks to a job instead of treating each task as a separate job? ((job enlargement))
What is the rationale behind job enlargement strategies? ((Jobs are more satisfying as the number of tasks performed by an individual increases))
Which of the following is true regarding how job enlargement strategies differ from job rotation strategies? ((Job enlargement strategies increase job satisfaction more than job rotation strategies.))
Paula works for a home decor store. Her normal tasks include cleaning the store fixtures, helping customers find what they are looking for, and ringing up customer purchases. One day, she notices that several of the shelves are empty, so she goes into the back room and finds merchandise to fill the shelves. When she shows her manager, Tricia, what she has done, Tricia is so impressed that she asks Paula to check all the shelves on a weekly basis and stock any empty shelves with new merchandise. This scenario best describes job ((enlargement))
Job _____ is the incorporation of motivational factors such as opportunity for achievement, recognition, responsibility, and advancement into a job. ((enrichment))
Which of the following is likely to be a result of job enrichment? ((Employees will have more control and authority over their job))
A work program that allows employees to choose their starting and ending times, as long as they are at work during a specified core period is called ((flextime))
Kart Inc., a publishing house, allows employees to log in at their preferred time and log off after completing eight hours. It does not have any fixed office timings. Kart Inc. offers _____ to its employees. ((flextime))
It improves What is an advantage of using a flextime schedule? the ability to recruit and retain workers who wish to balance work and home life. ((It improves the ability to recruit and retain workers who wish to balance work and home life.))
Apps for Apples, a technology company that produces educational applications for Macs and other Apple products, allows employees to work on a flextime schedule. All employees must be at work from 10:00 a.m. to 2:00 p.m. to attend meetings, but other than that, employees have the flexibility to choose which hours they work. All of the following are benefits of this type of scheduling EXCEPT it ((reduces the total number of hours that employees work.))
A compressed workweek is ((a four-day or shorter period in which employees work 40 hours))
Lindsay and Tim are married, and they recently found out that they are going to have twin daughters. To have more time to spend with their children, they are looking for new jobs that will provide greater flexibility in scheduling. All of the following are jobs that would be good options for Lindsay and Tim EXCEPT being ((teachers))
Job _____ occurs when two people do one job. ((sharing))
Little Corp., a toy manufacturing company, allocates one task to an employee working from 8:00 a.m. to 12:30 p.m. and allocates the same task to be completed by another employee working from 12:30 p.m. to 5:00 p.m. This is an example of job ((sharing))
Which of the following is a flexible scheduling strategy? ((job sharing))
Which of the following options is likely to be most effective for retaining employees who are trying to juggle their work duties with other responsibilities and needs? ((job sharing))
Laurel and Jennifer both work as receptionists at a law office. Laurel works from 8:00 a.m. to 12:30 p.m., so she can be home when her daughter gets home from school. Jennifer works from 12:30 p.m. to 5:00 p.m., so she can attend graduate school classes in the morning. Which of the following is a benefit this arrangement has for the company? ((The company has the skills of two people for one job.))
Allowing some employees to work at home part-time is ((a flexible scheduling strategy.))
Camden Inc., a pharmaceutical company, allows its employees to work at home a few days per week, staying connected via computers, modems, and telephones. This arrangement is called ((telecommuting))
What is an advantage of allowing employees to telecommute? ((It helps reduce overhead costs for businesses.))
Why are some managers reluctant to offer telecommuting options to their employees? ((It increases security concerns.))
Planning, as a function of human resources management, includes determining the status of current personnel when planning human resource needs for the future. ((TRUE))
Classroom training allows employees to learn by actually performing the tasks involved in their jobs. ((FALSE))
When managers speak of diverse workforces, they typically mean differences in gender and race. ((TRUE))
Natalie is a human resources manager at a large legal services company. She must decide how many new employees her company will need to fill vacant positions in the near future. In this scenario, Natalie is engaging in ((planning))
_____ involves determining, through observation and study, the specific tasks that comprise a job; the knowledge, skills, and abilities necessary to perform the job; and the environment in which the job will be performed. ((Job analysis))
Zen W Inc., a publication house, has posted an advertisement inviting applicants to pursue a role as a graphic designer. The company has specified that the job will involve illustrating content by designing aesthetically tasteful layouts. This is an example of a job ((description))
Which of the following information is most likely to be included in a job specification rather than a job description? ((education qualification required for a job))
Which of the following statements is true of a job specification as well as a job description? ((Both the job description and job specification are used to develop recruiting materials))
Legal Partners Inc. is looking to hire a lawyer, so it has posted the job in several legal publications. In its ad, the company requests applicants who have completed law school and passed the bar examination, have courtroom experience, and are confident and hardworking. This describes ((a job specification.))
Which of the following functions of human resources management includes testing? ((selection))
_____ refers to forming a pool of qualified applicants from which management can select new employees. ((Recruiting))
Sally manages a hair salon, and she is looking for a new head stylist to lead her team of hairdressers. She posts an ad on several job websites and uses those websites, along with LinkedIn, to look for qualified candidates. She also watches her employees work to see if any of them exhibit the skills she is looking for in a head stylist. Once she has received a few applications, found a few qualified candidates, and determined that one of her employees might be a good fit, she sets up interviews with these potential candidates to determine who to hire. Which human resources activity does this scenario describe? ((recruiting))
When compared to the external sources of recruitment, recruiting from internal sources ((improves employee morale))
Which of the following is an internal source of recruitment for an organization? ((current employees))
Why do many firms have a policy of giving first consideration to their own employees? ((The cost of hiring current employees to fill job openings is inexpensive.))
Lazar Inc., a software firm, has taken to recruiting potential employees from engineering colleges to fill its entry-level vacancies. On the basis of the given information, which of the following observations holds true? ((The management at Lazar is hiring its employees from external sources))
A characteristic of search firms known as headhunters is that they ((look for qualified candidates who are working for other companies))
Solutions Inc., an HR consultancy firm, specializes in luring qualified people away from other companies. Many companies hire Solutions Inc. to fill their managerial or professional positions above the entry level. On the basis of the given information, we can conclude that Solutions Inc. is included in the category of ((headhunters))
Jethro is the chief of police in a small town. He is looking for a new sheriff, and he already has three candidates to consider. He is in the process of carefully considering their applications, conducting interviews, administering performance tests, and checking their references. What is this process called? ((selection))
A publishing house is in the process of selecting a new development editor from a pool of potential candidates. All of the following would be part of this selection process EXCEPT ((posting an ad for the position on publishing career websites))
An employer beginning the selection process for new employees would begin with the ((application form.))
Justin has worked as a marketing manager for eight years. He has decided to move to be closer to his family, so he is looking for a new job. When filling out an online application for a marketing firm, what information might he be asked to provide? ((his current salary))
Nadia has recently graduated from college, and she is ready to look for her first job in her field. Her college provides her with career counseling to help her draft her résumé and prepare for the application and interviewing processes. Which of the following is an interview tip she might learn from her career counselor? ((Evaluate the organizational culture.))
Candace has applied for a sales position at computer software company, and she has been invited to an in-person interview. All of the following are questions she can expect to be asked during the interview EXCEPT ((what are the sales figures of your previous employer?))
All of the following are mistakes that are commonly made in interviews EXCEPT ((not enough concern about compensation.))
Which stage of the selection process may require a candidate to undergo physical examinations to determine his or her suitability for a specific job? ((testing))
Joanna has just completed graduate school and earned a Masters in publishing and writing. She recently interviewed for an editorial assistant position at a small publishing house, and the human resources manager sent Joanna home from the interview with a copyediting test, giving her a week to complete it. Which of the following is a reason the human resources manager may have given Joanna this ability test? ((to determine whether she has the skills necessary for the job))
Reference checking usually involves ((checking the previous work experience of a candidate))
Doug is a human resources manager at a large advertising agency. He is considering applicants for a consumer behavior analyst position. He has already conducted interviews and administered personality tests, and he has narrowed it down to the two strongest applicants. To help him decide which applicant to choose, he is planning to call their references to verify their educational background and previous work experience. He also plans to do an Internet search to determine their social media activities and any other public activities. All of the following are red flags Doug may uncover during the reference checking stage EXCEPT ((overly enthusiastic references))
The _____ is a federal agency established by the Civil Rights Act of 1964 that is dedicated to increasing job opportunities for women and minorities and eliminating job discrimination based on race, religion, color, sex, national origin, or disability. ((Equal Employment Opportunity Commission))
The Americans with Disabilities Act ((prohibits using a positive AIDS test as reason to deny an applicant employment))
In the United States, wage differences are acceptable only if they ((attributed to seniority.))
Which of the following practices is considered to be illegal in the United States? ((forcing employees to retire on the basis of age))
Stanley owns a plumbing company, which he has run for 50 years. Although he has hired some younger plumbers, he tends to prefer hiring older plumbers because he finds that they ((take pride in their work))
Orientation is the human resources function that ((includes building tours, introductions, and socialization))
Which of the following is an advantage of the orientation process? ((It allows firms to familiarize new employees with prevalent office culture))
Fluffy Bakes, a popular bakery, allows its newly recruited employees to learn the processes involved at its premises by actually working on live assignments. The training process at Fluffy Bakes best exemplifies ((on-the-job training))
Life Line, a chain of healthcare providers, sends its employees who are interested in learning about hospital management to some of the leading institutes in the nation. The training process at Life Line best exemplifies ((classroom training))
Ramone has recently started working at an accounting firm. To get him up and running quickly, the firm has paired him with an experienced accountant to offer him support, training, and guidance as he learns the tasks of his new job. This is an example of ((mentoring))
All of the following are true of mentoring EXCEPT it ((teaches employees with lectures, conferences, videos, case studies, and web-based training.))
What is the primary reason that training is considered a vital function of human resources management? ((It is used to improve the skills of employees in their present positions and to prepare them for increased responsibility.))
Which of the following is a newer and increasingly popular method of providing training and development? ((experiential and involvement-oriented training exercises))
Lucy is a manager at an advertising firm. Every year, she must assess her employees in a performance review to let them know how they are doing and what they can improve. These performance reviews are tied to raises and promotions, so Lucy takes them very seriously. All of the following are characteristics she assesses in these reviews EXCEPT ((educational background))
Which of the following statements is true regarding performance appraisals? ((Performance appraisals may be objective or subjective))
Which of the following is an example of an objective assessment of employee performance? ((Darrell, a car salesman, is judged by the number of cars he has sold.))
A _____ is a performance appraisal method that provides feedback from a panel that typically includes superiors, peers, and subordinates. ((360-degree feedback system))
In the manufacturing unit of a tire company, the appraisal process primarily focuses on rewarding employees on the basis of the rate at which they regularly produce work. Thus, the appraisal process is focusing on the _____ aspect. ((productivity))
Lucas teaches high school geometry. In his performance review, his principal praises him for using resourceful ideas, solutions, and methods to make dry topics more interesting for his students. What performance characteristic is the principal highlighting? ((creativity))
Kevin is a barista at a coffee shop. His manager has noticed that he has been showing up late for work, getting coffee orders wrong, and being rude to customers. What should his manager do to help Kevin improve his work? ((give Kevin constructive criticism, so he knows what to expect and how he is viewed))
All of the following are results of turnover, when employees quit or are fired and must be replaced by new employees, EXCEPT ((more employees working remotely))
Sophia, a software engineer, moves to a new job that involves more responsibility and an increase in compensation. This transition would typically be considered a(n) ((promotion))
Which of the following is likely to be an unacceptable reason for firing an employee? ((being a union organizer))
A well-organized human resources department strives to ((minimize losses due to separations and transfers.))
Financial compensation falls into two general categories, which are ((wages and salaries))
A wage/salary survey indicates ((how much compensation similar firms are paying for jobs that the firms have in common.))
Jonathan and Eli both work as farmers, taking care of animals, harvesting crops, and making farm repairs for their employers. Although they work at different farms, their employers are neighbors, so they are friends. When comparing their wages, they determined that Eli makes $15 per hour, while Jonathan only makes $12 per hour. Which of the following is most likely the reason for this difference in wages? ((Eli is more productive than Jonathan))
Which of the following statements is true of payment using time wages? ((It is appropriate when employees are continually interrupted.))
What is an advantage of time wages? ((They can be computed easily))
What is a disadvantage of time wages? ((They provide no incentive to increase productivity.))
Which of the following is a major advantage of piece wages? ((They motivate employees to increase output))
At Drive Away Car Dealership, employees are paid a percentage of their sales in order to motivate them to sell as many cars as they can. This company incorporates _____ as an incentive system. ((commissions))
It is associated with white-collar workers ((It is associated with white-collar workers))
Maureen works in a law office as a paralegal. She makes a salary of $50,000 per year, which is paid on a biweekly basis. What is one disadvantage of this form of payment? ((It may require her to work beyond usual hours without additional financial compensation.))
Christopher works as a news anchor for a television network. In addition to his regular pay, at the end of each year, his company pays him and his co-workers additional compensation as a "thank you" for good work. This monetary reward would most likely be categorized as a ((bonus))
Which of the following forms of compensation includes an employee stock ownership plan? ((profit sharing))
Which of the following statements is true of employee stock ownership plans? ((They create a sense of partnership between a firm and its employees))
_____ are nonfinancial forms of compensation provided to employees. ((Benefits))
Bruce works for an accounting firm that provides sick leave, vacation pay, pension plans, health plans, and other forms of extra compensation. These particular items are ((fringe benefits.))
Which of the following is a form of financial compensation? ((commissions))
Rachel works for a large publishing company that employs many different types of employees. As a result, her company provides a financial amount to employees so they can select the specific benefits that fit their needs. This company uses ((a cafeteria benefit plan.))
Paul works in a factory that produces nuts and dried fruits. He feels that he deserves better pay, hours, and working conditions. Although he has talked to management several times about these items, they still have not made any improvements. He decides to join his co-workers in a(n) _____, so they can hire specialists to represent them in their dealings with management. ((labor union))
_____ is the negotiation process through which management and unions reach an agreement about compensation, working hours, and working conditions for concerned employees. ((Collective bargaining))
All of the following are items management tries to retain control over in a labor contract EXCEPT ((better pay rates for overtime.))
What is the significance of the "cost-of-living adjustment" clause in labor contracts? ((It calls for automatic wage increases during periods of inflation.))
Which of the following statements is true of labor unions? ((They help their members achieve better pay, hours, and working conditions.))
Which of the following objectives do labor unions strive to achieve? ((to schedule pay increases))
Union growth has slowed in recent years because ((factories have become more automated))
The formal, written document that spells out the relationship between the union and management for a specified period of time is called a(n) ((labor contract.))
During tough economic times, unions may be forced to accept _____, wage and benefit concessions made to employers to allow them to remain competitive or, in some cases, to survive and continue to provide jobs for union workers. ((givebacks))
All of the following are steps in the collective bargaining process EXCEPT ((management rejects the contract by striking, boycotting, or picketing.))
Which of the following statements is true of a strike? ((It halts or disrupts the normal working of a firm.))
Glen manages a candy packaging factory. After failed negotiations between Glen and the labor union, the union members strike, partially shutting down the factory. In response, Glen shuts down the factory completely, restricting all his employees from coming to work. This situation best represents a(n) ((lockout))
When is a firm most likely to vote for a lockout? ((when a union strike has partially shut down a plant and it seems less expensive for the plant to close completely))
Why is strikebreaking generally considered to be a last-resort measure for management? ((It does great damage to the relationship between management and labor))
The negotiations between a union and the management representatives of a snack food firm come to a standstill and a third party is brought in to propose solutions to help resolve the impasse. However, the solutions are not binding on the parties. This third party would most likely be a(n) ((mediator))
Which of the following is a similarity between conciliators and mediators? ((Neither of them has formal power over labor or management))
The management at Gomars Inc., a firm manufacturing toys, is involved in a dispute with the union members representing its employees. The union members want a pay hike, but the management is not ready to approve the hike. Failing to arrive at a compromise, the disputing parties involve a third party to resolve the issue and offer a solution that is legally binding. The third party in the given scenario best represents a(n) ((arbitrator))
Management of a firm and labor submit to compulsory arbitration when the federal government ((requests arbitration as a means of eliminating a prolonged strike that threatens to disrupt the economy.))
The management at LKS Inc., a firm manufacturing hard disks, is involved in a dispute with the union members representing its employees. The management and the labor union do not seem to agree on the amount of compensations that need to be paid to employees. Failing to arrive at a compromise, the disputing parties involve a third party to resolve the issue and offer a solution to the problems. The solution provided by the third party is open to the approval of both parties involved and is not likely to be binding. The third party in the given scenario best represents a(n) ((mediator))
The management at Links Inc. is faced with a strike threat from union members over a dispute concerning working hours for employees. The disputing parties approach a neutral third party to ensure that the discussions between them are carried forward. The third party, however, has no formal power over union representatives or over management. Which of the following processes is best exemplified in the scenario? ((conciliation))
Which of the following is a primary characteristic of diversity? ((age))
Which of the following is a secondary characteristic of diversity? ((income))
All of the following describe people using primary characteristics of diversity EXCEPT ((Laura is married with two children who are 5-years-old and 8-years-old))
If an organization fosters and values workforce diversity, it is most likely to ((reduce conflict among employees of different cultural groups.))
Companies that do not value their diverse employees are most likely to ((experience greater prejudice))
_____ programs are legally mandated plans that try to increase job opportunities for minority groups. ((Affirmative action))
Which of the following is a concern that companies have raised against affirmative action plans? ((Companies argue that affirmative action plans stifle their ability to hire the best employees.))
Which of the following factors contributed to the controversies surrounding affirmative action plans? ((increased reverse discrimination))
Historically, GloboTech, a large technology company, has employed mostly white men. More recently, it has adopted affirmative action to make up for past hiring and promotion prejudices, overcome workplace discrimination, and provide equal employment opportunities for all employees. Which of the following is something GloboTech should avoid doing in its quest for a more diverse workplace? ((setting hiring quotas))
The customer value of a product refers to anything a buyer must give up to obtain the benefits the product provides. ((FALSE))
Companies segment markets on the basis of demographic, geographic, psychographic, and behavioristic variables. ((TRUE))
Buying behavior includes the behavior of both consumers purchasing products for personal or household use as well as organizations buying products for business use. ((TRUE))
It is necessary for nonprofits, government institutions, and even people to market themselves to spread awareness and achieve desired outcomes. ((TRUE))
_____ is best described as a group of activities designed to expedite transactions by creating, distributing, pricing, and promoting goods, services, and ideas. ((Marketing))
Marketing activities best create value by ((allowing individuals and organizations to obtain what they need and want))
For the products launched by companies to succeed, it is important that ((all the functional areas of the business are coordinated with marketing decisions))
Nadine is the marketing manager for a company that produces innovative educational technology products. Her colleagues have created a new set of simulations that allow philosophy students to respond to ethical dilemmas. All of the following are things Nadine and her colleagues must do to ensure that this new product succeeds in the market EXCEPT ((simply selling and advertising the product))
_____ has the important function of providing revenue to sustain a firm. ((Marketing))
Dennis goes to his favorite local bakery and hands the cashier $5 for a loaf of bread. Which of the following has just taken place? ((exchange))
Claudia and Jason are celebrating their 1-year wedding anniversary, and they have just gone for a couples massage at a spa. Jason hands his credit card to the cashier at the spa to pay for their massages. This is an example of a(n) ((exchange))
At the heart of all business is the_____, the act of giving up one thing in return for something else. ((exchange))
Which of the following is a condition required for an exchange to take place? ((Buyers and sellers must be able to communicate about the "something of value" available to each.))
Daniel and Mark both collect baseball cards. Daniel has a valuable and rare card that Mark has been wanting for a long time. What must Mark do to convince Daniel to part with this card? ((Mark must offer Daniel something he values enough to give up his valuable and rare card.))
Social Smoothies is a small, local company that, in addition to making delicious smoothies, offers its customers live music, open microphone nights, free Internet access, and comfortable seating so they can enjoy their smoothies with friends or while working. In the case of this company, which of the following statements is most likely true? ((The tangible product itself may not be as important as the benefits associated with the product.))
Ryan has produced 1,000 jars of pickled oranges, using the oranges in his orchard. By concentrating his efforts on advertising and publicity, he is trying to encourage people to notice and buy his product. Through his promotional activities, Ryan is primarily engaging in the marketing function of ((selling))
Burgerama Inc., a popular fast-food chain, is famous for its fries and hash browns. It is one of the few fast-food chains to exclusively use the potatoes from its own farms. However, its potatoes can be harvested for only three months of the year. Which of the following marketing functions can Burgerama employ to achieve time utility and satisfy year-round demand for its exclusive fries and hash browns? ((storing))
Randall is a packer in the frozen foods section of a grocery store. He labels meat steaks with little or no fat as "AA" and meat steaks with more fat as "AB." When he does this, he is primarily engaging in the marketing function of ((grading))
All of the following are activities marketing performs to accomplish objectives and generate exchanges EXCEPT ((trading))
_____ is the chance of loss associated with marketing decisions. ((Risk))
The exchange process is expedited through ((selling))
Amanda owns a rustic furniture store in New Hampshire. When customers buy her unique furniture pieces, she arranges a time for her employees to deliver the furniture to their homes using her delivery truck. This is an example of ((transporting))
Warehouses hold some products for lengthy periods in order to ((create time utility.))
_____ refers to standardizing products by dividing them into subgroups and displaying and labeling them so that consumers clearly understand their nature and quality. ((Grading))
Carl is the marketing manager of a large department store that sells a variety of household appliances, including refrigerators, washing machines and dryers, and stoves. What might Carl do to help customers finance these large, expensive items? ((arrange credit))
Through _____, marketers ascertain the need for new goods and services. ((research programs))
Megan is shopping for a new laptop computer. All the computer brands offer laptops with standard screens and keyboards, but each brand offers different software packages, memory amounts, processor speeds, hard drive sizes, and operating systems. Megan has noticed that the price of laptops varies depending on the features they include. In order to choose the best laptop computer for her, Megan will most likely ((judge which type of laptop offers her the best value according to the benefits she wants and her ability to pay for those benefits.))
For greater accessibility, Hakin Inc. has placed its products at every popular superstore. The convenience provided by the company is an example of reducing _____ costs. ((nonmonetary))
_____ costs include anything a buyer must give up to obtain the benefits the product provides. ((Customer))
All of the following are true regarding how customers receive benefits from products EXCEPT they ((deduct what they must give up to obtain the benefits from their perceived value of the product.))
In the context of the marketing concept, which is the first step that a business must take? ((find out what consumers desire))
To gain an edge, businesses must ((continually alter, adapt, and develop products to keep pace with changing consumer needs.))
In which of the following scenarios is a business least likely to survive? ((when a business focuses only on customer satisfaction and not on its own objectives such as boosting productivity))
_____ is the goal of the marketing concept. ((Customer satisfaction))
Which of the following is an ineffective practice to follow while implementing the marketing concept? ((adopting a product orientation instead of a consumer orientation to deliver the right good or service))
All of the following are true regarding the marketing concept EXCEPT ((regardless of their goals and resources, all companies must implement the marketing concept in the same way.))
In the context of the evolution of the marketing concept, the era of production orientation was characterized by ((the development of new technologies that made it possible to manufacture goods with ever increasing efficiency.))
During the period of _____ orientation, the supply of manufactured goods caught up with and then exceeded demand, which made businesspeople realize they would have to promote their products to get buyers to desire them. ((sales))
During the first half of the 20th century, businesspeople viewed _____ as the primary means of increasing profits. ((sales))
The market orientation approach emphasizes ((determining what customers want first and then producing it.))
A market orientation requires organizations to ((gather and use customer information to help build long-term relationships with customers.))
Why is customer relationship management important in a market orientation? ((It can result in loyal and profitable customers.))
According to the market orientation, profits can be obtained through ((acquiring new customers.))
_____ remains a major element of any strategy to develop and manage long-term customer relationships. ((Communication))
A _____ is a plan of action for developing, pricing, distributing, and promoting products that meet the needs of specific customers. ((marketing strategy))
The first step in developing a marketing strategy is ((selecting a target market.))
Kristoff wants to start a new copyediting business. He identifies that his target market is publishing companies that outsource their copyediting projects. Now that he has identified his target market, what must he do next? ((develop an appropriate marketing mix))
A _____ is best described as a group of people who have a need, purchasing power, and the desire and authority to spend money on goods, services, and ideas. ((market))
A _____ refers to a specific group of consumers on whose needs and wants a company focuses its marketing efforts. ((target market))
Red Unicorn Inc. is a company that manufactures wooden planks for construction. It has categorized its customers on the basis of their product requirements into construction companies, wholesalers, and government institutions. Which of the following marketing strategies is best illustrated in this scenario? ((market segmentation))
True Pharmaceuticals manufactures and sells a variety of drugs for a large market consisting of people of different genders, ages, educational backgrounds, lifestyles, geographic locations, and income levels. It assumes that all buyers have similar medical needs and wants. In this scenario, True Pharmaceuticals is most likely demonstrating a _____ to marketing. ((total-market approach))
Which of the following products is most suitable for selling through a total-market approach? ((salt and sugar))
Why does the total-market approach work best for sellers of salt, sugar, and many agricultural products? ((Everyone is a potential consumer of these products.))
Plentiful Auto Parts manufactures engines, brakes, mufflers, and batteries and sells them to car manufacturers. This is an example of ((business-to-business marketing))
Home Decor Emporium sells rugs, art, lamps, and other decorative pieces to homeowners, so they can express their personal styles in their homes. This is an example of ((business-to-consumer marketing.))
Which of the following is a challenge marketers in the United States will face in the future? ((effectively addressing an increasingly racially diverse population))
In the _____ approach to market segmentation, a company develops one marketing strategy for a single market segment. ((concentration))
Electronica Inc. is a mobile phone retailer that sells smartphones to high school students at easily affordable prices. The company has one marketing strategy for the entire market of high school students and focuses all its efforts on this one segment. In this scenario, Electronica Inc. is most likely using the _____ approach for marketing its smartphones. ((concentration))
In the _____ approach, the marketer aims its marketing efforts at two or more segments, developing a marketing strategy for each. ((multisegment))
Kemmen Foods, a company that manufactures and sells breakfast cereals, has customized its cereal flavors to suit different lifestyles, personal tastes, and age groups. For example, it makes organic cereals for its health-conscious customers and colorful cereals with cartoon characters for children. Which of the following market segment approaches is Kemmen Foods using in this scenario? ((multisegment approach))
In _____ marketing, all marketing efforts focus on one small, well-defined market segment that has a unique, specific set of needs. ((niche))
Highland Green is a group of luxury hotels that caters exclusively to high-end customers who form a small part of the market. The hotel has incorporated all possible luxuries to satisfy these customers. The marketing approach employed by Highland Green best exemplifies ((niche marketing))
If a company develops an advertising campaign exclusively for a segment of consumers with a certain income and education, which of the following segmentation variables is the company using? ((demographic))
The Beautiful Skin Company develops several different face creams, including an acne treatment cream for adolescents and a wrinkle-reducing cream for older people. These face creams have been developed on the basis of _____ segmentation. ((demographic))
Which of the following is a basis for the psychographic segmentation of markets? ((lifestyle))
Gear Power Inc., an automobile company, manufactures different cars for different market segments. It markets its SUVs for customers who live in the mountains, sedans for customers in the coastal plains, and smaller hatchback cars for those in big cities. In this scenario, Gear Power Inc. is primarily using _____ variables of market segmentation. ((geographic))
Coral Soul Inc. manufactures a range of body care products. It has separate product lines dedicated to men and women, and each product line has its own range of shower gels, shampoos, and cosmetics. Which of the following bases of market segmentation has Coral Soul Inc. adopted? ((demographic))
Thirst Tip, a beverage manufacturing company, has placed its products strategically in areas where the population density is high. The segmentation done by Thirst Tip is an example of _____ segmentation. ((geographic))
Uncommon Confections is a bakery that produces all gluten-free baked goods to target customers who have trouble digesting gluten but still want to be able to enjoy tasty treats. What type of segmentation is this company using? ((benefit segmentation))
In businesses, the _____ is an important variable—often the central focus—of the marketing mix, based on which the other variables are adjusted. ((product))
Gift Nation Inc. is deciding whether to sell its products through vending machines outside subway stations or in stores at popular malls. To which of the following elements in the marketing mix does this decision most closely relate? ((distribution))
Den Inc., a firm dealing with fashion clothing, is contemplating whether to sell its products in brick and mortar stores or to sell them at its online store. To which of the following elements in the marketing mix does this decision most closely relate? ((distribution))
All of the following are true regarding products EXCEPT they ((only have favorable attributes.))
_____ is a key element of the marketing mix because it relates directly to the generation of revenue and profits. ((Price))
Which of the following statements is true regarding the distribution aspect of the marketing mix? ((Intermediaries, usually wholesalers and retailers, perform many activities required to move products efficiently from producers to consumers or industrial buyers.))
Which of the following is a social variable of buying behavior? ((culture))
Which of the following is true of marketing research? ((Marketing research is a systematic, objective process of getting information about potential customers to guide marketing decisions.))
Mark runs a home improvement business. He is happy with the level of business he receives, but he wants to learn more about his customers, so he studies information compiled by the U.S. census bureau and other government agencies, databases created by marketing research firms, and sales and other internal reports. What kind of data is Mark using in this scenario? ((secondary data))
_____ refers to the decision processes and actions of people who purchase and use products. ((Buying behavior))
In the context of the psychological variables of buying behavior, _____ is the process by which a person selects, organizes, and interprets information received from his or her senses. ((perception))
Which of the following is a psychological variable of buying behavior? ((attitude))
Every year, Jordan has bought a calendar to keep track of his appointments and events. However, this year, he bought a smartphone, which has a built in calendar, so he did not buy a new paper calendar. Which psychological variable does this scenario most closely describe? ((learning))
_____ are a set of expectations for individuals based on some position they occupy. ((Social roles))
_____ are commonly determined by ranking people into higher or lower positions of respect. ((Social classes))
The groups with whom buyers identify and whose values or attitudes they adopt are referred to as ((reference groups))
Lisa, Stanley, and Kendall have developed a common pattern of behavior. They all own large houses, drive fancy cars, and belong to an elite country club. Lisa, Stanley, and Kendall most likely belong to the same ((social class))
Maria is a Mexican American who manages a Mexican food restaurant. She often eats the food her restaurant sells, and several times a year, she travels to Mexico to visit her family and eat authentic Mexican cuisine. Which social variable does this scenario most closely describe? ((culture))
Which of the following is a social role that a person may have? ((a student))
Which of the following is least likely to be a competitive or economic force in the marketing environment? ((the laws related to employment in a country))
Which of the following best exemplifies regulatory forces in a marketing environment? ((political actions of interest groups))
According to the foreign trade policy of the Republic of Cambria, multinational companies can enter its market only by merging with a local business partner. Which of the following environmental forces does this best reflect? ((political))
Specific laws require that advertisements be truthful and that all health claims be documented. This is an example of _____ forces. ((legal))
Which of the following statements is true of the marketing environment? ((A marketing manager can influence some environmental variables.))
Why does marketing require creativity and consumer focus? ((because environmental forces can change dramatically))
Air Auto has developed a new car that runs on air, instead of fuel or electricity. This is an example of how laws addressing environmental issues can lead to ((opportunities for new products))
The terms accounting and bookkeeping are interchangeable. ((FALSE))
In accounting, a ledger is a time-ordered list of accounting transactions. ((FALSE))
Non-business entities typically obtain revenues through the sale of their assets. ((FALSE))
Accounting refers to the process of ((recording, measuring, and interpreting financial information.))
Which of the following is a function of the Public Company Accounting Oversight Board? ((It makes rules and policies for accounting firms and businesses))
Evelyn is a self-employed, state-certified accountant who files tax returns, prepares financial records, and audits corporate financial records. She is known as a _____ accountant. ((certified public))
All of the following are part of "the Big Four" EXCEPT ((Plante Moran))
Garrett wants to become a certified public accountant. To accomplish this, he must be certified by the _____ in which he plans to provide accounting services. ((state))
The _____ Act required firms to be more rigorous in their accounting and reporting practices. ((Sarbanes-Oxley))
Which of the following is a requirement stated by the Sarbanes-Oxley Act ((This act requires firms to separate their consulting and auditing businesses.))
Stanley is an executive of Fit Corp., a chain of fitness clubs. For years, he successfully hid illegal and misleading accounting practices, but eventually, he was exposed and punished with a jail sentence under the _____ Act. ((Sarbanes-Oxley))
All of the following contributed to the financial crisis in 2008 EXCEPT ((government bailouts))
The _____ Act limits the types of assets commercial banks can buy; the amount of capital they must maintain; and the use of derivative instruments such as options, futures, and structured investment products. ((Dodd-Frank))
_____ accounting involves analyzing financial documents in search of fraudulent entries or financial misconduct. ((Forensic))
Kevin works for the Internal Revenue Service. His main responsibility is rooting out evidence of cooked books at major companies. Kevin is a _____ accountant. ((forensic))
Jordan has been an accountant for many years. Since the accounting scandals in 2000, his firm added fraud-detection services, and Jordan decided to focus on these services. Which of the following organizations could certify Jordan, so he can focus on detecting fraud? ((the Association of Certified Fraud Examiners))
Which of the following types of accountants has titles such as controller, tax accountant, and internal auditor? ((private accountants))
Carleigh is a private accountant who passed a rigorous examination by the Institute of Management Accountants. After passing this examination, Carleigh is a ((certified management accountant))
Dexter works for a publishing company, recording its routine, day-to-day business transactions. His main responsibility is obtaining and recording the information his company needs to analyze its financial position. Based on this description, Dexter is most likely a(n) ((bookkeeper))
Which of the following statements is true of bookkeeping? ((Bookkeeping is much more mechanical than accounting))
_____ is the movement of money through an organization over a daily, weekly, monthly, or yearly basis ((Cash flow))
The principal value of a budget lies in its ((breakdown of cash inflows and outflows))
Deviations between expected operating expenses and _____ serve as a "feedback loop" to launch more detailed financial analyses in an effort to pinpoint trouble spots and opportunities. ((operating revenues))
The single most important component of an annual report is the ((signature of a certified public accountant))
What are the fundamentals of the accounting process? ((the accounting equation and the double-entry bookkeeping system))
Ian got a loan from the Small Business Administration to start his own house contracting business. This loan is a(n) ((liability))
Double-entry bookkeeping is a system of recording and classifying business transactions ((in separate accounts in order to balance the accounting equation.))
Connor is using the double-entry bookkeeping system to account for all the transactions during his first month in business as an electrician. All of the following could be specific asset categories for Connor EXCEPT ((supplier credit.))
Which of the following is the first step in the accounting cycle? ((examine source documents))
In the accounting cycle, after examining source documents and recording transactions in an accounting journal, what is the next step taken by a financial manager? ((posting recorded transactions))
Nina, a manager at a small restaurant, is engaged in conducting the accounting cycle. She begins by examining checks, credit card receipts, sales slips, and other related evidence concerning specific transactions. Which of the following steps should she follow immediately after this step? ((She must record the financial transactions in a journal.))
Nancy owns a bakery. She has already gathered and examined the checks, credit card receipts, sales slips, and other related evidence concerning specific transactions. She has also recorded each financial transaction in a journal and transferred that information into a ledger. What must Nancy do at the end of the accounting period? ((She must prepare a trial balance))
Which of the following is true regarding financial statements? ((One size definitely does not fit all))
An accounting term that is interchangeable with revenue is ((sales))
Cole owns a large chain of grocery stores. A significant part of his job is determining the amount of income left after taxes and whether the company will retain this income or pay it out in dividends to its stockholders. The income in this scenario is _____ income. ((net))
_____ is the total amount of money received from the sale of goods or services, as well as from other business activities such as the rental of property and investments. ((Revenue))
Nonbusiness entities typically obtain revenue through ((donations))
Do-It-Yourself Home Repair Shop started the accounting period with $10,000 worth of shovels in inventory. During the accounting period, it purchased $5,000 worth of shovels and sold $7,500 worth. In this scenario, what is the cost of goods sold? (($7,500))
In the context of expense accounts, salaries of executives and their staff are included under the category of _____ expenses. ((general and administrative))
Hannah has borrowed money from a bank to start her new hair salon and spa. Her direct costs of borrowing this money is known as her _____ expenses. ((interest))
The process of spreading the costs of long-lived assets such as buildings and equipment over the total number of accounting periods in which they are expected to be used is called ((depreciation))
Depreciation, a type of expense account, is included in the _____ category. ((general and administrative))
Derek, an accountant, allocates the cost of a piece of earth-moving equipment over a specific period of time. Based on the given information, it can be concluded that Derek is implementing the process of ((depreciation))
Net income is ((earnings after all expenses have been deducted.))
Pals with Paws, a small pet store, is trying to determine its net income. To figure out this value, the company should equate the total profit or loss minus ((expenses, including taxes))
Which of the following are the results of calculations made from the revenues and expenses accounts? ((gross profit and net income))
Companies record their _____ activities in the revenue and expense accounts during an accounting period. ((operational))
At the end of each accounting period, the dollar amounts in all the revenue and expense accounts are moved into an account called ((retained earnings))
Gamers Inc., a videogame company, has decided to pay dividends to its shareholders. Which of the following accounts will doing so decrease? ((its cash and capital accounts))
Which of the following statements is true of the balance sheet? ((It represents all transactions conducted by an organization since its founding))
Molly is preparing a balance sheet for her pet grooming company. Following long-established traditions, she lists items on her balance sheet on the basis of their ((original cost minus accumulated depreciation))
Which of the following is one of the most common format for balance sheets? ((vertical))
All asset accounts are listed in descending order of ((liquidity))
_____ refers to money owed a company by its clients or customers who have promised to pay for the products at a later date. ((Accounts receivable))
Melanie manages a used car dealership that allows customers to buy cars for no money down and pay in installments throughout the year. Her company builds in a bad-debts adjustment that is deducted from the accounts receivable balance to present a more realistic view of the payments likely to be received in the future for these cars. The payments the company expects to receive are called ((net receivables.))
_____ represent a commitment of organizational funds of at least one year. ((Fixed assets))
All of the following are items that are classified as fixed assets EXCEPT ((temporary investments.))
Which of the following is an asset in the balance sheet? ((accounts receivable))
Tom is completing a balance sheet for his home improvement company. He must include the amounts he owes to suppliers for lumber, tools, equipment, and other materials that he purchased with credit. These liabilities are called ((accounts payable))
An account that represents all unpaid financial obligations incurred by an organization is called ((accrued expenses))
Which of the following categories of cash flow is calculated from changes in the long-term or fixed asset accounts? ((cash from investing activities))
When Pizza Palace decided to open several new locations, it spent billions of dollars on property and equipment. Which category of cash flow does this best describe? ((cash from investing activities))
All of the following are true of ratios EXCEPT they ((almost never provide as much useful information as an absolute value.))
How does ratio analysis relate to the income statement and balance sheet? ((It brings the income statement and balance sheet into sharper focus))
Profitability ratios measure ((how much operating income or net income a firm is able to generate relative to its assets, equity, and sales.))
Which of the following equations is correct? ((profit margin = net income/sales))
A company with a low return on assets ((is not using its assets very productively))
Return on assets and return on equity are examples of _____ ratios. ((profitability))
Asset utilization ratios measure ((how well a firm uses its assets to generate each $1 of sales.))
The _____, an asset utilization ratio, indicates how many times a firm collects its accounts receivable in one year. ((receivables turnover))
Total asset turnover ((measures how well an organization uses all of its assets in creating sales.))
_____ ratios compare current assets to current liabilities to indicate the speed with which a company can turn its assets into cash to meet debts as they fall due. ((Liquidity))
Which of the following is a debt utilization ratio? ((times interest earned ratio))
If a company is relying on borrowing and credit too extensively, this will probably be reflected in the _____ ratio. ((debt utilization))
Alma is an investor who wants to compare the performance of one publishing company with another on an equal, or per share, basis. What is a general finding associated with this practice? ((The more shares of stock a company issues, the less income is available for each share.))
_____ are profits of a corporation that are distributed in the form of cash payments to stockholders. ((Dividends))
A(n) _____ is a legal entity, created by the state, whose assets and liabilities are separate from its owners. ((corporation))
_____ are typically owned by many individuals and organizations who own shares of the business, called stock. ((Corporations))
Which of the following statements is true about corporations? ((They account for the majority of all U.S. sales and income))
Local Lumber, a small lumber company, decides to incorporate. It wants to have greater flexibility than traditional corporations. What type of corporation would work best for this company? ((S corporation))
All of the following are true of creating a corporation EXCEPT ((the procedure for chartering a corporation is the same in all states))
Alan, Jason, and Greg want to incorporate their small roofing business. To do so, they must file legal documents with the appropriate state office. What are these legal documents called? ((articles of incorporation))
Which of the following items is found in the Model Business Corporation Act issued by the American Bar Association? ((provisions for the regulation of internal corporate affairs))
_____ are legal documents that the state issues to companies based on information the company provides in the articles of incorporation. ((Corporate charters))
A corporation doing business outside the state in which it is chartered is known as a(n) ((foreign corporation))
A corporation doing business in the state in which it is chartered is known as a(n) ((domestic corporation))
Mega Mart, Inc. is a large grocery chain that was incorporated in the United States. Over the years, the corporation has grown, and it has now opened locations in five other nations. In those nations, Mega Mart would be considered to be a(n) ((alien corporation))
Which of the following statements is true of private corporations? ((They are owned and managed by just one or a few people))
Which of the following is a defining characteristic of public corporations? ((Their stock can be bought, sold, or traded by anyone))
GloboTron is a large company that produces home entertainment technology. GloboTron has stock that anyone can buy, sell, or trade, but since it is a large company, its stockholders are far removed from its management. However, it must disclose its financial information to the public under specific laws that regulate the trade of stocks and other securities. What type of business is this? ((public corporation))
Organizations such as the National Aeronautics and Space Administration and the U.S. Postal Service that are owned by the federal, state, or local governments are known as ((quasi-public corporations))
Assure Pharma Corp., a private corporation, sold its stock to raise the money it required for global expansion. As a result, anyone could buy, sell, or trade the stocks of the company. The process adopted by Assure Pharma Corp. to raise money is referred to as ((initial public offering))
Which of the following is a corporation that provides a service, but is neither owned by the government nor focuses on earning profits? ((a nonprofit corporation))
SunRay Foundation, a corporation owned and managed by volunteers, supplies medicines and vaccines to impoverished people in Asia and Africa. Its focus is not on earning a surplus but on serving the community. SunRay Foundation is an example of a(n) ((nonprofit corporation))
All of the following are true of 501(c)(3) organizations EXCEPT ((they must answer to their shareholders))
A _____ is elected by the stockholders of a corporation to oversee its general operation ((board of directors))
Which of the following is true regarding the role of the board of directors? ((Board members have a duty of care and loyalty to oversee the management of the firm))
Amanda is a professor at a local college who serves on the board of directors for EduPub, a leading academic publisher. She is a valued member of the board because she brings her unique perspective from the field, and she is qualified, knowledgeable, and independent of the company. Amanda is a(n) ((outside director))
Common stockholders within a corporation ((receive dividends after preferred stockholders))
Which of the following statements is true of preferred stockholders? ((They have a claim to profits before other stockholders do))
Which type of stockholders usually has the right to vote and elect the board of directors? ((a common stockholder))
The right of common stockholders to have the opportunity to purchase new shares of stock is called a ((preemptive right))
Ken wants to buy stock in AgroCorp, a large agricultural company. Since he lives in the city where the company is headquartered, he wants to attend their annual meetings and vote for the board of directors. He also wants to maintain a 5% ownership of the company, so having priority in purchasing new shares on the marketplace is very important to him. What type of stock should he purchase? ((common stock))
The form of business ownership that has the ability to raise capital most easily is a ((public corporation))
Which of the following is an advantage that corporations provide as a form of business ownership? ((It is easy to transfer ownership in corporations.))
Which of the following is a disadvantage of corporations? ((The formation of a corporation can be costly and it faces double taxation))
Which of the following is an advantage of corporations over sole proprietorships? ((Unlike sole proprietorships, existence of corporations is unaffected by the death or withdrawal of any of the stockholders.))
Which of the following is an advantage of a corporation over a partnership? ((As a legal entity, a corporation can enter into contracts without as much difficulty as a partnership.))
Megan works for a large corporation, and sometimes, she finds it difficult to see how she fits into the corporate picture. However, recently, her company has established an employee stock ownership plan. Which of the following is a benefit of this type of plan? ((It can boost employee productivity.))
A _____ is a partnership established for a specific project or for a limited time. ((joint venture))
Which of the following statements is true of joint ventures? ((They are popular in situations that call for large investments))
A(n) _____ is a form of business ownership that is taxed as though it were a partnership, is popular among entrepreneurs, and represents almost half of all corporate filings. ((S corporation))
Which of the following is true of S corporations? ((Their shareholders can shift income and appreciation to others.))
In an S corporation, net profits and losses of the corporation pass to the owners. What effect does this have? ((It eliminates double taxation))
Which of the following forms of business ownership provides limited liability but is taxed like a partnership? ((a limited liability company))
Which of the following is a difference between limited liability companies (LLCs) and S corporations? ((LLCs are very simple to run, whereas S corporations are not))
A similarity between limited liability companies and S corporations is that both ((offer limited liability to owners.))
An organization composed of individuals or small businesses that have banded together to reap the benefits of belonging to a larger organization is known as a(n) ((cooperative.))
North Farm Society is an association of farmers from rural communities. Its agenda is not to make profits but to help its members market their produce and purchase large quantities of fertilizer, seeds, and other supplies at discounted prices. Thus, North Farm Society is an example of a(n) ((cooperative.))
Which of the following is the best definition of a merger? ((when two companies combine to form a new company))
Big Lite Inc. purchases Canton Bulbs, its direct competitor, by buying most of its stock. In this scenario, Big Lite Inc. is involved in a(n) ((acquisition))
Big Sandwiches merges with its supplier, FC Ingredients, in an effort to have a constant supply of sauce for its sandwiches. This is an example of a ((vertical merger))
Razor, Inc. and Tyros, Inc., two automobile companies, merge to reduce the number of corporations competing within the automobile industry. This merger is an example of a ((horizontal merger))
United Movies Inc., a movie production company, and Fox Apparel, a premium clothing brand, joined their businesses together. When these two companies of different industries join together, it is an example of a ((conglomerate merger))
Auto Mart, a large auto parts distributor, is attempting to acquire Rubber Meets the Road, a tire manufacturer. However, Rubber Meets the Road does not want to be taken over, so it issues a poison pill to try to head off the hostile takeover attempt. What does this entail? ((allowing stockholders to buy shares of stock at prices lower than the market value))
A corporate raider is trying to take over Big City Financial. In an attempt to avoid this hostile takeover, Big City Financial is requiring a large majority of stockholders to approve the takeover. Which of the following methods of evading a takeover attempt does this describe? ((a shark repellant))
In the context of corporations, what is a white knight? ((a more acceptable firm that is willing to acquire the firm threatened by a hostile takeover))
Tim and Andy, wealthy industrialists, borrow money from True Global Bank to acquire Univo Corp. They promise to repay the bank using the assets of Univo Corp. as collateral. In this scenario, Tim and Andy are involved in a ((leveraged buyout.))
Entrepreneurship is the process of creating and managing a business to achieve desired objectives. ((TRUE))
One of the most significant drawbacks of small businesses is their inability to innovate and to bring significant benefits to customers. ((FALSE))
Retailing attracts entrepreneurs because gaining experience and exposure in retailing is relatively easy. ((TRUE))
Industries such as wholesaling, services, and manufacturing are unattractive to entrepreneurs because these fields tend to be relatively difficult to enter and require high initial financing. ((FALSE))
A drawback of franchising is the need for franchisees to strictly adhere to standardized operations. ((TRUE))
All of the following are ways innovative entrepreneurs can use emerging trends to provide products that serve customer needs EXCEPT ((inventing a major new technology.))
Megha is a taekwondo expert who starts her own martial arts training academy. She conducts taekwondo classes for women and teaches them the art of self-defense. Megha can be regarded as a(n) ((entrepreneur))
Carol and Evan recently had their first child. As two working parents, they struggled to find good, economical child care. As a result, they developed their own software to link parents with licensed, fairly priced child care providers. This is an example of the ((sharing economy))
A _____ is best described as any independently owned and operated business that is not dominant in its competitive area and does not employ more than 500 people. ((small business))
Michael owns a small catering business that mostly serves his community. He has been hired for company picnics, backyard barbecues, and family celebrations. As word of mouth has spread about his delicious food, good service, and fair prices, he has started getting more business. To handle this, he has hired four employees to help cook and serve food at these events. What kind of entrepreneur is Michael? ((a microentrepreneur))
The Small Business Administration is a(n) ((independent agency of the federal government that offers managerial and financial assistance to small businesses))
Which of the following is true about the role of small businesses in the American economy? ((They provide opportunities for women to succeed in business))
Which of the following businesses has created a majority of net new jobs annually? ((small businesses))
Strings and Keys, an independent music store, acquires instruments and other music equipment from producers and wholesalers and sells them to consumers. Strings and Keys is a ((retailer))
Megan and Chris are entrepreneurs who want to start their own business. Which of the following would attract them to retailing? ((Retailing requires low initial financing))
Many opportunities exist for _____, which involves selling products outside of a retail facility. ((nonstore retailing))
Carolyn makes unique clothing pieces, jewelry, and hair accessories. To avoid the costs of opening a store, she sells these pieces on Etsy. This is an example of ((direct marketing))
Clothes Horse, a clothing store in the mall, wants to give customers an opportunity to place orders. What is one form of direct marketing the store can use to provide this service? ((It can send catalogs to its customers))
While Tyler attends college, he is working part-time for a life insurance company. He goes door-to-door and offers consumers face-to-face presentations in their homes. This is an example of ((direct selling.))
All of the following are true of direct selling EXCEPT ((it requires buying a lot of inventory))
Which of the following best illustrates a small business operating in a retail industry as opposed to wholesaling its products? ((Neon Love Inc. imports candles and directly sells them to customers through kiosks.))
Which of the following is true of wholesalers? ((Wholesalers are extremely important because of the marketing activities they perform.))
Many service providers are considered retailers because they ((provide their services directly to ultimate consumers))
In the manufacturing sector, small businesses sometimes have an advantage over large firms because small businesses ((can customize products to meet specific customer needs and wants.))
Which of the following is a difference between high technology businesses and other small businesses? ((High technology businesses require greater capital and have higher initial startup costs than other small businesses.))
Which of the following is a challenge associated with the sharing economy? ((pressure over whether workers are independent contractors or employees))
Which of the following traits will help entrepreneurs succeed? ((persistence))
One of the major reasons people want to own and operate their own business is to ((be their own boss))
Which of the following is an advantage of a small business? ((greater adaptability to changing market demands))
Which of the following statements is true of organizational culture? ((It helps ensure that all members of a company share values.))
_____ are responsible for tactical and operational planning that will implement the general guidelines established by top management. (( Middle Managers ))
Management takes place only in business settings. ((False))
_____ refer to the ability of an individual to think in abstract terms and to see how parts fit together to form the whole. ((Conceptual skills))
is the process of evaluating and correcting activities to keep an organization on course ((Controlling))
Dividing work into small units and assigning it to specific individuals is a task related to the management function of ((organizing))
Planning involves forecasting events and determining the best course of action from a set of options or choices. ((True))
include the president and other top executives, such as the chief executive officer, chief financial officer, and chief operations officer, who have overall responsibility for an organization ((top managers))
In an organization, decisions regarding adding new products, acquiring companies, and moving into foreign markets would most typically be made by ((the top management))
_____ occurs when employees are provided with the ability to take on responsibilities and make decisions about their jobs. ((empowerment))
_____ occurs when employees are provided with the ability to take on responsibilities and make decisions about their jobs. ((employee empowerment))
_____, a type of plan for meeting objectives, establish the long-range objectives and the overall course of action by which a firm fulfills its mission. (((Strategic plans)))
For any company, owners and shareholders are the only sources of primary funding. ((False))
Managers need adequate _____ resources to pay for essential activities. ((financial))
Positions such as foremen, supervisors, and office service managers are a part of the _____ of a company. ((first-line management))
How long a time period do strategic plans generally cover? ((one year or longer))
The equilibrium price is represented by the point where a product's supply and demand curves intersect. ((TRUE))
While ethical behavior can enhance a company's reputation, it often has a negative effect on the company's financial performance. ((FALSE))
Businesses differ from nonprofit organizations in that a business's focus is on ((profit.))
All the groups that have an interest in an organization's success and outcomes are known as the organization's ((stakeholders.))
Which is the BEST example of a business trying to balance the needs of various stakeholders? ((Pete's Pest Control Company is trying to develop an eco-friendly insecticide that will eliminate crop-killing pests and still adhere to environmental regulations.))
Ronan works for a custom furniture manufacturer. His job is to oversee the process of taking raw materials—lumber, fabric, and metal components—and ensure that they are transformed to the customer's exact specifications. Ronan's job falls under the business activity known as ((operations))
A firm's ________ resources are also known as labor ((human))
Oscar is a real estate entrepreneur in a country that promotes the individual right to own property, earn profits, make decisions about business operations, and choose productivity systems. Which system best describes Oscar's business environment? ((free enterprise))
In which market structure do individual businesses have control over their products' price because each business supplies a large portion of the products sold in the marketplace? ((oligopoly))
Which statement is true about GDP as a measure of a country's economic health? ((It measures only those goods and services made within a country.))
When it comes to ethical behavior, co-workers and superiors exert significant control over one's choices at work through authority and example. ((TRUE))
Which of the following best exemplifies a company with a strong ethical culture? ((The top management at Randy's company clearly communicates to all employees the ethical behavior expected of them.))
A local landscaping company donates and plants a tree in the community for every new client that uses its landscaping design services. This exhibits the company's ((social responsibility.))
The term ________ refers to a business's obligation to maximize its positive impact and minimize its negative impact on society. ((social responsibility))
How do the terms business ethics and social responsibility differ from each other? ((Business ethics relates to an individual's or a work group's decisions that society evaluates as right or wrong, whereas social responsibility concerns the impact of the entire business's activities on society.))
Tracor Industries decided to buy less expensive materials from a foreign supplier that resulted in a significant reduction in the quality of its product. As a result, company sales dropped, many retailers refused to carry the product, and consumers left negative reviews online about the product. This is an example of ((how negative judgments can affect an organization's ability to achieve its goals.))
After a major horse race, it was discovered that the winner had drugged his horse to make it run faster. Even though there was not a specific rule against drugging horses, the horse's owner, trainer, and jockey were banned from the racing industry. This exemplifies how ((negative judgment directly affects an organization's ability to achieve its business goals.))
Daniel notices that one of his colleagues arrives late and leaves early from work every day. In deciding whether or not to report this behavior, what must he do first? ((know his company's policy regarding the ethics of his colleague's choice))
Which act is associated with bullying? ((discrediting others' ideas and opinions))
Marissa needs William's help to resolve an urgent work issue. She has emailed and called him several times, but William doesn't respond to any of her communications. William's behavior is an example of ((bullying.))
A hairdryer manufacturer produced a new model that could dry a person's hair in under five minutes. When customers used this hairdryer and experienced scalp and hand burns, the manufacturer was required to create and enforce detailed plans to prevent future burns. This example relates to ((fairness and honesty.))
________ involves taking someone else's work and presenting it as your own. ((Plagiarism))
If a manager attempts to take credit for a subordinate's ideas, this is a form of ((plagiarism.))
Lance's boss often goes on Facebook at work. As a result, the rest of his team goes on Facebook as well. This is an example of which factor of influence regarding ethical behavior? ((the influence of managers))
Kian's company, Microtech, does not have established rules and policies on ethics. It is a small, independent company that depends on its employees to use their own judgment. This structure can lead to ((opportunity for misconduct.))
According to the National Business Ethics Survey (NBES), ________ is the greatest determinant of future misconduct. ((a company's ethical culture))
________ is the act of an employee exposing an employer's wrongdoing to outsiders. ((Whistleblowing))
Obeying the law is a business's ((legal responsibility.))
Master Corporation created its own continuing education program that offers eligible employees full tuition to pursue continuing education related to their roles at the company. It also created a scholarship program to help employees send their children to college. These programs exemplify the company's ((philanthropic activities.))
All of the following are criteria the Ethisphere Institute uses to select its annual list of the world's most ethical companies EXCEPT ((innovation that contributes to the company's well-being.))
The assurance of both satisfactory quality and service at a fair price is a part of the consumer's right to ((choose.))
John F. Kennedy's consumer bill of rights outlined four rights: the right to be informed, the right to choose, the right to be heard, and the right to ((safety.))
Braxton Foods, Inc. started using a new preservative in its products. However, it didn't include anywhere on its packaging that this preservative could cause an allergic reaction to people who are allergic to milk products. This is a violation of the consumers' right to ((be informed.))
What is true about the role of the Federal Trade Commission's Bureau of Consumer Protection? ((It protects consumers against unfair, deceptive, or fraudulent practices.))
Beth has realized that she does not like working for others. She wants to open a business in which she will have maximum control and the least interference from government regulations. Which form of business is best suited for Beth's needs? ((a sole proprietorship))
All of the following are ways technology is helping today's sole proprietors succeed EXCEPT ((landline telephones allow them to make free long-distance calls to customers, suppliers, and others.))
Angelica owns a small hair salon. When a chain hair salon opened across the street from her business, Angelica lowered her prices and added more specialty services, like manicures and facials. Her ability to make these decisions on the spot without anyone else's approval gave her a ((competitive advantage.))
Mel owns an auto repair shop. He started the business with a personal loan and is wholly responsible for repaying it. Mel also needs to pay personal income tax on the profits generated from the business. Mel's business is an example of a(n) ((sole proprietorship.))
Victor is asked to be a part of a new business venture for a revolutionary mechanism to repair Internet cables that are placed on the ocean floor. Since the business involves high risk, Victor does not want to be held completely liable for the firm's debts if the project fails. In this case, Victor is most likely to prefer to participate as a(n) ((limited partner.))
David, Shannon, and Greg are starting a new accounting firm together, and they are drafting articles of partnership. All of the following are items they should include EXCEPT ((the total money and assets of each partner's estate.))
Ken is a partner in a local hardware store. He decides to sell his partnership interest. What is one challenge that Ken will face? ((It may be difficult for the partners to place a value on Ken's share of the partnership.))
Sarah is a partner in a local coffee shop. Without consulting her partners, she ordered some expensive, specialty roasted coffee beans that she thought would greatly improve their business. However, their customers felt the coffee had a bitter taste and are no longer buying it. What impact will Sarah's decision have on her partners? ((Her decision may put her partners' personal resources in jeopardy.))
Frank, Martin, and Paul secure a charter for their business. They hold an organizational meeting to establish the corporation's bylaws and elect a board of directors. What is one thing the bylaws might do? ((set up committees of the board of directors))
Brewer Bikes, Inc. is a company that sells bicycles and accessories. It has been owned and operated by the Brewer family for 25 years. The Brewers own all the corporation's stock, and they are not required to disclose financial information publicly. What type of business is this? ((a private corporation))
Summer Foods is a public corporation that is experiencing difficulty growing its vegetable products during an especially dry summer season. Wanting the flexibility to make decisions for restructuring the company's operations, its managers buy all the stock, which means the company is ((taking the company private.))
Jeff and Maggie both work in the real estate market. One day, a great investment property comes on the market, and they decide to pool their money to buy and renovate it. They plan to spend six weeks on the renovation, and they hope to make a nice profit when they turn around and sell it. Jeff and Maggie's partnership is an example of a(n) ((joint venture.))
April is a doctor who is forming her own medical practice. She wants her practice to be flexible, simple to run, and free from required meetings, minutes, or resolutions. Her goal is to blend the best characteristics of corporations, partnerships, and sole proprietorships. What form of business ownership seems most in-line with April's goals? ((limited liability company))
When a corporate raider wants to acquire or take over another company, it first offers to buy some or all of the other company's stock at a premium over its current price in a(n) ((tender offer.))
While growing up, Carlos noticed that many families in his community struggled to make ends meet and provide for their children's education. As a result, he started a business to provide low-cost tutoring and scholarship opportunities for low-income families in his community. As his company has grown, Carlos has been able to extend free tutoring and college scholarships to five impoverished students per year. Carlos is a ((social entrepreneur.))
Robert's Music Center is a local music shop. Along with selling musical instruments, it also rents instruments to students in school band programs. Robert is the owner and he employs three full-time and five part-time employees. Robert's business is a ((small business.))
Dante runs Night Moves, his own DJ and party planning company. He has built a good reputation for organizing weddings and corporate events and his business has grown. He recently moved his business from his home to an office located in a corporate building and has increased his staff to 40 employees. Dante's company can be categorized as a ((small business.))
Mahie owns an Indian restaurant that employs around 20 people. She is solely responsible for the management of the restaurant. Her business competes with a larger chain of restaurants that offers the same cuisine. Mahie's restaurant can be classified as a ((small business.))
Oliver came up with the idea of producing mayonnaise in neon colors. He was sure that kids would love the idea and this would become the hottest product on the market. Unfortunately, the neon-colored mayonnaise wasn't such a hit. What is the most likely cause of Oliver's product failure? ((poor business concept))
Elana recently opened a fashion boutique. She is successful in creating a strong brand image and a loyal customer base. In order to meet the increasing demands of her business, she hires a store manager. However, she finds it hard to let the manager make important decisions about the store. She often finds herself disagreeing with the manager's decisions, and in the process, she loses several existing customers. Which cause of small-business failure does this scenario best illustrate? ((inability to cope with growth))
Brenda is starting her own small home decorating company. She is developing the company's business plan. What is something the business plan should accomplish? ((estimating the business's income and expenses))
The act of financing one's business by using real personal assets is known as ((equity financing.))
Tamika borrows $40,000 from her father for her new business venture. She promises that he will be a partner in the business and be entitled to a significant percent of the business's profits at the end of the year. This is an example of ((equity financing.))
Fiona's small catering business requires more financing than she can provide from her savings. As a result, she is looking for investors to provide financing in exchange for stock ownership. What is a disadvantage of this arrangement? ((Fiona will have to share the profits of her business.))
Antonio is an interior designer who starts his own consulting firm. His friend Vanessa is the owner of a coffee shop. Antonio decorates Vanessa's coffee shop in exchange for a supply of coffee for his office. Which source of funding does this scenario best illustrate? ((bartering))
Noel decides to buy an existing home goods store, instead of starting his own from scratch. After he takes over the business, he learns that several customers have complained that one of the company's distributors does not pack items sufficiently for shipping, and their items arrived broken. What is Noel's responsibility in resolving this problem? ((Noel is responsible for taking on and resolving any problems the business already has.))
A license to sell another's products or to use another's name in business, or both, is called a(n) ((franchise))
Peter has recently quit his job and plans to open a restaurant so he can be his own boss. He has two options: he can either start his own new restaurant from scratch or purchase a franchise from an already established restaurant or fast-food chain. His wife Linda supports the former plan. Which statement strengthens the argument in favor of Linda's choice of opening a restaurant independently? ((It is easier and more flexible to make and execute decisions in an independent business.))
Jess is a human resource (HR) manager at a company. A survey of the company's employees reveals that more than 80 percent of the employees belong to the demographic segment Generation Y. Based on this, which HR strategy should Jess implement in order to best serve the needs of the employees? ((Jess should use recognition and advancement as the driving forces to motivate employees.))
Samuel is a human resource manager at a large multinational company. After a drastic drop in revenue, his company thinks of ways to correct the situation. At a meeting with the top management, the CEO suggests that the company should reduce its workforce by 20 percent. This move would help the company cut costs and continue functioning on the current projects. In this scenario, which strategy is Samuel's company planning to use? ((downsizing))
Top-level managers make an organization's strategic decisions that focus on a key idea for using resources in order to take advantage of opportunities. ((TRUE))
________ make decisions about the use of an organization's resources and are concerned with planning, organizing, directing, and controlling the organization's activities. ((Managers))
Wes is a manager at a large investment banking firm. The firm is expanding, and he is tasked with hiring more people to carry out the organization's increased workload. Wes must also determine what to pay the new hires and develop a plan to train them. Wes is engaged in ((staffing.))
Determining an organization's objectives and deciding how to accomplish them is the management function known as ((planning))
Marla manages an online shoe store. She must determine what her company's warehouses and distributing facilities need and which type of automation can maximize order efficiency. What major phase of the planning process does this example describe? ((data gathering))
A declaration of an organization's fundamental purpose and basic philosophy is known as a(n) ((mission statement.))
Jake starts his own catering business. He creates a website for the business, which states that his purpose is to provide his customers with delicious food at a fair price. For Jake's company, this is its ((mission statement.))
Vince starts his own tutoring business. He wants to write a good mission statement that explains his company's reason for existence. What is one guideline he should follow in writing his mission statement? ((It should be clear and concise.))
Naveen is part of the planning team in his organization that is working to develop the firm's mission statement. The team knows that along with describing the business and identifying its customers, the team should also articulate the business's operating philosophy and list any core competencies the business has. What other item should Naveen and his team include in the mission statement? ((the firm's responsibilities with respect to being a good steward of environmental, financial, and human resources))
Joey's jewelry business has grown from a small startup in his basement to a chain of outlets all over the state. His business has the potential to expand further, provided he has the resources. To attract investors and other resources, he creates a mission statement for his company. He highlights that his gems come from countries with fair labor practices. Which basic question of a well-developed mission statement is he addressing? ((What are our responsibilities with respect to being a good steward of environmental, financial, and human resources?))
A firm's ________ should contain three key components: an attribute sought, a target to be achieved, and a time frame. ((goals))
Howie's hot dog stand wants to earn a profit and satisfy its customers. As a result, it has developed several signature hot dogs that use quality ingredients and mix inventive flavors. What key component of a goal is this example most closely related to? ((an attribute))
The ends or results desired by an organization that are derived from the organization's mission are referred to as ((objectives.))
Melvin's fishing store has profit as an objective. As a result, he wants to have ((money and assets left over after paying off business expenses.))
Nellie's Sweet Shop, a large candy store, has historically sold a lot of chocolate and gummy candies. However, over the years, people have become more health conscious. As a result, Nellie's develops a new line of sugar-free candies that taste as good as the original but without the guilt. What type of objective does this exemplify? ((growth))
The top management of Darryl's Detailing Center decides that the company's objective for the next two years will be to expand the overall business to include locations in other states. This is an example of ________ planning. ((strategic))
School Works, a large publisher of educational resources, decides to sell its high school division because it isn't as profitable as the elementary school division. This will allow the company to focus its efforts and funds on becoming the leading elementary school publisher. What type of plan is this? ((strategic))
Short-range plans covering a period of one year or less that are designed to implement the activities and objectives specified in an organization's strategic plan are referred to as ((tactical plans.))
What is an effective practice with regards to tactical plans? ((They should be periodically reviewed and updated by an organization's management.))
) Live Wire Collectibles, a company that manufactures action figures inspired by popular video games, creates a strategic plan to be the leader in the product category within a period of three years. The company's middle management also creates a six-month plan to ensure that the objectives of the strategic plan are properly implemented. The six-month plan created by Live Wire Collectibles can be regarded as a(n) ((tactical plan.))
The top management of Clear Chemical Inc. creates a strategic plan to enter new product markets within a period of five years. The company's middle management creates six-month plans to ensure that the objectives of the strategic plan are properly implemented. Along with these plans, each department creates short-term plans that specify the actions to be taken by specific employees and teams to meet the objectives of the strategic plan and the six-month plans. These short-term plans established at the departmental level are referred to as ((operational plans.))
Stan works in the supply warehouse for a large manufacturing facility. Each day, Stan is given a production quota. This ensures that enough materials will be available for the next day's production run, so the company can increase its product availability and be able to ship products faster than any other company. A production quota is an example of a(n) ((operational plan.))
Crisis management plans generally cover maintaining business operations during a crisis and ((communicating with others about the situation and the company's response to the crisis.))
A huge fire broke out in Fancy Fixtures, a small home decorating store. The fire destroyed the store's inventory, and ultimately, the company went out of business. What is most likely the reason this company went out of business? ((Fancy Fixtures didn't have an updated contingency plan to handle such a crisis.))
Pamela manages a large blood donation facility. One of her roles is structuring the company's resources and activities to accomplish its objectives efficiently and effectively. What management function does this example pertain to? ((organizing))
Kelli's kitchen appliance company is reorganizing for greater efficiency. How would this company most likely organize the work? ((into teams))
) Kishan is a top-level manager at an accounting firm. He has many years of varied experience and he spends most of his time making strategic decisions about how to use the company's resources. On which management function does he spend most of his time? ((planning))
Norah was less educationally qualified and experienced than her colleagues when she joined Micro Systems Inc. However, due to her ability to negotiate effectively with clients, convince her team members to work harder, and understand their needs, she is a senior-level manager today. Which skills can be most attributed to Norah's success in this scenario? ((human relations skills))
Wayne is a manager of a large pharmaceutical firm. He often meets with consumer groups to discuss the safety of the company's new drugs. When he does this, he is acting as a ((spokesperson.))
Sean is interviewing for a position at a law firm. In the interview, he is asked: "How would you resolve the issue of successfully arguing a case in which you disagree with our client's position?" Answering this question requires Sean to use ((analytical skills.))
Which trait would be part of a person's human relations skills? ((the ability to understand the needs of others))
Pierre is a manager at a women's clothing store. He is very particular about how things are done, right down to how the shirts are folded and the positioning of the mannequins. He uses his authority and economic rewards to get his employees to comply with his directions. What type of manager is Pierre? ((autocratic))
Sounds Deluxe, a stereo and audio electronics company, encourages its employees to work together to make decisions that reinforce the company's mission and values. Each employee brings his or her own unique insights, and by discussing these insights with other employees, they collectively develop innovative ideas or decisions that most likely would not have been reached by one or two people. This scenario explains ((teamwork.))
Aiden is assessing the appropriateness of a proposed option to resolve his company's product safety issues. When considering the consequences of the option, he should ((consider its impact on the organization as a whole.))
An organization's structure determines how well it makes decisions and responds to problems, and influences employees' attitudes toward their work. ((TRUE))
Disadvantages of product departmentalization are that it duplicates functions and does not focus on an organization's overall objectives. ((TRUE))
An organization's ________ refers to its shared values, beliefs, traditions, philosophies, rules, and role models for behavior. ((culture))
Joylene is interviewing to work at a dentist's office. During the interview process, the hiring manager had her come in and talk to some of the employees about the office's culture. ((her job will consist of answering phones, checking patients in and out, and filing patient information.))
Which is a means of informally expressing an organization's culture? ((employee dress codes))
Which is a formal expression of an organization's culture? ((codes of ethics))
Christy works for a magazine publishing company. When pressed to meet a deadline, every employee is expected to work together to get the job done, even if it means working overtime. This commitment to teamwork is part of the company's ((culture.))
Which item is least associated with an organization's culture? ((its structure))
The managers of Trendy, a large department store, decide to have a sale. The store manager works with the advertising department to make the public aware of the sale, with department managers to schedule enough salespeople to handle the increased customer traffic, and with merchandise buyers to ensure that enough merchandise is available to meet expected consumer demand. What does this scenario say about organizational structure? ((An organizational structure develops when managers get people to work together to achieve the firm's objectives.))
Kris manages a frozen pizza company. In order to earn profits, she must determine what activities are required to achieve the company's objectives. All of the following are activities this company would need to do EXCEPT ((talk to competitors to make sure their recipe is unique.))
Judy works at a small printing company that groups jobs by functional activities. Her company has departments focused on sales, marketing, finance, printing, legal, and human resources. What is most likely true of the departments in Judy's company? ((Each functional department is managed by an expert in the work done by the department.))
Comfort Food Reimagined is a new restaurant that prepares childhood favorites in new ways to appeal to adults. The restaurant's owner has empowered Bill, an employee, to create a new advertising campaign to attract customers. What does Bill need to carry out this assignment? ((the authority to make decisions about the advertising materials and costs))
An advantage of decentralization is that it ((may increase the organization's productivity.))
Nicole is a manager of an online clothing store. She has five employees who work remotely in their homes in Minneapolis, Milwaukee, Chicago, Detroit, and Cleveland. In addition to managing her employees and having a phone meeting with each of them weekly, Nicole also orders merchandise, manages the inventory, coordinates shipping to customers, and manages the company's advertising and finances. What type of span of management does this scenario describe? ((narrow))
Franco is a manager at a large electronics store, where his main responsibility is managing 25 employees. His employees are highly competent, so he doesn't need to interact with them frequently and few problems arise. His company has a set of specific operating procedures that govern his activities and the activities of his employees. What type of span of management does this scenario describe? ((wide))
Dilbert's, a large department store chain, has store managers, district managers, regional managers, and functional managers. What type of organization does this describe? ((all))
Chuck manages a hardware superstore, where he supervises 45 employees. Because he manages a large number of employees, fewer management layers are needed, and thus, his organization is flat with a wide span of management. As a result, what is most likely true of Chuck's duties, in comparison with managers in tall organizations? ((He performs more administrative duties.))
Vince is the owner of a popular restaurant. He hired a manager who takes care of the day-to-day operations of the restaurant and reports to Vince at the end of each week. The manager is supported by an assistant manager, who supervises the rest of the staff in the restaurant. Thus, Vince's restaurant is using a ________ structure. (( line))
Lucie and Cara are both managers at a home improvement store. Lucie oversees the flooring, lighting, appliances, and hardware departments. Cara provides each of these departments with advice and support in the areas of finance and human resources. What is true regarding Lucie and Cara's roles in the company? ((Lucie is a line manager, while Cara is a staff manager.))
Rob's company has grown in recent years, and the firm is looking to restructure the organization into larger groups. Rob and his team are deciding whether to organize its departments on the basis of geography, customer type, or product. Regardless of which basis the firm uses, it will be organized using which structure? ((multidivisional))
Lacey is taking a college business class. Her professor splits the class into work teams to complete a project. Each team must create a fictional company, create a business plan for a new product, and market that product in a class presentation. What is true of Lacey's work team? ((Team members share leadership and create collective work products.))
William's company uses internal networks to share information with employees. What is an advantage of using this type of network? (( It can help with the flow of everyday business activities.))
The vice president of research and development at an automobile company wants to plan and coordinate the launch of a new car with the company's vice president of marketing. Any related communication between them will be referred to as ________ communication. ((horizontal))
Mary, a project manager at a software company, informs her subordinates of the organization's new policies and code of conduct. This is an example of ________ communication. ((downward))
Which statement is true regarding the monitoring of communications in the workplace? ((Failing to monitor employees' use of e-mail, social media, and the Internet can be costly.))
According to Maslow's hierarchy, a person buying insurance with income protection is attempting to satisfy physiological needs ((FALSE))
An inner drive that directs a person's behavior toward goals is called ((motivation.))
Companies that are concerned with boosting workplace morale, maximizing employees' productivity and creativity, and motivating employees to be more effective are concerned with ((human relations.))
Martin is an independent contractor in the plumbing industry. He receives work from several companies and individual customers. There are often multiple ways to fix a plumbing problem, and when given the option, Taylor sometimes chooses to use the fastest and easiest solution, rather than the best possible solution. What might explain Martin's tendency to avoid doing what he should do? (( lack of motivation))
Lately, Frank has trouble starting his car, making him late for work. Today, his car won't start at all. Since there is no public transportation for him to get to work, he needs to get a new car. After shopping and comparing models, Frank is now ready to sign the paperwork on a new SUV. What is the "need" in this scenario? ((requiring a car for work and not having one that functions))
Jon is a salesman for a security company and has been working in his field for over 20 years. Recently, he noticed that his sales haven't been as high as the sales of some of his younger co-workers. He recognizes the need to increase his sales. What is something he might do to satisfy this need and achieve success? ((obtain additional training to improve his sales skills))
Natasha is a cake designer. She often creates custom cakes for weddings, birthdays, anniversaries, and other large events. She is emotionally involved and committed to her work, which allows her to meet her customers' high expectations. Natasha can be described as ((engaged.))
Culinary House, a cookbook publisher, has noticed that since more people are getting their recipes from the Internet, it is struggling to stay profitable. As a result, the company has decided to streamline its operations, laying off 50 employees and changing the way the company works to focus more on offering innovative digital cookbook applications. Many of the company's employees are frustrated with the changes, so in addition to the employees the company let go, several other employees have quit. This scenario is most closely related to ((low morale.))
Marco works at a grocery store. He started six months ago as a bagger, but recently, his boss recognized his hard work, promoted him to cashier, and gave him a raise. Marco's promotion and raise are examples of ((extrinsic rewards.))
Nancy works in a factory that produces dolls and other toys. To improve productivity, Nancy's manager broke down each job into its component tasks, determined the best way to perform each task, and specified the output to be achieved by each worker performing the task. As a result, Nancy is expected to add arms and legs to 50 dolls per day. She is paid $2.50 per doll, but if she exceeds 50 dolls in a day, she receives an additional $1.50 per doll. This scenario relates to ((the classical theory of motivation.))
Elton Mayo and his team conducted research on workplace conditions and productivity at the Hawthorne Works Plant of the Western Electric Company from 1924 to 1932. Throughout the study, as Mayo and his team introduced various physical conditions, the workers' productivity increased. When the researchers asked the workers about their behavior, they expressed ((satisfaction because their supervisor had asked for their help and cooperation in the study.))
According to Maslow's hierarchy of needs, a person's desire for acceptance by others is a reflection of their ________ needs. ((social))
Which need is at the top of Maslow's hierarchy of needs, meaning that if you are at this level, you have reached your full potential? ((self-actualization needs))
According to Maslow's theory, a ________ person feels that he or she is living life to its fullest in every way. ((self-actualized))
Maslow's theory maintains that (())
Brandon works as an electrician, going into people's homes and offices to set up and repair electrical systems. When he sees a dangerous condition, he reports it to the home or business owner immediately and begins working to fix the situation. He wears safety goggles and gloves while he is working, and he has a comprehensive insurance plan with income protection in the event that he becomes unable to work. Brandon's actions are most related to Maslow's ________ needs. ((security))
Liam got laid off from his job at a chemical supply company. His company has promised to pay him for three months following the layoff. Before he was laid off, he was focused on selling more than any of his co-workers. Since the layoff, he has shifted his focus to finding a new source of income. What does this scenario highlight about laid off workers, in terms of Maslow's hierarchy of needs? ((They shift their focus from high-level esteem needs to the need for security.))
According to Herzberg's theory, what is a hygiene factor? ((job security))
According to Herzberg's theory, what is a motivational factor? ((recognition))
According to Herzberg's theory, the absence of which factor may be a potential source of dissatisfaction and high turnover? ((job security))
According to Herzberg's theory, the presence of which factor is likely to motivate employees but its absence may not necessarily result in dissatisfaction? ((achievement))
Tiana has a job as a lab technician. She makes a good annual salary and her job is secure. Which of Maslow's needs would these hygiene factors help Tiana satisfy? ((physiological and security))
Juanita manages a large boarding stable that employs several people who clean stalls, feed the horses, teach riding lessons, and do training rides on boarders' horses. According to Herzberg's theory, what can Juanita do to motivate her employees to work harder, in terms of content of the work itself? (( She can involve them in the stable's processes and decisions.))
Tahlia manages a grocery store, which employs several baggers, cashiers, floor managers, and stockroom workers. According to Herzberg's theory, what can Tahlia do to ensure that her employees are satisfied with their work setting? ((She can provide adequate wages.))
Herzberg's motivational factors are similar to Maslow's ((esteem and self-actualization needs.))
In the United States, workers' ________, as defined by Maslow, have largely been met by minimum-wage laws and federal occupational safety standards. ((physiological and security needs))
According to managers who adopt McGregor's Theory X, ((workers must be forced to do their jobs.))
McGregor's Theory X assumes that workers ((avoid responsibility.))
McGregor's Theory Y assumes that workers ((naturally like to work.))
When compared to Maslow's hierarchy of needs, the Theory X style of management focuses on ((physiological and security needs.))
The managers of a meat packing plant believe that the average person dislikes work and will avoid it when possible. As a result, they focus on coercing, controlling, directing, and threatening their workers with punishment to get them to work toward achieving the company's objectives. One method they initiated was an employee ranking system that ranked 50 percent of employees as below expectations. This company has adopted ((McGregor's Theory X perspective.))
If a manager follows McGregor's Theory Y approach to management, he or she is likely to assume that ((workers can contribute through imagination and ingenuity.))
Felicia is the principal of a charter school. She believes that her employees like to work and that under proper conditions, they will seek out responsibility in an attempt to satisfy their social, esteem, and self-actualization needs. What theory of management would Felicia most likely follow? ((McGregor's Theory Y))
What is a major difference between the Theory X and Theory Y views of management? ((Theory Y takes into account people's needs for companionship, esteem, and personal growth, whereas Theory X does not.))
What is the difference between Theory X and Theory Y managers as it relates to Maslow's hierarchy of needs? ((Theory X managers focus on physiological and security needs, while Theory Y managers address Maslow's high-level needs as well as physiological and security needs.))
Chester's Crab House has several locations in Massachusetts and New Jersey. Each location has a restaurant manager who runs his or her location as he or she sees fit. The restaurant chain has achieved success by empowering its employees to make their own decisions and follow their passions. Which management theory does Chester's Crab House follow? ((McGregor's Theory Y))
Which is an assumption of Theory Y? ((Organizations today do not make full use of workers' intellectual potential.))
What theory states that motivation depends not only on how much a person wants something but also on the person's perception of how likely he or she is to get it? ((expectancy theory))
Javier works as an assistant manager at a home improvement center. He wants to be promoted to the position of manager but feels that it is unlikely to happen. As a result, Javier is not motivated to achieve this goal. Which theory best explains Javier's situation? ((expectancy theory))
Sean is a marketer at Second Chance Car Company, and he wants to inform the market of the benefits of buying used cars over expensive new cars. He invests his efforts in creating advertisements that emphasize the company's low prices and service guarantee to reach the company's target market and compete with the large car dealerships in the area. This scenario relates most directly to ((goal-setting theory.))
Naveen, the manager of Not Just Any Burger, a chain of restaurants, gives awards on a monthly basis to employees with the highest productivity. Naveen's action is based on ((Skinner's behavior modification theory.))
Frontline Industries annually rewards the top ten employees, in terms of goal achievement, with a trip to Hawaii. What effect will Frontline Industries' action most likely have on employee behavior? ((It will motivate all employees to work hard to achieve company goals.))
April, a coffee shop manager, penalizes employees for reporting late to work. This measure has gone a long way in ensuring that employees arrive on time. The step taken by April best reflects ((Skinner's behavior modification theory.))
Which statement accurately reflects the two strategies used in Skinner's behavior modification theory? ((In general, rewarding appropriate behavior is a more effective way to modify behavior than punishing unacceptable behavior.))
Soumya manages a small diner. She notices some of her employees stealing fries and other food off patron's plates before bringing the food out to the tables. When she sees employees doing this, she punishes them by taking a portion of their tips and distributing that money to the employees who don't steal food from plates. What is an undesirable long-term side effect that may result from such punishments? ((there may be increased turnover))
Historically, Tom's Tool Works has used specialization to increase productivity. However, Tom has noticed that his employees seem bored with their regular tasks and productivity is actually declining. Tom decides to reduce employee boredom by allowing workers to perform a greater variety of tasks and learn new skills. He is most likely adopting ((job rotation))
What is an advantage of job enrichment? ((It enhances a worker's feeling of responsibility.))
What strategy benefits both companies and employees since it increases a company's ability to recruit and retain workers and allows employees to balance work and home life by allowing them to choose their starting and ending times, as long as they are there during a specified core period? ((flextime))
A compressed workweek is most likely to ((reduce a company's operating expenses.))
Vicki works as a medical transcriptionist. She is required to log in 40 hours in four days and take three days off. Vicki's work schedule is an example of ((a compressed workweek.))
Compensation for individuals within a specific job category depends on the compensation for that job and not on the individual's productivity. ((FALSE))
All of the following are reasons human resources management has increased in importance over the past few decades EXCEPT ((today's employees are only concerned with what a job pays and show little concern for other aspects of the job.))
LiMing was recently laid off from her job, and she is using job search websites to find a new job. When she finds a job title that looks interesting, she can click on it to read an overview of the job's tasks, relationships with other jobs, the physical and mental skills required, duties, responsibilities, and working conditions. If this job ________ looks like something she wants to do, she can submit an application. ((description))
Marlene has been an editorial assistant at a small trade publisher for two years. When an associate editor position opens up on her team, she applies for the position and gets it. What is one issue the company will have as a result of promoting Marlene into this new position? ((The company will have to fill Marlene's vacant editorial assistant position.))
Lately, Ron's Right Renovations has been receiving numerous requests to tile customers' kitchens and bathrooms. However, none of the contractors who work for this company specialize in tiling. The owner, Ron, starts recruiting for a tiling expert. Why does Ron decide to hire an external candidate instead of hiring from within? ((There are better-qualified people outside of the organization.))
Which stage of the employee selection process allows management to obtain detailed information about an applicant's experience and skills, reasons for changing jobs, and an idea of whether the person would fit in with the company? ((interviewing))
Jayla recently interviewed for a receptionist position at a physician's office. The interview went well, and now, the human resources manager asks her to complete the Myers-Briggs Type Indicator Test. Why does the human resources manager ask Jayla to complete this test? ((to assess her ability to fit into the organization's culture))
Camila just completed graduate school, earning a master's degree in publishing and writing. She interviews for an editorial assistant position at a small publishing house, and the human resources manager gives Camila a copyediting test to complete. A reason the human resources manager gives Camila this ability test is to ((determine whether she has the skills necessary for the job.))
Derek is a human resources manager. He is considering applicants for an open position and has narrowed it down to the two strongest applicants. To help him decide which applicant to choose, he calls their references to verify their educational backgrounds and previous work experiences. Derek knows it is important to check the applicants' references because research has shown that those who are willing to exaggerate or lie on their resumes are more likely to ((engage in unethical behavior.))
Martina arrives for her first day at a new job. The hiring manager, Francisco, takes her on a tour of the building, introducing her to her new co-workers and showing her where the kitchen, meeting rooms, and her cubicle are located. After the tour, Francisco leaves Martina in her cubicle with the company's employee handbook, so she can read about the organization's policies and benefits. All these activities are part of ((orientation.))
Luis has worked at Mel's auto shop for five years. He has been asked by Mel to train, support, and guide Tim—a new employee. In this way, Luis will be taking on the role of a(n) ((mentor.))
Jamie is an employee at a research lab. In an effort to improve his managerial skills and prepare him for a promotion, his company sends him on a series of experiential management training exercises at the company's expense. By helping him increase his knowledge, Jamie's company is focusing on ((development.))
Angelo was recently moved to a new position in his company that involved learning new skills and undertaking work assignments. However, he received no salary raise. Angelo's change in position can best be described as a(n) ((transfer.))
As it applies to the workplace, a separation is a(n) ((employee's departure from an organization.))
Samantha was terminated from her job because she was repeatedly late to work. She impacts her company's turnover rates through ((separation.))
What should managers do to avoid lawsuits from individuals who feel they have been fired unfairly? ((document all problems and warnings in the employee's work records))
Recently, Bella's company has downsized, laying off several employees. After the layoffs, Bella's workload increased, but her compensation stayed the same. As a result, she decides to leave the company. On her last day, the human resources manager schedules a meeting with Bella to determine why she is leaving the organization. This meeting is called a(n) ((exit interview.))
________ is a public protest against management practices and involves union members marching at an employer's plant or work site. ((Picketing))
Jason works at a bottling plant for a soft drink company. He is part of a labor union, along with his co-workers. After the plant's management and the labor union fail to agree on contract terms, the union uses picketing and advertising to ask its members and the public not to buy the company's products. This scenario best describes ((a boycott.))
Management's version of a strike is called a ((lockout.))
The management and labor union at a snack foods manufacturing plant are struggling to reach an agreement regarding benefits. They bring in a neutral third party to settle their dispute. The third party's solution is legally binding and enforceable. Thus, the third party is a(n) ((arbitrator.))
The difference between an arbitrator and a mediator is that ((an arbitrator's solution to a labor dispute is binding on the participants, while a mediator's solution is not binding.))
Marcia is a human resources manager at a large firm that does business internationally. A significant part of her job is dealing with the diversity of the company's customers and employees. Understanding this diversity means ((recognizing and accepting differences and valuing the unique perspectives such differences can bring to the workplace.))
Creating an innovative product that meets many users' needs is sufficient in today's global marketplace. ((FALSE))
Trying to determine customers' true needs is difficult because no one fully understands what motivates people to buy things. ((TRUE))
Lucia went to a new salon in the area to get her hair cut and colored. However, the stylist cut her hair shorter than she wanted and wouldn't use the bold red color Lucia wanted. As a result, Lucia did not feel good about her purchase. What does this most likely mean for the future of this exchange relationship? ((Lucia will go to a different salon and stylist for her next cut and color.))
Larry has a farm where he grows fresh vegetables and fruit. The process he uses to move the produce to the local farmer's market where he will sell them to customers is the marketing activity known as (( transporting))
A customer's subjective assessment of benefits relative to costs in determining the worth of a product is known as ((value.))
Preston decides to buy a new car. He does a lot of research online and goes to several car dealerships to test drive cars. He finally decides on a car to purchase, but, in order to buy it, Preston must take out a loan. He also must get the car inspected and registered before he can start driving it. In this scenario, which is a nonmonetary cost associated with Preston's car purchase? ((the time taken to test drive various car models))
Marissa owns an equestrian clothing store. After many customers complained that her clothing only fit petite riders, she decided to add a new brand of riding clothes that makes clothing for all body shapes and sizes. In regard to the marketing concept, which statement describes Marissa's decision? ((To remain competitive, companies must be prepared to add to or adapt their product lines to satisfy customer desires.))
The Car Lot is a large car dealership. It could sell its cars for $500 and give customers a lifetime guarantee, but based on the marketing concept, why shouldn't it do this? ((The company should strike a balance between achieving organizational objectives and satisfying customers.))
Michael worked for a textile manufacturer during the first half of the 20th century. When the supply of his company's textiles exceeded customer demand, he realized that he would have to invest more money in advertising his products and go directly to his customers to convince them to purchase his products. Based on this scenario, Michael has most likely adopted the ________ orientation. ((sales))
Victoria is a marketing manager. A large part of her role is trying to assess what customers want and building long-term relationships with customers. What would likely complicate Victoria's job? ((Trends, fashions, and tastes can change quickly.))
Quentin owns a computer store that sells new and used computers and has a service department. When customers buy computers at his store, he offers a service package at a discounted rate. Additionally, he has an ongoing promotion that allows customers to trade in their old computers for new ones at much lower prices than his competitors can offer. He is able to do this because his service department can fix customers' old computers, which allows him to resell them to other customers. Based on this scenario, Quentin is focused on ((enhancing the profitability of existing customers and extending the duration of customer relationships.))
Precision Movement Inc. is a company that markets its products—luxury wristwatches—exclusively to high-income individuals and celebrities. Thus, high-income individuals and celebrities are most likely Precision Movement Inc.'s ((target market.))
Eclectic Furniture produces and sells unique furniture pieces to both individual homeowners and hotels. As a result, this company's products can be considered ((both consumer and business products.))
Which basis for market segmentation pertains to some characteristic of the consumer's actions toward the product—a characteristic that commonly involves some aspect of product use? ((behavioristic))
Strength and Fitness Gym provides its clients with personal training sessions, small group exercise classes, fitness machines and weights to use, and a separate kids' gym with day care specialists, so members can bring their children to the gym with them. These items are most closely associated with the ________ aspect of the marketing mix. ((product))
Millennial Phones wants to target college students and young professionals. The company has therefore decided to advertise its cell phones over social networking and other popular websites. Relative to television advertising, this strategy will help Millennial Phones save costs and thus allow the firm to sell its phones at more competitive prices. Millennial Phones' decision to advertise through different media than the ones adopted by other firms illustrates the ________ activities of the marketing mix. ((promotion))
Albert's Foods, a multinational chain of supermarkets, wants to open a new store in a new country. Before launching in the new market, the management of Albert's Foods wants to collect primary data about that country's market. Which source of information will it most likely use? ((the data collected from anonymous surveys conducted in malls in the country))
Valerie is a marketer for an online shoe store. An important task she performs regularly is measuring the website's traffic and performance. This task is known as ((marketing analytics.))
Suzanne is an environmental activist who avoids buying paper-based products unless they are made from recycled paper. Which psychological factor is primarily responsible for Suzanne's buying behavior in this scenario? ((attitude))
Steven dislikes buying designer clothes because he claims that they are a waste of money. In this scenario, Steven's buying behavior is most likely related to his ((attitude.))
Carolyn is highly influenced by celebrities in movies and advertisements. She wishes to dress like them and adopt their lifestyle. When she goes shopping, these celebrities are her point of comparison. In this scenario, the celebrities are Carolyn's ((reference group.))
Every time Rob wants to change his cell phone, he takes the opinions of his colleagues and friends to decide which phone to buy. Thus, Rob's colleagues and friends are his ((reference group.))
Which is an example of how a person's social role can influence his or her buying behavior? ((Monica has five children, so she bought a station wagon.))
Matrix Clothing Inc. decides to launch its premium line of clothing and bags in a country that is experiencing increasing purchasing power and per capita income by its residents. Which environmental force primarily influenced Matrix Clothing Inc.'s decision? ((economic))
Which factor is a social force that influences the marketing environment? ((the public's opinions about the environment))
Farm Fresh, a supermarket in Oregon, was criticized by the community for its use of plastic bags. In view of the concerns raised by the members of the community, the supermarket's management made the decision to start using biodegradable bags. This decision has been influenced by ________ forces. ((social))
Aiden's music store developed a smartphone application to improve its distribution, promotion, and new-product development. In this scenario, Aiden's store is using ________ forces. ((technological))
Each type of stock sold by a corporation represents a claim on the organization and must be represented by a separate owners' equity account. ((TRUE))
Ratio analysis calculations measure an organization's financial health. ((TRUE))
John is an electrician who runs his own small shop. When tax season comes, he hires an accountant from one of "the Big Four" accounting firms to help him file his business's tax return. In this scenario, John has most likely hired a ________ accountant. ((certified public))
Claire is a certified public accountant who provides accounting services to individuals and businesses. Certification gives Claire the right to ((express, officially, an unbiased opinion regarding the accuracy of a client's financial statements.))
Viktor is an accountant employed by a large corporation. He works for the billing and taxation department, preparing and analyzing the company's financial statements. He is most likely a(n) ________ accountant. ((private))
Lucia is an accountant who completed coursework beyond a basic four- or five-year college accounting degree. What does Lucia's additional training allow her to do? ((understand, interpret, and even develop the sophisticated accounting systems necessary to classify and analyze complex financial information))
Pierre owns a chain of coffee shops. He hires accountants to summarize the information from his company's business transactions into a financial statement. Why would Pierre use a financial statement? ((to aid in internal planning and control))
Management's greatest single concern is ________, which is the movement of money through an organization over a daily, weekly, monthly, or yearly basis. ((cash flow))
Carlee's Cake Creations hired a managerial accountant to help forecast the company's expenses and income for its first year in business. This plan for how Carlee's Cake Creations plans to move from Point A to Point B over the course of the year is its ((budget.))
Potential investors typically study the ________ in a firm's annual report to determine whether the company meets their investment requirements. ((financial statements))
Community Bank, a short-term lender, examines Tiger Golf Shop's ________ to assess its ability to repay a loan quickly with cash generated from sales. ((cash flow))
Terrance is a manager at a bank. He has to decide whether to lend $10,000 to Weed and Prune, a landscaping company. He should ((review the company's financial statements.))
When lending money, a long-term lender is more interested in a company's ________ and indebtedness to other lenders. ((profitability))
Qingyi works for a large technology company as an engineer. As an employee, why should he look at his company's financial statements? ((to establish reasonable expectations for salary and other benefit requests))
Make It Your Own, a craft store, displays various types of paper, stickers, glue sticks, yarn, canvases, paints, and other craft supplies on shelves. These items are examples of the company's ((assets.))
What is meant by owners' equity? ((It refers to the money contributed to a firm that never has to be paid back.))
When Hunter opened his fish and tackle store, he convinced his family and friends to invest in the store. They invested money and assets to help him start and grow his business. The money and assets these investors contributed to Hunter's store comprise his ((owners' equity.))
Which equation is equivalent to the accounting equation? ((owners' equity = assets – liabilities))
What is the last step that must be taken before a public corporation's books can be closed for an accounting cycle? ((A CPA must attest that generally accepted accounting principles were used.))
An income statement shows ((an organization's profitability over a period of time.))
The total amount of money received from the sale of goods or services, as well as from other business activities such as the rental of property and investments is a firm's ((revenue.))
Ken's Home and Garden started the accounting period with $10,000 worth of shovels in inventory. During the accounting period, it purchased $5,000 worth of shovels and sold $7,500 worth. In this scenario, what is the cost of goods sold? (($7,500))
Sheila owns a childcare business. Since this market is competitive, she has hired an advertising and sales manager to make sure potential customers know about her company. As a result, she must pay for advertising materials and her advertising and sales manager's salary. These expenses would be classified as ________ expenses. ((selling))
When a piece of equipment is fully depreciated, it ((has a zero value on the firm's balance sheet.))
The ________ presents a snapshot of an organization's financial position at a given moment. ((balance sheet))
A company's assets that are used or converted into cash within the course of a calendar year are called ((current assets.))
A company's financial obligations to short-term creditors, which must be repaid within one year, are called its ((current liabilities.))
________ includes the owners' contributions to a company along with income earned by the company and retained to finance continued growth and product development. ((Owners' equity))
Each type of stock issued by a company represents a different claim on the organization, thus each must be represented by a separate owners' equity account called ((contributed capital.))
The financial statement that explains how a firm's cash changed from the beginning of the accounting period to the end is called the ((statement of cash flows.))
Geri's Gift Store wants to compute the revenues and expenses for its most recent accounting period to determine if it is making extra cash that it can use to invest in increased long-term capacity or to pay off its debts. To determine this, Geri should calculate cash from ________ activities by combining the changes in the revenue accounts, expense accounts, current asset accounts, and current liability accounts. ((operating))
Brandon's Boot Shop is paying off its long-term debt from the bank loan Brandon secured to open the business several years ago. What does this scenario indicate? ((Brandon's Boot Shop has a negative amount of cash from financing activities.))
When Juan's Taco Hut decided to open several new locations, it spent millions of dollars on property and equipment. Which category of cash flow does this best describe? ((cash from investing activities))
________ analysis refers to calculations that measure an organization's financial health. ((Ratio))
Which calculation helps managers compare an organization's productivity, profitability, and financing mix with other similar entities? ((ratio analysis))
One of the ways to analyze a company's financial performance is to compare its performance with those of the ((companies in the same industry.))
Brittany is asked to extend trade credit to a restaurant she hasn't serviced before. She asks to see its balance sheet to determine if it could pay its bills. She divides its current assets by current liabilities to get its ((current ratio.))
Steve's Shelving and Storage has a high inventory turnover ratio because Steve frequently sells and replaces his inventory over the course of a year. What might this scenario indicate? ((great efficiency))
The measure of liquidity that reflects a company's liquidity ratio after excluding its inventory is the ________ ratio. ((quick))
Stan's Skateboards doesn't have a lot of inventory. As a result, its ________ ratio is almost exactly the same as its current ratio. ((quick))
________ ratios provide information about how much debt an organization is using relative to other sources of capital, such as owners' equity. ((Debt utilization))
The earnings per share ratio is important because it ((determines a company's overall stock price.))
The equilibrium price is represented by the point where a product's supply and demand curves intersect. ((TRUE))
While ethical behavior can enhance a company's reputation, it often has a negative effect on the company's financial performance. ((FALSE))
The outcomes of a business's efforts are ((products))
Businesses differ from nonprofit organizations in that a business's focus is on ((profit.))
All the groups that have an interest in an organization's success and outcomes are known as the organization's ((stakeholders.))
All of the following are people and forces beyond an organization's control that have an impact on the business's daily operations EXCEPT ((employees.))
A firm's _____ resources are also known as labor. ((human))
Gary is a real estate entrepreneur in a country that promotes the individual right to own property, earn profits, make decisions about business operations, and choose productivity systems. Which of the following systems best describes Gary's business environment? ((free enterprise))
If you are a member of a wholesale club, like BJs or Sam's Club, you tend to buy items like toilet paper and groceries in bulk to save money. This is an element of ((demand.))
In which of the following market structures do individual businesses have control over their products' price because each business supplies a large portion of the products sold in the marketplace? ((oligopoly))
Which of the following statements is true about GDP as a measure of a country's economic health? ((It measures only those goods and services made within a country.))
A business's obligations to its owners and investors include maintaining proper accounting procedures. ((TRUE))
A local pet store, Pals with Paws, donates a portion of every sale to pet rescues in its community. This exhibits the company's ((social responsibility))
The term _____ refers to a business's obligation to maximize its positive impact and minimize its negative impact on society. ((social responsibility))
How do the terms business ethics and social responsibility differ from each other? ((Business ethics relates to an individual's or a work group's decisions that society evaluates as right or wrong, whereas social responsibility concerns the impact of the entire business's activities on society.))
When it hit the headlines that Auto Corp. pays its board members $200,000 per year, it created ethical concerns. People fear that compensation over a certain amount ((affects directors' objectivity and duty to their firms.))
After a major horse race, it was discovered that the winner had drugged his horse to make it run faster. As a result, the horse's owner, trainer, and jockey were banned from the racing industry. This exemplifies how ((negative judgment directly affects an organization's ability to achieve its business goals.))
Which of the following acts is associated with bullying? ((discrediting others' ideas and opinions))
Megan needs Steve's help to resolve an urgent work issue. She has emailed and called him several times, but Steve doesn't respond to any of her communications. Steve's behavior is an example of ((bullying))
A hairdryer manufacturer produced a new model that could dry a person's hair in under five minutes. When customers used this hairdryer and experienced scalp and hand burns, the manufacturer was required to create and enforce detailed plans to prevent future burns. This example relates to ((fairness and honesty.))
_____ involves taking someone else's work and presenting it as your own ((Plagiarism))
If a manager pressures a subordinate to engage in activities that he or she may otherwise view as unethical, such as engaging in accounting fraud or stealing a competitor's secrets, this would be an ethical issue related to ((business relationships))
If a manager attempts to take credit for a subordinate's ideas, he or she is engaging in ((plagiarism))
Jim's boss often goes on Facebook at work. As a result, the rest of his team goes on Facebook as well. This is an example of ((the influence of managers))
Kim's company, Globotech, does not have established rules and policies on ethics. It is a small, independent company that depends on its employees to use their own judgment. This structure can lead to ((opportunity for misconduct))
According to the National Business Ethics Survey (NBES), _____ is the greatest determinant of future misconduct. ((a company's ethical culture))
_____ is the act of an employee exposing an employer's wrongdoing to outsiders ((Whistleblowing))
Obeying the law is a business's ((legal responsibility))
Globotech created its own continuing education program that offers eligible employees full tuition to pursue continuing education related to their roles at the company. It also created a scholarship program to help employees send their children to college. These programs exemplify the company's ((philanthropic activities.))
All of the following are criteria the Ethisphere Institute uses to select its annual list of the world's most ethical companies EXCEPT ((innovation that contributes to the company's well-being))
Social responsibility is an area of business with issues that ((change constantly in response to society's demands))
The assurance of both satisfactory quality and service at a fair price is a part of the consumer's right to ((choose))
John F. Kennedy's consumer bill of rights outlined four rights: the right to be informed, the right to choose, the right to be heard, and the right to ((safety))
Healthy Foods, Inc. started using a new experimental sugar substitute in its products. However, it didn't include that this sugar substitute could cause dizziness and nausea anywhere on its packaging. This is a violation of the consumers' right to ((be informed))
Which of the following is true about the role of the Federal Trade Commission's Bureau of Consumer Protection? ((It protects consumers against unfair, deceptive, or fraudulent practices))
Judy has realized that she does not like working for others. She wants to open a business in which she will have maximum control and the least interference from government regulations. Which of the following forms of business is best suited for Judy's needs? ((a sole proprietorship))
Marta owns a small pet grooming company. When a chain pet store with pet grooming services opened across the street from her business, Marta lowered her prices and added more specialty grooming services. Her ability to make these decisions on the spot without anyone else's approval gave her a ((competitive advantage.))
Josh has been asked to be a part of a new business venture that develops wind energy technologies. Since the business involves high risk, he does not want to be held completely liable for the firm's debts if the project fails. In this case, Josh is most likely to prefer to participate as a(n) ((limited partner))
Alan, Gary, and Paul are starting a new law firm together, and they are drafting articles of partnership. All of the following are items they should include EXCEPT ((the total money and assets of each partner's estate))
Paula is a partner in a local hardware store. She has decided to sell her partnership interest. What is one challenge that Paula will face? ((It is difficult to place a value on a partner's share of the partnership))
Stephanie is a partner in a local hair salon. Without consulting her partners, she ordered some expensive hair products that she thought would greatly improve their business. However, the products were too expensive for the salon's customers, so they sat on the shelves. What impact will Stephanie's decision have on her partners? (("Her decision may put her partners' personal resources in jeopardy.
Catalina opens a fashion boutique with the money she inherited from her parents. She is successful in creating a strong brand image and a loyal customer base. In order to meet the increasing demands of her business, she hires a store manager. However, she finds it hard to let the manager make important decisions about the store. She often finds herself disagreeing with the manager's decisions, and in the process, she loses a lot of existing customers. Which of the following causes of small-business failure does this scenario best illustrate? ((inability to cope with growth))
Candace started a small house cleaning company, Neat and Tidy, and hired a few of her friends to help her. When the business started, most of its clients were having their houses cleaned to prepare them to sell. However, as more people hired Neat and Tidy and saw what a good job they did, the company started to see more repeat clients who wanted their houses cleaned on a weekly or biweekly basis. As this company grows, all of the following are changes that might occur EXCEPT ((the company's reputation will suffer, no matter how well the growth is managed.))
Sarah is starting her own small house painting company, Pickett Fence Painting. She is working on developing the company's business plan. Which of the following is something the business plan should accomplish? ((estimating the business's income and expenses))
Megan is opening her own small retail store that sells women's purses and accessories. To start her business, she needs at least $50,000 in initial financing to do all of the following EXCEPT ((pay off any debts so they don't affect her business))
The act of financing one's business by using real personal assets is known as ((equity financing))
Shawna's small flooring business requires more financing than she can provide from her savings. As a result, she is looking for investors to provide financing in exchange for stock ownership. What is a disadvantage of this arrangement? ((Shawna will have to share the profits of her business))
Chang is an interior designer who starts his own consulting firm. His friend Rachel is the owner of a coffee shop. Chang decorates Rachel's new coffee shop in exchange for a supply of coffee whenever his clients visit his office. Which of the following sources of funding does this scenario best illustrate? ((bartering))
Heather wants to run her own picture framing business, but she doesn't want to start it from scratch. She decides to buy an existing business. What is one advantage of doing this? ((It provides a built-in network of customers, suppliers, and distributors.))
Gary decides to buy an existing home goods store, instead of starting his own from scratch. Once he takes over the business, he learns that several customers have complained that one of the company's distributors doesn't pack items sufficiently for shipping, and their items arrived broken. What is Gary's responsibility in resolving this problem? ((Gary is responsible for taking on and resolving any problems the business already has))
A license to sell another's products or to use another's name in business, or both, is called a ((franchise))
Since Eva has decided to use her savings to purchase the rights to own and operate a McDonald's outlet rather than starting her own restaurant, she is most likely to be a(n) ((franchisee))
Paul has recently quit his job as an investment banker. He plans to open a restaurant. He has two options: he can either start his own new restaurant from scratch or purchase a franchise from an already established restaurant or fast food chain. His wife Lisa supports the former plan. Which of the following statements strengthens the argument in favor of Lisa's choice of opening a restaurant independently? ((It is easier and more flexible to make and execute decisions in an independent business.))
_____ are often the Small Business Administration's principal means of providing direct management assistance to small businesses and entrepreneurs. ((Small Business Development Centers))
Sally is a human resource (HR) manager at a company. A survey of the company's employees reveals that more than 80 percent of the employees belong to the demographic segment Generation Y. In this scenario, which of the following HR strategies should Sally implement in order to best serve the needs of the employees? ((Sally should use recognition and advancement as the driving forces to motivate employees.))
Norton is a human resource manager at a large multinational company. After a drastic drop in revenue, his company thinks of ways to correct the situation. At a meeting with the top management, the CEO suggested that the company should reduce its workforce by 20 percent. This move would help the company cut costs and continue functioning on the current projects. In this scenario, which of the following strategies is Norton's company planning to use? ((downsizing))
_____ make decisions about the use of an organization's resources and are concerned with planning, organizing, directing, and controlling the organization's activities. ((Managers))
Marilyn is a manager at a large accounting firm. The firm is expanding, and she is tasked with hiring more people to carry out the organization's increased workload. She must determine all of the following items EXCEPT ((which employees to replace with new candidates.))
Determining an organization's objectives and deciding how to accomplish them, a management function, is known as ((planning.))
Megan manages an online clothing store. She must determine what her company's warehouses and distributing facilities need and which type of automation can maximize order efficiency. What major phase of the planning process does this example describe? ((data gathering))
A declaration of an organization's fundamental purpose and basic philosophy is known as a(n) ((mission))
Cameron started his own photography business. He wants to write a good mission statement that explains his company's reason for existence. What is one guideline he should follow in writing his mission statement? ((It should be clear and concise.))
Nolan's jewelry business has grown from a small startup in his room to a chain of outlets all over the country. His business has the potential to expand further, provided he has the resources. To attract investors and other resources, he is required to create a mission statement for his company. If he highlights that his gems come from countries with fair labor practices, which basic question of a well-developed mission statement is he addressing? ((What are our responsibilities with respect to being a good steward of environmental, financial, and human resources?))
Harlan's hot dog stand wants to earn a profit and satisfy its customers. As a result, it has developed several signature hot dogs that use quality ingredients and mix inventive flavors. What key component of a goal is this example most closely related to? ((an attribute))
In the context of a business's objectives, which of the following is an ineffective practice? ((isolating them from the business's mission))
_____ are the ends or results desired by an organization that are derived from the organization's mission. ((Objectives))
Stephanie's candy store has profit as an objective. As a result, she wants to have ((money and assets left over after paying off business expenses))
Kendall's fitness company wants to make the best use of its resources. As a result, she hired college students to work at the front desk, so her personal trainers can focus on helping members in the gym and teaching fitness classes. What type of objective is the most relevant to this example? ((efficiency))
The top management of Dusk Automobiles Inc. has decided that the company's objective for the next two years will be to expand the overall business internationally. This is an example of ((strategic planning))
EduPub, a large educational publisher, decided to sell its college division because it isn't as profitable as the school division. This will allow the company to focus its efforts and funds on becoming the leading school publisher. What type of plan is this? ((strategic))
Short-range plans covering a period of one year or less that are designed to implement the activities and objectives specified in an organization's strategic plan are referred to as ((tactical plans.))
Which of the following is an effective practice with regard to tactical plans? ((They should be periodically reviewed and updated by an organization's management))
_____ are very short-term plans that specify what actions individuals, work groups, or departments need to accomplish in order to achieve an organization's tactical plan and ultimately the strategic plan. ((Operational plans))
Xtreme Toys, a company that manufactures toys inspired by popular video games, creates a strategic plan to be the leader in the product category within a period of three years. The company's middle management also creates a half-yearly plan to ensure that the objectives of the strategic plan are properly implemented. The half-yearly plan created by Xtreme Toys can be regarded as a(n) ((tactical plan))
The top management of XSports Autos Inc. creates a strategic plan to enter new product markets within a period of five years. The company's middle management creates half-yearly plans to ensure that the objectives of the strategic plan are properly implemented. Along with these plans, each department creates short-term plans that specify the actions to be taken by specific employees and teams to meet the objectives of the strategic plan and the half-yearly plans. These short-term plans established at the departmental level are referred to as ((operational plans.))
Crisis management plans generally cover maintaining business operations during a crisis and ((communicating with others about the situation and the company's response to the crisis.))
A huge fire broke out in Rustic Rooms, a small rustic furniture store. The fire destroyed all of the furniture, and ultimately, the company went out of business. Which of the following is most likely the reason this company went out of business? ((Rustic Rooms didn't have an updated contingency plan to handle such a crisis.))
Paul manages a large technology company. One of his roles is structuring the company's resources and activities to accomplish its objectives efficiently and effectively. What management function does this example pertain to? ((organizing))
Harold's kitchen appliance company is reorganizing for greater efficiency. How would this company most likely organize the work? ((into teams))
Kevin is a top-level manager at an advertising firm. He has many years of varied experience and he spends most of his time making strategic decisions about how to use the company's resources. On which management function does he spend most of his time? ((planning))
Lara was less educationally qualified and experienced than her colleagues when she joined Aurora Systems Inc. However, due to her ability to negotiate effectively with clients, convince her team members to work harder, and understand their needs, she is a senior-level manager today. Which of the following skills can be most attributed to Lara's success in this scenario? ((human relations skills))
Carl is a manager of a large car dealership. He often meets with consumer groups to discuss the safety features of the company's cars. When he does this, he is acting as a ((spokesperson))
Jasper is a first-line manager at a heating and HVAC company. Although he no longer goes out to customers' homes and does their repairs, since he worked in the field for over 20 years, he is able to train employees, answer questions, provide guidance, and solve problems. These abilities most closely describe his ((technical expertise.))
Landon is interviewing for a position at a law firm. In the interview, he is asked: "How would you resolve the issue of successfully arguing a case in which you disagree with our client's position?" Answering this question requires Landon to use ((analytical skills.))
EduTech is a publisher that provides high quality digital educational products to instructors and students. When new employees are hired, they must attend extensive trainings and demos of the company's products to learn about their functionality and the customers' experience with the products. Annually, the company sends all its employees and managers on a retreat to take mandatory classes on customer relations and office leadership that address skills related to listening, consistent communication, and handling change without compromising values. This scenario is most closely related to developing ((human relations skills))
Carlos is a manager at a men's clothing store. He is very particular about how things are done, right down to how the shirts are folded and the positioning of the mannequins. He uses his authority and economic rewards to get his employees to comply with his directions. What type of manager is Carlos? ((autocratic))
Audio Xperience, a stereo and audio electronics company, encourages its employees to work together to make decisions that reinforce the company's mission and values. Each employee brings his or her own unique insights, and by discussing these insights with other employees, they collectively develop innovative ideas or decisions that most likely would not have been reached by one or two people. This scenario explains ((teamwork))
Gerald is assessing the appropriateness of a proposed option to resolve his publishing company's plagiarism issue. When considering the consequences of the option, he should ((consider its impact on the organization as a whole.))
Brenda manages a flower shop. Recently, the flower shop has had trouble competing with the local grocery store's florist. Brenda wants to develop a list of possible courses of action to draw more customers to her shop, so she gathers her team. They discuss the issue and suggest ideas to solve the problem. This is called ((brainstorming))
Maddie's pet store has decided to abandon one of its dog food brands. To implement this decision, Maddie must do all of the following EXCEPT ((produce a similar brand to fool customers.))
Disadvantages of product departmentalization are that it duplicates functions and does not focus on an organization's overall objectives. ((TRUE))
_____ refers to a firm's shared values, beliefs, traditions, philosophies, rules, and role models for behavior. ((Organizational culture))
Jenna is interviewing to work at a doctor's office. During the interview process, the hiring manager had her come in and talk to some of the employees about the office's culture. All of the following things she learned relate to the office's culture EXCEPT ((her job will consist of answering phones, checking patients in and out, and filing patient information.))
Which of the following is a means of informally expressing an organization's culture? ((employee dress codes))
Which of the following is a formal expression of an organization's culture? ((codes of ethics))
The managers of Rug Emporium, a large rug store, decide to have a sale. The store manager works with the advertising department to make the public aware of the sale, with department managers to schedule enough salespeople to handle the increased customer traffic, and with merchandise buyers to ensure that enough rugs are available to meet expected consumer demand. What does this scenario best describe? ((how people must work together to achieve the store's objectives))
Carolyn manages a frozen pizza company. In order to earn profits, she must determine what activities are required to achieve the company's objectives. All of the following are activities this company would need to do EXCEPT ((talking to competitors to make sure their recipe is unique.))
Kendall works at a small publishing house that groups jobs by functional activities. Her company has departments focused on editorial, marketing, finance, production, legal, and human resources. Which of the following is most likely true of the departments in Kendall's company? ((Each functional department is managed by an expert in the work done by the department.))
Melanie's Macaroni is a new restaurant that prepares childhood favorites in new ways to appeal to adults. The restaurant's owner has assigned an employee to create a new advertising campaign to attract customers. Which of the following is something the employee will need to carry out this assignment? ((the authority to make decisions about the advertising materials and costs))
Which of the following is an advantage of decentralization? ((It may increase the organization's productivity.))
Donna is a manager of an online clothing store. She has five employees who work remotely in their homes in New York, Texas, California, Colorado, and Florida. In addition to managing her employees and having a phone meeting with each of them weekly, Donna also orders merchandise, manages the inventory, coordinates shipping to customers, and manages the company's advertising and finances. What type of span of management does this scenario describe? ((narrow))
Philip is a manager at a large office supplies store, where his main responsibility is managing 30 employees. His employees are highly competent, so he doesn't need to interact with them frequently and few problems arise. His company has a set of specific operating procedures that govern his activities and the activities of his employees. What type of span of management does this scenario describe? ((wide))
Carlos manages a hardware superstore, where he supervises 50 employees. Because he manages a large number of employees, fewer management layers are needed, and thus, his organization is flat with a wide span of management. As a result, what is most likely true of Carlos's duties, in comparison with managers in tall organizations? ((He performs more administrative duties.))
Gerald is the owner of a popular restaurant. He has appointed a manager who takes care of the day-to-day operations of the restaurant and reports to Gerald at the end of each week. The manager is assisted by a junior manager, who supervises the rest of the staff in the restaurant. Thus, Gerald's restaurant is using a _____ structure. ((line))
Leonard and Carol are both managers at a home improvement store. Leonard oversees the flooring, lighting, appliances, and hardware departments. Carol provides each of these departments with advice and support in the areas of finance and human resources. Which of the following is true regarding Leonard and Carol's roles in the company? ((Leonard is a line manager, while Carol is a staff manager.))
Liz is taking a college business class. Her professor splits the class into work teams to complete a project. Each team must create a fictional company, create a business plan for a new product, and market that product in a class presentation. Which of the following would be true of Liz's work team? ((Team members share leadership and create collective work products.))
Keith's company uses internal networks to share information with employees. What is an advantage of using this type of network? ((It can help with the flow of everyday business activities))
Mary, a project manager at a software company, informs her subordinates of the organization's new policies and code of conduct. This is an example of _____ communication. ((downward))
Which of the following is true regarding monitoring communications? ((Failing to monitor employees' use of e-mail, social media, and the Internet can be costly.))
Maslow's hierarchy lists the most basic needs people strive to satisfy first at the top of the pyramid. ((FALSE))
An inner drive that directs a person's behavior toward goals is called ((motivation))
Taylor is an independent contractor in the plumbing industry. He receives work from several companies and individual customers. There are often multiple ways to fix a plumbing problem, and when given the option, Taylor sometimes chooses to use the fastest and easiest solution, rather than the best possible solution. What might explain Taylor's tendency to avoid doing what he should do? ((lack of motivation))
Larry is a computer salesman who has been working in his field for over 20 years. Recently, he has noticed that his sales haven't been as high as the sales of some of his younger coworkers. He recognizes the need to increase his sales. Which of the following is something he might do to satisfy this need and achieve success? ((obtain additional training to improve his sales skills))
Victoria is a cake designer and baker at a bakery. She often creates custom cakes for weddings, birthdays, anniversaries, and other large events. She is emotionally involved and committed to her work, which allows her to meet her customers' high expectations. Victoria can be described as ((engaged))
Creative Cookbooks Inc. is a large publisher that focuses on developing cookbooks with recipes from around the world. Recently, since more people are getting their recipes from the Internet, Creative Cookbooks is struggling to stay profitable. As a result, the company has decided to streamline its operations, laying off 50 employees and changing the way the company works to focus more on offering innovative digital cookbook applications. Many of the company's employees are frustrated with the changes, so in addition to the employees the company let go, several other employees have quit. This scenario is most closely related to ((low morale.))
Javier works at a grocery store. He started six months ago as a bagger, but recently, his boss recognized his hard work, promoting him to cashier and giving him a raise. Javier's promotion and raise are examples of ((extrinsic rewards))
Becky works in a factory that produces dolls and other toys. To improve productivity, Becky's manager broke down each job into its component tasks, determined the best way to perform each task, and specified the output to be achieved by each worker performing the task. As a result, Becky is expected to add arms and legs to 50 dolls per day. She is paid $2.50 per doll, but if she exceeds 50 dolls in a day, she receives an additional $1.50 per doll. This scenario relates to ((the classical theory of motivation))
Larissa worked at the Hawthorne Works Plant of the Western Electric Company in 1924, when Elton Mayo and his team started their research on workplace conditions and productivity. Throughout the study, as Mayo and his team introduced various physical conditions, Larissa's productivity increased. When the researchers asked her about her behavior, she expressed ((satisfaction because her supervisor had asked for her help and cooperation in the study.))
The American Painters' Association provides discounted house painting services to military and veteran families, which enlivens its employees. This shows how important it is for employees to feel like ((their work matters))
According to Maslow's hierarchy of needs, which of the following needs do people strive to satisfy first? ((physiological needs))
According to Maslow's hierarchy of needs, the need for love, companionship, and friendship—the desire for acceptance by others—is a part of the _____ needs. ((social))
Which of the following needs is at the top of Maslow's hierarchy of needs? ((self-actualization needs))
Blujay Pharmaceutical features the salesperson who has been the most successful during the previous quarter in the company newsletter. According to Maslow's hierarchy of needs, which of the following needs of employee is Blujay Pharmaceutical trying to meet? ((self-actualization needs))
Maslow's theory maintains that ((the more basic needs at the bottom of the hierarchy must be satisfied before higher-level goals can be pursued.))
Garrett works as an electrician, going into people's homes and offices to set up and repair electrical systems. When he sees a dangerous condition, he reports it to the home or business owner immediately and begins working to fix the situation. He wears safety goggles and gloves while he is working, and he has a comprehensive insurance plan with income protection in the event that he becomes unable to work. Garrett's actions are most related to Maslow's _____ needs. ((security))
Travis got laid off from his job at a hay and grain supply company. Before he was laid off, he was focused on selling more hay and grain than any of his co-workers. Since the layoff, he has shifted his focus to finding a new source of income. What does this scenario highlight about laid off workers, in terms of Maslow's hierarchy of needs? ((They shift their focus from high-level esteem needs to the need for security.))
According to Herzberg's theory, which of the following is a hygiene factor? ((salary))
According to Herzberg's theory, which of the following is a motivational factor? ((responsibility))
According to Herzberg's theory, the absence of which of the following factors may be a potential source of dissatisfaction and high turnover? ((security))
According to Herzberg's theory, the presence of which of the following factors is likely to motivate employees though their absence may not result in dissatisfaction? ((achievement))
Hailey has a job as a dental hygienist. She makes a good annual salary and her job is secure. Which of Maslow's needs would these hygiene factors help Hailey satisfy? ((physiological and security))
Maritza manages a large boarding stable that employs several people who clean stalls, feed the horses, teach riding lessons, and do training rides on boarders' horses. Which of the following is a way Maritza can motivate her employees to work harder, in terms of content of the work itself? ((She can involve them in the stable's processes and decisions.))
Herzberg's motivational factors are similar to Maslow's ((esteem and self-actualization needs))
In the United States, workers' _____, as defined by Maslow, have largely been met by minimum-wage laws and federal occupational safety standards. ((physiological and security needs))
According to managers who adopt McGregor's Theory X ((workers must be forced to do their jobs.))
McGregor's Theory X corresponds to ((the traditional view of management))
McGregor's Theory Y corresponds to ((the humanistic view of management.))
When compared to Maslow's hierarchy of needs, the Theory X style of management focuses on ((physiological and security needs.))
The managers of Automotive Parts Emporium believe that the average person dislikes work and will avoid it when possible. As a result, they focus on coercing, controlling, directing, and threatening their workers with punishment to get them to work toward achieving the company's objectives. One method they initiated was an employee ranking system that ranked 60 percent of employees as below expectations. This company has adopted ((McGregor's Theory X perspective.))
Mary, a manager at a publishing house, follows McGregor's Theory Y approach to management. She is likely to assume that ((workers can contribute through imagination and ingenuity))
Jill is the principal of a charter school. She believes that her employees like to work and that under proper conditions, they will seek out responsibility in an attempt to satisfy their social, esteem, and self-actualization needs. What theory of management would Jill most likely follow? ((McGregor's Theory Y))
Which of the following is a major difference between the Theory X and Theory Y views of management? ((Theory Y takes into account people's needs for companionship, esteem, and personal growth, whereas Theory X does not.))
In regards to Maslow's hierarchy of needs, what is the difference between Theory X and Theory Y managers? ((Theory X managers focus on physiological and security needs, while Theory Y managers address Maslow's high-level needs as well as physiological and security needs.))
Lorenzo's Italian Restaurant has several locations in Massachusetts and New York. Each location has a restaurant manager who runs his or her location as he or she sees fit. The restaurant chain has achieved success by empowering its employees to make their own decisions and follow their passions. Which management theory does Lorenzo's Italian Restaurant follow? ((McGregor's Theory Y))
_____ states that motivation depends not only on how much a person wants something but also on the person's perception of how likely he or she is to get it. ((Expectancy theory))
Rahul works as an assistant manager in a car manufacturing firm. He wishes to be promoted to the position of manager but feels that it is unlikely to happen. As a result, Rahul is not motivated to achieve the desired end. Which of the following theories best explains Rahul's situation? ((expectancy theory))
Calvin is a marketer at Like-New Car Company, and he wants to inform the market of the benefits of buying used cars over expensive new cars. He invests his efforts in creating advertisements that emphasize the company's low prices and service guarantee to reach the company's target market and compete with the large car dealerships in the area. This scenario relates most directly to ((goal-setting theory.))
Elias, the manager of Eat Big, a chain of restaurants, gives awards on a monthly basis to employees with the highest productivity. Elias's action is based on ((Skinner's behavior modification theory))
Stag Industries annually rewards the top ten employees, in terms of goal achievement, with a trip to the Caribbean. What effect will Stag Industries' action most likely have on employee behavior? ((It will motivate all employees to work hard to achieve company goals.))
Jennifer, the store manager at a supermarket, penalizes employees for reporting late to work. This measure has gone a long way in ensuring that employees arrive on time. The step taken by Jennifer best reflects ((Skinner's behavior modification theory.))
Which of the following statements is true regarding the two strategies used in Skinner's behavior modification theory? ((In general, rewarding appropriate behavior is a more effective way to modify behavior than punishing unacceptable behavior.))
Mandy manages a small diner. She has noticed that some of her employees steal fries and other food off patron's plates before bringing the food out to the tables. When she sees employees doing this, she punishes them by taking a portion of their tips and distributing that money to the employees who don't steal food from plates. Which of the following is an undesirable long-term side effect that may result from such punishments? ((there may be increased turnover))
Historically, Aaron's Automotive has used specialization to increase productivity. However, Aaron has noticed that his employees seem bored with their regular tasks and productivity is actually declining. If Aaron decides to reduce employee boredom by allowing workers to undertake a greater variety of tasks and by giving them the opportunity to learn new skills, he is most likely adopting ((job rotation))
What is an advantage of job enrichment? ((It enhances a worker's feeling of responsibility))
A compressed workweek is most likely to ((reduce a company's operating expenses.))
Simone works as a medical transcriptionist. She is required to log in 40 hours in four days and take three days off. Simone's work schedule is an example of ((a compressed workweek.))
It is not necessary for a company to verify an applicant's references because most applicants tell the truth on applications or résumés. ((FALSE))
Compensation for individuals within a specific job category depends on the compensation for that job and not on the individual's productivity. ((FALSE))
Human resources management refers to ((all the activities involved in determining an organization's human resources needs, as well as acquiring, training, and compensating people to fill those needs.))
All of the following are reasons human resources management has increased in importance over the past few decades EXCEPT ((today's employees need to be directed by autocratic managers.))
Megan has worked in the publishing field for over 15 years. She was recently laid off from her job, and she is using job search websites to find a new job. When she finds a job title that looks interesting, she can click on it to read an overview of the job's tasks, relationships with other jobs, the physical and mental skills required, duties, responsibilities, and working conditions. If this job _____ looks like something she would want to do, she can submit an application. ((description))
Mary has been an editorial assistant at a small trade publisher for two years. When an associate editor position opens up on her team, she applies for the position and gets it. What is one issue the company will have as a result of promoting Mary into this new position? ((The company will have to fill Mary's vacant editorial assistant position.))
Lenny's Carpentry and Painting has recently received many calls looking for help tiling kitchens and bathrooms. Since none of the contractors that work for this company specialize in tiling, Lenny is looking to hire a new tiling expert to complete these customer projects. To recruit potential candidates, the company has created a career website to provide employment information and take applications. Which of the following is a reason Lenny might hire an external candidate instead of hiring from within? ((There are better-qualified people outside of the organization.))
Which stage of the employee selection process allows management to obtain detailed information about an applicant's experience and skills, reasons for changing jobs, and an idea of whether the person would fit in with the company? ((interviewing))
Tara has recently interviewed for a receptionist position at a dental office. The interview went well, and now, the human resources manager has asked her to complete the Myers-Briggs Type Indicator Test. Which of the following is a reason the human resources manager would ask Tara to complete this test? ((to assess her ability to fit into the organization's culture))
Naomi has just arrived for her first day at work at GloboTech, a large technology company. The first thing the hiring manager, Evan, does is take her on a tour of the building, introducing her to her co-workers and showing her where the kitchen, meeting rooms, and her cubicle are located. After the tour, Evan leaves Naomi in her cubicle with the company's employee handbook, so she can read about the organization's policies and benefits. Evan tells Naomi that he will be back in two hours to bring her to lunch with the entire team. All these activities are part of ((orientation))
Sampson's Sporting Goods Store has recently hired several new employees who have the knowledge, skills, and abilities the company needs. Why is it still important that these new employees undergo training? ((to teach them how to do their specific job tasks))
Vikram is an employee at Monsanto research lab. In an effort to improve his managerial skills and prepare him for a promotion, his company sends him on a series of experiential management training exercises at the company's expense. By helping him increase his knowledge, Vikram's company is focusing on ((development))
_____ is one of the most difficult tasks for managers. However, doing so is essential because it gives employees feedback and generates information about the quality of the firm's selection, training, and development activities. ((Performance appraisal))
In the context of a performance review, _____ refers to an employee's willingness to identify and address opportunities for improvement. ((initiative))
Ryan works at an electronics store. During his performance review, his manager praises him for always responding quickly to requests, showing up on time for all his shifts, and always doing his best to complete his tasks correctly and thoroughly. In this scenario, the manager is praising Ryan's ((dependability))
Joe was recently moved to a new position in his company that involved learning new skills and undertaking work assignments. However, he received no salary raise. Joe's change in position can best be described as a(n) ((transfer))
Which of the following statements is true of a separation? ((It is an employee's departure from an organization))
Socorro was terminated from her job because she was repeatedly late to work. She contributes to her company's turnover through ((separation))
Which of the following should managers do to avoid lawsuits from individuals who may feel they have been fired unfairly? ((document all problems and warnings in the employee's work records))
Kendall has worked for a heating, oil, and propane company for five years. Recently, her company has downsized, laying off several employees. After the layoffs, Kendall's workload increased but her compensation stayed the same. As a result, Kendall has decided to leave the company. On her last day, the human resources manager has scheduled a meeting with Kendall to determine why she is leaving the organization. This meeting is called a(n) ((exit interview.))
_____ is a public protest against management practices and involves union members marching at an employer's plant or work site. ((Picketing))
Nick works at a bottling plant for a soft drink company. He is part of a labor union, along with his co-workers. After the plant's management and the labor union failed to agree on contract terms, the union used picketing and advertising to ask its members and the public not to buy the company's products. This scenario best describes ((a boycott.))
Which of the following is management's version of a strike? ((a lockout))
The management and a labor union at a car parts manufacturing plant are struggling to reach an agreement regarding benefits. They bring in a neutral third party to settle their dispute. The third party's solution is legally binding and enforceable. Thus, the third party is a(n) ((arbitrator))
The difference between an arbitrator and a mediator is that ((an arbitrator's solution to a labor dispute is binding on the participants, while a mediator's solution is not binding.))
Meredith is a human resources manager at a large advertising firm that does business internationally. A significant part of her job is dealing with the diversity of the company's customers and employees. Understanding this diversity means ((recognizing and accepting differences and valuing the unique perspectives such differences can bring to the workplace.))
William is the human resources manager at a large law firm that has historically been dominated by white men. As the firm's clients have become more diverse, William sees the need to recruit, develop, and retain more diverse employees. All of the following are things William can do to attract and retain a more diverse group of employees EXCEPT he can ((hire minority candidates for all open positions, regardless of qualifications))
Abby is a doctor at a hospital where most of the doctors are men. The other doctors have made a lot of sexist comments, indicating that she should be a nurse, not a doctor, and that the job is too difficult and complex for a woman. Hurt by these comments, Abby discussed the situation with her boss, who told her to brush off the comments and just do her job. Since this hospital doesn't seem to value its diverse employees, like Abby, all of the following are consequences it may face EXCEPT ((decreased turnover.))
Creating an innovative product that meets many users' needs is sufficient in today's volatile global marketplace. ((FALSE))
Trying to determine customers' true needs is difficult because no one fully understands what motivates people to buy things. ((TRUE))
Jasmine went to a new salon in the area to get her hair cut and colored. However, the stylist cut her hair shorter than she wanted and wouldn't use the bold red color Jasmine wanted. As a result, Jasmine did not feel good about her purchase. What does this most likely mean for the future of this exchange relationship? ((Jasmine will go to a different salon and stylist for her next cut and color.))
All of the following are examples of the activities marketing performs to accomplish objectives and generate exchanges EXCEPT ((Jim doesn't want his car anymore, so he leaves it on the side of the road with a "free" sign on it.))
A customer's subjective assessment of benefits relative to costs in determining the worth of a product is known as ((value))
Jeremy has decided to buy a new car. Since he travels a lot, he is looking for a car that gets great gas mileage and has enough room for his luggage. He has done a lot of research online and gone to several car dealerships to test drive cars. He has finally decided to buy a Toyota Prius because he liked driving it, and it fits his criteria for great gas mileage and storage space. However, in order to buy the car, Jeremy must take a car loan. He also must get the car inspected and registered before he can start driving it. In this scenario, which of the following is a nonmonetary cost associated with Jeremy's car purchase? ((the time taken to test drive various car models))
Monica owns an equestrian clothing store. After many customers complained that her clothing only fit petite riders, she decided to add a new brand of riding clothes that makes clothing for all body shapes and sizes. In regards to the marketing concept, which of the following statements describes Monica's decision? ((To remain competitive, companies must be prepared to add to or adapt their product lines to satisfy customers' desires.))
Auto Giants is a large car dealership. It could sell its cars for $100 and give customers a lifetime guarantee, but based on the marketing concept, why shouldn't it do this? ((The company should strike a balance between achieving organizational objectives and satisfying customers.))
Gary worked for a textile manufacturer during the first half of the 20th century. When the supply of his company's textiles exceeded customer demand, he realized that he would have to invest more money in advertising his products and go directly to his customers to convince them to purchase his products. Based on this scenario, Gary has most likely adopted the _____ orientation. ((sales))
Melinda is a marketing manager at a company that sells bicycles. A large part of her role is trying to assess what customers want and building long-term relationships with customers. Which of the following would likely complicate Melinda's job? ((Trends, fashions, and tastes can change quickly.))
Bobby owns a computer store that sells new and used computers and has a service department. When customers buy computers at his store, he offers a service package at a discounted rate. Additionally, he has an ongoing promotion that allows customers to trade in their old computers for new ones at much lower prices than his competitors can offer. He is able to do this because his service department can fix customers' old computers, which allows him to resell them to other customers. Based on this scenario, Bobby is focused on ((enhancing the profitability of existing customers and extending the duration of customer relationships.))
Tory is a marketing manager for a large chain of sporting goods stores. A significant part of her job is analyzing the purchase data of the company's customers. All of the following are benefits of doing this EXCEPT it ((allows the company to focus on targeting groups of similar customers.))
Ferava Inc. is a company that markets its products—luxury wristwatches—exclusively to high-income individuals and celebrities. Thus, high-income individuals and celebrities are most likely Ferava Inc.'s ((target market.))
Rustic Rooms Furniture produces and sells unique furniture pieces to both individual homeowners and hotels. As a result, this company's products can be considered ((both consumer and business products))
Mitch's Sporting Goods Store decides to focus its new marketing campaigns on its yoga, softball, and tennis equipment in an attempt to attract more female customers. Which of the following is most likely the reason this store might want to focus on this market segment? ((Women are the largest market segment))
For a firm to successfully use a concentration or multisegment approach to market segmentation, all of the following requirements must be met EXCEPT ((consumers' needs for the product must be homogeneous))
Which of the following bases for market segmentation pertains to some characteristic of the consumer's actions toward the product—a characteristic that commonly involves some aspect of product use? ((behavioristic))
Prism Phones wants to target college students and young professionals. The company has therefore decided to advertise its cell phones over social networking and other popular websites. Relative to television advertising, this strategy will help Prism Phones save costs and thus allow the firm to sell its phones at more competitive prices. Prism Phones' decision to advertise through different media than the ones adopted by other firms illustrates the _____ activities of the marketing mix. ((promotion))
Personal Fitness Gym provides its clients with personal training sessions, small group exercise classes, fitness machines and weights to use, and a separate kids' gym with day care specialists, so members can bring their children to the gym with them. These items are most closely associated with the _____ aspect of the marketing mix. ((product))
Oliver's Oil Company wants to increase customer demand for oil. What can Oliver do to accomplish this? ((reduce the price of his oil))
Global Mart Inc., a multinational chain of supermarkets, wants to open a new store in a country called Alheroni. Before launching itself in the new market, the management of Global Mart Inc. wants to collect primary data about Alheroni's market. Which of the following sources of information will it most likely use? ((the data collected from anonymous surveys conducted in malls in Alheroni))
All of the following are examples of companies using online marketing research EXCEPT ((Quality Cars uses mystery shoppers to visit its car dealerships and report on whether the dealerships are adhering to the company's standards of service.))
Gabrielle is a marketer for an online shoe store. An important task she performs regularly is measuring the website's traffic and performance. This task is known as ((marketing analytics))
Anna is an environmental activist who avoids buying paper-based products unless they are made from recycled paper. Which of the following psychological factors is primarily responsible for Anna's buying behavior in this scenario? ((attitude))
Derek dislikes buying branded clothes because he claims that they are a waste of money. In this scenario, Derek's buying behavior is most likely related to his ((attitude.))
Jennifer is highly influenced by celebrities in movies and advertisements. She wishes to dress like them and adopt their lifestyle. When she goes shopping, these celebrities are her point of comparison. In this scenario, the celebrities are Jennifer's ((reference group))
Every time David wants to change his cell phone, he takes the opinions of his colleagues and friends to decide which phone to buy. Thus, David's colleagues and friends are his ((reference group))
Which of the following is an example of how a person's social role can influence his or her buying behavior? ((Andrea has five children, so she bought a station wagon))
Due to the increasing purchasing power and per capita income of the population of United Nombavia, True Couture Inc. decided to launch its premium line of clothing and bags in the country. Which of the following environmental forces primarily influenced True Couture Inc.'s decision? ((economic forces))
Which of the following is a social force that influences the marketing environment? ((the public's opinions about the environment))
Daily Stop, a supermarket in California, was criticized by the community for its use of plastic bags. In view of the concerns raised by the members of the community, the supermarket's management made the decision to start using biodegradable bags. This decision has been influenced by _____ forces. ((social))
Kevin's music store developed a smartphone application to improve its distribution, promotion, and new-product development. In this scenario, Kevin's store is using _____ forces. ((technological))
The balance sheet presents a "snapshot" of an organization's financial position at a given moment. ((TRUE))
Cash from investing activities is calculated from changes in the long-term liability accounts and the contributed capital accounts in owners' equity. ((FALSE))
Matt is an auto mechanic who runs his own small shop. When tax season comes, he hires an accountant from one of "the Big Four" accounting firms to help him file his business's tax return. In this scenario, Matt has most likely hired a _____ accountant. ((certified public))
Amanda is a certified public accountant who provides accounting services to individuals and businesses. Which of the following rights does her certification give her? ((the right to express, officially, an unbiased opinion regarding the accuracy of a client's financial statements))
Vikram is an accountant employed by a large corporation. He works for the billing and taxation department, preparing and analyzing the company's financial statements. He is most likely to be a _____ accountant. ((private))
Carol is an accountant who completed course work beyond a basic four- or five-year college accounting degree. Which of the following is something Carol's additional training allows her to do? ((understand, interpret, and even develop the sophisticated accounting systems necessary to classify and analyze complex financial information))
Harold owns a chain of coffee shops. He hires accountants to summarize the information from his company's business transactions into a financial statement. Which of the following is a reason Harold would use a financial statement? ((to aid in internal planning and control))
Gary's Gadgets hired a managerial accountant to help forecast the company's expenses and income for its first year in business. This plan for how Gary's Gadgets plans to move from Point A to Point B over the course of the year is its ((budget))
Potential investors typically study the _____ in a firm's annual report to determine whether the company meets their investment requirements. ((financial statements))
Small Bank, a short-term lender, examines Frank's Kitchen Fixtures' _____ to assess its ability to repay a loan quickly with cash generated from sales. ((cash flow))
Thomson is a manager at a bank. He has to decide whether to lend $10,000 to Safe Toys, a company that produces toys that don't have a choking hazard. He should ((review the company's financial statements.))
When lending money, a long-term lender is more interested in a company's _____ and indebtedness to other lenders. ((profitability))
Kerry works for a large technology company as an engineer. As an employee, why should he look at his company's financial statements? ((to establish reasonable expectations for salary and other benefit requests))
Emily's Embellishments, a craft store, displays various types of paper, stickers, glue sticks, yarn, canvases, paints, and other craft supplies on shelves. These items are examples of the company's ((assets))
What is meant by owners' equity? ((It refers to the money contributed to a firm that never has to be paid back.))
When Harvey opened Harvey's Hardware Store, he convinced his family and friends to invest in the store. They invested money and assets to help him start and grow his business. The money and assets these investors contributed to Harvey's store comprise his ((owners' equity))
Which of the following equations is equivalent to the accounting equation? ((owners' equity = assets - liabilities))
What is the last step that must be taken before a firm's books can be closed for an accounting cycle? ((A certified public accountant must attest that generally accepted accounting principles were used.))
An income statement shows ((an organization's profitability over a period of time.))
Leslie owns a childcare company. Since this market is competitive, she has hired an advertising and sales manager to make sure potential customers know about her company. As a result, she must pay for advertising materials and her advertising and sales manager's salary. These expenses would be classified as _____ expenses. ((selling))
Craig's Construction Company bought a crane for $200,000, which it expects to be able to use for 50 years. Rather than showing an expense of $200,000 in the first year and no expense for the crane over the next 49 years, Craig can report depreciation expenses of _____ per year in each of the next 50 years because that better matches the cost of the machine to the years it is used. (($4,000))
Which of the following is true of a piece of equipment when it is fully depreciated? ((It has a zero value on the firm's balance sheet.))
The _____ presents a snapshot of an organization's financial position at a given moment. ((balance sheet))
A company's assets that are used or converted into cash within the course of a calendar year are called ((current assets.))
A company's financial obligations to short-term creditors, which must be repaid within one year, are called its ((current liabilities))
_____ includes the owners' contributions to a company along with income earned by the company and retained to finance continued growth and product development. ((Owners' equity))
Each type of stock issued by a company represents a different claim on the organization, thus each must be represented by a separate owners' equity account called ((contributed capital.))
The financial statement that explains how a firm's cash changed from the beginning of the accounting period to the end is called the ((statement of cash flows))
Paul's Plant Store wants to compute the revenues and expenses for its most recent accounting period to determine if it is making extra cash that it can use to invest in increased long-term capacity or to pay off its debts. To determine this, Paul should calculate cash from _____ activities by combining the changes in the revenue accounts, expense accounts, current asset accounts, and current liability accounts. ((operating))
Christy's Craft Company is paying off its long-term debt from the bank loan Christy secured to open the business several years ago. What does this scenario indicate? ((Christy's Craft Company has a negative amount of cash from financing activities))
_____ analysis refers to calculations that measure an organization's financial health ((Ratio))
Which of the following calculations helps managers compare an organization's productivity, profitability, and financing mix with other similar entities? ((ratio analysis))
One of the ways to analyze a company's financial performance is to compare its performance with those of the ((companies in the same industry))
Aaron's Automobiles experienced a huge recall on one of its best selling cars, and it had to spend $10 billion to fix all the recalled cars. Which of the following is most likely the result of this unexpected and expensive charge? ((decreased earnings per share and significantly reduced profitability ratios))
Camille's Closets, a closet design company, has a low return on equity. What does this indicate? ((Immediate managerial attention is needed.))
Which of the following equations is used to calculate return on equity? ((return on equity = net income/owners' equity))
Maurice was asked to extend trade credit to a restaurant she hadn't serviced before. She asked to see its balance sheet to determine if it could pay its bills. She divided its current assets by current liabilities to get its ((current ratio))
Debbie's Doormats has a high inventory turnover ratio because Debbie frequently sells and replaces her inventory over the course of a year. Which of the following might this scenario indicate? ((great efficiency))
The measure of liquidity that reflects a company's liquidity ratio after excluding its inventory is the _____ ratio. ((quick))
Chloe's Cosmetics doesn't have a lot of inventory. As a result, its _____ ratio is almost exactly the same as its current ratio. ((quick))
_____ ratios provide information about how much debt an organization is using relative to other sources of capital, such as owners' equity. ((Debt utilization))
Which of the following explains why the earnings per share ratio is important? ((It determines a company's overall stock price.))
Louis, Carl, and David have secured a charter for their car dealership. They hold an organizational meeting to establish the corporation's bylaws and elect a board of directors. What is one thing the bylaws might do? ((set up committees of the board of directors))
Wilson's Wishing Wells, Inc. is a company that produces wells and well equipment. It has been owned and operated by the Wilson family for 50 years. The Wilsons own all the corporation's stocks, and they are not required to disclose financial information publicly. What type of business is this? ((private corporation))
AgroCorp is a public corporation that experienced difficulty growing its agricultural products during an especially dry summer season. Wanting the flexibility to make decisions for restructuring AgroCorp's operations, its managers bought all the stock, thus ((taking it private.))
A written authorization that assigns a stockholder's voting privilege to another is called ((proxy.))
Due to a severe recession, GloboTech is struggling to operate and earn profits. In the current economic state, people don't have extra money to buy its televisions, stereos, and home theater systems. If these economic conditions continue, GloboTech will be forced to ((file bankruptcy))
Jim and Carol both work in the real estate market. One day, a great investment property comes on the market, and they decide to pool their money to buy and renovate it. They plan to spend six weeks on the renovation, and they hope to make a nice profit when they turn around and sell it. Jim and Carol's partnership is an example of a(n) ((joint venture.))
Which of the following is true of limited liability companies? ((They protect the members' personal assets in case of lawsuits))
Gwen is a doctor who is forming her own medical practice. She wants her practice to be flexible, simple to run, and free from required meetings, minutes, or resolutions. Her goal is to blend the best characteristics of corporations, partnerships, and sole proprietorships. What form of business ownership seems most in-line with Gwen's goals? ((limited liability company))
When a corporate raider wants to acquire or take over another company, it first offers to buy some or all of the other company's stock at a premium over its current price in a(n) ((tender offer.))
Growing up, Cameron noticed that many families in his community struggled to make ends meet and provide for their children's education. As a result, after he finished school, he started a business to provide low-cost tutoring and scholarship opportunities for low-income families in his community. As his company has grown, Cameron has been able to extend free tutoring and college scholarships to five impoverished students per year. Cameron is a ((social entrepreneur))
Vernon runs Buzz, his own event management company, which frequently organizes large corporate events. He has built a good reputation for organizing corporate events with a fair degree of professionalism at reasonable costs. He works out of his own house with a staff of about 55 employees. Vernon's company can be categorized as a ((small business))
Salma owns a Lebanese restaurant that employs around 25 people. She is solely responsible for the management of the restaurant. Her business has to compete with a larger chain of restaurants that offer the same cuisine. Salma's restaurant can be classified as a ((small business.))
When Miguel first moved from Mexico to the United States, he struggled to find and keep work because he couldn't speak English fluently. After learning English, he started a new business to help other Mexican Americans learn English and to help place them in jobs. Miguel's business is a ((minority-owned small business.))
Jack owns a barbershop that specializes in men's haircuts and shaving. This is an example of a ((service business))
Ben and Molly live and work in Texas, but they love vacationing in Hawaii, and they go there as often as time and money will allow. Finally, they take the plunge and buy a vacation home in Hawaii. When they aren't there, they rent the property to other people for $1000 per week. This is an example of ((the sharing economy.))
Simon has worked for a large corporation for years, and he feels like he just doesn't fit in the corporate mold" anymore. He decides to start his own business because he wants the freedom to choose whom he works with and when and where he works. What advantage of small business ownership seems the most prevalent in this example? ((independence))
________ make decisions about the use of an organization's resources and are concerned with planning, organizing, directing, and controlling the organization's activities. ((Managers))
Which of the following is a means of informally expressing an organization's culture? (( employee dress codes))
make decisions about the use of an organization's resources and are concerned with planning, organizing, directing, and controlling the organization's activities (( managers ))
are very short-term plans that specify what actions individuals, work groups, or departments need to accomplish in order to achieve an organization's tactical plan and, ultimately, the strategic plan. ((operational plans))
Short-range plans covering a period of one year or less that are designed to implement the activities and objectives specified in an organization's strategic plan are referred to as ((tactical plans)
Determining an organization's objectives and deciding how to accomplish them, a management function, is known as ((planning.)
The equilibrium price is represented by the point where a product’s supply and demand curves intersect. ((TRUE))
While ethical behavior can enhance a company’s reputation, it often has a negative effect on the company’s financial performance. ((FALSE))
Businesses differ from nonprofit organizations in that a business’s focus is on ((profit.))
All the groups that have an interest in an organization’s success and outcomes are known as the organization’s ((stakeholders.))
Which is the BEST example of a business trying to balance the needs of various stakeholders? ((Pete’s Pest Control Company is trying to develop an eco-friendly insecticide that will eliminate crop-killing pests and still adhere to environmental regulations.))
Ronan works for a custom furniture manufacturer. His job is to oversee the process of taking raw materials—lumber, fabric, and metal components—and ensure that they are transformed to the customer’s exact specifications. Ronan’s job falls under the business activity known as ((operations))
A firm’s ________ resources are also known as labor ((human))
Oscar is a real estate entrepreneur in a country that promotes the individual right to own property, earn profits, make decisions about business operations, and choose productivity systems. Which system best describes Oscar’s business environment? ((free enterprise))
In which market structure do individual businesses have control over their products’ price because each business supplies a large portion of the products sold in the marketplace? ((oligopoly))
Which statement is true about GDP as a measure of a country’s economic health? ((It measures only those goods and services made within a country.))
When it comes to ethical behavior, co-workers and superiors exert significant control over one’s choices at work through authority and example. ((TRUE))
Which of the following best exemplifies a company with a strong ethical culture? ((The top management at Randy’s company clearly communicates to all employees the ethical behavior expected of them.))
A local landscaping company donates and plants a tree in the community for every new client that uses its landscaping design services. This exhibits the company’s ((social responsibility.))
The term ________ refers to a business’s obligation to maximize its positive impact and minimize its negative impact on society. ((social responsibility))
How do the terms business ethics and social responsibility differ from each other? ((Business ethics relates to an individual’s or a work group’s decisions that society evaluates as right or wrong, whereas social responsibility concerns the impact of the entire business’s activities on society.))
Tracor Industries decided to buy less expensive materials from a foreign supplier that resulted in a significant reduction in the quality of its product. As a result, company sales dropped, many retailers refused to carry the product, and consumers left negative reviews online about the product. This is an example of ((how negative judgments can affect an organization’s ability to achieve its goals.))
After a major horse race, it was discovered that the winner had drugged his horse to make it run faster. Even though there was not a specific rule against drugging horses, the horse’s owner, trainer, and jockey were banned from the racing industry. This exemplifies how ((negative judgment directly affects an organization’s ability to achieve its business goals.))
Daniel notices that one of his colleagues arrives late and leaves early from work every day. In deciding whether or not to report this behavior, what must he do first? ((know his company’s policy regarding the ethics of his colleague’s choice))
Which act is associated with bullying? ((discrediting others’ ideas and opinions))
Marissa needs William’s help to resolve an urgent work issue. She has emailed and called him several times, but William doesn’t respond to any of her communications. William’s behavior is an example of ((bullying.))
A hairdryer manufacturer produced a new model that could dry a person’s hair in under five minutes. When customers used this hairdryer and experienced scalp and hand burns, the manufacturer was required to create and enforce detailed plans to prevent future burns. This example relates to ((fairness and honesty.))
________ involves taking someone else’s work and presenting it as your own. ((Plagiarism))
If a manager attempts to take credit for a subordinate’s ideas, this is a form of ((plagiarism.))
Lance’s boss often goes on Facebook at work. As a result, the rest of his team goes on Facebook as well. This is an example of which factor of influence regarding ethical behavior? ((the influence of managers))
Kian’s company, Microtech, does not have established rules and policies on ethics. It is a small, independent company that depends on its employees to use their own judgment. This structure can lead to ((opportunity for misconduct.))
According to the National Business Ethics Survey (NBES), ________ is the greatest determinant of future misconduct. ((a company’s ethical culture))
________ is the act of an employee exposing an employer’s wrongdoing to outsiders. ((Whistleblowing))
Obeying the law is a business’s ((legal responsibility.))
Master Corporation created its own continuing education program that offers eligible employees full tuition to pursue continuing education related to their roles at the company. It also created a scholarship program to help employees send their children to college. These programs exemplify the company’s ((philanthropic activities.))
All of the following are criteria the Ethisphere Institute uses to select its annual list of the world’s most ethical companies EXCEPT ((innovation that contributes to the company’s well-being.))
The assurance of both satisfactory quality and service at a fair price is a part of the consumer’s right to ((choose.))
John F. Kennedy’s consumer bill of rights outlined four rights: the right to be informed, the right to choose, the right to be heard, and the right to ((safety.))
Braxton Foods, Inc. started using a new preservative in its products. However, it didn’t include anywhere on its packaging that this preservative could cause an allergic reaction to people who are allergic to milk products. This is a violation of the consumers’ right to ((be informed.))
What is true about the role of the Federal Trade Commission’s Bureau of Consumer Protection? ((It protects consumers against unfair, deceptive, or fraudulent practices.))
Beth has realized that she does not like working for others. She wants to open a business in which she will have maximum control and the least interference from government regulations. Which form of business is best suited for Beth’s needs? ((a sole proprietorship))
All of the following are ways technology is helping today’s sole proprietors succeed EXCEPT ((landline telephones allow them to make free long-distance calls to customers, suppliers, and others.))
Angelica owns a small hair salon. When a chain hair salon opened across the street from her business, Angelica lowered her prices and added more specialty services, like manicures and facials. Her ability to make these decisions on the spot without anyone else’s approval gave her a ((competitive advantage.))
Mel owns an auto repair shop. He started the business with a personal loan and is wholly responsible for repaying it. Mel also needs to pay personal income tax on the profits generated from the business. Mel’s business is an example of a(n) ((sole proprietorship.))
Victor is asked to be a part of a new business venture for a revolutionary mechanism to repair Internet cables that are placed on the ocean floor. Since the business involves high risk, Victor does not want to be held completely liable for the firm’s debts if the project fails. In this case, Victor is most likely to prefer to participate as a(n) ((limited partner.))
David, Shannon, and Greg are starting a new accounting firm together, and they are drafting articles of partnership. All of the following are items they should include EXCEPT ((the total money and assets of each partner’s estate.))
Ken is a partner in a local hardware store. He decides to sell his partnership interest. What is one challenge that Ken will face? ((It may be difficult for the partners to place a value on Ken’s share of the partnership.))
Sarah is a partner in a local coffee shop. Without consulting her partners, she ordered some expensive, specialty roasted coffee beans that she thought would greatly improve their business. However, their customers felt the coffee had a bitter taste and are no longer buying it. What impact will Sarah’s decision have on her partners? ((Her decision may put her partners’ personal resources in jeopardy.))
Frank, Martin, and Paul secure a charter for their business. They hold an organizational meeting to establish the corporation’s bylaws and elect a board of directors. What is one thing the bylaws might do? ((set up committees of the board of directors))
Brewer Bikes, Inc. is a company that sells bicycles and accessories. It has been owned and operated by the Brewer family for 25 years. The Brewers own all the corporation’s stock, and they are not required to disclose financial information publicly. What type of business is this? ((a private corporation))
Summer Foods is a public corporation that is experiencing difficulty growing its vegetable products during an especially dry summer season. Wanting the flexibility to make decisions for restructuring the company’s operations, its managers buy all the stock, which means the company is ((taking the company private.))
Jeff and Maggie both work in the real estate market. One day, a great investment property comes on the market, and they decide to pool their money to buy and renovate it. They plan to spend six weeks on the renovation, and they hope to make a nice profit when they turn around and sell it. Jeff and Maggie’s partnership is an example of a(n) ((joint venture.))
April is a doctor who is forming her own medical practice. She wants her practice to be flexible, simple to run, and free from required meetings, minutes, or resolutions. Her goal is to blend the best characteristics of corporations, partnerships, and sole proprietorships. What form of business ownership seems most in-line with April’s goals? ((limited liability company))
When a corporate raider wants to acquire or take over another company, it first offers to buy some or all of the other company’s stock at a premium over its current price in a(n) ((tender offer.))
While growing up, Carlos noticed that many families in his community struggled to make ends meet and provide for their children’s education. As a result, he started a business to provide low-cost tutoring and scholarship opportunities for low-income families in his community. As his company has grown, Carlos has been able to extend free tutoring and college scholarships to five impoverished students per year. Carlos is a ((social entrepreneur.))
Robert’s Music Center is a local music shop. Along with selling musical instruments, it also rents instruments to students in school band programs. Robert is the owner and he employs three full-time and five part-time employees. Robert’s business is a ((small business.))
Dante runs Night Moves, his own DJ and party planning company. He has built a good reputation for organizing weddings and corporate events and his business has grown. He recently moved his business from his home to an office located in a corporate building and has increased his staff to 40 employees. Dante’s company can be categorized as a ((small business.))
Mahie owns an Indian restaurant that employs around 20 people. She is solely responsible for the management of the restaurant. Her business competes with a larger chain of restaurants that offers the same cuisine. Mahie’s restaurant can be classified as a ((small business.))
Oliver came up with the idea of producing mayonnaise in neon colors. He was sure that kids would love the idea and this would become the hottest product on the market. Unfortunately, the neon-colored mayonnaise wasn’t such a hit. What is the most likely cause of Oliver’s product failure? ((poor business concept))
Elana recently opened a fashion boutique. She is successful in creating a strong brand image and a loyal customer base. In order to meet the increasing demands of her business, she hires a store manager. However, she finds it hard to let the manager make important decisions about the store. She often finds herself disagreeing with the manager’s decisions, and in the process, she loses several existing customers. Which cause of small-business failure does this scenario best illustrate? ((inability to cope with growth))
Brenda is starting her own small home decorating company. She is developing the company’s business plan. What is something the business plan should accomplish? ((estimating the business’s income and expenses))
The act of financing one’s business by using real personal assets is known as ((equity financing.))
Tamika borrows $40,000 from her father for her new business venture. She promises that he will be a partner in the business and be entitled to a significant percent of the business’s profits at the end of the year. This is an example of ((equity financing.))
Fiona’s small catering business requires more financing than she can provide from her savings. As a result, she is looking for investors to provide financing in exchange for stock ownership. What is a disadvantage of this arrangement? ((Fiona will have to share the profits of her business.))
Antonio is an interior designer who starts his own consulting firm. His friend Vanessa is the owner of a coffee shop. Antonio decorates Vanessa’s coffee shop in exchange for a supply of coffee for his office. Which source of funding does this scenario best illustrate? ((bartering))
Noel decides to buy an existing home goods store, instead of starting his own from scratch. After he takes over the business, he learns that several customers have complained that one of the company’s distributors does not pack items sufficiently for shipping, and their items arrived broken. What is Noel’s responsibility in resolving this problem? ((Noel is responsible for taking on and resolving any problems the business already has.))
A license to sell another’s products or to use another’s name in business, or both, is called a(n) ((franchise))
Peter has recently quit his job and plans to open a restaurant so he can be his own boss. He has two options: he can either start his own new restaurant from scratch or purchase a franchise from an already established restaurant or fast-food chain. His wife Linda supports the former plan. Which statement strengthens the argument in favor of Linda’s choice of opening a restaurant independently? ((It is easier and more flexible to make and execute decisions in an independent business.))
Jess is a human resource (HR) manager at a company. A survey of the company’s employees reveals that more than 80 percent of the employees belong to the demographic segment Generation Y. Based on this, which HR strategy should Jess implement in order to best serve the needs of the employees? ((Jess should use recognition and advancement as the driving forces to motivate employees.))
Samuel is a human resource manager at a large multinational company. After a drastic drop in revenue, his company thinks of ways to correct the situation. At a meeting with the top management, the CEO suggests that the company should reduce its workforce by 20 percent. This move would help the company cut costs and continue functioning on the current projects. In this scenario, which strategy is Samuel’s company planning to use? ((downsizing))
________ make decisions about the use of an organization’s resources and are concerned with planning, organizing, directing, and controlling the organization’s activities. ((Managers))
Wes is a manager at a large investment banking firm. The firm is expanding, and he is tasked with hiring more people to carry out the organization’s increased workload. Wes must also determine what to pay the new hires and develop a plan to train them. Wes is engaged in ((staffing.))
Determining an organization’s objectives and deciding how to accomplish them is the management function known as ((planning))
Marla manages an online shoe store. She must determine what her company’s warehouses and distributing facilities need and which type of automation can maximize order efficiency. What major phase of the planning process does this example describe? ((data gathering))
A declaration of an organization’s fundamental purpose and basic philosophy is known as a(n) ((mission statement.))
Jake starts his own catering business. He creates a website for the business, which states that his purpose is to provide his customers with delicious food at a fair price. For Jake’s company, this is its ((mission statement.))
Vince starts his own tutoring business. He wants to write a good mission statement that explains his company’s reason for existence. What is one guideline he should follow in writing his mission statement? ((It should be clear and concise.))
Naveen is part of the planning team in his organization that is working to develop the firm’s mission statement. The team knows that along with describing the business and identifying its customers, the team should also articulate the business’s operating philosophy and list any core competencies the business has. What other item should Naveen and his team include in the mission statement? ((the firm’s responsibilities with respect to being a good steward of environmental, financial, and human resources))
Joey’s jewelry business has grown from a small startup in his basement to a chain of outlets all over the state. His business has the potential to expand further, provided he has the resources. To attract investors and other resources, he creates a mission statement for his company. He highlights that his gems come from countries with fair labor practices. Which basic question of a well-developed mission statement is he addressing? ((What are our responsibilities with respect to being a good steward of environmental, financial, and human resources?))
A firm’s ________ should contain three key components: an attribute sought, a target to be achieved, and a time frame. ((goals))
Howie’s hot dog stand wants to earn a profit and satisfy its customers. As a result, it has developed several signature hot dogs that use quality ingredients and mix inventive flavors. What key component of a goal is this example most closely related to? ((an attribute))
The ends or results desired by an organization that are derived from the organization’s mission are referred to as ((objectives.))
Melvin’s fishing store has profit as an objective. As a result, he wants to have ((money and assets left over after paying off business expenses.))
Nellie’s Sweet Shop, a large candy store, has historically sold a lot of chocolate and gummy candies. However, over the years, people have become more health conscious. As a result, Nellie’s develops a new line of sugar-free candies that taste as good as the original but without the guilt. What type of objective does this exemplify? ((growth))
The top management of Darryl’s Detailing Center decides that the company’s objective for the next two years will be to expand the overall business to include locations in other states. This is an example of ________ planning. ((strategic))
School Works, a large publisher of educational resources, decides to sell its high school division because it isn’t as profitable as the elementary school division. This will allow the company to focus its efforts and funds on becoming the leading elementary school publisher. What type of plan is this? ((strategic))
Short-range plans covering a period of one year or less that are designed to implement the activities and objectives specified in an organization’s strategic plan are referred to as ((tactical plans.))
What is an effective practice with regards to tactical plans? ((They should be periodically reviewed and updated by an organization’s management.))
) Live Wire Collectibles, a company that manufactures action figures inspired by popular video games, creates a strategic plan to be the leader in the product category within a period of three years. The company’s middle management also creates a six-month plan to ensure that the objectives of the strategic plan are properly implemented. The six-month plan created by Live Wire Collectibles can be regarded as a(n) ((tactical plan.))
The top management of Clear Chemical Inc. creates a strategic plan to enter new product markets within a period of five years. The company’s middle management creates six-month plans to ensure that the objectives of the strategic plan are properly implemented. Along with these plans, each department creates short-term plans that specify the actions to be taken by specific employees and teams to meet the objectives of the strategic plan and the six-month plans. These short-term plans established at the departmental level are referred to as ((operational plans.))
Stan works in the supply warehouse for a large manufacturing facility. Each day, Stan is given a production quota. This ensures that enough materials will be available for the next day’s production run, so the company can increase its product availability and be able to ship products faster than any other company. A production quota is an example of a(n) ((operational plan.))
Crisis management plans generally cover maintaining business operations during a crisis and ((communicating with others about the situation and the company’s response to the crisis.))
A huge fire broke out in Fancy Fixtures, a small home decorating store. The fire destroyed the store’s inventory, and ultimately, the company went out of business. What is most likely the reason this company went out of business? ((Fancy Fixtures didn’t have an updated contingency plan to handle such a crisis.))
Pamela manages a large blood donation facility. One of her roles is structuring the company’s resources and activities to accomplish its objectives efficiently and effectively. What management function does this example pertain to? ((organizing))
Kelli’s kitchen appliance company is reorganizing for greater efficiency. How would this company most likely organize the work? ((into teams))
) Kishan is a top-level manager at an accounting firm. He has many years of varied experience and he spends most of his time making strategic decisions about how to use the company’s resources. On which management function does he spend most of his time? ((planning))
Norah was less educationally qualified and experienced than her colleagues when she joined Micro Systems Inc. However, due to her ability to negotiate effectively with clients, convince her team members to work harder, and understand their needs, she is a senior-level manager today. Which skills can be most attributed to Norah’s success in this scenario? ((human relations skills))
Wayne is a manager of a large pharmaceutical firm. He often meets with consumer groups to discuss the safety of the company’s new drugs. When he does this, he is acting as a ((spokesperson.))
Sean is interviewing for a position at a law firm. In the interview, he is asked: "How would you resolve the issue of successfully arguing a case in which you disagree with our client’s position?" Answering this question requires Sean to use ((analytical skills.))
Which trait would be part of a person’s human relations skills? ((the ability to understand the needs of others))
Pierre is a manager at a women’s clothing store. He is very particular about how things are done, right down to how the shirts are folded and the positioning of the mannequins. He uses his authority and economic rewards to get his employees to comply with his directions. What type of manager is Pierre? ((autocratic))
Sounds Deluxe, a stereo and audio electronics company, encourages its employees to work together to make decisions that reinforce the company’s mission and values. Each employee brings his or her own unique insights, and by discussing these insights with other employees, they collectively develop innovative ideas or decisions that most likely would not have been reached by one or two people. This scenario explains ((teamwork.))
Aiden is assessing the appropriateness of a proposed option to resolve his company’s product safety issues. When considering the consequences of the option, he should ((consider its impact on the organization as a whole.))
An organization’s structure determines how well it makes decisions and responds to problems, and influences employees’ attitudes toward their work. ((TRUE))
Disadvantages of product departmentalization are that it duplicates functions and does not focus on an organization’s overall objectives. ((TRUE))
An organization’s ________ refers to its shared values, beliefs, traditions, philosophies, rules, and role models for behavior. ((culture))
Joylene is interviewing to work at a dentist’s office. During the interview process, the hiring manager had her come in and talk to some of the employees about the office’s culture. ((her job will consist of answering phones, checking patients in and out, and filing patient information.))
Which is a means of informally expressing an organization’s culture? ((employee dress codes))
Which is a formal expression of an organization’s culture? ((codes of ethics))
Christy works for a magazine publishing company. When pressed to meet a deadline, every employee is expected to work together to get the job done, even if it means working overtime. This commitment to teamwork is part of the company’s ((culture.))
Which item is least associated with an organization’s culture? ((its structure))
The managers of Trendy, a large department store, decide to have a sale. The store manager works with the advertising department to make the public aware of the sale, with department managers to schedule enough salespeople to handle the increased customer traffic, and with merchandise buyers to ensure that enough merchandise is available to meet expected consumer demand. What does this scenario say about organizational structure? ((An organizational structure develops when managers get people to work together to achieve the firm’s objectives.))
Kris manages a frozen pizza company. In order to earn profits, she must determine what activities are required to achieve the company’s objectives. All of the following are activities this company would need to do EXCEPT ((talk to competitors to make sure their recipe is unique.))
Judy works at a small printing company that groups jobs by functional activities. Her company has departments focused on sales, marketing, finance, printing, legal, and human resources. What is most likely true of the departments in Judy’s company? ((Each functional department is managed by an expert in the work done by the department.))
Comfort Food Reimagined is a new restaurant that prepares childhood favorites in new ways to appeal to adults. The restaurant’s owner has empowered Bill, an employee, to create a new advertising campaign to attract customers. What does Bill need to carry out this assignment? ((the authority to make decisions about the advertising materials and costs))
An advantage of decentralization is that it ((may increase the organization’s productivity.))
Nicole is a manager of an online clothing store. She has five employees who work remotely in their homes in Minneapolis, Milwaukee, Chicago, Detroit, and Cleveland. In addition to managing her employees and having a phone meeting with each of them weekly, Nicole also orders merchandise, manages the inventory, coordinates shipping to customers, and manages the company’s advertising and finances. What type of span of management does this scenario describe? ((narrow))
Franco is a manager at a large electronics store, where his main responsibility is managing 25 employees. His employees are highly competent, so he doesn’t need to interact with them frequently and few problems arise. His company has a set of specific operating procedures that govern his activities and the activities of his employees. What type of span of management does this scenario describe? ((wide))
Dilbert’s, a large department store chain, has store managers, district managers, regional managers, and functional managers. What type of organization does this describe? ((all))
Chuck manages a hardware superstore, where he supervises 45 employees. Because he manages a large number of employees, fewer management layers are needed, and thus, his organization is flat with a wide span of management. As a result, what is most likely true of Chuck’s duties, in comparison with managers in tall organizations? ((He performs more administrative duties.))
Vince is the owner of a popular restaurant. He hired a manager who takes care of the day-to-day operations of the restaurant and reports to Vince at the end of each week. The manager is supported by an assistant manager, who supervises the rest of the staff in the restaurant. Thus, Vince’s restaurant is using a ________ structure. (( line))
Lucie and Cara are both managers at a home improvement store. Lucie oversees the flooring, lighting, appliances, and hardware departments. Cara provides each of these departments with advice and support in the areas of finance and human resources. What is true regarding Lucie and Cara’s roles in the company? ((Lucie is a line manager, while Cara is a staff manager.))
Rob’s company has grown in recent years, and the firm is looking to restructure the organization into larger groups. Rob and his team are deciding whether to organize its departments on the basis of geography, customer type, or product. Regardless of which basis the firm uses, it will be organized using which structure? ((multidivisional))
Lacey is taking a college business class. Her professor splits the class into work teams to complete a project. Each team must create a fictional company, create a business plan for a new product, and market that product in a class presentation. What is true of Lacey’s work team? ((Team members share leadership and create collective work products.))
William’s company uses internal networks to share information with employees. What is an advantage of using this type of network? (( It can help with the flow of everyday business activities.))
The vice president of research and development at an automobile company wants to plan and coordinate the launch of a new car with the company’s vice president of marketing. Any related communication between them will be referred to as ________ communication. ((horizontal))
Mary, a project manager at a software company, informs her subordinates of the organization’s new policies and code of conduct. This is an example of ________ communication. ((downward))
Which statement is true regarding the monitoring of communications in the workplace? ((Failing to monitor employees’ use of e-mail, social media, and the Internet can be costly.))
According to Maslow’s hierarchy, a person buying insurance with income protection is attempting to satisfy physiological needs ((FALSE))
An inner drive that directs a person’s behavior toward goals is called ((motivation.))
Companies that are concerned with boosting workplace morale, maximizing employees’ productivity and creativity, and motivating employees to be more effective are concerned with ((human relations.))
Martin is an independent contractor in the plumbing industry. He receives work from several companies and individual customers. There are often multiple ways to fix a plumbing problem, and when given the option, Taylor sometimes chooses to use the fastest and easiest solution, rather than the best possible solution. What might explain Martin’s tendency to avoid doing what he should do? (( lack of motivation))
Lately, Frank has trouble starting his car, making him late for work. Today, his car won’t start at all. Since there is no public transportation for him to get to work, he needs to get a new car. After shopping and comparing models, Frank is now ready to sign the paperwork on a new SUV. What is the "need" in this scenario? ((requiring a car for work and not having one that functions))
Jon is a salesman for a security company and has been working in his field for over 20 years. Recently, he noticed that his sales haven’t been as high as the sales of some of his younger co-workers. He recognizes the need to increase his sales. What is something he might do to satisfy this need and achieve success? ((obtain additional training to improve his sales skills))
Natasha is a cake designer. She often creates custom cakes for weddings, birthdays, anniversaries, and other large events. She is emotionally involved and committed to her work, which allows her to meet her customers’ high expectations. Natasha can be described as ((engaged.))
Culinary House, a cookbook publisher, has noticed that since more people are getting their recipes from the Internet, it is struggling to stay profitable. As a result, the company has decided to streamline its operations, laying off 50 employees and changing the way the company works to focus more on offering innovative digital cookbook applications. Many of the company’s employees are frustrated with the changes, so in addition to the employees the company let go, several other employees have quit. This scenario is most closely related to ((low morale.))
Marco works at a grocery store. He started six months ago as a bagger, but recently, his boss recognized his hard work, promoted him to cashier, and gave him a raise. Marco’s promotion and raise are examples of ((extrinsic rewards.))
Nancy works in a factory that produces dolls and other toys. To improve productivity, Nancy’s manager broke down each job into its component tasks, determined the best way to perform each task, and specified the output to be achieved by each worker performing the task. As a result, Nancy is expected to add arms and legs to 50 dolls per day. She is paid $2.50 per doll, but if she exceeds 50 dolls in a day, she receives an additional $1.50 per doll. This scenario relates to ((the classical theory of motivation.))
Elton Mayo and his team conducted research on workplace conditions and productivity at the Hawthorne Works Plant of the Western Electric Company from 1924 to 1932. Throughout the study, as Mayo and his team introduced various physical conditions, the workers’ productivity increased. When the researchers asked the workers about their behavior, they expressed ((satisfaction because their supervisor had asked for their help and cooperation in the study.))
According to Maslow’s hierarchy of needs, a person’s desire for acceptance by others is a reflection of their ________ needs. ((social))
Which need is at the top of Maslow’s hierarchy of needs, meaning that if you are at this level, you have reached your full potential? ((self-actualization needs))
According to Maslow’s theory, a ________ person feels that he or she is living life to its fullest in every way. ((self-actualized))
Maslow’s theory maintains that (())
Brandon works as an electrician, going into people’s homes and offices to set up and repair electrical systems. When he sees a dangerous condition, he reports it to the home or business owner immediately and begins working to fix the situation. He wears safety goggles and gloves while he is working, and he has a comprehensive insurance plan with income protection in the event that he becomes unable to work. Brandon’s actions are most related to Maslow’s ________ needs. ((security))
Liam got laid off from his job at a chemical supply company. His company has promised to pay him for three months following the layoff. Before he was laid off, he was focused on selling more than any of his co-workers. Since the layoff, he has shifted his focus to finding a new source of income. What does this scenario highlight about laid off workers, in terms of Maslow’s hierarchy of needs? ((They shift their focus from high-level esteem needs to the need for security.))
According to Herzberg’s theory, what is a hygiene factor? ((job security))
According to Herzberg’s theory, what is a motivational factor? ((recognition))
According to Herzberg’s theory, the absence of which factor may be a potential source of dissatisfaction and high turnover? ((job security))
According to Herzberg’s theory, the presence of which factor is likely to motivate employees but its absence may not necessarily result in dissatisfaction? ((achievement))
Tiana has a job as a lab technician. She makes a good annual salary and her job is secure. Which of Maslow’s needs would these hygiene factors help Tiana satisfy? ((physiological and security))
Juanita manages a large boarding stable that employs several people who clean stalls, feed the horses, teach riding lessons, and do training rides on boarders’ horses. According to Herzberg’s theory, what can Juanita do to motivate her employees to work harder, in terms of content of the work itself? (( She can involve them in the stable’s processes and decisions.))
Tahlia manages a grocery store, which employs several baggers, cashiers, floor managers, and stockroom workers. According to Herzberg’s theory, what can Tahlia do to ensure that her employees are satisfied with their work setting? ((She can provide adequate wages.))
Herzberg’s motivational factors are similar to Maslow’s ((esteem and self-actualization needs.))
In the United States, workers’ ________, as defined by Maslow, have largely been met by minimum-wage laws and federal occupational safety standards. ((physiological and security needs))
According to managers who adopt McGregor’s Theory X, ((workers must be forced to do their jobs.))
McGregor’s Theory X assumes that workers ((avoid responsibility.))
McGregor’s Theory Y assumes that workers ((naturally like to work.))
When compared to Maslow’s hierarchy of needs, the Theory X style of management focuses on ((physiological and security needs.))
The managers of a meat packing plant believe that the average person dislikes work and will avoid it when possible. As a result, they focus on coercing, controlling, directing, and threatening their workers with punishment to get them to work toward achieving the company’s objectives. One method they initiated was an employee ranking system that ranked 50 percent of employees as below expectations. This company has adopted ((McGregor’s Theory X perspective.))
If a manager follows McGregor’s Theory Y approach to management, he or she is likely to assume that ((workers can contribute through imagination and ingenuity.))
Felicia is the principal of a charter school. She believes that her employees like to work and that under proper conditions, they will seek out responsibility in an attempt to satisfy their social, esteem, and self-actualization needs. What theory of management would Felicia most likely follow? ((McGregor’s Theory Y))
What is a major difference between the Theory X and Theory Y views of management? ((Theory Y takes into account people’s needs for companionship, esteem, and personal growth, whereas Theory X does not.))
What is the difference between Theory X and Theory Y managers as it relates to Maslow’s hierarchy of needs? ((Theory X managers focus on physiological and security needs, while Theory Y managers address Maslow’s high-level needs as well as physiological and security needs.))
Chester’s Crab House has several locations in Massachusetts and New Jersey. Each location has a restaurant manager who runs his or her location as he or she sees fit. The restaurant chain has achieved success by empowering its employees to make their own decisions and follow their passions. Which management theory does Chester’s Crab House follow? ((McGregor’s Theory Y))
Which is an assumption of Theory Y? ((Organizations today do not make full use of workers’ intellectual potential.))
What theory states that motivation depends not only on how much a person wants something but also on the person’s perception of how likely he or she is to get it? ((expectancy theory))
Javier works as an assistant manager at a home improvement center. He wants to be promoted to the position of manager but feels that it is unlikely to happen. As a result, Javier is not motivated to achieve this goal. Which theory best explains Javier’s situation? ((expectancy theory))
Sean is a marketer at Second Chance Car Company, and he wants to inform the market of the benefits of buying used cars over expensive new cars. He invests his efforts in creating advertisements that emphasize the company’s low prices and service guarantee to reach the company’s target market and compete with the large car dealerships in the area. This scenario relates most directly to ((goal-setting theory.))
Naveen, the manager of Not Just Any Burger, a chain of restaurants, gives awards on a monthly basis to employees with the highest productivity. Naveen’s action is based on ((Skinner’s behavior modification theory.))
Frontline Industries annually rewards the top ten employees, in terms of goal achievement, with a trip to Hawaii. What effect will Frontline Industries’ action most likely have on employee behavior? ((It will motivate all employees to work hard to achieve company goals.))
April, a coffee shop manager, penalizes employees for reporting late to work. This measure has gone a long way in ensuring that employees arrive on time. The step taken by April best reflects ((Skinner’s behavior modification theory.))
Which statement accurately reflects the two strategies used in Skinner’s behavior modification theory? ((In general, rewarding appropriate behavior is a more effective way to modify behavior than punishing unacceptable behavior.))
Soumya manages a small diner. She notices some of her employees stealing fries and other food off patron’s plates before bringing the food out to the tables. When she sees employees doing this, she punishes them by taking a portion of their tips and distributing that money to the employees who don’t steal food from plates. What is an undesirable long-term side effect that may result from such punishments? ((there may be increased turnover))
Historically, Tom’s Tool Works has used specialization to increase productivity. However, Tom has noticed that his employees seem bored with their regular tasks and productivity is actually declining. Tom decides to reduce employee boredom by allowing workers to perform a greater variety of tasks and learn new skills. He is most likely adopting ((job rotation))
What is an advantage of job enrichment? ((It enhances a worker’s feeling of responsibility.))
What strategy benefits both companies and employees since it increases a company’s ability to recruit and retain workers and allows employees to balance work and home life by allowing them to choose their starting and ending times, as long as they are there during a specified core period? ((flextime))
A compressed workweek is most likely to ((reduce a company’s operating expenses.))
Vicki works as a medical transcriptionist. She is required to log in 40 hours in four days and take three days off. Vicki’s work schedule is an example of ((a compressed workweek.))
Compensation for individuals within a specific job category depends on the compensation for that job and not on the individual’s productivity. ((FALSE))
All of the following are reasons human resources management has increased in importance over the past few decades EXCEPT ((today’s employees are only concerned with what a job pays and show little concern for other aspects of the job.))
LiMing was recently laid off from her job, and she is using job search websites to find a new job. When she finds a job title that looks interesting, she can click on it to read an overview of the job’s tasks, relationships with other jobs, the physical and mental skills required, duties, responsibilities, and working conditions. If this job ________ looks like something she wants to do, she can submit an application. ((description))
Marlene has been an editorial assistant at a small trade publisher for two years. When an associate editor position opens up on her team, she applies for the position and gets it. What is one issue the company will have as a result of promoting Marlene into this new position? ((The company will have to fill Marlene’s vacant editorial assistant position.))
Lately, Ron’s Right Renovations has been receiving numerous requests to tile customers’ kitchens and bathrooms. However, none of the contractors who work for this company specialize in tiling. The owner, Ron, starts recruiting for a tiling expert. Why does Ron decide to hire an external candidate instead of hiring from within? ((There are better-qualified people outside of the organization.))
Which stage of the employee selection process allows management to obtain detailed information about an applicant’s experience and skills, reasons for changing jobs, and an idea of whether the person would fit in with the company? ((interviewing))
Jayla recently interviewed for a receptionist position at a physician’s office. The interview went well, and now, the human resources manager asks her to complete the Myers-Briggs Type Indicator Test. Why does the human resources manager ask Jayla to complete this test? ((to assess her ability to fit into the organization’s culture))
Camila just completed graduate school, earning a master’s degree in publishing and writing. She interviews for an editorial assistant position at a small publishing house, and the human resources manager gives Camila a copyediting test to complete. A reason the human resources manager gives Camila this ability test is to ((determine whether she has the skills necessary for the job.))
Derek is a human resources manager. He is considering applicants for an open position and has narrowed it down to the two strongest applicants. To help him decide which applicant to choose, he calls their references to verify their educational backgrounds and previous work experiences. Derek knows it is important to check the applicants’ references because research has shown that those who are willing to exaggerate or lie on their resumes are more likely to ((engage in unethical behavior.))
Martina arrives for her first day at a new job. The hiring manager, Francisco, takes her on a tour of the building, introducing her to her new co-workers and showing her where the kitchen, meeting rooms, and her cubicle are located. After the tour, Francisco leaves Martina in her cubicle with the company’s employee handbook, so she can read about the organization’s policies and benefits. All these activities are part of ((orientation.))
Luis has worked at Mel’s auto shop for five years. He has been asked by Mel to train, support, and guide Tim—a new employee. In this way, Luis will be taking on the role of a(n) ((mentor.))
Jamie is an employee at a research lab. In an effort to improve his managerial skills and prepare him for a promotion, his company sends him on a series of experiential management training exercises at the company’s expense. By helping him increase his knowledge, Jamie’s company is focusing on ((development.))
Angelo was recently moved to a new position in his company that involved learning new skills and undertaking work assignments. However, he received no salary raise. Angelo’s change in position can best be described as a(n) ((transfer.))
As it applies to the workplace, a separation is a(n) ((employee’s departure from an organization.))
Samantha was terminated from her job because she was repeatedly late to work. She impacts her company’s turnover rates through ((separation.))
What should managers do to avoid lawsuits from individuals who feel they have been fired unfairly? ((document all problems and warnings in the employee’s work records))
Recently, Bella’s company has downsized, laying off several employees. After the layoffs, Bella’s workload increased, but her compensation stayed the same. As a result, she decides to leave the company. On her last day, the human resources manager schedules a meeting with Bella to determine why she is leaving the organization. This meeting is called a(n) ((exit interview.))
________ is a public protest against management practices and involves union members marching at an employer’s plant or work site. ((Picketing))
Jason works at a bottling plant for a soft drink company. He is part of a labor union, along with his co-workers. After the plant’s management and the labor union fail to agree on contract terms, the union uses picketing and advertising to ask its members and the public not to buy the company’s products. This scenario best describes ((a boycott.))
Management’s version of a strike is called a ((lockout.))
The management and labor union at a snack foods manufacturing plant are struggling to reach an agreement regarding benefits. They bring in a neutral third party to settle their dispute. The third party’s solution is legally binding and enforceable. Thus, the third party is a(n) ((arbitrator.))
The difference between an arbitrator and a mediator is that ((an arbitrator’s solution to a labor dispute is binding on the participants, while a mediator’s solution is not binding.))
Marcia is a human resources manager at a large firm that does business internationally. A significant part of her job is dealing with the diversity of the company’s customers and employees. Understanding this diversity means ((recognizing and accepting differences and valuing the unique perspectives such differences can bring to the workplace.))
Creating an innovative product that meets many users’ needs is sufficient in today’s global marketplace. ((FALSE))
Trying to determine customers’ true needs is difficult because no one fully understands what motivates people to buy things. ((TRUE))
Lucia went to a new salon in the area to get her hair cut and colored. However, the stylist cut her hair shorter than she wanted and wouldn’t use the bold red color Lucia wanted. As a result, Lucia did not feel good about her purchase. What does this most likely mean for the future of this exchange relationship? ((Lucia will go to a different salon and stylist for her next cut and color.))
Larry has a farm where he grows fresh vegetables and fruit. The process he uses to move the produce to the local farmer’s market where he will sell them to customers is the marketing activity known as (( transporting))
A customer’s subjective assessment of benefits relative to costs in determining the worth of a product is known as ((value.))
Preston decides to buy a new car. He does a lot of research online and goes to several car dealerships to test drive cars. He finally decides on a car to purchase, but, in order to buy it, Preston must take out a loan. He also must get the car inspected and registered before he can start driving it. In this scenario, which is a nonmonetary cost associated with Preston’s car purchase? ((the time taken to test drive various car models))
Marissa owns an equestrian clothing store. After many customers complained that her clothing only fit petite riders, she decided to add a new brand of riding clothes that makes clothing for all body shapes and sizes. In regard to the marketing concept, which statement describes Marissa’s decision? ((To remain competitive, companies must be prepared to add to or adapt their product lines to satisfy customer desires.))
The Car Lot is a large car dealership. It could sell its cars for $500 and give customers a lifetime guarantee, but based on the marketing concept, why shouldn’t it do this? ((The company should strike a balance between achieving organizational objectives and satisfying customers.))
Michael worked for a textile manufacturer during the first half of the 20th century. When the supply of his company’s textiles exceeded customer demand, he realized that he would have to invest more money in advertising his products and go directly to his customers to convince them to purchase his products. Based on this scenario, Michael has most likely adopted the ________ orientation. ((sales))
Victoria is a marketing manager. A large part of her role is trying to assess what customers want and building long-term relationships with customers. What would likely complicate Victoria’s job? ((Trends, fashions, and tastes can change quickly.))
Quentin owns a computer store that sells new and used computers and has a service department. When customers buy computers at his store, he offers a service package at a discounted rate. Additionally, he has an ongoing promotion that allows customers to trade in their old computers for new ones at much lower prices than his competitors can offer. He is able to do this because his service department can fix customers’ old computers, which allows him to resell them to other customers. Based on this scenario, Quentin is focused on ((enhancing the profitability of existing customers and extending the duration of customer relationships.))
Precision Movement Inc. is a company that markets its products—luxury wristwatches—exclusively to high-income individuals and celebrities. Thus, high-income individuals and celebrities are most likely Precision Movement Inc.’s ((target market.))
Eclectic Furniture produces and sells unique furniture pieces to both individual homeowners and hotels. As a result, this company’s products can be considered ((both consumer and business products.))
Which basis for market segmentation pertains to some characteristic of the consumer’s actions toward the product—a characteristic that commonly involves some aspect of product use? ((behavioristic))
Strength and Fitness Gym provides its clients with personal training sessions, small group exercise classes, fitness machines and weights to use, and a separate kids’ gym with day care specialists, so members can bring their children to the gym with them. These items are most closely associated with the ________ aspect of the marketing mix. ((product))
Millennial Phones wants to target college students and young professionals. The company has therefore decided to advertise its cell phones over social networking and other popular websites. Relative to television advertising, this strategy will help Millennial Phones save costs and thus allow the firm to sell its phones at more competitive prices. Millennial Phones’ decision to advertise through different media than the ones adopted by other firms illustrates the ________ activities of the marketing mix. ((promotion))
Albert’s Foods, a multinational chain of supermarkets, wants to open a new store in a new country. Before launching in the new market, the management of Albert’s Foods wants to collect primary data about that country’s market. Which source of information will it most likely use? ((the data collected from anonymous surveys conducted in malls in the country))
Valerie is a marketer for an online shoe store. An important task she performs regularly is measuring the website’s traffic and performance. This task is known as ((marketing analytics.))
Suzanne is an environmental activist who avoids buying paper-based products unless they are made from recycled paper. Which psychological factor is primarily responsible for Suzanne’s buying behavior in this scenario? ((attitude))
Steven dislikes buying designer clothes because he claims that they are a waste of money. In this scenario, Steven’s buying behavior is most likely related to his ((attitude.))
Carolyn is highly influenced by celebrities in movies and advertisements. She wishes to dress like them and adopt their lifestyle. When she goes shopping, these celebrities are her point of comparison. In this scenario, the celebrities are Carolyn’s ((reference group.))
Every time Rob wants to change his cell phone, he takes the opinions of his colleagues and friends to decide which phone to buy. Thus, Rob’s colleagues and friends are his ((reference group.))
Which is an example of how a person’s social role can influence his or her buying behavior? ((Monica has five children, so she bought a station wagon.))
Matrix Clothing Inc. decides to launch its premium line of clothing and bags in a country that is experiencing increasing purchasing power and per capita income by its residents. Which environmental force primarily influenced Matrix Clothing Inc.’s decision? ((economic))
Which factor is a social force that influences the marketing environment? ((the public’s opinions about the environment))
Farm Fresh, a supermarket in Oregon, was criticized by the community for its use of plastic bags. In view of the concerns raised by the members of the community, the supermarket’s management made the decision to start using biodegradable bags. This decision has been influenced by ________ forces. ((social))
Aiden’s music store developed a smartphone application to improve its distribution, promotion, and new-product development. In this scenario, Aiden’s store is using ________ forces. ((technological))
Each type of stock sold by a corporation represents a claim on the organization and must be represented by a separate owners’ equity account. ((TRUE))
Ratio analysis calculations measure an organization’s financial health. ((TRUE))
John is an electrician who runs his own small shop. When tax season comes, he hires an accountant from one of "the Big Four" accounting firms to help him file his business’s tax return. In this scenario, John has most likely hired a ________ accountant. ((certified public))
Claire is a certified public accountant who provides accounting services to individuals and businesses. Certification gives Claire the right to ((express, officially, an unbiased opinion regarding the accuracy of a client’s financial statements.))
Viktor is an accountant employed by a large corporation. He works for the billing and taxation department, preparing and analyzing the company’s financial statements. He is most likely a(n) ________ accountant. ((private))
Lucia is an accountant who completed coursework beyond a basic four- or five-year college accounting degree. What does Lucia’s additional training allow her to do? ((understand, interpret, and even develop the sophisticated accounting systems necessary to classify and analyze complex financial information))
Pierre owns a chain of coffee shops. He hires accountants to summarize the information from his company’s business transactions into a financial statement. Why would Pierre use a financial statement? ((to aid in internal planning and control))
Management’s greatest single concern is ________, which is the movement of money through an organization over a daily, weekly, monthly, or yearly basis. ((cash flow))
Carlee’s Cake Creations hired a managerial accountant to help forecast the company’s expenses and income for its first year in business. This plan for how Carlee’s Cake Creations plans to move from Point A to Point B over the course of the year is its ((budget.))
Potential investors typically study the ________ in a firm’s annual report to determine whether the company meets their investment requirements. ((financial statements))
Community Bank, a short-term lender, examines Tiger Golf Shop’s ________ to assess its ability to repay a loan quickly with cash generated from sales. ((cash flow))
Terrance is a manager at a bank. He has to decide whether to lend $10,000 to Weed and Prune, a landscaping company. He should ((review the company’s financial statements.))
When lending money, a long-term lender is more interested in a company’s ________ and indebtedness to other lenders. ((profitability))
Qingyi works for a large technology company as an engineer. As an employee, why should he look at his company’s financial statements? ((to establish reasonable expectations for salary and other benefit requests))
Make It Your Own, a craft store, displays various types of paper, stickers, glue sticks, yarn, canvases, paints, and other craft supplies on shelves. These items are examples of the company’s ((assets.))
What is meant by owners’ equity? ((It refers to the money contributed to a firm that never has to be paid back.))
When Hunter opened his fish and tackle store, he convinced his family and friends to invest in the store. They invested money and assets to help him start and grow his business. The money and assets these investors contributed to Hunter’s store comprise his ((owners’ equity.))
Which equation is equivalent to the accounting equation? ((owners’ equity = assets – liabilities))
What is the last step that must be taken before a public corporation’s books can be closed for an accounting cycle? ((A CPA must attest that generally accepted accounting principles were used.))
An income statement shows ((an organization’s profitability over a period of time.))
The total amount of money received from the sale of goods or services, as well as from other business activities such as the rental of property and investments is a firm’s ((revenue.))
Ken’s Home and Garden started the accounting period with $10,000 worth of shovels in inventory. During the accounting period, it purchased $5,000 worth of shovels and sold $7,500 worth. In this scenario, what is the cost of goods sold? (($7,500))
Sheila owns a childcare business. Since this market is competitive, she has hired an advertising and sales manager to make sure potential customers know about her company. As a result, she must pay for advertising materials and her advertising and sales manager’s salary. These expenses would be classified as ________ expenses. ((selling))
When a piece of equipment is fully depreciated, it ((has a zero value on the firm’s balance sheet.))
The ________ presents a snapshot of an organization’s financial position at a given moment. ((balance sheet))
A company’s assets that are used or converted into cash within the course of a calendar year are called ((current assets.))
A company’s financial obligations to short-term creditors, which must be repaid within one year, are called its ((current liabilities.))
________ includes the owners’ contributions to a company along with income earned by the company and retained to finance continued growth and product development. ((Owners’ equity))
Each type of stock issued by a company represents a different claim on the organization, thus each must be represented by a separate owners’ equity account called ((contributed capital.))
The financial statement that explains how a firm’s cash changed from the beginning of the accounting period to the end is called the ((statement of cash flows.))
Geri’s Gift Store wants to compute the revenues and expenses for its most recent accounting period to determine if it is making extra cash that it can use to invest in increased long-term capacity or to pay off its debts. To determine this, Geri should calculate cash from ________ activities by combining the changes in the revenue accounts, expense accounts, current asset accounts, and current liability accounts. ((operating))
Brandon’s Boot Shop is paying off its long-term debt from the bank loan Brandon secured to open the business several years ago. What does this scenario indicate? ((Brandon’s Boot Shop has a negative amount of cash from financing activities.))
When Juan’s Taco Hut decided to open several new locations, it spent millions of dollars on property and equipment. Which category of cash flow does this best describe? ((cash from investing activities))
________ analysis refers to calculations that measure an organization’s financial health. ((Ratio))
Which calculation helps managers compare an organization’s productivity, profitability, and financing mix with other similar entities? ((ratio analysis))
One of the ways to analyze a company’s financial performance is to compare its performance with those of the ((companies in the same industry.))
Brittany is asked to extend trade credit to a restaurant she hasn’t serviced before. She asks to see its balance sheet to determine if it could pay its bills. She divides its current assets by current liabilities to get its ((current ratio.))
Steve’s Shelving and Storage has a high inventory turnover ratio because Steve frequently sells and replaces his inventory over the course of a year. What might this scenario indicate? ((great efficiency))
The measure of liquidity that reflects a company’s liquidity ratio after excluding its inventory is the ________ ratio. ((quick))
Stan’s Skateboards doesn’t have a lot of inventory. As a result, its ________ ratio is almost exactly the same as its current ratio. ((quick))
________ ratios provide information about how much debt an organization is using relative to other sources of capital, such as owners’ equity. ((Debt utilization))
The earnings per share ratio is important because it ((determines a company’s overall stock price.))
The equilibrium price is represented by the point where a product’s supply and demand curves intersect. ((TRUE))
While ethical behavior can enhance a company’s reputation, it often has a negative effect on the company’s financial performance. ((FALSE))
The outcomes of a business’s efforts are ((products))
Businesses differ from nonprofit organizations in that a business’s focus is on ((profit.))
All the groups that have an interest in an organization’s success and outcomes are known as the organization’s ((stakeholders.))
All of the following are people and forces beyond an organization’s control that have an impact on the business’s daily operations EXCEPT ((employees.))
A firm’s _____ resources are also known as labor. ((human))
Gary is a real estate entrepreneur in a country that promotes the individual right to own property, earn profits, make decisions about business operations, and choose productivity systems. Which of the following systems best describes Gary’s business environment? ((free enterprise))
If you are a member of a wholesale club, like BJs or Sam’s Club, you tend to buy items like toilet paper and groceries in bulk to save money. This is an element of ((demand.))
In which of the following market structures do individual businesses have control over their products’ price because each business supplies a large portion of the products sold in the marketplace? ((oligopoly))
Which of the following statements is true about GDP as a measure of a country’s economic health? ((It measures only those goods and services made within a country.))
A business’s obligations to its owners and investors include maintaining proper accounting procedures. ((TRUE))
A local pet store, Pals with Paws, donates a portion of every sale to pet rescues in its community. This exhibits the company’s ((social responsibility))
The term _____ refers to a business’s obligation to maximize its positive impact and minimize its negative impact on society. ((social responsibility))
How do the terms business ethics and social responsibility differ from each other? ((Business ethics relates to an individual’s or a work group’s decisions that society evaluates as right or wrong, whereas social responsibility concerns the impact of the entire business’s activities on society.))
When it hit the headlines that Auto Corp. pays its board members $200,000 per year, it created ethical concerns. People fear that compensation over a certain amount ((affects directors’ objectivity and duty to their firms.))
After a major horse race, it was discovered that the winner had drugged his horse to make it run faster. As a result, the horse’s owner, trainer, and jockey were banned from the racing industry. This exemplifies how ((negative judgment directly affects an organization’s ability to achieve its business goals.))
Which of the following acts is associated with bullying? ((discrediting others’ ideas and opinions))
Megan needs Steve’s help to resolve an urgent work issue. She has emailed and called him several times, but Steve doesn’t respond to any of her communications. Steve’s behavior is an example of ((bullying))
A hairdryer manufacturer produced a new model that could dry a person’s hair in under five minutes. When customers used this hairdryer and experienced scalp and hand burns, the manufacturer was required to create and enforce detailed plans to prevent future burns. This example relates to ((fairness and honesty.))
_____ involves taking someone else’s work and presenting it as your own ((Plagiarism))
If a manager pressures a subordinate to engage in activities that he or she may otherwise view as unethical, such as engaging in accounting fraud or stealing a competitor’s secrets, this would be an ethical issue related to ((business relationships))
If a manager attempts to take credit for a subordinate’s ideas, he or she is engaging in ((plagiarism))
Jim’s boss often goes on Facebook at work. As a result, the rest of his team goes on Facebook as well. This is an example of ((the influence of managers))
Kim’s company, Globotech, does not have established rules and policies on ethics. It is a small, independent company that depends on its employees to use their own judgment. This structure can lead to ((opportunity for misconduct))
According to the National Business Ethics Survey (NBES), _____ is the greatest determinant of future misconduct. ((a company’s ethical culture))
_____ is the act of an employee exposing an employer’s wrongdoing to outsiders ((Whistleblowing))
Obeying the law is a business’s ((legal responsibility))
Globotech created its own continuing education program that offers eligible employees full tuition to pursue continuing education related to their roles at the company. It also created a scholarship program to help employees send their children to college. These programs exemplify the company’s ((philanthropic activities.))
All of the following are criteria the Ethisphere Institute uses to select its annual list of the world’s most ethical companies EXCEPT ((innovation that contributes to the company’s well-being))
Social responsibility is an area of business with issues that ((change constantly in response to society’s demands))
The assurance of both satisfactory quality and service at a fair price is a part of the consumer’s right to ((choose))
John F. Kennedy’s consumer bill of rights outlined four rights: the right to be informed, the right to choose, the right to be heard, and the right to ((safety))
Healthy Foods, Inc. started using a new experimental sugar substitute in its products. However, it didn’t include that this sugar substitute could cause dizziness and nausea anywhere on its packaging. This is a violation of the consumers’ right to ((be informed))
Which of the following is true about the role of the Federal Trade Commission’s Bureau of Consumer Protection? ((It protects consumers against unfair, deceptive, or fraudulent practices))
Judy has realized that she does not like working for others. She wants to open a business in which she will have maximum control and the least interference from government regulations. Which of the following forms of business is best suited for Judy’s needs? ((a sole proprietorship))
Marta owns a small pet grooming company. When a chain pet store with pet grooming services opened across the street from her business, Marta lowered her prices and added more specialty grooming services. Her ability to make these decisions on the spot without anyone else’s approval gave her a ((competitive advantage.))
Josh has been asked to be a part of a new business venture that develops wind energy technologies. Since the business involves high risk, he does not want to be held completely liable for the firm’s debts if the project fails. In this case, Josh is most likely to prefer to participate as a(n) ((limited partner))
Alan, Gary, and Paul are starting a new law firm together, and they are drafting articles of partnership. All of the following are items they should include EXCEPT ((the total money and assets of each partner’s estate))
Paula is a partner in a local hardware store. She has decided to sell her partnership interest. What is one challenge that Paula will face? ((It is difficult to place a value on a partner’s share of the partnership))
Stephanie is a partner in a local hair salon. Without consulting her partners, she ordered some expensive hair products that she thought would greatly improve their business. However, the products were too expensive for the salon’s customers, so they sat on the shelves. What impact will Stephanie’s decision have on her partners? (("Her decision may put her partners’ personal resources in jeopardy.
Catalina opens a fashion boutique with the money she inherited from her parents. She is successful in creating a strong brand image and a loyal customer base. In order to meet the increasing demands of her business, she hires a store manager. However, she finds it hard to let the manager make important decisions about the store. She often finds herself disagreeing with the manager’s decisions, and in the process, she loses a lot of existing customers. Which of the following causes of small-business failure does this scenario best illustrate? ((inability to cope with growth))
Candace started a small house cleaning company, Neat and Tidy, and hired a few of her friends to help her. When the business started, most of its clients were having their houses cleaned to prepare them to sell. However, as more people hired Neat and Tidy and saw what a good job they did, the company started to see more repeat clients who wanted their houses cleaned on a weekly or biweekly basis. As this company grows, all of the following are changes that might occur EXCEPT ((the company’s reputation will suffer, no matter how well the growth is managed.))
Sarah is starting her own small house painting company, Pickett Fence Painting. She is working on developing the company’s business plan. Which of the following is something the business plan should accomplish? ((estimating the business’s income and expenses))
Megan is opening her own small retail store that sells women’s purses and accessories. To start her business, she needs at least $50,000 in initial financing to do all of the following EXCEPT ((pay off any debts so they don’t affect her business))
The act of financing one’s business by using real personal assets is known as ((equity financing))
Shawna’s small flooring business requires more financing than she can provide from her savings. As a result, she is looking for investors to provide financing in exchange for stock ownership. What is a disadvantage of this arrangement? ((Shawna will have to share the profits of her business))
Chang is an interior designer who starts his own consulting firm. His friend Rachel is the owner of a coffee shop. Chang decorates Rachel’s new coffee shop in exchange for a supply of coffee whenever his clients visit his office. Which of the following sources of funding does this scenario best illustrate? ((bartering))
Heather wants to run her own picture framing business, but she doesn’t want to start it from scratch. She decides to buy an existing business. What is one advantage of doing this? ((It provides a built-in network of customers, suppliers, and distributors.))
Gary decides to buy an existing home goods store, instead of starting his own from scratch. Once he takes over the business, he learns that several customers have complained that one of the company’s distributors doesn’t pack items sufficiently for shipping, and their items arrived broken. What is Gary’s responsibility in resolving this problem? ((Gary is responsible for taking on and resolving any problems the business already has))
A license to sell another’s products or to use another’s name in business, or both, is called a ((franchise))
Since Eva has decided to use her savings to purchase the rights to own and operate a McDonald’s outlet rather than starting her own restaurant, she is most likely to be a(n) ((franchisee))
Paul has recently quit his job as an investment banker. He plans to open a restaurant. He has two options: he can either start his own new restaurant from scratch or purchase a franchise from an already established restaurant or fast food chain. His wife Lisa supports the former plan. Which of the following statements strengthens the argument in favor of Lisa’s choice of opening a restaurant independently? ((It is easier and more flexible to make and execute decisions in an independent business.))
_____ are often the Small Business Administration’s principal means of providing direct management assistance to small businesses and entrepreneurs. ((Small Business Development Centers))
Sally is a human resource (HR) manager at a company. A survey of the company’s employees reveals that more than 80 percent of the employees belong to the demographic segment Generation Y. In this scenario, which of the following HR strategies should Sally implement in order to best serve the needs of the employees? ((Sally should use recognition and advancement as the driving forces to motivate employees.))
Norton is a human resource manager at a large multinational company. After a drastic drop in revenue, his company thinks of ways to correct the situation. At a meeting with the top management, the CEO suggested that the company should reduce its workforce by 20 percent. This move would help the company cut costs and continue functioning on the current projects. In this scenario, which of the following strategies is Norton’s company planning to use? ((downsizing))
_____ make decisions about the use of an organization’s resources and are concerned with planning, organizing, directing, and controlling the organization’s activities. ((Managers))
Marilyn is a manager at a large accounting firm. The firm is expanding, and she is tasked with hiring more people to carry out the organization’s increased workload. She must determine all of the following items EXCEPT ((which employees to replace with new candidates.))
Determining an organization’s objectives and deciding how to accomplish them, a management function, is known as ((planning.))
Megan manages an online clothing store. She must determine what her company’s warehouses and distributing facilities need and which type of automation can maximize order efficiency. What major phase of the planning process does this example describe? ((data gathering))
A declaration of an organization’s fundamental purpose and basic philosophy is known as a(n) ((mission))
Cameron started his own photography business. He wants to write a good mission statement that explains his company’s reason for existence. What is one guideline he should follow in writing his mission statement? ((It should be clear and concise.))
Nolan’s jewelry business has grown from a small startup in his room to a chain of outlets all over the country. His business has the potential to expand further, provided he has the resources. To attract investors and other resources, he is required to create a mission statement for his company. If he highlights that his gems come from countries with fair labor practices, which basic question of a well-developed mission statement is he addressing? ((What are our responsibilities with respect to being a good steward of environmental, financial, and human resources?))
Harlan’s hot dog stand wants to earn a profit and satisfy its customers. As a result, it has developed several signature hot dogs that use quality ingredients and mix inventive flavors. What key component of a goal is this example most closely related to? ((an attribute))
In the context of a business’s objectives, which of the following is an ineffective practice? ((isolating them from the business’s mission))
_____ are the ends or results desired by an organization that are derived from the organization’s mission. ((Objectives))
Stephanie’s candy store has profit as an objective. As a result, she wants to have ((money and assets left over after paying off business expenses))
Kendall’s fitness company wants to make the best use of its resources. As a result, she hired college students to work at the front desk, so her personal trainers can focus on helping members in the gym and teaching fitness classes. What type of objective is the most relevant to this example? ((efficiency))
The top management of Dusk Automobiles Inc. has decided that the company’s objective for the next two years will be to expand the overall business internationally. This is an example of ((strategic planning))
EduPub, a large educational publisher, decided to sell its college division because it isn’t as profitable as the school division. This will allow the company to focus its efforts and funds on becoming the leading school publisher. What type of plan is this? ((strategic))
Short-range plans covering a period of one year or less that are designed to implement the activities and objectives specified in an organization’s strategic plan are referred to as ((tactical plans.))
Which of the following is an effective practice with regard to tactical plans? ((They should be periodically reviewed and updated by an organization’s management))
_____ are very short-term plans that specify what actions individuals, work groups, or departments need to accomplish in order to achieve an organization’s tactical plan and ultimately the strategic plan. ((Operational plans))
Xtreme Toys, a company that manufactures toys inspired by popular video games, creates a strategic plan to be the leader in the product category within a period of three years. The company’s middle management also creates a half-yearly plan to ensure that the objectives of the strategic plan are properly implemented. The half-yearly plan created by Xtreme Toys can be regarded as a(n) ((tactical plan))
The top management of XSports Autos Inc. creates a strategic plan to enter new product markets within a period of five years. The company’s middle management creates half-yearly plans to ensure that the objectives of the strategic plan are properly implemented. Along with these plans, each department creates short-term plans that specify the actions to be taken by specific employees and teams to meet the objectives of the strategic plan and the half-yearly plans. These short-term plans established at the departmental level are referred to as ((operational plans.))
Crisis management plans generally cover maintaining business operations during a crisis and ((communicating with others about the situation and the company’s response to the crisis.))
A huge fire broke out in Rustic Rooms, a small rustic furniture store. The fire destroyed all of the furniture, and ultimately, the company went out of business. Which of the following is most likely the reason this company went out of business? ((Rustic Rooms didn’t have an updated contingency plan to handle such a crisis.))
Paul manages a large technology company. One of his roles is structuring the company’s resources and activities to accomplish its objectives efficiently and effectively. What management function does this example pertain to? ((organizing))
Harold’s kitchen appliance company is reorganizing for greater efficiency. How would this company most likely organize the work? ((into teams))
Kevin is a top-level manager at an advertising firm. He has many years of varied experience and he spends most of his time making strategic decisions about how to use the company’s resources. On which management function does he spend most of his time? ((planning))
Lara was less educationally qualified and experienced than her colleagues when she joined Aurora Systems Inc. However, due to her ability to negotiate effectively with clients, convince her team members to work harder, and understand their needs, she is a senior-level manager today. Which of the following skills can be most attributed to Lara’s success in this scenario? ((human relations skills))
Carl is a manager of a large car dealership. He often meets with consumer groups to discuss the safety features of the company’s cars. When he does this, he is acting as a ((spokesperson))
Jasper is a first-line manager at a heating and HVAC company. Although he no longer goes out to customers’ homes and does their repairs, since he worked in the field for over 20 years, he is able to train employees, answer questions, provide guidance, and solve problems. These abilities most closely describe his ((technical expertise.))
Landon is interviewing for a position at a law firm. In the interview, he is asked: "How would you resolve the issue of successfully arguing a case in which you disagree with our client’s position?" Answering this question requires Landon to use ((analytical skills.))
EduTech is a publisher that provides high quality digital educational products to instructors and students. When new employees are hired, they must attend extensive trainings and demos of the company’s products to learn about their functionality and the customers’ experience with the products. Annually, the company sends all its employees and managers on a retreat to take mandatory classes on customer relations and office leadership that address skills related to listening, consistent communication, and handling change without compromising values. This scenario is most closely related to developing ((human relations skills))
Carlos is a manager at a men’s clothing store. He is very particular about how things are done, right down to how the shirts are folded and the positioning of the mannequins. He uses his authority and economic rewards to get his employees to comply with his directions. What type of manager is Carlos? ((autocratic))
Audio Xperience, a stereo and audio electronics company, encourages its employees to work together to make decisions that reinforce the company’s mission and values. Each employee brings his or her own unique insights, and by discussing these insights with other employees, they collectively develop innovative ideas or decisions that most likely would not have been reached by one or two people. This scenario explains ((teamwork))
Gerald is assessing the appropriateness of a proposed option to resolve his publishing company’s plagiarism issue. When considering the consequences of the option, he should ((consider its impact on the organization as a whole.))
Brenda manages a flower shop. Recently, the flower shop has had trouble competing with the local grocery store’s florist. Brenda wants to develop a list of possible courses of action to draw more customers to her shop, so she gathers her team. They discuss the issue and suggest ideas to solve the problem. This is called ((brainstorming))
Maddie’s pet store has decided to abandon one of its dog food brands. To implement this decision, Maddie must do all of the following EXCEPT ((produce a similar brand to fool customers.))
Disadvantages of product departmentalization are that it duplicates functions and does not focus on an organization’s overall objectives. ((TRUE))
_____ refers to a firm’s shared values, beliefs, traditions, philosophies, rules, and role models for behavior. ((Organizational culture))
Jenna is interviewing to work at a doctor’s office. During the interview process, the hiring manager had her come in and talk to some of the employees about the office’s culture. All of the following things she learned relate to the office’s culture EXCEPT ((her job will consist of answering phones, checking patients in and out, and filing patient information.))
Which of the following is a means of informally expressing an organization’s culture? ((employee dress codes))
Which of the following is a formal expression of an organization’s culture? ((codes of ethics))
The managers of Rug Emporium, a large rug store, decide to have a sale. The store manager works with the advertising department to make the public aware of the sale, with department managers to schedule enough salespeople to handle the increased customer traffic, and with merchandise buyers to ensure that enough rugs are available to meet expected consumer demand. What does this scenario best describe? ((how people must work together to achieve the store’s objectives))
Carolyn manages a frozen pizza company. In order to earn profits, she must determine what activities are required to achieve the company’s objectives. All of the following are activities this company would need to do EXCEPT ((talking to competitors to make sure their recipe is unique.))
Kendall works at a small publishing house that groups jobs by functional activities. Her company has departments focused on editorial, marketing, finance, production, legal, and human resources. Which of the following is most likely true of the departments in Kendall’s company? ((Each functional department is managed by an expert in the work done by the department.))
Melanie’s Macaroni is a new restaurant that prepares childhood favorites in new ways to appeal to adults. The restaurant’s owner has assigned an employee to create a new advertising campaign to attract customers. Which of the following is something the employee will need to carry out this assignment? ((the authority to make decisions about the advertising materials and costs))
Which of the following is an advantage of decentralization? ((It may increase the organization’s productivity.))
Donna is a manager of an online clothing store. She has five employees who work remotely in their homes in New York, Texas, California, Colorado, and Florida. In addition to managing her employees and having a phone meeting with each of them weekly, Donna also orders merchandise, manages the inventory, coordinates shipping to customers, and manages the company’s advertising and finances. What type of span of management does this scenario describe? ((narrow))
Philip is a manager at a large office supplies store, where his main responsibility is managing 30 employees. His employees are highly competent, so he doesn’t need to interact with them frequently and few problems arise. His company has a set of specific operating procedures that govern his activities and the activities of his employees. What type of span of management does this scenario describe? ((wide))
Carlos manages a hardware superstore, where he supervises 50 employees. Because he manages a large number of employees, fewer management layers are needed, and thus, his organization is flat with a wide span of management. As a result, what is most likely true of Carlos’s duties, in comparison with managers in tall organizations? ((He performs more administrative duties.))
Gerald is the owner of a popular restaurant. He has appointed a manager who takes care of the day-to-day operations of the restaurant and reports to Gerald at the end of each week. The manager is assisted by a junior manager, who supervises the rest of the staff in the restaurant. Thus, Gerald’s restaurant is using a _____ structure. ((line))
Leonard and Carol are both managers at a home improvement store. Leonard oversees the flooring, lighting, appliances, and hardware departments. Carol provides each of these departments with advice and support in the areas of finance and human resources. Which of the following is true regarding Leonard and Carol’s roles in the company? ((Leonard is a line manager, while Carol is a staff manager.))
Liz is taking a college business class. Her professor splits the class into work teams to complete a project. Each team must create a fictional company, create a business plan for a new product, and market that product in a class presentation. Which of the following would be true of Liz’s work team? ((Team members share leadership and create collective work products.))
Keith’s company uses internal networks to share information with employees. What is an advantage of using this type of network? ((It can help with the flow of everyday business activities))
Mary, a project manager at a software company, informs her subordinates of the organization’s new policies and code of conduct. This is an example of _____ communication. ((downward))
Which of the following is true regarding monitoring communications? ((Failing to monitor employees’ use of e-mail, social media, and the Internet can be costly.))
Maslow’s hierarchy lists the most basic needs people strive to satisfy first at the top of the pyramid. ((FALSE))
An inner drive that directs a person’s behavior toward goals is called ((motivation))
Taylor is an independent contractor in the plumbing industry. He receives work from several companies and individual customers. There are often multiple ways to fix a plumbing problem, and when given the option, Taylor sometimes chooses to use the fastest and easiest solution, rather than the best possible solution. What might explain Taylor’s tendency to avoid doing what he should do? ((lack of motivation))
Larry is a computer salesman who has been working in his field for over 20 years. Recently, he has noticed that his sales haven’t been as high as the sales of some of his younger coworkers. He recognizes the need to increase his sales. Which of the following is something he might do to satisfy this need and achieve success? ((obtain additional training to improve his sales skills))
Victoria is a cake designer and baker at a bakery. She often creates custom cakes for weddings, birthdays, anniversaries, and other large events. She is emotionally involved and committed to her work, which allows her to meet her customers’ high expectations. Victoria can be described as ((engaged))
Creative Cookbooks Inc. is a large publisher that focuses on developing cookbooks with recipes from around the world. Recently, since more people are getting their recipes from the Internet, Creative Cookbooks is struggling to stay profitable. As a result, the company has decided to streamline its operations, laying off 50 employees and changing the way the company works to focus more on offering innovative digital cookbook applications. Many of the company’s employees are frustrated with the changes, so in addition to the employees the company let go, several other employees have quit. This scenario is most closely related to ((low morale.))
Javier works at a grocery store. He started six months ago as a bagger, but recently, his boss recognized his hard work, promoting him to cashier and giving him a raise. Javier’s promotion and raise are examples of ((extrinsic rewards))
Becky works in a factory that produces dolls and other toys. To improve productivity, Becky’s manager broke down each job into its component tasks, determined the best way to perform each task, and specified the output to be achieved by each worker performing the task. As a result, Becky is expected to add arms and legs to 50 dolls per day. She is paid $2.50 per doll, but if she exceeds 50 dolls in a day, she receives an additional $1.50 per doll. This scenario relates to ((the classical theory of motivation))
Larissa worked at the Hawthorne Works Plant of the Western Electric Company in 1924, when Elton Mayo and his team started their research on workplace conditions and productivity. Throughout the study, as Mayo and his team introduced various physical conditions, Larissa’s productivity increased. When the researchers asked her about her behavior, she expressed ((satisfaction because her supervisor had asked for her help and cooperation in the study.))
The American Painters’ Association provides discounted house painting services to military and veteran families, which enlivens its employees. This shows how important it is for employees to feel like ((their work matters))
According to Maslow’s hierarchy of needs, which of the following needs do people strive to satisfy first? ((physiological needs))
According to Maslow’s hierarchy of needs, the need for love, companionship, and friendship—the desire for acceptance by others—is a part of the _____ needs. ((social))
Which of the following needs is at the top of Maslow’s hierarchy of needs? ((self-actualization needs))
Blujay Pharmaceutical features the salesperson who has been the most successful during the previous quarter in the company newsletter. According to Maslow’s hierarchy of needs, which of the following needs of employee is Blujay Pharmaceutical trying to meet? ((self-actualization needs))
Maslow’s theory maintains that ((the more basic needs at the bottom of the hierarchy must be satisfied before higher-level goals can be pursued.))
Garrett works as an electrician, going into people’s homes and offices to set up and repair electrical systems. When he sees a dangerous condition, he reports it to the home or business owner immediately and begins working to fix the situation. He wears safety goggles and gloves while he is working, and he has a comprehensive insurance plan with income protection in the event that he becomes unable to work. Garrett’s actions are most related to Maslow’s _____ needs. ((security))
Travis got laid off from his job at a hay and grain supply company. Before he was laid off, he was focused on selling more hay and grain than any of his co-workers. Since the layoff, he has shifted his focus to finding a new source of income. What does this scenario highlight about laid off workers, in terms of Maslow’s hierarchy of needs? ((They shift their focus from high-level esteem needs to the need for security.))
According to Herzberg’s theory, which of the following is a hygiene factor? ((salary))
According to Herzberg’s theory, which of the following is a motivational factor? ((responsibility))
According to Herzberg’s theory, the absence of which of the following factors may be a potential source of dissatisfaction and high turnover? ((security))
According to Herzberg’s theory, the presence of which of the following factors is likely to motivate employees though their absence may not result in dissatisfaction? ((achievement))
Hailey has a job as a dental hygienist. She makes a good annual salary and her job is secure. Which of Maslow’s needs would these hygiene factors help Hailey satisfy? ((physiological and security))
Maritza manages a large boarding stable that employs several people who clean stalls, feed the horses, teach riding lessons, and do training rides on boarders’ horses. Which of the following is a way Maritza can motivate her employees to work harder, in terms of content of the work itself? ((She can involve them in the stable’s processes and decisions.))
Herzberg’s motivational factors are similar to Maslow’s ((esteem and self-actualization needs))
In the United States, workers’ _____, as defined by Maslow, have largely been met by minimum-wage laws and federal occupational safety standards. ((physiological and security needs))
According to managers who adopt McGregor’s Theory X ((workers must be forced to do their jobs.))
McGregor’s Theory X corresponds to ((the traditional view of management))
McGregor’s Theory Y corresponds to ((the humanistic view of management.))
When compared to Maslow’s hierarchy of needs, the Theory X style of management focuses on ((physiological and security needs.))
The managers of Automotive Parts Emporium believe that the average person dislikes work and will avoid it when possible. As a result, they focus on coercing, controlling, directing, and threatening their workers with punishment to get them to work toward achieving the company’s objectives. One method they initiated was an employee ranking system that ranked 60 percent of employees as below expectations. This company has adopted ((McGregor’s Theory X perspective.))
Mary, a manager at a publishing house, follows McGregor’s Theory Y approach to management. She is likely to assume that ((workers can contribute through imagination and ingenuity))
Jill is the principal of a charter school. She believes that her employees like to work and that under proper conditions, they will seek out responsibility in an attempt to satisfy their social, esteem, and self-actualization needs. What theory of management would Jill most likely follow? ((McGregor’s Theory Y))
Which of the following is a major difference between the Theory X and Theory Y views of management? ((Theory Y takes into account people’s needs for companionship, esteem, and personal growth, whereas Theory X does not.))
In regards to Maslow’s hierarchy of needs, what is the difference between Theory X and Theory Y managers? ((Theory X managers focus on physiological and security needs, while Theory Y managers address Maslow’s high-level needs as well as physiological and security needs.))
Lorenzo’s Italian Restaurant has several locations in Massachusetts and New York. Each location has a restaurant manager who runs his or her location as he or she sees fit. The restaurant chain has achieved success by empowering its employees to make their own decisions and follow their passions. Which management theory does Lorenzo’s Italian Restaurant follow? ((McGregor’s Theory Y))
_____ states that motivation depends not only on how much a person wants something but also on the person’s perception of how likely he or she is to get it. ((Expectancy theory))
Rahul works as an assistant manager in a car manufacturing firm. He wishes to be promoted to the position of manager but feels that it is unlikely to happen. As a result, Rahul is not motivated to achieve the desired end. Which of the following theories best explains Rahul’s situation? ((expectancy theory))
Calvin is a marketer at Like-New Car Company, and he wants to inform the market of the benefits of buying used cars over expensive new cars. He invests his efforts in creating advertisements that emphasize the company’s low prices and service guarantee to reach the company’s target market and compete with the large car dealerships in the area. This scenario relates most directly to ((goal-setting theory.))
Elias, the manager of Eat Big, a chain of restaurants, gives awards on a monthly basis to employees with the highest productivity. Elias’s action is based on ((Skinner’s behavior modification theory))
Stag Industries annually rewards the top ten employees, in terms of goal achievement, with a trip to the Caribbean. What effect will Stag Industries’ action most likely have on employee behavior? ((It will motivate all employees to work hard to achieve company goals.))
Jennifer, the store manager at a supermarket, penalizes employees for reporting late to work. This measure has gone a long way in ensuring that employees arrive on time. The step taken by Jennifer best reflects ((Skinner’s behavior modification theory.))
Which of the following statements is true regarding the two strategies used in Skinner’s behavior modification theory? ((In general, rewarding appropriate behavior is a more effective way to modify behavior than punishing unacceptable behavior.))
Mandy manages a small diner. She has noticed that some of her employees steal fries and other food off patron’s plates before bringing the food out to the tables. When she sees employees doing this, she punishes them by taking a portion of their tips and distributing that money to the employees who don’t steal food from plates. Which of the following is an undesirable long-term side effect that may result from such punishments? ((there may be increased turnover))
Historically, Aaron’s Automotive has used specialization to increase productivity. However, Aaron has noticed that his employees seem bored with their regular tasks and productivity is actually declining. If Aaron decides to reduce employee boredom by allowing workers to undertake a greater variety of tasks and by giving them the opportunity to learn new skills, he is most likely adopting ((job rotation))
What is an advantage of job enrichment? ((It enhances a worker’s feeling of responsibility))
A compressed workweek is most likely to ((reduce a company’s operating expenses.))
Simone works as a medical transcriptionist. She is required to log in 40 hours in four days and take three days off. Simone’s work schedule is an example of ((a compressed workweek.))
It is not necessary for a company to verify an applicant’s references because most applicants tell the truth on applications or résumés. ((FALSE))
Compensation for individuals within a specific job category depends on the compensation for that job and not on the individual’s productivity. ((FALSE))
Human resources management refers to ((all the activities involved in determining an organization’s human resources needs, as well as acquiring, training, and compensating people to fill those needs.))
All of the following are reasons human resources management has increased in importance over the past few decades EXCEPT ((today’s employees need to be directed by autocratic managers.))
Megan has worked in the publishing field for over 15 years. She was recently laid off from her job, and she is using job search websites to find a new job. When she finds a job title that looks interesting, she can click on it to read an overview of the job’s tasks, relationships with other jobs, the physical and mental skills required, duties, responsibilities, and working conditions. If this job _____ looks like something she would want to do, she can submit an application. ((description))
Mary has been an editorial assistant at a small trade publisher for two years. When an associate editor position opens up on her team, she applies for the position and gets it. What is one issue the company will have as a result of promoting Mary into this new position? ((The company will have to fill Mary’s vacant editorial assistant position.))
Lenny’s Carpentry and Painting has recently received many calls looking for help tiling kitchens and bathrooms. Since none of the contractors that work for this company specialize in tiling, Lenny is looking to hire a new tiling expert to complete these customer projects. To recruit potential candidates, the company has created a career website to provide employment information and take applications. Which of the following is a reason Lenny might hire an external candidate instead of hiring from within? ((There are better-qualified people outside of the organization.))
Which stage of the employee selection process allows management to obtain detailed information about an applicant’s experience and skills, reasons for changing jobs, and an idea of whether the person would fit in with the company? ((interviewing))
Tara has recently interviewed for a receptionist position at a dental office. The interview went well, and now, the human resources manager has asked her to complete the Myers-Briggs Type Indicator Test. Which of the following is a reason the human resources manager would ask Tara to complete this test? ((to assess her ability to fit into the organization’s culture))
Naomi has just arrived for her first day at work at GloboTech, a large technology company. The first thing the hiring manager, Evan, does is take her on a tour of the building, introducing her to her co-workers and showing her where the kitchen, meeting rooms, and her cubicle are located. After the tour, Evan leaves Naomi in her cubicle with the company’s employee handbook, so she can read about the organization’s policies and benefits. Evan tells Naomi that he will be back in two hours to bring her to lunch with the entire team. All these activities are part of ((orientation))
Sampson’s Sporting Goods Store has recently hired several new employees who have the knowledge, skills, and abilities the company needs. Why is it still important that these new employees undergo training? ((to teach them how to do their specific job tasks))
Vikram is an employee at Monsanto research lab. In an effort to improve his managerial skills and prepare him for a promotion, his company sends him on a series of experiential management training exercises at the company’s expense. By helping him increase his knowledge, Vikram’s company is focusing on ((development))
_____ is one of the most difficult tasks for managers. However, doing so is essential because it gives employees feedback and generates information about the quality of the firm’s selection, training, and development activities. ((Performance appraisal))
In the context of a performance review, _____ refers to an employee’s willingness to identify and address opportunities for improvement. ((initiative))
Ryan works at an electronics store. During his performance review, his manager praises him for always responding quickly to requests, showing up on time for all his shifts, and always doing his best to complete his tasks correctly and thoroughly. In this scenario, the manager is praising Ryan’s ((dependability))
Joe was recently moved to a new position in his company that involved learning new skills and undertaking work assignments. However, he received no salary raise. Joe’s change in position can best be described as a(n) ((transfer))
Which of the following statements is true of a separation? ((It is an employee’s departure from an organization))
Socorro was terminated from her job because she was repeatedly late to work. She contributes to her company’s turnover through ((separation))
Which of the following should managers do to avoid lawsuits from individuals who may feel they have been fired unfairly? ((document all problems and warnings in the employee’s work records))
Kendall has worked for a heating, oil, and propane company for five years. Recently, her company has downsized, laying off several employees. After the layoffs, Kendall’s workload increased but her compensation stayed the same. As a result, Kendall has decided to leave the company. On her last day, the human resources manager has scheduled a meeting with Kendall to determine why she is leaving the organization. This meeting is called a(n) ((exit interview.))
_____ is a public protest against management practices and involves union members marching at an employer’s plant or work site. ((Picketing))
Nick works at a bottling plant for a soft drink company. He is part of a labor union, along with his co-workers. After the plant’s management and the labor union failed to agree on contract terms, the union used picketing and advertising to ask its members and the public not to buy the company’s products. This scenario best describes ((a boycott.))
Which of the following is management’s version of a strike? ((a lockout))
The management and a labor union at a car parts manufacturing plant are struggling to reach an agreement regarding benefits. They bring in a neutral third party to settle their dispute. The third party’s solution is legally binding and enforceable. Thus, the third party is a(n) ((arbitrator))
The difference between an arbitrator and a mediator is that ((an arbitrator’s solution to a labor dispute is binding on the participants, while a mediator’s solution is not binding.))
Meredith is a human resources manager at a large advertising firm that does business internationally. A significant part of her job is dealing with the diversity of the company’s customers and employees. Understanding this diversity means ((recognizing and accepting differences and valuing the unique perspectives such differences can bring to the workplace.))
William is the human resources manager at a large law firm that has historically been dominated by white men. As the firm’s clients have become more diverse, William sees the need to recruit, develop, and retain more diverse employees. All of the following are things William can do to attract and retain a more diverse group of employees EXCEPT he can ((hire minority candidates for all open positions, regardless of qualifications))
Abby is a doctor at a hospital where most of the doctors are men. The other doctors have made a lot of sexist comments, indicating that she should be a nurse, not a doctor, and that the job is too difficult and complex for a woman. Hurt by these comments, Abby discussed the situation with her boss, who told her to brush off the comments and just do her job. Since this hospital doesn’t seem to value its diverse employees, like Abby, all of the following are consequences it may face EXCEPT ((decreased turnover.))
Creating an innovative product that meets many users’ needs is sufficient in today’s volatile global marketplace. ((FALSE))
Trying to determine customers’ true needs is difficult because no one fully understands what motivates people to buy things. ((TRUE))
Jasmine went to a new salon in the area to get her hair cut and colored. However, the stylist cut her hair shorter than she wanted and wouldn’t use the bold red color Jasmine wanted. As a result, Jasmine did not feel good about her purchase. What does this most likely mean for the future of this exchange relationship? ((Jasmine will go to a different salon and stylist for her next cut and color.))
All of the following are examples of the activities marketing performs to accomplish objectives and generate exchanges EXCEPT ((Jim doesn’t want his car anymore, so he leaves it on the side of the road with a "free" sign on it.))
A customer’s subjective assessment of benefits relative to costs in determining the worth of a product is known as ((value))
Jeremy has decided to buy a new car. Since he travels a lot, he is looking for a car that gets great gas mileage and has enough room for his luggage. He has done a lot of research online and gone to several car dealerships to test drive cars. He has finally decided to buy a Toyota Prius because he liked driving it, and it fits his criteria for great gas mileage and storage space. However, in order to buy the car, Jeremy must take a car loan. He also must get the car inspected and registered before he can start driving it. In this scenario, which of the following is a nonmonetary cost associated with Jeremy’s car purchase? ((the time taken to test drive various car models))
Monica owns an equestrian clothing store. After many customers complained that her clothing only fit petite riders, she decided to add a new brand of riding clothes that makes clothing for all body shapes and sizes. In regards to the marketing concept, which of the following statements describes Monica’s decision? ((To remain competitive, companies must be prepared to add to or adapt their product lines to satisfy customers’ desires.))
Auto Giants is a large car dealership. It could sell its cars for $100 and give customers a lifetime guarantee, but based on the marketing concept, why shouldn’t it do this? ((The company should strike a balance between achieving organizational objectives and satisfying customers.))
Gary worked for a textile manufacturer during the first half of the 20th century. When the supply of his company’s textiles exceeded customer demand, he realized that he would have to invest more money in advertising his products and go directly to his customers to convince them to purchase his products. Based on this scenario, Gary has most likely adopted the _____ orientation. ((sales))
Melinda is a marketing manager at a company that sells bicycles. A large part of her role is trying to assess what customers want and building long-term relationships with customers. Which of the following would likely complicate Melinda’s job? ((Trends, fashions, and tastes can change quickly.))
Bobby owns a computer store that sells new and used computers and has a service department. When customers buy computers at his store, he offers a service package at a discounted rate. Additionally, he has an ongoing promotion that allows customers to trade in their old computers for new ones at much lower prices than his competitors can offer. He is able to do this because his service department can fix customers’ old computers, which allows him to resell them to other customers. Based on this scenario, Bobby is focused on ((enhancing the profitability of existing customers and extending the duration of customer relationships.))
Tory is a marketing manager for a large chain of sporting goods stores. A significant part of her job is analyzing the purchase data of the company’s customers. All of the following are benefits of doing this EXCEPT it ((allows the company to focus on targeting groups of similar customers.))
Ferava Inc. is a company that markets its products—luxury wristwatches—exclusively to high-income individuals and celebrities. Thus, high-income individuals and celebrities are most likely Ferava Inc.’s ((target market.))
Rustic Rooms Furniture produces and sells unique furniture pieces to both individual homeowners and hotels. As a result, this company’s products can be considered ((both consumer and business products))
Mitch’s Sporting Goods Store decides to focus its new marketing campaigns on its yoga, softball, and tennis equipment in an attempt to attract more female customers. Which of the following is most likely the reason this store might want to focus on this market segment? ((Women are the largest market segment))
For a firm to successfully use a concentration or multisegment approach to market segmentation, all of the following requirements must be met EXCEPT ((consumers’ needs for the product must be homogeneous))
Which of the following bases for market segmentation pertains to some characteristic of the consumer’s actions toward the product—a characteristic that commonly involves some aspect of product use? ((behavioristic))
Prism Phones wants to target college students and young professionals. The company has therefore decided to advertise its cell phones over social networking and other popular websites. Relative to television advertising, this strategy will help Prism Phones save costs and thus allow the firm to sell its phones at more competitive prices. Prism Phones’ decision to advertise through different media than the ones adopted by other firms illustrates the _____ activities of the marketing mix. ((promotion))
Personal Fitness Gym provides its clients with personal training sessions, small group exercise classes, fitness machines and weights to use, and a separate kids’ gym with day care specialists, so members can bring their children to the gym with them. These items are most closely associated with the _____ aspect of the marketing mix. ((product))
Oliver’s Oil Company wants to increase customer demand for oil. What can Oliver do to accomplish this? ((reduce the price of his oil))
Global Mart Inc., a multinational chain of supermarkets, wants to open a new store in a country called Alheroni. Before launching itself in the new market, the management of Global Mart Inc. wants to collect primary data about Alheroni’s market. Which of the following sources of information will it most likely use? ((the data collected from anonymous surveys conducted in malls in Alheroni))
All of the following are examples of companies using online marketing research EXCEPT ((Quality Cars uses mystery shoppers to visit its car dealerships and report on whether the dealerships are adhering to the company’s standards of service.))
Gabrielle is a marketer for an online shoe store. An important task she performs regularly is measuring the website’s traffic and performance. This task is known as ((marketing analytics))
Anna is an environmental activist who avoids buying paper-based products unless they are made from recycled paper. Which of the following psychological factors is primarily responsible for Anna’s buying behavior in this scenario? ((attitude))
Derek dislikes buying branded clothes because he claims that they are a waste of money. In this scenario, Derek’s buying behavior is most likely related to his ((attitude.))
Jennifer is highly influenced by celebrities in movies and advertisements. She wishes to dress like them and adopt their lifestyle. When she goes shopping, these celebrities are her point of comparison. In this scenario, the celebrities are Jennifer’s ((reference group))
Every time David wants to change his cell phone, he takes the opinions of his colleagues and friends to decide which phone to buy. Thus, David’s colleagues and friends are his ((reference group))
Which of the following is an example of how a person’s social role can influence his or her buying behavior? ((Andrea has five children, so she bought a station wagon))
Due to the increasing purchasing power and per capita income of the population of United Nombavia, True Couture Inc. decided to launch its premium line of clothing and bags in the country. Which of the following environmental forces primarily influenced True Couture Inc.’s decision? ((economic forces))
Which of the following is a social force that influences the marketing environment? ((the public’s opinions about the environment))
Daily Stop, a supermarket in California, was criticized by the community for its use of plastic bags. In view of the concerns raised by the members of the community, the supermarket’s management made the decision to start using biodegradable bags. This decision has been influenced by _____ forces. ((social))
Kevin’s music store developed a smartphone application to improve its distribution, promotion, and new-product development. In this scenario, Kevin’s store is using _____ forces. ((technological))
The balance sheet presents a "snapshot" of an organization’s financial position at a given moment. ((TRUE))
Cash from investing activities is calculated from changes in the long-term liability accounts and the contributed capital accounts in owners’ equity. ((FALSE))
Matt is an auto mechanic who runs his own small shop. When tax season comes, he hires an accountant from one of "the Big Four" accounting firms to help him file his business’s tax return. In this scenario, Matt has most likely hired a _____ accountant. ((certified public))
Amanda is a certified public accountant who provides accounting services to individuals and businesses. Which of the following rights does her certification give her? ((the right to express, officially, an unbiased opinion regarding the accuracy of a client’s financial statements))
Vikram is an accountant employed by a large corporation. He works for the billing and taxation department, preparing and analyzing the company’s financial statements. He is most likely to be a _____ accountant. ((private))
Carol is an accountant who completed course work beyond a basic four- or five-year college accounting degree. Which of the following is something Carol’s additional training allows her to do? ((understand, interpret, and even develop the sophisticated accounting systems necessary to classify and analyze complex financial information))
Harold owns a chain of coffee shops. He hires accountants to summarize the information from his company’s business transactions into a financial statement. Which of the following is a reason Harold would use a financial statement? ((to aid in internal planning and control))
Gary’s Gadgets hired a managerial accountant to help forecast the company’s expenses and income for its first year in business. This plan for how Gary’s Gadgets plans to move from Point A to Point B over the course of the year is its ((budget))
Potential investors typically study the _____ in a firm’s annual report to determine whether the company meets their investment requirements. ((financial statements))
Small Bank, a short-term lender, examines Frank’s Kitchen Fixtures’ _____ to assess its ability to repay a loan quickly with cash generated from sales. ((cash flow))
Thomson is a manager at a bank. He has to decide whether to lend $10,000 to Safe Toys, a company that produces toys that don’t have a choking hazard. He should ((review the company’s financial statements.))
When lending money, a long-term lender is more interested in a company’s _____ and indebtedness to other lenders. ((profitability))
Kerry works for a large technology company as an engineer. As an employee, why should he look at his company’s financial statements? ((to establish reasonable expectations for salary and other benefit requests))
Emily’s Embellishments, a craft store, displays various types of paper, stickers, glue sticks, yarn, canvases, paints, and other craft supplies on shelves. These items are examples of the company’s ((assets))
What is meant by owners’ equity? ((It refers to the money contributed to a firm that never has to be paid back.))
When Harvey opened Harvey’s Hardware Store, he convinced his family and friends to invest in the store. They invested money and assets to help him start and grow his business. The money and assets these investors contributed to Harvey’s store comprise his ((owners’ equity))
Which of the following equations is equivalent to the accounting equation? ((owners’ equity = assets - liabilities))
What is the last step that must be taken before a firm’s books can be closed for an accounting cycle? ((A certified public accountant must attest that generally accepted accounting principles were used.))
An income statement shows ((an organization’s profitability over a period of time.))
Leslie owns a childcare company. Since this market is competitive, she has hired an advertising and sales manager to make sure potential customers know about her company. As a result, she must pay for advertising materials and her advertising and sales manager’s salary. These expenses would be classified as _____ expenses. ((selling))
Craig’s Construction Company bought a crane for $200,000, which it expects to be able to use for 50 years. Rather than showing an expense of $200,000 in the first year and no expense for the crane over the next 49 years, Craig can report depreciation expenses of _____ per year in each of the next 50 years because that better matches the cost of the machine to the years it is used. (($4,000))
Which of the following is true of a piece of equipment when it is fully depreciated? ((It has a zero value on the firm’s balance sheet.))
The _____ presents a snapshot of an organization’s financial position at a given moment. ((balance sheet))
A company’s assets that are used or converted into cash within the course of a calendar year are called ((current assets.))
A company’s financial obligations to short-term creditors, which must be repaid within one year, are called its ((current liabilities))
_____ includes the owners’ contributions to a company along with income earned by the company and retained to finance continued growth and product development. ((Owners’ equity))
Each type of stock issued by a company represents a different claim on the organization, thus each must be represented by a separate owners’ equity account called ((contributed capital.))
The financial statement that explains how a firm’s cash changed from the beginning of the accounting period to the end is called the ((statement of cash flows))
Paul’s Plant Store wants to compute the revenues and expenses for its most recent accounting period to determine if it is making extra cash that it can use to invest in increased long-term capacity or to pay off its debts. To determine this, Paul should calculate cash from _____ activities by combining the changes in the revenue accounts, expense accounts, current asset accounts, and current liability accounts. ((operating))
Christy’s Craft Company is paying off its long-term debt from the bank loan Christy secured to open the business several years ago. What does this scenario indicate? ((Christy’s Craft Company has a negative amount of cash from financing activities))
_____ analysis refers to calculations that measure an organization’s financial health ((Ratio))
Which of the following calculations helps managers compare an organization’s productivity, profitability, and financing mix with other similar entities? ((ratio analysis))
One of the ways to analyze a company’s financial performance is to compare its performance with those of the ((companies in the same industry))
Aaron’s Automobiles experienced a huge recall on one of its best selling cars, and it had to spend $10 billion to fix all the recalled cars. Which of the following is most likely the result of this unexpected and expensive charge? ((decreased earnings per share and significantly reduced profitability ratios))
Camille’s Closets, a closet design company, has a low return on equity. What does this indicate? ((Immediate managerial attention is needed.))
Which of the following equations is used to calculate return on equity? ((return on equity = net income/owners’ equity))
Maurice was asked to extend trade credit to a restaurant she hadn’t serviced before. She asked to see its balance sheet to determine if it could pay its bills. She divided its current assets by current liabilities to get its ((current ratio))
Debbie’s Doormats has a high inventory turnover ratio because Debbie frequently sells and replaces her inventory over the course of a year. Which of the following might this scenario indicate? ((great efficiency))
The measure of liquidity that reflects a company’s liquidity ratio after excluding its inventory is the _____ ratio. ((quick))
Chloe’s Cosmetics doesn’t have a lot of inventory. As a result, its _____ ratio is almost exactly the same as its current ratio. ((quick))
_____ ratios provide information about how much debt an organization is using relative to other sources of capital, such as owners’ equity. ((Debt utilization))
Which of the following explains why the earnings per share ratio is important? ((It determines a company’s overall stock price.))
Louis, Carl, and David have secured a charter for their car dealership. They hold an organizational meeting to establish the corporation’s bylaws and elect a board of directors. What is one thing the bylaws might do? ((set up committees of the board of directors))
Wilson’s Wishing Wells, Inc. is a company that produces wells and well equipment. It has been owned and operated by the Wilson family for 50 years. The Wilsons own all the corporation’s stocks, and they are not required to disclose financial information publicly. What type of business is this? ((private corporation))
AgroCorp is a public corporation that experienced difficulty growing its agricultural products during an especially dry summer season. Wanting the flexibility to make decisions for restructuring AgroCorp’s operations, its managers bought all the stock, thus ((taking it private.))
A written authorization that assigns a stockholder’s voting privilege to another is called ((proxy.))
Due to a severe recession, GloboTech is struggling to operate and earn profits. In the current economic state, people don’t have extra money to buy its televisions, stereos, and home theater systems. If these economic conditions continue, GloboTech will be forced to ((file bankruptcy))
Jim and Carol both work in the real estate market. One day, a great investment property comes on the market, and they decide to pool their money to buy and renovate it. They plan to spend six weeks on the renovation, and they hope to make a nice profit when they turn around and sell it. Jim and Carol’s partnership is an example of a(n) ((joint venture.))
Which of the following is true of limited liability companies? ((They protect the members’ personal assets in case of lawsuits))
Gwen is a doctor who is forming her own medical practice. She wants her practice to be flexible, simple to run, and free from required meetings, minutes, or resolutions. Her goal is to blend the best characteristics of corporations, partnerships, and sole proprietorships. What form of business ownership seems most in-line with Gwen’s goals? ((limited liability company))
When a corporate raider wants to acquire or take over another company, it first offers to buy some or all of the other company’s stock at a premium over its current price in a(n) ((tender offer.))
Growing up, Cameron noticed that many families in his community struggled to make ends meet and provide for their children’s education. As a result, after he finished school, he started a business to provide low-cost tutoring and scholarship opportunities for low-income families in his community. As his company has grown, Cameron has been able to extend free tutoring and college scholarships to five impoverished students per year. Cameron is a ((social entrepreneur))
Vernon runs Buzz, his own event management company, which frequently organizes large corporate events. He has built a good reputation for organizing corporate events with a fair degree of professionalism at reasonable costs. He works out of his own house with a staff of about 55 employees. Vernon’s company can be categorized as a ((small business))
Salma owns a Lebanese restaurant that employs around 25 people. She is solely responsible for the management of the restaurant. Her business has to compete with a larger chain of restaurants that offer the same cuisine. Salma’s restaurant can be classified as a ((small business.))
When Miguel first moved from Mexico to the United States, he struggled to find and keep work because he couldn’t speak English fluently. After learning English, he started a new business to help other Mexican Americans learn English and to help place them in jobs. Miguel’s business is a ((minority-owned small business.))
Jack owns a barbershop that specializes in men’s haircuts and shaving. This is an example of a ((service business))
Ben and Molly live and work in Texas, but they love vacationing in Hawaii, and they go there as often as time and money will allow. Finally, they take the plunge and buy a vacation home in Hawaii. When they aren’t there, they rent the property to other people for $1000 per week. This is an example of ((the sharing economy.))
Simon has worked for a large corporation for years, and he feels like he just doesn’t fit in the corporate mold" anymore. He decides to start his own business because he wants the freedom to choose whom he works with and when and where he works. What advantage of small business ownership seems the most prevalent in this example? ((independence))
________ make decisions about the use of an organization’s resources and are concerned with planning, organizing, directing, and controlling the organization’s activities. ((Managers))
Which of the following is a means of informally expressing an organization’s culture? (( employee dress codes))
Top-level managers make an organization’s strategic decisions that focus on a key idea for using resources in order to take advantage of opportunities. (( True ))
make decisions about the use of an organization’s resources and are concerned with planning, organizing, directing, and controlling the organization’s activities (( managers ))
are very short-term plans that specify what actions individuals, work groups, or departments need to accomplish in order to achieve an organization’s tactical plan and, ultimately, the strategic plan. ((operational plans))
Short-range plans covering a period of one year or less that are designed to implement the activities and objectives specified in an organization’s strategic plan are referred to as ((tactical plans)
Determining an organization’s objectives and deciding how to accomplish them, a management function, is known as ((planning.)
What is true of wholesalers? (((Wholesalers are extremely important because of the marketing activities they perform.)))
Many service providers are considered retailers because they (((provide their services directly to consumers)))
A merger occurs when (((two companies combine to form a new company)))
A(n) ________ is a partnership established for a specific project or for a limited time. (((joint venture)))
An entrepreneur has been primarily associated with the willingness to((take risks.))
What is one of the difficulties faced by small business owners?((worries about employee problems))
What is a difference between high technology businesses and other small businesses?((High technology businesses require greater capital and have higher initial startup costs thanother small businesses))
What is true of a capitalist economic system?((Prices of goods and services are determined by supply and demand))
The quantity of goods and services that consumers are willing to buy at different prices at a specific time is referred to as((demand.))
The quantity of products that businesses are willing to sell at different prices at a specific time is referred to as((supply.))
What is a defining characteristic of public corporations?((Their stock can be bought, sold, or traded by anyone))
Which business provides a service, but is neither owned by the government nor focuses on earning profits?((a nonprofit corporation))
What is a true statement about the board members of a corporation?((They have a duty of care and loyalty to oversee the management of the firm.))
Preferred stockholders of a corporation((have a claim to profits before other stockholders do.))
Which of the following is NOT a question to consider in determining whether an action is ethical? ((Is there significant financial upside for your firm to engage in this particular activity?))
A firm's ________ resources are also known as labor. ((human))
A clear and concise declaration that explains the organization’s reason for existence is a(n) (( mission statement ))
A company needs to hire three new employees. It wants to select people from a pool of qualified applicants. What is the process of forming this pool of applicants? (( recruitment ))
Social networks such as Facebook and Twitter enable managers and subordinates to _________blank, or get information and advice from other people. ((network))
The first step in the decision-making process is developing options. ((False))
Involvement and appreciation boosts morale for employees. (( True ))
Max is a manager in a warehouse, and he needs to hire a new assistant manager. What is the first stage of the selection process? (( the application ))
In the United States, wage differences between women and men are acceptable if they are (( attributed to seniority. ))
Magnus supervises employees who work on the assembly line in a manufacturing plant. Magnus is a middle manager. (( False ))
A _________ structure is also called a project management structure. (( matrix ))
During the COVID-19 pandemic, many companies had to eliminate employees from their organizations to stay in business. This is called downsizing. (( true ))
Dalida is assigned to a demanding project with a tight deadline. During the project, she works overtime, postpones a vacation, and agrees to learn how to perform new tasks to help speed up the project timeline. At the end of the year, she learns that a co-worker who put in less work on the project is receiving a higher annual raise than she is, so she begins searching for a new job. This is an example of (( equity theory ))
What is a formal expression of an organization's culture? (( codes of ethics ))
A major challenge with the increasing use of smartphones and tablet computers is ((the blurring between leisure and work time.))
Communication is just as important in an organization as finance, human resources, or marketing. (( True ))
________ programs are legally mandated plans that try to increase job opportunities for underrepresented groups. (( Affirmative action ))
What is the first step in decision making? (( recognizing and defining the decision situation ))
Job ________ incorporates motivational factors such as opportunity for achievement, recognition, responsibility, and advancement into a job. (( enrichment ))
Strategic objectives are expressed in general terms and do not contain specific, quantifiable metrics of where the firm is now or where it is going. (( False ))
The field of human relations has become increasingly important over the years as businesses strive to understand how to boost company sales. (( False ))
All teams are groups, but not all groups are teams. (( True ))
High-level managers make an organization's strategic decisions that focus on a key idea for using resources in order to take advantage of opportunities. (( True ))
The ________blank is a federal agency established by the Civil Rights Act of 1964 that is dedicated to increasing job opportunities for women and underrepresented racial and ethnic groups and eliminating job discrimination based on race, religion, color, gender identity, sexual orientation, national origin, or disability. (( Equal Employment Opportunity Commission ))
Wages that are too high may result in the company's products being priced too high, making them uncompetitive in the market. (( True ))
Which of the following is a component of the marketing mix? (( Pricing ))
What is the main characteristic of mixed economies? ((Elements from multiple systems))
Technology has no significant impact on small business competitiveness. ((False))
Sole proprietorships are subject to more regulatory scrutiny compared to corporations. ((False))
What is equity financing? ((Investing your own cash or selling stock))
If a business owner wants the ability to have her business continue indefinitely beyond her lifetime, she should form a: ((Corporation))
What disadvantage do sole proprietorships face regarding funding? ((Scarce external funding))
Abusive or intimidating behavior in the workplace is associated with: ((Bullying))
Why is studying business important? ((It helps in becoming a better consumer and understanding the role of businesses in society))
Bribery is defined as: ((Payments or favors intended to influence the outcome of a decision))
How does plagiarism relate to business ethics? ((It is taking someone else's work and presenting it as your own, an unethical practice))
What legal document is necessary to establish a partnership?((A partnership agreement))
What is the purpose of business ethics? ((To guide acceptable behavior/conduct in business))
What is a benefit of franchising for the franchisee?((Brand-name appeal))
A corporation's profits are taxed at the corporate level and then again as personal income when distributed as dividends. ((True))
A local bakery decides to use organic ingredients, distinguishing its products from competitors. This strategy exemplifies: ((An oligopoly market.))
Economic systems do not influence a business's operational environment. ((False))
Khalid is the sole owner of a bakery, which business structure is this? ((Sole Proprietorship))
In what way do mixed economies differ from pure economic systems? ((They blend elements from different economic systems to suit their needs.))
The majority of small businesses fail within their first year. ((False))
An employee consistently uses work time for personal phone calls. What is the ethical issue? ((Misuse of company time))
How can a business plan benefit a small business? ((Serving as a roadmap for success and attracting financing))
How can unethical behavior negatively impact a company? ((Leading to conflicts of interest, legal issues, and damaged reputation ))
What is a stakeholder in business? ((Groups interested in business outcomes ))
Misuse of company time refers to: ((Spending excessive time on personal activities during working hours ))
Why do many small businesses fail? ((Managerial inexperience or incompetence ))
Samah is thinking of becoming a franchisee. What is one thing she can expect to receive from the franchiser? ((Training and ongoing support))
The role of marketing is limited to the physical selling of products. ((False))
Which of these is an advantage of sole proprietorships? ((Flexibility and control of the business ))
Franchises offer complete creative freedom to franchisees. ((False))
When evaluating the potential of a small business, investors consider undercapitalization. This refers to: (( The business having insufficient funds to operate normally ))
What is a key feature of monopolistic competition? ((Many businesses selling products that are similar but not identical. ))
What is a primary characteristic of a corporation? ((It is a legal entity separate from its owners))
Ethical conduct in business relationships helps to build: ((Trust))
What does social responsibility in business refer to? ((A business's obligation to pursue policies that benefit society))
Ethics and social responsibility are interchangeable terms with the same meaning. ((False))
Small businesses are immune to market fluctuations due to their size and operational flexibility. ((False))
What is the primary challenge in a monopolistic competition environment? ((Differentiation of products through branding and quality))
What is a characteristic of the taxation in sole proprietorships? ((Taxes paid only once as personal income))
Small businesses usually have multiple layers of management like large corporations. ((False))
What is a primary characteristic of a corporation? ((It is a legal entity separate from its owners))
During an economic expansion, a retail store experiences increased sales. This growth is primarily due to which factor? ((Increased consumer spending ))
Which form of business ownership typically allows for the fastest decision-making process? ((Partnership))
The acceptability of behavior of a business rests solely with the business itself. ((False))
What is essential for earning a profit in business? ((Management skills, marketing expertise, and financial resources))
A family business wants to ensure it stays within the family, with members sharing in the management. They want a structure that allows them to have shareholders but maintain control. What is their best option? ((Private Corporation))
Which of the following is not a type of economic system? ((Monetarism))
A business plan is unnecessary for small business success. ((False))
What is the result of effective business ethics programs? ((Enhanced business performance and trust))
What is the main focus of marketing activities? ((Maximizing profit))
What is a characteristic of the formation process of sole proprietorships? ((Easy and inexpensive ))
Whistleblowers damage a company's reputation without cause. ((False))
What does economics focus on as a field of study? ((How resources are distributed for the production of goods and services))
A conflict of interest in a business setting occurs when: ((Personal interests clash with professional duties))
An entrepreneur attends a business course to understand how to start and manage a successful company. This action highlights: ((The importance of understanding business fundamentals.))
Jana is assessing the risks of starting her own business. What is a risk she should consider? ((Encountering high stress levels))
What is a potential advantage of buying an existing business? ((A built-in network of customers and suppliers))
What is a franchiser responsible for? ((Providing training and support to the franchisee ))
Corporations face double taxation because profits are taxed both at the corporate level and as dividends to shareholders. ((True))
Entrepreneurs must always start with a large amount of capital to ensure the success of a new business. ((False))
If a supplier suggests a bribe for a contract renewal, what should be done ethically? ((Say no to the bribe and judge the supplier fairly.))
It's ethical for a company to prioritize its interests over societal well-being if it increases profitability. ((false))
What role does marketing play in a business? ((It involves creating, pricing, promoting, and distributing products))
Stakeholders of a business only include its shareholders and employees. ((false))
Why is fairness and honesty critical in business ethics? ((They build the foundation for ethical decision-making and trust))
Which is a characteristic unique to small businesses? ((The ability to make quick operational changes))
What is a disadvantage of partnerships related to decision-making? ((Responsible for each other's decisions))
A franchisee must pay ongoing fees to the franchiser. This practice: ((Includes initial and ongoing fees for support and brand use.))
Sole proprietorships are the easiest and least expensive type of business to form. ((True))
Ethical behavior in business is solely determined by an organization's internal code of conduct. ((False))
Why are management and employees in the same segment of the business functions circle? ((To separate their roles clearly))
Which type of entrepreneurship addresses social problems? ((Social entrepreneurship))
What does buying a franchise offer? ((A proven business model and support))
Ethical business practices do not contribute to a company's long-term success. ((False))
What is an oligopoly? ((A market dominated by a few large companies.))
Small businesses by their very nature are small and therefore do not contribute significantly to job creation. ((False))
In which type of business structure are the owners and managers not always the same? ((Sole Proprietorship))
Which of the following entities primarily benefits from a business earning profits? ((Society, by creating jobs and supporting social institutions.))
Why might an entrepreneur prefer starting a business from scratch over buying a franchise? ((More creative and operational freedom))
A business structure where professionals like lawyers and accountants share both management and liability is most likely a: ((General Partnership))
Ethical behavior in business can lead to a competitive advantage. ((True))
What does finance refer to in business? ((Activities concerned with obtaining and using money effectively))
Social responsibility is considered an unnecessary expense by most successful businesses. ((False))
Limited partners in a limited partnership: ((Have their liability limited to their investment in the business))
It's ethical for a company to prioritize its interests over societal well-being if it increases profitability. ((False))
What is an essential step in starting a small business? ((Creating a business plan))
What is a major disadvantage of sole proprietorships? ((Unlimited liability for business debts))
What is the difference between ethics and social responsibility? ((Ethics and social responsibility are interchangeable terms.))
How do codes of ethics support ethical behavior in organizations? ((By outlining acceptable behaviors and guiding decision-making))
A major hurdle when expanding a small business is: ((The ability to maintain quality during growth.))
What is the significance of "smallness" in business? (( It is relative and not strictly defined by numbers))
What does the concept of supply refer to in economics? ((The quantity of products businesses are willing to sell at different prices))
Business ethics apply only at the organizational level, not to individuals. ((False))
A local government hospital is expanding its free services. This is indicative of which type of organization's activities? ((A nonprofit organization))
What constitutes the economic system of a society? ((a. How it distributes resources to produce goods and services))
The board of directors in a corporation is elected by the shareholders. ((True))
In a limited partnership, what is the role of a general partner? ((To assume unlimited liability and manage the business.))
What is social entrepreneurship? ((Using entrepreneurship to address social problems))
What is an IPO? ((The process a corporation uses to go public))
A small business owner is concerned about high stress levels. This is most directly related to: ((The challenge of maintaining a work-life balance))
The principles and standards that determine acceptable conduct in business organizations are referred to as ((business ethics.))
In a sole proprietorship, who keeps all the profits? ((Owner))
What should a marketing team do when faced with pressure to exaggerate product capabilities in ads to outdo competitors? ((Advertise truthfully, highlighting the product's genuine strengths.))
What does social responsibility in business refer to? ((A business's obligation to pursue policies that benefit society))
In what way do mixed economies differ from pure economic systems? ((They blend elements from different economic systems to suit their needs.))
Small businesses cannot operate internationally.((False))
Which of the following is an example of pure competition? ((Bottled water))
In a limited partnership, what is the role of a general partner? ((To assume unlimited liability and manage the business.))
How do integrity-based programs improve ethical behavior? ((By making ethics a core part of organizational culture))
What should a marketing team do when faced with pressure to exaggerate product capabilities in ads to outdo competitors? ((Advertise truthfully, highlighting the product's genuine strengths.))
Why is undercapitalization a significant reason for small business failure? ((Insufficient funds to cover operational costs))
Social Responsibility is best described as: ((A business's obligation to pursue long-term goals that are good for society))
Sole proprietorships offer the advantage of easy access to large markets and extensive capital. ((False))
What is the primary goal of a business? ((To earn a profit by providing products that satisfy people's needs))
What role does a code of ethics play in a business? ((It outlines acceptable and unacceptable behavior within the organization))
What is a stakeholder in business? ((Groups interested in business outcomes))
Services can be physically touched and owned. ((False))
Two entrepreneurs form a business where one provides capital without participating in daily operations, highlighting which partnership aspect? ((Limited partnership))
Social entrepreneurship focuses solely on maximizing profits. ((False))
Technology is now less accessible to small businesses. ((False))
Sole proprietorships do not have a separate legal existence from their owners. ((False))
Trust is an unnecessary component in building professional business relationships. ((False))
A startup is deciding on its pricing strategy for a new product. Which of the following factors should it consider to ensure the product's success? ((Competitors' pricing strategies and consumer demand))
Which business form is characterized by having a perpetual existence? ((Corporation))
Susan is considering forms of business ownership. What factor is she evaluating? ((The legal structure of her business))
Services, unlike goods, are intangible and cannot be stored or held. ((True))
Competition in the market leads to increased prices for consumers. ((False))
The Small Business Administration (SBA) and similar entities provide financial assistance to large corporations. ((False))
An employee is offered a bribe to overlook a safety violation. What is the ethical action? ((Report the offer and the violation to the authorities.))
What is a primary benefit of whistleblowing programs? ((Encouraging reporting of misconduct))
What role does a code of ethics play in a business? ((It outlines acceptable and unacceptable behavior within the organization))
In a limited partnership, what is the role of a general partner? ((To assume unlimited liability and manage the business.))
Which of the following best describes the flexibility and control of a sole proprietorship? ((The owner has complete control over the business))
What is a financial aspect of franchises? ((Franchise fees and profit sharing with the franchiser))
1. Which of the following is a major reason for the increased interest in small business?(((There is a growing trend toward self-employment.)))
2. Which of the following statements is true about the impact of small businesses and entrepreneurship on educational institutions?(((Entrepreneurship and small business management are accepted as academically respected disciplines by many high schools, colleges, and universities.)))
3. Which of the following statements is true about small businesses?(((The manager usually owns the business.)))
4. Which of the following features must a business have to be classified as small?(((Management is independent, and the area of operation is primarily local.)))
5. A is any business that is independently owned and operated and is not dominant in its field of operation.(((small business)))
6. An entrepreneurial venture is characterized by:(((innovative strategic practices and/or products.)))
7. Which of the following is characteristic of a small business owner?(((Perceives the business as being an extension of his or her personality)))
8. Smaller firms tend to:(((keep larger firms competitive.)))
9. Which of the following best describes one of the contributions of small businesses?(((They are sources of new ideas or services that larger businesses may be unable to provide.)))
10. Which of the following statements is true about small businesses?(((Inadequate management in small businesses creates problems for small business owners and entrepreneurs.)))
11. Which of the following is a problem faced by small businesses today?(((Unexpected growth and financing issues)))
12. Which of the following is a reason for the drastic changes in small business management trends?(((Improvements in communications and computer technology)))
13. is the fundamental redesign of a business, often resulting in reduction in size and markets.(((Reinvention)))
14. refers to the process of reducing the number of employees to increase efficiency.(((Rightsizing)))
15. Which of the following is the result of the trend in business to become more active globally?(((A growing number of large and small U.S. businesses are or are becoming foreign owned.)))
16. Which of the following is a reason for starting a small business?(((To satisfy personal objectives)))
21. Which of the following is true about the first stage of the typical growth pattern that small businesses tend to follow?(((Owners manage the business and do all the work.)))
22. Which of the following is true about the final stage of the typical growth pattern that small businesses tend to follow?(((Owners hire managers to run the firms.)))
23. Which of the following statements is true about the typical growth pattern of a small business?(((Professional managers who work in small companies move from one company to another as they progress upward in rank and earnings.)))
24. is a voluntary decision to terminate a business.(((Discontinuance)))
26. John opened All-for-Shop, a small retail outlet, in his neighborhood. He could not sustain it for more than a year because of huge losses and chose to shut it down. Fortunately, he did not have any creditors. Which of the following does the situation describe?(((Personal failure)))
27. Which of the following statements is true about failure and small businesses?(((A discontinuance may result from the apparent advantages of working for someone else.)))
28. nurture young firms and help them to survive and grow during the startup period when they are most vulnerable.(((Business incubators)))
31. Small firms generate most new private employment.(((TRUE)))
32. Today more students think self-employment is a safer haven than working for big corporations.(((TRUE)))
33. The growth rate of the general workforce is always greater than the growth rate of self- employment.(((FALSE)))
34. A small business is one that is owned and operated by a group of entrepreneurs and is dominant in its field of operation.(((FALSE)))
35. An entrepreneurial venture may never grow large, and the owners may not want it to, as they prefer a more relaxed and less aggressive approach to running the business.(((FALSE)))
36. One of the unique contributions of small businesses is that they keep larger firms competitive.(((TRUE)))
37. Employees in large workplaces have higher job satisfaction than those in smaller firms.(((FALSE)))
".
38. A small business provides employees with a variety of learning experiences not open to individuals holding more specialized jobs in larger companies."(((TRUE)))
".
39. The reality that small businesses train people to become better leaders and managers and to develop their talents and energies more effectively has led more college graduates to seek full-time jobs with small businesses."(((TRUE)))
40. Managers of small firms must be specialists rather than generalists.(((FALSE)))
".
41. Nowadays, smaller firms are exempt from many federal regulations and even some state and local ones."(((FALSE)))
42. Few jobs in small firms are unaffected by improvements in communications and computer technology.(((TRUE)))
43. Downsizing or rightsizing is reducing the number of employees in order to achieve efficiency.(((TRUE)))
44. Technological advances in automation, computers, robotics, and electronic communication, along with changing markets resulting from cultural, demographic, and economic changes, have affected traditional "smokestack" industries.(((TRUE)))
45. Reengineering is the fundamental redesign of a business, often resulting in reduction in size and markets.(((FALSE)))
46. Those interested in small business management need to understand what the challenge of being active globally is and what the rewards may be.(((TRUE)))
47. One result of the current trends in business is the decreasing number of large and small U.S. businesses that are or become foreign owned.(((FALSE)))
48. Not all small business owners and managers make a lot of money, nor do they all intend to.(((TRUE)))
49. Growth depends on attaining both profit and social objectives, which are not necessarily incompatible.(((TRUE)))
50. Small business owners expect quick and concrete results from their investment of time and capital instead of engaging in the long-range planning.(((TRUE)))
51. With so much time, money, and energy devoted to the business, small business owners tend to have more zeal and devotion than do managers of big companies.(((TRUE)))
52. Most of the growing industries are dominated by large companies.(((FALSE)))
".
53. With advances in technology, many tasks in the health care industry are now performed by highly paid and skilled workers."(((FALSE)))
54. Most creative entrepreneurs are very good managers.(((FALSE)))
".
55. The length of service of professional managers (as opposed to owner-managers) in small businesses tends to be relatively long."(((FALSE)))
".
56. In formal failures, an owner who cannot succeed voluntarily terminates the business."(((FALSE)))
".
57. Personal failures are failures ending up in court with some kind of loss to creditors."(((FALSE)))
".
58. Personal failures are more common than formal failures."(((TRUE)))
".
59. While the usual incubation period of a business is two to three years, 30 percent of incubator clients typically graduate each year."(((TRUE)))
".
60. One of the causes of discontinuance or failure is that the amount of time and/or physical effort demanded of the small business manager was not recognized and/or planned for."(((TRUE)))
_____ is the process of finding a small—but profitable—demand for something and producing a custom-made product for that market.(((Niche marketing)))
John wants to start his own business. After doing some research, he has decided to specialize in cleaning kitchens since most homemakers are not satisfied with the quality of the service offered by the residential cleaning companies. This process of finding a small—but profitable—demand for something and producing a custom-made product or service is known as _____.(((niche marketing)))
3. Which of the following refers to a group of retired—but active—managers from all walks of life who help people develop their business ideas?(((Service Corps of Retired Executives)))
4. The website of the Federal Small Business Administration has a self-help program called the , which helps prospective entrepreneurs focus on some of the factors they need to consider before starting their own business.(((Small Business Startup Kit)))
5. Which of the following is a good source of information through which prospective business owners get help and advice?(((State Economic Development Agencies)))
_____ is the system of gathering, recording, classifying, analyzing, and interpreting data related to merchandising of goods and services.(((Market research)))
8. Prospective entrepreneurs can access data from to obtain information on subjects such as population, business, and housing.(((Census Bureau)))
9. Which of the following statements is true of market research in the context of small businesses?(((Formal research programs are a valuable but an expensive way to obtain guidance.)))
10. While working in a fast food outlet, Harold observed that most of the customers were working in offices in the neighborhood. Sensing a profitable opportunity, he decided to open a catering business. He offered to deliver lunch directly to the offices so that the businesspeople would not have to interrupt their meetings to go out for lunch. In the context of the above scenario, which of the following was the reason for Harold to start his own business?(((To select a competitive environment)))
11. Which of the following is a reason for starting a new business?(((The owner has the freedom to select his or her own competitive environment.)))
12. Catherine, who loves dogs, wants to offer dog-walking services in her neighborhood. Her research leads her to a small professional pet care store operating in the area, which she decides to buy. Which of the following is a good reason for Catherine to buy the pet care store?(((The facilities are already available.)))
14. Which of the following is a benefit of buying a franchise?(((The franchiser brings proven methods of operation to aid the franchisee.)))
15. Which of the following is a drawback of buying a franchise?(((Franchisees face issues in understanding their role in the responsibilities of operating the franchise.)))
16. In franchising, which of the following is a benefit to the franchisee?(((Share in local or national promotion)))
17. In a franchise, which of the following is an advantage for a franchiser?(((Low operating costs)))
_____ is a marketing system based on a legal arrangement that permits one party to conduct business as an individual owner while abiding by the terms and conditions set by the second party.(((Franchising)))
19. Which of the following statements is true of franchising?(((A franchise is an agreement whereby an independent businessperson is given exclusive rights to sell a specified good or service.)))
22. Product and trademark franchising:(((grants the franchisee the right to sell a widely recognized brand.)))
23. Which type of franchising system grants the franchisee the right to market the product and trademark and use a complete operating system?(((Business format franchising)))
24. Which of the following is true of business format franchising?(((The franchiser provides assistance to the franchisee to train franchise personnel.)))
25. What is due diligence?(((The research and analysis of the company that is done before a business transaction)))
26. Which of the following is true of a franchise fee?(((It is a one-time fee paid to the franchiser.)))
28. Which document provides information to possible new franchisees on 20 items required by the Federal Trade Commission?(((Disclosure statement)))
29. is the concept that two or more people, working together in a coordinated way, can accomplish more than the sum of their independent efforts.(((Synergy)))
30. Which of the following is true of combination franchising?(((Cultural clashes can lead to the breakup of this arrangement.)))
31. is the most important professional assistance one needs before investing in a franchise.(((Legal advice)))
33. When a person wants to start a new business, the first step is to decide whether to start a new business, buy an existing one, or buy a franchise.(((FALSE)))
35. Niche marketing is the systematic gathering, recording, and analyzing of data related to the marketing of goods and services.(((FALSE)))
36. Reintroducing an old product is a good business idea since customer tastes run in cycles.(((TRUE)))
37. The Small Business Administration is a group of retired managers who help people develop their business ideas.(((FALSE)))
39. Small businesses never select one product niche because they usually have sufficient resources to cover the whole market.(((FALSE)))
40. The Service Corps of Retired Executives publishes many useful books on planning and organizing small businesses.(((FALSE)))
41. In a given market area, the fact that a high number of similar firms have gone out of business usually signals market weakness.(((TRUE)))
42. Small businesses must strive for low overhead, use no-frills assets, and look for better real estate deals.(((TRUE)))
43. Lack of an established product line does not typically stop a person from starting a new business.(((FALSE)))
44. One should buy an existing business even if the physical facilities of the firm are old or obsolete.(((FALSE)))
45. Even if an established or an existing business is in a desirable location, it is not a good idea to buy it.(((FALSE)))
46. An advantage of buying a franchise is that it already has many of the requirements for success.(((TRUE)))
47. In the context of small businesses, franchises tend to fail because they lack sufficient funds to cover all needs.(((TRUE)))
48. In the context of small businesses, franchises tend to fail because of unsuccessful marketing.(((TRUE)))
51. Product and trademark franchising is usually observed in the restaurant industry.(((FALSE)))
53. The industry group with the largest volume of sales in a business format franchising is automobile and truck dealerships.(((FALSE)))
54. A disadvantage of franchising is the voluminous paperwork needed to provide disclosure documents to potential franchisees.(((TRUE)))
55. Due diligence is the concept that two or more people, working together in a coordinated way, can accomplish more than the sum of their independent efforts.(((FALSE)))
56. The disclosure agreement is a document that provides information on 20 items required by the Federal Trade Commission.(((TRUE)))
57. In the context of franchising, a disclosure statement provides demographic information to the Federal Trade Commission.(((FALSE)))
58. A problem area faced by franchisees is the high price of supplies that must be bought from the franchiser.(((TRUE)))
59. In the context of franchising, the success of fast food restaurants is related to the increasing number of women working outside the home.(((TRUE)))
60. Combination franchising is the ideal arrangement for big-name franchise operations to avoid cultural clashes.(((FALSE)))
61. Some franchisers prefer franchisees who own a large number of franchise outlets because it speeds their growth.(((TRUE)))
62. Small businesses in the United States are moving away from synergy as they are trying to reengineer themselves.(((FALSE)))
Which of the following does a business incubator do to help small business owners succeed? ((Provide management assistance and shared office services))
_____ should be the first management function performed by an effective business owner-manager. (((Planning)))
The process of setting objectives and devising courses of action to achieve those objectives is termed _____. (((planning)))
_____ is the process of setting standards, measuring performance against standards, and taking corrective action to see that planned performance is achieved. (((Controlling)))
_____ provides comprehensive long-term direction to help a business accomplish its mission. (((Strategic planning)))
Strategic planning helps a business set its _____. (((missions)))
Which of the following is an example of operational planning in a company? (((Setting the personnel policy)))
A SWOT analysis is used to: (((identify new markets.)))
The external environment of a SWOT analysis consists of: (((opportunities and threats.)))
_____ give a short-term direction to a business and serve as benchmarks for measuring performance. (((Objectives)))
Many consultants and advisers push small companies to give more emphasis to their: (((external environment.)))
_____ are general statements that serve as a guide to managerial decision making and supervisory activities. (((Policies)))
_____ provide employees with standing instructions for performing their jobs, comprise detailed explanations of how to do the work properly, and inform in what order it should be done. (((Methods and procedures)))
_____ set the requirements needed to follow strategies and accomplish objectives(((Budgets)))
The largest expense for companies selling products usually is related to: (((materials, supplies, and/or goods.)))
Financial planning involves: (((estimating income and expenses.)))
Raw materials used to produce products are: (((variable expenses.)))
Expenses that do not alter in value as sales volume rise or fall are called _____. (((fixed expenses)))
_____ is the amount of money available at a given time to pay expenses. (((Cash flow)))
In order to maintain control, the owner of a business should: (((invest more personal funds than all other sources combined.)))
Through equity financing, a company can be partly owned by: (((the investors.)))
Matt plans to open a convenience store. The startup cost is approximately $10,000. Since Matt has only $2,500 with him, he decides to invite two of his friends—Brett and Brian—to join him in this venture. In this scenario, who should be in charge of writing the business plan? (((Matt, Brett, and Brian, since it is always a team effort)))
A business plan can be successfully prepared by: (((developing short-term detailed plans for those involved with the business.)))
The _____ is a part of a business plan that outlines the entire plan, its major objectives, and the expected results. (((executive summary)))
In which part of a business plan should the expected revenues be included? (((Financial data)))
An effective business plan should be about _____. (((25 to 30 pages long or less)))
The oral presentation of any business plan should be(((validated by the market research findings.)))
Organizing should be the first management function performed while setting up a business, as it sets the future course of action for all aspects of the business. (((FALSE)))
Controlling is the process of setting standards and measuring performance against those standards. (((TRUE)))
Making a plan is not always the safest step as planning for the future is very risky. (((FALSE)))
The process of planning is very convenient and does not consume much time. (((FALSE)))
Selecting the type of business to enter is an example of operational planning. (((FALSE)))
SWOT analysis is a useful tool to aid the process of strategic planning and can be organized into the form of a matrix. (((TRUE)))
In a SWOT analysis, strengths and weaknesses of a company are part of its external environment. (((FALSE)))
Legal objectives are included in the operational planning process during the setting up of a business plan. (((FALSE)))
The mission statement of a business corporation should be revised frequently. (((FALSE)))
Setting up the human resource management (HRM) objectives of a company is an activity that falls under strategic planning. (((TRUE)))
Objectives are the goals that give shorter-term direction to a business and serve as benchmarks for measuring performance. (((TRUE)))
External environmental elements do not affect the operation of a small business. (((FALSE)))
Internal environmental factors of a business include clients, technology, and competition. (((FALSE)))
A competitive edge is a particular characteristic that makes a firm more attractive to customers than are its rivals. (((TRUE)))
Strategies are general statements that serve as short-term guides to managerial decision making. (((FALSE)))
In the context of a business setting, operational planning involves developing strategies to meet competition. (((FALSE)))
Methods and procedures provide employees with standing instructions for performing their jobs. (((TRUE)))
The more amount of time a small business owner devotes to the business, the more important it is to have capable employees. (((FALSE)))
Simple startup businesses do not need records and information systems(((FALSE)))
Income from sales is termed revenue. (((TRUE)))
Net profit is defined as the total amount of revenue generated. (((FALSE)))
Variable expenses remain stagnant in relation to volume of output. (((FALSE)))
Expenses such as depreciation on buildings are known as variable expenses(((FALSE)))
A business plan serves as a tool for attracting people and money(((TRUE)))
f a business is to be organized and run by more than one person, preparing the business plan should be an individual effort. (((FALSE)))
The executive summary of a business plan outlines the entire plan, its major objectives, how these objectives will be accomplished, and the expected results. (((TRUE)))
One important purpose of a business plan is to indicate the expected financial results from operations. (((TRUE)))
____ are those capital requirements that are of a relatively permanent nature and are necessary for the functioning of a business. (((Fixed assets)))
_____ of a company is its current assets, less current liabilities, that a firm uses to produce goods and services and to finance the extension of credit to customers. (((Working capital)))
Working capital needs of a company are projected by estimating what out-of-pocket expenses will be incurred and when the revenues from sales are to be collected, which is done by _____(((cash budgets)))
Equity financing in a corporation is evidenced by shares of either common or preferred _____. (((stock)))
In the context of small businesses, _____ comes from lenders who will be repaid at a specified interest rate within a specified time span. (((debt financing)))
In the context of small businesses, which of the following statements is true of preferred stock? (((It usually conveys no voting rights to its holder.)))
Which of the following statements is true of small company offering registration (SCOR)? (((It is the sale of common stock to the public through a regulated board.)))
Which of the following statements is true of debt securities? (((Debt securities are usually in the form of bonds or loans.)))
_____ is a form of debt security with a standard denomination, method of interest payment, and method of principal repayment. (((Bond)))
A(n) _____ is a long-term debt that is secured by real property. (((mortgage loan)))
A debt backed by some physical asset other than land, such as machinery, equipment, or inventory, is termed _____. (((chattel mortgage loan)))
In the context of small businesses, which of the following statements is true of asset-based financing? (((It accepts the assets of a firm as collateral in exchange for a loan.)))
In the context of small businesses, which of the following statements is true of debt securities? (((Small companies rely on loans from commercial banks.)))
Small business investment companies are:Â (((private firms licensed and regulated by the Small Business Administration to make "venture" investments in small firms.)))
Which of the following statements is true of venture capital firms? (((They prefer to back small firms with opportunities for fast growth.)))
_____ are wealthy individuals who invest part of their assets in high-risk, high-return ventures. (((Angel capitalists)))
In the context of small businesses, which of the following statements is true of business incubators? (((They nurture young firms and help them grow.)))
_____ is a source of financing that reaps tax advantages from selling shares to workers. (((Employee stock ownership plan)))
_____ consists of two or more companies exchanging items of roughly equal value. (((Barter)))
_____ refers to purchases of inventory, equipment, and/or supplies on an open account in accordance with customary terms for retail. (((Trade credit)))
In the context of small businesses, which of the following statements is true of consignment selling? (((It allows payments to be made to suppliers only when products are sold.)))
In the context of small businesses, which of the following statements is true of a line of credit? (((It permits a business to borrow up to a set amount without red tape.)))
What is the purpose of specialized small business investment companies? (((They assist socially and economically disadvantaged businesses with venture capital.)))
Which of the following statements is true of the Small Business Administration? (((It helps small firms find financing at reasonable rates.)))
Which of the following statements is true of small business investment companies (SBICs)? (((The Small Business Administration matches each dollar an SBIC puts into a loan.)))
John, who needs funding for his small business, approaches a few lenders for funds. Which of the following will be true in this situation? (((Lender(s) will typically look at the track record of his business because the money being borrowed is for an existing business.)))
If a business has an operating history, its future needs can be estimated with relative accuracy, even with substantial growth. (((TRUE)))
Growing companies must be financed entirely with debt financing. (((FALSE)))
When sales are made on credit, a firm must carry the costs of production itself for an extended period. (((TRUE)))
Working capital estimates what the out-of-pocket expenses will be for a company during the following year to produce a product(s) for sale and when revenues from these sales are to be collected. (((FALSE)))
Debt financing comes from lenders who will be repaid at a specified interest rate within an agreed-on time span. (((TRUE)))
The role of equity financing is to serve as a buffer that protects debtors from loss in case of financial difficulty.(((FALSE)))
The claims of common and preferred stockholders on the assets of a firm take preference over those of the creditors in the event of default on a contractual obligation. (((FALSE)))
Small firms use debt financing because interest payments are tax-deductible expenses. (((TRUE)))
Employing a varying charge to increase the return to common stockholders is known as financial leverage. (((FALSE)))
Common stockholders have a legally enforceable claim to dividends. (((FALSE)))
Common stock usually conveys no voting rights to its holder. (((FALSE)))
No dividends can be paid to common stockholders until preferred stock dividends have been paid. (((TRUE)))
Small company offering registration (SCOR) is the sale of common stock to the public through a regulated board such as Nasdaq. (((TRUE)))
Short-term securities have a maturity period of one to five years. (((FALSE)))
Long-term debt backed by some physical asset other than land is a mortgage loan. (((FALSE)))
Most asset-based loans are financed against inventory and less often against accounts receivable. (((FALSE)))
Many "unsecured" loans that banks extend to small businesses require personal guarantees by the managers of the firms. (((TRUE)))
People who start a small business usually seek outside funding before using their own funds. (((FALSE)))
Small business investment companies (SBICs) tend to make very small investments as they are not profit-making institutions. (((FALSE)))
Venture capital firms generally require a substantial return as either equity or profit. (((TRUE)))
Venture capitalists insist on looking at business plans rather than executive summaries to help them make investment decisions. (((FALSE)))
The percentage of business plans accepted by venture capitalists for investment purposes is very high. (((FALSE)))
Professional venture capital firms provide more investment capital for small businesses than do angel capitalists. (((FALSE)))
The purpose of business incubators is to shelter failing enterprises by providing financial services. (((FALSE)))
The growth rate of the general workforce is always greater than the growth rate of self-employment. ((false))
17. A new small business owner's primary motive is usually .(((independence, that is, freedom from interference or control by superiors)))
18. Which of the following refers to an introspective personal analysis that can help a small business owner determine whether the combination of qualities he or she has will help him or her succeed as a small business owner?(((Analysis of one's attitude)))
19. According to the Small Business Administration's (SBA) Office of Advocacy (www.sba.gov), only tend to be dominated by larger businesses.(((construction and personnel/supply services)))
25. Mark opened a café in his neighborhood. Soon his café became very popular, and his revenues started increasing. Steve, the owner of a popular restaurant, saw Mark's success and asked him to join his restaurant. The financial incentive that Steve offered Mark was more than what he earned at his café. Moreover, Mark would not have to face the daily hassles of managing everything on his own. Mark decided to shut down his café and join the restaurant. This is an example of:(((discontinuance.)))
6. are private associations of local (or regional) business and professional people who are familiar with an area's needs.(((Chambers of commerce)))
Jane, who wants to start her own business, considers buying an Internet café which is up for sale in her neighborhood. However, Jane discovers that Internet connectivity tends to be erratic in the store since she lives in a remote town and decides against it. Which of the following is most likely the reason for Jane's refusal to buy the business?(((Problems due to old and obsolete facilities)))
20. What does a franchiser do?(((Owns the company's name and licenses others to sell its products)))
21. Burger's Inc. owns a number of fast food stores across the country. Ryan, who wants to start his own business, decides to open a store under the Burger's Inc. trademark. He has to sell and market the fast food products according to the terms of the licensing agreement. In this scenario, Ryan is a:(((franchisee.)))
27. The continuous fee paid by the franchisee to the franchiser based on a percentage of the franchisee's gross revenue is termed:(((royalty fee.)))
32. The in the United States has a code of ethics that covers a franchiser's obligations to its franchisees.(((International Franchise Association)))
34. Planning to start one's own business begins with searching for or identifying a good or service to sell.(((TRUE)))
38. The Small Business Administration in the United States is a particularly effective source of local information for prospective business owners because it is familiar with an area's needs.(((FALSE)))
49. One way to help avoid franchise scams is to carefully review the franchiser's disclosure document.(((TRUE)))
50. A franchisee owns a franchise's name and distinctive elements and licenses others to sell its products.(((FALSE)))
52. A franchiser exercises complete control over a franchisee's business operations.(((FALSE)))
A business's _____ defines the present business scope and broadly describes an organization's present capabilities, focus, and activities. (((mission statement)))
A software engineer is part of a company's _____ resources. (((human)))
Which of the following is part of a company's financial resources? (((Available equity)))
Which of the following statements is true of a company's business plan? (((It provides a detailed design of activities to be carried to finance the business and market products.)))
Strategic planning consists of two parts: a firm's mission and objectives and its strategies. (((TRUE)))
In developing a business plan, one should consider the firm's origin and background to attract investors. (((TRUE)))
A business plan should be delivered from the entrepreneur's perspective and not the investor's perspective. (((FALSE)))
_____ is an owner's share of the assets of a company. (((Equity)))
In the context of small businesses, what is financial leverage?(((Using fixed-charge financing, usually debt, to fund a business's operation)))
A _____ is a contract that permits the use of someone else's property for a specified time period. (((lease)))
In the context of small businesses, which of the following statements is true of common stock? (((It represents an owner's interest and consists of many identical shares.)))
Small businesses' assets such as buildings should be financed with short-term loans. (((FALSE)))
Working capital is an owner's share of the assets of a company. (((FALSE)))
A company's stocks represent an owner's share of the assets of the company. (((FALSE)))
A lenders' income in debt financing varies with the success or failure of a business. (((FALSE)))
From an investor's point of view, debt investments should have a higher financial return than common stock investments. (((FALSE)))
A lease is a contract that permits use of someone else's property for a specified period of time. (((TRUE)))
17. A new small business owner’s primary motive is usually .(((independence, that is, freedom from interference or control by superiors)))
18. Which of the following refers to an introspective personal analysis that can help a small business owner determine whether the combination of qualities he or she has will help him or her succeed as a small business owner?(((Analysis of one’s attitude)))
19. According to the Small Business Administration’s (SBA) Office of Advocacy (www.sba.gov), only tend to be dominated by larger businesses.(((construction and personnel/supply services)))
25. Mark opened a café in his neighborhood. Soon his café became very popular, and his revenues started increasing. Steve, the owner of a popular restaurant, saw Mark’s success and asked him to join his restaurant. The financial incentive that Steve offered Mark was more than what he earned at his café. Moreover, Mark would not have to face the daily hassles of managing everything on his own. Mark decided to shut down his café and join the restaurant. This is an example of:(((discontinuance.)))
6. are private associations of local (or regional) business and professional people who are familiar with an area’s needs.(((Chambers of commerce)))
13. Jane, who wants to start her own business, considers buying an Internet café which is up for sale in her neighborhood. However, Jane discovers that Internet connectivity tends to be erratic in the store since she lives in a remote town and decides against it. Which of the following is most likely the reason for Jane’s refusal to buy the business?(((Problems due to old and obsolete facilities)))
20. What does a franchiser do?(((Owns the company’s name and licenses others to sell its products)))
21. Burger’s Inc. owns a number of fast food stores across the country. Ryan, who wants to start his own business, decides to open a store under the Burger’s Inc. trademark. He has to sell and market the fast food products according to the terms of the licensing agreement. In this scenario, Ryan is a:(((franchisee.)))
27. The continuous fee paid by the franchisee to the franchiser based on a percentage of the franchisee’s gross revenue is termed:(((royalty fee.)))
32. The in the United States has a code of ethics that covers a franchiser’s obligations to its franchisees.(((International Franchise Association)))
34. Planning to start one’s own business begins with searching for or identifying a good or service to sell.(((TRUE)))
38. The Small Business Administration in the United States is a particularly effective source of local information for prospective business owners because it is familiar with an area’s needs.(((FALSE)))
49. One way to help avoid franchise scams is to carefully review the franchiser’s disclosure document.(((TRUE)))
50. A franchisee owns a franchise’s name and distinctive elements and licenses others to sell its products.(((FALSE)))
52. A franchiser exercises complete control over a franchisee’s business operations.(((FALSE)))
A business’s _____ defines the present business scope and broadly describes an organization’s present capabilities, focus, and activities. (((mission statement)))
A software engineer is part of a company’s _____ resources. (((human)))
Which of the following is part of a company’s financial resources? (((Available equity)))
Which of the following statements is true of a company’s business plan? (((It provides a detailed design of activities to be carried to finance the business and market products.)))
Strategic planning consists of two parts: a firm’s mission and objectives and its strategies. (((TRUE)))
In developing a business plan, one should consider the firm’s origin and background to attract investors. (((TRUE)))
A business plan should be delivered from the entrepreneur’s perspective and not the investor’s perspective. (((FALSE)))
_____ is an owner’s share of the assets of a company. (((Equity)))
In the context of small businesses, what is financial leverage?(((Using fixed-charge financing, usually debt, to fund a business’s operation)))
A _____ is a contract that permits the use of someone else’s property for a specified time period. (((lease)))
In the context of small businesses, which of the following statements is true of common stock? (((It represents an owner’s interest and consists of many identical shares.)))
Small businesses’ assets such as buildings should be financed with short-term loans. (((FALSE)))
Working capital is an owner’s share of the assets of a company. (((FALSE)))
A company’s stocks represent an owner’s share of the assets of the company. (((FALSE)))
A lenders’ income in debt financing varies with the success or failure of a business. (((FALSE)))
From an investor’s point of view, debt investments should have a higher financial return than common stock investments. (((FALSE)))
A lease is a contract that permits use of someone else’s property for a specified period of time. (((TRUE)))
What is true of wholesalers? (((Wholesalers are extremely important because of the marketing activities they perform.)))
Many service providers are considered retailers because they (((provide their services directly to consumers)))
A merger occurs when (((two companies combine to form a new company)))
A(n) ________ is a partnership established for a specific project or for a limited time. (((joint venture)))
An entrepreneur has been primarily associated with the willingness to((take risks.))
What is one of the difficulties faced by small business owners?((worries about employee problems))
What is a difference between high technology businesses and other small businesses?((High technology businesses require greater capital and have higher initial startup costs thanother small businesses))
What is true of a capitalist economic system?((Prices of goods and services are determined by supply and demand))
The quantity of goods and services that consumers are willing to buy at different prices at a specific time is referred to as((demand.))
The quantity of products that businesses are willing to sell at different prices at a specific time is referred to as((supply.))
What is a defining characteristic of public corporations?((Their stock can be bought, sold, or traded by anyone))
Which business provides a service, but is neither owned by the government nor focuses on earning profits?((a nonprofit corporation))
What is a true statement about the board members of a corporation?((They have a duty of care and loyalty to oversee the management of the firm.))
Preferred stockholders of a corporation((have a claim to profits before other stockholders do.))
Mark opened a café in his neighborhood. Soon his café became very popular, and his revenues started increasing. Steve, the owner of a popular restaurant, saw Mark's success and asked him to join his restaurant. The financial incentive that Steve offered Mark was more than what he earned at his café. Moreover, Mark would not have to face the daily hassles of managing everything on his own. Mark decided to shut down his café and join the restaurant. This is an example of: ((discontinuance.))
A _____ is any business that is independently owned and operated and is not dominant in its field of operation. ((small business))
_____ nurture young firms and help them to survive and grow during the startup period when they are most vulnerable. ((Business incubators))
Test marketing a product is a cost-effective method to carry out primary market research. (( False ))
The four variables of a marketing mix are—customer, company, competition, and constraints. (( False ))
A uniform marketing strategy is recommended for people of all age groups as it proves to be very beneficial. (( False ))
Consumerism involves performing product tests. (( True ))
The marketing concept emphasizes that the survival of any business depends on providing service to customers. (( True ))
The marketing concept states that a firm must put consumer needs first, even if it means foregoing satisfactory profits. (( False ))
Watching people is a useful marketing research technique. (( True ))
A product's benefits are identified and emphasized to develop a competitive niche in the introduction stage of a product's life cycle. (( False ))
Test marketing is the systematic gathering, recording, and analyzing of data relating to the marketing of goods and services. (( False ))
Small firms generally should not attempt to be price leaders. (( True ))
Employees are not a good source of information about customer preferences as they are usually just trying to sell a product. (( False ))
When customers walk out of the store without buying anything, it may be an indication that a business is in trouble. (( True ))
Marketing research cannot be done by using customer credit records and sales records since they are not helpful in yielding any information. (( False ))
When people buy something, there is no sense of satisfaction in the purchase made or in the goods and services provided to them. (( False ))
A target market is a part of the total market toward which promotional efforts are concentrated. (( True ))
Marketing objectives need not be tied in with a business's competitive edge. (( False ))
Small businesses should carry out primary market research on their own instead of hiring experts to do it. (( False))
The marketing concept is opposed to the systems approach in managing a business. (( False ))
Market research can be useful before starting a small firm, but it is not required once the firm is in operation. (( False ))
Market segmentation simulates the conditions under which a product is to be marketed. (( False ))
A marketing mix consists of uncontrollable variables that a firm combines to satisfy its target market. ((False))
The way a product is packaged can influence customers in making buying decisions. (( True ))
Competition becomes more aggressive during the decline stage of a product's life cycle as prices and profits are on a decline. (( False))
A fact is a piece of evidence that can be verified.(((True.)))
A fact is just a subjective and unsupported point of view.(((False.)))
A familiar problem-solution pattern consists of: Situation, Problem,…………. , Solution, and Evolution. (((Causes)))
A problem often creates a ………….. towards a situation. (((Negative attitude)))
A problem often creates a positive attitude towards a situation. Select one:(((False.)))
A solution-justification text is an answer to three questions:(((What is the problem? What is your solution? & why do you think so?)))
A truth claim indicate that you believe something is or is not true. The language used in the sentence above represents:(((weak claim)))
An argument can be made strongly, identify boosting words can build an argument:(((Definitely, indeed.)))
An argument can be weak, identify hedging words can build an argument:(((Apparently, Seemingly)))
An argument is a claim that is --------- and ------------- in a variety of ways, such as the convincing explanations, evidence and examples.((( justified and supported)))
An argument means a discussion between people who hold the same views.(((False.)))
An example of hedging (weaker or tentative) claims:(((seems.)))
An opinion is just subjective and unsupported point of view(((True.)))
Answering people in mannerly way is an example of good:(((Listening and team working skills)))
Any written claim was supported by evidence we call it……(((Verified fact)))
As a result of the economic downturn, people have less money to spend. The cheaper supermarkets are wherefores attracting more customers is example of:(((Causal Chain)))
Before you read carefully, look quickly through the text to see what it is generally about, Good places to look at are:(((The title, introduction, fist sentences of each paragraph and the final paragraph.)))
Boosting language is used to show your:(((Confidence in your claims and results.)))
Brompton bicycle is the biggest bicycle manufacturer in Britain. Select one:(((Strengths)))
Brompton Bicycle is well-known for the high quality of its niche product, a folding bicycle popular with commuters Select one:(((Strengths)))
Business can study texts describe:(((General situations of a company)))
Business case study can have different format such as:(((Video)))
Business case study texts describe:(((General situations of a company)))
Cause and effect describes a relationship between actions or events Select one:(((TRUE)))
Checking the meaning of difficult words and expressions is a useful strategy for:(((Reading skills)))
Choice of reporting verbs that describe neutral relation to your argument:(((All the answers (identified, stated, mentioned))))
Choose the right/correct answer from the below examples on cause & effect:(((Cause leads to problem)))
Choose the right/correct answer from the below examples on cause & effect:(((The cause affect the effect)))
Claim and support are the ability to recognize these two components in an argument helps you to understand other people’s plan and writing your own argument effectively:(((True.)))
Comparison is to compare and contrast plan, or draw introduction.(((False. (Draw conclusion))))
Components of an argument text include:(((Argument contains both a claim and a supporting explanation or evidence.)))
Covid-19 is just a flu. This sentence represents:(((A fact.)))
Crises, satisfaction and productivity are examples of:(((Abstract nouns)))
Explaining something clearly in workplace context is considered one of:(((Speaking skills)))
Finding solutions to a problem is common only in the work place: Select one:(((False.)))
Fishbone / lshikawa diagrams are:(((Cause & Effect Diagram)))
From the two sentences written below, sentence number 1 is: S (1) The internet is a good invention. S (2) It is a source of endless information.(((Claim)))
Good presentation involves:(((Using bold and size to differentiate sections)))
Good reading skills involve:(((Analysing business cases)))
Good reading strategies include:(((Making notes)))
Good writing skills include:(((Plan and structure ideas)))
Happiness is an examples of:(((Abstract nouns)))
Hedging is used to show certainty, confidence, modesty, and respect for others’ views Select one:(((TRUE)))
Hedging language in argument is used to:(((show uncertainty, modesty and respect for others’ views.)))
How successful is the suggestion? Does it solve the case? Select one:(((Evaluation)))
Identifying the problem & Sum it up in one or two sentences mean …. Select one:(((Problem)))
In Fat Face case study showed that, there was an increased business/brand awareness among consumers:Select one:(((Opportunities)))
In Fat face case study, competition from more sought after brands is Select one:(((Threats)))
In Fat Face case study, the increased marketing opportunities Select one:(((Opportunities)))
In Fat Face case study, the transactional website increases sales is: Select one:(((Strengths)))
In Fat Face case study, there is little sales training for staff is:(((Weaknesses)))
In fat face study, the increased interest in online presences is Select one:(((Opportunities)))
In order to communicate effectively you should be:(((Ready)))
In order to communicate effectively you should be:(((Prepare)))
In order to complete the analyzing process, what step must be completed first ?(((Identify What is the starting situation.)))
In SWOT analysis, weaknesses and threats are harmful to achieve the objective of the organization.(((TRUE)))
In SWOT, we need to apply a framework to identify Select one:(((The organization internal and external problems.)))
In the ……….. you explain how you are doing your analysis(((Introduction)))
In the language of argument, a term that shows confidence in your claims and results:(((Boosting)))
In your Café, if you are offering a free Wi-Fi to your computers. This will be considered as:(((Strengths)))
In your own cafe, if you are NOT providing proper customer services this will be considered as: ((Weaknesses))
In your own café, if you are NOT providing an adequate employee training, such as showing your staff how they should attend to tables or explaining to them how the food has to be prepared and presented This will be considered as:(((Weaknesses)))
In your own café, if you are not providing an adequate employee training, such as showing your staff how they should attend to tables or explaining to them how the food has to be prepared and presented. This will be considered as:(((Weaknesses)))
In your own café, if you are not providing an adequate employee training, such as showing your staff how they should attend to tables or explaining to them how the food has to be prepared and presented. This will be considered as: Select One:(((Weaknesses)))
In your own café, if you are offering a free Wi-Fi to your customers. This will be considered as.(((Strengths)))
In your own café, if you are offering a free Wi-Fi to your customers.. This will be considered as : Select one:(((Strength)))
In your own café, if you are offering more organic snacks or salads on the menu in order to take advantage of trends which linked to eat healthier. This will be considered as:(((Strengths)))
In your own café, if you are presenting more organic snacks or salad son the menu in your own café in order to take advantage of trends which linked to eating healthier. This will be considered as:(((Strengths)))
In your own café, if you are presenting more organic snacks or salads on the menu in your own café in order to take advantage of trends which linked to eating healthier.. This will be considered as:(((Strengths)))
In your own café, if you are providing an excellent customer services. This will be considered as:(((Strengths)))
In your own café, if you are providing excellent customer services. This will be considered as:(((Strengths)))
In your own café, if you are serving shrimp salad and something negative impacts the seafood market.. This will be considered as:(((Threats)))
In your own case, if you are serving shrimp salad and something negative impacts the seafood market. This will considered as: Select One:(((Threats)))
In-text referencing should be under the heading ‘references or ‘Bibliography’(((FALSE)))
is just a subjective and unsupported point of view.(((Opinion)))
It is better that the company change the product’s label: that example of …...(((Proposing solution)))
It is worth pointing out that paraphrasing is not the same as summarizing. When summarizing a text, you reduce the length by covering the key points in a concise form. Paraphrasing maintains the same meaning and the length may stay the same.(((False.)))
Justification is:(((What is right or reasonable by using evidences)))
Justification is:(((It is the action of showing something to be right or reasonable by using evidences.)))
Knowing the staring case & what is the text about. Are questions that lead to :(((Solution)))
linking word shows the sequence of events or ideas(((Time)))
Lowell, a burger company, is expanding rapidly. However, the orders are often cold by the time they are delivered and customers complain about this. There is a shortage of motorcycle delivery riders. So, Lowell will recruit more motorcycle delivery riders. The starting Situation in the above case study is? Select one:(((Lowell company, is expanding rapidly)))
Lowell, a burger company, is expanding rapidly. However, the orders are often cold bythe time they are delivered and customers complain about this. There is a shortage of motorcycle delivery riders. So, Lowell will recruit more motorcycle delivery riders. The starting Situation in the above case study is? Select one:(((Lowell company, is expanding rapidly.)))
Methods of making notes include:(((Sketches or diagrams)))
One of the words is “Problem – signaling Word” Select one:(((Positive impact)))
One of these words is “Problem – signaling Word” Select one:(((Undervalued.)))
Opportunities and threats Select one:(((External factors influencing the organization.)))
Opportunities are potential openings like Select one:(((All answers are applicable)))
Opportunities mean Select one:(((What an organization can do to make its position stronger in order to gain more customers)))
Organization’s report should include the following sections in order:(((Method, Finding and Conclusion)))
Paragraphs usually start with ---------- then move on to ----------(((General information, Specific details)))
Paraphrasing involves stating ideas from the original source in your own words:(((True.)))
Problem is a negative situation strengthens an individual, group or organization.(((FALSE)))
Problem solving is the key to a business’s success.(((True.)))
Problems are a matter of perception means :(((One person may perceive a situation as a problem while someone else may be quite satisfied with the same situation.)))
Read the text below, which has sentences numbered1,2 etc. Type the numbers of the sentences in the box below to show which is support in the text: Covid-19 is just a flu. (2) Where most common symptoms of COVID-19 are fever, dry cough, and tiredness. (3) In addition, most people (about 80%) recover from the disease without needing hospital treatment.(((S (2) and S (3))))
Read the text below, which has sentences numbered1,2 etc.Type the numbers of the sentences in the box below to show which sentences have linking words in the text: (s1) Covid-19 is just a flu. (2) Where most common symptoms of COVID- 19 are fever, dry cough, and tiredness. (3) In addition, most people (about 80%) recover from the disease without needing hospital treatment.(((S2 and S3)))
Recognizing signposting words and groups of words is a helpful way in.(((Reading skills)))
Recommendation is beneficial for the stakeholders, so you need to use cause-and-effect language effectively(((TRUE)))
Reporting verbs also indicate how much you agree or disagree with the other sources in relation to your argument, so you may be positive if you use:(((confirm)))
Responding to people correctly is an example of good:(((Listening and team working skills)))
Stakeholder a popular way of identifying groups of people who have a legitimate interest in the way a business is being run. Stakeholder might typically be identified as employees and shareholders(((TRUE)))
Starbucks, the very popular American coffee. However, they have not been successful to reach south Africa market. This is mainly because of the very slow service and high price. Moreover, South-Africans like their coffee made of full fat milk and do not believe in adding a little creamer. The starting Situation in the above case study is? Select one:(((Starbucks had hopes of opening many stores in many sites in south Africa.)))
Starbucks, the very popular American coffee. However, they have not been successful to reach south Africa market. This is mainly because of the very slow service and high price. Moreover, South-Africans like their coffee made of full fat milk and do not believe in adding a little creamer. The starting Situation in the above case study is? Select one:(((Starbucks had hopes of opening many stores in many sites in south Africa)))
STEEPLE analysis consists of identifying …………… affecting a business.(((External factors)))
Strategy to avoid unintended plagiarism can be done by only two of the following paraphrasing and summarizing the information from different sources, and referencing these resources accurately: (((True.)))
Strengths and Weaknesses are Select one:(((Internal factors influencing the organization)))
Structure of a Problem-Solution text has steps SEQUENTIALLY listed as below:(((Situation, problem, cause, solution, evaluation)))
Students, tutors and children are known as:(((Concrete nouns)))
Super Wash is a very popular “Car Wash Service” group. However as there are few vansand or drivers they are not able to meet the demand always. The customers complain that sometimes during weekends they have to wait too long for a Super Wash van to arrive or are simply unavailable. The starting Situation in the above case study is? Select one:(((Super Wash is a popular car wash service group.)))
Super Wash is a very popular “Car Wash Service” group. However as there are few vansand or drivers they are not able to meet the demand always. The customers complain that sometimes during weekends they have to wait too long for a Super Wash van to arrive or are simply unavailable. The starting Situation in the above case study is? Select one:(((Super Wash is a popular car wash service group.)))
The -------- describes the situation in general.(((High level generalization)))
The ----------- Explains the key points by providing examples and more details.(((Low level generalization)))
The ----------- Explains the key points by providing examples.(((Low level generalization)))
The ----------- gives more details.(((Low level generalization)))
The advantage of using STEEPLE is that it helps you to identify ……… more clearly.(((Cause-and-effect factors)))
The aim of an argument is to persuade other people to refuse your claim, or at least seriously not to consider it(((False.)))
The analyzing process has a starting point in order to be completed, which is:(((Identifying the starting situation)))
The Brompton Bicycle case study show that, Brompton Bicycle, is the biggest bicycle manufacturer in Britain:(((Strengths)))
The Brompton Bicycle case study show that, Brompton have been facing seasonal demand issues.(((Threats)))
The Brompton Bicycle case study show that, Brompton, have been facing competition from many other bike manufacturers.(((Threats)))
The Brompton Bicycle case study show that, Brompton, have been facing seasonal demand issues.(((Threats)))
The Brompton Bicycle case study show that, the market has been growing 20% to 25% a year and they haven’t been growing the same rate.(((Weaknesses)))
The Brompton Bicycle case study show that, there is growth potential in a rapidly expanding global market.(((Opportunities)))
The Brompton Bicycle case study showed that, Brompton Bicycle is biggest bicycle manufactured is Britain. Select One:(((. Strength)))
The Brompton Bicycle case study showed that, Brompton have been facing seasonal demand issues.. Select one:(((Threats)))
The Brompton bicycle case study showed that, Brompton, have been facing competition from many other bike manufacturers.(((Threats)))
The Brompton Bicycle case study showed that, Brompton, have been facing seasonal demand issues.Select one:(((Threats)))
The Brompton bicycle case study showed that, the market, has been growing 20% to 25% a year and they haven’t been growing at the same rate..(((Weaknesses)))
The Brompton bicycle case study showed that, there is growth potential in a rapidly expanding global market(((Opportunities)))
The Brompton Bicycle study showed that, there is growth potential in a rapidly expanding global market. This will be considered as(((Opportunities)))
The Causes of the problem in the above case study are?(((There is a shortage of motorcycle delivery riders.)))
The Causes of the problem in the above case study are? Select one:(((A very slow service and high price. South-Africans like their coffee made of fullfat milk and do not believe in adding a little creamer.)))
The Causes of the problem in the above case study are? Select one:(((There are few vans and or drivers they are not able to meet the demand always.)))
The Causes of the problem in the above case study are? Select one:(((There is a shortage of motorcycle delivery riders.)))
The Fact Face case study shows that, there was little sales training for staff(((Weaknesses)))
The Fat Face case study showed that, Fat Face Limited was foundation in 1988 by Tim Slade and Jules Leaver, the company has grown to over 200 stores.(((Strengths)))
The Fat Face case study showed that, Fat Face Limited was founded in 1988 by Tim Slade and Jules Leaver, the company has grown to over 200 stores.(((Strengths)))
The Fat Face case study showed that, the middle market prices can be undercut by high street stores.(((Threats)))
The Fat Face case study showed that, there was an increase interest in online presences:(((Opportunities)))
The Fat Face case study showed that, there was little sales training for staff. This will be considered as:(((Weakness)))
The follows can be examples on language for proposing Solutions:(((It might/may/could be beneficial/better/effective/profitable, etc.)))
The key concepts and frameworks which enable analyzing a case study Select one:(((Select key information and organize your thinking in addition to understand the business context and make recommendations)))
The Lodge Bistro chain case study showed that, in 2010, Joyce Lodge retired. Her daughter, Miss Patricia Lodge committed as the chief executive officer of the Lodge chain.(((Strengths)))
The Lodge Bistro chain case study showed that, Joyce Lodge opened her first café in 1989. Moreover, in early 2000s, the business grew into a twenty-five restaurants across England:(((Strengths)))
The Lodge Bistro chain case study showed that, Patricia, the chief executive officer of Lodge Bistro chain, became increasingly concerned about the effects of the lack of communication between managers.(((Weaknesses)))
The Lodge Bistro chain case study showed that, the Lodge Bistro chain business, improve their brand.(((Opportunities)))
The Lodge Bistro chain case study showed that, the Lodge Bistro chain business, remained reasonably successful but experienced Poor location issues(((Threats)))
The Lodge Bistro chain study show that, the Lodge Bistro chain business, improve their brand.(((Strengths)))
The Lodge Bistro chain study show that,the Lodge Bistro chain business, remained reasonably successful but experienced Poor location issues.(((Threats)))
The Logo Bistro chain case study show that, in 2010, Joyce Lodge retired. Her daughter, Miss Patricia Lodge committed as the chief executive officer of the Lodge chain.(((Opportunities)))
The Logo Bistro chain case study show that, in 2010, Joyce Lodge retired. Her daughter, Miss Patricia Lodge committed as the chief executive officer of theLodge chain.(((Threats)))
The Logo Bistro chain case study show that, Joyce Lodge opened her first café in 1989. Moreover, in early 2000s, the business grew into twentyfive restaurants across England.(((Strengths)))
The Logo Bistro chain case study show that, the Logo Bistro chain business, remained reasonably successful but experienced poor location issues.(((Threats)))
The Logo Bistro chain case study shows that, Patricia, the chief executive officer of Lodge Bistro chain, became increasingly concerned about the effect of the Lack of communication between manager.(((Weaknesses)))
The main body of the report: analysis and financial data, should be listed in which following section?(((In the findings section)))
The method in Report structure means:(((How the information was collected.)))
The organization must/should/has to/needs to " you can use it if you want to provide:(((Recommendations)))
The paragraph that contains what is the company should change or start:(((Recommendations.)))
The Problem in the above case study is? Select one:(((They have not been successful to reach south Africa market.)))
The problem in the above case study is? Select one:(((Customers have to wait too long for a Super to arrive or are simply unavailable.)))
The problem in the above case study is? Select one:(((The orders are often cold by the time they are delivered.)))
The problem is:(((A negative situation threatening an individual, group or organization)))
The purpose of Referencing is: (((Allow others to check sources and their validity and authenticity.)))
The purpose of the citations is:(((. Helps the reader to find the source easily.)))
The purpose of the conclusion is:(((It summarizes the key claims made in the report)))
The purpose of the introduction is:(((It gives a brief context to the report and the company)))
The starting Situation in the above case study is(((Starbucks had hopes of opening many stores in many sites in south Africa.)))
The starting Situation in the above case study is?(((Super Wash comes to the doorstep of the customers with all accessories in vans.)))
The starting Situation in the above case study is?(((Lowell company, is expanding rapidly.)))
The threat of redundancy is causing great anxiety among unskilled workers in a distribution warehouse The word that does not describe problem situation is:(((Distribution warehouse)))
This technique involves stating ideas from the original source in your own words, it gives same meaning and change structure of the sentence:(((Paraphrasing.)))
To convince your reader with a claim, your argument should contain…...(((Evidence and Explanation)))
To have effective delivery of a presentation you should not:(((Reed from the slides)))
To persuade the reader with the claim, you need a sport for it:(((Explanation and Evidence.)))
To present the reader with the claim, you need a support for it:(((all the answers are true)))
To use evidence, you need data from research and other sourcesin the business world. Evidence can include facts, figures and reports(((True.)))
Using -------------- is useful for making good notes.(((Abstact Nouns)))
Using key concepts or a framework from a business context to examine a situation is called:(((Framing the case)))
We have to avoid plagiarism by:(((Paraphrasing and summarizing the information from different sources, and referencing those sources accurately including in-text citation and referencing list.)))
What are the Problem-signaling words and phrases?(((Shortage of motorcycle delivery riders)))
What are the solution and evaluation?(((Tim & Sons will recruit more motorcycle delivery riders)))
What is a SWOT analysis Select one:(((This acronym stands for strengths, weaknesses, opportunities and threats and it is a tool often used in business)))
What is an argument?(((A claim that is justified and supported in a variety of ways, such as)))
What is the Cause?(((Sam & Young delivery team they are NOt able to meet the demand always)))
What is the Effect?(((Customers sometimes complain they have to wait too long for Sam & Young drivers to arrive or are simply unavailable.)))
what is the situation? Select one:(((Tim & Sons, a bread delivery company, is expanding rapidly)))
What solution is proposed?(((Lowell will recruit more motorcycle delivery riders.)))
What solution is proposed? Select one:(((Making coffee with milk, lowering the price and increasing the speed of theservice.)))
What solution is proposed? Select one:(((Lowell will recruit more motorcycle delivery riders.)))
What we propose leads to …(((Solution)))
When a student use theories and frameworks such as Maslow’s theory of psychological needs and external business environment that is an example of:(((Explanation)))
When one effect may have several causes or one cause may have several effects, we call this relationship as:(((Multi-Causality)))
Which of the following example is showing the linking word that indicates to sequence of events or ideas.(((When it trains then snows)))
Which of the following linking words indicate to cause and effect:(((In order to, as a result, consequently.)))
Which of the following linking words indicate to cause and effect?(((Therefore, because.)))
Which of the following pattern represents this example: the meal is often cold by the time they are delivered, and customers complain about this.(((Restaurant Problem)))
Which of the following pattern represents this example: the meal is often could by the time they are delivered, and customers complain about this.(((Restaurant Problem)))
Which of the following reason is not important for listing references…(((To demonstrate your wide reading)))
Which of the following sentence is an example of claim (1) Heather was satisfied before Renata took over as supervisor. (2) This can be explained by using Maslow’s theory (1943) based on psychological needs. (3) She had good working conditions with a regular income: she seemed to have had job security: a good relationship with all levels of staff within the branch, socializing with them at Christmas meals and a small party for Jenny’s retirement.(((Sentence 1)))
Which type of linking word is However?(((Comparison)))
Which type of linking word is moreover?(((Addition)))
While preparing a Power Points Presentations and when you have a lot of information:(((You should use handouts or papers with detailed information)))
Why is the problem(((Cause)))
You might perceive a situation as a problem while someone else might be quite satisfiedwith the same situation. Select one:(((True.)))
STEEPLE analysis is a categorizing framework that divides the external environment of a business into seven areas ((True))
Fat Face Limited was founded in 1988 by Tim Slade and Jules Leaver, the company has grown to over 200 stores, selling womenswear, menswear, childrenswear, accessories and footwear within the United Kingdom and Ireland alone, as well as stores in Singapore and Dubai. ((Strengths))
The Lodge Bistro chain case study showed that, Patricia, the chief executive officer of Lodge Bistro chain, became increasingly concerned about the effects of the Lack of communication between managers. This will be considered as:((Weaknesses))
Stakeholder A popular way of identifying groups of people who have a legitimate interest in the way a business is being run. Stakeholders might typically be identified as employees and shareholders ((True))
In SWOT analysis, strengths and weaknesses are internal to the orgnization. ((True))
Problems are a matter of perception means: ((One person may perceive a situation as a problem while someone else may be quite satisfied with the same situation.))
The Lodge Bistro chain case study showed that, Joyce Lodge opened her first café in 1989. Moreover, in early 2000s, the business grew into a twenty-five restaurants across England this will be considered as: ((Strengths))
Identifying the situation is the first step in structuring problem-solution pattern: ((True))
The Brompton Bicycle case study showed that, Brompton Bicycle, is the biggest bicycle manufacturer in Britain. This will be considered as: ((Strengths))
A familiar problem-solution pattern consists of: situation, problem, _________, solution, and evaluation. ((causes))
The first step in structuring problem-solution pattern is: ((Identifying the situation))
In SWOT analysis, strengths mean what businesses are good at ((True))
In Fat Face case study, the increased business/brand awareness among consumers is: ((Opportunities))
In your own cafe, if you are NOT providing an adequate employee training, it is considered ... ((Weaknesses))
In SWOT analysis, weaknesses mean what orgnization is not so successful at ((True))
The Political factors facing a business in its environment are rules and regulations imposed by the government and affect the business ((True))
In SWOT analysis, weaknesses and threats are harmful to achieve the objective of the orgnization. ((True))
The Brompton Bicycle case study showed that, Brompton, have been facing competition from many other bike manufacturers. This will be considered as: ((Threats))
In your own cafe, if you are offering more organic snacks or salads on the menu that is... ((Strengths))
The Fat Face case study showed that, there was an increased interest in online presences. This will be considered as: ((Opportunities))
The Sociological factors facing a business in its environment are the forces within society that might affect the business. ((True))
In your own cafe, if you are NOT providing proper customer services. This will ... ((Weakness))
The Technological factors facing a business in its environment are the forces within society that might affect the business. ((False))
After finishing your report, you have to check spelling, grammar and facts and figures and proofread:(((TRUE)))
The section of the report that includes what should be changed is:(((recommendations)))
In-text referencing is an alphabetical list at the end of your report of all the sources used with their information.(((FALSE)))
The section of a report that shows how information was collected is:(((method)))
To convince the reader with the argument, you need:(((To have claim and a supporting explanation or evidence)))
Which sentence indicates boosting:(((I am sure, it is more important for students to understand ideas and concepts than learning the facts only.)))
To add a connecting idea to an argument, the linking word used is …(((Moreover)))
................… is used to show uncertainty, modesty and respect for others’ views in an argument?(((Hedging)))
The aim of an argument is to:(((All answers are correct)))
Read the sentences, that are numbered 1, 2, etc., which of the concepts below implement in sentences 2 and 3: (1) The graph shows the number of telephones owned per thousand of the population in different countries over a five-year period. (2) Overall, the number of phone owners per thousand of the population differ considerably. (3) However, numbers tended to fall in countries with the highest level of phone ownership, whereas numbers generally rose in countries which had fewer phone owners in 2000.(((Evidence)))
The executive summary in a report is:(((abstract)))
References are commonly used in assignments, including business reports.(((TRUE)))
Passing off someone else’s work as your own without acknowledgement of the source is called:(((plagiarism)))
(De Tienne and Lewis, 2005, p. 362) is an example of: ((in-text citation))
Business presentations can include images that are not relevant to the messages ((false))
To have effective delivery of a presentation you should NOT use (( No need to monitor audience and time ))
In order to communicate effectively you should be less aware of the Purpose ((false))
Business presentations rarely include some evidence like figures or graphs ((false))
Inability to work with people from different backgrounds and culture is essential in team-working skills ((false))
The beginning of the case study is important as it sets the scene. ((true))
Business presentations usually include: ((Some evidence like figures or graphs))
The ______________ develops the main points by examples and more details. ((Low level generalization))
Sketches or diagrams are: ((Methods of making notes))
To have effective delivery of a presentation you should NOT: ((monitor audience and time))
______________ are things that you feel or think about. ((Abstract nouns))
Business presentations rarely include some evidence like figures or graphs.((false))
_____________ means looking back at something you have done or experienced. ((Reflection))
--------- are things you can see, hear, smell, taste and/or touch them.(((Concrete nouns)))
-------- means looking back and think about what you have learned and what to do next as a result of that learning.(((Reflection)))
The Brompton Bicycle case study showed that, the market, has been growing 20% to 25% a year and they haven't been growing at the same rate. This will be considered as: ((Weakness))
Read the sentence below, which of the linking words/ phases connect the sentence perfectly: This person is poor and has no house. …. he had money; he wouldn't buy a house.(((Even if)))
’--------- are things you can see, hear, smell, taste and/or touch them.(((Concrete nouns)))
’-------- means looking back and think about what you have learned and what to do next as a result of that learning.(((Reflection)))
The Brompton Bicycle case study showed that, the market, has been growing 20% to 25% a year and they haven’t been growing at the same rate. This will be considered as: ((Weakness))
Read the sentence below, which of the linking words/ phases connect the sentence perfectly: This person is poor and has no house. …. he had money; he wouldn’t buy a house.(((Even if)))
• Strengths • Threats In SWOT analysis, opportunities and threats are external to the organization.(((TRUE)))
What is true of wholesalers? (((Wholesalers are extremely important because of the marketing activities they perform.)))
Many service providers are considered retailers because they (((provide their services directly to consumers)))
A merger occurs when (((two companies combine to form a new company)))
A(n) ________ is a partnership established for a specific project or for a limited time. (((joint venture)))
An entrepreneur has been primarily associated with the willingness to((take risks.))
What is one of the difficulties faced by small business owners?((worries about employee problems))
What is a difference between high technology businesses and other small businesses?((High technology businesses require greater capital and have higher initial startup costs thanother small businesses))
What is true of a capitalist economic system?((Prices of goods and services are determined by supply and demand))
The quantity of goods and services that consumers are willing to buy at different prices at a specific time is referred to as((demand.))
The quantity of products that businesses are willing to sell at different prices at a specific time is referred to as((supply.))
What is a defining characteristic of public corporations?((Their stock can be bought, sold, or traded by anyone))
Which business provides a service, but is neither owned by the government nor focuses on earning profits?((a nonprofit corporation))
What is a true statement about the board members of a corporation?((They have a duty of care and loyalty to oversee the management of the firm.))
Preferred stockholders of a corporation((have a claim to profits before other stockholders do.))
To make an effective and successful meeting, participants should distribute the minutes during the meeting(((FALSE)))
Section in the workplace proposals that shows what author found after data analysis. IT represents... (((The report's recommendation)))
To: The Executive Board; From: Senior Human Resources Officer; "Proposal to improve the recruitment and retention of careers" The above text is an example to(((Identify the audience and goal)))
Participating in group discussion during a meeting can decreases the possibility of failing meeting(((TRUE)))
Employees need to communicate in order to achieve business profits and to create a healthy body and constructive working environment.(((TRUE)))
Which of the following behavior can lead to a Fail meeting?(((Looking up for different direction. EX: Ceiling or mobile)))
Which of the following meeting contains agenda and minutes should be circulated?(((Brief meeting with in the workplace)))
Informal Meetings are prepared for official operational meetings in the organization or the brief meetings to discuss a big problem needs to be solved.(((FALSE)))
The reason that the meeting may fail....(((All members have no contribution at all)))
Usually during meeting, an effective management should(((Define the purpose and the meeting out comes)))
Legal binding decision means that any result done by.....(((Formal meeting in the workplace)))
Over talkers in the meeting they can cause.(((Meeting Failure)))
Formal meetings when board of directors are gathered to discuss financial issues, writing agenda, meeting with a committee where decisions are made.(((TRUE)))
Problem-solving documents are:(((Workplace proposal)))
Prior to any meeting, active management should place(((A documented written agenda)))
The Lodge Bistro chain case study showed that, Patricia, the chief executive officer of Lodge Bistro chain, became increasingly concerned about the effects of the Lack of communication between managers this will be considered as: ((Weaknesses))
You might perceive a situation as a problem while someone else might be quite satisfied with the same situation. ((True))
Stakeholder a popular way of identifying groups of people who have a legitimate interest in the way a business is being run Stakeholders might typically be identified as employees and shareholders ((True))
The Fat Face case study showed that the company has grown to over 200 stores this will be considered as: ((Strength))
The Brompton Bicycle case study showed that, Brompton, have been facing competition from many other bike manufacturers this will be considered as: ((Threats))
The Brompton Bicycle case study showed that, there is growth potential in a rapidly expanding global market this will be considered as: ((Opportunity)
The Lodge Bistro chain case study showed that, Joyce Lodge opened her first café in 1989. Moreover, in early 2000s, the business grew into a twenty-five restaurants across England this will be considered as: ((Strength))
The Fat Face case study showed that, there was an increased business/brand awareness among consumers. This will be considered as: ((Opportunity))
Which of the following pattern represents this example: the meal is often cold by the time they are delivered and customers complain about this. ((Restaurant Problem))
In your own cafe, if you are offering more organic snacks or salads on the menu in order to take advantage of trends which linked to eating healthier this will be considered as: ((Strengths))
The ______________ describes the situation in general. (((High level generalization)))
Paragraphs usually start with ______________ then move on to ______________ (((General information, Specific details)))
______________ are things you can see, hear, smell, taste and/or touch them (((Concrete nouns)))
Using______________ is useful for making good notes. (((Abstact Nouns)))
The purpose of citations is: ((helps the reader to find the source easily.))
The step in the formal planning process known as __________ involves studying past events, examining current conditions, and forecasting future trends.(((analyzing the situation)))
The outcome of a situational analysis is the(((identification and diagnosis of planning assumptions, issues, and problems.)))
Situational analysis, as part of the formal planning process, focuses on(((the internal forces at work within an organization and examines influences from the external environment. )))
In the context of the planning process, __________ are the targets or ends a manager wants to reach.(((goals)))
Smooth Coat, a paint manufacturer , developed a strategic plan that assumed continued growth in its industry. Additionally, it generated several __________ for different economic settings. When a recession started, Smooth Coat quickly implemented the plan based on those conditions. As a consequence, it survived more easily than other firms in the industry.(((scenarios)))
Year in and year out, Coca-Cola outperforms in the soft drink industry. While this success is due in part to the quality of their products, it is also due to Coca-Cola's consistently strong marketing campaigns. These campaigns represent Coca-Cola's ongoing, or __________, plan to develop and execute a world-class marketing and promotion strategy.(((standing)))
A plan to integrate employees into a new payroll system, assuming that it is not likely to be repeated in the foreseeable future, is an example of a __________ plan.(((single-use)))
According to the __________ plan of Castleton Industries, if expansion to new markets in Australia does not materialize, the company will strive for expansion in Asia.(((contingency)))
The managers of Tanika Enterprises are considering how well alternative plans may meet its high-priority goals, as well as identifying the cost of each initiative and its likely investment return. Tanika Enterprises is in the __________ stage of the planning process.(((evaluating goals and plans)))
During the __________ step of the planning process, managers identify the priorities and trade-offs among goals and plans.(((selection of goals and plans)))
After Jenn's department performed poorly two years, she reviewed her plan and saw it had inadequate funding and unclear priorities. The next year, she acquired sufficient funds and incentivized her top managers with bonuses for goals met. The results made Jenn ecstatic, showing how the implementation of plans is successful when(((they are linked to other organizational systems like budgets and rewards.)))
The final step in the formal planning process involves(((monitoring and controlling performance.)))
In the context of planning, if there is alternative generation, evaluation, and selection steps which reveal several different future scenarios, a different __________ plan is attached to each scenario.(((contingency)))
The planning step known as monitoring and controlling performance(((involves managers continually monitoring the performance of their work units against the unit's goals and plans.)))
Which level of manager is known as a tactical manager?(((middle-level)))
Typical strategic goals involve(((increasing market share.)))
Strategic goals tend to have a strong __________ orientation.(((external)))
Strategic goals tend to relate to(((the long-term survival of an organization.)))
The level of planning involving the longest time horizon, usually ranging from three to seven years, is __________ planning.(((strategic)))
__________ planning involves a process of translating broad strategic goals and plans into specific goals and plans relevant to a particular portion of an organization(((Tactical)))
__________ planning is the process of identifying the specific procedures and processes required at lower levels of an organization.(((Operational)))
In __________ planning, frontline managers usually focus on routine tasks such as production runs, delivery schedules, and human resource requirements.(((Operational)))
"For organizations, the formal planning model is generally __________, with top-level strategies flowing down through the levels of the organization into more specific goals and plans.
"(((hierarchical)))
To be effective, an organization's strategic plans should focus on goals that are(((aligned with its tactical plans and operational plans.)))
Which statement regarding the role of managers in strategic planning is accurate?(((Managers throughout the organization may be involved in developing strategic plan elements. )))
The formal planning model emphasizes a __________ approach.(((top-down)))
A mission statement describes the organization's(((basic purpose.)))
Senior management at Durable Manufacturing knows that a successful implementation of the business plan in the coming year is critical to the company's survival in a competitive industry. Durable cannot afford for different parts of the organization to "stray" from the plan; in fact, each organizational unit must perform in sync with the goals of the unit above it and the units below it. Thus, the individual plans of the corporation, each division, each department, and each section must be(((aligned)))
The strategic vision of an organization(((provides a perspective on where the organization is headed.)))
The tangible assets of an organization include(((real estate)))
Resources provide competitive advantage only under certain circumstances, including when they are(((difficult to imitate.)))
The sleek appearance of Apple products helps set it apart from its competitors and makes the products top sellers. For example, the curved design and dual-camera system in the newest iPhone are examples of Apple's(((core capabilities.)))
Homestyle Treats, a chain of cookie shops, periodically compares the quality of its food and service with its competitors in order to continually improve its performance. This corporate strategy is known as(((benchmarking.)))
Cameron's manager and mentor, Oliva, is helping Cameron develop his first business plan. Oliva said, "One of the most powerful tools that you can use in developing a strategic plan for your business is to analyze the external environment and your internal resources. I will help you formulate this strategy by using a technique called(((SWOT analysis.")))
A __________ strategy identifies the set of businesses, markets, or industries in which the organization competes and the distribution of resources among those businesses.(((corporate)))
In order to reduce production costs, Autoworks an automobile manufacturer, decided to buy out a glass plant and begin manufacturing the glass for the windows of cars on its own. The corporate strategy adopted by the company is known as(((vertical integration.)))
Southern Spirits had focused only on alcoholic beverages for the last two decades. Now, as part of a(n) __________ strategy, the company is considering entering the soft drink market.(((related diversification)))
The __________ matrix is not a substitute for management judgment; it helps managers of businesses evaluate their strategy alternatives.(((BCG)))
Deisha viewed her staff as a competitive advantage, as they had the necessary academic degrees and experience. Her division manager said, "Deisha, until you have staff with rare and valuable academic research experience and corresponding degrees, and with talents that are world-class and in extremely high demand by your clients, such that they are willing to pay top dollar for that talent, your staff do not represent a competitive advantage. Your staff are too __________ by your competitors."(((easy to imitate)))
The goal of __________ is to thoroughly understand the best practices of other firms and to undertake actions to achieve better performance and lower costs.(((benchmarking)))
The corporate strategy of __________ involves expanding the domain of an organization into supply channels and to distributors.(((vertical integration)))
Lodestar Inc. recently decided to expand from its original base in construction to several wide-ranging industries such as trucking, lawn mower manufacturing, and catering. Lodestar is using the __________ corporate strategy.(((unrelated diversification)))
Harris Industries is evaluating the purchase of a small firm, StrongTech Inc. which manufactures products involving advanced technology. These products are expected to revolutionize the home construction industry. StrongTech has only a minor share of this market, but the market is expected to grow rapidly. Substantial investment will be required. In the BCG matrix, StrongTech Inc. would be placed in the __________ category.(((question marks)))
Groups and individuals who affect and are affected by achievement of the organization’s mission, goals, and strategies are considered (((stakeholders.)))
If an organization's mission statement describes the organization as it currently operates then its strategic vision points to the(((future.)))
In the BCG matrix, a star represents a company(((with high growth and a strong competitive position in the market.)))
Functional strategies are implemented(((by each appropriate area or unit.)))
Businesses using a __________ strategy try to be efficient and offer a standard, no-frills product.(((low-cost)))
The most effective strategy that can be adopted by strategy managers is one that(((competitors are unable to imitate.)))
The first step in strategy implementation involves articulating in simple language what a particular business must do to create or sustain a competitive advantage. This step involves(((defining strategic tasks.)))
When a problem is frequent, repetitive, and routine, with much certainty regarding cause-and-effect relationships, a __________ decision may be taken.(((programmed)))
At Toni's retirement party, she reflected upon her long career in starting up successful companies. She said, "I salute my long-time mentor who counseled me early in my career about risk. He said, “When you are confronted with risk when making management decisions, you should(((anticipate it, minimize it, and control it.'")))
What is typical of most managerial decisions?(((They lack structure.)))
A small U.S.-based company is trying to decide whether to open its first international branch in Europe or Asia. The managers of the company are operating under a condition of(((uncertainty.)))
In managerial decision making, when one can estimate the likelihood of various consequences but still does not know with certainty what will happen, the manager is facing(((risk)))
"In managerial decision making, the state that exists when decision makers have insufficient information is known as "(((uncertainty.)))
"(((uncertainty.)))
__________ is one of the six stages of formal decision making.(((Evaluating the decision)))
The first step in an ideal decision-making process is to(((identify and diagnose the problem)))
The manager of a restaurant noticed that the number of customers in the evening was decreasing. She promptly ordered the chef to revise the evening menu. Later, customer feedback indicated that the problem was not the menu but poor service from the waitstaff. The manager's decision to have the menu revised suggests that she failed to(((identify and diagnose the problem.)))
When generating alternative solutions during the decision-making process, __________ solutions must be designed for specific problems.(((custom-made)))
Jean is evaluating a new customer relationship management software solution for her business. She knows several business owners and managers who have implemented the software in their organizations. Instead of conducting a trial in her own company, she asks them for their advice. Jean is most likely searching for a(n) __________ solution to her problem.(((creative)))
When making a choice, __________ requires searching thoroughly for a complete range of alternatives, carefully assessing each alternative, comparing one to another, and then choosing or creating the very best.(((maximizing)))
Patrice has to select paper clips to purchase for her office. She looks at the office supply catalog and picks the first one that is priced reasonably. Patrice is using the __________ method of decision making.(((satisficing)))
For the decision-making process in an organization, those who __________ a decision must understand the choice and must be committed to its successful implementation.(((implement)))
With managerial decision making, decision evaluation is useful(((whether the conclusion is positive or negative.)))
Decision makers who search for __________ solutions use ideas they have tried before or follow the advice of others who have faced similar problems.(((ready-made)))
Managers who use the method of optimizing to make their decisions will attempt to choose(((an alternative that achieves the best possible balance among several goals.)))
In the context of formal decision making, negative feedback indicates that(((implementation of the decision may need more resources.)))
__________ is a psychological bias that applies to students who do not spend sufficient time studying.(((Discounting the future)))
The psychological bias known as illusion of control refers to a(((belief that one can influence events even when one has no influence over what will happen.)))
Illusion of control, discounting the future, and framing effects are examples of(((psychological biases.)))
__________ refer(s) to a decision bias influenced by the way in which a problem or decision alternative is phrased or presented.(((Framing effects)))
Discounting the future refers to(((weighing short-term benefits more heavily than long-term benefits.)))
A company recently made changes to its organizational structure. While deciding upon the changes to be implemented, the company invited many employees to discuss the issue with the senior management. This enabled the employees to hear the relevant arguments both for the chosen alternative and against the rejected alternatives. An advantage of using employees in this group discussion was that(((the employees understood why the decision was made.)))
An advantage in using a group for decision making is that it(((leads to a higher level of commitment to the decision.)))
Jason wanted the advantage that comes with the collective wisdom of a management team participating in a decision. Jason's division had to decide if solar panel, wind turbine blade, or battery power research would be its next major investment. He brought together his managers in a meeting; however, Tarek used his strength of character and oratory skills to push for solar panel research. Unfortunately, the other managers were quiet, and Tarek managed to overpower them with his views. Given the dynamics of the meeting and its outcome, __________ had prevented Jason's meeting from achieving what he had hoped it would.(((domination)))
__________ is the phenomenon that occurs in decision making when group members avoid disagreement as they strive for consensus.(((Groupthink)))
When decision making is done in groups, the job of a devil's advocate is to create(((the likelihood of constructive conflict.)))
Five executives attended a meeting that was being held at the company. A decision needed to be made about the following year's marketing campaign. Brant, an outspoken and aggressive person, insisted that his ideas be followed because there could not be a better alternative. No one else contributed to the discussion, and Brant's views were accepted. This group discussion suffered from(((domination.)))
In brainstorming, group members generate(((as many ideas about a problem as they can. )))
When a group accepts an alternative in order to end a meeting, the group has reached a(n) __________ decision.(((satisficing)))
__________ is a condition that occurs when a decision-making group loses sight of its original purpose and a new, less important aim emerges.(((Goal displacement)))
Leaders of decision-making groups who want to encourage constructive conflict should(((encourage multiple alternatives from a variety of perspectives.)))
"In an effort to combat groupthink and an overall lack of creativity, the group leader assigned Raisa the job of criticizing ideas, for the sake of argument, throughout the meeting. Raisa is playing the role of a
"(((devil's advocate.)))
In an attempt to make a conflict between two employees less personal and emotional, their manager instructs them to approach the same problem from a different viewpoint. They have a structured debate and come to a conclusion that suits both employees. This is an example of using the __________ approach.(((dialectic)))
What is a feature of effective brainstorming?(((A group is encouraged to say anything that comes to mind, except to criticize other people. )))
The senior management of Galaxy Industries has conducted a strategic planning meeting. Based upon the discussion, decisions were made regarding(((whether the company should expand into a new region in the next 10 years.)))
The lab manager devised a schedule so that the lab chemists and staff could continually run the experiments over the next two weeks. This decision is part of operational planning because it(((involves a specific procedure that is required at lower levels of the organization.)))
As part of a strategic planning process, Northeast Power's senior executives determined positive findings from their SWOT analysis: (1) New regulations will provide tax credits for renewable ("green") power sources. (2) Their customers will pay higher prices for green power. (3) A competing power utility that owns renewable power sources is struggling and might be a target to be acquired by Northeast Power. (4) As compared to their competitors, Northeast Power's management team is one of the best in the industry. A strength of Northeast Power, per the SWOT analysis, is that(((their management team is one of the most skilled in the industry.)))
The CEO of GoodLife Corporation, said, "I like what I heard from our SWOT analysis. It found: (1) We have an excellent sales staff. (2) Our management team is one of the best in the industry. (3) Demand for our products in the underserved overseas market is projected to soar. (4) Our factories use the latest and most efficient equipment. (5) And our cost control system is world class." An opportunity for Goodlife, per the SWOT analysis, is that(((demand for their products in the underserved overseas markets is projected to soar.)))
The owner of Frank's Fine Foods, a family-owned grocery store, said to his management team following their SWOT analysis, "We're in trouble. We have been the leading grocer in this geographical area, but Wholesome Groceries is opening a superstore a mile away. Our refrigeration system is 15 years old and needs replacing. Our store manager and produce manager are retiring this year. And our employee benefits package is not competitive." A threat to Frank's Fine Foods, per the SWOT analysis, is that(((Wholesome Groceries is opening a superstore nearby.)))
"Our business needs a steady supply of raw milk," said the CEO of Happy Cow Ice Cream, "But Midwest Dairy Farms is unable to deliver with certainty, and they keep raising prices. This year, I want us to expand our business by buying our own dairy farm." The CEO is advocating a(n) __________ strategy.(((vertical integration)))
"I am pleased to announce that we have acquired Fancy Fruit Juices, a premier producer of natural beverages," said Louis Garcia, CEO of Granddad's Root Beer. "This will allow us to take advantage of our strengths by applying them in a new business that is similar to our core business." Louis is speaking of a(n) __________ strategy.(((related diversification)))
"The last three years of severe drought conditions have been brutal to our lawn care business. We survived, but our business is too concentrated," said the owner of Anderson & Sons. "I propose that we expand in new areas—trucking, catering, and dry cleaning—to minimize our risks due to market fluctuations in one industry." This business owner is advocating a(n) __________ strategy.(((unrelated diversification)))
The management team of Community Care Medical Center is deciding whether to include more advanced, and substantially more costly, technologies. This is a nonprogrammed decision, because the underlying problem(((is novel and unstructured.)))
A finance manager, told his boss, "Jake, just like last month, I have determined appropriate revenue recognition for each of the 55 projects this month, based upon company policy and our standard procedures. Each of the projects has routine revenue accrual. I have prepared financial affidavits for each of these 55 projects, and I recommend that you approve the revenues by signing the affidavits." Jake's decision to approve the revenues is an example of a programmed decision because the underlying problem(((is frequent, repetitive, and routine. )))
"We have 125 qualified candidates for the quarterly training program but only 25 slots," said the production manager. "In that case, randomly select 25, and we will train the other candidates later this year during the next rounds of training," said the division manager. By selecting the first 25 qualified candidates at random rather than examining each candidate's file to determine if more qualified candidates should be selected first, the managers are engaging in(((satisficing.)))
During a managers' meeting, there is difficulty reaching a conclusion on which project to fund in the upcoming year. In arguing for funding for her project, Lorene states that she has experience in making all projects succeed. She even states that she is immune to failure. Lorene is displaying what barrier to decision making?(((illusion of control )))
"We must jump into this new business area right now, or we will fall behind," argued Dave. "Let's price our product well below our competitors' prices, even if we don't make money at first. We will figure out how to make money later." Dave is displaying which bias in decision making?(((discounting the future)))
When James presented his plan to his boss, he emphasized its huge money-making potential without pointing out its inherent risks and ethical challenges. James was using __________ to sell his position.(((framing effects)))
The new CEO told his executive team, "Every two years, on a rolling basis, we will develop a new plan that will address our high-level mission, objectives, and approaches to conduct business for the next five years. The plan will guide decision making that is consistent with our long-term goals and strategies." The five-year plan is an example of a __________ plan.(((strategic )))
"The operator's manual has a purpose—to allow the equipment to meet its performance goals and last for a long time," said the plant manager to an employee. "You didn't perform scheduled maintenance on the equipment as specified in the manual, and now the equipment is ruined." Following the instructions described in the operator's manual is an example of a __________ plan.(((standing)))
"The dam is on the verge of failing, and the downstream residents are in danger!" cried the on-scene reporter. The power plant manager, replied, "Stay calm. We implemented the emergency protocol over 24 hours ago, and we have evacuated all potentially affected residents." The emergency protocol is an example of a __________ plan.(((contingency)))
In an organization, the process of planning moves in a linear fashion.(((FALSE)))
Effective planning allows for changing conditions.(((TRUE)))
Standing plans are designed to achieve a set of short-term goals.(((FALSE)))
Making decisions about an organization's short-term goals and tactics is known as strategic planning.(((FALSE)))
A strategic control system must encourage efficient operations while allowing flexibility to adapt to changing conditions.(((TRUE)))
Individuals usually make higher-quality decisions when compared to groups, provided there is enough time.(((FALSE)))
__________ refers to the moral principles and standards that guide the behavior of an individual or group.(((Ethics)))
A(n) __________ is a situation, problem, or opportunity in which an individual must choose among several actions that must be evaluated as morally right or wrong.(((ethical issue)))
Morgan, a member of the sales team, must choose between going along with his colleagues or informing his manager about inaccuracies on some coworkers' expense reports. In this context, Morgan is faced with a(n)(((ethical issue.)))
What are the moral principles and standards that guide behavior in the world of business?(((business ethics)))
__________ refers to the principles, rules, and values people use in deciding what is right or wrong.(((The legal system)))
Delores owns a small tax preparation firm. After receiving the catering bill for a recent staff event, she believes they were significantly undercharged, and notifies the catering firm to correct the error. In taking this action, Delores is relying upon(((moral philosophy. )))
__________ is an ethical system which states that all people should uphold certain values that society needs to function.(((Universalism)))
Two basic ethical ideals underpin the Caux Principles for Business. They are(((kyosei and human dignity.)))
Rand confronts his supervisor when he observes new employees being intimidated into performing dangerous work without proper training. He believes all members of the organization must uphold some values for it to function well. Rand was following the __________ ethical system.(((universalism)))
According to the Caux Principles for Business, the ethical ideal of kyosei means(((living and working together for the common good.)))
The notion of __________, an ethical system, is similar to Adam Smith's concept of the invisible hand in business.(((egoism)))
Which statement would a proponent of egoism believe?(((People should engage in the behavior that promotes the greatest good for themselves. )))
The ethical system of utilitarianism supports the idea that(((seeking the greatest good for the greatest number of people should be the overriding concern of decision makers.)))
In the workplace, the Chinese concept of guanxi reflects what standard governing ethical behavior?(((Ethical behavior is defined based on the opinions and behaviors of associated people.)))
"Zari is a police officer, and usually makes decisions based on what her fellow officers would expect her to do. Her actions reflect her use of the __________ ethical system.
"(((relativism)))
What ethical system acknowledges the existence of different ethical viewpoints?(((relativism )))
Manual's workplace requires that all employees be fully vaccinated, including annual flu shots, before they are allowed to enter company premises. They must also submit proof of first aid training. This policy is an example of which ethical system?(((utilitarianism)))
The virtue ethics perspective suggests that an individual's moral judgment and ethical behavior should be based upon(((a strong personal value system.)))
According to Kohlberg's model of cognitive moral development, people in the __________ stage make decisions based on concrete rewards and punishments and immediate self-interest.(((preconventional)))
According to Kohlberg's model of cognitive moral development, people in the __________ stage see beyond authority, laws, and norms and follow their self-chosen ethical principles.(((principled)))
As the owner of a home health aide business, Grant is concerned that his employees exercise the highest levels of judgment and moral reasoning. He evaluates his staff based on both industry regulations and their ability to apply their own personal values system to their work. Grant runs his business based on which ethical value system?(((virtue ethics)))
In the context of universalism, kyosei and human dignity are the two basic ideals underpinning the __________ Principles for Business.(((Caux)))
In Kohlberg's model of cognitive moral development, the very highest stage of moral development is the __________ stage.(((principled)))
A company believes ethical lapses within its daily operations will be brought to public attention by workers acting as whistleblowers. Thus, it does not need to establish a formal code of conduct for employees, instead relying on workers' values to guide ethical decision making. Management of this company displays what ethical perspective?(((relativism)))
__________ is the ethical system wherein one seeks the greatest good for the greatest number of people.(((Utilitarianism)))
A tenet of __________ is that all people should uphold certain values, such as honesty, fairness and respectfulness that society needs to function.(((universalism)))
A tenet of __________ is that ethical behavior is based on the opinions and behaviors of associated people.(((relativism)))
According to Kohlberg's model of cognitive moral development, people in the conventional stage conform to(((expectations of ethical behavior held by groups or institutions. )))
Taylor conforms to the norms established by her religion and ensures that her behavior always meets the expectations set forth by her parents and the church. According to Kohlberg's model of cognitive moral development, she resides in the __________ stage.(((conventional)))
Lawrence sees a forgotten package lying near the locked mailboxes inside his apartment building. He walks past it only because he's sure there's a secret security camera watching the lobby. According to Kohlberg's model of cognitive development, Lawrence resides in the __________ stage.(((preconventional)))
Andrew prefers theater to business, and dreams of becoming a performer one day. However, he decides to major in accounting because his parents expect him to join the family business right after college. According to Kohlberg's model of cognitive moral development, Andrew resides in the __________ stage.(((conventional)))
Janelle, a scientist working in the research department of a personal care products manufacturer, is asked to overlook the ill-effects of using a new product that's ready for launch. Janelle's boss offers a bonus and extra vacation time in exchange for skipping a few details in the product report. Janelle refuses to do this and asks the firm to withhold the product from entering the market. According to Kohlberg's model of cognitive moral development, Janelle resides in the __________ stage.(((principled)))
Which statement describes an ethical dilemma facing modern managers?(((Nearly three-fourths of Americans say executives' pay packages are excessive.)))
What was the intended goal of the Sarbanes-Oxley Act?(((to establish strict accounting and reporting rules to make senior managers more accountable)))
The __________ of an organization refers to the processes by which decisions are evaluated and made on the basis of right and wrong.(((ethical climate)))
Tiana, a graphic artist at an advertising firm, realizes that the head designer has been copying designs from the Internet and passing them off as his own. Tiana informs the marketing head about this, which leads to the designer getting fired. However, after this event, Tiana is ostracized by her coworkers. This indicates that the organization has a poor(((ethical climate.)))
A company regularly conducts classes in moral philosophy for its employees. In addition, it ensures that whistleblowers are protected from ostracism or mistreatment. These measures have been specifically taken to(((create an ethical climate in the organization.)))
Carlos reported that the industrial wastewater treatment system operated by his employer was no longer operating properly, allowing toxics-laden water to be discharged to the river. He was pleased to see the plant manager, Darnell, immediately shut down production for an indefinite period, pending repair of the wastewater treatment system. In this situation, Darnell demonstrated(((willingness to take an ethical stand that may impose financial costs.)))
For businesses with an international presence, maintaining a positive ethical climate is(((particularly complex and challenging.)))
What is a legal advantage to companies that make an effort to comply with stipulations of the 2002 Sarbanes-Oxley (SOX) Act?(((The companies can reduce the consequences if an employee does break the law.)))
Managers need to __________ so that employees will discuss problems in meeting goals, rather than resorting to unethical and possibly illegal behavior.(((keep the lines of communication open)))
In the context of business ethics, an ethical leader(((influences others to also act ethically.)))
In the context of business ethics, the Sarbanes-Oxley Act requires(((that public companies disclose whether they have adopted a code of ethics.)))
In the context of corporate ethics programs, ethics committees(((develop policies, evaluate actions, and investigate violations)))
__________ ethics programs are designed by corporate counsel to prevent legal violations.(((Compliance-based)))
The primary focus of compliance-based ethics programs is on(((detecting and punishing legal violations.)))
What strategy is used by compliance-based corporate ethics programs?(((increased surveillance and controls and imposing punishments on wrongdoers)))
An ethics program that is designed to instill in people a personal responsibility for ethical behavior is called a(n) __________ ethics program.(((integrity-based)))
A company was required by law to change its physical work environment in such a way so as to enable disabled people to function properly. Because the company followed a(n) __________ ethics program, it did not stop at just making changes that would help it stay out of trouble with the law. It went further and trained its employees to be sensitive toward disabled people.(((integrity-based)))
Integrity-based ethics programs in business organizations are based upon the premise that(((companies and people govern themselves through a set of guiding principles that they embrace.)))
In the context of ethics programs in organizations, programs tend to be better integrated into operations, thinking, and behavior when(((top management has a personal commitment to responsible ethical behavior.)))
"A(n) __________ ethics program is designed to instill in people a personal responsibility for ethical behavior. ((integrity-based))
"(((integrity-based)))
In the context of making ethical decisions, knowing what actions are morally defensible is referred to as moral(((judgment.)))
Priya is considering having to lay off Larry, because Larry's skills no longer match the company's needs in the face of a changing marketplace. Larry is a long-time employee who organizes many office social gatherings and company-sponsored charitable events. Priya knows that letting Larry go would have a huge negative impact on the other employees and that there would likely be unintended consequences of this decision. In the context of making an ethical decision, this stage is known as moral(((awareness.)))
In the context of making ethical decisions, moral __________ is the strength and persistence to act in accordance with one's ethics despite the challenges.(((character)))
The concept of __________ holds that only a person unaware of his or her own identity can make a truly ethical decision.(((the "veil of ignorance")))
A supervisor of a manufacturing facility is facing an ethical dilemma. She uses the formal process of ethical decision making to resolve the issue. Her first step will be(((understanding all the moral standards.)))
Anna, a senior executive, has a difficult ethical decision to make regarding one of her new clients. In evaluating her ethical duties, in addition to considering all the financial requirements, she must also(((be proud of seeing this decision or action reported in the newspaper.)))
In the context of the business costs of ethical failures, __________ is incurred at Level 3.(((reputation cost )))
"In the context of the business costs of ethical failures, Level 1 costs that are less damaging and get more executive attention are the costs of
"(((government fines and penalties.)))
As a manager charged with making an ethical decision, moral awareness begins with(((considering whether your decision has unfavorable ramifications.)))
Sienna has always been praised for her moral awareness, judgment, and character at work. However, turning in her own boss, a division manager, for fraudulent behavior will be very difficult for her to do. Sienna knows that she will need to draw upon every bit of __________ that she can muster.(((courage)))
Some organizations have platforms that allow employees to provide anonymous feedback about the work and behavior of colleagues and seniors. The primary purpose of such methods is(((protection of whistleblowers.)))
The Write Stuff, Inc., a manufacturer of designer journals and notebooks, switched to using recycled paper to manufacture its products. It also changed the materials and design of its covers and binding material to eliminate non-recyclable plastics. The changes made by the Write Stuff, Inc. are an example of __________ responsibility.(((corporate social)))
The __________ responsibilities of business are to produce goods and services that society wants at a price that perpetuates the business and satisfies its obligations to investors.(((economic )))
Corporate social responsibility refers to(((fulfilling the obligation toward society assumed by businesses.)))
In the context of corporate social responsibility, __________ responsibilities are to obey local, state, federal, and relevant international laws.(((legal)))
In the context of corporate social responsibility, __________ responsibility involves meeting other societal expectations that are not written as law.(((fiscal)))
Safe Earth Extermination believes in meeting societal expectations whether or not mandated by law. It uses only organic products that are environmentally friendly. Safe Earth Extermination is operating at the __________ level of the pyramid of corporate social responsibility.(((ethical)))
What corporate responsibility includes activities such as supporting local food pantries that society finds desirable and that the values of the business support?(((philanthropic)))
When a construction business regularly gives its employees paid time off to volunteer with Habitat for Humanity, it is an example of its(((philanthropic responsibility.)))
The __________ model is a theory of corporate social responsibility that holds that managers are agents of stockowners whose primary objective is to maximize profits.(((shareholder)))
A major pharmaceutical company donates supplies of epinephrine auto-injectable pens, used to combat severe allergic reactions, to schools in lower-income areas of the state. The donation generates significant publicity for the firm and sales of the product continue at a profitable rate. This donation is part of the firm's __________ responsibility.(((economic )))
A transcendent education teaches students to think beyond self-interest and profitability—in fact, to(((leave a legacy that extends beyond the bottom line.)))
In accordance with the goals of transcendent education, __________ is described as learning how to give as well as take.(((generativity)))
In accordance with the goals of transcendent education, __________ is described as thinking not just in terms of "don'ts" (lie, cheat, steal, kill), but also in terms of positive contributions.(((civil aspiration)))
Although he has not suffered any personal injustices, Lonnie joins a protest at school that seeks to publicize unethical actions taken against nonnative students. Lonnie's action demonstrates which goal of transcendent education?(((intolerance of ineffective humanity)))
Which theorist said, "The social responsibility of business is to increase profits."?(((Milton Friedman)))
An individual who strives to feel his or her decisions as potential victims might feel them is demonstrating(((empathy. )))
The chairman of the board said to the CEO, "We must cease production of those items; they are not healthy for our customers. We must transition to new, safe product lines. I know that other firms are producing those unhealthy items, and I know that no laws prevent us or them from producing them. However, just because something is legal, doesn't mean that it is ethical. The chairman is promoting a(n) __________ ethics program.(((integrity-based)))
"It is perfectly legal for us to accept gifts from our suppliers, as long as we don't get too greedy, and as long as we don't do anything that could get us in trouble with the law," said Charles Jebson, CEO of Starlite Manufacturing. "That's why I have instructed our corporate counsel to design mechanisms to prevent, detect, and punish legal violations." Charles is promoting a(n) __________ ethics program.(((compliance-based)))
The __________ area of responsibility, corresponding to doing what global stakeholders desire, is at the top of the pyramid of global corporate social responsibility and performance.(((philanthropic)))
Which corporate action is likely to be categorized as a philanthropic endeavor?(((making a wide variety of charitable contributions to community organizations)))
A reality of living in a capitalistic society is that(((the creation and distribution of wealth generate by-products that can cause injury, loss, or danger to people and the environment. )))
The goal of __________ is to create sustainable economic development and improve the quality of life worldwide for all organizational stakeholders.(((ecocentric management)))
__________ is economic growth and development that meet an organization's present needs without harming the ability of future generations to meet their needs.(((Sustainable growth)))
__________ is the process of evaluating all inputs and outputs, through the entire "cradle-to-grave" journey of a product, in order to determine the total environmental impact of its production and use.(((Ethical management)))
During a job interview, the recruiting manager from BioCorn Energy told Salvador that ethanol was a more efficient transportation fuel and was, therefore, environmentally superior to petroleum-derived fuel. Afterwards, in doing his own research, Salvador learned that this viewpoint was disputed. Critics of ethanol-from-corn production cited a recent __________ study, contending that ethanol was in fact more environmentally harmful than gasoline, when considering the amount of land required, the amount of chemicals (fertilizer, herbicides, and pesticides) required, the higher transportation costs incurred, and amount of agricultural wastes generated.(((life cycle analysis)))
The modern view of business toward incorporating environmental values is to treat it as(((a way to gain a competitive edge.)))
Dallas, store manager for Newberry Foods, learns that the store's refrigeration system failed overnight, and, for several hours, the cold meat and produce were exposed to temperatures that were a few degrees higher than required by company standards. She decides to discard the food, at cost of many thousands of dollars, because she believes it is the right thing to do, and the health of consumers is at stake. Dallas uses __________ as an ethical system to make her decision.(((universalism)))
Alec has great respect for his long-time manager and mentor, Eli. When Alec is confronted with a challenging ethical issue, he asks himself how Eli would have handled the situation. Alec uses __________ as an ethical system to guide him in his decision making.(((relativism)))
Calvin's bicycle company that specializes in mountain bikes sold a few models with a defective component. If he informs the customers and issues a recall, it will cost him a substantial amount of money. If the bike malfunctions, there is a very small chance that it could cause serious injury to a cyclist who might use it on rough terrain. Calvin decides to use the egoism approach to decision making and remain silent about the defect, likely because he(((is motivated by self-interest. )))
Nathan received a holiday gift from a company supplier. He estimated the value of the gift at $25. As he was not sure he could ethically accept the gift, Nathan reviewed company policy and discussed the matter with his boss before making his decision. In this situation, Nathan used what ethical framework to guide his decision?(((relativism)))
Maya, owner of a small craft shop, inadvertently sold decorative wall hangings that included hazardous lead-based paint. The paint was a very small part of the finished product, and the crafts were intended to hang well above the reach of small children and pets. Nonetheless, when Maya learned that the crafts contained the hazardous paint, she contacted each buyer, took back the crafts she had sold them, and replaced them with products that did not contain the hazardous paint. She acted as a mature businessperson with good moral character by using the __________ approach to ethical decision making.(((virtue ethics)))
"That's a problem that is endangering the public! Our company needs to immediately stop production and solve it!" warned Kevin. His manager said, "Kevin, we're not going to solve any problem that means spending money. Our profits are behind plan this quarter." An unwillingness to take an ethical stand that may impose financial costs is a sign that the company has a(n) __________ that is conducive to unethical behavior.(((ethical climate)))
The codified laws of the country in which a business is located are the evaluative tool guiding business ethics.(((FALSE)))
Unethical managerial behavior helps create a toxic work environment that negatively impacts the entire organization.(((TRUE)))
An ethical leader is one who is both a moral person and a moral manager who influences others to behave ethically.(((TRUE)))
In ethical decision making, moral judgment involves understanding that a problem has ethical implications.(((FALSE)))
The shareholder model of business makes management's economic responsibility to the firm a priority.(((TRUE)))
A company can easily avoid environmental issues because the creation and distribution of wealth generates very few by-products.(((FALSE)))
Life cycle analysis (LCA) quantifies the total use of resources and the releases into the air, water, and land.(((TRUE)))
__________ is/are composed of all relevant forces outside a firm's boundaries, such as competitors, customers, the government, and the economy.(((The external environment)))
Advances in genetic engineering are expected to produce some food products that will become available all year round even in continental climates. These changes will provide grocers with an opportunity to reduce their shipping costs while offering fresher produce to their customers. These advances are examples of changes in the __________ environment.(((technological)))
A clothing importer/exporter in England is worried that the British exit from the EU (Brexit) will result in new tariffs applied to clothing unless a trade agreement with the EU is reached. Which component of the macroenvironment concerns this company?(((economic)))
When considering workforce demographics, hiring managers must remember that the fastest-growing age group will be workers who are(((75 years old and older)))
When Johnson & Johnson created reservable lactation rooms for employees so that mothers could work quietly and nurse privately, it was addressing which element of the macroenvironment?(((social)))
__________ is the immediate environment surrounding a firm, including suppliers, customers, rivals, and the like.(((Competitive environment)))
Netflix, Amazon, and Disney+ have emerged as serious and successful content providers, attracting viewers away from traditional networks such as ABC, CBS, and NBC. The success of these nontraditional providers demonstrates the power in the competitive environment held by new competitors, or new entrants to the marketplace. To effectively compete with these new entrants, executives at traditional networks will take advantage of barriers to entry, such as(((distribution channels.)))
Jocko, a new manager at Super Sandwiches, has been asked to focus on the competitive environment of the organization. Which factor is among those that Jocko should focus on?(((suppliers)))
A popular low-cost airline, Savor Airways, has gone out of business. Although the service and price provided by the airline was what customers wanted, the larger airlines were able to drive the low-cost airline out of business through an aggressive price war. Which component of the competitive environment does this illustrate?(((competitors)))
__________ occurs when weaker companies are eliminated, and the strong companies survive.(((Industry shakeout)))
__________ are conditions that prevent new companies from establishing themselves in an industry.(((Barriers to entry)))
__________ is/are a common barrier to entry.(((Distribution channels )))
Juan Carlo owns a janitorial service company. Over the past five years, the bulk of his business, about 80 percent, has derived from a long-term service contract with the federal government. He learned today that his company will not be awarded an extension of the contract, and thus his company's work for the government client must cease within 30 days. The very negative impact of this decision on Juan Carlo's company is a reminder that(((an organization is at a disadvantage if it depends too heavily upon powerful customers.)))
Hospitals and universities are very expensive to start and keep running because of the capital and equipment required. The personnel requirements, such as specialized medical doctors and researchers, are also high. This demonstrates(((barriers to entry.)))
Carrell Foods is a major supplier to many grocery store chains. It has most of the valuable shelf space in the cereal aisle. What is true based on this information?(((The threat of new entrants is low.)))
The main difference between final consumers and intermediate consumers is that final consumers(((use products themselves.)))
Caralee presented to her boss the possibility of building a new manufacturing plant to serve a new, hot market. Her boss replied, "Caralee, I can't support that. Three multinational firms, who are world-class in terms of global reach, influence, stability, and financial depth, will be competing for that market. Not only would we be a new entrant to the playing field, the barriers to entry are just too high for us. We are small and financial resource-limited, so we would be challenged to meet the market's __________ requirements."(((capital)))
The manager of operations at Beta Industries, has decided to change careers. For years, the company has been a market leader in manufacturing DVD players, but sales—and the manager's advancement opportunities—have been declining for years. He told his wife, "Recent __________ have allowed smartphones and streaming to emerge as substitutes for DVD players. That's reality, and I need to face it."(((barriers to entry)))
Waldo, a regional sales manager, was asked to analyze whether his company should launch a marketing effort to become Fancy Foods' produce supplier this year. He found that Fresh Green Veggies currently has a supply contract with Fancy Foods that has three more years in its term; Fancy Foods would have to pay Fresh Green a half million dollars to break the contract. Also, Fresh Green has installed automated ordering/billing software in Fancy Food's home office; Fancy Foods would have to spend $100,000 to replace it and retrain its staff. He concluded that Fancy Foods' __________ costs would be too high to seriously consider a change in supplier, thereby recommending that a marketing effort not be launched this year.(((switching)))
Jasper Products is the only provider of a patented leakproof valve for industrial applications. Arnold Construction wants to include Jasper Products in a project for Arnold's customer, thereby increasing the chance that Arnold will be selected to perform the project. However, Jasper Products charges a premium price for its valves, eroding Arnold's profitability. This situation is indicative of the high bargaining power of(((suppliers.)))
Associated Bank Corp. utilizes the janitorial services of Clean-It Corp. to keep its buildings clean. Clean-It Corp. provides all the necessary cleaning supplies, training, background security checks, and all other associated expenses. The other available janitorial services do not typically provide security checks, and a client such as a bank would have to hire guards to watch the work of the janitors. If Associated Bank were to consider hiring another cleaning service to replace Clean-It Corp., the bank would most likely incur(((high switching costs.)))
Josiah is the marketing manager in a firm that sells air pollution control systems to the coal-fired power industry. His business is highly competitive, with existing rivalries and new players. Coal-fired power plants are being shut down in favor of substitute forms of energy, e.g., wind and natural gas. Thus, new project opportunities in coal-fired power plants are rare. His clients (electric utility companies) have many choices of providers and technologies. Josiah's firm has long-term pricing agreements with its equipment suppliers, so Josiah knows what external costs his firm will incur on potential new projects. The competitive force that appears to represent the lowest risk to Josiah's firm is(((power of suppliers.)))
Your company has asked you to launch a new product campaign at a trade show. You have found that there are established competitors with many advantages, making it difficult to break into the market. However, you recommend displacing these competitors' business by(((offering promotions and price breaks, intensive selling, and other tactics.)))
"Last month, I opened a résumé-writing business out of my dorm room. And, now, on this floor alone, I find that I have three start-up competitors!" complained Aurora. Her roommate replied, "Well, at least you haven't had to buy any new equipment for your business. But apparently neither have your competitors." This scenario indicates the(((high threat of new entrants when capital requirements are low.)))
Competition is most intense when there are __________ direct competitors, industry growth is slow, and the product or service cannot be differentiated.(((many)))
__________ is/are a major barrier to entry.(((Brand identification)))
A complement is a potential __________ because customers use it as an alternative, buying less of one kind of product but more of another.(((opportunity)))
If buyers change suppliers, they are faced with(((switching costs. )))
A __________ is an intermediate consumer.(((retailer)))
Identify the environmental factor that contributes to an attractive environment for existing firms.(((many barriers to entry )))
Identify the environmental factor that is a characteristic of an attractive environment for a business.(((low customer bargaining power )))
Dennis, a senior manager at VPR Corporation, is considering the acquisition of a revolutionary water treatment technology from a small development firm. The asking price is $5 million. However, the environmental uncertainty of the water treatment marketplace is both complex and dynamic. Dennis wants to acquire the new technology but seeks to manage the risk associated with his decision, or "hedge his bets." Key ways to cope with the uncertainty would be to(((gain competitive intelligence through environmental scanning, develop business scenarios, forecast sales, and benchmark the technology.)))
Sanjay, a long-term and successful sales professional for DiscDrives Inc., has gained a vast degree of competitive intelligence in his 25 years with DiscDrives. He walked into his manager's office and said, "Daniel, for family reasons, I need to move out of state. Therefore, I've accepted a position with a competitor. I'm submitting my two weeks' notice. This isn't personal; you've been a great friend." Daniel replied, "Sanjay, I'm sorry to hear this. I will truly miss you and your contributions. However, your two weeks' notice will not be required. I will walk with you to human resources right now to process your resignation, and then we will escort you to the door." Daniel's rather terse response was intended to quickly remove Sanjay from DiscDrives' premises. Why?(((Daniel wants to ensure that a soon-to-be competitor will not have ongoing access to DiscDrives' competitive intelligence and staff through Sanjay.)))
A division manager for Rinte Corporation told his subordinate, "Samantha, because this is your first year of budget planning for your department, I need you to __________, defining a best-case, a worst-case, and a plausible-case business, or middle-ground business, outlook. The plausible-case business outlook will be what you will use for budget planning."(((produce different scenarios)))
Because his firm's product market share has declined in the past 12 months, the sales manager for AtoZ Group instructed his marketing professional to perform a benchmarking analysis. What would you instruct the marketing professional do for this analysis?(((identify competitors' best processes and compare with RLC's processes)))
__________ refers to searching for information that is not immediately evident and sorting through that information to interpret what is important.(((Environmental scanning)))
In an attempt to improve customer service, Terrific Toys decided to assign a team to investigate the kinds of services offered by competing companies. The team discovered that a smaller company, GV Games Corp., seemed to have outstanding customer service. The team then determined the major differences between the two companies and developed a plan to incorporate the best elements of GV Games into Terrific Toys. What tool is being implemented by Terrific Toys?(((benchmarking)))
__________ is the number of issues to which a manager must attend and the degree to which they are interconnected.(((Complexity)))
__________ is the degree of discontinuous change that occurs within the industry.(((Dynamism)))
What method is used to predict exactly how some variable or variables will change in the future?(((forecasting)))
__________ includes searching for and sorting through information about the environment.(((Environmental scanning )))
Forecasts are most useful when the __________ will look radically different from the __________.(((future; past)))
Antonia Sorez is the CEO of a nationwide computer purchasing and repair chain. Observing that the number of computer technology stores in the Northeast is so great that prices and margins have been driven downward, she has decided not to open any more outlets in that region. In the context of the competitive environment, this fact is an indication of the(((high bargaining power of customers.)))
__________ is the information necessary to decide how to best manage the competitive environment.(((Competitive intelligence)))
You have been asked to report on the competitive environment in the restaurant industry; therefore, you develop the __________ necessary to decide how to manage in the given environment.(((competitive intelligence)))
Cho, a financial analyst at a small food-processing company, was asked by the CFO to forecast the costs of the company's required raw materials over the next five years. You would advise her to collect data carefully,(((use simple forecast models, and make multiple forecasts to get an average. )))
__________ is the process of sharing authority with employees to enhance their confidence in their ability to perform their jobs and contribute to an organization.(((Empowerment )))
In __________, supplies of excess resources are created in case of unpredictable needs.(((buffering)))
Suri owns a small gift shop in Chicago. She hopes the economy will rebound this fall as she has to order her Christmas merchandise and other holiday gifts in February. She wants to make sure she has enough goods for the surge in sales that the holiday season usually brings to her retail store, so she orders more than the previous year. Here, Suri is using __________ to be sure she has enough gifts for those extra customers that she hopes will bear the economic woes and shop for holiday presents.(((buffering)))
The use of contingent workers in the U.S. labor force due to unpredictable labor demand is an illustration of(((buffering)))
When America's Fruit Juice Company acts on its own to promote the fruit juice industry as a whole, the company is practicing(((competitive pacification.)))
When Vermont Syrups advertises its syrup products as better and healthier than the syrup products of Maple Goodness, Vermont Syrups is demonstrating(((competitive aggression. )))
__________ is leveling normal fluctuations of demand at the boundaries of an environment, such as when clothing stores put their coats on sale in late winter.(((Smoothing)))
__________ are methods for adapting the technical core of a company's products to changes in an environment.(((Flexible processes)))
__________ are approaches used by an organization that acts on its own to influence stakeholders or change some aspect of its current environment.(((Independent strategies)))
A university is struggling financially, and has just formed a board of trustees and invited 25 of its wealthiest alumni to join the board. What process is being adopted by the university?(((cooptation)))
Todd Hospital Systems and Gary Medicines Corp. have joined forces to fight for health care reform. The two companies(((are creating a coalition.)))
Domain selection, diversification, mergers or acquisitions, and divestitures are all examples of(((strategic maneuvering.)))
Country Farms Corp. announced that it was selling off its dairy division in order to realign itself more competitively in the marketing of its other products. This is an example of managing the task environment through(((divestiture.)))
Antonio owns a local men's clothing store. In the face of increasing competition, Antonio has tried several tactics: aggressively promoting price-slashing sales to drive his competitors' customers to his doors; attempting to cut costs by leveling out sales and inventory through seasonal sales; and lining up contracts with wholesalers in advance of seasonal rushes (e.g., summer swimwear) to prevent inventory depletion. None of these tactics has been successful. Now, Antonio is considering a deviation from his current business to one that might be more suitable, perhaps a formal wear/tuxedo rental and retail shop or a boutique Western wear store. Swen is using __________ as his final tactic to save his business.(((domain selection)))
__________ is an organization's conscious efforts to change the boundaries of its competitive environment.(((Strategic maneuvering)))
__________ occurs when a company sells one or more businesses.(((Divestiture)))
__________ are companies that stay within a limited, stable product domain as a strategic maneuver.(((Defenders)))
When sales of its personal computers started to level off, Apple Computer added music players and cellular phones to its product line. In terms of strategic maneuvering, what type of company is Apple?(((Prospector)))
__________ is independent action to improve relations with competitors.(((Competitive pacification)))
__________ refers to absorbing new elements into an organization's leadership structure to avert threats to its stability or existence.(((Cooptation)))
Barley owns and operates a food truck in a large city. Competitors are many, and competition is fierce. Due to her small business's size, she cannot realistically influence her business environment. However, Barley could take steps to adapt to the uncertain environment by buffering, including(((hiring part-time help and maintaining extra inventory for peak periods.)))
A large national restaurant chain was moving into a new community and looking for ways to promote itself. It decided to sponsor a local Little League team, give out free samples during the games, and purchase the team's uniforms with the company logo on them. Which action is the company using to influence its environment?(((public relations)))
__________ is leveling normal fluctuations at the boundaries of the environment.(((Smoothing)))
During the holiday season, Dora's Department Store works with a contracted employment agency to bring extra workers on board to handle overflow business, and extra duties such as wrapping presents. Dora's is using __________ during these rush times.(((buffering)))
A company uses __________ when it acts on its own to change some aspect of its current environment.(((independent strategies)))
For many years, Mars, Incorporated manufactured M&M's candies in assorted but fixed colors. M&M's were known and branded worldwide as individual candies featuring single solid colors. Then, in response to technological advancements and evolving customer tastes and expectations, Mars adapted its technical core by producing M&M's in an infinite variety of configurations and colors, even offering M&M's with personal photographs imprinted upon them as a product of "mass customization." Mars accomplished this by implementing(((flexible processes.)))
A company is experiencing an increase in competition, and at the same time it is building more production facilities in Southeast Asia. In this scenario, the top management team is most likely to(((give lower-level managers the authority to make decisions to benefit the firm. )))
__________ is described as exploiting a distinctive competence or improving efficiency for competitive advantage.(((Competitive aggression)))
A company uses __________ when it acts on its own to change some aspect of its current environment.(((independent action)))
__________ are the strongly held and taken-for-granted beliefs that guide behavior in a firm.(((Unconscious assumptions)))
An organization's __________ refers to all relevant forces inside a firm's boundaries, such as its managers, employees, resources, and organizational culture.(((internal environment)))
The five-by-eight inch card with one rule on it—Use good judgment in all situations—that employees at Omniplex Corporation receive along with the employee handbook tells the employees a great deal about their company's(((organization culture.)))
__________ are the components of an organization that can be seen and heard, such as office layout, dress, orientation, stories, and written material.(((Visible artifacts)))
__________ are the first level of organizational culture.(((Visible artifacts)))
__________ are the underlying qualities and desirable behaviors that are important to an organization.(((Values)))
At the third and deepest level of organizational culture are(((unconscious assumptions.)))
A useful clue about an organization's culture is(((found by seeing who is hired and who is fired. )))
The leadership focus on __________ is/are associated with a hierarchical culture.(((control, formality, and stability)))
Goeff and Trad successfully partnered as a small independent oil exploration firm for 30 years. They worked informally together like family, and they were intensely loyal toward each other, their customers, and their suppliers. However, later in their careers, Goeff and Trad sold their firm and were hired by MegaOil, Inc., a top 5 global energy conglomerate.. They quit after 3 months because they could not adapt to MegaOil's culture, which is characterized by production/achievement, results orientation, competitive advantage, and market superiority (market/compete culture). In hindsight, Goeff and Tad probably realized that they missed the(((tradition, trust, teamwork, cohesiveness, and sense of family of a clan culture.)))
Which organizational culture is strongly characterized by being internally oriented and flexible, and emphasizing trust, commitment, and tradition?(((clan)))
Which organizational culture's objectives are productivity, planning, and efficiency?(((market)))
A hierarchical organizational structure values __________ and assumes that individuals will comply with the organizational mandates when roles are stated formally and enforced through rules and procedures.(((stability)))
The key attributes of a(n) __________ are flexibility, risk taking, and entrepreneurial.(((adhocracy)))
The founders of Rainy Corporate pride themselves on their commitment to a stewardship of the environment, displaying a photo montage in the lobby that displays before-and-after pictures of the site where the headquarters complex is built—from an ugly landfill into a parkland. As a reflection of the company's culture, the photo montage represents a(((visible artifact.)))
The Chief Operating Office at Hank Corporation, effectively sustains and manages the company culture by(((crafting a vision, "walking the talk," and celebrating employees who exemplify the culture.)))
Jean Paul's French Foods founded in 1963 has grown steadily to 250 employees. Despite its size, Jean Paul's operates as a family business with a strong clan culture. Jean Paul's CEO, John Smythe, has just learned that the company's long-standing and best customer, Saul's Bistro, with 15 locations in the region, is filing for bankruptcy. As a result, Jean Paul's will immediately experience a 15 percent reduction in sales. Based upon this description, what do you advise John to do?(((Ask members of the entire company to agree to cut their hours by 15 percent until business improves, avoiding any layoffs.)))
Open systems are all relevant forces outside an organization's boundaries.(((FALSE)))
The government has very little immediate effect on business opportunities in the U.S.(((FALSE)))
When an industry matures, profits drop. Weaker companies are eliminated, and the strong companies survive.(((TRUE)))
Established companies are less threatened by new competitors when there are many barriers to entry.(((TRUE)))
Environmental uncertainty means that managers need to implement controls to be sure they aren't harming the physical environment.(((FALSE)))
Benchmarking is the process comparing an organization's practices and technologies with those of other companies.(((TRUE)))
Divestiture is entering a new market or industry with existing expertise.(((FALSE)))
A small company that makes face masks for health care workers has decided to donate hundreds of masks to nursing homes in its area. This demonstrates the independent strategy of influencing stakeholders known as competitive pacification.(((FALSE)))
When Maria started her telemarketing business, she organized the space with individual cubicles for privacy; provided ergonomic chairs for all employees; and encouraged a casual, comfortable dress code. Maria was focusing on the visible artifacts of the office.(((TRUE)))
One recommended way to understand a company's culture is to listen to the stories people tell about the company.(((TRUE)))
Management is the(((process of working with people and resources to accomplish organizational goals.)))
Planning involves(((analyzing current situations.)))
__________ is specifying the goals to be achieved and deciding in advance the appropriate actions needed to achieve those goals.(((Planning)))
WorldMart Inc. periodically reviews the goals of the company. During the process, the managers of the company analyze their current strategies as compared to their competitors' strategies, determine goals that they will pursue, and decide upon specific actions for each area of the company to take in pursuit of these goals. With these actions, the managers are engaged in the management function of(((planning.)))
A large company recently embarked on an effort to increase coordination and cooperation within the company. During the process, the managers of the organization reviewed and specified job responsibilities, grouped jobs into work units, and reallocated resources within the company. The managers were exercising the management function of(((organizing)))
__________ is assembling and coordinating the human, financial, physical, informational, and other resources needed to achieve goals.(((organizing)))
Brian is a general manager for a tool-manufacturing firm. He is considering some changes to the production floor, which include layout adjustments and the purchase of new equipment to improve efficiency. He also wants to promote one of his employees to team leader. Which function of management is Brian performing?(((organizing)))
Building a dynamic organization is the goal of which function of management?(((organizing)))
Mike focuses on assembling and coordinating the people, equipment, and supplies that his company needs to achieve its goals. What management function does Mike emphasize in his work?(((organizing)))
What will be the result when managers use new forms of organizing?(((They will build organizations that are flexible and adaptive. )))
Natasha is very good at inspiring the people in her department to learn new skills and to perform better than expected on the job. As a manager, what is Natasha especially good at doing?(((leading)))
As one of the key management functions, leading focuses on a manager's efforts to(((mobilize people to contribute their ideas.)))
__________ is a manager's ability to stimulate people to be high performers.(((Leading)))
Yolanta manages a team of six professionals in a health care company. Her subordinates have a wide variety of experiences and skill sets. By effectively __________ her team, Yolanta regularly seeks to inspire her subordinates to draw upon their various backgrounds in new and innovative ways.(((leading)))
When Kabir, the marketing manager of SideStreet Inc., realized that his plan to increase sales levels was not producing the results he desired, he took quick action to make necessary adjustments. According to this scenario, Kabir was exercising the management function of(((controlling)))
__________ involves monitoring performance and making necessary changes.(((Controlling)))
When Martin, manager of his company's sales department, realized that his plan to increase his department's sales levels was not producing the desired results, he instituted a training course to help the salespeople achieve better results. Which management function is illustrated in this scenario?(((controlling)))
Through careful monitoring of the financial budget of a firm, managers can detect potential problems in reaching their financial goals and take actions to reverse the problem. This is an example of the management function of(((controlling)))
When PG&E created a wildfire safety plan after devastating fires in California, it was demonstrating the management function of(((controlling)))
Sanga is a middle-level manager who is known for his efficiency; however, he is not necessarily effective. But, Rachelle, another middle-level manager, is known for her effectiveness, rather than her efficiency. Therefore, Rachelle is more likely to(((achieve organizational goals.)))
Fashion Forward wants to be the sales leader in women's fashion. To achieve this goal, the CEO has analyzed the current situation and determined objectives and resources. What planning activities still need to be completed?(((anticipate the future, decide on activities, and choose a business strategy)))
__________ is coordinating the human, financial, physical, informational, and other resources in a company.(((organizing)))
"Leading involves
"(((stimulating people to be high performers.)))
Controlling helps a manager to(((monitor performance and implement necessary changes.)))
To become an effective manager, one should choose __________ of the four management functions and commit one hundred percent.(((all)))
Which statement best describes the management function of controlling?(((Managers make sure the organization's resources are being used as planned and that the organization is meeting its goals for quality and safety.)))
Leading involves(((mobilizing and inspiring people to engage fully in their work and contribute their ideas.)))
The CEO said, "Every six months or so, my senior management team and I meet to discuss the goals that will be achieved over the next year, three years, and beyond. We then make sure we are clear on who will take responsibility to see that appropriate actions are undertaken to achieve our goals within the time frame we set." The CEO is describing the management function of(((planning.)))
"Recently, I spent a great deal of time looking at how to define jobs to most efficiently utilize the employees in those jobs," said the CEO of Baker Products. "And now I need managers to be responsible for the various job groupings." The CEO is describing the management function of(((organizing.)))
"As CEO, I maintain a big-picture view of how we are performing as a company, determining what changes we need to make if we begin deviating from our plans, and ensuring we meet our goals for quality and safety." The CEO is describing the management function of(((controlling.)))
"As CEO, I must mobilize, inspire, and stimulate my management team, as well as the entire staff, to continually perform at high levels. I seek to empower my staff through communication and motivation." The CEO is describing the management function of(((leading.)))
Labyrinth is a new, very successful, brand of organic clothing recently introduced globally. The company is unique in the way it has collaborated with its suppliers and potential customers to bring the clothing to market. Labyrinth's founders have always focused on efficiency. What is the best advice you should give them, given the quick success they have had with Labyrinth?(((To continue their success, they should maintain a clear focus on both effectiveness and efficiency, especially while their company is experiencing such a high growth in sales.)))
__________ managers are senior executives responsible for the overall management and effectiveness of an organization.(((Strategic)))
A company's CEO and Director of Human Resources are concerned that their company will not survive the current recession. As experienced __________ managers, they are working collaboratively to develop new business strategies and work processes to effectively and efficiently support the company so that it can survive the current economic downturn and thrive in the coming years. Their employees are depending upon them to provide overall management of the company.(((top-level)))
The four levels of managers found in large organizations are(((top-level, middle-level, frontline, and team leader.)))
"Top-level managers focus on
"(((the long-term survival of an organization.)))
"__________ managers are typically concerned with the interaction between an organization and its external environment. ((top level))
"(((Top-level)))
Chief executive officer, president, chief operating officer, and vice president are all titles typical of __________ management.(((top-level)))
__________ managers are responsible for translating the general goals and plans developed for an organization into more specific activities.(((Tactical)))
Middle-level managers are often referred to as __________ managers.(((tactical)))
As a manager, Jason spends much of his time coaching his employees and making sure that any required information from upper management reaches them in an understandable format. Jason would best be described as a(n)(((tactical manager.)))
__________ managers are lower-level managers who supervise the operational activities of an organization.(((Frontline)))
Operational managers play a crucial role in an organization because they(((provide the link between management and nonmanagement personnel.)))
One of Yani's primary activities in his career as a supervisor at Front yard Bird Food is working with his middle manager, Delilah, to introduce new growth opportunities in the business (such as expanding into exotic bird foods) and to help the people who actually manufacture the food. Yani is most likely a(n)(((frontline manager.)))
Titles such as sales manager or supervisor typically belong to __________ managers.(((operational)))
In smaller entrepreneurial firms and even in more adaptive larger firms, managers(((have strategic, tactical, and operational responsibilities.)))
Being a(n) __________ is one of the roles of a manager that involves searching for new business opportunities and initiating new projects to create change.(((entrepreneur)))
In a major announcement at an annual medical conference, Dr. Harold Weinter, Research Director of Assop Pharmaceuticals, informs the medical community of a breakthrough in the treatment of diabetes. As __________ for his organization, he answers questions posed to him by his medical research colleagues and members of the press.(((spokesperson)))
Audrey is the lead attorney representing RockWork, Inc. and she is in back-and-forth discussions with parties, inside as well as outside the firm, to finalize a contract with Murphy Realty. Audrey knows that, in her role as __________, she is making important business decisions for RockWork.(((negotiator)))
Jeffrey's role as __________ entails creating a presentation to give to his management team that will communicate the company's new business objectives, and more importantly, how Jeffrey has interpreted them to apply to his business unit.(((disseminator)))
The president of Key Pharma, acting as the corporation's __________, attended the opening of a customer's new office complex.(((figurehead)))
When a customer service manager works to defuse a situation with an irate customer, he or she is assuming a __________ role.(((disturbance handler)))
"A project manager determines the number of employees to be assigned to a certain project. The managerial role being implemented by this project manager is
"(((resource allocator.)))
Maintaining a network of outside contacts and alliances that provide information and favors defines a __________ managerial role.(((liaison)))
The need for interpersonal and communication skills(((is important at every level of management.)))
A __________ serves as the spokesperson and champion for a work group dealing with external stakeholders.(((team leader)))
"I'm sorry to tell you that four employees from our division were laid off today," said Bill Harrington, division manager, to his assembled staff. "And a total of 25 employees were laid off corporate-wide. No further staff reductions are planned, and we expect the corporation's financial performance to remain on plan." When the manager notifies his employees of these developments and plans, he is performing in the role of(((spokesperson.)))
"Quick, Mr. Delaney, Margaret is in the lobby, shouting at a customer!" said George. Rushing to intervene, Paul Delaney demanded that Margaret return to her office immediately. "Mr. Crenshaw, I'm very sorry. Please, allow me to escort you to our executive suite, and I will straighten this out," said Mr. Delaney. Mr. Delaney was performing in the role of(((disturbance handler.)))
"I must attend the fundraiser," said Maura O'Brien, Senior Vice President of Marketing. "One of our key clients is sponsoring the event for a worthy cause, and our firm should be represented by a senior member of our management team." When Maura performs symbolic duties on behalf of her organization, she is performing in the role of(((figurehead.)))
Julia mused to herself, "It has taken over 30 years, but I've finally been named CEO of a major firm. Now, I can concentrate on what I really want to do as a(n) __________ manager, to focus on the survival, growth, and overall effectiveness of our firm."(((strategic)))
"A(n) __________ skill is the ability to perform a specialized task that involves a certain method or process. ((technical))
"(((technical)))
__________ represent the three general categories of skills that are crucial to managers.(((Technical, interpersonal and communication, and conceptual and decision)))
Using a particular software program at an expert level, compilation of an accounting statement, and writing advertising copy are all examples of __________ skills.(((technical )))
Rashid started out as a front office worker, and over the years worked his way up to the CEO position. As such, Rashid likely found that as he moved up in responsibility, he required a higher level of __________ skills.(((conceptual and decision )))
The senior managers at a printing company recognized a lack of employee enthusiasm about the new website. There was a fair amount of infighting and accusations of who was responsible for the layout, given that the firm is considered to be highly competent in design and production. Marley, a manager, got the team together and engaged them in a lively discussion to determine how to change the website so that it reflected more of the company's unique design ability. As the meeting wound down, each team member volunteered to take on a part of the project to fix the site. The ability to identify this problem and resolve it is an effective use of __________ skills.(((conceptual and decision )))
Listening to employee suggestions, gaining support for organizational objectives, and fostering an atmosphere of teamwork are all considered __________ skills.(((interpersonal and communication)))
Which statement exemplifies the importance of managerial skills?(((Interpersonal skills are important throughout a manager's career, at every level of management.)))
__________ is the skills of understanding oneself, managing oneself, and dealing effectively with others.(((Emotional intelligence)))
Pete has built up a wide network of contacts, clients, and local business neighbors since moving to Chicago five years ago. Throughout his home-based marketing career, he has regularly and frequently networked with these business associates online, by phone, and in person. As he considers leaving his marketing career to become an owner/manager of a local business, he will rely upon the __________ that he has developed with these individuals to ensure a successful transition to a new career.(((social capital)))
One component of emotional intelligence is(((making good decisions. )))
__________ is among the necessary skills of emotional intelligence.(((Showing empathy)))
Which person illustrates a manager with emotional intelligence?(((Stephen can exercise self-control.)))
"A new boss from outside the industry won't be able to train us to perform specialized tasks involving our particular methods and processes, or to evaluate how well we were performing these tasks. In other words, would she have the necessary __________?" asked Muffy.(((technical skills)))
__________ is one of the key elements that makes the current business landscape different from those of the past.(((Technological change)))
Which statement about globalization is true?(((It occurs through cross-border partnerships.)))
Which statement about the Internet is true?(((The Internet is a virtual marketplace and speeds up globalization.)))
__________ is the goodwill stemming from social relationships.(((Social capital)))
__________ is the set of practices aimed at discovering and harnessing an organization's intellectual resources, fully utilizing the intellects of the organization's people.(((Knowledge management )))
Hannah excels at identifying the talents of employees and finding the jobs where they can best use those talents to benefit the organization. Hannah excels at __________ management.(((knowledge)))
Which statement about collaboration is true?(((Collaboration supports knowledge management and vice versa.)))
Technology both complicates things and creates new opportunities. The challenges come from(((the rapid rate at which technology changes. )))
As the success of modern business so often depends on the knowledge used for innovation and the delivery of services, organizations need to manage that(((knowledge.)))
Collaboration can occur(((with customers.)))
Creating outstanding products and services can start with involving __________ in company decisions.(((customers )))
Harry is surprised one day to learn that the parts that he has ordered from a small local supplier are delayed. He immediately calls his supplier, and the supplier admits that his shop is overbooked with orders, not only from Harry's business but also from new customers in China. As Harry realizes that his small U.S.-based business is competing for shop space with companies in China, he faces the management challenge involved with(((globalization. )))
__________ is the introduction of new goods and services.(((Innovation)))
Service refers to the(((speed and dependability with which an organization delivers what customers want.)))
Which statement about quality is true?(((Quality can be measured in terms of product performance, customer service, and reliability.)))
Lucy has a sandwich shop in Seattle. One thing that attracts busy people to her shop is that she has perfected a way to make sandwiches in half the time her competitors take. Which fundamental driver of success has Lucy emphasized?(((speed)))
Cost competitiveness means that one(((prices his or her products or services at a level attractive to consumers.)))
"Tyrone, manager at Engineering Systems, Inc. (ESI), is struggling to maintain a competitive advantage in ESI's marketplace. He wants to hire and retain the best staff that he can, but the best staff command high salaries. Tyrone knows that high staff salaries can erode his company's profitability. But passing along these higher costs to his clients means they will start doing less business with higher-priced ESI and more business with Tyrone's lower-priced competitors. Tyrone is struggling to
"(((be cost competitive.)))
Which statement about the sources of competitive advantage is true?(((The best managers and companies deliver all six competitive advantages.)))
Amazon is not necessarily known for its development of new products. However, Amazon has transformed the way that products are purchased and delivered. Thus, Amazon has leapfrogged its competition through its(((innovation.)))
Because products __________, a firm must adapt to new competitors and to consumer demands.(((do not sell forever)))
Stacy manages a specialized environmental consulting practice. Client feedback obtained in the past 12 months indicates that her business is on the decline. Stacy decides to implement various changes within her practice to improve service quality. One of the key elements of these changes will be to refocus her staff on(((making it easy and enjoyable for clients to experience a service. )))
Cost competitiveness means pricing a firm's products(((low but enough to make a profit.)))
Latosha's department has been introducing new goods and services on a rapid-fire basis over the past two years. However, business is down, largely because customer feedback on the usefulness, reliability, and durability of the new goods and services is negative. Latosha's division manager, Viceroy, reviews the situation and concludes that the innovation of Latosha's department is good; however, __________ is poor.(((quality)))
Jonas Wilkins, Vice President of Production, said, "Alice, my employees have been manufacturing the same products for two years without a new product being introduced. Products don't sell forever, especially since globalization and technological advances have accelerated the pace of change. I'm concerned that, without __________, we will wither and die as an organization."(((innovation)))
Safety Standards is updating its automated contracting/ordering system. Max Truesdale, CEO of Defense Supply Corporation, told his chief information officer, "Travis, you must lead an effort to reengineer our computer infrastructure to remain compatible with that of Safety Standards. This will be a complex effort but, if we can get this done before our competitors can, we will pick up a lot of new business worldwide. That's the nature of __________, it complicates things and creates opportunities."(((technological change)))
The unique ideas and products that Leverage Group has patented provide a significant advantage to its business because none of its competitors are able to offer them. Because Leverage's CEO recognizes the __________, he has authorized development of practices to discover and harness his organization's intellectual resources.(((importance of knowledge management)))
During an intense six-month contract negotiation, Carlisle developed a strong working relationship with his client, Bart, an outside vendor. Because of their professional bonding, Carlisle and Bart were able to openly explore and constructively hammer out agreements on very tough contract issues. The resulting contract was far superior to what either of them might have hoped for six months earlier and was a good example of(((collaboration across organizational boundaries.)))
Mary Smith, Vice President of Human Resources, is finding that competition for human talent is becoming fiercer. She has implemented creative solutions to leverage diversity of the labor force to the benefit of her organization. For example, Mary has found that __________ allows her to supplement her available staff, particularly during times of work overload.(((hiring older workers on a part-time basis)))
The best managers prioritize efficiency over effectiveness.(((FALSE)))
The four fundamental management principles include analyzing current situations, determining objectives, choosing business strategies, and determining the resources needed to achieve the organization's goals.(((FALSE)))
Maura has weekly meetings with her managers to discuss their work and to share ideas for improvement. Maura exhibits the leading function of management.(((TURE)))
Successful managers do not need to utilize all four functions of management, just the ones relevant to their business.(((FALSE)))
Top-level managers are also called tactical managers because they must translate general goals into specific objectives and activities.(((FALSE)))
A team leader's job is less challenging than frontline and other types of managers' jobs because team leaders always have direct control over team members.(((FALSE)))
Lower-level managers who possess technical skills tend to earn less credibility from their subordinates than comparable managers without technical know-how.(((FALSE)))
One of the five key elements that make the current business environment different from those of the past involves an increasingly diverse workforce.(((TURE)))
Social capital is the goodwill you gain from your social relationships.(((TURE)))
Creativity is defined as the introduction of new goods and services.(((FALSE)))
Information conveyed by a conventional organization chart includes the ((( work performed by each unit in the organization. )))
In organizing, coordination refers to ((( the procedures that link the various parts of an organization to achieve the organization's overall mission. )))
Harvard professors Lawrence and Lorsch found that organizations in complex, dynamic environments ((( had to develop high differentiation in order to succeed. )))
__________ is the assignment of different tasks to different people or groups in an organization. ((( Division of labor )))
__________ is the process in which different individuals and units perform different tasks. ((( Specialization )))
In a(n) __________ organization, managers encourage employees to work more as teammates than as subordinates who take orders from the boss. ((( organic )))
What is a characteristic of a mechanistic organization? ((( obedience to authority as an expression of commitment )))
"The organization chart depicts the
" ((( positions in the firm and the way they are arranged. )))
The more highly differentiated a firm is, ((( the greater the need for integration among its units. )))
In the vertical structure of a firm, authority is the ((( legitimate right of a manager to make decisions and to tell other people what to do. )))
__________ is a structure authority directly related to the three broad levels of the organizational pyramid. ((( Hierarchy )))
In the context of a firm's vertical structure, a powerful trend for U.S. businesses over the past few decades has been to ((( reduce the number of hierarchical layers. )))
In a vertical structure, narrow spans of control build a(n) __________ organization with many reporting levels. ((( tall )))
In the context of the vertical structure of a firm, a wide span of control builds a __________ organization with few reporting levels. ((( flat )))
The span of control should be wide when ((( jobs are similar, and performance measures are comparable. )))
In __________, new or additional responsibilities to a subordinate are assigned. ((( delegation )))
An employee who has __________ is supposed to carry out specified tasks. ((( responsibility )))
__________ is the expectation that employees will perform a job, take corrective action when necessary, and report upward on the status and quality of their performance. ((( Accountability )))
Which statement about delegation of responsibility and authority is correct? ((( It can occur between any two individuals in any type of structure with regard to any task. )))
In order for delegated tasks to be done, a manager must also ensure the individual can draw on the necessary resources and make decisions to complete the work. In other words, the manager also delegates ((( authority. )))
Marcus, a human resource intern, was given an assignment by his manager that meant he must obtain information from other departments to complete it, but some departments refused to share information with an intern. Marcus was frustrated because he did not have the power to get the information he needed. In the context of this situation, Marcus' manager made the mistake of ((( assigning responsibility without delegating adequate authority. )))
In the context of delegation, a common problem faced by a subordinate in carrying out a delegated task is ((( being given more responsibility than authority. )))
An advantage of delegation is that it ((( conserves managers' most valuable asset—their time. )))
The first step in the delegation process, defining the goal, requires a manager to ((( clearly understand the outcome he or she wants. )))
Which statement is accurate about effective delegation? ((( Throughout the delegation process, the manager and the subordinate must work together. )))
In a(n) __________ organization, important decisions usually are made at the top. ((( centralized )))
Janelle recently accepted a job with a reputed bank as an internal auditor. Her new company had strictly defined job responsibilities and lines of communication. For every decision she needed to make, approval had to be obtained from upper management. Overall, she found the atmosphere to be quite formal. Janelle's organization has a ((( high degree of centralization. )))
In __________ organizations, more decisions are made at lower levels of management. ((( decentralized )))
During tough times, senior managers often take charge within the organization. During times of rapid growth, decisions are often ((( pushed to lower levels. )))
In a centralized organization, __________ make all the important decisions. ((( top managers )))
Jonas does not want a management position. He is highly respected by clients, competitors, and coworkers, due to his expertise, experience, and personal qualities. When a major business decision is required, even the division manager above Jonas in the hierarchy asks his opinion. Even though Jonas has no formal management authority, he has __________ authority. ((( informal )))
The __________, led by the chairperson, makes major decisions, subject to corporate charter and bylaw provisions. ((( board of directors )))
After a different vice president is assigned to head the division for the second time in five years, Maria's job as a middle manager does not really change. She carries out the same tasks, attends the same meetings, and sends in the same reports to the new vice president as she always does. What can be accurately determined about this organization? ((( Authority resides in the position, not the people. )))
Howard wanted to delegate an important task to a subordinate, and he had several candidates in mind. He decided to delegate the task to Arjun, who is not only capable but who also offers the advantage of ((( benefiting from developing skills through assuming additional responsibility. )))
In decentralized organizations, decision making occurs at the level of the people who are most directly __________ and have the most __________ about the problem. ((( affected; intimate knowledge )))
In the context of horizontal structure of a firm, __________ are those that have responsibility for the principal activities of the firm. ((( line departments )))
In a horizontal structure, __________ deal directly with the organization's primary goods or services; they make things, sell things, or provide customer service. ((( line departments )))
Subdividing an organization into smaller units in a horizontal structure is ((( departmentalization. )))
In a __________ organization, jobs (and departments) are specialized and grouped as production, marketing, human resources, research and development, finance, accounting, and so forth based on the skills they require. ((( functional )))
An advantage of functional departmentalization is that ((( economies of scale can be realized. )))
Exceptional Performance, a worldwide leader in athletic apparel products, operates divisions in North America, South America, Europe, and Asia-Pacific. Each division is managed independently and has its own line and staff departments to better align with ((( geographies. )))
A __________ organization has departmentalization that groups units around products, customers, or geographic regions. ((( divisional )))
To be effective, the __________ approach to departmentalization requires proper management of all the issues surrounding decentralization and delegation. ((( product )))
Global Gold, Inc. manufactures and distributes athletic wear, sporting equipment, and protective sports gear worldwide. They are departmentalized based on products sold to professional athletes, products sold to people who pursue sports only for leisure, and products sold to sports teams. Which divisional approach does the company follow? ((( customer )))
Which statement is accurate about a matrix organization? ((( It is composed of dual reporting relationships in which some employees report to two superiors. )))
A characteristic feature of matrix organizations is that ((( more career options become available. )))
A disadvantage of a matrix organization is that ((( reporting to two superiors can cause confusion. )))
A __________ is composed of temporary arrangements among members that can be assembled and reassembled to meet a changing competitive environment. ((( modular network )))
The CEO of Virtual Solutions decides to reorganize her firm into a(n) __________ organization. In this approach, Virtual Solutions will collaborate with other independent, mostly single-function firms to provide a desired service. ((( network )))
Greenfield Farm Equipment operates worldwide and is organized into a(n) __________ structure. Within this structure, each of the four division presidents and their respective employees are committed to the tractor, planter, combine, and silo businesses, respectively. Within these four divisions, task responsibilities are clear, and managers are more independent and accountable. ((( product )))
The unity-of-command principle states that a person should have ((( only one boss. )))
In the __________ approach to organization structure, decision making is decentralized to a level where information is processed properly and relevant knowledge is applied. ((( matrix )))
A(n) __________ is a person who assembles and coordinates participants in a network. ((( broker )))
In the horizontal structure of a firm, line departments ((( make things, sell things, or provide customer service. )))
A disadvantage of the traditional functional approach to departmentalization is that ((( communication between functions may fall off due to conflicts. )))
Which statement is accurate about a network organization? ((( In a network organization, managers within the organization interact closely with members outside it. )))
In organizational integration, __________ means establishing common routines and operating procedures that apply uniformly to everyone. ((( standardization )))
__________ refers to a situation when interdependent units create deadlines and objectives that contribute to shared goals. ((( Coordination by plan )))
The approach to __________ involves feedback and discussions to jointly figure out how to approach problems and devise solutions agreeable to everyone. ((( coordination by mutual adjustment )))
Managers can cope with high uncertainty and heavy information demands by reducing the need for information or ((( increasing information-processing capability. )))
Galaxy Corporation and Adventure Inc. often compete with one another in the marketplace. A new business area is emerging that appeals to both firms. However, organizations that will successfully compete in the new business area will need to have a robust laboratory capability as well as an established construction capability. Galaxy has the former, and Adventure has the latter. Although unusual for two competitors, they negotiate and create a(n) __________. Under the terms of this agreement, the two firms formally and jointly pursue their mutual goals. ((( strategic alliance )))
A __________ is an organization skilled at creating, acquiring, and transferring knowledge as well as at modifying its behavior to reflect new knowledge and insights. ((( learning organization )))
In the context of organizational agility, learning organizations are skilled at ((( experimenting with new approaches. )))
"A __________ organization is one in which top management ensures that there is consensus about the direction in which the business is heading. ((high-involvement))
" ((( high-involvement )))
A characteristic of a high-involvement organization is that ((( the organization has a flat, decentralized structure built around a customer, good, or service. )))
__________ is a way of managing in which everyone is committed to continuous improvement of his or her part of the operation. ((( Total quality management )))
One of W. Edwards Deming's "14 points" of quality is to ((( allow autonomy and spontaneity. )))
__________ use(s) statistical tools to analyze the causes of product defects. ((( Six sigma quality )))
Morgan owns and manages a custom woodworking business. She and her staff of four craftspeople work informally together, and each worker tends to make decisions in an independent (decentralized) manner. Morgan's business is a __________ operation. ((( small batch )))
In the context of technology configurations, which statement is true of large batch technologies? ((( Hierarchical authority is more prominent. )))
What characterizes firms such as International Paper that use continuous process technologies? ((( People are removed from the work itself. )))
__________ is the production of both high-volume and high-variety products at the same time. ((( Mass customization )))
__________ strives to achieve the highest possible productivity and total quality, cost-effectively, by eliminating unnecessary steps in the production process and continually striving for improvement. ((( Lean manufacturing )))
Which statement is true of flexible factories? ((( They are organized around products, in work cells or teams. )))
For the lean manufacturing approach to work well, ((( people should be broadly trained rather than specialized. )))
A just-in-time philosophy strives to increase a firm's competitive advantage based on eliminating waste and increasing ((( production. )))
Neven's specialty pharmaceuticals packaging firm needs to be more agile in order to be competitive in the marketplace. Foregoing buying a full-service laboratory because of the investment and risk, Neven decides to form a __________ with a chemical firm in China that has a laboratory but lacks a packaging business. Each company brings a complementary capability to the other, allowing them to pursue mutual goals. ((( strategic alliance )))
__________ is a system that calls for subassemblies and components to be manufactured in very small lots and delivered to the next stage of the production process as they are needed. ((( Just-in-time )))
With the impending retirements of very talented senior staff, Delia was concerned that Good Home Inc., her employer, was not planning for the future. Spurred on by her concern, Delia convinced management to budget and build a knowledge management system to create, acquire, store, and transfer knowledge in order to modify Good Home's behavior to reflect new knowledge and insights. Two years later, Delia was proud to reflect on her success in helping the company become a(n) __________ organization. ((( learning )))
A plant manager directed his top production engineer to sign up for an upcoming statistical quality workshop. The plant manager said, "I've long been a student of total quality management. I've decided to introduce a statistical approach called __________ into our production process to analyze the causes of product defects, so that we can then eliminate those defects." ((( six sigma )))
Trish manages a factory that manufactures automotive wiring harnesses. Her customers require that their suppliers be __________ certified, so that the suppliers' products are compliant with specified voluntary quality standards—standards that are recognized worldwide. ((( ISO 9001 )))
Enrique, a management intern, was tasked with investigating ways for the firm to more effectively compete on a global basis. He was intrigued to learn about __________ as a framework by which to improve quality and to gain certification in accordance with these quality standards. ((( ISO 9001 )))
Jose complained to his fellow division manager, "Our company is bogged down with cumbersome bureaucratic requirements. We must convince our executive leadership that we need to act fast to meet customer needs and to respond to other outside pressures! We need to correct past mistakes quickly and prepare for an uncertain future! We must respond to threats and capitalize on opportunities! We must place a premium on __________!" ((( agility )))
Charlotte manages a large chemical plant that manufactures a few specialty products for the roofing industry. As a __________ operation, the plant manufactures products in large volumes but with little variety. Production is not continuous but is standardized. Charlotte manages the plant in a formal and hierarchical manner, and most major decisions are made by her. ((( large batch )))
Nate told his boss, "Unless we invest in new technology, such as computer-integrated manufacturing (CIM), we will not be able to deliver more high-variety and high-volume products. With CIM, we will have computer-aided __________, so that the work of our engineers and factory workers will be more in sync." ((( design and manufacturing )))
Armand told his management team, "Our design and production process occurs in an orderly, yet slow, process, from research to engineering to manufacturing. To be more competitive, we must speed up our research and product development through the use of __________. In this manner, we will incorporate the issues and perspectives of all the functions—including suppliers and customers—from the beginning of the process." ((( simultaneous engineering )))
With the cost of warehouse space and maintaining inventory increasing, Drake has decided to change the operation of his company to a "just-in-time" (JIT) mode, whereby manufacturing is initiated only upon customer order and only in very small lots, minimizing the company's warehouse and inventory costs. However, Drake is nervous about the change because he knows that JIT offers costs savings only when the costs of storing product are greater than the costs of ((( frequent delivery. )))
The CEO for Carlson Corporation has announced to the management team that it has been decided to implement lean manufacturing at the Southside plant. The CEO stated, "I want to eliminate unnecessary steps in the production process and continually strive for __________. We must emphasize quality, speed, and flexibility." ((( improvement )))
The CEO of TruTone Manufacturing tells her executive management team that the company must become more agile to stay competitive. She says, "We have several options to improve our agility, a couple that we have already started to implement. But, most importantly, I believe that we need to organize around our primary strengths and expertise: our __________." ((( core capabilities )))
In terms of organizational agility, technology refers to the methods, processes, systems, and skills used to ((( transform resources into products. )))
Latisha has been promoted to a managerial position in Betty's Bakery. One of her jobs is to coordinate employee vacation schedules. However, the owner of the company overrides her decisions. Consequently, her subordinates do not respect her position as a manager. Latisha has identified the problem, and it is that the ((( owner has not delegated enough authority to enable her to do her work. )))
Karl is a recently promoted manager at Good for You Foods, winning the position over his former peers. His peers do not see Karl as their supervisor; they continue to think of him and treat him as a peer. As a result, Karl must rely more on __________ to get his work done and to manage his staff. ((( informal influence tactics )))
In a five-year time period, Zeke's organization has grown from a one-man motorcycle repair shop to a custom bike manufacturing/assembly and rebuild business employing 32 mechanics and technicians. Due to Zeke's difficulty in delegating responsibility, he makes almost all business and design/technical decisions. As a result, he is working 80 hours per week, and the business has stopped expanding Zeke's business is a(n) __________ organization. ((( centralized )))
Prior to going on vacation for two weeks, the division manager assembled his management team and said, "In my absence, Sophie will serve as acting division manager. I know that all of you will give Sophie your full support and complete any projects she assigns." In this situation, Sophie will gain ((( authority and responsibility. )))
As vice president of human resources at Oswald Industries, Asher enjoys working alongside his peer vice presidents, largely because they all have different skills and all manage different parts of the organization, including research, engineering, production, construction, procurement, and finance. Oswald Industries is a(n) __________ organization. ((( functional )))
After a major international expansion, the CEO of Scofield Corporation announced the new corporate structure by naming vice presidents for the newly established Far East Division, Middle East Division, European Division, and the Americas Division. Scofield Corporation's new structure follows the __________ approach to departmentalization. ((( geographic )))
Tomas meets with his group leader and says, "I'm not sure what to do. I have conflicting direction from several managers. James wants me in the field next week across town. Anya wants me in the field next week out of state. And I know that you would like me to work next week in the office." His group leader replies, "We will solve this problem through communications and prioritization. We work for a __________ organization and having dual or even more lines of command can be confusing." ((( matrix )))
Rachael works in a matrix organization, and although she found it confusing at first, she has seen the inherent opportunities. Instead of being promoted from biology head to research head, Rachael prefers a project career, with opportunities to serve in increasingly responsible and diverse project manager/project director positions. Fortunately, one of the strengths of a matrix organization is ((( more career choices, on both sides of organization. )))
In an organization, when there are many subunits and many specialists who think differently, differentiation is said to be high. ((( TRUE )))
Ten employees work under the supervision of Eddie, the manager of Elegant Day Salon. Eddie reports directly to the owner; therefore, in the context of span of control, Elegant Day Salon is a flat organization. ((( TRUE )))
Although she has an accounting degree, Dallas works as a recruiter in the human resources department of a major accounting firm. Dallas works for a line department. ((( FALSE )))
In general, coordination methods within an organization include standardization, plans, and mutual adjustment. ((( TRUE )))
Coordination by plan deals with establishing common routines and procedures that apply uniformly to everyone. ((( FALSE )))
Sometimes, competitors create strategic alliances. ((( TRUE )))
In a high-involvement organization, top management determines and dictates the direction in which the business is heading. ((( FALSE )))
According to the findings of Kouzes and Posner, which action is taken by the best leaders? ((( They challenge conventional beliefs and practices. )))
__________ is best described as a mental image of a possible and desirable future state of an organization. ((( Vision )))
In leadership, vision expresses ((( a leader's ambitions for an organization. )))
Which statement is accurate regarding leadership vision? ((( A person or team can develop a vision for any organization. )))
Academic research shows that __________ lead(s) to higher organizational performance. ((( a clear vision and communication of that vision )))
The CEO of Box Well Packaging worked diligently with her board to develop a vision for the firm. She tells her chairman, "I know where I want us to go. And I must continue to __________ this vision for others within the organization.” ((( articulate )))
Supervisory leadership is behavior that provides ((( guidance, support, and corrective feedback for day-to-day activities. )))
__________ leadership gives purpose and meaning to organizations by anticipating and envisioning a viable future for the organization and working with others to initiate changes that create such a future. ((( Strategic )))
What skill distinguishes excellent managers from other managers? ((( the ability to lead )))
Managing requires __________, while leading includes setting the direction. ((( planning and budgeting routines )))
Everett told his coworker, "I really admire Hilda's leadership, and I'm glad that she is our boss. She has articulated her vision and motivated us to ((( overcome our obstacles.” )))
The most effective followers are distinguished by the ability to ((( think independently but remain committed to organizational goals. )))
A leadership scholar asserts that “Executives are given subordinates; they have to __________ followers.” ((( earn )))
In organizations, __________ often means the ability to get things done or accomplish one's goals despite resistance from others. ((( power )))
A leader with __________ power has the right, or the authority, to tell others what to do. ((( legitimate )))
A leader who has __________ power influences others because he or she controls valued benefits; people comply with the leader's wishes so as to receive those benefits. ((( reward )))
A leader with __________ power has control over punishments; people comply to avoid those punishments. ((( coercive )))
A leader with __________ power has personal characteristics that appeal to others. ((( referent )))
The top management of a company determined that their end-of-year bonuses would not be given due to low sales. Emily, a department manager, informed her staff that even high-performing employees should not expect a bonus. Emily's __________ power was limited by the decision of the top management. ((( reward )))
At a restaurant, employees who perform poorly during a particular shift are scheduled for the unpopular weekend shifts by the manager. In this case, the manager exercises his __________ power. ((( coercive )))
A leader who has __________ power has certain knowledge; people comply because they believe in, can learn from, or can otherwise gain from that knowledge. ((( expert )))
Darla, a young manager at an organization, has a lot of influence over her employees. She is an admired manager: her subordinates like working with her and want to emulate her enthusiasm and personal style. In this scenario, Darla has __________ power. ((( referent )))
The __________ is a leadership perspective that focuses on individual leaders and attempts to determine the personal characteristics that great leaders share. ((( trait approach )))
In the trait approach to leadership, __________ refers to a set of characteristics that reflect a high level of effort, including high need for achievement, constant striving for improvement, ambition, energy, tenacity, and initiative. ((( drive )))
In the trait approach to leadership, __________ distinguishes effective leaders from other people. ((( self-confidence )))
__________, in the trait approach to leadership, is the correspondence between actions and words. ((( Integrity )))
Which statement describes a characteristic of effective leaders? ((( They have a high need for power. )))
Ryder, an employee at a company, is extremely eager to take up higher positions in the organization. He is an extravert and tries to influence the decisions made by his boss. He prefers to lead his colleagues rather than follow them. According to the trait approach to leadership, Ryder displays ((( motivation. )))
According to the trait approach to leadership, a(n) __________ leader overcomes obstacles, makes decisions despite uncertainty, and instills assurance in others. ((( self-confident )))
The dark triad of negative leadership traits include ((( narcissism. )))
Cultural expectations differ for leadership approaches. For example, the Chinese embrace the concept of guanxi, an expectation of ((( continuously exchanged favors. )))
__________ is a leadership perspective that attempts to identify what good leaders do—that is, what behaviors they exhibit. ((( Behavioral approach )))
The __________ approach downplays personal characteristics in favor of the actual conduct that leaders exhibit. ((( behavioral )))
__________ are actions taken to ensure that a work group or organization reaches its goals. ((( Task performance behaviors )))
In the behavioral approach to leadership, __________ behavior is variously referred to as concern for production, directive leadership, initiating structure, or closeness of supervision. ((( task performance )))
Which question can be used to identify a manager's level of task performance leadership? ((( To what extent does your superior give you detailed instructions and updates? )))
__________ behavior is sometimes referred to as concern for people, supportive leadership, or consideration. ((( Group maintenance )))
__________ behavior includes a focus on people's feelings and comfort, appreciation of them, and stress reduction. ((( Group maintenance )))
Which question can be used to identify a manager's level of group maintenance leadership? ((( Does your supervisor generally support you? )))
The __________ highlights the importance of leader behaviors not just toward the group as a whole but toward individuals on a personal basis. ((( LMX (leader-member exchange) theory )))
According to the __________, maintenance behaviors such as trust, open communication, mutual respect, mutual obligation, and mutual loyalty form the cornerstone of relationships that are satisfying and perhaps more productive. ((( leader-member exchange theory )))
__________ leadership seeks information, opinions, and preferences, sometimes to the point of meeting with the group, leading discussions, and using consensus or majority vote to make the final choice. ((( Democratic )))
Chan's boss makes no managerial decisions but asks his employees to take the necessary steps in all aspects of work. Chan's boss follows a(n) __________ decision-making style. ((( laissez-faire )))
Which statement is accurate regarding various decision styles? ((( A democratic style results in employees' positive attitudes. )))
In a research program that was conducted at the University of Michigan, planning, scheduling, coordinating, providing resources, and setting performance goals were all elements of what they called __________ behavior. ((( task-oriented )))
In a research program that was conducted at the University of Michigan, it was found that relationship-oriented behavior also made for effective managers. This behavior is characterized by ((( demonstrating trust and confidence in employees. )))
A __________ approach is a leadership perspective that proposes universally important traits and behaviors do not exist and that effective leadership behavior varies. ((( situational )))
The __________ model of leadership uses the basic situational approach of assessing the situation before determining the best leadership style. ((( Vroom )))
According to __________, leadership effectiveness depends on two factors: the personal style of the leader and the degree to which a situation gives the leader power, control, and influence over the situation. ((( Fiedler's contingency model )))
In Fiedler's contingency model of leadership effectiveness, leadership styles were measured with an instrument assessing the leader's ((( least preferred coworker )))
In Fiedler's contingency model, __________ leadership is more likely exhibited by leaders with low least preferred coworker (LPC) scores. ((( task-motivated )))
In Fiedler's contingency model, __________ leadership is more likely from high least preferred coworker (LPC) leaders. ((( relationship-motivated )))
In __________, the key situational factor is the maturity of the followers. ((( Hersey and Blanchard's situational theory )))
According to Hersey and Blanchard's situational theory, __________ is the level of the followers' skills and technical knowledge relative to the task being performed. ((( job maturity )))
In Hersey and Blanchard's situational theory, __________ refer(s) to an employee's self-confidence and self-respect. ((( psychological maturity )))
__________ is concerned with how leaders influence followers' perceptions of their work aims and the courses they follow toward attaining that end. ((( Path-goal theory )))
In the path-goal theory, __________ leadership refers to behaviors geared toward motivating people, such as setting challenging goals and rewarding good performance. ((( achievement-oriented )))
In the path-goal theory, __________ is the degree to which individuals respect, admire, and defer to power. ((( authoritarianism )))
In the path-goal theory, __________ is the extent to which individuals see events as being under their influence. ((( locus of control )))
In the __________, after answering a series of questions, several leader decisions styles may be appropriate, but the model recommends using the one that takes the least amount of time. ((( Vroom model )))
According to the trait approach to studying leadership, __________ relates to leadership emergence and leadership effectiveness. ((( extraversion )))
According to the behavioral approach to studying leadership, __________ behavior includes a focus on work speed, quality and accuracy, quantity of output, and following the rules. ((( task performance )))
__________ is a form of leadership in which the leader makes decisions on his or her own and then announces those decisions to the group. ((( Autocratic leadership )))
"__________ is a situational approach to leadership that emphasizes the participative dimension of leadership: how leaders go about making decisions. ((The Vroom model))
" ((( The Vroom model )))
Which question can be used to identify a manager's level of group maintenance leadership? ((( Does your superior give credit to team members for helping with projects and problem solving? )))
Which workplace factor can be an effective substitute for leadership? ((( a closely knit group with a professional orientation )))
Kenn told his subordinate, Greg, "Let me give you some advice. I know that you take pride in your delegating leadership style, and indeed you're good at it. But, always remember that effective leaders rely on __________ style(s) of leadership, depending on the situation." ((( different )))
Farouk, manager of a 50-person engineering department, exhibits group maintenance behaviors; he ensures employee satisfaction, harmonious work relationships, and the department's social stability. However, his division manager notes that Farouk's department rarely achieves its yearly goals, in terms of work speed, quality and accuracy, output, and compliance with company policies and requirements. How should the division manager coach Farouk to perform his job at a higher level and to increase the organization's performance? ((( Focus more on task performance behaviors. )))
__________ has a strong positive impact on follower satisfaction and motivation and also on leader effectiveness. ((( Group maintenance behavior )))
Raegan told her protégé, Tyler, "You are particularly effective in exhibiting __________ behaviors, an important part of the LMX theory. It's obvious that you are respected for your trust, open communication, and mutual respect towards your staff. But remember that, in your relocation to our new office in Tokyo, these behaviors will need to change due to cross-cultural differences." ((( group maintenance )))
"James, you have a strong management team supporting you, and I applaud your efforts to solicit information, opinions, and preferences from your team, and in seeking consensus in reaching decisions," said his boss. "But sometimes decisions need to be made more quickly, and sometimes your team simply wants you to step up and make certain tough decisions. In other words, a(n) __________ style of leadership might be appropriate." ((( autocratic )))
Research indicates that the performance and maintenance dimensions of leadership are ((( independent of each other. )))
"Dr. Redstone, welcome to Doctors Without Borders," said the program administrator. "I will serve as your organizational manager 'in name only,' as we both know that I will not be providing much management, much less leadership, to your team in a foreign country. I'm confident, however, that you will not lack for leadership. After all, you will be working closely with several highly experienced professional colleagues in an environment that demands the close cooperation of a tight-knit group. Furthermore, the rules and procedures that govern your work are rigid, so your operating flexibility will be limited." In this case, the program administrator is counting upon __________ to provide a positive influence on Dr. Redstone. ((( substitutes for leadership )))
According to the proponents of the __________ approach to leadership, universally important traits and behaviors do not exist. ((( situational )))
The __________ asks a leader to use a decision tree to assess the problem and determine the appropriate decision style to employ. ((( Vroom model )))
Lacie, a supervisor new to her job, knows that, in leading a team that is new to her, she must adapt her leadership style to suit the circumstances. She knows that she should first analyze each situation and then decide what to do. Lacie is adopting the __________ leadership approach. ((( situational )))
"I'm not sure that I'm going to like our new boss," said Lawrence to his coworkers. "I've noticed that when a decision needs to be made, boom!—she makes it, announces it, and moves on without consulting any of us." Lawrence's new boss is using a(n) __________ leadership approach to decision making. ((( autocratic )))
Ian oversees two very different workers. Horatio wants to be told exactly what to do because he is hesitant to make decisions, and Nina wants to be told exactly what to do because she wants to get on with it. So, even though they are very different people, they both respect decisiveness. According to the path-goal theory of leadership, Ian should use the __________ leadership style with these workers. ((( directive )))
Carter has decided to start using the situational leadership approach he learned about last month. He likes the idea of a leader adapting his or her leadership style depending upon the __________ and __________ of the follower in a specific situation. The former refers to the follower's skills and technical knowledge relative to the task being performed. The latter refers to the follower's self-confidence and self-respect. ((( job maturity; psychological maturity )))
A __________ leader has a special ability to generate excitement and revitalize organizations. ((( transformational )))
A __________ leader is a person who gets others to transcend their personal interests for the good of the group. ((( transformational )))
A __________ leader manages through activities, using his or her legitimate, reward, and coercive powers to give commands and exchange rewards for services rendered. ((( transactional )))
__________ leadership is a combination of determination and personal humility that builds enduring greatness. ((( Level 5 )))
__________ leadership refers to a style in which the leader is true to himself or herself while leading. ((( Authentic )))
A(n) __________ leader seems to care about organizational interests, but tends to ignore followers’ real needs in order to put their own self-interests at the forefront. ((( pseudotransformational )))
Transformational leaders generate excitement and ((( revitalize organizations. )))
At the annual shareholders' meeting of Galaxy Industries, one shareholder turned to the other and whispered, "Our CEO, Levi Nilsson, has done an outstanding job of leading his division presidents to set aside their personal interests for the betterment of Galaxy. And Levi is just such an exciting and passionate man. He has done wonders in turning around this organization." The other shareholder replied, "Indeed, Levi truly stands out as a(n) __________. I'm so glad that I've invested heavily in Galaxy!" ((( transformational leader )))
__________ serve others' needs while strengthening the organization. ((( Servant-leaders )))
Tam, Leslie, and Sawyer are three qualified engineers working in a team at a firm. Though Tam has the most experience, he does not always lead the group. Some projects require specialized knowledge that the other two members have, so the leadership role is passed around the group as appropriate. They are using the __________ style of leadership. ((( shared )))
"Did you hear that Braden is returning back to the States, to work in corporate headquarters as vice president of environmental services in the international division?" asked Zach. His coworker, replied, "Yes. Obviously those four years that Braden has spent abroad, in our Rio and Tokyo offices, have served him well." In leaving his culture and working in other cultures for a significant period of time, Braden will bring __________ leadership to his organization. ((( bridge )))
The CEO noted to one of his managers, "Judith, you and the other senior managers have done an outstanding job of developing leadership in your divisions, especially at the very top. But we need more than just one or two superstars at the top. You need to spread out the responsibilities so that ((( leadership permeates the organization." )))
A __________ leader lives, goes to school, travels, or works in other cultures, then returns home to lead. ((( bridge )))
Lisa's friend asked her, "What is the key to your success? After all, you have been promoted twice in the past three years." Lisa replied, "It's simple. When I tell my employees to do something, they do it. I'm the boss; they know it; and they behave accordingly." From this dialogue, it is clear that Lisa has __________ power. ((( legitimate )))
Marlene, a shift manager at Bargain Interiors, told her roommate, "Work isn't much fun right now. I acknowledge that I have very little experience in the retail industry, but I just can't get my floor staff to perform their duties unless I use discipline. Over and over, I'm threatening to cut a nonperformer's hours unless he or she improves performance. That seems to be the only way that I can exert influence over them to make them perform their duties, but even then they don't respect my position as their manager." Based upon this discussion, why are Marlene's floor staff willing to comply with her direction? ((( They want to avoid punishment. )))
George said, "I've never had a client leader like Aimee before. She has what is called referent power. As a result, the work that I do for Aimee is my very best because ((( I admire her and desire her approval." )))
Ellie asked her coworker, "Why do you always consult with Harry before making any major business decisions? Harry is 65 years old. He's very quiet and unassuming. He's been working here since the firm was founded 40 years ago and isn't even in a management position." Her coworker explained that Harry ((( has expertise and knowledge that others believe in, can learn from, and can gain from. )))
Albert candidly thinks to himself, "I just don't know if I have the skills to progress in my career. I've made it to the second level of management, but I want to contribute at a greater level. I want to create a vision for the firm and institute important change.” Albert is striving to ((( become a leader. )))
Bob, a division manager, talks about positive change for his staff and business, but he allows self-interest to take precedence over the needs of the division. Bob is a __________ leader. ((( pseudotransformational )))
"Depending upon your political persuasion, Colleen is either the second coming of Ronald Reagan, or the second coming of Barack Obama!" said Savannah. "She is such a __________ leader. She is dominant, self-confident, convinced of the moral righteousness of her beliefs, and able to rally her followers with excitement." ((( charismatic )))
Joyce uses task performance behaviors to ensure that the work of her business unit gets done. Therefore, she ((( focuses on work speed, quality and accuracy, quantity of output, and compliance with the rules. )))
Blake uses group maintenance behaviors as a major part of his leadership activities. Therefore, he ((( ensures the satisfaction of all employees, focusing on their feelings and comfort. )))
Three employees were discussing their managers' decision-making styles. The first said, "My boss is very autocratic. She makes all the decisions, and I'm a bit scared of her. Our financial performance is fine but could be better." The second stated, "My boss is the opposite; she is always democratic. She always involves all of us in discussion and decision making, and morale is good. But I think our financial performance should be better." The third concluded, "My boss takes a very laissez-faire approach. He makes no decisions. In our business unit, ((( morale and attitudes are poor, and financial performance is poor." )))
True leaders are those who are living examples of the ideals they support. ((( TRUE )))
Effective managers are necessarily true leaders. ((( FALSE )))
If company policy dictates that everyone is to receive the same salary increase, a leader's reward power increases. ((( FALSE )))
Research indicates that a laissez-faire leadership approach has a negative influence on attitudes and performance. ((( TRUE )))
Advanced degrees are more important for leaders than acquired expertise in matters relevant to an organization. ((( FALSE )))
Charisma can contribute to transformational leadership. ((( TRUE )))
For the person who wants to both lead and serve others, servant-leadership is a way to serve their needs and enhance their personal growth while strengthening the organization. ((( TRUE )))
Motivation refers to forces that ((( energize, direct, and sustain a person's efforts. )))
For a workplace goal to be motivating, it ((( has to be acceptable to employees. )))
Goals should be __________ to be motivating. ((( quantifiable )))
Which goal is most likely to stimulate performance, according to the guidelines of goal-setting theory? ((( Obtain sales levels that are 15 percent higher than last year. )))
Stretch goals are the ((( targets that are exceptionally demanding and novel. )))
Vertical stretch goals are ((( aligned with current activities, including productivity and financial results. )))
Horizontal stretch goals involve what area of an organization? ((( people's professional development )))
In his second year of managing the accounting department, Scott set a goal for his team to slightly improve its "days billing" from last year's 36 days to 35 days. Midway through the year, Scott is disappointed that the "days billing" has actually worsened to 37 days. Scott's manager advised that to motivate his staff to perform at a higher level, Scott should set a goal that is ((( more challenging. )))
What premise specifically states that people have conscious goals that energize them and direct their thoughts and behaviors toward a particular end? ((( goal-setting theory )))
In Declan's annual performance review, he submitted his draft goals for the upcoming year. One of his goals was to "Improve my punctuality." His boss responded, "No, that goal is not acceptable, based on your poor attendance record. Your goal needs to be __________ so that you and I can monitor and measure whether or not you are attaining it." ((( specific and quantifiable )))
Every year, Tandy's previous manager developed and given her a list of performance goals for the upcoming year, and then proceeded to criticize her for not fully achieving her previous year's goals—goals that he had developed and given to her. And, in fact, Tandy was not always motivated to achieve the goals prescribed for her. But Tandy's new manager is a breath of fresh air. Tandy tells her roommate, "My new manager lets me participate in goal-setting for my work and, as a result, ((( we tend to generate goals that I accept and pursue willingly." )))
Stretch goals can shift people away from ((( mediocrity. )))
In a group setting, individualized performance goals can create __________ and reduce __________. ((( competition; cooperation )))
Andi reflected on last year's one goal that she established for her department—to achieve $1 million in profit. And she required all staff members to establish this same personal goal—and only this goal—for themselves. Looking back a year later, Andi realized that her dogged pursuit of a "number" caused her department to fall a year behind in training and new business development. Andi knows that this focus on one dimension of her business cannot be repeated. For the coming year, she must avoid ((( establishing a single productivity goal. )))
"__________ are positive consequences that motivate behavior.
" ((( Reinforcers )))
__________ formulated the law of effect. ((( Edward Thorndike )))
The law of __________ states that behavior that is followed by positive consequences will likely be repeated. ((( effect )))
Organizational behavior modification primarily attempts to ((( improve performance. )))
Hector, a manager at Roseland Corp., recently illustrated __________ by promoting two employees because their performance had greatly improved. ((( positive reinforcement )))
__________ is the act of applying a consequence that increases the likelihood that the person will repeat the behavior that led to that result. ((( Positive reinforcement )))
__________ is the practice of removing or withholding an undesirable consequence. ((( Negative reinforcement )))
Arun, the manager at Advanced Solutions, took an employee off probation because of her improved performance, illustrating ((( negative reinforcement. )))
__________ is an example of positive reinforcement. ((( Giving letters of commendation )))
__________ is an example of extinction. ((( Forgetting to say thanks for a favor )))
__________ is the act of administering an aversive consequence. ((( Punishment )))
__________ involves the instances of withdrawing or failing to provide a reinforcing consequence. ((( Extinction )))
What is most likely to occur when positive reinforcement is employed? ((( The same behavior is likely to be repeated. )))
As an employee receiving feedback, one should ideally ((( avoid negative emotions. )))
Viktor's work performance has greatly improved over the course of the past two months, so his boss decided to upgrade his status from that of a temporary employee to that of a permanent employee. This is an example of ((( negative reinforcement. )))
For optimum effectiveness, managers should provide employees with feedback ((( on a regular and ongoing basis. )))
__________ is an example of punishment. ((( Assigning an unappealing task )))
Jeanette was pleased to reward her top sales professional with a hefty bonus for the third year in a row because of his increasingly large sales figures. He said, "Jeanette, you keep providing __________ as a consequence of my efforts, and I will keep posting excellent sales!" ((( positive reinforcement )))
Jed, an operations manager, struggles with staffing the unpopular night shift, where all new employees are initially assigned. The turnover is high, morale is poor, and performance is dismal. Jed has tried positive reinforcement, offering rewards for punctuality and good work habits; this approach failed. He has also tried the extinction approach, also with little success. Finally, he punished wrongdoers, and that also failed to bring about any desired change. If Jed now tries negative reinforcement, what might be a proper approach? ((( Offer reassignment to the day shift for the best night-shift performers. )))
Mildred knows that she will not have bonus dollars to reward her high-performing management team this year due to a shortfall in company profits. Furthermore, a companywide hiring and promotion freeze will prohibit her from hiring entry-level workers to allow existing staff to be promoted to higher levels. In response, she has arranged for her management team to attend the annual corporate planning meeting, affording them a unique opportunity to interact with and gain recognition by company executives. Mildred instituted this nonmonetary reward because ((( nonmonetary rewards can motivate when pay and promotions are scarce. )))
In contrast to reinforcement theory, expectancy theory ((( considers some of the cognitive processes that go on in people's heads. )))
__________ theory proposes that people will behave based on their perceived likelihood that their effort will lead to a certain outcome and on how highly they value that outcome. ((( Expectancy )))
Expectancy theory includes the following events, in order: ((( effort, performance, and outcome valence. )))
According to expectancy theory, instrumentality is ((( the perceived likelihood that performance will be followed by a particular outcome. )))
__________ is the link between performance and outcomes in expectancy theory. ((( Instrumentality )))
According to expectancy theory, valence ((( can be positive or negative )))
__________ is the perceived likelihood that performance will be followed by a particular outcome. ((( Instrumentality )))
According to expectancy theory, what must occur for motivation to be high? ((( Expectancy must be high. )))
__________ is a consequence a person receives for his or her performance. ((( Outcome )))
For motivation to be high, total valence of all outcomes must be ((( high. )))
Dale, a marketing professional who reports to you, believes that he can meet his $1 million sales goal this year (expectancy). And he would truly value the $10,000 bonus (valence) that he might earn as a result. In the past, your company has failed to pay out promised rewards. Therefore, in accordance with the expectancy theory, he is understandably concerned—and hence demotivated—that ((( the company will not follow through on its commitment to provide a bonus (instrumentality). )))
In the context of Maslow's need hierarchy, the need found at the bottom of the pyramid is the __________ need. ((( physiological )))
__________ is the highest-level need in Maslow's hierarchy. ((( Self-actualization )))
In Maslow's need hierarchy, __________ need refers to a need for friendship, affection, belonging, and love. ((( social )))
In Maslow's need hierarchy, __________ includes the need for recognition and self-esteem. ((( esteem )))
Which statement is accurate according to Maslow's need hierarchy? ((( People are motivated to satisfy lower-level needs before higher-level needs. )))
Which statement is accurate regarding the importance of Maslow's need hierarchy for managers? ((( It sensitizes managers to the importance of personal growth. )))
Alderfer's ERG theory primarily focuses on ((( existence needs, relatedness needs, and growth needs. )))
The Maslow's need hierarchy is different from Alderfer's ERG theory in that ((( ERG theory asserts that various needs operate simultaneously. )))
According to McClelland, the most important needs for managers are the needs for ((( achievement, affiliation, and power. )))
According to McClelland's research, the need for achievement is ((( important to most U.S. managers. )))
According to McClelland, __________ power is used as a negative force. ((( personalized )))
Garrett's company was hemorrhaging money and losing many valuable employees through layoffs and resignations. He tried, but failed, to motivate Allison, his best employee, by discussing her next year's goals, appealing to her full potential as a professional. She told him she could not think about next year—she was worried about keeping her job this year. In accordance with Maslow's need hierarchy, why is Garrett failing to motivate Allison? ((( Garrett is speaking to higher-level needs that Allison is not in a position to currently appreciate. )))
A low need for affiliation and a moderate to high need for power are associated with ((( managerial success. )))
Generally speaking, managers in the United States are motivated most by ((( achievement, esteem, and self-actualization. )))
In the context of Alderfer's ERG theory, existence needs typically ((( are physiological desires. )))
Brady loves his job. It offers him the opportunity to meet lots of people, to work outdoors, and to do things his own way. Brady often jokes that he would be glad to do his job even if he is not given any pay. This sense of fulfillment is an example of ((( intrinsic reward. )))
__________ are the rewards given to a person by the boss, the company, or some other person. ((( Extrinsic rewards )))
__________ is intended to alleviate boredom by giving people different things to do at different times. ((( Job rotation )))
__________ is the act of giving people additional tasks at the same time to alleviate boredom. ((( Job enlargement )))
__________ is the act of changing a task to make it inherently more rewarding, motivating, and satisfying for the people concerned. ((( Job enrichment )))
Jones is the marketing manager at a small firm. In addition to his marketing-related duties, he has recently been put in charge of recruitment, at his request. This is an example of ((( job enlargement. )))
__________ means that jobs are restructured or redesigned by adding higher levels of responsibility. ((( Job enrichment )))
Tanner's job responsibilities have recently been changed as part of a decentralization effort taking place at his office. He has been given the authority to decide when a file's status is to be altered, rather than only altering files at his supervisor's discretion. This is an example of ((( job enrichment. )))
Herzberg's __________ theory describes hygiene and motivators as factors affecting people's work motivation and satisfaction. ((( two-factor )))
In Herzberg's two-factor theory, __________ are characteristics of the workplace, such as company policies, working conditions, pay, and supervision, that can make people dissatisfied. ((( hygiene factors )))
According to Herzberg, the key to true job satisfaction and motivation to perform lies in ((( motivators. )))
In the Hackman and Oldham model, __________ is the independence and discretion in making decisions. ((( autonomy )))
Growth need strength is best defined as the ((( degree to which people desire personal and psychological development. )))
__________ is the process of sharing power with employees, thereby enhancing their confidence in their ability to perform their jobs and their belief that they are influential contributors to the organization. ((( Empowerment )))
"In job __________, workers are given additional tasks at a higher level of responsibility.
" ((( enrichment )))
According to __________, characteristics of the workplace termed hygiene factors will not necessarily motivate workers. ((( Herzberg )))
Jameson said to his employee, "Stephan, I will be out next week on client business. In my absence, I'd like you to serve as acting manager. On Monday, I'd like you to lead our weekly sales meeting. Also, please assign client contacts to the sales staff for the upcoming trade show. Lastly, decide upon final terms for our contract with People's Power, and negotiate that contract to closure. I'm glad that I can count on you." In empowering his employee, Jameson has ((( shared power. )))
Clinton said to his employee, "Ryan, I don't know what else I can do for you. You have a job, a good paycheck, competitive benefits, a suitable office space, and reasonable people with whom to work." Ryan replied, "I'm sorry but those things don't motivate me." According to Herzberg's two-factor theory, what might Ryan be looking for? ((( opportunities for personal growth and recognition )))
Which statement is true of growth need strength? ((( A person's growth need strength will help determine just how effective a job enrichment program might be. )))
Which statement is supported by equity theory? ((( Inequity causes dissatisfaction. )))
__________ is the practice of using a fair process in decision making and making sure others know that the process is as fair as possible. ((( Procedural justice )))
As the on-site project manager, Brandon reprimanded the construction superintendent, "Gabe, I'm going to have to write you up for not properly securing yourself on that elevated work site. I've written up others, and the crew will be watching to see what I will do." Gabe replied, "I've been up there dozens of times, and we're on a tight schedule. There's no time to fool with every safety procedure." In writing up Gabe as he did other employees, Brandon was implementing ((( procedural justice. )))
"I haven't missed a day of work in the past five years, and I'm committed to this company, so I determined that a 20 percent salary increase is warranted for reasons of fairness," said Tony to his supervisor. In reply, she asked, "How did you determine that this increase is fair?" If Tony is using equity theory as a basis for his request, he would reply, ((( “I made comparisons with others' salaries." )))
After a rough week and against her better judgment, Leesa tells Tarik she is tired of arguing with him over trip reports. She makes an exception for him and, against policy, says he no longer needs to fill them out. A week later, another sales rep comes to her and asks to stop preparing reports, just like Tarik. Why do you think Leesa now has a managerial problem? ((( She failed to provide procedural justice. )))
The general goal of quality of work life (QWL) programs is to ((( satisfy the full range of employee needs. )))
Constitutionalism in the workplace refers to ((( the rights of personal privacy, dissent, and due process. )))
A(n) __________ refers to a set of perceptions of what employees think their employers owe them and vice versa. ((( psychological contract )))
"Charles, I have some disturbing data," said the vice president of human resources. "Our company has suffered increases in turnover, absenteeism, lawsuits by employees and former employees, injury and illness claims, and customer complaints. About the only thing that hasn't increased is our productivity." Charles replied, "I have unfortunately come to recognize these workforce characteristics. They are a sign of ((( job dissatisfaction." )))
Merrick tells his sales manager, "Our staff are dissatisfied, and they have lacked good citizenship since we lost the big account." What does Merrick mean regarding lacking good citizenship? ((( His employees are not going the "extra mile" and helping others at work. )))
Advocates of quality of work life (QWL) programs assert that it benefits the organization by ((( improving productivity. )))
The manager of operations, who is a critic of quality of work life programs, told the firm's manager of human resources, "I'm not convinced that a QWL program will necessarily inspire employees to work harder. Rather, I want us to tie rewards directly to ((( individual performance." )))
A survey by Mental Health America found that a top contributor to employee dissatisfaction is ((( not being rewarded for daily contributions. )))
__________ is a set of perceptions of what employees owe their employers and what their employers owe employees. ((( A psychological contract )))
__________ create a workplace that enhances employee well-being and satisfaction. ((( Quality of work life (QWL) programs )))
The ideal psychological contract for both employers and employees can be summarized as, ((( “If you help me grow, I'll help the company grow.” )))
Donte was given a surprise $500 bonus, and his supervisor told him that if he continued to post numbers as he did that month, he would receive a bonus each month. Donte was motivated to go all out and do his best every month. This shows the power of ((( positive reinforcement. )))
Cody racked up three safety violations in one month and was taken off the construction crew and put in the office, meaning he was no longer eligible for overtime. He pled with his boss, saying he needed the money for rent. Stating that Cody's financial situation was not a company concern, he was put in the office until he could demonstrate that safety was a priority. This scenario illustrates ((( punishment. )))
Malik is demoralized and unmotivated. He tells his coworker, "I'm at a dead end in this job. Believe me, I would love to get a raise or a promotion. But no matter how well I do, I'll never get that kind of recognition." In the context of the expectancy theory, this illustrates ((( low instrumentality. )))
Andrew told his fellow roofer, "I'm telling the boss at the end of the day that I'm quitting. This work is too physically demanding and dangerous for what they're paying me." In the context of the expectancy theory, Andrew's experience illustrates ((( negative valence. )))
The most powerful goals are those which appeal to people's higher values. ((( TRUE )))
Organizational behavior modification focuses on influencing behavior. ((( TRUE )))
Negative reinforcement involves administering an aversive consequence, such as reducing work hours or docking pay. ((( FALSE )))
An expectancy can be high as well as low. ((( TRUE )))
Effective managers can create positive outcomes by realizing that everyone passes through the five needs in Maslow's hierarchy in order. ((( FALSE )))
Intrinsic rewards have a close relationship to inspiring creativity. ((( TRUE )))
Options for restoring workplace equity including increasing your own outcomes or decreasing others' outcomes. ((( TRUE )))
Quality of work life (QWL) programs are designed to increase profits and productivity. ((( FALSE )))
What is the best definition of control? ((( Control is any process that directs the activities of individuals toward the achievement of organizational goals. )))
Cheryl, an employee, received an e-mail from an angry client about a certain product. Although it was not Cheryl's fault, she hesitated to report it to her manager because she knew that she would be blamed and could even be fired. In this case, what seems to be true of Cheryl's company? ((( "Shoot the messenger" management exists, implying a lack of control. )))
At a companywide meeting, the CEO spoke energetically about adopting cost-cutting measures due to the recent economic slump. However, a few days later, the employees found out that the CEO had just left for an exotic vacation paid for by the company. Based on this scenario, what might be assumed about this company? ((( Senior managers set a bad example, implying a lack of control. )))
A company is said to lack control when there is a(n) __________, that is the firm's expectations are not established in writing. ((( absence of policies )))
Control and __________ have been called the Siamese twins of management. ((( planning )))
Bureaucratic control refers to ((( the use of rules, standards, regulations, hierarchy, and legitimate authority to guide performance. )))
__________ control is the use of rules, regulations, and authority to guide performance. ((( Bureaucratic )))
__________ control includes items such as budgets, statistical reports, and performance appraisals to regulate behavior and results. ((( Bureaucratic )))
What is an accurate statement about market control? ((( It uses prices, competition, and exchange relationships to regulate activities. )))
__________ control is based on the norms, values, shared goals, and trust among group members. ((( Clan )))
During challenging times when resources are limited and budgets need to be stretched, __________ control becomes crucial for organizational survival. ((( managerial )))
"Are you telling me that we have to write off over $2 million in revenue because our eastern division manager did not accrue it properly?" asked the CEO. His CFO replied, "Yes. I know that we are a start-up, still working on developing policies, rules, and procedures, but, in this case, the division manager had no processes in place to monitor the project's progress." The CEO concluded, "The __________ has caused irreparable damage to our firm." ((( lack of controls )))
Jensen thought, "I'm not surprised that the corporate office canceled our funding. Costs had spiraled upward, and unanticipated technical problems had cropped up. We just didn't keep our eye on the ball, even though we had a great plan and sound strategies. Fundamentally, we lacked an effective means of control to ensure that ((( the plans were carried out." )))
__________ lay(s) out a framework for the future and provide(s) a blueprint for control. ((( Planning )))
After first setting performance standards, the next step in a typical control system is ((( measuring performance. )))
The last step in a typical control system is ((( taking action to correct problems. )))
A(n) __________ is the level of expected performance for a given goal: a target that establishes a desired performance level, motivates performance, and serves as a benchmark against which actual performance is assessed. ((( standard )))
"In implementing the control process for our new operation, I first established performance standards," said Muriel. "Next, I measured performance. But I'm not sure that my measurement of performance is accurate." Her boss replied, "In measuring performance, you can gather performance data from three sources: __________. Furthermore, the data need to be timely." ((( written reports, oral reports, and personal observation )))
A division manager whispered to his peer as they left the monthly executive meeting, "Our new CEO is going to play hardball. Any of us whose division is performing with significant negative deviation from the established budget is going to get a lot of her concentrated attention." "What do you expect?" countered his coworker. "She told us in our meeting that she would be using __________ as part of her control system." ((( the principle of exception )))
The __________ control process is used while plans are being carried out, including directing, monitoring, and fine-tuning activities as they are performed. ((( concurrent )))
While plans are being carried out, __________ control is taking place. ((( concurrent )))
"Listen up," said Ian, the dive team leader, as his team finished suiting up. "This is a hazardous dive. We've been properly trained and properly equipped with the necessary gear and tools to rescue the trapped cavers. Stay in physical contact with the divers in front of and behind you. After we complete the rescue, meet me to debrief what we did well and what we can improve." In this case, the proper training and equipment is an example of __________ control. ((( feedforward )))
__________ control is future oriented and aims to prevent problems before they arise. ((( Feedforward )))
Which statement is accurate regarding six sigma? ((( It indicates how often defects in a process are likely to occur. )))
The six sigma approach is based on a(n) __________ that contribute(s) to customer satisfaction. ((( statistical analysis )))
A(n) __________ is an evaluation of the effectiveness and efficiency of various systems within an organization. ((( management audit )))
External audits are used for ((( identifying possible mergers or acquisitions. )))
External audits are useful for preliminary control because they ((( can prevent problems from occurring. )))
Assessing the planning, organizing, leading, and controlling efforts of one's own organization is the essential function of ((( internal audit. )))
A(n) __________ is an evaluation conducted by one organization of another organization. ((( external audit )))
__________ detect and correct significant variations, or discrepancies, in the results of plans and activities. ((( Control systems )))
"After a long and costly two weeks, I have lots of data, that I don't know how to analyze. I don't know what constitutes good performance as opposed to bad performance," said Joseph to his boss. His boss replied, "It sounds as if you failed to first establish a(n) __________ by which to assess your data." ((( standard )))
In __________ control, instead of waiting for results and comparing them with goals, a manager or employees can exert control by limiting activities in advance. ((( feedforward )))
Darby, shift supervisor at Hoosier Meatpacking, has the highest performing team. One reason for this is how she controls her team's activities. First, she ensures that her subordinates are properly trained and properly equipped. Then, during the shift, she carefully monitors her workers' actions, taking notes and even measurements as needed. Finally, at the end of each work week, she analyzes her notes and measurement data, and she uses the results of her analysis to make needed production changes the following week. When Darby is monitoring her workers' actions, she is exercising __________ control. ((( concurrent )))
What provides essential feedback control when they identify legal and ethical lapses that could harm the organization and its reputation? ((( external audits )))
__________ are targets that establish desired performance levels, motivate performance, and serve as benchmarks against which to assess actual performance. ((( Standards )))
Policies restricting workplace romantic involvement, tattoos and body art, and public disclosure of corporate information are examples of __________ control. ((( feedforward )))
__________ control focuses on the use of information about results to correct deviations from the acceptable standard after they arise. ((( Feedback )))
__________ is the second stage of budgetary control. ((( Performing budgetary operations )))
A __________ budget is used for areas of the organization that incur expenses but produce no revenue. ((( cost )))
Kristina is the financial manager at a candy manufacturer. One of her duties is to make a budget that shows the forecasted receipts and expenditures of the company, the amount of working capital available, and the degree to which it should obtain funds from outside of the firm. Kristina makes this budget after all other budget estimates have been completed. Kristina is working on a __________ budget. ((( cash )))
In his new employee orientation training, Preston heard his new accounting supervisor, explain, "At Fitzgerald Industries, we do not use traditional cost accounting. Rather, we use activity-based costing, or ABC. According to ABC, we __________ costs according to what our employees do, rather than what the company spends." ((( allocate )))
Required information for preparing a __________ budget includes types and capabilities of machines and availability of materials. ((( production )))
Budgeting information is ((( not confined to finances. )))
The __________ budget is essential to every business. ((( cash )))
Traditional cost accounting may be inappropriate today because ((( it is based on rigid hierarchical organization structures. )))
"I have to help my operations manager reduce expenses," said Tyrone, "but my expense categories—salaries, fringes, supplies, and fixed costs—are too general to give me guidance." Mavis, his accounting supervisor, said, "Let me show you a different approach, so that you can develop data that will identify areas for expense reduction. I suggest that you use __________. It will identify streams of activity to which you can then allocate costs across particular business processes according to the time employees devote to those activities." ((( activity-based costing )))
A(n) __________ is a report that shows the financial picture of a company at a given time and itemizes assets, liabilities, and stockholders' equity. ((( balance sheet )))
__________ are the values of the various items the corporation owns. ((( Assets )))
__________ are the amounts the corporation owes to various creditors. ((( Liabilities )))
__________ is the amount accruing to the corporation's owners. ((( Stockholders' equity )))
Select the relationship represented by the balance sheet. ((( assets = liabilities + stockholders' equity )))
A(n) __________ is an itemized financial document of the income and the expenses of a company's operations. ((( profit and loss statement )))
__________ indicate a company's ability to pay short-term debts. ((( Liquidity ratios )))
Nan has recently reviewed a number of indicators of financial control. The one that troubled her, the __________, indicated that the company could have trouble meeting its short-term financial obligations. ((( current ratio )))
__________ ratio is the name of the most common liquidity ratio, current assets to current liabilities. ((( Current )))
A __________ is a liquidity ratio that indicates the extent to which short-term assets can decline and still be adequate to pay short-term liabilities. ((( current ratio )))
__________ ratios show the funds supplied by creditors and shareholders. ((( Leverage )))
The __________ ratio indicates the company's ability to meet its long-term financial obligations. ((( debt-equity )))
__________ ratios indicate management's ability to generate a financial return on sales or investment. ((( Profitability )))
__________ refers to the situation when managers focus on short-term earnings and profits at the expense of their longer-term strategic obligations. ((( Management myopia )))
A company enforced a rule that employees were required to submit their cell phones for safekeeping before entering their work bay. Employees followed the rule in order to avoid getting into trouble with the management. As a result, many of them missed important phone calls from their clients and customers. This scenario is an example of ((( rigid bureaucratic behavior. )))
When a control system prompts employees to do only tasks strictly required in their formal job descriptions, it reflects ((( rigid bureaucratic behaviors. )))
__________ behavior occurs when employees feel forced to attempt to "beat the system." ((( Tactical )))
Uri noticed a pattern at the annual budgeting session of his company. Mid-level managers were asking for unrealistically high budgets while top management was attempting to limit budgets under last year's actual expenditures. Management used __________ strategies. ((( tactical behaviors )))
What is the most plausible reason why employees resist control systems? ((( Control systems can change the power structure of the organization. )))
__________ budget sheet items over time uncovers important trends. ((( Summarizing )))
"Many of you management trainees do not have formal educations or degrees in accounting or finance. And that is OK," said the corporate trainer. "That's why this training module will cover control by __________, which is most commonly used for the entire enterprise and, in the case of a diversified corporation such as ours, its divisions." ((( profit and loss )))
"The amount of information in the accounting reports is so overwhelming that I cannot understand it," said Woody to his division manager, Sam. "Yes," said Sam, "The sheer volume of the numbers in the reports is daunting. That's why we have decided that an effective approach for checking our firm's overall performance is to use __________, which indicate strengths and weaknesses." ((( key financial ratios )))
"With the rollout of this new __________, we will be able to get a much more accurate picture of how each division—and, in fact, each operations manager, each sales manager, and each sales professional—is performing. And we will be able to hold each manager and employee more accountable. As a result, do not be surprised when we receive a lot of resistance from across the organization," said the CFO of Magnetic International. ((( comprehensive control system )))
"My business relationship with the Midwest sales rep has really deteriorated. We used to cooperate; now we __________," said the manager of the Southwest region. "And I blame our new control system. Every month, managers across the entire enterprise get profit and loss statements for all divisions and business units. We look at one another's performance and are less likely to help one another when we know that our business unit or division may suffer a loss in profitability as a result. After all, promotions, salary increases, and bonus pools are tied to our profitability." ((( compete )))
The __________ is an itemized financial statement of the income and expenses of a company's operations. ((( profit and loss statement )))
To be effective, control systems must ((( use multiple approaches. )))
__________ motivates people and provides information that enables them to correct their own deviations from performance standards. ((( Feedback )))
A __________ refers to a control system combining four sets of performance measures: financial, customer, business process, and learning and growth. ((( balanced scorecard )))
The most effective standards are expressed in __________ terms; they are objective more than subjective. ((( quantitative )))
"Well, no wonder our customers are leaving us!" said the CEO of Performance Sports. "For the past year, our control system has measured and reported the days between order receipt and order shipped, rather than the days between order receipt and date requested by our customers. What we are measuring may make us look and feel good, but we are measuring the wrong thing!" Performance Sports is committing the "deadly sin" of performance measurement known as ((( vanity. )))
Cole, an outside salesman for Rainforest Products, angrily told his supervisor, "I received a memo from our accounting department that, effective immediately, all employees and their managers will receive a monthly report measuring each employee's days-in-office statistics. And over the next 12 months, the days-in-office metric is expected to increase by 50 percent to reduce our firm's travel costs. How can I do outside sales, when I am expected to spend more days inside the office?" Cole is expressing a view that his firm is committing the "deadly sin" of __________ by measuring days-in-office for outside sales people. ((( inanity )))
Market controls use __________ to regulate performance. ((( economic forces )))
Using market control mechanisms is criticized for ((( not adequately reflecting organizations' total value. )))
A(n) __________ price is internally charged by one organizational unit for a good or service that it supplies to another unit. ((( transfer )))
Traditionally, boards try to control CEO performance mainly through ((( incentive pay. )))
Empowerment is important in organizations today because ((( management has changed. )))
__________ involves creating relationships built on mutual respect and encouraging each individual to take responsibility for his or her actions. ((( Clan control )))
A practical guideline for managing in an empowered world is to ((( reinforce responsiveness and teamwork. )))
Which statement is accurate regarding managing in an empowered world? ((( Team performance and adding value to the customer should be emphasized. )))
"This is the third time that our business unit has been sold in the past five years," said Mahima. "Each time, our managers lose their jobs, and workers like us have to learn to work under new managers and new systems. Our business unit has been just a minor piece of several large diversified companies, and our performance has been compared to other business units' performance in these companies. As a result, we have been repeatedly bought and sold based on market controls, and employees often __________ as a result." ((( suffer )))
Nigel said to his manager, "Tomas, I've received an unsolicited offer from a competitor. I've enjoyed my eight years here, and I'd like to stay. But the offered salary is 20 percent higher than my current salary. The other firm cited what I have learned this past year on the public works job as a particularly valuable skill." Nigel is a highly valued employee, and Tomas has been grooming him as his successor. If Tomas were to consider market rates as an important indicator of worth in responding to Nigel, Tomas would most likely ((( seek to retain Nigel by increasing his salary by 25 percent. )))
__________ keep CEOs focused on the company's longer-term health. ((( Balanced scorecards )))
A(n) __________ is the internal charge by one organizational unit for a good or service that it supplies to another unit. ((( transfer price )))
__________ has become a necessary part of a manager's control repertoire. ((( Empowerment )))
Triton Gypsum is a corporation that produces material for wallboard and other applications. They are a hierarchical organization, with a strict chain of command culture and a strict set of rules and regulations. They track a large amount of data using statistical techniques, and they use this data to regulate behavior and results. Triton follows a __________ control system. ((( bureaucratic )))
Northeastern Health is a regional medical center. They tie pricing of services and profits to specific services in the medical center. Northeastern Health uses a __________ control system. ((( market )))
"Once per year, our CEO stations himself for an entire day on the production floor, handing out safety gear and demonstrating its proper use if workers are failing to do so," said Norm. "Boy, that really makes an impression. He ‘walks the talk' regarding our corporate value of safety." The bureaucratic control that the CEO is using is known as __________ control. ((( concurrent )))
"The new phones are really high-tech, but they are also very complicated to use," said Nadia, administrative manager for Baker Enterprises. "I will schedule several training workshops that the staff can attend, and I will teach them how to use the phones." The bureaucratic control that Nadia will use is __________ control. ((( feedforward )))
"Over the past five years, some of our new entry-level employees have been stars, some have been solid, and some have not met our expectations," said Claudia, manager of operations. "We will be gathering data on entry-level employees' performances and correlating those performances with college GPAs and college leadership positions. We will use our results to fine-tune our campus recruiting program so that we hire more star employees." The bureaucratic control that Claudia is describing is __________ control. ((( feedback )))
"I think that we can manufacture 12,000 units next year," said Maritza, the factory manager. "But I will need to buy significantly more tons of raw material and more replacement parts." The operations division manager replied, "Maritza, I'm glad to hear that. You will need to include all of these physical units in your __________ budget for next year." ((( production )))
"If top management wants us to produce a new line of lawn tractors over the next ten years, we will need to invest in another factory assembly line. And that's going to cost big bucks," said the factory manager for Home Oasis Inc. "Please start greasing the skids internally. I don't want the home-office executives to be surprised when they see high-dollar fixed assets in my __________ budget." ((( capital )))
"Why is the __________ budget for next year 30 percent higher than this year's actual expenses?" asked Pilar, CEO of Lyonnais Group. The CFO replied, "We need to hire two additional human resources professionals to oversee the new benefits program and an additional attorney to handle claims. Also, our new lease for Building A will go into effect, and that includes a 10 percent increase in the first year." Pilar replied, "Ouch. It's tough to absorb those kinds of increases from parts of our organization that generate no revenue." ((( cost )))
"All right, we have budget inputs from all six divisions. I have consolidated all of these inputs into a draft spreadsheet, and I have added other corporate budgets into the spreadsheet," said the CFO of Fresh Grove Products, to her fellow executives. "We need to review this ‘budget of budgets' and get back to the division manager with our comments. After they incorporate our comments and submit their revised budget inputs, I will consolidate the division budgets and all other budgets into a __________ budget." ((( master )))
Effective managers make sure that activities are going as planned with the help of control. ((( TRUE )))
Customer service standards and performance standards fall into the general categories of quantity, quality, cost, and time used. ((( TRUE )))
Within U.S. organizations, the standard unit of measure for the budgeting process is dollars. ((( FALSE )))
Controlling by profit and loss is most commonly used for the entire enterprise and, in the case of a diversified corporation, its divisions. ((( TRUE )))
Allowing people to initiate their own corrective action encourages self-control and reduces the need for outside supervision. ((( TRUE )))
Empowering employees to make decisions implies giving up control. ((( FALSE )))
A compliment is a potential ________ because customers purchase and use it alongside another item the company sells. ((Opportunity ))
__________ are methods for adapting the technical core of a company’s products to changes in an environment. (( Flexible Process ))
What is true of wholesalers? (((Wholesalers are extremely important because of the marketing activities they perform.)))
Many service providers are considered retailers because they (((provide their services directly to consumers)))
A merger occurs when (((two companies combine to form a new company)))
A(n) ________ is a partnership established for a specific project or for a limited time. (((joint venture)))
An entrepreneur has been primarily associated with the willingness to((take risks.))
What is one of the difficulties faced by small business owners?((worries about employee problems))
What is a difference between high technology businesses and other small businesses?((High technology businesses require greater capital and have higher initial startup costs thanother small businesses))
What is true of a capitalist economic system?((Prices of goods and services are determined by supply and demand))
The quantity of goods and services that consumers are willing to buy at different prices at a specific time is referred to as((demand.))
The quantity of products that businesses are willing to sell at different prices at a specific time is referred to as((supply.))
What is a defining characteristic of public corporations?((Their stock can be bought, sold, or traded by anyone))
Which business provides a service, but is neither owned by the government nor focuses on earning profits?((a nonprofit corporation))
What is a true statement about the board members of a corporation?((They have a duty of care and loyalty to oversee the management of the firm.))
Preferred stockholders of a corporation((have a claim to profits before other stockholders do.))
A leadership scholar asserts that “Executives are given subordinates; they have to __________ followers.” ((earn))
A mission statement describes the organization’s (( basic purpose. ))
1. Given that OB experts have been accumulating a distinct knowledge about behavior within organizations, OB is referred to as a field of study. (((1. True)))
10. According to the systems perspective, most organizations have one working part rather than many sub-components. (((10. False)))
100. Employee behaviors that extend beyond normal job duties: (((100. are called organizational citizenship behaviors.)))
101. Sabotage, threatening harm, and insulting others represent: (((101. three forms of counterproductive work behaviors.)))
102. Which of the following is considered a counterproductive work behavior? (((102. All of the above)))
103. If Dave's employees quit their jobs, according to research, the main reason why they quit their jobs may be that: (((103. they are dissatisfied with the job or work context.)))
104. Which of the following is NOT a work-related behavior? (((104. Competencies)))
105. Which of the following would be considered a work-related behavior? (((105. All of the above.)))
106. Generous sick leave policies are known to: (((106. increase absenteeism.)))
107. Which of these statements about globalization and organizational behavior is TRUE? (((107. Globalization gives rise to the question of how corporate leaders and employees can work effectively in the global workplace.)))
108. Globalization occurs when an organization: (((108. increases its connectivity with people and organizations in other parts of the world.)))
109. _______ refers to economic, social, and cultural connectivity with people in other parts of the world. (((109. Globalization)))
11. The best organizational practices are those built on the notion that organizations are closed systems. (((11. False)))
110. Which of these represent the one-third of the American population? (((110. Hispanics)))
111. Workforce diversity: (((111. all of the above.)))
112. Which of the following statements about America's population and workforce is FALSE? (((112. Within the next decade, Asian-Americans will replace African-Americans as the second largest ethnic group.)))
113. Which of the following statements is FALSE? (((113. Employment relationships are shifting towards the idea that companies must provide employees a high degree of job security, possibly even a job for life.)))
114. According to research, telecommuting offers all of these benefits EXCEPT: (((114. enhance employee recognition.)))
115. Which discipline has provided organizational behavior with much of its theoretical foundation for team dynamics, organizational power, and organizational socialization? (((115. Sociology)))
116. Which of the following is NOT a conceptual anchor in organizational behavior? (((116. Economic anchor)))
117. Which of the following does NOT represent a belief that anchors organizational behavior? (((117. OB should view organizations as closed systems.)))
118. Which of these statements is consistent with the five anchors of organizational behavior? (((118. The field of organizational behavior should rely on other disciplines for some of its theory development.)))
119. To collect and analyze information systematically, organizational behavior researchers rely on: (((119. both 'A' and 'C'.)))
12. The open systems perspective emphasizes that organizations survive by adapting to changes in the external environment. (((12. True)))
120. The contingency anchor of organizational behavior states that: (((120. a particular action may have different consequences in different situations.)))
121. According to the multiple levels of analysis anchor: (((121. OB topics typically relate to the individual, team and organizational levels of analysis.)))
122. To help Allison understand some preliminary information about OB, which of these statements about the field is FALSE? (((122. Given the specific utility of the field, OB is useful for the managers in the organizations and not the employees.)))
123. Allison thinks that organization is the same as an organism. However, in the field of organizational behavior, organizations are best described as: (((123. groups of people who work interdependently towards some purpose.)))
124. Allison needs to know that according to the authors of the text, organizational behavior knowledge: (((124. is relevant to everyone who works in organizations.)))
125. Bob has been interested in this trend of globalization. He should know that globalization occurs when an organization: (((125. increases its connectivity with people and organizations in other parts of the world.)))
126. In collecting his information on OB trends, Bob should be told that all of these statements about America's population and workforce are true EXCEPT which one? (((126. Within the next decade, Asian-Americans will replace African-Americans as the second largest ethnic group.)))
127. If Bob wants to consider deep-level diversity he would (((127. look at different attitudes and expectations of his employees.)))
128. Bob has never liked the idea of telecommuting for employees of Tricky Toys. According to research, telecommuting offers all of these benefits EXCEPT: (((128. enhanced employee recognition.)))
129. All of the following are conceptual anchors in organizational behavior EXCEPT: (((129. Societal level anchor.)))
13. One of the fastest ways to acquire knowledge is to hire individuals or purchase entire companies that have valued knowledge. (((13. True)))
130. Which of the following does NOT represent a belief that anchors organizational behavior? (((130. OB should view organizations as closed systems.)))
131. Which of these statements is consistent with the four anchors of organizational behavior? (((131. The field of organizational behavior should rely on other disciplines for some of its theory development.)))
132. To collect and analyze information systematically, organizational behavior researchers rely on: (((132. both 'A' and 'C'.)))
133. Which of the following is NOT a conceptual anchor in organizational behavior? (((133. Organizational effectiveness anchor)))
14. Organizational effectiveness depends on the organization's capacity to acquire, share, use and store valuable knowledge. (((14. True)))
15. Intellectual capital includes, among other things, the knowledge captured in an organization's systems and structures. (((15. True)))
16. The knowledge, skills and abilities of employees is an example of structural capital. (((16. False)))
17. Intellectual capital represents the stock of knowledge held by an organization. (((17. True)))
18. Structural capital includes knowledge embedded in the organization's systems and structures. (((18. True)))
19. One way organizations can retain intellectual capital is to have high employee turn around. (((19. False)))
2. In order for something to be called an organization it must have buildings and equipment. (((2. False)))
20. A perspective that effective organizations incorporate several workplace practices that leverage the potential of human capital is called high-performance work practices. (((20. True)))
21. One widely mentioned high-performance work practice is employee involvement. (((21. True)))
22. Working on a self-directed team actually reduces employee motivation because employees feel they have no direction. (((22. False)))
23. Organizations are more effective when they invest in employee skills and knowledge development. (((23. True)))
24. Stakeholders are shareholders, customers, suppliers, governments and any other groups with a vested interest in the organization. (((24. True)))
25. Values represent an individual's short-term beliefs about what will happen in the future. (((25. False)))
26. Values guide an individual but are not an important subject within an organization. (((26. False)))
27. Ethics refers to the study of moral principles or values that determine whether actions are right or wrong and outcomes are good or bad. (((27. True)))
28. The "triple bottom line" philosophy says that in addition to their own profitability, successful organizations focus on the financial performance of their suppliers and customers. (((28. False)))
29. There are five individual-level dependent variables found in most OB research. (((29. True)))
3. All organizations have a collective sense of purpose, whether it's producing oil or creating the fastest Internet search engine. (((3. True)))
30. Task performance refers to goal-directed behaviors under the individual's control that support organizational objectives. (((30. True)))
31. One example of organizational citizenship behaviors is assisting co-workers with their work problems. (((31. True)))
32. Counterproductive work behaviors are voluntary. (((32. True)))
33. Companies thrive when employees with talent and potential leave the company. (((33. False)))
34. Dark-side workplace behaviors are collectively known as counterproductive work behaviors (CWBs). (((34. True)))
35. Counterproductive work behaviors include threats and work avoidance. (((35. True)))
36. Joining the organization is a type of work-related behavior. (((36. True)))
37. Absenteeism is lower in organizations with generous sick leave. (((37. False)))
38. Organizations need to anticipate and adjust to environmental changes in order to fit in their environment. (((38. True)))
39. Employees who experience job dissatisfaction or work-related stress are more likely to be absent or late for work. (((39. True)))
4. Social entities are called organizations only when their members have complete agreement on the goals they want to achieve. (((4. False)))
40. Globalization may have both positive and negative implications for people working in organizations. (((40. True)))
41. Globalization refers to economic, social, and cultural connectivity with people in other parts of the world. (((41. True)))
42. Reduced job security and increased work intensification are partly caused by globalization. (((42. True)))
43. Deep level diversity is the observable demographics such as age, gender and race. (((43. False)))
44. People born between 1946 and 1964 are referred to as Generation X employees. (((44. False)))
45. Research indicates that Baby Boomers and Generation-X employees bring the same values and expectations to the workplace. (((45. False)))
46. Workforce diversity potentially improves decision making and team performance on complex tasks. (((46. True)))
47. Work/life balance refers to minimizing conflict between work and non-work demands. (((47. True)))
48. According to research, although telecommuting significantly increases employee stress and reduces productivity and job satisfaction, it makes employees feel more empowered. (((48. False)))
49. Most organizational behavior theories have been developed by OB scholars rather than from other disciplines. (((49. False)))
5. Accounting, OB, and Marketing are three business fields that have a clearly-defined career path. (((5. False)))
50. Psychology and sociology have contributed many theories and concepts to the field of organizational behavior. (((50. True)))
51. Communications and information systems are two emerging fields from which organizational behavior is now acquiring knowledge. (((51. True)))
52. Marketing is one of the disciplines that have not made any contribution to organizational behavior knowledge. (((52. False)))
53. The field of organizational behavior relies on qualitative rather than quantitative research to understand organizational phenomena. (((53. False)))
54. The systematic research anchor relies mainly on qualitative data and subjective procedures to test hypothesis. (((54. False)))
55. The contingency anchor in organizational behavior suggests that we need to diagnose the situation to identify the most appropriate action under those specific circumstances. (((55. True)))
56. Most organizational events may be studied from all three levels of analysis: individual, team and organization. (((56. True)))
57. _____ is the study of what people think, feel, and do in and around organizations. (((57. OB)))
58. Which of these statements about the field of organizational behavior is FALSE? (((58. Given the specific utility of the field, OB is useful for the managers in the organizations and not the employees.)))
59. Which of these statements about the field of organizational behavior is TRUE?(((59. Organizational behavior emerged as a distinct field during the 1940s.)))
6. An important principle in organizational behavior is that OB theories should never be used to predict or influence organizational events. (((6. False)))
60. Which of the following statements about the field of organizational behavior is FALSE? (((60. OB is a self-contained discipline, independent of other disciplines.)))
61. Organizational behavior knowledge: (((61. helps us to understand, predict, and influence the behaviors of others in organizational settings.)))
62. In the field of organizational behavior, organizations are best described as: (((62. groups of people who work interdependently towards some purpose.)))
63. Which of these describes groups of people who work interdependently towards some purpose? (((63. Organizations)))
64. According to the authors of your text, organizational behavior knowledge: (((64. is relevant to everyone who works in organizations.)))
65. Which of these refers to the perspective that companies take their sustenance from the environment and, in turn, affect that environment through their outputs? (((65. Open system)))
66. Organizational behavior views organizations as: (((66. open systems.)))
67. The open systems anchor of organizational behavior states that: (((67. organizations affect and are affected by their external environments.)))
68. ACME Software Inc. has developed a training program to make employees more aware of how their job performance affects customers and other employees within the organization. This training program relates most closely with which of the following concepts? (((68. Open systems)))
69. Which of the following relates to the idea that organizations are open systems? (((69. All of the above.)))
7. Everyone is a manager. (((7. True)))
70. From the open systems view of the organizations, which of these is NOT an input? (((70. Profits)))
71. Which organizational behavior perspective discusses inputs, outputs, and feedback? (((71. Open systems)))
72. Knowledge management is an extension of: (((72. the open systems perspective of organizational behavior.)))
73. Which of the following is a form of knowledge acquisition? (((73. Experimentation)))
74. As part of the knowledge management process, experimentation is conducive to: (((74. knowledge acquisition.)))
75. Eastern University performs a daily computer search through newspaper articles to identify any articles about the university or its faculty members. University administrators use this information to receive feedback about how the public reacts to university activities. In knowledge management, searching for newspaper articles and other external writing about the organization is mainly a form of: (((75. knowledge acquisition.)))
76. Twice each year, a major car parts manufacturer brings together production and engineering specialists from its eight divisions to discuss ideas, solutions, and concerns. This helps to minimize the 'silos of knowledge' problem that exists in many organizations. This practice is primarily an example of: (((76. knowledge sharing.)))
77. Companies 'manage' knowledge by: (((77. all of the above.)))
78. Organizations retain intellectual capital by: (((78. transferring employee capital into structural capital.)))
79. Intellectual capital refers to: (((79. the stock knowledge that resides in an organization.)))
8. Three challenges organizations are facing are globalization, increasing workforce diversity and emerging employment relationships. (((8. True)))
80. Intellectual capital consists of: (((80. all of the above.)))
81. A computer maintenance company wants to 'capture' the knowledge that employees carry around in their heads by creating a database where employees document their solutions to unusual maintenance problems. This practice tries to: (((81. transfer human capital into structural capital.)))
82. Intellectual capital is: (((82. the company's stock of knowledge.)))
83. Which of the following typically results in a loss of intellectual capital? (((83. All of the above.)))
84. The perspective that effective organizations incorporate several workplace practices that leverage the potential of human capital is called (((84. HPWP.)))
85. Which of the following statements is a proposition of high-performance work practices? (((85. Human capital is rare.)))
86. Stakeholders include: (((86. all of the above.)))
87. Employees, suppliers and governments: (((87. are organizational stakeholders.)))
88. Stable, long-lasting beliefs about what is important in a variety of situations are: (((88. called values.)))
89. The topic of ethics is most closely associated with: (((89. values.)))
9. Open system organizations are unable to maintain a close alignment of the organization's systems with the external environment. (((9. False)))
90. ______ refers to the study of moral principles or values that determine whether actions are right or wrong and outcomes are good or bad. (((90. Ethics)))
91. Corporate social responsibility is most closely related to which of these organizational behavior trends? (((91. Workplace values and ethics)))
92. ______ refers to an organization's moral obligation toward all of its stakeholders. (((92. CSR)))
93. The triple bottom line philosophy says that: (((93. companies should try to support the economic, social, and environmental spheres of sustainability.)))
94. Which of the following concepts are closely associated with corporate social responsibility? (((94. Both 'B' and 'C')))
95. Which of the following is an example of an organizational citizenship behavior? (((95. cooperation toward the organization)))
96. __________ refers to goal-directed behaviors under the individual's control that support organizational objectives. (((96. Task performance)))
97. Which of the following refers to goal-directed activities under the individual's control that support organizational objectives? (((97. Task performance)))
98. Showing up late to work or not showing up at all represent: (((98. forms of counterproductive work behaviors.)))
99. Organizational citizenship refers to: (((99. employee behaviors that extend beyond normal job duties.)))
1. Conflict begins whenever both parties realize that they have opposing interests. (((1. False)))
10. Constructive conflict helps people to recognize problems, identify a variety of solutions and better understand the issues involved. (((10. True)))
100. Direct communication minimizes conflict by: (((100. doing only 'B' and 'C'.)))
101. Dialogue potentially reduces conflict by: (((101. improving communication and understanding between the parties.)))
102. According to your text, compared to those with individualist cultures, employees in collectivist and high power distance cultures tend to be: (((102. less comfortable with the practice of resolving differences through direct and open communication.)))
103. Increasing inventories between sequential work units tends to reduce potential conflict by: (((103. reducing task interdependence.)))
104. A major game software firm experienced conflict between the young Internet-savvy employees who designed the games and the older computer-illiterate executives who ran the company. The company hired team leaders who could work with the executives but were computer literate, and as a result were respected by the development staff members. By hiring these team leaders as liaisons between the executives and employees, the company would: (((104. reduce conflict by reducing task interdependence.)))
105. The bargaining zone model states that: (((105. the negotiation process moves each party along a continuum with an area of potential overlap.)))
106. The bargaining zone model identifies: (((106. the relative locations of various bargaining positions between the parties.)))
107. Initial, target, and resistance represent three elements in: (((107. the bargaining zone model.)))
108. Negotiators implicitly or explicitly consider three bargaining positions. These positions include: (((108. initial, target, and resistance.)))
109. In a purely win-lose situation, the bargaining zone states that the opposing parties: (((109. try to discover the other side's resistance point so they can determine how much they can gain without breaking off negotiations.)))
11. Constructive conflict tests the logic of arguments and encourages participants to re-examine their basic assumptions. (((11. True)))
110. Which of these conditions is most likely to weaken your strength in a win-lose bargaining situation? (((110. You have to complete negotiations quickly to meet your time deadline.)))
111. In the context of negotiations, which of the following statements about time deadlines and time is FALSE? (((111. Negotiators become less committed to reaching an agreement as they devote more time to it.)))
112. Which of the following tends to make negotiators more competitive and less willing to give concessions? (((112. The audiences of each party directly observe the negotiations.)))
113. When negotiators have an audience watching their progress in the negotiations, the audience's negotiator: (((113. pays more attention to saving face.)))
114. Negotiators tend to be more effective when: (((114. they discuss issues rather than people AND spend a lot of time clarifying and asking questions about the other party's offer.)))
115. Effective negotiators: (((115. set negotiation goals for themselves.)))
116. What effect does making concessions have on negotiations? (((116. All of the above.)))
117. Which of the following is a third-party conflict resolution strategy with high process control and high outcome control? (((117. Inquisition)))
118. Third-party interventions are defined in terms of their: (((118. level of process control and level of decision control.)))
119. Which of the following is a third-party conflict resolution strategy with low process control and low outcome control? (((119. None of these has low process control and low outcome control.)))
12. Relationship conflict refers to conflict between individuals, whereas constructive conflict refers to conflict between departments and organizations. (((12. False)))
120. Which third-party conflict resolution strategy manages the process and context of interaction between the disputing parties but does not impose a solution on the parties? (((120. Mediation)))
121. Managers tend to prefer which of the following types of third-party intervention? (((121. Inquisitor)))
122. The Third Party Conflict Resolution intervention that has a high level of control over the final outcome but little control over the process is known as: (((122. arbitration.)))
123. Two team members who disagree over their work schedule approach their team leader to help resolve the conflict. Based on your knowledge of third-party conflict resolution, the team leader should: (((123. help the employees resolve their own conflict with the team leader serving as a mediator to help them clarify their differences and common interests in the dispute.)))
124. For everyday disputes between two employees, managers should use ______ as a third-party intervention. (((124. mediation)))
125. If the parties cannot resolve their differences through direct dialogue and negotiation, the process that seems to work best is to: (((125. arbitrate the dispute.)))
126. In the conflict process, ________ occurs immediately before conflict outcomes. (((126. manifest conflict)))
127. In the conflict process, conflict sources lead to: (((127. Both 'B' and 'D'.)))
128. According to the conflict process, which of these manifests conflict? (((128. All of the above)))
129. According to the conflict process, which of these is a positive outcome? (((129. Cohesiveness)))
13. When people experience relationship conflict, they tend to increase their reliance on communication with the other party. (((13. False)))
130. The conflict at NewTel appears to be: (((130. relationship conflict.)))
131. Which of these is NOT a strategy identified by the conflict management experts that might reduce the level of conflict at NewTel? (((131. Third-party conflict resolution)))
132. The conflict at NewTel concerning the four new hires' attempts to have the company put more money into marketing and customer services instead of technology investment is mainly an example of conflict due to: (((132. goal incompatibility.)))
133. Conflict at NewTel appears to have all of these sources EXCEPT: (((133. ambiguity.)))
134. The conflict at ABC International appears to be: (((134. relationship conflict.)))
135. Management at ABC International appears to prefer which of the following types of third-party intervention? (((135. Inquisitor)))
136. Employees at ABC International appear to prefer which of the following types of third-party intervention? (((136. Mediator)))
137. Generally speaking, which of these is suggested as the best type of third-party intervention? (((137. Mediator)))
14. When conflict reduces each side's motivation to communicate, they rely more on stereotypes to reinforce their perceptions of the other side. (((14. True)))
15. Most people can avoid experiencing relationship conflict if they focus on constructive debate instead. (((15. False)))
16. Although constructive conflict can degenerate into relationship conflict, it is easy to prevent this from happening. (((16. False)))
17. Emotional intelligence helps employees to avoid escalating constructive conflict into relationship conflict. (((17. True)))
18. Constructive conflict is more likely to escalate into relationship conflict among employees in highly cohesive teams. (((18. False)))
19. Some team norms encourage tactics that diffuse relationship conflict when it first appears. (((19. True)))
2. Conflict occurs when one party perceives that its interests are being opposed or negatively affected by another party. (((2. True)))
20. Conflict is sometimes apparent by the style each side uses to resolve the conflict. (((20. True)))
21. The conflict process is really a series of episodes that potentially link together into an escalation cycle. (((21. True)))
22. Two employees from different departments who are evaluated on different performance criteria and compensated based on different reward systems are likely to experience conflict due to goal incompatibility. (((22. True)))
23. Mergers and acquisitions tend to increase conflict due to different values and beliefs. (((23. True)))
24. Poor work ethic is one of the main sources of conflict between Baby-boomer and Generation-X employees. (((24. False)))
25. It is possible to agree on a common goal yet still experience conflict due to differentiation. (((25. True)))
26. Conflict is more likely to occur between two departments with pooled interdependence than reciprocal interdependence. (((26. False)))
27. Conflict may occur because the conflicting parties lack the opportunity, ability and/or motivation to communicate with each other. (((27. True)))
28. Communication problems partly explain why conflict is more likely to occur in a multicultural workforce. (((28. True)))
29. Conflict tends to escalate more with a win-lose than with a win-win orientation. (((29. True)))
3. Conflict begins when two parties experience manifest conflict. (((3. False)))
30. The problem-solving conflict management style involves a high degree of assertiveness and cooperativeness. (((30. True)))
31. The five interpersonal conflict management styles are distinguished by their level of interdependence with the other party. (((31. False)))
32. Avoiding is usually the best interpersonal conflict management style when the issue is trivial to everyone involved. (((32. True)))
33. Although forcing is one of five interpersonal styles of conflict management, it should never actually be used to manage conflict. (((33. False)))
34. The yielding conflict management style is preferred when both parties have equal power and enough trust to share information. (((34. False)))
35. The preferred conflict management style tends to depend partly on the individual's cross-cultural values. (((35. True)))
36. People from collectivist cultures tend to apply a forcing style of conflict with colleagues more than do people from individualist cultures. (((36. False)))
37. The yielding conflict management style can produce more conflict rather than resolve it. (((37. True)))
38. Resolving differences with the opposing party through direct communication is not as comfortably applied in collectivist cultures. (((38. True)))
39. Communication and understanding interventions should be applied only after differentiation between the parties has been reduced. (((39. True)))
4. Conflict perceptions and emotions lead to conflict sources. (((4. False)))
40. Buffers tend to resolve conflict by reducing the level of interdependence between the conflicting parties. (((40. True)))
41. Increasing resources and creating more precise rules for the allocation of those resources represent two ways to increase conflict. (((41. False)))
42. Conflicts can be minimized by negotiating a collective agreement between union and management which both parties must follow. (((42. True)))
43. Negotiation occurs whenever two or more conflicting parties try to redefine the terms of their interdependence. (((43. True)))
44. Negotiators share information more fully by adopting a win-lose orientation. (((44. False)))
45. The bargaining zone model of negotiations describes the best physical zone in which negotiations should occur. (((45. False)))
46. According to the bargaining zone model, the parties should begin negotiations by describing their resistance point to each other. (((46. False)))
47. According to the bargaining zone model, the target point is the team's realistic goal or expectation for a final agreement. (((47. True)))
48. Most negotiators prefer using email, videoconferences and other forms of electronic communication when negotiating rather than meeting face-to-face. (((48. False)))
49. When negotiators get closer to their time deadline, they become less committed to resolving the conflict. (((49. False)))
5. The view that conflict in organizations is bad is overly simplistic and in some cases incorrect. (((5. True)))
50. Negotiators tend to be more competitive and less willing to give concessions when their audience directly observes the negotiations. (((50. True)))
51. Negotiators tend to avoid "hardline" behaviors when they know an audience is watching them. (((51. False)))
52. An important rule for effective negotiations is to avoid using persuasive communication tactics since this often demonstrates weakness. (((52. False)))
53. An important rule in negotiations is to make several major concessions early in the proceedings to communicate your willingness to resolve the conflict. (((53. False)))
54. In most cultures, making concessions symbolizes a negotiator's motivation to bargain in good faith. (((54. True)))
55. Arbitration has a high level of process control but a low level of decision control. (((55. False)))
56. Mediation has a high level of process control but a low level of decision control. (((56. True)))
57. Research suggests that managers tend to avoid the inquisitional approach to third-party conflict resolution. (((57. False)))
58. The preference for arbitration or mediation in work disputes partly depends on cross-cultural values. (((58. True)))
59. ADR usually begins with a meeting between employee and employer. (((59. True)))
6. Approximately 42% of a manager's time is spent dealing with workplace conflict. (((6. True)))
60. Managers should arbitrate decisions when employees cannot resolve their differences alone. (((60. True)))
61. Conflict is defined as: (((61. a process in which one party perceives that its interests are being opposed or negatively affected by another party.)))
62. The process in which one party perceives that its interests are being opposed or negatively affected by another party is called: (((62. conflict.)))
63. The current perspective on organizational conflict is that: (((63. moderate levels of conflict are necessary and produce favorable outcomes.)))
64. The two main forms of conflict are: (((64. constructive and socioemotional.)))
65. Constructive conflict exists: (((65. None of the above.)))
66. According to the literature on organizational conflict, constructive conflict: (((66. is a positive application of conflict in organizations.)))
67. Constructive conflict: (((67. encourages people to think about different points of view.)))
68. Constructive conflict: (((68. encourages employees to re-examine their basic assumptions about a problem and its possible solution.)))
69. Constructive conflict: (((69. should be encouraged for better decision making.)))
7. Moderate levels of conflict produce improved decision-making. (((7. True)))
70. A large computer company was well known for the frequent debate among employees regarding various issues. Employees weren't afraid to disagree with their colleagues and to clearly argue their point. Although the discussions would sometimes get loud, employees never spoke angrily at each other and the discussants would always end the meeting in friendship. This computer company has many instances of: (((70. constructive conflict.)))
71. Which of the following best describes relationship conflict? (((71. The conflict episodes are viewed as personal attacks rather than attempts to resolve the problem.)))
72. Relationship conflict usually causes people to: (((72. reduce communication and information sharing with the other party.)))
73. Effective managers should: (((73. encourage constructive conflict for better decision making and minimize relationship conflict.)))
74. Which of these is NOT a strategy identified by the conflict management experts as something that might reduce the level of relationship conflict during constructive conflict episodes? (((74. Third-party conflict resolution)))
75. The conflict process begins: (((75. when scarce resources, different values, or other conditions exist that potentially causes conflict.)))
76. In the conflict process, what occurs immediately before conflict outcomes? (((76. Manifest conflict)))
77. In the conflict process, what immediately follows conflict perceptions and emotions? (((77. Manifest conflict)))
78. Which of the following is not explicitly included in the conflict process model? (((78. Conflict zone)))
79. Which of these is NOT a source of conflict? (((79. Cohesiveness)))
8. If conflict was eliminated, organizations would be more productive. (((8. False)))
80. The conflict process model includes: (((80. conflict outcomes, conflict emotions, and sources of conflict.)))
81. Salespeople at Widget Co. complain that they lose sales bonuses when the production department is out of stock of a particular item. This sometimes causes customers to buy elsewhere rather than wait for the next production run. Meanwhile, production employees complain that salespeople don't appreciate the need to minimize inventory costs, for which production staff is rewarded. This is mainly an example of conflict due to: (((81. goal incompatibility.)))
82. Incompatible goals, differentiation, and ambiguous rules are: (((82. sources of potential conflict.)))
83. Employees in engineering and marketing often disagree with each other mainly because they have unique backgrounds, experiences, and training. The most apparent source of conflict here is: (((83. differentiation.)))
84. Which of the following sources of conflict is typically associated with mergers and acquisitions? (((84. Differentiation)))
85. GlobalCo formed a task force consisting of eight employees from four culturally diverse countries who have not previously met. The task force must work closely together for a month to solve a troublesome customer problem. These employees have similar technical training, but they have different religious beliefs, different languages, and different standards of living. If these employees experience conflict, this conflict might be caused by: (((85. all of the above.)))
86. Which of the following most likely produces the highest risk of conflict? (((86. Scarce resources and high task interdependence.)))
87. The potential for conflict between two employees would be highest under conditions of: (((87. reciprocal interdependence.)))
88. In organizational conflict, scarce resources are typically identified as: (((88. a source of conflict.)))
89. Communication is a potential source of conflict because: (((89. all of the above.)))
9. The modern perspective on conflict is that an optimal level exist which is beneficial to the organization. (((9. True)))
90. A win-win orientation exists when a person or team believes that: (((90. a mutually beneficial solution may be found if both parties apply a collaborative orientation.)))
91. Which of the following interpersonal conflict management styles has some degree of win-lose orientation? (((91. All of the above)))
92. The problem-solving interpersonal style of conflict has: (((92. high assertiveness and high cooperativeness)))
93. Which of the following interpersonal conflict management styles represents a purely win-win orientation? (((93. Problem solving)))
94. Which conflict management style has low cooperativeness and low assertiveness? (((94. Avoiding)))
95. Which conflict management style may be necessary when it is apparent that the other party will take advantage of information sharing and other cooperative strategies? (((95. Forcing)))
96. The yielding conflict management style should be used: (((96. when the issue is much less important to you than to the other party.)))
97. Maintenance employees and drivers at ABC Bus Company were constantly bickering with each other. However, the conflict subsided when another bus company opened for business to compete with ABC and ABC executives warned employees that the competitor could force lay-offs at ABC if productivity and customer service didn't improve. Which of the following conflict management strategies was operating here? (((97. The company introduced superordinate goals for both parties.)))
98. According to research on gender and conflict management styles, women tend to adopt the _____ style, whereas men tend to adopt the ______ style. (((98. problem-solving, forcing)))
99. Which of the following actions does NOT emphasize superordinate goals to reduce conflict? (((99. Hiring employees with a technical specialty and developing their career paths within that specialty.)))
1. (p. 234) Teams are groups of two or more people who have equal influence over each other regarding the team's goals and means of achieving those goals. (((1. False)))
10. (p. 235) Some informal groups exist primarily to satisfy the drive to bond. (((10. True)))
100. (p. 239) Organizational leadership, organizational structure, and reward systems are: (((100. three elements of the organizational and team environment.)))
101. (p. 241) Of what importance is task interdependence to teams or team dynamics? (((101. Jobs with high task interdependence are usually completed more effectively by teams than by individuals working alone.)))
102. (p. 242) In most situations, larger teams: (((102. consume more time coordinating their roles than do smaller teams.)))
103. (p. 242) In terms of team size, the general rule is that teams: (((103. should have the fewest number of people possible to perform the work.)))
104. (p. 241) Production employees working on an assembly line usually have which of the following types of task interdependence? (((104. Sequential interdependence)))
105. (p. 241) Two company divisions produce completely different products but must seek funding from head office for a capital expansion project. The relationship between these two divisions would be best described as: (((105. pooled interdependence.)))
106. (p. 241) Pooled interdependence is: (((106. the weakest form of interdependence other than complete independence.)))
107. (p. 241) Employees should almost always be organized into teams when they have: (((107. reciprocal interdependence.)))
108. (p. 245) "Faultlines" are more likely to occur when teams: (((108. are heterogeneous.)))
109. (p. 244) An effective team member __________ and manages the team's work so it is performed efficiently and harmoniously. (((109. coordinates)))
11. (p. 235) Social identity theory partly explains why people join informal groups. (((11. True)))
110. (p. 245) Teams with strong faultlines: (((110. experience more dysfunctional conflict within the team.)))
111. (p. 245) A diverse team is better than a homogeneous team: (((111. on complex projects and tasks requiring innovative solutions.)))
112. (p. 246) The first three stages of team development in sequential order are: (((112. forming, storming, norming.)))
113. (p. 246) Conforming, performing, and reforming are all: (((113. None of the above.)))
114. (p. 246-247) What generally occurs during the 'storming' stage of team development? (((114. Members try to assume specific responsibilities and influence the team's goals and means of goal attainment.)))
115. (p. 248) Which of these statements about team roles is FALSE? (((115. A team role is almost always assigned to the same person for the life of the team.)))
116. (p. 248) The primary objective of team building is to: (((116. accelerate the team development process.)))
117. (p. 248) Which type of common team building activity aims to improve relations among team members? (((117. Paintball wars.)))
118. (p. 248) Team building should be viewed as: (((118. None of the above.)))
119. (p. 249) How do norms affect the behavior of team members? (((119. Norms help the team regulate and guide the behaviors of its members.)))
12. (p. 235) Our desire for informal groups is mostly influenced by our drive to defend. (((12. False)))
120. (p. 249) Which of these statements about team norms is FALSE? (((120. Norms apply only to thoughts or feelings, not behaviors.)))
121. (p. 250) If a dysfunctional norm is very deeply ingrained, the best strategy is probably to: (((121. disband the group and replace it with people having more favorable norms.)))
122. (p. 250) Team cohesiveness tends to be higher: (((122. in smaller teams.)))
123. (p. 250) Team success, team size, and member similarity are three: (((123. of the main factors influencing team cohesiveness.)))
124. (p. 251) Which of the following does NOT occur as team cohesiveness increases? (((124. Team members experience more dysfunctional conflict among themselves.)))
125. (p. 251) Compared to people in low-cohesion teams, members of high-cohesion teams: (((125. are more likely to resolve conflicts swiftly and effectively.)))
126. (p. 252) Calculus, knowledge and identification are: (((126. three foundations of trust in teams.)))
127. (p. 252) Calculus-based trust: (((127. is the minimum level of trust to hold a relationship together.)))
128. (p. 252) Which foundation of trust is determined mainly by the other party's predictability? (((128. Knowledge-based)))
129. (p. 252) Liam works with four other accounting professionals as a team within one company. Liam doesn't particularly agree with many of his teammates' ideas, such as leaving work early and failing to double-check some account entries. However, he works comfortably with the group because their behavior and decisions are predictable. What foundation of trust does Liam have in this team? (((129. Knowledge-based)))
13. (p. 236) Under stressful or dangerous conditions, people are more likely to stay together than disperse, even when the other people are strangers. (((13. True)))
130. (p. 253) Employees tend to join a virtual or conventional team with: (((130. a moderate or high level of trust in their new team members.)))
131. (p. 253) Self-directed teams: (((131. have substantial autonomy over the execution of a complete task.)))
132. (p. 253) Which of the following allows employees to collectively plan, organize, and control work activities with little or no direct involvement of a higher-status supervisor? (((132. Self-directed teams)))
133. (p. 253-254) Members of self-directed teams have jobs that are: (((133. both enlarged and enriched.)))
134. (p. 255) Self-directed teams are best suited to situations where: (((134. employees perform highly interdependent tasks.)))
135. (p. 255) Virtual teams are best described as: (((135. cross-functional groups of employees that operate across space, time and organizational boundaries.)))
136. (p. 255) Two features that distinguish virtual teams from conventional teams are: (((136. lack of co-location and dependence on information technology.)))
137. (p. 255) Globalization and knowledge management have made __________ necessary for organizations to remain competitive. (((137. virtual teams)))
138. (p. 257) Production blocking and evaluation apprehension: (((138. do none of these.)))
139. (p. 257) Which of the following statements about evaluation apprehension in team settings is TRUE? (((139. Evaluation apprehension is more likely to occur when team members formally evaluate each other's performance throughout the year.)))
14. (p. 237) Teams typically provide better customer service. (((14. True)))
140. (p. 257) Groupthink is caused by: (((140. All of these factors cause groupthink.)))
141. (p. 258) Groupthink characteristics cause team members to be ___________ their decisions. (((141. highly confident in)))
142. (p. 258) Teams tend to make better decisions when: (((142. none of these conditions exist.)))
143. (p. 258) The main advantage of constructive conflict is that it: (((143. encourages team members to re-examine the assumptions and logic of their preferences in the decision.)))
144. (p. 258-259) Brainstorming requires team members to: (((144. do none of these.)))
145. (p. 260)  Which of the following explicitly encourages team members to 'piggyback' or 'hitchhike' on the ideas presented by other team members?  (((145. Brainstorming.)))
146. (p. 260) Which of these team decision-making structures explicitly discourages criticism and debate? (((146. Both brainstorming and nominal group technique explicitly discourages criticism and debate.)))
147. (p. 260) Which of the following is NOT a feature of nominal group technique? (((147. Participants openly debate and criticize ideas constructively.)))
148. (p. 260) In which decision-making structure do participants typically meet, but only interact with each other for part of the meeting? (((148. Nominal group technique.)))
15. (p. 237) Employees are more motivated in teams because they are accountable to fellow team members who also monitor their performance. (((15. True)))
16. (p. 237) Process losses are the resources expended to develop and maintain an effective team. (((16. True)))
17. (p. 237) Organizational behavior scholars have concluded that employees always work better in teams than alone. (((17. False)))
18. (p. 238) Social loafing is least common in situations where team members work alone towards a common output. (((18. False)))
19. (p. 238) Social loafing is more common among people with collectivist values. (((19. False)))
2. (p. 234) All teams exist to fulfill some purpose, either for the organization or for its members. (((2. True)))
20. (p. 237) Brooks' Law states that, "Whatever can go wrong in groups will, so one should be prepared for it." (((20. False)))
21. (p. 238) A team's effectiveness is partly measured by how well its members' needs are fulfilled. (((21. True)))
22. (p. 239) Team effectiveness refers to how well a team accomplishes its objectives for the organization, even if this undermines the team's ability to survive for future tasks. (((22. False)))
23. (p. 239) Companies with the best team dynamics are more likely to have team-based rewards and encourage interaction among team members. (((23. True)))
24. (p. 239) Teams are generally more effective when the task is complex and lacks definition. (((24. False)))
25. (p. 240-241) Teams are well suited when complex work can be divided into more specialized roles. (((25. True)))
26. (p. 241) Teams are best suited for tasks with low interdependence among team members. (((26. True)))
27. (p. 241) Reciprocal interdependence is the highest level of task interdependence in organizations. (((27. True)))
28. (p. 241) Forming, storming, and norming are the three main levels of task interdependence. (((28. False)))
29. (p. 241) Students experience pooled interdependence when they are lined up at the laser printers trying to get their assignments done just before a class deadline. (((29. True)))
3. (p. 235) All teams are groups, but some types of groups are not teams. (((3. True)))
30. (p. 242) The optimal team size exists when the team is as small as possible, yet has enough people to accomplish the task. (((30. True)))
31. (p. 242) In effective teams, each member must possess the full set of competencies to perform the team's entire task alone. (((31. False)))
32. (p. 244) Communicating, comforting and conflict resolution are task related characteristics of effective team members. (((32. False)))
33. (p. 244) The five most frequently mentioned characteristics of effective team members are communicating, comforting, conflict resolution, coordinating, and cooperating. (((33. False)))
34. (p. 245) Diverse teams take longer to work through the stages of team development than do heterogeneous teams. (((34. False)))
35. (p. 245) Diverse teams have faultlines that may split the team into subgroups along gender, ethnic or other dimensions. (((35. True)))
36. (p. 246) The norming stage of team development is marked by interpersonal conflict as team members compete for leadership and other positions on the team. (((36. False)))
37. (p. 246) The longer team members work together, the better they develop common mental models to help them complete the work together. (((37. True)))
38. (p. 246) Teams develop their first real sense of cohesion during the norming stage of team development. (((38. True)))
39. (p. 246-247) During the adjourning stage of team development, team members shift their attention away from relationships and instead focus mainly on completing the task. (((39. False)))
4. (p. 235) All groups are teams, but some types of teams are not groups. (((4. False)))
40. (p. 248) Team members typically hold one or more formal roles in the team as well as roles that they informally fulfill at various times. (((40. True)))
41. (p. 248) Team roles are typically negotiated among team members. (((41. False)))
42. (p. 248) Some team-building interventions clarify the team's performance goals and increase the team's motivation to accomplish these goals. (((42. True)))
43. (p. 248) Team building interventions often fail because many times they are offered as a three-day jump-start rather than an ongoing process. (((43. True)))
44. (p. 249) One advantage of team building activities is that they can be used as general solutions to general team problems. (((44. False)))
45. (p. 249) Norms are the informal rules and standards established by a team to regulate the behavior of its members. (((45. True)))
46. (p. 249) Team members rarely conform to team norms unless other team members apply reinforcement or punishment. (((46. False)))
47. (p. 249) The only way to alter team norms is to disband the group. (((47. False)))
48. (p. 249-250) One way to change team norms in existing teams is to explicitly discuss the counterproductive norms with team members using persuasive communication strategies. (((48. True)))
49. (p. 250) To maximize cohesiveness, the team should be as small as possible without jeopardizing its ability to accomplish the task. (((49. True)))
5. (p. 235) Team members are held together by their interdependence and need for collaboration to achieve common goals. (((5. True)))
50. (p. 250) Cohesiveness tends to be higher in teams where the office design facilitates communication among team members. (((50. True)))
51. (p. 250) Diversity among team members tends to undermine cohesion. (((51. True)))
52. (p. 250) Team cohesiveness decreases with increased interaction because there are more chances for conflicts to emerge. (((52. False)))
53. (p. 251) Highly cohesive teams invariably perform organizational objectives better than teams with moderate or low cohesiveness. (((53. False)))
54. (p. 251) When highly cohesive teams have norms that conflict with organizational goals, team performance is reduced. (((54. True)))
55. (p. 251-252) Trust occurs when we have positive expectations about another party's intentions and actions toward us in situations involving vulnerability. (((55. True)))
56. (p. 252) Calculus-based trust is based on the belief that the other party will deliver its promises because punishments would be applied if they fail to deliver those promises. (((56. True)))
57. (p. 252) Knowledge-based trust is based on the belief that the other party will deliver its promises because punishments would be applied if they fail to deliver those promises. (((57. False)))
58. (p. 252) Knowledge-based trust develops over time. (((58. True)))
59. (p. 252) Identification-based trust is the most robust or sturdy form of trust in work relationships. (((59. True)))
6. (p. 235) Teams are groups with some degree of task interdependence and a common objective. (((6. True)))
60. (p. 253) When people join teams, they usually begin with a very low level of trust in the other team members. (((60. False)))
61. (p. 253) The trust that new team members feel towards their teammates is fragile and easily weakened. (((61. True)))
62. (p. 253) Members of self-directed work teams have enriched and enlarged jobs. (((62. True)))
63. (p. 253) Self-directed work teams plan, organize, and control activities with little or no direct involvement of supervisors. (((63. True)))
64. (p. 255) In most self-directed work teams, the supervisor assigns tasks that individual team members perform. (((64. False)))
65. (p. 255) Virtual teams are usually permanent functional groups that communicate mainly through weekly face-to-face meetings. (((65. False)))
66. (p. 257) Production blocking occurs when employees are unable to complete their tasks because they spend too much time in meetings. (((66. False)))
67. (p. 257) Evaluation apprehension is most common in meetings attended by people with different levels of status or expertise. (((67. True)))
68. (p. 257) Having groupthink and evaluation apprehension are two characteristics of effective decision-making teams. (((68. False)))
69. (p. 258) One symptom of groupthink is that the team feels comfortable with risky decisions because possible weaknesses are suppressed or glossed over. (((69. True)))
7. (p. 235) Employees in a department are considered a team only when they directly interact and coordinate work activities with each other. (((7. True)))
70. (p. 258) Constructive conflict occurs when team members hold different opinions or assumptions and debate the issues through an open, healthy dialogue. (((70. True)))
71. (p. 258) An important rule in brainstorming is that no one is allowed to evaluate or criticize another team member's ideas. (((71. True)))
72. (p. 259) One of the rules of brainstorming is that no one is allowed to piggyback or build on the ideas of other team members. (((72. False)))
73. (p. 260) Electronic brainstorming significantly reduces the problem of production blocking. (((73. True)))
74. (p. 260) The nominal group technique is essentially a variation of brainstorming. (((74. True)))
75. (p. 260) The nominal group technique tends to produce more and better ideas than do traditional interacting groups. (((75. True)))
76. (p. 260) The nominal group technique removes the problems of evaluation apprehension and production blocking. (((76. False)))
77. (p. 260) The nominal group technique involves a three-stage process. (((77. True)))
78. (p. 234) Teams have which of the following features? (((78. All of the above)))
79. (p. 235) Which of the following statements about teams and groups is FALSE? (((79. Some teams exist without any goal or purpose.)))
8. (p. 235) Informal groups exist primarily to complete tasks for the organization that management doesn't know about. (((8. False)))
80. (p. 234) Departments are considered teams only when: (((80. employees directly interact with each other and coordinate work activities.)))
81. (p. 235) Some ______ are just people assembled together without any necessary ________. (((81. groups, interdependence)))
82. (p. 236) Which of these statements is TRUE? (((82. Some groups are just people assembled together in the same physical area.)))
83. (p. 235) A task force refers to: (((83. any temporary team that investigates a particular problem and typically disbands when the decision is made.)))
84. (p. 235) Royal Dutch/Shell Group formed a team to improve revenues for its service stations along major highways in Malaysia. This team, which included a service station dealer, a union truck driver and four or five marketing executives, disbanded after it had reviewed the Malaysian service stations and submitted a business plan. This team is called: (((84. a task force.)))
85. (p. 235) Which type of team or group is best known for having a champion who bootlegs people and resources to develop new products, services, or procedures? (((85. Skunkworks)))
86. (p. 235) Which of the following can be either a formal or informal group? (((86. A community of practice)))
87. (p. 235) Skunkworks teams typically have all of the following characteristics EXCEPT: (((87. they are formed when top management decides where employees should be reassigned in the organization.)))
88. (p. 235) Informal groups: (((88. All of the above.)))
89. (p. 235) ______ is one explanation of why people belong to informal groups. (((89. Social identity theory)))
9. (p. 235) Task forces are temporary groups that typically investigate a particular problem and disband when the decision is made. (((9. True)))
90. (p. 235) According to social identity theory: (((90. people define themselves by their group affiliations.)))
91. (p. 235) The drive to bond and the dynamics of social identity theory both explain why people: (((91. join informal groups.)))
92. (p. 237) In team dynamics, process losses are best described as: (((92. resources expended towards team development and maintenance.)))
93. (p. 237) Brooks's Law says that adding more people to a late software project only makes it later. This law is mainly referring to: (((93. process losses.)))
94. (p. 238) Social loafing is more likely to occur: (((94. when the task is boring.)))
95. (p. 238) Keeping the team size sufficiently small and designing tasks such that each team member's performance is measurable are two ways to: (((95. minimize social loafing.)))
96. (p. 238) Which of the following does NOT minimize social loafing? (((96. Forming larger work teams.)))
97. (p. 238) The phenomenon where people exert less effort when working in groups than when working alone is called: (((97. social loafing.)))
98. (p. 238) A team that achieves its organizational goals, satisfies member needs, and survives in its environment: (((98. has a strong communication system but inappropriate reward system.)))
99. (p. 239) All of the following organizational environment features potentially affects team effectiveness EXCEPT which one? (((99. Team size)))
Marketing is an activity that only large firms with specialized departments can execute. (((FALSE)))
The marketing plan is broken down into various components—how the product or service will be conceived or designed, how much it should cost, where and how it will be promoted, and how it will get to the consumer.(((TRUE)))
Understanding the marketplace and a customer’s needs and wants is fundamental to marketing success. (((TRUE)))
As Godiva and Hershey’s battle for chocolate lovers, they divide the population into a host of categories such as luxury versus cost-conscious and service-oriented versus self-service.(((TRUE)))
The four Ps of the marketing mix include product, promotion, planning, and place. (((FALSE)))
Supply chain management is the set of approaches and techniques that firms employ to efficiently and effectively integrate their suppliers, manufacturers, warehouses, stores, and other firms involved in the transaction into a seamless value chain in which merchandise is produced and distributed in the right quantities, to the right locations, and at the right time. (((TRUE)))
Value is what you get for what you give. (((TRUE)))
In value cocreation, the customer participates in the creation of a good or service, which provides additional value to the customer.(((TRUE)))
Marketing advises production about how much of the company’s product to make and then tells supply chain mangers when to ship it. (((TRUE)))
Entrepreneurs are people who organize, operate, and assume the risk of a business venture. (((TRUE)))
When a manufacturer sells truck and car parts to Toyota, this is an example of B2C marketing. (((FALSE)))
The power adapters Dell sells with its computers are built by small companies that specialize in power-related accessories. When Dell purchases its power adapters from these small companies, it is engaged in B2B marketing. (((TRUE)))
Garage sales and online classified ads are examples of C2C marketing. (((TRUE)))
Key to the success of many entrepreneurs is that they launch ventures that aim to satisfy unfilled needs. (((TRUE)))
HappyCow is an example of a location-based social media application.(((TRUE)))
As it relates to marketing, the trade of things of value between the buyer and the seller so that each is better off as a result is known as an exchange. (((TRUE)))
Marketing’s fundamental purpose is to create value by developing a variety of offerings that will earn income for the company. (((FALSE)))
Margo likes her morning coffee, and she always stops at Starbucks because it is on her way to work. Margo is being influenced by the place element of the marketing mix.(((TRUE)))
To build relationships, firms focus on the lifetime profitability of the relationship, not how much money is made during each transaction. (((TRUE)))
Firms have come to realize that good corporate citizenship through socially responsible actions should be a priority because it will help their bottom line in the long run.(((TRUE)))
Effectively managing supply chain relationships has a minimal effect on profitability. (((FALSE)))
A group of firms that make and deliver a given set of goods and services is known as a distribution network. (((FALSE)))
Buffalo Wild Wings suggests that its diners check in to its locations using their phones. This demonstrates the use of social media to market a product. (((TRUE)))
Approximately half of marketers use social media tools for their businesses.(((FALSE)))
Firms become value driven, in part, by focusing on the competition. (((TRUE)))
When a T-shirt manufacturer states, "We sell it only in black because that way we can buy plenty of black fabric and run our plant efficiently," its statement reflects the views that were popular in which era of the evolution of marketing? (((production-oriented)))
Marketing involves all of the following except (((production scheduling.)))
Jami sells construction equipment. Whenever she calls on her building contractor customers, she asks if they are having any problems. In doing so, Jami is addressing which of the following core aspects of marketing? (((satisfying customer needs and wants )))
Julia is considering a career in marketing. She is concerned about the image of marketers as fast-talking, high-pressure people who care only about making a sale. When reading about the core aspects of marketing, Julia is relieved to see that in marketing (((all parties to the transaction should be satisfied.)))
Xavier is analyzing potential market segments. He should carefully seek potential customers who have both an interest in his products and (((the ability to buy them. )))
When referring to "exchange," marketers are focusing on (((the trading of things of value.)))
Whenever Valerie has a new massage therapy customer, she invites the person to be on her e-mail distribution list. In the process, in addition to exchanging her massage therapy service for payment, Valerie is gathering (((information.)))
Which of the following is a core aspect of marketing? (((making product, place, promotion, and price decisions )))
Marketing has traditionally been divided into a set of four interrelated decisions known as the marketing mix, or four Ps, including all of the following except (((performance.)))
The four Ps make up the marketing mix, which is the __________ set of decisions or activities that the firm uses to respond to the wants of its target markets.(((controllable )))
The fundamental goal of marketers when creating goods, services, or combinations of both is to (((create value.)))
Brian is struggling with the choice of publishing his new book, How to Cook Polish Barbeque, as an e-book or a paperback. Brian is addressing which core marketing aspect?(((making product decisions)))
The basic difference between a good and a service is that a good (((can be physically touched. )))
Four Winds Art Gallery recently began offering appraisals of customers’ art collections, in addition to continuing to sell paintings. Four Winds is (((expanding from offering just goods to also offering services.)))
Marketers must determine the price of a product carefully, based on potential buyers’ beliefs about (((its value.)))
Some discount stores put products in large bins and let consumers hunt and find bargains. The price these consumers pay includes (((the value of their time and energy. )))
Henriette offers financial counseling and management on a fee-only basis. She has found that different customers are willing to pay different rates for her services. This shows that her pricing decisions should depend primarily on (((how different customers perceive the value of her services.)))
Marketing channel management is also known as (((supply chain management.)))
___________ represents all the activities necessary to get the product to the right customer when that customer wants it. (((Place)))
Not only does GIS technology help Starbucks determine the ideal locations for new stores, but it also can enable the company to decide (((which kinds of stores to open. )))
The marketing goal of getting the "right quantities to the right locations, at the right time" relates to(((managing the supply chain. )))
Value-oriented marketers constantly measure(((perceived customer benefits against the costs of their offerings. )))
Many marketing students initially overlook the importance of marketing channel management because (((many of these activities take place behind the scenes. )))
Which element of the marketing mix deals with supply chain management? (((place)))
UPS, FedEx, DHL, and other shipping companies support other firms’ __________ marketing goals. (((supply chain management )))
__________ is communication by a marketer that informs, persuades, and reminds potential buyers about a product or service so as to influence their opinions and elicit a response. (((Promotion )))
Effective promotion enhances a product or service’s (((perceived value. )))
By promoting perfume based on youth, style, and sex appeal, Calvin Klein is attempting to (((increase the perceived value of its products.)))
When retailers accumulate merchandise from producers in large amounts and sell to consumers in smaller amounts it is considered _______ marketing.(((B2C)))
By allowing consumers to sell their unwanted goods to other consumers, auction sites like eBay and Etsy cater to __________ marketing. (((C2C)))
As use of the Internet took off, car manufacturers were tempted to sell directly to consumers, but decided instead to continue to sell through their existing dealer networks. The car manufacturers considered switching from __________ to __________ marketing. (((B2B; B2C)))
Many universities provide physical or electronic bulletin boards to facilitate ride-sharing and exchange of used books among students. These bulletin boards increase __________ marketing. (((C2C)))
The "Milk Life" advertising campaign, designed to increase awareness of the health benefits of milk, was intended to help market a(n) (((industry.)))
The “onboard, online retail business” refers to (((the options airlines offer passengers to shop through their mobile devices while they fly toward their destination.)))
The idea that a good product will sell itself is associated with the __________ era of marketing. (((production-oriented )))
Henry Ford’s statement, "Customers can have any color they want so long as it’s black," typified the __________ era of marketing. (((production-oriented)))
Melanie works for a small computer software company. Her boss is constantly improving the company’s products but neglects customers, billing, and promoting the company. Her boss is probably stuck in the __________ era of marketing. (((production-oriented)))
During the __________ era, firms had excess capacity and used personal selling and advertising to generate customers. (((sales-oriented)))
The prevailing marketing strategy of the __________ era was to find customers for inventories that went unsold. (((sales-oriented )))
Near the end of the model year, the Move-Them-Out automobile dealership had an unusually high inventory level. The manager increased her advertising spending and gave extra incentives to its salespeople. Move-Them-Out operates as if it were in the __________ era. (((sales-oriented)))
Many U.S. companies first discovered marketing during the __________ era. (((market-oriented)))
Which of the following statements reflects the philosophy of the market-oriented era?(((The customer is king. )))
Data about how, when, why, where, and what people buy refers to(((marketing analytics. )))
During the __________ era, manufacturers and retailers recognized they needed to give their customers greater value than their competitors did. (((value-based marketing )))
Creative locations, such as kiosks at the baggage claim in airports or small booths in grocery stores, represent Starbucks’ effort to improve its offering on which dimension of the marketing mix? (((place)))
Trey sells consumer electronics. He knows his customers weigh the costs versus the benefits associated with the different options available. He decides which products to offer and what prices to charge based on the way his customers think. Trey operates as if he were in the __________ era. (((value-based marketing)))
Serena studies her customer profiles, market research data, complaints, and other information, attempting to better understand what her customers want. Serena most likely operates in the __________ era of marketing. (((value-based marketing)))
In delivering value, marketing firms attempt to find the most desirable balance between (((the need to provide benefits to customers and keep down costs. )))
Yolanda is the new restaurant manager in a major hotel. When considering changes in the restaurant that will increase value to customers, Yolanda will likely attempt to either provide the same quality at a lower cost or (((improve products and services at the same cost. )))
Christie has just started with a travel agency, and she has been offering clients and prospective clients a range of packaged tours. She is concerned because the commissions she is earning on her sales are lower than she had hoped. Her colleague Peter, who has been with the agency for several years, is having a great deal of success by working closely with the clients, seeking their ideas, and building customized tour packages for each one based on their suggestions. Peter’s approach is based on (((value cocreation.)))
To become a more value-driven organization, Pokrah University is holding regular coffee-hour discussions with its students and surveying its graduates regarding students’ educational needs and desires. By doing so, Pokrah University is becoming more value driven through (((building relationships with customers. )))
As owner of a retail franchise food store, Mary Gray purchases supplies based on specials advertised nationally throughout the franchise system. One Monday, she was surprised to find customers asking for specials she hadn’t been informed of in advance. The franchise company failed to live up to the value-driven activity of (((sharing information across the organization. )))
In the past, manufacturer’s representatives did not have real-time inventory data about the products they were selling. Today, manufacturer’s representatives are often provided online access to inventory data for the companies they represent. These online inventory systems allow companies to become more value driven by (((sharing information across the organization. )))
To build relationships, firms focus on the ___________ of the relationship, not how much money is made during each transaction. (((lifetime profitability )))
One of the benefits of value-driven marketing is that attention to customer needs and wants will likely result in (((long-term relationships. )))
Even though they operate from out-of-the-way airports and offer few extra services, discount, no-frill airlines like Ryanair and EasyJet have been successful. Consumers obviously consider (((the benefit of lower prices to be greater than the cost of reduced services and less convenience. )))
A relational orientation is based on the philosophy that buyers and sellers develop (((a long-term relationship. )))
A local art gallery keeps information on its customers regarding their preferences for certain artists as well as the style of art that interests them. The gallery uses this information to inform the customers when new products arrive from their favorite artists and targets them with special promotions. In this way, the gallery is using _______ to build loyalty among its customers. (((customer relationship management )))
If a firm adopts a CRM business philosophy, it most likely has a(n) _______ orientation with its customers. (((relational )))
After major hurricanes like Katrina, many ethical home repair and building supply businesses continue to charge pre-hurricane prices to their customers, even though due to the huge increase in demand they could charge much more. These firms probably recognize that (((lifetime profitability of relationships matters more than profits from a particular transaction. )))
What is the focus of a firm that develops a relational orientation with its customers? (((gaining lifetime profitability from customer relationships)))
The goal of customer relationship management is to (((identify and build loyalty among a firm’s most valued customers. )))
Franco uses a database software system to remind him when his customers should be ready to reorder his industrial cleaning products. With this reminder system, Franco contacts his customers when they are most likely to be "in the buying mode." Franco’s system is part of (((customer relationship management.)))
Many catalog companies create special-run issues based on what customers have purchased in the past. For example, customers who frequently order bedding items like sheets and pillows receive a catalog with a larger section of bedding items than do customers who mostly order kitchen tools. This is an example of (((customer relationship management. )))
It was during the market-oriented era that firms first discovered "marketing." In what timeframe did this occur? (((just after World War II )))
Many inventors struggle with the question, "I made it; now how do I get rid of it?" They have made the error of considering marketing as (((an afterthought. )))
Georgia, the outside sales rep for a major building supply company, reads a report stating that building permits are down dramatically in her sales territory. She had noticed that things were slowing down, but now she has data confirming her impression. Based on this information, Georgia will help her company become more value driven if she(((advises the production and purchasing departments to produce or order smaller quantities of products. )))
Jenny, the delivery and sales representative for a beer distributor, is calling on a retailer and sees the shelves are almost empty. An unexpected sporting event held nearby resulted in a huge increase in sales. She calls her company’s distribution manager and requests a special delivery for her customer. Jenny is providing the important marketing function of (((alerting the logistics department when to ship products. )))
After the previous sales representative in his territory infuriated an important customer, Benjamin visited the customer once a month, never asking for business but hoping to rebuild trust through listening and expressing concern. Finally, after more than two years, the customer gave Benjamin an order. Benjamin was providing the important marketing function of (((engaging customers and developing long-term relationships. )))
Which of the following best represents how marketing expands firms’ global presence? (((when companies manufacture and sell their products all over the world )))
Leah is the marketing manager for an electronics company. While on vacation in Ecuador, she visited electronics stores in the major malls in Quito, the capital city. Most of her company’s products were available, except for smartphones. When she returned to work, she mentioned this observation to her international sales manager. Leah was providing the important marketing function of (((identifying opportunities to expand.)))
Greenbelt Construction has been a successful small home-building firm for years. The owner pays subcontractors slightly more than the usual rate for different tasks, reducing the company’s gross margin. Greenbelt rarely changes subcontractors, has relatively few complaints from home buyers, and is able to get quick responses from subcontractors when buyers do have problems. Greenbelt is engaged in (((effective supply chain management. )))
Marketing enriches society by (((sponsoring charitable events. )))
A friend of yours is starting a new business. She is confident that her product is superior to her competitors’ and doesn’t want to budget any money for marketing. You would recommend that she rethink her position on marketing for all of the following reasons except (((Marketing isn’t essential now, but it will be in a year or two when the product takes off.)))
When we say that marketing can be entrepreneurial, it means that entrepreneurs (((launch ventures to satisfy unfilled needs. )))
People who organize, operate, and assume the risk of a business venture are called (((entrepreneurs.)))
If a radio station holds an online contest in which you must log in to its website and submit personal details such as name, phone number, and e-mail in order to participate, the radio station is (((offering an exchange. )))
Which of the following is true of marketing? (((Marketing affects various stakeholders. )))
Traditionally, marketing activities have been divided into product, price, place, and promotion. Select the term that best describes the four Ps. (((marketing mix)))
Internet sites, physical stores, and kiosks are most closely associated with which element of the marketing mix? (((place)))
The primary purpose of the __________ plan is to specify the marketing activities for a specific period of time. (((marketing)))
The process of value __________, in which customers collaborate in product design, often provides additional value to the firm’s customers. (((cocreation )))
How a product or service will be conceived or designed, how much it should cost, where and how it will be promoted, and how it will get to the consumer are all elements of (((a marketing plan.)))
The activity, set of institutions, and processes for creating, capturing, communicating, delivering, and exchanging offerings that have value for customers, clients, partners, and society at large is called (((marketing. )))
Jeff is going to sell sporting apparel, which he has already purchased from manufacturers, and has signed a deal agreeing to the volume he will sell monthly. He has researched his competition, talked to some customers, and decided on prices he will charge. Jeff has also developed a plan for promoting his business. Based on this description, which element of the marketing mix does Jeff still need to work on? (((place)))
Jeff opened a sporting apparel store and signed a lease on the property. He also signed an agreement with the manufacturer on the amount of merchandise he will sell and the promotions he will conduct. Based on this description, which aspect of the marketing mix does he still need to work on? (((price)))
At one point in the evolution of marketing, the United States entered a buyer’s market and the customer became king. Which era is being described? (((market-oriented)))
Which of the following is true with regard to mobile advertising? (((Within just a couple of years, mobile advertising will be a bigger market than digital advertising. )))
Marketing channel management is related to which of the four Ps? (((place)))
Marney bought a dress from a retail store. Which type of transaction was Marney participating in? (((B2C )))
According to your text, in the broadest terms, the "marketplace" refers to (((the world of trade.)))
Which element of the marketing mix is most relevant to the activity "capturing value"? (((price)))
Which element of the marketing mix is most relevant to the activity "delivering value"? (((place)))
Which element of the marketing mix is most relevant to the activity "creating value"? (((product)))
Which marketing activity is most directly served by the promotion element of the marketing mix? (((communicating value )))
Janine was tired of her winter coat, so she sold it to her friend, Marissa. This is an example of ________ marketing. (((C2C)))
Strong supplier relations and efficient supply chains help firms such as Walmart achieve operational excellence. (((TRUE)))
To build a sustainable competitive advantage, companies should focus on a single strategy. (((FALSE)))
It is not always necessary to go through all the steps in the marketing planning process. (((TRUE)))
A mission statement describes the specific actions a firm will take to achieve its goals.(((FALSE)))
iTunes software is often credited with the success of the Apple iPod MP3 player, because it made the iPod easier to use than competing players, and was difficult for competitors to duplicate. This is an example of a sustainable competitive advantage. (((TRUE)))
STP refers to segmentation, testing, and promotion. (((FALSE)))
The components of a SWOT analysis are strengths, weaknesses, opportunities, and tactics. (((FALSE)))
Firms are typically more successful when they focus on opportunities that build on their strengths relative to those of their competition. (((TRUE)))
Duke’s is a surfer-themed restaurant chain in Hawaii. Most of its customers are tourists. In a SWOT analysis for Duke’s, the possibility that the recession might cut back on tourism in Hawaii would be considered a weakness.(((FALSE)))
Price should be based on the value that the customer perceives as giving them a good value for the product they receive. (((TRUE)))
Geraldo manages the electrical turbine engine division of General Electric Corporation. He makes most decisions independently, without consulting headquarters. Geraldo manages a strategic business unit. (((TRUE)))
The strategic planning process always proceeds sequentially through the five steps. (((FALSE)))
Isaac is looking for ways to offer new goods and services to his existing customers. He is pursuing a market development strategy. (((FALSE)))
SanDisk’s MP3 player product line (called the Sansa) has a low relative market share. The MP3 player market is expected to decline over the next few years. In Boston Consulting Group (BCG) portfolio analysis, the Sansa would be considered a dog. (((TRUE)))
The "implement marketing mix" step of the strategic marketing planning process is part of the control phase. (((FALSE)))
Relative market share is an example of a marketing metric. (((TRUE)))
Product penetration is one of the four major growth strategies. (((FALSE)))
If a firm wants to develop a sustainable competitive advantage, it should (((examine its operations and customer relations to identify significant things competitors cannot easily copy. )))
Which of the following is not one of the four overarching strategies to create and deliver value and to develop sustainable competitive advantages? (((planning excellence)))
When Ramona, the keynote speaker at a major business leaders’ conference, arrived in the middle of the night at the Ritz-Carlton, she was exhausted and her suit was wrinkled from her 10-hour plane trip. The night clerk found someone to dry clean Ramona’s suit and have it ready for her morning presentation. She has been a loyal Ritz-Carlton customer ever since. In this example, Ritz-Carlton demonstrated the macro strategy of (((customer excellence. )))
Amazon’s latest attempt to shore up and enhance its competitive barriers by introducing a home service marketplace that may help it achieve a lasting, powerful advantage is an example of _______________ excellence. (((customer )))
Some banks offer special accounts designed to attract junior high school students. These kids save in such small amounts that the accounts cost banks more to maintain than they are worth; however bankers know that consumers are creatures of habit and hope that the young people they serve now will become adult customers. These banks recognize (((the lifetime value of customers. )))
Nordstrom, an upscale department store, has a well-known reputation for going the extra mile to serve its customers. This reputation for excellent customer service will most likely result in (((a sustainable competitive advantage. )))
Firms achieve ___________ through efficient procedures and excellent supply chain management. (((operational excellence)))
Marketers want their firms to develop excellent supply chain management and strong supplier relations so they can(((create a sustainable competitive advantage. )))
For many years, Southwest Airlines distinguished itself as the low-cost airline. Now, many other low-cost competitors have entered the market. Similarly, Southwest was one of the first airlines to offer online ticketing. Now, all airlines have online ticketing. These examples suggest that (((no single strategy is likely to be sufficient to build a sustainable competitive advantage. )))
Customers around the world know Pepsi and consider it a primary “go-to” brand if they want a refreshing drink. This positioning reflects Pepsi’s (((careful targeting and marketing mix implementation. )))
Carla has been directed by her regional marketing manager to cut prices on seasonal items, place an ad in the local paper, and tell distributors to reduce deliveries for the next month. Which step of the strategic marketing planning process is Carla engaged in? (((implement marketing mix and resources )))
When conducting a SWOT analysis, in what phase of the strategic marketing process is an organization presently engaged? (((planning)))
The automobile manufacturing industry closely watches annual consumer satisfaction surveys. For years, Japanese car companies consistently had the highest levels of customer satisfaction, creating a(n) ________ for these companies. (((sustainable competitive advantage )))
Which of the following is least likely to provide a sustainable competitive advantage? (((lowering prices)))
Even when large discount retailers enter a market, a few small, local retailers survive and prosper. These small retailers have probably developed a(n) ________ that allows them to survive. (((sustainable competitive advantage )))
As part of her company’s SWOT analysis, Valerie is assessing the company’s internal environment, including (((strengths and weaknesses.)))
Samantha is charged with assessing her company’s external environment as part of a SWOT analysis. Samantha will study her company’s(((opportunities and threats.)))
Manufacturers that use just-in-time manufacturing systems coordinate closely with suppliers to ensure that materials and supplies arrive just before they are needed in the manufacturing process. While just-in-time systems can offer major advantages in terms of inventory costs, they must be carefully managed. If a firm found that its just-in-time system was badly managed, leading to frequent manufacturing delays due to missing parts, this would represent a(n) __________ in a SWOT analysis. (((weakness )))
For U.S. businesses with strong export capabilities, expansion of U.S. trade agreements with other countries creates (((opportunities. )))
In 2006, Ford Motor Company announced it would severely cut back its automobile production. For parts companies supplying Ford its parts, this represented a(n) (((threat. )))
Lionel is asked to conduct an STP analysis for his firm. The first step he should perform in this analysis is to (((divide the marketplace into subgroups. )))
In 2006, Walmart announced that it would begin selling organic food products. In doing so, Walmart was most likely trying to (((attract a different market segment. )))
For years, when considering new products, marketers at Celestial Seasonings asked themselves, "What would Stacy think?" Stacy was a fictional character representing 25- to 50-year-old educated, upper-income women who rarely watched television but did a lot of reading. "Stacy" represented Celestial’s primary(((target market segment.)))
After identifying various market segments that her company could pursue, Lisa evaluated each segment’s attractiveness based on size, income, and accessibility. Lisa was involved in (((target marketing. )))
LeBron James, Stephen Curry, and other athletes are paid huge sums of money by companies for celebrity endorsements. If endorsements by these athletes create a clear understanding among consumers of the companies’ products in comparison to competing products, they can help with the firm’s __________ strategy. (((positioning )))
__________ involves the process of defining the marketing mix variables so that target customers have a clear, distinctive understanding of what a product does or represents in comparison with competing products. (((Positioning )))
Imagine that you are in a convenience store choosing your favorite "comfort" food instead of being in a classroom taking this test. You might notice the packaging, colors, labels, even the fonts used on labels. All of these efforts are part of the marketer’s (((positioning strategy. )))
When positioning products relative to competitors’ offerings, firms typically are most successful when they focus on opportunities (((that build on their strengths relative to those of their competition. )))
Many small businesses whose competitors are national franchises advertise "we are locally owned" or "we have been here since 1951." This is part of these firms’ (((positioning strategy.)))
Among Internet users, some do research online, some shop, some look for entertainment, and many do all three. Each of these groups would be called a(((market segment. )))
Suppose your university made a sizable investment in its career services—additional counselors, increased efforts to bring in recruiters, and other services aimed at helping students find jobs. This investment would enhance the university’s _____________ in an attempt to create value for students and recent graduates.(((product value)))
The idea of value-based marketing requires firms to charge a price that (((captures the value customers perceive that they are receiving. )))
E-books, in addition to being an alternative product form, provide __________ value creation since they can be downloaded via the Internet immediately when and where they are needed. (((place)))
In value-based marketing, the promotion element of the four Ps communicates the ________ to customers through a variety of media. (((value proposition )))
When marketers use a variety of communication disciplines—advertising, personal selling, sales promotion, public relations, direct marketing, and online marketing including social media—in combination to communicate a value proposition to the customer, it is referred to as (((integrated marketing communications. )))
Google and other search engines allow marketers to bid to have their ads shown when consumers search keywords related to the firm’s products. These marketers are attempting to create value through (((promotion.)))
Craig sees that his company’s quarterly sales and profits are significantly above projections and says, "That’s great. Let’s keep doing what we’ve been doing." Craig is ignoring the __________ step of the marketing planning process. (((evaluate performance )))
The first objective in the evaluate performance phase of the marketing planning process is to (((review implementation programs and results using metrics. )))
As it pertains to the marketing plan, understanding the causes of performance, regardless of whether that performance exceeded, met, or fell below the firm’s goals (((enables firms to make appropriate adjustments. )))
A regional manager at GNC, a chain of retail stores selling nutritional supplements, is reviewing sales data after a recent in-store promotion. The data show success in some stores and limited response in others. To understand the differences between stores, the manager will probably next review the company’s (((implementation programs, to see if the promotion was handled consistently in the different stores. )))
After conducting STP analysis for her custom auto parts store and developing strategies for each of the four Ps, Monique now has to make _____________ decisions. (((resource allocation )))
Lamar owns four dry cleaning stores in the suburbs of Orlando, Florida. He recently updated his STP analysis, and has just finished adjusting his marketing mix based on the STP results. His next strategic marketing decision will most likely involve determining (((how to allocate resources among his four stores. )))
In most companies, portfolio management is typically done at the SBU or ___________ level of the firm. (((product line)))
A(n) ___________ is a group of products that consumers may use together or perceive as similar in some way. (((product line)))
Heather has been assessing several of her firm’s products using the Boston Consulting Group (BCG) approach to portfolio analysis. She has been trying to assess the strength in a particular market and is looking at the sales of the product and the overall market as well as the sales of competitors. Heather is trying to determine(((the product’s relative market share. )))
To determine how attractive a particular market is using the BCG portfolio analysis, __________ is(are) established as the vertical axis. (((market growth rate )))
In BCG portfolio analysis, products in low-growth markets that have received heavy investment and now have excess funds available to support other products are called (((cash cows.)))
Fernando was thrilled to find out that his company had just decided to invest a great deal of money in the product he was managing. He knows that even with its recent high rate of growth and the fact that it dominates its market, he would need more money to establish it firmly. Using the BCG portfolio analysis, his product would be classified as a(n) (((star)))
Using the BCG portfolio analysis, a "dog" should be phased out unless (((it complements or boosts the sales of another product. )))
The strategic marketing planning process (((is not always sequential.)))
Which of the following is not one of the four major growth strategies marketers typically utilize? (((segment development )))
Adrienne decides to add new sales representatives and increase advertising to increase sales in her existing market for her current line of security systems. Adrienne is pursuing a __________ growth strategy. (((market penetration)))
A __________ growth strategy employs the existing marketing offering to reach new market segments. (((market development)))
Quitman Enterprises sells its business language dictionary to college students throughout the United States. Joseph Quitman, the owner, wants to start selling the book to international students abroad. Quitman wants to pursue a __________________ growth strategy. (((market development)))
When Marvel launched several successful series on Netflix, including , , , and , it employed a ___________ strategy. (((product development)))
H&R is a small, local heating and air conditioning business. The local military base is a potential source of growth, and H&R already installs and services the type of equipment the military would require, but it is difficult to get established as a certified government contractor. H&R is considering a ____________ growth strategy. (((market development)))
Many states create licensing requirements for a variety of professionals (such as lawyers and accountants) designed to restrict entry into their market by professionals from other states. This strategy limits ____________ growth strategies. (((market development)))
Marketers who design and offer new products and services to their existing customers are pursuing a ____________________ growth strategy. (((product development)))
Most banks now have customer relationship software that, when a customer contacts the bank, tells the service representative what types of accounts, loans, and credit cards the customer currently has. Service representatives use this information to sell some of the other services the bank currently offers to these customers. This is a ___________________ growth strategy. (((market penetration)))
Maryam phoned her auto insurance agent to renew her policy. The agent told her about new types of insurance now available—to cover her apartment, or even the engagement ring she just got from her fiancé. The agent was pursuing a ________________ growth strategy. (((product development )))
Introducing newly developed products or services to a market segment the company is not currently serving is called (((diversification. )))
Zara is a women’s clothing retailer headquartered in Spain, with stores located in many countries. Zara has developed a "quick response" system that allows store merchandise to be adjusted rapidly to fit changing customer preferences. Every aspect of Zara’s operation is optimized for this system, making it difficult for competitors like The Gap to duplicate. Zara has established (((a sustainable competitive advantage. )))
Fourteenth National Bank prides itself on offering better service than any of its competitors. If this is accurate, and if customers recognize and value Fourteenth National’s superior service, the bank creates and delivers value through (((customer excellence.)))
Anita has gone to the same hair salon for the past ten years. She believes that her stylist, the salon owner, does a better job of cutting and styling her hair than anyone else could. Other salons have opened closer to Anita’s home, some offering more plush facilities or lower prices, but she isn’t tempted to switch. Anita’s attitude toward the salon is an example of (((customer loyalty.)))
Customer retention programs are based on what concept? (((Customer relationships should be viewed from a lifetime value perspective rather than on a transaction-by-transaction basis. )))
Most banks implement customer retention programs aimed at their best customers. They do this because they know that retaining customers usually results in(((increased long-term profits. )))
Some universities offer online degree programs, competing with traditional colleges based on the convenience of taking online courses. These online programs are most likely pursuing which macro strategy? (((locational excellence)))
When a pharmaceutical company develops a new medication, it typically applies for a patent in order to prevent competitors from developing copycat products for several years. Pharmaceutical companies apply for patents to establish (((a sustainable competitive advantage. )))
One example of a customer loyalty program is (((a "frequent diner" card at a restaurant, offering a free appetizer for every $100 in food purchases. )))
Which of the following is the third step in the marketing planning process?(((identify and evaluate opportunities)))
In mid-2010, Hewlett-Packard Company (HP) acquired Palm Computing Inc., a manufacturer of personal devices and smartphones. Before deciding to acquire the company, strategic planners at HP spent time thinking about how Palm and HP would "fit" together, and how the acquisition might change HP’s core goals and objectives. The strategic planners were engaged in the ____________ step of the marketing planning process. (((define the business mission)))
After defining the business mission, what should a firm do next to develop a marketing plan? (((perform a situation analysis )))
In a SWOT analysis, increasing gasoline prices would represent a potential __________ for manufacturers of electric cars. (((opportunity)))
A former advertising campaign for GEICO Insurance used the slogan "So easy, even a caveman could do it" to emphasize the ease of buying insurance on GEICO’s website. This campaign was part of GEICO’s (((positioning strategy.)))
Abercrombie & Fitch, a clothing retailer, includes a "SHARE" link on the product pages of its website. This link encourages an Abercrombie customer to post a link (perhaps showing a new style of jeans) on Facebook or Twitter. Abercrombie & Fitch hopes that the customer’s friends (who are probably very much like current customers) will click the link, visit the page, and make purchases. This is an example of a __________ growth strategy. (((market penetration )))
Sodexo is a corporation that manages school cafeterias, university dining halls, mess halls at military bases, concession stands at sports arenas, and other large-scale food service facilities. If Sodexo were to begin to sell individual frozen meals in supermarkets, it would be pursuing a __________ growth strategy. (((diversification)))
Singapore Airlines seeks to differentiate itself from competing airlines, in part through innovative design of its airplane seats and in-flight entertainment systems. Through continuous innovation in these areas, Singapore Airlines is pursuing a(n) __________ macro strategy. (((product excellence)))
3M involves its customers in the process of developing new products. In this way, it can benefit from current customers’ insights and develop new products that will meet these customers’ needs. 3M is pursuing a __________ growth strategy. (((product development )))
Over the last few months, Juan and his colleagues have analyzed the current business situation and identified target markets for his firm’s personal care products. Finally, they developed the products, prices, distribution, and promotion that should appeal to each of those target markets. In doing so, Juan has also identified what he believes is an advantage his competitors cannot match. Juan and his colleagues have been developing (((a marketing strategy. )))
Gerald’s Tire Store sets itself apart from competitors by the extra attention it pays to retaining loyal customers by providing fast, courteous service in a clean, professional-looking environment. Gerald’s has used this strategy for over 40 years, expanding to 15 outlets. Gerald’s Tire Store has created a ___________ macro strategy to create and deliver value and to develop a sustainable competitive advantage. (((customer excellence)))
Which of the following is the best way to build a sustainable competitive advantage using product excellence? (((positioning the product using a clear, distinctive brand image )))
A competitive advantage based on location is often sustainable because (((it is not easily duplicated. )))
Four companies dominate the cereal industry. These firms produce in large volumes, promote heavily, and control access to the supermarket shelves through "slotting allowances," which are payments to retailers in return for shelf space. Combined, these four firms have (((a sustainable competitive advantage.)))
Value creation through place decisions for a consumer product involves (((making sure the product is available in the stores where customers will want to find it, and that it is always in stock so they can buy it when they want to. )))
In recent years, cellular (mobile) service providers have worked hard to eliminate "dead zones," providing customers with service wherever they traveled. By working to make the network available in more locations, cellular service companies were focusing on __________ value creation.(((place)))
When firms successfully implement poor strategies (perhaps due to good luck) or do a poor job of implementing good strategies, it can be difficult to (((evaluate performance and make adjustments.)))
In 2007, Apple Computer introduced its new iPhone, adding a cellular telephone, a camera, and Internet access to its iPod. The company was pursuing a(n) __________ strategy. (((product development)))
Allen is in the marketing department of a midsized firm that develops and sells communications systems. He is proud of the human resource area in the company that provides the firm with excellent employees. Allen himself really enjoys his work, but he knows the work of HR helps create an operational advantage for the firm because of all the following except (((the company provides products with a high perceived value.)))
Which of the following actions would be the most likely to support and enhance an operational excellence macro strategy? (((3M Corporation implementing new software to improve communication with its suppliers )))
Which of the following factors, listed in a situation analysis for a major U.S. auto manufacturer, is the best example of a threat? (((A New York law firm has filed a $10 million class action suit against the company on behalf of car owners whose gas tanks exploded. )))
Which of the following factors, listed in a situation analysis for a major U.S. auto manufacturer, is the best example of an opportunity? (((Recent consumer studies have indicated that Chinese consumers prefer American cars. )))
The global athletic footwear market is expected to experience only very slow growth over the next several years. Nike is the market leader. According to Boston Consulting Group portfolio analysis, how should Nike treat its athletic shoe business? (((Nike’s athletic shoe business still requires some investment, but it is likely to produce excess resources that can be invested in other divisions of the company. )))
Subway is a large chain of franchise sandwich shops. Marcia owns three Subway stores in a large city. At the end of the year, she notes that sales rose from 2 to 5 percent over last year’s sales at Stores 1 and 2, but fell 2 percent at Store 3. Based on this information, which of the following is the best course of action as to how Marcia should reward (or punish) her store managers?(((She should seek more information about why the stores had different results before making a decision.)))
Coca-Cola sells two different zero-calorie versions of Coke: Diet Coke and Coke Zero. It has chosen to attempt to appeal to men with Coke Zero. In addition to launching an ad campaign featuring men enjoying Coke Zero, Coca-Cola also designed a masculine-looking can for Coke Zero, with bold red lettering on a black background. This specially designed can is an example of (((positioning. )))
Delta Airlines is among the companies experimenting with selling products and services on their Facebook pages. The idea is to make purchasing even easier for customers who may spend large portions of the day with Facebook active on their computers or mobile devices. There is no need even to navigate to Delta’s website—users can book a trip in Delta’s "Ticket Agent" application without ever leaving Facebook. Which element of the marketing mix does this represent? (((place and value delivery )))
Because it was able to deliver merchandise overnight, in the required quantities and at a lower delivered cost than its competitors, Joe’s Sports Shack was given exclusive rights to sell clothing with school logos for the school district. Joe’s demonstrates ________ excellence. (((operational )))
During which phase of the marketing planning process does a firm engage in segmentation, targeting, and positioning? (((identify and evaluate opportunities phase )))
During a SWOT analysis, a company should assess the opportunities and uncertainties of the marketplace due to changes in several factors, given the acronym CDSTEP. Which of the following is not one of these factors? (((ethical)))
The process of dividing the market into groups of customers with different needs, wants, or characteristics is called (((market segmentation.)))
________ involves the process of defining the marketing mix variables so that target customers have a clear, distinctive, desirable understanding of what the product does or represents in comparison with competing products. (((Market positioning )))
All of the following statements regarding marketing metrics are true except (((they cannot be used to project the future.)))
A product that is in a high-growth market but has a low market share would be classified as a ________ on the BCG matrix. (((question mark)))
By paying close attention to customer needs and continuously monitoring the business environment in which it operates, a good marketer can identify potential opportunities. (((TRUE)))
When analyzing the immediate environment and the macroenvironment, marketers must be careful to keep the firm at the center of all analyses. (((FALSE)))
The marketing firm must consider the entire business process, all from a consumer’s point of view. (((TRUE)))
In the immediate environment, the competition has no effect on consumers. (((FALSE)))
The three elements of the consumer’s immediate environment are the company, competition, and corporate partners. (((TRUE)))
Demographics are transmitted by words, literature, and institutions from generation to generation. (((FALSE)))
Demographic profiles of customers provide an easily understood "snapshot" of the typical consumer in a specific target market. (((TRUE)))
As the euro becomes more expensive relative to the dollar, Americans are likely to purchase more European wines. (((FALSE)))
Macroenvironmental factors include the company, competition, and corporate partners. (((FALSE)))
Members of Generation Y are also called Millennials. (((TRUE)))
As the children of the Baby Boomers, Generation X is the biggest cohort since the original postwar World War II boom. (((FALSE)))
The members of Generation X are also referred to as Digital Natives. (((FALSE)))
One of the social trends discussed in the text that is shaping consumer values in the United States and around the world is a concern about the vast disparity in income. (((FALSE)))
Culture can broadly be defined as the shared meanings, beliefs, morals, values, and customs of a group of people. (((TRUE)))
Companies such as Burger King no longer use cartoon characters to promote products such as burgers and fries because the characters do not retain popularity among young children for very long. (((FALSE)))
Socially responsible marketing involves a strategic effort by firms to supply customers with environmentally friendly, sustainable merchandise and services. (((FALSE)))
Firms that disingenuously market products or services as environmentally friendly, with the goal of gaining public approval and sales rather than actually improving the environment, are engaging in greenwashing. (((TRUE)))
The next broad wave of mobile applications is likely to expand the use of wireless payments through applications such as Apple Pay, Google Wallet, Master Card’s Easy Pay, and Softcard Mobile Wallet, all of which enable customers’ phones to serve as m-wallets. (((TRUE)))
Inflation refers to the cost of borrowing money. (((FALSE)))
The 1890 Sherman Antitrust Act prohibits monopolies and other activities that would restrain trade or competition and makes fair trade within a free market a national goal. (((TRUE)))
Consumers’ needs and wants, as well as their ability to purchase, depend on a host of factors that remain consistent over time. (((FALSE)))
Parties that work with the focal firm are its corporate partners. (((TRUE)))
Jeanne, who lives in Boston, refers to Pepsi as "tonic," while Paul, who lives in Chicago, calls it "pop." This is an example of a regional culture difference.(((TRUE)))
Lisle Hair Company keeps track of the gender and age of its customers so it can target e-mails to them and be sure to have the right hair products in stock when they visit. This is an example of the use of demographics in marketing. (((TRUE)))
Applying age as a basis to identify customers is unethical, as it involves stereotyping. (((FALSE)))
As a retail clothing store manager, Randy frequently asks his staff what customers are saying and what they are asking for. He also attends the quarterly clothing show at the regional merchandise mart. Randy’s efforts will likely help him (((identify potential opportunities. )))
The centerpiece of the marketing environment analysis framework is (((consumers.)))
A firm’s macroenvironment includes all of the following except (((competition. )))
The center of all marketing efforts is (((the consumer. )))
The creation of CVS Health’s mobile apps in connection with its onsite MinuteClinics is a response to a _________ trend that surrounds health and wellness concerns. (((social)))
One of the goals of value-based marketing is (((to offer greater value than competitors offer. )))
Ryan knows that one of the goals of value-based marketing is to provide greater value to consumers than competitors offer. To accomplish this goal, Ryan’s firm must look at everything it does (((from a consumer’s point of view. )))
Yvonne knows her firm must look at everything it does from a consumer’s point of view. One major difficulty is that consumers’ __________ change(s) over time. (((needs, wants, and ability to purchase )))
Select the statement that best describes the key traits of Generation Y. (((This generation varies the most in age, ranging from teenagers to adults who have their own families. )))
Which of the following statements accurately describes the current income distribution in the United States? (((Wealthy households are outpacing both poor and middle-class households. )))
When you register your telephone number with the National Do Not Call Registry, you are responding to a need for (((privacy.)))
In the immediate marketing environment, the first factor that affects the consumer is (((the firm itself. )))
Successful firms focus their efforts on satisfying customer needs that (((match their core competencies.)))
The Mayo Clinic in Minnesota is known for top-quality medical care. For decades, even presidents and dictators from around the world flew to the Mayo Clinic to utilize its services. The Mayo Clinic used its reputation to create additional medical facilities in Jacksonville, Florida, and elsewhere. This is an example of a firm focusing its efforts on satisfying customer needs that (((match its core competencies.)))
Yuri is considering a new promotional campaign in which he will compare his products to those of his competitors. Before initiating the promotional campaign, Yuri will assess his competitors’ strengths, weaknesses, and (((likely response to his promotional activities. )))
The firms that work along with the focal firm to provide goods and services to consumers are viewed as (((corporate partners. )))
Yellow Trucking provides transportation services for Dan’s Delicious Donuts so that Dan’s stores always have fresh stock. Yellow Trucking is an example of a (((corporate partner. )))
The difference between a firm’s immediate marketing environment and its macroenvironment is that the macroenvironment (((is external. )))
The shared meanings, beliefs, morals, values, and customs of a group of people constitute their (((culture. )))
Political candidates are notorious for appealing to the different beliefs and values of groups of potential voters in different areas of the country. These politicians are appealing to differences in (((regional culture.)))
Insight Guides, a line of travel books, provides travelers with background information about people’s beliefs, values, and customs in various parts of the world. Insight’s books educate travelers about a country’s (((culture.)))
When studying culture, the challenge for marketers is to determine whether culture (((can help identify a particular group that might be interested in the marketer’s products. )))
Late in the day on September 11, 2001, the day of the World Trade Center attacks, Americans purchased all the American flags Walmart stores had available, nationwide. These purchases were indicative of (((country culture. )))
The United States has often been called the "melting pot" society, integrating people from many different cultures into the social fabric of the country. The challenge for marketers is to determine whether a group’s culture (((can be used as a relevant identifier for a particular target group. )))
When considering the use of a radio commercial in England that was designed for U.S. markets, a marketer would likely need to consider which of the following aspects of culture that might be different between the two countries? (((language )))
In New England, foot-long sandwiches are called "grinders," while in many other parts of the country they are called "subs." This is an example of the impact of (((regional culture. )))
The shift of population from the Rust Belt in the North to the Sun Belt in the South and Southwest will likely (((reduce regional cultural differences. )))
Typical demographic data include all of the following except (((language differences.)))
When marketers look at advertising media they often begin with viewer or listener profiles such as age, income, gender, and race. They then compare the media profile with their target audience. These marketers are using __________ to see if the media "fit" with their advertising agenda. (((demographics )))
Generational cohorts are groups of people of the same generation who have similar __________ because they have shared experiences and are in the same stage of life.(((purchase behaviors)))
Viewers of the World Series are likely to see ads for beer and cars, and viewers of the Academy Awards broadcast are likely to see ads for clothing and hair care products, due to (((differing demographic data for potential and past viewers. )))
________ is distinguished from the other generational cohorts in that the group’s oldest members are now collecting Social Security. (((Baby Boomers)))
Marketers wanting to use social media to reach consumers should understand that people in the _______ group were born into a world that was already full of electronic gadgets and digital technologies such as the Internet and social networks. (((Generation Z)))
The first "latchkey" children belong to which generational cohort? (((Generation X )))
According to your text, the wealthiest 1 percent of the population control approximately what percentage of Americans’ total net worth? (((34.6 percent )))
ABC Company knew that its customers were interested in environmentally friendly business practices, so it began marking all of its products as environmentally friendly because they were made with all-natural ingredients, even though ABC’s plant was one of the worst polluters in town. The term for this practice is (((greenwashing.)))
When considering income as a demographic variable affecting marketing efforts, marketers need to recognize that (((income in the United States has become more unevenly distributed. )))
Astute marketers recognize that the increasing disparity of income between upper- and lower-income groups (((creates opportunities to provide value to each group. )))
Population estimates show that by the year 2050, nearly 30 percent of the population in the United States will come from which group? (((Hispanics )))
For some products, marketers can combine education level with other data like occupation and income to obtain (((accurate predictions of purchase behavior.)))
Marketers know that, compared to high school graduates who are working full time, college students (((spend their disposable dollars differently. )))
Which of the following is true regarding modern gender roles? (((Gender roles have been blurred in the past several years. )))
The results of the 2010 census suggest what about the 42 million African American U.S. households? (((They are more affluent and suburban. )))
Marketers in the United States are paying increasing attention to ethnic groups because (((approximately 80 percent of all population growth in the next 20 years is expected to come from minority groups.)))
Marketers are more likely to find higher concentrations of foreign-born Americans and recent immigrants in (((New York, Los Angeles, San Francisco, and Chicago. )))
Though Asian Americans comprise only about 5.6 percent of the U.S. population, they represent (((the fastest-growing minority population.)))
Strategic efforts to supply consumers with environmentally friendly merchandise are called (((green marketing. )))
By offering environmentally responsible products, green marketers (((add value that other products do not have. )))
Many American consumers are purchasing hybrid automobiles even though they are more expensive than conventional compact cars. These consumers (((value contributing to a greener environment. )))
Every time a consumer surfs the web and clicks on a site, online marketers can place _____ on that user’s computer, showing them where the user starts, proceeds, and ends the online encounter—as well as what the user buys and doesn’t buy. (((cookies)))
What retailer, in 2013, suffered a massive security breach that allowed hackers to steal credit and debit card information for millions of customers? (((Target)))
What technological advancement enables a firm to track an item from the moment it was manufactured, through the distribution system, to the retail store, and into the hands of the final consumer? (((RFID tags )))
An advance in technology called RFID makes it easier for manufacturers, distributors, and retailers to track items through production, distribution, and sales. This advance helps the firm (((meet inventory needs.)))
When marketers monitor the economic situation affecting their target markets, they are likely to monitor changes in all of the following except (((age.)))
If the inflation rate in Mexico was twice the rate in the United States, but the Mexican monetary authorities kept the peso-dollar exchange rate almost constant, which of the following would be true? (((Mexican products would be more expensive, while U.S.-made products would become comparatively less expensive. )))
Laws that prohibit the formation of monopolies or alliances that would damage a competitive marketplace benefit consumers through (((increased choices. )))
Which of the following groups has never lived without easy access to the Internet and other digital technologies? (((Generation Z)))
Which of these is a macroenvironmental factor? (((culture )))
When Kimberly-Clark introduced rolls of toilet paper without the cardboard core, what social trend did this product respond to? (((green marketing )))
If McDonald’s wanted to change its marketing strategy in response to the social trends outlined in the text, it might consider all of the following ideas except (((creating an advertising campaign targeting elementary schoolchildren.)))
Suppose that you are the vice president of marketing for Target, the large retail store chain. You want to keep your website and in-store services current with technological advances. You might consider experimenting with what is most likely the next expected development in mobile technology, which is (((wireless payments from mobile devices. )))
Some companies have been accused of taking advantage of the current social trend of green marketing, positioning their products as environmentally friendly when this may not actually be the case. This is called (((greenwashing.)))
Part of the social trend toward health and wellness is a concern about the increasing degree of obesity in the United States. Related to this, the text notes the increasing popularity of (((yoga.)))
The political/regulatory environment comprises political parties, governmental organizations, and (((legislation and laws.)))
Christy, a marketing executive who was born in 1955, advocated that her company focus on a print campaign for its new line of lipsticks rather than spending money on online ads. This point of view may be attributed to the fact that she is a member of which generational cohort? (((Baby Boomer )))
Which generation’s members are also known as Millennials? (((Gen Y)))
The poorest 10 percent of the U.S. population earned less than _______ per week in 2014. ((($412 )))
Which statement about the changing ethnicity in the United States is true? (((African American U.S. households are more affluent than previous studies suggested. )))
According to the text, 14 cable companies agreed to provide low-cost Internet access to impoverished families for all of the following reasons except (((increased income from a larger number of subscribers. )))
The government has enacted laws that promote both fair trade and competition by prohibiting the formation of monopolies or alliances that would damage a competitive marketplace, fostering fair pricing practices for all suppliers and consumers. These actions are part of the ________ environment of business. (((political/regulatory)))
Which of the following specifically prohibits monopolies? (((Sherman Antitrust Act)))
What is most likely to happen when inflation increases? (((Consumers buy lower-priced foods. )))
________ is the cost to the customers or the fee the bank charges those customers for borrowing money. (((Interest)))
In the summer of 2002 the euro was valued at slightly less than US$1. By 2008, it had risen to an all-time high of $1.60, but in early 2016, the euro and the dollar were nearly equivalent. This change in value is called (((foreign currency fluctuations.)))
When Diana went to college, tuition was $650 per semester. Now that same college charges $6,500 per semester. This number reflects a persistent increase in price known as (((inflation. )))
From a firm’s perspective, the biggest advantage of RFID is (((it enables the firm to track an item from the moment it was manufactured. )))
Which of the following is an example of greenwashing? (((A company donates money to a school reading project so it can advertise itself as environmentally friendly. )))
Nearly what percentage of U.S. adults now recycle their soda bottles and newspapers? (((50 percent)))
Which of the following is a social trend listed in the text? (((health and wellness)))
Which ethnic group in the United States tends to earn more, have more schooling, and be more likely to be professionally employed or own a business?(((Asian American)))
Which ethnic group tends to be not only more affluent and suburban than previously thought, but also younger, with 47 percent between the ages of 18 and 49 years? (((African American)))
A generational cohort is a group of people(((who are of the same generation. )))
The loyalty program My Starbucks Rewards, which allows members to pay via their mobile device and earn points toward discounts and free offers, enables Starbucks to gather the purchase behavior data, appropriately segment that customer, and offer customers deals within a week. (((FALSE)))
When a marketer adjusts the marketing mix to give customers a clear, distinctive understanding of what the product does, the marketer is engaging in psychographic segmentation. (((FALSE)))
For products like pencils and paper clips, marketers should probably use an undifferentiated targeting strategy. (((TRUE)))
When deciding how to promote his new medical oncology center, Dr. Jones decided that he did not need to throw a large grand opening reception. Instead, he promoted the center to the internal medicine doctors in the area, who were the doctors most likely to refer patients to the center. Dr. Jones was engaged in a concentrated targeting strategy. (((TRUE)))
It would be logical for bathing suit marketers to use geographic segmentation. (((TRUE)))
NASCAR redirected its marketing efforts when a survey indicated that almost 50 percent of its fans were female. This is an example of psychographic segmentation. (((FALSE)))
The Value and Lifestyle Survey (VALS) conducted by Strategic Business Insights (SBI) is a widely used tool for geographic segmentation.(((FALSE)))
Giant Food Stores in suburban Washington, DC, adjusts its ethnic food aisle offerings based on the ethnic groups living near each store. Giant Foods is using geodemographic segmentation. (((TRUE)))
Positioning strategies generally focus either on how the product benefits the consumer or on how it is better than competitors’ products.(((TRUE)))
As it relates to positioning, a self-values map displays the position of products or brands in the consumer’s mind. (((FALSE)))
Perceptual maps include positions of current brands as well as ideal points where a consumer segment would want a product to be. (((TRUE)))
The STP process is made up of strategy, targeting, and promotion. (((FALSE)))
The first step in the segmentation process is to articulate the vision or objectives of the company’s marketing strategy clearly. (((TRUE)))
Automobile manufacturers could build cars specially designed for very tall people (for example, over seven feet), but it is likely that this segment is not substantial. (((TRUE)))
A complex product that is best described in writing will be difficult to explain to consumers who are unable to read. For this reason, a segment in which many consumers could not read would be considered unattractive because the consumers would not be responsive. (((FALSE)))
A company that sells only multigrain, low-calorie bread should use an undifferentiated targeting strategy. (((FALSE)))
A value proposition compares the price of a product to its benefits.(((FALSE)))
Maintaining a unique value proposition can be sustained in the long term only in monopoly situations or monopolistic competition situations. (((TRUE)))
Sally is in the new marketing department of a midsized lawn and garden company. She is working on the first marketing plan the firm has ever used. She has defined the mission and objectives, and she just finished a situation analysis for the firm. The next step is to (((identify and evaluate opportunities by conducting segmentation, targeting, and positioning analysis. )))
Segmentation, targeting, and positioning(((are part of a firm’s marketing strategy. )))
Alex has found that both commercial and residential real estate buyers respond positively to his marketing communication message. Alex has identified __________ that respond similarly to his marketing efforts. (((market segments )))
For each target market, General Imaging Corporation, a manufacturer of imaging equipment, will engage in positioning, adjusting its marketing mix variables in order to give customers (((a clear, distinctive, and desirable understanding of their products relative to competing products. )))
The first step in the STP process is to(((establish the overall strategy or objectives. )))
For products like clothes pins, which provide the same benefit for all consumers, marketers should probably use a(n) ________ strategy. (((undifferentiated targeting )))
In a classic example of segmentation strategy, years ago Swift Meat Company transformed turkey meat into "Butterball" turkeys. In doing so, the company(((differentiated its product. )))
Adidas Group owns Reebok, Rockport, and TaylorMade brands. Adidas uses the different brands to pursue a(n) __________ strategy. (((differentiated targeting)))
Adidas Group owns Reebok and Rockport—both of which offer different types of shoes. Having a variety of brands allows adidas to use a differentiated targeting strategy to(((obtain a bigger share of the shoe market. )))
One of the benefits of a differentiated targeting strategy is that it allows the firm to diversify its business and (((lower overall risk. )))
A differentiated targeting strategy can allow a firm to obtain a bigger share of the overall market; this strategy (((is likely more costly than an undifferentiated strategy. )))
Firms use a differentiated targeting strategy because (((it helps obtain a bigger share of the market. )))
When Penguin Catering Services first opened, the owner decided to target only events at resorts in its geographic region. Penguin Catering was using a(n) __________ targeting strategy. (((concentrated)))
__________ is an extreme form of a targeting strategy. (((Micromarketing )))
"Corporate pilot fish" are former employees who start a new business based primarily on contacts and contracts with their old company. A corporate pilot fish would probably pursue a(n) __________ targeting strategy. (((micromarketing)))
Many companies use the Internet to allow consumers to design customized products; for example, Nike allows customers to order shoes with custom color combinations. This is a form of (((micromarketing. )))
Four frequently used targeting strategies are the micromarketing, undifferentiated, differentiated, and __________ targeting strategies. (((concentrated )))
When Mr. How, a Pennsylvania-based discount lumber and hardware chain, sent snow blowers to its Augusta, Georgia, store in April, it was engaged in (((misguided geographic segmentation. )))
Differences in weather and climate create opportunities for (((geographic segmentation. )))
Kellogg’s marketing communications about its Special K cereal appeal almost exclusively to women. This is an example of __________ segmentation. (((demographic)))
__________ segmentation is the most common basis of consumer market segmentation because these segments are easy to define and usually easy to reach. (((Demographic)))
Beer marketers know that high-school educated, working-class males from the ages of 25 to 40 make an attractive market for their products. This is a __________ segment of the beer market. (((demographic )))
Television advertising has recently expanded to include "mini-ads," which are short ads lasting five to ten seconds. These ads are most useful in advertising to men, since men are more likely than women to channel surf during commercial breaks. Given this fact, this type of advertising will be more useful to marketers engaged in __________ segmentation. (((demographic )))
Demographic segmentation is segmentation based on all of the following except (((lifestyle. )))
Psychographics is the segmentation method that delves into how consumers (((describe themselves. )))
To develop psychographic segments, the marketer must understand consumers’ (((self-values, self-concept, and lifestyles. )))
Paul is considering psychographics as a way to segment the market for his small travel agency. This approach to segmentation offers him an advantage because(((it is based on underlying motivations of target customers. )))
Education marketers know that the primary motivations of most nontraditional college students are self-respect and self-fulfillment, so they craft their marketing message to emphasize the benefits these consumers are looking for in a college education. What type of segmentation method was used to segment this market? (((psychographics)))
Baby Boomers represent a huge demographic segment for travel marketers. Baby Boomers are also heavily motivated by self-fulfillment, which creates the possibility of __________ segmentation.(((psychographic )))
Beverage companies often use television commercials with images of young people laughing and enjoying themselves on a beach or in a club. These images are designed to appeal to consumers’ __________, suggesting "be like me." (((self-concepts)))
If values provide an end goal, and self-concept is the way one sees oneself in the context of that goal, lifestyles are (((how we live our lives to achieve our goals. )))
VALS is the most widely used __________ segmentation tool.(((psychographic )))
Marketers have found that __________ segmentation is often more useful for predicting consumer behavior than __________ segmentation. (((psychographic; demographic )))
Although marketers have found that psychographics are often more useful for predicting consumer behavior than demographics, psychographic segmentation (((is a more expensive method for identifying potential customers. )))
__________ segmentation is the segmentation method most directly related to value creation for consumers. (((Benefit)))
A university using benefit segmentation and targeting students who want to get a degree quickly while still working full-time would most likely focus on (((providing classes at convenient times and offering online courses. )))
The light beer commercial with the slogan "less filling, tastes great" was based on __________ segmentation. (((benefit )))
Toothpastes sold in supermarkets and pharmacies promise various results, such as whiter teeth, protection against gum disease, tartar control, or fresh breath. Toothpaste marketers are using __________ segmentation. (((benefit)))
Golden Years Vitamin Corporation targets consumers living in Florida who are over age 65. Golden Years is using __________ segmentation. (((geodemographic )))
The phrase "birds of a feather flock together" captures the idea of __________ segmentation. (((geodemographic )))
The Tapestryâ„¢ Segmentation system divides and sorts small geographic areas into categories using 60 or more demographic and lifestyle characteristics. This market research tool is an example of __________ segmentation. (((geodemographic )))
Gloria is opening an upscale women’s clothing store in a growing suburban residential area. Gloria knows her target market is upper-income women living within 20 miles of her store. She can’t afford to purchase Tapestry analysis, so she will most likely use __________ as a basis for targeting her market.(((real estate values by subdivision )))
Geodemographic segmentation can be particularly useful for __________, because customers typically patronize stores close to their neighborhood. (((retailers )))
Retail store chains often use __________ data to tailor each store’s assortment to the preferences of the local community. (((geodemographic)))
Marketers often create a special marketing mix for loyalty segments because these segments are (((generally profitable. )))
When Ron complained to his bank about the unprofessional behavior of one teller, the branch manager added a code into his accounts alerting all tellers that Ron was a valued customer and to provide whatever service he requests. The branch management included Ron in its __________ segmentation. (((loyalty )))
When airlines created frequent-flyer programs, they were among the first retailers to embrace (((loyalty segmentation.)))
One of the reasons marketers use loyalty segmentation is (((the high cost of finding new customers. )))
While demographic and geographic segmentation of retail customers are relatively easy, these characteristics do not help marketers determine (((what their customers need.)))
Kim’s Kayak Tours initially identified active retirees living in the retirement community nearby as one of its target markets. Kim then tailored her service and marketing message to the interests and schedules of that audience. Kim initially used __________ segmentation and then used __________ segmentation.(((geodemographic; lifestyle )))
Marketers often employ a combination of segmentation methods, using __________ to identify and target their customers, and __________ to design products and messages to appeal to them. (((demographics and geography; lifestyle or benefit segmentation )))
Karen has identified four potential market segments for her Rent-A-Chef home meal catering service. The next step is to(((evaluate the attractiveness of each segment. )))
Karen has identified four potential market segments for her Rent-A-Chef home meal catering service. She will now evaluate whether or not each segment is attractive relative to all of the following except (((realistic. )))
Carlos has identified four potential market segments for his Rent-A-Nurse service. He will now compare the segments to see if they are distinct from each other. Carlos is evaluating whether or not each segment is (((identifiable. )))
Greta is concerned that one of the potential market segments she has identified for her dog grooming service is too small and has too little income to have sufficient buying power. Greta is concerned with whether the segment is (((substantial. )))
Ryan wants to sell personal website services to American soldiers in Afghanistan. Because of the difficulty of communicating with people in a war zone, Ryan may have trouble with this segment not being (((reachable. )))
With access to the Internet nearly universal in the United States, many potential market segments have become more (((reachable.)))
Talbots, an upscale women’s clothing store, targets college-educated women between 35 and 55 years old with average household income of $75,000 or more. This is a form of __________ segmentation. (((demographic )))
When Talbots, an upscale women’s clothing store, tries to portray an image of "traditional, conservative, and with good taste," it is appealing to its target market’s(((lifestyle. )))
College students can be a less __________ market segment because students’ media habits are quite diverse and firms might have to use a wide variety of media to attract this segment. (((reachable)))
For a segmentation strategy to be successful, the customers in the segment must react similarly and positively to the firm’s marketing mix. In other words, the market segment must be (((responsive. )))
When Starbucks first opened, many critics suggested, "No one will pay $4 for a cup of coffee." Starbucks’s critics suggested consumers would not be __________ to the company’s offerings. (((responsive )))
The manager of Plantation River Country Club wanted members of the very upscale club to use the bar and dining facilities more frequently. He offered a two-for-one "happy hour" special but few members showed up. The manager did not have a grasp of what would make his target market (((responsive. )))
Caroline is assessing market growth, market competitiveness, and market access for each segment she has identified. Caroline is assessing the __________ of each potential market segment. (((profitability )))
After assessing the market growth potential in Mexico for his company’s baby products, Harmon wanted to evaluate market competitiveness. To do this, Harmon would consider (((the number of competitors, entry barriers, and product substitutes. )))
After assessing the market growth potential and market competitiveness in Mexico for his company’s baby products, Harmon wanted to evaluate market access. To do this, Harmon would consider (((ease of accessing or developing distribution channels and brand familiarity. )))
Which of the following would not be used in calculating the profitability of a segment? (((segment perceptions )))
Fiona is looking to expand her pet grooming business and wants to evaluate the profitability of a potential new market. The area she is looking at has 2,000 homes and Fiona estimates that 20 percent of them would be likely to use her service. She charges $45 per grooming and on average customers groom their pets six times a year. Fiona estimates her variable costs to expand her business will be $10 per grooming and her fixed costs are $10,000. How much profit would Fiona make on this new segment? ((($72,240 )))
Paula is trying to determine whether the segments she is considering for her day care center will be profitable. Which of the following will not specifically help her in this analysis? (((the average number of school-age children in families sending a child to the day care center )))
Ronaldo wants to develop an Internet auction-based business and is working through the STP process. After establishing his objectives, describing potential market segments, and evaluating the attractiveness of each segment, Ronaldo now has to (((select a target market. )))
When selecting a target market, firms will be most successful if they(((match the firm’s competencies with a market segment’s attractiveness. )))
__________ involves defining the firm’s marketing mix variables so that target customers have a clear, distinctive, desirable understanding of the firm’s offerings relative to competitors’ offerings. (((Positioning)))
Whenever Donald calls on potential pest control customers, he emphasizes the fact that, unlike the national franchise competitors, he is a local business person and has been in business over 20 years. Donald is __________ his business relative to his competition. (((positioning )))
Regina wants to position her financial services company. Regina can position her services according to all of the following except (((profitability. )))
Whenever the president of the local public university promotes the institution, he emphasizes the university’s price (much lower than neighboring private colleges) and high quality. He is positioning the institution based primarily on (((the value proposition. )))
We often see advertisements touting a product as being made with natural ingredients, or being long-lasting. Marketers using these types of promotions are positioning their products based primarily on (((product attributes.)))
Marco was assigned to help create a positioning strategy for his employer’s products based on product attributes. He should consider (((what the target market would consider the most important features. )))
Almost all U.S. political candidates use some combination of red, white, and blue in their campaign posters. They are trying to position themselves in voters’ minds using(((symbols.)))
Jim wants to position his firm against his competitors. In doing so he should (((avoid looking too much like the competitor so as not to confuse the target segment.)))
A __________ is often used to illustrate the position of a firm’s products or brands in consumers’ minds. (((perceptual map )))
Within a perceptual map, a(n) __________ represents where a particular market segment’s desired product would lie. (((ideal point)))
"Why create a perceptual map?" asked David. Andreas, the senior marketing manager, gave him four reasons. All of the following are valid reasons except((("It’s a way to show the position of the company throughout the product’s life cycle." )))
The perceptions of __________ are being measured in a perceptual map.(((consumers)))
Which of the following segmentation methods is being used when M&M’s offers special wedding-themed packaging for customized M&M’s (imprinted with the names of the bride and groom)? (((occasion segmentation )))
Godiva Chocolates produces specially decorated boxes of candies for Valentine’s Day, Easter, and Christmas. This is an example of ________ segmentation. (((occasion)))
Which of the following defines a value proposition? (((the unique value that a product or service provides to customers )))
Overriding desires that drive how we live our lives are called (((self-values.)))
When Victoria’s Secret offers different product lines, such as its Pink line, it is trying to capture the market by(((using segmentation, targeting, and positioning. )))
When evaluating the attractiveness of the segment, if a segment is expected to react positively to the firm’s offering, we say that the segment is (((responsive. )))
What is the best situation in the Circles for a Successful Value Proposition framework? (((The firm’s offering overlaps with customer needs/wants but not with competitors’ offerings. )))
In the Circles for a Successful Value Proposition framework, the value proposition is represented by (((the intersection between customer needs/wants and the firm’s product’s benefits. )))
In the Circles for a Successful Value Proposition framework, the portion of the customer needs/wants circle that doesn’t overlap with anything else represents (((unmet customer needs/wants. )))
All of the following are one of the main components of a value proposition except (((the price of the product or service being offered. )))
Let’s Dish is a meal-preparation service operating in three states. Customers visit a Let’s Dish store to select and partially prepare their meals, which are then packaged for the freezer. At home, the customer can pull a meal out of the freezer and warm it in the oven or microwave. If Let’s Dish wanted to expand into new areas of the United States, what segmentation method would probably be most useful in choosing new locations? (((geodemographic segmentation )))
Social networking sites that compete directly with Facebook have a difficult time surviving; however, LinkedIn has survived by focusing on the needs of business professionals, who seek to network with colleagues, share their expertise, post online résumés, and look for new jobs. What segmentation method is LinkedIn using? (((benefit segmentation )))
Roderick is charged with assessing his company’s external environment as part of a SWOT analysis. Roderick will study her company’s (((opportunities and threats)))
Which of the following growth strategies employs the existing marketing mix and focuses the firm’s efforts on existing customers? (((market penetration)))
What type of analysis does management use to evaluate the firm’s various products and businesses and allocate resources according to which products are expected to be the most profitable for the firm in the future? ((Portfolio Analysis))
Amazon’s Alexa and Google Home are examples of technology that enable consumers to call out a command to deal with all sorts of in-home tasks. ((True))
Amazon’s Alexa and Google Home are examples of technology that enable consumers to call out a command to deal with all sorts of in-home tasks. (( True ))
During which step of the marketing planning process does a firm engage in segmentation, targeting, and positioning? ((identify and evaluate opportunities phase))
What type of marketers constantly measure the benefits that customer perceive against the cost of their offerings? (( value-oriented marketers))
Social media is of increasing importance. About ________ of the world’s population uses Facebook. ((30 percent))
the four overarching strategies to create and deliver value and to develop sustainable competitive advantage include locational, customer, operational and ________ excellence. ((planning))
Promotion deals specifically with retailing and marketing channel management, also known as supply chain management ((False))
Sustainability, health and wellness, and how to more efficiently utilize and distribute food are all considered political trends. ((False))
Parties that work along with the focal firm to provide goods and services to consumers are viewed as ((corporate partners.))
Apple makes its innovations compatible with existing products to encourage consumers to maintain a long-term relationship with the company across all their electronic needs. ((TRUE))
Marketing is an activity that only large firms with specialized departments can perform. ((False))
What type of marketers constantly measure the benefits that customers perceive against the cost of their offerings ((value-oriented marketers))
Macroenvironmental factors include culture, demographics, social trends, and the ((economic situation))
A set of approaches and techniques that firms use to make and deliver a given set of goods and services is known as a distribution network ((False))
A product is anything that is of value to a consumer and can be offered through a voluntary marketing exchange. ((( TRUE )))
Effective packaging and labels send consumers the signal “Buy me!” ((( TRUE )))
A company’s product line consists of its various product mixes. ((( FALSE )))
Brands enable customers to quickly differentiate one firm or product from another. ((( TRUE )))
Brand equity is calculated by subtracting sales of generic brands from the sales of branded items in a category. ((( FALSE )))
The more familiar customers are with a brand, the harder their decision-making process will be. ((( FALSE )))
Harold is a loyal Coca-Cola customer. Loyal customers like Harold require lower marketing expenditures, benefiting businesses like Coca-Cola. ((( TRUE )))
Private-label brands are imitations often sold by street vendors. ((( FALSE )))
For a company like Pepsi-Cola, brand dilution occurs when it sells more product than the competition. ((( FALSE )))
In entertainment licensing, the major risk to licensees is that the brand will become overexposed. ((( FALSE )))
A product label is just a required sticker on a package. ((( FALSE )))
When purchasing a mobile phone, the product you are buying is more than just the phone. ((( TRUE )))
A customized carved cedar wood sign for your house is an example of a convenience product. ((( FALSE )))
For the average college student, a retirement account would be a shopping product. ((( FALSE )))
Kellogg’s sells many different breakfast cereals, including Corn Flakes, Rice Krispies, Frosted Flakes, Raisin Bran, and Special K. This is an example of the breadth of the Kellogg’s product mix. ((( FALSE )))
If Microsoft decided to sell mobile phones under the Xbox brand (which it uses for its gaming systems), this would be an example of a line extension. ((( FALSE )))
Sales of private-label brands continue to increase, particularly in big-box stores such as Costco and Trader Joe’s. ((( TRUE )))
When Jaclyn selects a case of Sprite at the grocery store for her party, she is attracted by the holiday-themed box featuring green and white bells. What Jaclyn is seeing is referred to as the secondary package. ((( TRUE )))
Jenny traded in her old car for her heart’s desire, a new Lexus sedan. The Lexus is considered a specialty product. ((( TRUE )))
Nike sells shoes, clothing, and athletic gear. These represent the breadth of Nike’s products. ((( TRUE )))
A decrease in a company’s product depth will always hurt its marketing efforts. ((( FALSE )))
A familiar brand name always helps the sales of a product. ((( FALSE )))
Claire always liked Lands’ End raincoats, so when she was shopping for a winter coat, she ordered one from Lands’ End without thinking too much about it. This demonstrates how brands facilitate purchases. ((( TRUE )))
Brands are assets that can be legally protected through trademarks. ((( TRUE )))
When Mars manufactures M&M’s, Snickers, and other confectionary products, or when Trek designs bicycles, each company’s core question is: How do we produce this product in the least expensive manner? ((( FALSE )))
A product is ________ that can be offered through a voluntary marketing exchange. ((( anything of value to consumers )))
The complete set of all products offered by a firm is called its ((( product mix. )))
A __________ is a group of associated items that consumers tend to use together or think of as part of a group of similar products. ((( product line )))
For a major university, undergraduate studies, graduate studies, and professional programs would be __________ within the university’s product mix. ((( product lines )))
It is almost impossible to watch a sporting event on television without seeing Nike’s "swoosh" check mark, which is Nike’s ((( symbol. )))
In 2015, Nike signed an 8-year, $1 billion apparel deal with the NBA. This is an example of a deal for _______ rights. ((( licensing )))
Zappos is a successful online shoe company. One of the difficulties in running a shoe company is the need to have significant __________, a large number of items in each product line. ((( product line depth )))
Marketers with successful brands sometimes hesitate to expand their brands because ((( it is costly to maintain many product lines, and it might weaken the brand’s meaning. )))
All of the following scenarios illustrate a reason why a firm would eliminate an item within a product line except ((( the firm has decided to capture new markets. )))
For many years, General Electric had a corporate strategy of being among the top three firms in any market in which it operated; if it could not achieve a top-three position, it would exit the market. This strategy often resulted in the company __________ when certain product lines failed to meet this expectation. ((( decreasing product mix breadth )))
The decision to delete a product is never taken lightly because, generally, manufacturers ((( have made substantial investments in product development and manufacturing. )))
A university that has separate graduate and undergraduate admission offices recognizes that these are distinct ((( product lines. )))
One important purpose of a brand is to ((( increase consumer recognition and awareness of product offerings. )))
In addition to extensive online and outdoor advertising, Under Armour has intensified its ______ efforts by signing Washington Nationals baseball star Bryce Harper to the biggest endorsement deal for an MLB player. ((( branding )))
For a brand name to be effective, it needs to be ((( easy for consumers to recognize and remember. )))
One key feature of the value of a strong brand is that ((( it can protect the firm from competition. )))
When consumers associate a brand with a certain level of quality and familiar attributes, allowing consumers to make quick decisions, the brand ((( facilitates purchasing. )))
Parents of young children have been known to drive out of their way so their kids will not see McDonald’s Golden Arches and plead with their parents to stop. For McDonald’s, the Golden Arches reduce marketing costs because people (including young children) ((( know what the Golden Arches brand symbol means. )))
Firms that use the same brand name for new products can spend relatively less on marketing costs for the new product because ((( people already know what the brand means. )))
The XYZ car manufacturing company is advertising its new hybrid vehicle. It understands that its competition, Toyota’s hybrid car, the Prius, is known for being economical, a good value, stylish, and good for the environment. Toyota has the advantage of ((( brand associations. )))
When Amanda shops for lipstick, she always buys Really Red by Cover Girl. This is an example of ((( brand loyalty. )))
The value of a brand is often calculated by assessing the ((( earning potential of the brand over the next 12 months. )))
Marketing expenditures allocated carefully can result in greater brand recognition, awareness, perceived value, and consumer loyalty for the brand, which all enhance the brand’s ((( equity. )))
The Doritos advertising effort of "Live Mas," which is Spanish for "Live More," is meant to suggest a lifestyle aspiration. If consumers make a connection between the Doritos brand and this promotional effort, then Doritos has created ((( a brand association. )))
Firms spend millions of dollars annually to build brand equity, recognizing that brand equity contributes to ((( profitability. )))
All of the following are included in the main criteria used for determining how "good" a brand is, or how much equity it has except ((( brand conceptualization. )))
If many consumers in the marketplace are familiar with a brand and what it stands for and have an opinion about the brand, the brand has considerable ((( awareness. )))
Whether they like them or not, most U.S. automobile customers are familiar with and have an opinion about American-made cars. This familiarity makes it easier for consumers to ((( make purchase decisions. )))
Sometimes brand names become synonymous with a product itself. If that happens, the brand ((( could lose its trademark status. )))
In the case of Band-Aid adhesive bandages, the brand name has ((( become synonymous with the product itself. )))
Salina is working to create greater brand awareness for her company’s new line of fitness trackers. To increase brand awareness, the least important information that should be included in promotions is ((( the product history. )))
Why would a firm spend over $2 million for a 30-second ad on television during the Super Bowl? ((( because the Super Bowl offers an opportunity to create significant brand awareness )))
Relatively few consumers like to go to the dentist. Dental insurance plans that pay for regular checkups increase the __________ of dental care by reducing the cost to the consumer. ((( perceived value )))
In a competitive market, perceived value is determined by consumers mostly ((( in relation to the value of its close competitors. )))
Nora is deciding whether to purchase brand-name sneakers or a less expensive store brand. She has purchased other shoes with the same brand name in the past but was only marginally satisfied. In this situation, Nora is likely to purchase the store-brand sneakers because they offer ((( greater perceived value. )))
Matt was passionate about Hollister. It was the only place he’d buy his clothes. If anyone asked him about clothes, he would talk for what seemed like hours about why he only shopped there. From a strictly marketing perspective, this word of mouth is an element of ((( brand loyalty. )))
Jenna always buys Stacy’s brand pita chips. She does not even consider alternatives. Jenna is a __________ customer. ((( brand-loyal )))
All of the following statements regarding brand loyalty are true ((( the marketing costs of reaching loyal customers are typically very high. )))
For marketers, one of the benefits of having achieved brand loyalty is ((( lower marketing costs associated with reaching loyal customers. )))
For luxury carmakers, the lowest-end models in their product lines often represent only a small portion of their sales. Yet these models are critically important when it comes to attracting new customers and, potentially, establishing their lifelong ((( loyalty to the brand. )))
Imagine that you are the marketing manager of a hotel chain that wants to implement a customer reward and loyalty program. You research the firm Sabre Hospitality Solutions, which offers its services to global and boutique hotel chains to develop and execute these types of programs. Sabre Hospitality Solutions is a company that focuses on ((( customer relationship management. )))
Frequent buyer/user award programs are used to ((( maintain contact with loyal customers. )))
A tablet programmed with key customers’ birthdates, wine preferences, and food allergies is a(n) __________ tool. ((( CRM )))
Brands can be owned by ((( any firm in the supply chain. )))
Adidas, Pepsi, Kitchen-Aid, and Sony are considered famous ________ brands because they are owned and managed by the manufacturer. ((( national )))
Which of the following is the best example of a family brand? ((( Tostitos Multigrain Scoops and Tostitos Chunky Salsa )))
The basic reason manufacturers spend time and money building their own brands is to ((( build brand equity. )))
Procter & Gamble is a huge national brand manufacturer. By owning its brands like Tide and Crest, P&G ((( has greater control over its marketing strategy. )))
Brands that are owned by ___________ are called private-label brands. ((( retailers )))
________ is another term for private-label brands. ((( Store brands )))
Private-label brands are developed and managed by ((( retailers. )))
Consumers might not realize that Old Spice health and wellness products and Iams pet nutrition products are made by the same company—Procter & Gamble. Old Spice and Iams are known as ((( individual brands. )))
When Procter & Gamble added teeth-whitening products under the Crest brand, the firm was engaged in ((( brand extension. )))
Why is brand extension a popular marketing strategy? ((( It allows the firm to spend less on creating brand awareness and associations. )))
All of the following are potential benefits of brand extension except ((( eliminating competition. )))
One of the categories of products for which brand extension is especially logical is ((( complementary goods. )))
Jake developed a toothpaste using only natural ingredients, and he has been quite successful selling the product in health food stores and some grocery stores. He has recently developed a toothbrush using bamboo and natural components. Jake is considering a __________, giving the toothbrush the same brand name as the toothpaste in order to create greater brand awareness. ((( brand extension )))
Ferrari is well known as a brand of luxury sports cars; accordingly, it has leveraged its brand name to introduce clothing offerings emblazoned with its horse logo. It has also licensed its logo to 68 different products, from sunglasses to guitars. Ferrari might be at risk of which of the following? ((( brand dilution )))
Marco tried a new fruit-flavored beverage and thought it was awful. He was especially disappointed because he had liked the dried fruit snacks marketed under the same brand name. Now he wasn’t sure he even wanted to buy the snacks he had liked before. This highlights a problem in branding known as ((( brand dilution. )))
Harley Davidson’s cake decorating kit tried to appeal to the brand’s unparalleled consumer loyalty, but it was considered too tame by their consumer base. This is an example of ((( brand dilution. )))
A ________ refers to the use of the same brand in a different product line. ((( brand extension )))
Marketers with luxury brands use brand extension cautiously in order to avoid ((( diluting the core brand. )))
Shell MasterCard, created cooperatively by Shell Oil Company and MasterCard, is an example of ((( co-branding. )))
Efforts to change a brand’s focus to target new markets or change the image of a brand are called ((( brand repositioning. )))
When Disney flooded retail stores with products based on its movie, this illustrated ((( brand licensing. )))
Ella had been using an imported brand of shampoo for several years, but she could no longer find it anywhere. As she was considering what kind of shampoo to try, she was surprised to see ads for Procter & Gamble’s Head and Shoulders being promoted as a glamorous health-oriented product. She had always thought of Head & Shoulders as an antidandruff shampoo. The new ads suggest that Head & Shoulders has most likely been ((( rebranded. )))
All of the following statements regarding secondary packaging are true except ((( It is of little value to the average consumer. )))
A __________ package is the one a consumer uses. A __________ package is used by retailers to display and sell the product. ((( primary; secondary )))
All of the following are considered an important function of labels on products and packages except ((( protecting against damage to the product. )))
What U.S. government agency has the primary responsibility in reviewing food and package labels to ensure claims made by the manufacturer about the product are true? ((( Food and Drug Administration )))
Kendra is taking a backpacking trip across Europe. She wants to make sure that the backpack she chooses is the best one for her adventure, so she is spending a considerable amount of time comparing alternatives to make sure she selects the best one. For Kendra, this backpack represents what type of product? ((( shopping )))
Zappos, an online shoe company, knows shoes are typically a(n) __________ good, with consumers often spending time comparing alternatives. It overcomes that aspect of consumers’ search process by offering a free, no-questions-asked return policy. ((( shopping )))
Marketers selling milk, bread, and other consumer staples know that most customers do not spend much time searching or comparing alternatives. For most consumers, these are _____________ goods. ((( convenience )))
Andy likes gourmet popcorn, and will spend time trying to find his favorite brand. His girlfriend, Joanne, loves popcorn but doesn’t care about brands. For Andy, popcorn is a __________ good while for Joanne it is a __________ good. ((( specialty; convenience )))
Marketers need to think about the product offer on three levels. Which of the following levels includes associated services such as warranties, financing, support, and after-sale service? ((( augmented product )))
All of the following are considered part of the "actual product" level of the product offer except ((( product warranty. )))
When manufacturers try to understand what customers are looking for, they are thinking about the _______ component of the product offer. ((( core customer value )))
Suppose that Walgreens (a major drug store chain) wants to introduce its own brand of cough medicine that is similar in contents and packaging to a national brand, but at a lower cost. What kind of brand would Walgreens be introducing? ((( private-label brand )))
When a company launches a new product line with an existing brand name, this is known as ((( a brand extension. )))
Which of the following brand strategies is being used when the local ice cream shop decides to add a new flavor to its menu? ((( a line extension )))
When a restaurant chain, Big Burgers, launches its own brand of frozen meals, this is an example of a ((( brand extension. )))
Which of the following best describes when Johnson & Johnson introduces a travel-sized package for its existing baby oil product? ((( line extension )))
What aspect of the product is being described when a used-car salesperson explains that each car comes with a one-year extended warranty and a two-year maintenance plan? ((( associated services )))
Some credit cards provide free liability insurance, funded by the bank issuing the card, when a customer pays for a rental car using the card. This extra insurance is an example of ((( an associated service. )))
When a salesperson calls you to sell a cemetery plot, this is most likely what type of product? ((( unsought product )))
Where on a product’s packaging would one most likely find nutrient content and country of origin? ((( on the label )))
Returnable packaging, use of 3D printing, and flexible packaging are examples of ((( sustainable packaging. )))
When Kraft’s Philadelphia cream cheese changed its packaging from round to oval to ensure front-facing displays that can’t spin and to fit more into the same shelf space, this was a subtle way in which the firm _________ its product. ((( repositioned )))
How did consumers respond when Chobani reduced the size of its Greek yogurt containers by 12 percent? ((( They used social media to complain about the change. )))
A toothpaste tube is an example of a ________ package. ((( primary )))
When Meg planned her trip to Australia, she spent months evaluating packages, airfares, and hotel accommodations. Meg’s trip is an example of a(n) ________ product. ((( shopping )))
Price is the cash expenditure plus taxes that consumers have to pay for a good or service. ((( FALSE )))
The key to successful pricing is to match the product with the consumer’s perception of value. ((( TRUE )))
Price is the only part of the marketing mix that does not generate costs. ((( TRUE )))
Pricing strategies should be aligned with a firm’s overall goals and objectives. ((( TRUE )))
When a firm has a particular profit goal as its overriding concern, it will use target return pricing to meet the profit objective. ((( FALSE )))
Rarely is the lowest-price product offering the dominant brand in a given market. ((( TRUE )))
American Airlines just reduced its fares for summer flights by $100. Delta Airlines changes its pricing structure and reduces its flights by $100 as well. Delta is employing status quo pricing. ((( TRUE )))
A demand curve shows the relationship between income and demand. ((( FALSE )))
The demand curve for prestige products generally slopes downward due to higher prices. ((( FALSE )))
Because consumers are generally more sensitive to price increases than to price decreases, it is easier to lose current customers with a price increase than it is to gain new customers with a price decrease. ((( TRUE )))
When the price of milk goes up, demand does not fall significantly, because people still need to buy milk. However, if the price of T-bone steaks rises beyond a certain point, people will buy fewer of them because they can turn to the many substitutes for this cut of meat. This refers to price elasticity of demand. ((( TRUE )))
Dynamic pricing is also referred to as individualized pricing. ((( TRUE )))
Brands that have developed loyal customers have a higher price elasticity of demand. ((( FALSE )))
In U.S. markets, there are many substitute products for Fruit Loops cereal, suggesting the price elasticity of demand for Fruit Loops is high. ((( TRUE )))
Costs related to supply and costs related to demand are the two primary cost categories. ((( FALSE )))
At the break-even point, profits are maximized. ((( FALSE )))
Diana owns a boutique specializing in ball gowns. Sales are stable and Diana feels it is time she had a 20 percent increase in her salary. If Diana takes this increase in compensation, it will decrease the break-even quantity of gowns she needs to sell on a monthly basis. ((( FALSE )))
Pure competition occurs when there are many firms competing for customers in a given market but their products are differentiated. ((( FALSE )))
An example of an objective set by a sales-oriented company is to institute a company-wide policy that all products must provide for at least an 18 percent profit margin to reach a particular profit goal for the firm. ((( FALSE )))
If a firm is engaged in monopolistic competition, it should seek a way to differentiate itself. ((( TRUE )))
When CarMax promises a “no-haggle” pricing structure, it exhibits _________ because it provides additional value to potential used car buyers by making the process simple and easy. ((( a customer orientation )))
Earl was known for driving 30 miles just to save a dollar on the price of his favorite beverage. Earl perceived price as _______ for a good or service, while most consumers recognize price as the _______ made to acquire a good or service. ((( the money paid; overall sacrifice )))
All of the following are included in the full price of a product or service except ((( the price of alternative products and services. )))
Consumers generally believe that ________ is one of the most important factors in their purchase decisions. ((( price )))
Unlike product, promotion, or place, price is the only part of the marketing mix ((( that generates revenue. )))
In developing marketing strategies, why is price often the most challenging of the four Ps to manage? ((( because it is the least understood element of the marketing mix )))
Historically, prices were ((( rarely changed except in response to radical shifts in market conditions. )))
Margaret has been invited to a fancy dinner party and wants to bring a good bottle of wine as a gift for the host. Since she does not know much about wine, she will likely use the price of the wines as ((( an indicator of quality. )))
All of the following are included in the five Cs of pricing except ((( collaboration. )))
Tess is the marketing manager for a fast-food restaurant chain. She uses a target return pricing strategy because her firm’s primary objective is to ((( increase profits. )))
Gary is the marketing manager for an automobile dealership. His boss tells him the firm’s primary goal is to increase its local market share from 15 to 30 percent. His firm is using a ________ orientation. ((( sales )))
When Delta increases its average fares, American Airlines and United often follow with similar increases. This is an example of ((( status quo pricing. )))
Bernard’s firm has set corporate direction to become one of the leaders in each of its significant market segments. It was Bernard’s job to examine the firm’s pricing strategy to determine how to maximize market share, even at the expense of profits in the short run. What kind of company objective would guide Bernard’s effort? ((( sales-oriented )))
Naomi tells her sales representatives the goal is to generate at least a 20 percent return on investment for all of the industrial building supplies they sell. Naomi is using a _______ pricing strategy. ((( target return )))
A _______ strategy involves accurately measuring all the factors needed to predict sales and profits at various price levels, so that the price level that produces the highest return can be chosen. ((( maximizing profits )))
Health clubs often use a low introductory offer price to get people to join their club. These low prices represent a _______ pricing strategy. ((( sales orientation )))
Many years ago Honda’s Accord and Ford’s Taurus were the two top-selling cars in the United States. As the year was coming to an end, Ford cut the price of the Taurus, hoping to outsell the Accord and allow Ford to claim that "Taurus is the best-selling car in America." Ford was using a _______ pricing strategy. ((( sales orientation )))
Julia wants her firm’s gourmet snacks to be the leading brand in the U.S. market. When adopting a pricing strategy designed to gain market share, she should remember that ((( rarely is the lowest-price offering the dominant brand in a market. )))
Sharon knew that her established customers liked her product much better than her competitor’s. She was planning to expand into new markets, and she was considering pricing. She was leaning toward charging a higher price than competitors to help demonstrate that hers was a high-quality product. Sharon was considering ((( premium pricing. )))
When Ursula decides how to price new products in her gift store, she measures the value of her product offerings against those of the other stores in her area. Ursula uses a _______ pricing strategy. ((( competitor-oriented )))
Ryan gave the manager of his convenience store a set of binoculars so she could see the gasoline prices charged by the other convenience store at that intersection. Ryan told the manager to always match the gasoline prices of the other store. Ryan is using a _______ pricing strategy. ((( status quo )))
When firms set prices similar to those of competitors, they are following a strategy of ((( competitive parity. )))
In many high-end resort markets, Westin hotels compete directly with Crown Plaza hotels. When it comes to pricing, Westin tends to charge its guests similar rates to what the Crown Plaza hotels charge. Westin is using a _______ pricing strategy. ((( competitive parity )))
A customer orientation toward pricing implicitly invokes the concept of ((( value. )))
A “no-haggle” pricing policy is a type of _____ pricing strategy. ((( customer-oriented )))
Julia’s is an upscale women’s clothing store. Prices are based on customers’ beliefs about the value of the clothing. The store focuses on a limited target market and provides excellent customer service. Julia’s is using a _______ pricing strategy. ((( customer-oriented )))
A strategy of setting prices based on how customers develop their perceptions of value can often be the most effective pricing strategy, especially if the strategy ((( is supported by consistent advertising and distribution strategies. )))
Traditional demand curve economic theory is used by marketers to understand ________ in the five Cs of pricing. ((( customers )))
Customers must see value in a product or service before they are willing to exchange time or money to obtain it, but not all customers see the same value in a product. To analyze how many units will be sold at any given price point, marketers draw on ((( a demand curve. )))
A demand curve shows the relationship between _______ during a specific period of time. ((( price and demand )))
A demand curve is built on the assumption that ((( everything but price and demand remains the same. )))
According to a typical demand curve, the higher the price, ((( the lower the quantity consumers will buy. )))
There is an old saying "If you have to ask the price of a yacht, you cannot afford it." Products like yachts are most likely to be associated with ((( prestige pricing. )))
The price elasticity of demand for a teeth-whitening kit is -1. The market for this product is considered ((( elastic. )))
The price elasticity of demand for a specific brand of deodorant is −0.9. The market for this product is considered ((( inelastic. )))
When it comes to measuring consumers’ price sensitivity, products are viewed as either ((( elastic or inelastic. )))
For which of the following is demand likely to be least sensitive to price increases? ((( prescription drugs )))
For which of the following is demand likely to be most sensitive to price increases? ((( a specific brand of soft drink )))
A study found that, among addicted smokers, a 10 percent increase in the price of cigarettes resulted in a 2 percent decrease in quantity demanded. For these consumers, cigarettes have a(n) _______ price elasticity demand. ((( inelastic )))
What situation is occurring if a 1 percent decrease in price results in more than a 1 percent increase in quantity demand? ((( Demand is price elastic. )))
Assume the demand for electricity, a necessity with few substitutes, is -0.2. If the electric company raised its rates by 10 percent, we would expect ((( a 2 percent decrease in quantity demanded. )))
If a 1 percent decrease in price results in less than a 1 percent increase in the quantity demanded, demand is ((( price inelastic. )))
Barry customizes Harley-Davidson motorcycles. No two cycles are alike. He notices that very few customers even ask the price of his motorcycles before they decide to purchase them. Demand for his motorcycles is probably ((( price inelastic. )))
The observation that consumers are generally more sensitive to price increases than to price decreases suggests that ((( it is easier to lose customers with a price increase than to gain customers with a price decrease. )))
The Coffee Express company is located in a business district with few customers on the weekend. To attract customers on Saturday and Sundays, it reduces its prices on these two days. This is an example of ((( dynamic pricing. )))
French luxury goods manufacturer and retailer Hermès is known for making expensive leather goods. But paying $300,168 for a handbag at auction, which is over the standard retail price of $280,000, is extraordinary, and not for the casual shopper. The handbag is considered a ((( prestige product. )))
Natalie operates on a pretty tight budget. She is a price-conscious shopper and usually buys store or generic brands to save money. Recently, however, Natalie was given a pretty substantial raise. As such, she has altered her shopping patterns and now regularly buys more expensive, name-brand goods. This is an example of ((( the income effect. )))
Sales of national brands of orange juice tend to increase when the economy is doing well, while sales of generic orange juice increase when the economy is not doing well. This is an example of how _______ impacts demand for products. ((( the income effect )))
Rodi owns Hallman’s auto repair service. He has observed over the years that customers keep their high-mileage cars longer when the economy is doing poorly, creating demand for his maintenance and repair service. Rodi has observed the impact of _______ on demand for his service. ((( the income effect )))
There are many options available to consumers when it comes to breakfast cereals. So, if Kellogg’s significantly increases the price of Rice Krispies, consumers are more apt to buy alternate cereals instead. This illustrates which concept? ((( the substitution effect )))
The more substitutes that exist in a market, ((( the more sensitive consumers will be to changes in the price of a particular product. )))
Brad always buys and uses Nike brand golf balls. If he finds a Titleist or Callaway ball in the rough, he gives it away. Brand-loyal golfers like Brad allow Nike to charge a higher price and not lose many sales. By building a strong brand, Nike has effectively ((( reduced the price elasticity of demand for its products. )))
Marketers spend millions of dollars annually trying to create or reinforce brand loyalty. Brand loyalty changes the demand curve for the firm’s products by ((( reducing the price elasticity of demand. )))
Cross-price elasticity is the ((( percentage change in quantity demanded of Product A compared to the percentage change in price of Product B. )))
_______ products are products whose demands are positively related and as such, they rise or fall together. ((( Complementary )))
Which of the following is the most logical example of complementary products? ((( hot dogs and hot dog buns )))
Bill is a yacht broker in the southeastern United States. For years he has had difficulty selling large yachts locally because there were few places to dock these boats. Yachts and spaces to dock them are an example of ((( complementary products. )))
If the price for a product increases, the demand for the complementary product will ((( decrease. )))
Frank’s Heating and Air Conditioning Company specializes in electric heat pumps. Frank keeps track of the price of natural gas, knowing that ((( an increase in the price of natural gas will increase demand for his electrical heating systems. )))
Managers of Wendy’s fast-food restaurants keep track of prices at competitors such as McDonald’s, Burger King, and Arby’s, knowing that a decrease in the prices at these other fast-food restaurants will ((( decrease demand for Wendy’s products. )))
One problem in relying on price elasticity and demand curves when setting prices is ((( the way a product or service is marketed can have a profound impact on price elasticity. )))
In general, prices should not be based on costs because ((( consumers make their purchase decisions based on perceived value. )))
Consider a bakery like Entenmann’s: The majority of the ___________ costs are the cost of the ingredients, primarily flour. ((( variable )))
David manages a Shoney’s restaurant. He is considering staying open later in the evening. For David, the variable costs associated with staying open longer hours will include all of the following except ((( rent on the restaurant building. )))
Variable costs change with ((( changes in the quantity being produced. )))
Raymond estimates that the fixed costs associated with opening a new bank branch are $500,000. He expects the branch to attract 1,000 new customer accounts in the first year, each of which will cost $50 per year to service. He also expects to generate $100,000 per year in revenue. For Raymond, the total cost of opening the new branch and remaining open for one year will be ((( $550,000. )))
At the break-even point, ((( profits are zero. )))
The break-even point is estimated by ((( dividing fixed costs by contribution per unit. )))
If the fixed costs of manufacturing a new cell phone are $10,000, the sales price is $60, and variable cost per unit is $20, the break-even point is ((( 250 units. )))
Jacob rents rooms in his hotel for an average of $100 per night. The variable cost per rented room is $20, to cover maid service and utilities. His fixed costs are $100,000 and his profit last year was $20,000. For Jacob, the contribution per unit is ((( $80. )))
The contribution per unit is ((( price minus variable cost per unit. )))
Jason rents rooms in his hotel for an average of $100 per night. The variable cost per rented room is $20. His fixed costs are $100,000 and his target profit is $20,000. For Jason, to earn his target profit, he will need to rent out ________ rooms. ((( 1,500 )))
Break-even analysis is useful because it allows managers to ((( estimate the quantity they will need to sell at a given price to break even. )))
One of the limitations associated with break-even analysis is that ((( it assumes that there is only one price. )))
Which of the following markets is most likely to be characterized by oligopolistic competition in the United States? ((( smartphone service providers )))
Because there are only a few firms in markets with oligopolistic competition, ((( price wars may occur. )))
In a market with _______ there are many firms providing differentiated products. ((( monopolistic competition )))
Because there are many firms in monopolistic competition markets, ((( the many competitors will focus on product differentiation. )))
In _______ many firms provide similar products that are considered substitutes for each other. ((( pure competition )))
Which of the following is most likely to be characterized by pure competition in the United States? ((( soybeans )))
Because there are many firms with similar products in purely competitive markets, ((( price is determined by the laws of supply and demand. )))
If a firm in a purely competitive market can differentiate its product or service, it becomes part of a(n) _______ market. ((( monopolistic competition )))
How can a company find its way out of a market characterized by pure competition? ((( differentiate the product in some way, even by packaging, so customers will see it as distinct )))
When the price for Blu-ray players dropped, the demand for DVD players went down, so DVD players and Blu-ray players are ((( substitute products. )))
In one year, the Hotel by the Shore incurred $100,000 in fixed costs. Because the hotel booked 10,000 room nights, its total variable cost is $100,000 (10,000 room nights ´ $10 per room). Thus, its total cost is ((( $200,000. )))
There is often only one provider of cable television services in each region of the country: Time Warner is in New York, Comcast is in most of New England, and so forth. When Comcast recently proposed a plan to buy Time Warner, the purchase ultimately could not be completed, mostly due to concerns that it would have caused Comcast to become an overly large ________with too much power. ((( monopolist )))
Predatory pricing ((( is illegal in the United States under both the Sherman Antitrust Act and the Federal Trade Commission Act. )))
Diane owns a bakery where she sells cupcakes. Two blocks down there is another bakery, CC’s Bakery, that sells cupcakes for $1 less than Diane. Diane decides to lower her price and match CC’s Bakery prices. What type of pricing strategy is Diane implementing? ((( competitor-oriented pricing )))
The point at which the number of units sold generates enough revenue to equal the total costs of running an operation is known as the ((( break-even point. )))
The commercial airline industry is considered what type of market? ((( oligopolistic competition )))
In the four Ps of marketing, place refers to all the activities required to get the right product to the right customer when that customer wants it. ((( TRUE )))
Marketing channel management adds value because it gets products to customers efficiently, quickly, and at low cost. ((( TRUE )))
A distribution center and a fulfillment center are one and the same. ((( FALSE )))
Marketing channel management and supply chain management are virtually the same. ((( TRUE )))
In a direct marketing channel, there are no intermediaries between the buyer and the seller. ((( TRUE )))
Radio frequency identification (RFID) tags automatically transmit information about a container’s contents or individual products. ((( TRUE )))
Quick response (QR) inventory systems allow retailers to maximize their inventory holdings. ((( FALSE )))
Marketing channel relationships evolve when the parties have common goals. ((( TRUE )))
Conflicts within a supply chain tend to be more pronounced when the members are part of a corporate vertical marketing system. ((( FALSE )))
Universal product codes (UPCs) are used to describe products for inventory taxation purposes. ((( FALSE )))
Retailers generally have no prior knowledge of the merchandise they will be receiving from suppliers in any particular shipment. ((( FALSE )))
When a firm is just starting out or entering a new market, it doesn’t typically have the option of designing the best marketing channel structure. ((( TRUE )))
When manufacturers such as Pampered Chef sell directly to consumers, they perform both production and retailing activities. ((( TRUE )))
Distribution centers are always operated by distribution specialists. ((( FALSE )))
Wholesalers are often used in an indirect marketing channel. ((( TRUE )))
When two competing retailers have a disagreement, it is an example of a horizontal channel conflict. ((( TRUE )))
The vertical marketing system that exhibits the most formalization and control is the corporate vertical marketing system. ((( TRUE )))
When Walmart threatens to punish or punishes the other channel member for not undertaking certain tasks, such as if it were to delay payment for a late delivery, this is an example of expertise power. ((( FALSE )))
The computer-to-computer exchange of business documents from a retailer to a vendor and back is referred to as vendor-managed inventory (VMI). ((( FALSE )))
One reason retailers are reluctant to use distribution centers is that retail store space is typically much less expensive than space at a distribution center, so it is more cost-effective to store merchandise and get it ready for sale at the retail store rather than at a distribution center. ((( FALSE )))
Sophie made pies and sold them from her food truck to local businesses. This is an example of a(n) ((( direct marketing channel. )))
Which element of the marketing mix specifically deals with supply chain management? ((( place )))
Students of marketing often overlook or underestimate the importance of place in the marketing mix simply because ((( it happens behind the scenes. )))
In marketing’s four Ps, place refers to all activities required to get ((( the right product to the right customer when that customer wants it. )))
Dawn works for a firm that buys products directly from the manufacturer and resells them to retailers, who then sell the products to consumers. Dawn works for a ((( wholesaler. )))
Stores like Home Depot and Costco act as wholesalers when they ((( sell to contractors or restaurant owners. )))
When Costco sells to consumers directly, it acts as a _______; when it sells to other businesses, such as a restaurant owner, it acts as a _______. ((( retailer; wholesaler )))
A(n) _______ is used in the shipment of products directly to customers. ((( fulfillment center )))
Naomi knows she has to order her store’s Christmas holiday merchandise in April to ensure delivery before the holiday season. Naomi is concerned with the supply chain management goal of ((( providing products at the right time. )))
A ___________ program is a promotional strategy that encourages people to pass along a marketing message to other potential consumers. ((( virtual marketing )))
A ________ is a facility for the receipt, storage, and redistribution of goods to company stores. ((( distribution center )))
A distribution center is typically operated by ((( retailers, manufacturers, or distribution specialists. )))
Having no intermediaries between the buyer and seller is a defining characteristic of a(n) _______ marketing channel. ((( direct )))
In a(n) _______ marketing channel, one or more intermediaries work with manufacturers to provide goods and services to customers. ((( indirect )))
Trisha makes purses out of old blue jeans and sells them to customers at local craft fairs. What type of marketing channel does this represent? ((( direct )))
A local furniture store buys furniture from various manufacturers and resells the furniture to its customers. What type of marketing channel does this represent? ((( indirect )))
When a customer purchases a smartphone at Best Buy, all of the following information flows in the supply chain are started except ((( the purchase is added to the customer’s purchasing habit records. )))
Steven managed an auto parts store in the 1990s. At that time, stockouts—failure to have the parts auto mechanics needed to do their work that day—increased the likelihood of the mechanics becoming customers of competing auto supply stores. To avoid this problem and keep his business customers, Steven most likely ((( stockpiled inventory, adding to the cost of providing auto parts. )))
In addition to merchandise and payments, information flows throughout a supply chain. All of the following statements are a good characterization of the flow of information in a supply chain except ((( customers will send and receive information from stores and manufacturers. )))
Typically, manufacturers and retailers exchange business documents through a(n) __________ system, the computer-to-computer exchange of business documents from a retailer to a vendor and back. ((( electronic data interchange )))
Flora is frustrated with her company’s supply chain management information system. She wants to be able to receive sales data, initiate purchase orders, send and receive invoices, and receive returned merchandise documentation. Flora needs a(n) ((( electronic data interchange system. )))
Today, when a customer orders merchandise from an online vendor, the vendor usually sends an immediate order confirmation message by e-mail. Usually within a day or two, a second message arrives stating that the order is in the mail. This second message is a type of ((( advanced shipping notice. )))
Roland has just received notification from a vendor that his clothing merchandise order has been processed and dispatched. Roland has just received a(n) ((( advanced shipping notice. )))
In vendor-managed inventory systems, ((( retailers send sales information to the manufacturer. )))
All of the following are advantages of using a distribution center except ((( customers know they are dealing with a more sophisticated and financially well-off operation if the firm uses a distribution center. )))
All of the following are activities carried on in a distribution center except ((( distributing paychecks and paystubs for retail employees. )))
Ted is glad his company finally converted to an RFID system. Now, he no longer needs to go through all the new goods to make sure what they ordered was what they received. Ted is responsible for __________ in his company. ((( checking )))
Radio frequency identification tags are ((( tiny computer chips that transmit information about a container’s contents. )))
__________ is when vendors ship merchandise prepackaged in the quantity required for each store to the distribution center. ((( Cross-docking )))
In a __________ distribution center, merchandise moves from vendors’ trucks to retailers’ delivery trucks in a matter of hours. ((( cross-docking )))
Walmart’s customers have come to expect to find P&G products in stores, and P&G depends on Walmart to purchase a good portion of its output to sell to its own customers. This scenario represents the first phase of a ((( vertical marketing system. )))
Colin has just received a delivery from the company’s distribution center. He opens the containers and finds the popcorn and snacks are all bar-coded and priced and the package includes an end-of-the-aisle display rack. Colin has received a(n) __________ shipment. ((( floor-ready )))
Some retailers require their suppliers to ship merchandise ___________, thus eliminating the time and expense associated with ticketing and marking. ((( floor-ready )))
Getting merchandise floor-ready entails ((( ticketing and marking. )))
Which of the following is true regarding the ticketing and marking process? ((( It is more efficient for a retailer to perform these activities at a distribution center than in its stores. )))
Each time a politician or celebrity writes a book, bookstores can expect at least some customers to want the book, but whether or not it will become a best-seller is less certain. The bookstore’s primary inventory management challenge is ((( having enough books to satisfy customer demands versus the cost of having the inventory. )))
H&M has adopted the inventory management system that delivers less merchandise on a more frequent basis than in traditional inventory systems. The firm gets the merchandise just in time for it to be used in the manufacture of another product or for sale when the customer wants it. This is an example of a ___________ inventory system. ((( quick response (QR) )))
Forever 21 has adopted an inventory system that reduces the amount of time between the recognition that an order needs to be placed and the arrival of the needed merchandise at the seller’s store, available for sale. This increases product availability and lowers inventory investment. These benefits are attributed to the _______ system. ((( just-in-time (JIT) )))
At the BMW plant in Spartanburg, South Carolina, rather than use a typical order-to-delivery process, it has suppliers deliver parts every four hours when the plant is in operation. BMW uses a ______ inventory control system. ((( just-in-time (JIT) )))
Because manufacturers with JIT systems produce merchandise closer to the time of sale, they can ((( reduce inventories needed to satisfy retailers’ demand. )))
Especially in the fashion industry where styles and trends change rapidly, a quick response system can ((( align deliveries more closely with actual sales. )))
A recent innovation by Amazon, the Vendor Flex program, seeks to lower overall transportation costs but also creates new forms of channel conflict with competitors. As a partner in the Vendor Flex program, P&G agreed to allow Amazon to build fulfillment centers within P&G’s own warehouses, thus helping to eliminate some of the costs of transporting P&G’s products to Amazon’s fulfillment centers. Believing that the program was giving its competitor Amazon an unfair advantage, Target reacted vigorously, moving all P&G products from prominent end-cap positions in its stores to less prestigious and less visible locations. Target also stopped using P&G as its primary source of advice for planning merchandising strategies within each category. This is an example of ____________ channel conflict. ((( horizontal )))
The Stanley company wants Home Depot to carry all its tools but not the tools of its competitors so that Stanley can maximize its sales. But Home Depot carries a mix of tool brands so it can maximize the sales in its tool category. This misalignment of goals will likely lead to a ________ channel conflict. ((( vertical )))
Marketing channels and supply chains comprise various buying entities that form relationships with one another. The basic motivating factor in these interactions is ((( each party wants something from the others. )))
Although conflict is likely to occur in any marketing channel, it is generally more pronounced when ((( the the channel members are independent entities. )))
Yesterday, Lorinda overheard a surprisingly unpleasant encounter between the manager of the hardware store where she works and a sales rep who sells a well-known line of tools. The rep insisted that his tools should be more prominently displayed and that a better assortment would mean more sales. The manager had other plans and told him so, and the conversation turned into a loud argument. What Lorinda observed was an example of ((( channel conflict. )))
In a(n) __________ marketing channel, none of the participants have any control over the others. ((( conventional )))
Because of its size and relative power, Walmart can easily impose controls on small manufacturers, such as PenAgain, but with large, powerful suppliers such as P&G, the control is more balanced between parties. Walmart’s various marketing channel relationships offer examples of different forms of a(n) ______________ marketing system. ((( administered vertical )))
In a(n) __________ marketing channel, several independent members each attempt to satisfy their own objectives and maximize their profits, often at the expense of the other members. ((( independent )))
In a vertical marketing system, if the system is _______, the less likely conflict will occur. ((( more formal )))
In an administered vertical marketing system, ((( there is no common ownership, and the dominant member has significant power to impose its ideas and objectives. )))
Franchising is the most common type of contractual _______ marketing system. ((( vertical )))
In a corporate vertical marketing system, ((( participants—such as warehouses, transportation companies, and retail outlets—are typically owned by a parent company to ensure harmonious relations throughout the supply chain. )))
Felicia had just taken over her family’s business after spending ten years in the marketing department of a large corporation. She met with a representative from one of her firm’s biggest customers, who told her, "We should think about how we can make the pie bigger rather than fighting over the size of the slices." She had expected a more cutthroat approach rather than this call for a ((( partnering relationship. )))
All of the following are required to build a successful strategic relationship except ((( creation of a joint venture. )))
If a firm declared that it wanted to develop a strategic relationship, but was unwilling to commit funds or any effort to make it succeed, there would be an obvious lack of ((( credible commitments. )))
TT TOYS manufactures toys. The company recently started buying paint for its toys from a Chinese firm. This Chinese company is part of TT Toys’ ((( supply chain. )))
Walmart needs P&G to satisfy its customers, and P&G recognizes that if it can keep Walmart happy, it will have more than enough business for years to come. Walmart and P&G recognize that it is in their common interest to be ((( strategic partners. )))
All of the following statements about distribution centers are true except ((( they enable retailers to carry less merchandise in individual stores. )))
A(n) __________ is a supply chain whose members act like a unified system. ((( vertical marketing system )))
A(n) __________ is a document used by a forklift driver indicating how much of each item to get from specific storage areas. ((( pick ticket )))
A(n) __________ is a 13-digit code retailers can use to track inventory. ((( UPC )))
When Cynthia’s Boutique receives dresses, they already have price tags and are on hangers. Cynthia’s Boutique receives __________ merchandise. ((( floor-ready )))
When Joe’s Chicken Shack wants chicken delivered in smaller batches more frequently, it offers its distributors an incentive in the form of special bonuses to change their schedules. Joe’s demonstrates which kind of power? ((( reward )))
Because Walmart has vast information about the consumer goods market, it might exert ___________ power over P&G by providing or withholding important market information. ((( information )))
When her company’s dry goods deliveries were late for the third time, Melissa withheld payment from her supplier until it was back on schedule. This is an example of ________ power. ((( coercive )))
Walmart may have _________ power if a supplier desperately wants to be associated with Walmart, because being known as an important Walmart supplier enables that supplier to attract other retailers’ business. ((( referent )))
When supply chain members view their goals and ultimate success as intricately linked, or ________, they develop deeper long-term relationships. ((( interdependent )))
In a retail supply chain, the ________ records the purchase information and electronically sends it to the corporate office. ((( POS terminal )))
The local auto supply store gets merchandise delivered to it by its manufacturers as soon as it has a need, reducing stockouts with minimal inventory. This demonstrates the concept of ((( vendor-managed inventory. )))
Manufacturers trying to implement a just-in-time delivery system need to start with ((( knowledge of customer demand. )))
After installing an electronic data interchange, Carmella’s gift shop was able to reduce _______, the amount of time between the recognition that an order needs to be placed and the arrival of the needed merchandise. ((( lead time )))
The amount of time between the recognition that an order needs to be placed and the arrival of the needed merchandise at the seller’s store to be available for sale is referred to as ((( lead time. )))
One of the benefits of EDI is that it can reduce lead time, which is ((( the time between the decision to place an order and the receipt of the merchandise. )))
After installing a(n) _______ in its JIT system, Chocolate Tree (a retail chocolate store) was able to reduce lead time for merchandise orders. ((( electronic data interchange system )))
Shorter lead times allow retailers ((( to reduce the needed level of inventories. )))
With more frequent shipments associated with quick response (QR) systems, a retailer is ((( more likely to have what customers want. )))
While JIT systems have many benefits, they ((( make the logistics function more complicated. )))
Leona is the logistics manager for the Barnes & Noble bookstore chain. She is weighing the many benefits of the company’s JIT system, but will need to consider that just-in-time inventory management systems increase _______ costs. ((( transportation )))
When Walmart promises to purchase larger quantities if a manufacturer will lower its wholesale price, it is exhibiting ________ power. ((( reward )))
It is difficult, but not impossible, to integrate marketing communications across all platforms, including social media. ((( TRUE )))
Each element of an integrated marketing communication (IMC) strategy must have a well-defined purpose and support, and extend the message delivered by all the other elements. ((( TRUE )))
Integrated marketing communications (IMC) represents the product element of the four Ps. ((( FALSE )))
As the number of communication media has increased, the task of understanding how best to reach target customers has become easier. ((( FALSE )))
Encoding means converting the sender’s ideas into a message, which could be verbal, visual, or both. ((( TRUE )))
Though a picture may be worth a thousand words, the most important facet of encoding is not what is received but what is sent. ((( FALSE )))
In the communication process of marketing communications, the marketing department often functions in the role of transmitter. ((( TRUE )))
If there is a difference between the message that is sent and the message that is received, it is probably due to noise. ((( TRUE )))
The sender of an integrated marketing communication controls the meaning individual receivers take from the message. ((( FALSE )))
Though advertising experts wish it were true, there is not always a direct link between a particular marketing communication and a consumer’s purchase. ((( TRUE )))
Even the best marketing communication can be wasted if the sender does not gain the attention of the consumer. ((( TRUE )))
If the marketing communication has piqued the interest of its target market, the goal of subsequent IMC messages should be to move the consumer from "I want it" to "I like it." ((( FALSE )))
Top-of-mind awareness is when consumers indicate that they know the brand when the name is presented to them. ((( FALSE )))
Betty is assessing the effect of her firm’s marketing communications. She should remember that the ultimate goal is to drive the receiver to action. ((( TRUE )))
After seeing advertisements for the Toyota Prius, Joel becomes interested and does some Internet research. However, after seeing the price for a new model, he decides to purchase a Ford Fusion instead. This is an example of the lagged effect. ((( FALSE )))
Public relations is the component of IMC that has received the greatest increase in aggregate spending. ((( FALSE )))
Mobile marketing involves marketing to customers through wireless handheld devices such as mobile phones. ((( TRUE )))
The cost of communicating directly with a potential customer is low compared with other forms of promotion. ((( FALSE )))
The message originates from the transmitter, who must be clearly identified to the intended audience. ((( FALSE )))
Twitter is a form of blogging, and is increasingly used to reach current or potential customers. ((( TRUE )))
Because the goals of IMC are only part of the overall promotional plan, they do not need to be explicit or measurable. ((( FALSE )))
When using the rule-of-thumb method for IMC budgeting, prior sales and communication activities are used for guidance. ((( TRUE )))
It may take several exposures to marketing communications before consumers are moved to buy. ((( TRUE )))
Personal selling can be successful only through the use of print, newspapers, and radio. ((( FALSE )))
Gross rating points (GRP) measure how often the audience is exposed to a communication within a specified period of time. ((( FALSE )))
Frequency describes the percentage of the target population exposed to a specific marketing communication, such as an advertisement, within a specified period of time. ((( TRUE )))
The basic goal of integrated marketing communications is to ((( communicate the value proposition to the target market. )))
Especially for marketers with new products or services, IMC is needed because ((( consumers are unlikely to buy products they are not aware of. )))
Integrated marketing communications represents the __________ element in the four Ps of a firm’s marketing mix. ((( promotion )))
Integrated marketing communications include all of the following except ((( supply chain management. )))
Carlos owns a chain of retail electronic stores. He is evaluating how he allocates his firm’s IMC budget. He receives offers from a variety of advertising media, spends money on his firm’s public relations efforts, and is considering electronic media alternatives. Carl must recognize that each IMC alternative ((( is part of the whole. )))
The three elements of any IMC strategy are the consumer, the channels, and ((( evaluation of the results. )))
As the number of communication media has increased, the task of understanding how best to reach target customers has ((( become more complex. )))
The IMC communication process includes all of the following except ((( evaluation. )))
The IMC communication process begins with __________, who (which) must be clearly identified. ((( the sender )))
In the IMC communication process, the ________ encodes the marketing communication message. ((( transmitter )))
___________ means converting the sender’s ideas into a message, which could be verbal, visual, or both. ((( Encoding )))
In integrated marketing communications, encoding involves ((( converting the sender’s ideas into a message. )))
Though a picture can be worth a thousand words, the most important facet of encoding is ((( not what is sent, but rather what is received. )))
In the IMC communication process, the __________ is the medium that carries the message. ((( communication channel )))
All of the following are a communication channel used in the IMC process except ((( supply chain. )))
The right communication channel to use in IMC is ((( the one that will connect to the desired recipients. )))
In the IMC communication process, the __________ is the person who reads, hears, or sees and processes the message being communicated. ((( receiver )))
The sender of an IMC message hopes the receivers are ((( the people for whom the message was originally intended. )))
__________ refers to the process by which the receiver interprets the sender’s message. ((( Decoding )))
If you ever watched a television commercial and at the end of the message wondered what they were promoting, you may have had trouble __________ the IMC message. ((( decoding )))
__________ is any interference in the IMC process. ((( Noise )))
In the IMC process, noise can occur as a result of lack of message clarity, a flaw in the medium, or ((( competing messages. )))
Advertising agencies often provide the IMC function of comparing their customer’s target audience with the viewer, listener, or reader profile of the communication channel being considered. The agency is most likely trying to avoid noise problems associated with ((( a poor choice of medium. )))
National manufacturers and retailers often pay a service provider to monitor television ads around the country, to ensure that their ads are shown in their entirety during the time frames that were purchased. This service provider is monitoring for IMC noise problems associated with ((( a flaw in the medium. )))
The _____________ loop allows the receiver to communicate with the sender in the IMC process and thereby informs the sender whether the message was received and decoded properly. ((( feedback )))
Most manufacturing and retailing marketers worry constantly about whether or not their IMC efforts are paying off. They assess various forms of __________ to determine what is working and what is not. ((( feedback )))
Sales data, complaints, compliments, and redemption rates for coupons and rebates are types of __________ marketers use to assess the effectiveness of their IMC efforts. ((( feedback )))
Loretta would like to know which, if any, of her firm’s IMC efforts are working. She could use all of the following except __________ to provide feedback from her efforts. ((( the channel )))
Marketers often use focus groups to learn how consumers interpret their IMC messages. Experience has shown that ((( each receiver decodes IMC messages in his or her own way. )))
The sender of an integrated marketing communication ((( has little control over what meaning any individual receiver will take from the message. )))
Though advertising experts wish it were true, there is not always a direct link between a particular form of marketing communications and ((( a consumer’s purchase. )))
Julia is driving down Interstate 95 on her way from New England to Florida and sees a billboard saying "South of the Border Restaurant and Motel is just 100 miles away." Julia has never heard of South of the Border before. After seeing the sign, she will most likely need additional exposure to the product before deciding whether to stop at South of the Border. This is an example of ((( the lagged effect. )))
All of the following are steps in the AIDA model except ((( intention. )))
In simple terms, the AIDA model is also known as the __________ model. ((( think, feel, do )))
In the AIDA model, awareness leads to ((( interest, which hopefully leads to desire and then action. )))
In the AIDA model, the think stage is the __________ stage. ((( awareness )))
__________ refers to a potential customer’s ability to recall that the brand name is a particular type of retailer, product, or service. ((( Brand awareness )))
Even the best marketing communication can be wasted if the sender does not first ((( gain the attention of the consumer. )))
Cheryl asked Nadia to help her buy some aftershave for her boyfriend. Nadia was going through a list of different brand names, when Cheryl stopped her and said, "I recognize that one." Marketers call this ((( aided recall. )))
In the AIDA model, the do stage is the __________ stage. ((( action )))
The highest level of awareness occurs when customers mention a specific brand name when asked about a product or service. Marketers call this ((( top-of-mind awareness. )))
After creating awareness that the firm’s products or services exist, the next goal of integrated marketing communications is to ((( increase the level of interest among consumers. )))
In the movie Field of Dreams, one of the memorable phrases is, "If you build it, he will come." The AIDA model would suggest that after marketers "build" a product or service and create awareness among consumers, they need to ((( create interest among consumers, persuading them to investigate further. )))
Several years ago, changes in advertising restrictions allowed pharmaceutical companies to begin promoting their products through traditional media. Today, it is common to see ads on television ending with, "Ask your doctor about [our medicine]." In addition to creating awareness about their drugs, the companies are most likely hoping to ((( stimulate interest, persuading consumers to investigate further. )))
Once the marketing communication has captured the interest of its target market, the goal of subsequent IMC messages should be to move the consumer from ((( "I like it" to "I want it." )))
Betty is assessing the effectiveness of her firm’s marketing communications. She knows the ultimate goal is to ((( drive the receiver to action. )))
If marketing communications are effective, they will ((( result in purchases by some consumers receiving the communications. )))
One of the difficulties in measuring the effectiveness of IMC efforts is the __________, where consumers do not act immediately after receiving a marketing communication. ((( lagged effect )))
Reaching the right audience with marketing communications is becoming more difficult because ((( the media environment has become more complicated. )))
When comparing the various communication channels available to marketing professionals, it becomes apparent that ((( no single channel is better than another channel. )))
The goal of any marketing communication is to ((( get the right message to the right audience through the right media. )))
All of the following are interactive elements of an IMC strategy except ((( public relations. )))
The proliferation of new media alternatives has led many firms to shift their promotional budgets from ((( advertising to direct marketing and website development. )))
Compared to other IMC alternatives, advertising is extremely effective for ((( creating awareness and generating interest in a product. )))
One difficulty associated with using advertising as part of a marketer’s IMC efforts is ((( it is hard to break through the clutter of other messages targeted for the same audience. )))
Integrated marketing communications include all of the following except ((( supply chain management. )))
Which statement best describes personal selling? ((( It is the two-way flow of communication between a buyer and a seller. )))
Personal selling is an especially important part of IMC in ((( business-to-business settings. )))
Ryan is debating how to allocate the IMC budget for his new ski equipment store. He knows having knowledgeable salespeople in his store can simplify buyers’ purchase decisions. He should also consider that, compared to other IMC alternatives, personal selling is ((( expensive. )))
Sales promotions include all of the following except ((( online ads. )))
An advantage of Internet-based technologies is they ((( can be directed to a specific consumer. )))
In recent years, the component of IMC that has received the greatest increase in aggregate spending is ((( direct marketing. )))
Brenda wants to reduce mass media IMC and to increase the use of personalized marketing communication messages. To achieve this goal, Brenda will likely increase her use of ((( direct marketing. )))
Cora will be a bridesmaid next summer, and she purchased her dress online. The next time she turned on her computer, Cora was surprised to see special offers for matching accessories. This is an example of which marketing strategy? ((( direct marketing )))
The increased use of customer databases has enabled marketers to identify and track consumers over time and across purchase situations, and has contributed to the rapid growth in ((( direct marketing. )))
A major factor contributing to the growth in the use of direct marketing IMC efforts is ((( increased use of credit and debit cards, and online shopping by consumers. )))
When purchasing books on Amazon.com, customers are shown other books and a message saying "Customers who purchased [your book] also purchased . . ." This is an example of ((( direct marketing. )))
Public relations is the component of IMC that ((( supports other promotional efforts by generating "free" media attention. )))
Which of the following is most effective in building brand image, listing details about retail locations, and educating customers about products and services? ((( corporate website )))
A __________ can be used to create positive word of mouth, help customers form a community, and develop long-term relationships between customers and the company. ((( corporate blog )))
Firms can use __________ to bring customers together to share experiences around the products. ((( social media )))
The goals of IMC need to ((( be explicit and measurable. )))
Naomi is the IMC manager for a chain of regional income tax service providers. Franchisees pay a percentage of their revenue to an IMC account allocated to her. As she establishes the short-term goals for her firm’s IMC efforts, her goals are likely to include ((( increasing inquiries, awareness, and trial of her firm’s services. )))
Lamar is assessing the long-term effectiveness of his firm’s IMC efforts. He will probably analyze the firm’s success in ((( increasing market share, sales, and customer loyalty. )))
Julie is developing a budget for her firm’s IMC program. First she sets objectives. Then she chooses media, and finally she determines the cost for each product to be promoted. Julie is using the __________ method of establishing an IMC budget. ((( objective-and-task )))
Using prior sales and communication activities to determine the present communication budget describes which method of IMC budgeting? ((( rule-of-thumb )))
When using the objective-and-task method of IMC budgeting for multiple products and services, how often must the process be repeated? ((( once, for each individual product and service )))
Yolanda asked her firm’s advertising agency to estimate how often consumers saw her firm’s IMC message and what percentage of the target audience was exposed to the message. Yolanda is asking for __________ data. ((( frequency and reach )))
When Yolanda asked her firm’s advertising agency to estimate how often consumers saw her firm’s IMC message and what percentage of the target audience was exposed to the message, she was told the reach was 40, the frequency was 4, and the competitive parity was 10. The gross rating points for her firm’s campaign were ((( 160. )))
To estimate reach in terms of electronic media, marketers can use the click-through rate (CTR). To do so, they need to know the number of clicks and ((( the number of impressions. )))
Which of the following is true of competitive parity? ((( It does not allow firms to exploit the unique opportunities or problems they confront in a market. )))
An ad for Bud Light ran six times during a recently televised football game. When measuring IMC results for this ad, six would be the __________ for this telecast. ((( frequency )))
Sally could recall the brand of toothpaste she used only when a marketing researcher mentioned the brand. This is known as ((( aided recall. )))
If you send an e-mail and include a link, you can track how many people took the desired action of clicking on the link. This is known as ((( the click-through rate. )))
George wants to increase the number of visits to his insurance firm’s website, which specializes in rental insurance for college students. George decides to target Internet users who search for the terms "apartment," "insurance," and "student." Which of the following will be most helpful to George? ((( Google AdWords )))
Which of the following is being used by a store owner who sends out a text message to all of her preferred customers announcing the arrival of this season’s new clothing? ((( mobile marketing )))
The Salvation Army runs a campaign over the Christmas holidays called the Mobile Bell Ringer. Volunteers send text messages to their friends’ cell phones asking them to donate. Which type of marketing communication does this represent? ((( mobile marketing )))
Red Bull sends out student brand managers to distribute free samples to their peers. Which form of marketing communication is this? ((( sales promotion )))
Charming Charlie’s, a fashion accessories retailer, encourages visitors to its stores to "check in" using a smartphone app. In return, customers receive an instant coupon toward that day’s purchase. This is an example of ((( mobile marketing. )))
Which of the following is the least interactive IMC strategy? ((( direct marketing via catalog )))
A firm that places an emphasis on developing and maintaining positive relationships with the media is focusing on a(n) ________ strategy. ((( public relations )))
As firms become more sophisticated in their communication efforts, the trend is toward company blogs becoming more ((( interactive. )))
When Apple’s famous 1984 "Big Brother" ad aired during the Super Bowl, it reached an estimated 500 million viewers. It aired only once on commercial television, but was seen in later years in at least 10 television programs recalling great commercials. For its target audience, the frequency of this ad is measured as ((( 1. )))
A measure termed _______ describes how useful an ad message is to the consumer doing the search. ((( relevance )))
A firm’s marketing communication strategy is formulated specifically to ((( communicate the value of its product(s). )))
In a television commercial for Chobani Greek yogurt, the manufacturer Chobani plays which role in the communication process? ((( sender )))
The manufacturer of Beats by Dr. Dre headphones decides to advertise in the "lifestyles" sections of big-city newspapers. However, this generates a limited response in sales. Which of the following represents a likely breakdown in the communication process here? ((( The message was not received by its intended audience. )))
Lars wants to purchase a gift for a colleague whose home he will be visiting. He decides to bring luxury chocolates, as he knows most people enjoy them. Although he is not sure about specific products, Lars heads directly for a store selling Godiva Chocolates, because he knows this is a high-end brand. Lars can be said to have ((( brand awareness. )))
The individual elements of an IMC strategy can be viewed on two axes: __________ (from the consumer’s perspective) and ((( passive and interactive; offline and online. )))
A ketchup bottle is an example of a ________ package. (( Primary ))
A company’s product mix consists of ((its various product lines.))
What type of competition occurs when there are many firms competing for customers in a given market, but their products are differentiated? ((monopolistic competition))
Private-label brands are developed and marketed by a(n) ((retailer))
Which of the following can help suppliers save on the cost of a product? ((rebranding))
A demand curve shows the relationship between ________ during a specific period of time. ((price and demand))
In ________ many firms provide similar products that are considered substitutes for each other. ((pure competition))
What price competitive level would be indicated when the price is usually set according to the laws of supply and demand?((pure competition))
What is one of the main criteria used for determining how “good” a brand is, or how much equity it has? ((perceived value))
In a market with ________ there are many firms providing differentiated products. ((Monopolistic Competition))
If a firm in a purely competitive market can differentiate its product or service, it becomes part of a(n) ________ market. ((monopolistic competition))
______ shows the relationship between income and demand. ((A demand curve))
What is one of the main criteria used for determining how “good” a brand is, or how much equity it has? ((perceived value))
When clients work with their investment advisers, they ________ their investment portfolios. ((cocreate))
STP refers to ((segmentation, targeting, and positioning))
What type of firms recognize that including a strong social orientation in business is a sound strategy that is in both its own and its customers’ best interest? ((socially responsible firms))
The components of a SWOT analysis are ((strengths, weaknesses, opportunities, and threats.))
Which of the following is one of the four major growth strategies marketers typically utilize? (( market penetration ))
The publisher of the well-known Winnie-the-Pooh series wants to celebrate the 100th anniversary of the series, initiating a $200,000 campaign. This initiative represents the ________ component in the four Ps. ((Promotion))
Which of the following is considered part of the augmented product? ((product warranty))
Dynamic pricing is also referred to as ________ pricing. ((individualized))
________ is the cash expenditure plus taxes that consumers have to pay for a good or service. ((Price))
A useful technique that enables managers to examine the relationships among cost, price, revenue, and profit over different levels of production and sales is called ((break-even analysis.))
The poorest 25 percent of the U.S. population earned ________ or less per year in 2016. (($24,002))
Marketing has traditionally been divided into a set of four interrelated decisions known as the marketing mix, or four Ps, including product, place, price, and (( promotion ))
To promote their greener practices, nearly 60 private-sector companies have partnered with the ________, which seeks to reduce the deforestation caused by the production of many consumer products that rely on palm oil, soy, tree pulp, and paper. ((tropical forest alliance))
Which of the following are commonly found in distribution and fulfillment centers to fill orders for stores or individual customers? (( robots ))
In 2016, in response to the threats of climate change, the United Nations rectified the (( Paris Accord ))
The four elements of the consumer’s immediate environment are the company, competition, corporate partners, and the physical environment. (( True ))
According to your text, the wealthiest 10 percent of the population control approximately what percentage of Americans’ total net worth? (( 76 percent ))
baby boomers are also called millennials (( False ))
________ involves marketing to customers through wireless handheld devices such as cellular telephones. (( Mobile marketing ))
________ refers to a potential customer’s ability to recognize or recall that the brand name is a particular type of retailer or product/ service. (( Brand awareness ))
Integrated marketing communications (IMC) represents the ________ element of the six Ps. (( promotion ))
Integrated marketing communications (IMC) represents the promotion dimension of the six Ps. It encompasses a variety of communication disciplines—advertising, personal selling, sales promotion, public relations, direct marketing, and online marketing including social media. (( TRUE ))
After creating awareness of the firm’s products or services, the next goal of integrated marketing communications is to (( increase the level of interest among consumers. ))
Which of the following are steps in the AIDA model? (( awareness))
When it comes to measuring consumers’ price sensitivity, product prices are viewed as either (( elastic or inelastic. ))
Which of the following are communication channels used in the IMC process? (( internet ))
Marketing communications managers usually state their media objectives in terms of ________, which represents reach multiplied by frequency. (( GRP ))
What is true of wholesalers? (((Wholesalers are extremely important because of the marketing activities they perform.)))
Many service providers are considered retailers because they (((provide their services directly to consumers)))
A merger occurs when (((two companies combine to form a new company)))
A(n) ________ is a partnership established for a specific project or for a limited time. (((joint venture)))
An entrepreneur has been primarily associated with the willingness to((take risks.))
What is one of the difficulties faced by small business owners?((worries about employee problems))
What is a difference between high technology businesses and other small businesses?((High technology businesses require greater capital and have higher initial startup costs thanother small businesses))
What is true of a capitalist economic system?((Prices of goods and services are determined by supply and demand))
The quantity of goods and services that consumers are willing to buy at different prices at a specific time is referred to as((demand.))
The quantity of products that businesses are willing to sell at different prices at a specific time is referred to as((supply.))
What is a defining characteristic of public corporations?((Their stock can be bought, sold, or traded by anyone))
Which business provides a service, but is neither owned by the government nor focuses on earning profits?((a nonprofit corporation))
What is a true statement about the board members of a corporation?((They have a duty of care and loyalty to oversee the management of the firm.))
Preferred stockholders of a corporation((have a claim to profits before other stockholders do.))
The purpose of conscious marketing is to make a product by selling products and services.(((FALSE)))
Marketing ethics is concerned with distinguishing between right and wrong actions and decisions that arise in a business setting, according to broad and well-established moral and ethical principles that might arise in a business setting, and any special duties or obligations that apply to persons engaged in commerce.(((FALSE)))
Marketing ethics can involve societal issues such as the sale of products or services that may damage the environment or global issues such as the use of child labor.(((TRUE)))
The consideration of stakeholders and their interdependence is one of the four overriding principles of conscious marketing.(((TRUE)))
Key corporate social responsibility (CSR) stakeholders include employees, customers, the marketplace, and society.(((TRUE)))
When Walmart pressured its vendors to supply it with environmentally friendly merchandise with labels to prove it, this most relates to the concept of sustainability.(((TRUE)))
One way in which conscious marketing differs from corporate social responsibility (CSR) is that CSR takes a holistic, ecosystem view of business as a complex, adaptive system.(((FALSE)))
From a conscious marketing perspective, social responsibility is at the core of the business through the higher purpose and viewing the community and the environment as stakeholders.(((TRUE)))
Coca-Cola spent $102 million through The Coca-Cola Campaign focusing on water stewardship, healthy and active lifestyles, community recycling, and education. This is an illustration of a CSR program.(((TRUE)))
The first step in ethical decision making is to gather information and identify stakeholders.(((FALSE)))
The Golden Rule test asks the question, "Would I like to be on the receiving end of this action and all its potential consequences?"(((TRUE)))
A roofing company agreed to complete a job in one week and collected a 50 percent deposit, but never showed up to do the job. The same roofing company then donated $6,000 to a local children's hospital. The roofing company could be considered socially responsible.(((TRUE)))
BlendMate, a firm that manufactures high-end blenders, donates $10 per blender sold to a local food bank. This is a form of corporate social responsibility.(((TRUE)))
Ethos Water donates 2 percent of its profits to children in need of clean water. This action demonstrates that Ethos Water is a firm with a strong ethical climate.(((FALSE)))
Corporate social responsibility refers to the coordinated actions of government organizations to address the ethical, social, and environmental impacts of business operations.(((FALSE)))
Among the key differences between conscious marketing and CSR is the unique view on shareholders that is absolutely critical to CSR.(((FALSE)))
Stakeholders typically include the firm's employees and their families, customer groups, members of the community, the environment, and the firm's partners and competitors.(((TRUE)))
Brainstorming in the ethical decision-making framework occurs immediately following the identification of issues.(((FALSE)))
Badger Hardware was planning on raising the pay of its managers, but not its frontline employees. To determine the potential ethical issues, it should first identify the issues involved so that it can gather facts related to those issues.(((TRUE)))
A common view in today's business climate is that the only responsibility of a business is to its shareholders, so its only purpose is to make a profit.(((FALSE)))
Tipco Computer Company decided to market its tablet computers to preschoolers, even though the tablets were better suited for much older children. This potentially unethical activity takes place during the control phase of the strategic marketing planning process.(((FALSE)))
Once the marketing strategy is implemented, controls must be in place to be certain that the firm has actually done what it has set out to do.(((TRUE)))
An example in the text describes a campaign by Grey Poupon that involved a Facebook campaign targeted toward people who had "good taste." The campaign was criticized for privacy issues that made it unethical. This occurred during the implementation phase of the strategic marketing planning process.(((TRUE)))
The most basic corporate social responsibility to employees is to ensure the highest pay for the work performed.(((FALSE)))
Pepsi has cooperated with America on the Move to improve many of its products and their labels, such as reducing the saturated fat in its Frito-Lay Ruffles. This form of social responsibility most directly impacts shareholders.(((FALSE)))
Business ethics and marketing ethics are synonymous terms.(((FALSE)))
Conscious marketing encompasses all of the following overriding principles except(((recognition of the company's bottom line. )))
Business ethics is concerned with all of the following except(((societal issues such as the sale of products or services that may damage the environment )))
The sale of products that may damage the environment, the use of sweatshop labor, and the marketing of dangerous products are examples of(((marketing ethical issues. )))
When Walmart issued new standards for livestock products that were raised on food without antibiotics or artificial growth hormones, it considered multiple ______, including the ranchers that supply the food, its customers, and animal welfare groups.(((stakeholders )))
All of the following are included in the decision-making metric except(((the likability test. )))
____________ provides a detailed, multipronged “Statement of Ethics” that can serve as a foundation for marketers.(((The American Marketing Association)))
In the ethical decision-making metric, the question that asks "Would I want the person I admire most to see me doing this?" applies to the(((admired observer test. )))
The _________ for natural skin care company Burt’s Bees is to “create natural, Earth-friendly personal care products formulated to help you maximize your well-being and that of the world around you.”(((mission statement. )))
In the ___________ stage, a firm will decide what level of commitment to its ethical policies and standards it is willing to declare publicly and how the firm plans to balance the needs of its various stakeholders.(((planning )))
Which of the following statements regarding corporate social responsibility is true?(((It sees limited overlap between the business and society, and between business and the planet.)))
Which of the following statements regarding conscious marketing is correct?(((It recognizes that business is a subset of society, and that society is a subset of the planet. )))
When marketers work in controversial or polluting industries such as tobacco or fossil fuels, their central activities largely bar them from becoming(((conscious marketers. )))
Which of the following descriptions embodies conscious marketing principles?(((both ethically and socially responsible)))
New real estate disclosure regulations require sellers and their agents to tell prospective buyers about any existing problems with the property. Previously, they were expected only to answer buyers' questions. The new regulation addressed the marketing ethics problem of(((withholding information.)))
When making decisions, managers often have to decide between doing what is beneficial for them (and possibly the firm) in the short run, and doing what is right and beneficial for the firm and for society in the long run. To address this conflict, a firm(((must align the short-term goals of each employee with the long-term, overriding goals of the firm. )))
To avoid having ethical situations become problematic for a firm, the short-term goals of each employee must(((be aligned with the long-term goals of the firm. )))
Compared to the average company, firms with strong ethical climates tend to(((be more socially responsible. )))
Courses of action such as halting the market research project, making responses anonymous, and instituting training on the AMA Code of Ethics for all researchers would be identified during the __________ stage.(((brainstorming )))
The XYZ firm is in Step 4 of its ethical decision-making process. Executives were asked to take the Publicity test using an ethical-decision making metric. All scores were in the "No" column. What does this mean?(((The situation is ethically troubling to the executives.)))
Ironically, while the leaders of Enron Corporation were manipulating the company's finances for their personal benefit, the company was a major donor to Houston area charities. Enron had unethical business practices, but was also(((demonstrating corporate social responsibility. )))
The ethical decision-making framework includes all of the following steps except(((promote the firm's corporate social responsibility efforts. )))
Garrett has just purchased a beer distributorship. He wants to increase the visibility of his firm in local markets, but he knows there are a number of regulations and socially accepted practices associated with promoting alcoholic beverages. The first thing Garrett should do is to(((identify issues that need to be addressed. )))
Hisaoki picks up the local newspaper and reads a stinging letter to the editor criticizing his beverage company for supporting a sporting event for children with disabilities. The letter writer is critical of a banner displayed at the event with the logos of alcoholic beverages and Hisaoki's company name. Hisaoki never considered that this problem might arise. Hisaoki's company failed to(((identify issues. )))
Many corporations are shifting from defined benefit to defined contribution retirement programs. When considering changes to retirement programs, the primary stakeholders are the(((employees. )))
When Bernie Ebbers, WorldCom's CEO, was convicted of financial crimes, WorldCom was forced to merge with MCI. One of the ramifications of this merger was the loss of WorldCom's sponsorship of the Sea Pines Heritage PGA golf tournament. The tournament funds the Heritage Foundation, a major community charity. This example illustrates(((that the impact of unethical actions can reach far beyond the corporation. )))
All of the following statements regarding corporate social responsibility are true except(((a firm's responsibilities to society are not associated with the demands, expectations, requirements, and desires of various stakeholders. )))
The Harvest County School Board is concerned about deteriorating school facilities, combined with a shrinking budget. The board began by studying the issue, and then identified parents, children, teachers, staff, and taxpayers as groups who have a vested interest in the problem. The school board has listened to each group's concerns. In the ethical decision-making framework, its next action should be to(((engage in brainstorming alternatives. )))
After a firm has identified the various stakeholders and their issues and gathered available data related to an ethical decision-making situation, __________ should engage in brainstorming and evaluating alternatives.(((all parties relevant to the decision )))
After a firm has identified the various stakeholders and their issues and gathered available data related to an ethical decision-making situation, all parties relevant to the decision should(((brainstorm for alternatives.)))
After a firm has identified the various stakeholders and their issues and gathered the available data, all parties relevant to the decision should engage in brainstorming and evaluating alternatives. __________ then review and refine these alternatives, and choose a course of action.(((Company leaders and managers )))
Darwin's company is facing a difficult ethical issue. The firm has identified the various stakeholders and their issues and gathered the available data. Everyone with an interest in the issue has engaged in brainstorming and evaluating alternatives. Management reviewed and refined the alternatives. It should now choose the course of action that(((seems best after weighing the concerns of all stakeholders.)))
Rock-Bend Company is considering buying out a competing firm and closing most of the competitor's factories. The firm has identified the various stakeholders and their issues and gathered the available data. Everyone with an interest in the issue has engaged in brainstorming and evaluating alternatives. Management reviewed and refined the alternatives, and then chose a course of action. If the managers are not confident about the decision, they should(((reexamine their alternatives.)))
Imagine the use (or misuse) of data collected from consumers by a marketing research firm. One of the issues that might arise is the way the data are collected. At what step in the framework for ethical decision making would this issue be identified by tthe marketing research firm?(((Step 1 )))
Denny is considering the question, "Did our actions have a negative impact on any stakeholder group?" Denny is addressing marketing ethical issues in the __________ phase of the strategic marketing planning process.(((control )))
Every year, General Mills issues a report discussing how the firm has performed against its own standards of conscious marketing. This report is part of General Mills's __________ phase of its strategic marketing planning process.(((control )))
Charges that firms are using "sweatshop" labor to produce their products are likely to occur during the __________ phase of the strategic marketing planning process.(((implementation )))
Marketers that include ethical policies and standards in the firm's mission or vision statements are introducing these concepts at which stage of the strategic marketing planning process?(((planning phase )))
During the __________ phase of the strategic marketing planning process, marketers utilize systems to check whether each conscious marketing issue raised in earlier phases was addressed.(((control)))
Alicia has been asked to approve a marketing campaign that, although it is not illegal, promotes food products to children. She is concerned that the food products are not particularly nutritious, although they are not as bad for children as some others sold by competitors. By using the ethical decision-making metric, Alicia will(((evaluate the alternative using a series of questions.)))
CSR can be described as context-specific actions and policies, taking stakeholders’ expectations into account, to achieve what is referred to as the triple bottom line. The triple bottom line includes ________ performance.(((economic, social, and environmental )))
When Toyota owners began to report problems with sticking accelerator pedals and nonfunctioning brakes, Toyota at first ignored or rejected the claims. Eventually, the company evaluated the issue and issued a recall. Which of the four steps of the ethical decision-making framework was Toyota performing when it recalled several of its 2007-2010 models?(((choose a course of action)))
For every consumer who purchases a pair of TOMS shoes for $55, the company promises that a needy child will receive a pair of shoes. TOMS shoes is actively engaging in(((corporate social responsibility. )))
If Melissa decides to sell the best ice cream on earth, and intends to establish a strong ethical climate in her organization, during which phase of the strategic marketing planning process should she introduce ethical considerations?(((planning )))
Anupam's company manufactures industrial ladders. He is concerned that consumers who do not understand ladder safety will purchase these extra-tall ladders and injure themselves. During which phase of the strategic marketing planning process should this issue be addressed?(((implementation )))
A meat-packing company discovers that six months ago it unknowingly distributed meat from a cloned cow. The firm is unaware of any specific risks to humans consuming the meat; however, some scientists have raised questions, and some consumers are afraid of possible future problems. The meat company has to decide whether or not to make this matter public. How should it begin the process of making an ethical decision?(((Identify the issues raised by the situation. )))
Elena is the CEO of a small manufacturing firm. She is concerned with meeting the investment objectives of the firm's shareholders, and sees no value in corporate social responsibility. Elena's attitude is(((consistent with the views of other critics of corporate social responsibility. )))
How might a technology company like Appleensure that it behaves in a socially responsible way toward its employees?(((It can ensure that pay practices are fair at all levels of the company. )))
How might a technology company like Appleensure that it behaves in a socially responsible way toward its customers?(((It can protect the privacy of personal information collected on its website. )))
How might Starbucks ensure that it behaves in a socially responsible way toward members of its supply chain?(((It can purchase coffee beans from suppliers who pay coffee growers a fair price. )))
All of the following terms are generally associated with the definition of corporate social responsibility except(((profit.)))
When companies embrace __________, they appeal not only to their shareholders but also to all of their key stakeholders.(((conscious marketing )))
For corporate leaders, their firm’s ability to ___________ must be of paramount importance.(((balance shareholder interests with the needs of society. )))
All of the following are questions posed in the ethical decision-making metric except(((Will this action help advance my career? )))
When integrating conscious marketing into a marketing strategy, a key task is to ensure that all managers are evaluated on their actions from a conscious marketing perspective.This action would take place during which stage of the strategic marketing planning process?(((control phase)))
Generally, people buy one product or service instead of another because they perceive it to be a better value.(((TRUE)))
The consumer decision process begins with a comparison of available alternatives.(((FALSE)))
The greater the difference between a consumer’s unsatisfied need and the desired state, the greater the need recognition will be. (((TRUE)))
Brenda wants a new car that will be dependable transportation and look good. She wants to satisfy both functional and psychological needs.(((TRUE)))
5. When Glen is thirsty, he always buys a Coke. Like many consumers, Glen engages in considerable alternative evaluation when buying habitual products like his Coke.(((FALSE)))
6. One benefit of having satisfied customers is that they may spread positive word of mouth.(((TRUE)))
Setting high customer expectations is a good strategy that will help avoid customer dissatisfaction in the long run.(((FALSE)))
Customers are more likely to talk about service that exceeded their expectation than about service that did not meet their expectation.(((FALSE)))
Maslow's hierarchy of needs is an interesting concept for psychology, but it has little relevance for marketing.(((FALSE)))
Situational factors sometimes override psychological and social factors in the consumer decision process.(((TRUE)))
Frazier is out of milk and bread and needs to decide what is for dinner. He will be stopping at the grocery store on the way home. Frazier will likely engage in limited problem solving.(((TRUE)))
Consumers involved in habitual decision making engage in little conscious decision making.(((TRUE)))
As manager of a local donut shop, Arnie greets his regular customers by name and often begins making their order when he sees them drive into the parking lot. Arnie knows habitual purchasers with strong store loyalty are great customers.(((TRUE)))
Rachael is visiting colleges before applying to schools. Rachael is likely to be involved in an impulse buying process.(((FALSE)))
When making an important purchase, consumers often consult friends and family. This is considered an external search for information.(((TRUE)))
Mary will not consider purchasing an "American" car brand based on negative comments made by her parents; therefore, she has developed a negative attitude toward American brand cars.(((TRUE)))
A reference group may have direct or indirect influence on your attitude toward a particular clothing store.(((TRUE)))
After purchasing an expensive pair of shoes, you may question whether or not the shoes are any better than the less expensive shoes you could have purchased instead. This is an example of postpurchase cognitive dissonance.(((TRUE)))
Pam didn't go see the movie Gravity because her friends all said she wouldn't be able to handle it. When she found out how good it was, she blamed her friends. Pam is demonstrating an internal locus of control.(((FALSE)))
Another name for physiological risk is safety risk.(((TRUE)))
Jason usually buys Nike shoes, so when his friend asked him what shoes he should buy, he said Nike without thinking about it. Jason's response was an evoked set.(((FALSE)))
When Hakim asked Marta where she wanted to go for lunch, she said Chili's because she went there at least once a week and always liked it. Marta conducted an internal search for information.(((TRUE)))
Vladimir decided not to purchase the iPod shuffle because he didn't think it would hold all of his songs. Vladimir was using his assessment of the iPod shuffle's financial risk in making his decision.(((FALSE)))
Determinant attributes are product or service features that are important to the buyer and on which competing brands or stores are perceived to differ.(((TRUE)))
According to Maslow's hierarchy of needs, safety needs are the needs that people first seek to meet.(((FALSE)))
Marketers often use principles and theories from sociology and psychology to better understand consumers' actions and to(((develop basic strategies for dealing with their behavior. )))
Generally, people buy one product or service instead of another because they(((perceive it to be the better value for them. )))
The consumer decision process model represents(((the steps that consumers go through before, during, and after making purchases. )))
The consumer buying process begins when(((a consumer recognizes an unsatisfied need.)))
When Karen realized her dog had fleas, Karen was faced with(((an unsatisfied need. )))
The greater the discrepancy between a consumer's needy state and the desired state, the greater(((the consumer's need recognition will be. )))
The greater the discrepancy between a consumer's __________, the greater the consumer's need recognition will be.(((needy state and desired state )))
Upscale men's and women's clothing stores like Nordstrom, Neiman Marcus, or Saks Fifth Avenue are more likely to appeal to consumers' __________ needs.(((psychological )))
When mountain climbers purchase clothing for scaling Mount Everest, their purchases are primarily addressing __________ needs.(((functional )))
Laura has an almost-new economy car, but she wants a Ford Mustang because she thinks it would be exciting to own one. If she decides to purchase a sports car such as the Mustang, she will be primarily fulfilling a __________ need.(((psychological )))
A key to successful marketing is determining how to meet the correct balance of __________ needs that best appeals to the firm's target markets.(((functional and psychological)))
By producing motorcycles that do more than get riders to their destinations and back, Harley-Davidson is addressing consumers' __________ needs.(((functional and psychological)))
When the floor rusted through on her old car, Kelly knew she had a problem. Logically, Kelly's next step in the consumer decision process would be to(((search for information about cars. )))
When Kelly began searching for a new car to replace her old, rusty one, she probably relied on __________ sources of information.(((internal and external )))
When Brandon decided he needed a new car, he immediately called his old college roommate, who owns a BMW dealership, to ask questions about options and financing. Brandon was searching for information from(((an external source. )))
Peter wanted an unbiased source of information to help him decide what brand of appliances to buy for his new condominium. Peter would most likely search for information from(((the Consumer Reports website. )))
Once consumers have recognized a need, they begin to search for ways to satisfy that need. The internal search is characterized by(((examining personal memories and knowledge. )))
Every time Katie wants to eat salad for lunch, she and her friends go to Sweet Tomatoes, but if she’s craving dessert, she heads straight to The Cheesecake Factory. In making these choices, she relies on a(n)(((internal search for information. )))
Ryan believes he is responsible for his actions, and he will conduct extensive searches before making a purchase. Michael's favorite phrase, when confronted by the need to make a decision, is "Whatever." In marketing terms, Ryan is said to have a(n) __________ and Michael, a(n) __________.(((internal locus of control; external locus of control )))
In the consumer decision process, we decide how much time and effort to expend searching for information based partly on(((the degree of perceived risk associated with the product or service being considered. )))
Kathy has naturally curly hair and has often been disappointed with the haircuts she has received. When she moved to a new town, she approached her new office mates and several strangers with curly hair and asked them where they had their hair cut. She chose to spend considerable effort finding a new hair stylist based on the __________ associated with her purchase decision.(((performance risk )))
Before flying, Jaden researches the types of planes the airline uses, scans the plane for defects as it taxis up to the terminal, and follows the pilot and crew as they come through the airport. He has no experience as a pilot or airplane mechanic. Jaden probably has a misguided sense of his(((locus of control.)))
Every year before he puts his boat in the water, James has his mechanic put a new battery in the boat. James is most likely concerned with __________ risk.(((financial)))
Jordana is a travel agent. Whenever she sells an expensive vacation package, she encourages the customer to buy travel insurance, which provides reimbursement in case of trip cancellation due to illness or another emergency. Jordana is trying to reduce her customers' __________ risk.(((financial )))
André was afraid his new condominium would look shabby to his future in-laws, so he had it painted just before their visit. André was addressing his __________ risk.(((social)))
Consumers consider universal, retrieval, and evoked sets during which stage of the consumer decision process?(((evaluation of alternatives)))
When Magda decided to buy a new computer, she considered all the brands she could recall seeing advertised. This represents Magda's __________ set.(((retrieval )))
When Maya decided to buy a new computer, she thought about all the brands she could recall seeing advertised, but she would consider only those brands she could buy at her local Best Buy electronics store. This represents Maya's __________ set.(((evoked )))
Marketers particularly want their brands and products to be in consumers' __________ sets.(((evoked )))
Elena is in the process of buying a new car. There are many possible cars to choose from, but she is focused on a few she would actually consider buying. These make up her __________ set.(((evoked)))
The Wall Street Journal provides a set of guidelines each year for purchasing a laptop computer. The guidelines include recommendations for hard disk capacity, memory size, battery life, and several other attributes. The Wall Street Journal is providing consumers with(((evaluative criteria. )))
__________ attributes are product or service features that are important to buyers and that are used to differentiate among choices.(((Determinant )))
Andrea and Karl got married a year ago and are ready to move out of their apartment and into a new home. After looking at several houses, they have developed a list of features that are important to them and that are different among the homes they have visited. The features on their list are called(((determinant attributes. )))
Jonathan prefers shirts made with 100 percent cotton, but he will sometimes buy shirts with less cotton if they are less expensive. Jonathan uses __________ to decide which shirts to buy.(((a compensatory decision rule )))
Dawn flies regularly between Atlanta and Los Angeles. She almost always uses Delta Airlines and has lots of Delta Sky Miles credit (Delta's frequent-flyer program). Still, she uses an online fare comparison website each time to see if a competitor has a better price or a more convenient schedule. Dawn uses __________ to decide which airline to fly.(((a compensatory decision rule)))
Jordana is buying a laptop computer to take on trips. Although she has looked at several brands, she refuses to buy a computer that weighs more than five pounds. Jordana is basing her decision on(((a noncompensatory decision rule. )))
Marketers are particularly interested in postpurchase behavior because it(((involves actual rather than potential customers. )))
Sometimes consumers have second thoughts after buying goods that are expensive, infrequently purchased, or associated with a high level of risk. This is an especially critical time for marketers, as their customers are dealing with(((buyer's remorse. )))
Many teenagers, both male and female, have clothes they purchased in the past that they "would not be caught dead in" today. When they occasionally see those clothes hanging in the back of their closet, these teenagers probably feel(((postpurchase cognitive dissonance. )))
Postpurchase cognitive dissonance is especially likely for products that are(((expensive, infrequently purchased, or associated with high levels of risk. )))
Marketers frequently design customer relationship management programs to(((retain loyal customers. )))
Marketers fear negative word of mouth because when consumers are dissatisfied, they(((often want to complain to many people.)))
Most firms maintain customer complaint services online, in the store, or over the telephone. Firms attempt to respond quickly to complaints, hoping to(((minimize negative word of mouth and rumors. )))
Maslow’s hierarchy of needs includes physiological needs at the lowest level and self-actualization at the top. The three levels in between are (((safety, love, and esteem. )))
After Sharon graduated from college, she found a steady and good-paying job, got married and began to raise a family, and began to receive recognition at work and in the community. Eventually, she began to devote more time and effort to intellectual and aesthetic pursuits. Sharon is moving up to what level on Maslow's Hierarchy of Needs?(((self-actualization )))
A(n) __________ is a need or want strong enough to cause a person to seek satisfaction.(((motive )))
There are approximately 1 billion people living in India. Only about 200 million of these people earn more than the equivalent of $1,000 per year. According to Maslow's hierarchy, most of the other 800 million Indian consumers are primarily addressing their __________ needs.(((physiological )))
Negative attitudes are typically difficult for marketers to change because(((attitudes are learned and long lasting. )))
Though he has never owned a Jaguar, Jerry thinks they are poorly made and have many mechanical problems. For Jaguar to sell Jerry a car, the company would need to change the __________ component of Jerry's attitude.(((cognitive)))
Apple computer users tend to like the company and love its products. Apple has nurtured this __________ component of its customers' attitudes.(((affective)))
__________ refer(s) to the process by which consumers select, organize, and interpret information to form a meaningful picture of the world.(((Perception )))
American visitors to the Indonesian island of Bali are often aghast when they see the sign for the Swastika resort. Americans associate the swastika symbol with Nazi Germany while Indonesians associate the symbol with the four major elements on Earth. The two groups have different(((perceptions.)))
Learning refers to a change in a person’s thought process or behavior that arises from (((experience. )))
Stuart wanted to impress Janet with the perfect engagement ring. He had been saving money for months, and he noticed his attitudes and perceptions about diamond rings changing as he began paying attention to ads for rings. Marketers call this process(((learning. )))
Among the factors affecting the consumer decision-making process is the way consumers spend their time and money to live, otherwise known as(((lifestyle. )))
Christopher bought Timberland boots because he felt they were perfect for his outdoor activities. Patrick bought the same kind of boots because he felt they were stylish, especially with the logo clearly visible. The psychological factor driving Patrick's behavior is(((lifestyle. )))
A consumer’s external social environment includes (((the customer's family, reference groups, and culture. )))
Jennifer's spending decisions are heavily influenced by her family, her peers, and her religious education. These influences on her spending decisions are all ________ influences.(((external social environment)))
Natalie and her fiancé Dow are planning their wedding. She knows her mother wants her to have a traditional church wedding with a Roman Catholic priest officiating. Natalie would like to have an informal ceremony on the beach, since that type of wedding has become popular with her friends. Furthermore, Dow is from Thailand and would like to have a monk officiate. Natalie and Dow's wedding decisions are most influenced by(((family, reference groups, and culture. )))
Many Hollywood movie stars were among the first to buy electric and hybrid vehicles. These stars often become ________, influencing other consumers’ behavior. (((reference groups)))
The shared meanings, beliefs, morals, values, and customs of a group of people constitute its(((culture. )))
Beverage firms sometimes hire attractive young people to sit at fashionable bars, sipping the company's latest product offering. The firms hope these "models" will serve as a(n) __________ and influence consumers.(((reference group )))
Before going on his first business trip to China, Brad asked his Chinese American friend to advise him on customs and values common among the Chinese businesspeople he will likely encounter. Brad is trying to avoid __________ business blunders.(((cultural)))
Where Caroline grew up, everyone knew everyone else, no one locked their doors, and a person's word could be trusted. When she went to work in another part of the country, she was surprised by how few people had similar values and beliefs. Caroline had to adjust to __________ differences.(((cultural )))
The traditional marketing strategy of selling umbrellas when it is raining is an example of how __________ factors influence consumers' decisions.(((situational)))
__________ factors override or at least influence psychological and social issues.(((Situational )))
Paula is about to open a new hardware store. She is making decisions regarding lighting, colors, and layout of merchandise. Paula knows these __________ factors will influence consumers' purchase decisions.(((store atmosphere)))
Steve wasn't sure what kind of salsa he wanted to get for his upcoming Super Bowl party. It seemed like there were dozens of varieties to choose from. He noticed that he could sample a few at a station in the store. He tried four, rejected two, and bought several jars of the two he really liked. He also ended up buying a different—and more expensive—kind of tortilla chip after tasting it. Marketers identify this as a success story of(((in-store demonstrations. )))
Brenda was planning a small dinner party, and had gone to a new specialty food store with coupons she'd found in the food section of the paper. At the store she also found a "buy one, get one free" deal, and a gift offered with the purchase of a particular dessert. She altered the menu as a result of the __________ and ended up spending less than she'd planned.(((in-store promotion)))
Many states have laws regulating the prices businesses charge during emergencies like hurricanes. These laws are designed to protect consumers whose __________ state may impair their ability to make sound purchase decisions.(((temporal )))
There is a saying "Never go to the grocery store hungry." This saying suggests that a consumer's __________ state may adversely affect purchasing decisions.(((temporal)))
The consumer's level of involvement can lead to two types of buying decisions: __________ or(((extended problem solving; limited problem solving. )))
Consumers use and process different aspects of advertising or messages. __________ yields greater attention and deeper processing, and leads to strong attitudes and purchase intentions.(((High involvement)))
Thanh has to decide which college to attend. This is the most important, riskiest, and most expensive decision she has ever made. She will be engaged in(((extended problem solving. )))
Limited problem solving usually relies on(((past experience more than on external information. )))
Last semester, Henri bought his textbooks over the Internet and saved a considerable amount of money. Classes start in a few days, and he needs to decide right away how and where to purchase his books. Henri will most likely engage in a(n) __________ process.(((limited problem-solving)))
Most big box retailers regularly move products from one aisle to another. They also put personal care products in the pharmacy area, many aisles away from the grocery products. They do this because consumers who spend more time walking through the store are likely to buy things they hadn't planned to buy, or make(((impulse purchases. )))
Marketers love consumers who engage in __________, buying their company's product with little thought or consideration of alternatives.(((habitual decision making )))
To attract and maintain habitual purchasers, marketers spend considerable effort(((creating strong brands and store loyalty. )))
For which of the following purchases would consumers most likely engage in limited problem solving?(((college courses )))
The three components of an attitude are(((cognitive, affective, and behavioral. )))
Alex decides to make a donation to the Autism Research Institute. Which component of Alex's attitude toward autism research does this represent?(((behavioral component)))
An online retailer needs to be able to measure how well its website converts purchase intentions into actual purchases. This is known as the(((conversion rate. )))
Zappos.com constantly reminds customers of recently viewed items and informs them when stock is low in an effort to entice the customer to make a purchase. Zappos is trying to improve its(((conversion rate. )))
Tomas, a bank employee, doesn't feel that his coworkers accept him. He decides to dress more casually, as they do, hoping to be accepted. Which level of Maslow's hierarchy of needs is Tomas trying to work on?(((love)))
There are five types of risks associated with purchase decisions. Which of the following best describes a situation where your new car stalls in the middle of a busy intersection?(((physiological risk)))
Barak is considering buying a hybrid car, but he's not sure that he believes the gas mileage estimates. Barak is concerned about(((performance risk. )))
A home security company will advertise the need for home surveillance products to appeal to which level of Maslow's Hierarchy of Needs?(((safety)))
Joanna lives according to her own rules, unconcerned about designer labels, brand names, and luxury items. Joanna is at which level in Maslow's hierarchy of needs?(((self-actualization)))
Some websites allow consumers to shop while getting opinions from online friends. Which of the influences on the consumer buying process does this represent?(((social)))
__________ are the three types of attribute sets.(((Universal, retrieval, and evoked)))
While on vacation, Martha had her camera stolen. Not wanting to waste vacation time shopping for a new camera, Martha simply purchased another camera just like her old one. For Martha, the __________ was low.(((perceived benefit versus perceived cost of search)))
Reginald greets his regular customers by name every morning when they come in for coffee. He offers them a taste of anything special he is cooking that day. He has a database with their birthdays, and offers them free meals on their birthdays. Reginald hopes that this attention to his "regulars" will encourage them to(((spread positive word of mouth. )))
"Black Friday," as the day after Thanksgiving has come to be known in the United States, is a day marked by many special deals in most retail stores, including deep-discounted products available in limited quantities, called "doorbusters," and designed to get shoppers into the store. But one danger of running out of the doorbuster deals is that a shopper may become angry or discouraged at failing to get the special item, and decide not to do any more shopping. This is an example of the impact of __________ on the consumer decision process.(((setting high expectations)))
Americans often equate "bigger" with "better," and prefer larger cars, TV screens, homes, even meals. Researchers suspect that in doing so, we are trying to reduce __________ risk in the consumer decision process.(((psychological )))
Zappos online shoe and clothing store has a unique way of dealing with abandoned shopping carts. If a site visitor places items into the shopping cart and then leaves the site without making a purchase, several days later Zappos sends a humorous e-mail saying, "Let us show you what your shopping cart did while you were gone," along with a photo of a cute dog intended to represent the shopping cart. This attention-getting device is designed to improve the site's(((conversion rate. )))
Firms would prefer to manufacture in a country that has a trade surplus, or a higher level of exports than imports.(((TRUE)))
The greater the wealth of a country, generally, the better the opportunity a firm will have in that particular country.(((TRUE)))
The Big Mac Index is a measure of economic health in a country.(((FALSE)))
Colin wants information about the infrastructure in the countries his company is planning to export to, so he should gather information about the transportation and communications capabilities in each country.(((TRUE)))
Tariffs artificially lower prices and therefore lower demand.(((FALSE)))
Among the various international trade agreements, the North American Free Trade Agreement represents the highest level of integration across individual nations.(((FALSE)))
The BRIC countries are Bolivia, Russia, Italy, and China.(((FALSE)))
Even as growth in its gross domestic product has slowed, China maintains a thriving retail market, likely to reach the $8 trillion mark soon and surpass the United States as the world’s largest.(((TRUE)))
India is one of the fastest-growing markets and has one of the youngest populations in the world.(((TRUE)))
Brazil is characterized by strong upper and lower classes, but the middle class has declined in recent years.(((FALSE)))
Russia lags behind most European countries in use of the Internet.(((FALSE)))
Global expansion often begins when a firm receives an order for its product from another country.(((TRUE)))
Exporting refers to a situation where a company maintains ownership of its plants, operational facilities, and offices in a foreign country in which it sells its products.(((FALSE)))
In a joint venture, the burden of ownership, control, and profits is not shared.(((FALSE)))
Entering into a global franchise agreement exposes a company to higher risk than if the company had entered into direct investment in the country.(((FALSE)))
There is only one global product strategy: to sell a product or service similar to that sold in the home country, but include minor adaptations.(((FALSE)))
An effective global pricing strategy can be easily established by setting the same prices in every market around the world.(((FALSE)))
Glocalization refers to a global marketing strategy in which each of the four Ps is customized for each country. (((FALSE)))
To maximize potential, segments and target markets can and should be defined by more than just geography.(((TRUE)))
In parts of Europe, including Belgium, Italy, Spain, Greece, and France, sales are allowed only twice a year, in January and June or July.(((TRUE)))
Firms with global appeal can run global advertising campaigns and simply translate the wording in the advertisements and product labeling.(((TRUE)))
Global distribution networks that involve middlemen, exporters, importers, and different transportation systems generally lower costs and prices for products.(((FALSE)))
Sophia owns a string of boutiques in Italy. As such she is allowed to have sales only twice a year, in January and June or July.(((TRUE)))
A consumer products company produces inexpensive goods in underdeveloped markets, then repackages them as cost-effective innovations for Western buyers. This is an example of glocalization.(((FALSE)))
Direct investment offers the firm complete control over its operations in the foreign country.(((TRUE)))
Globalization refers to the processes by which goods, services, capital, people, information, and ideas (((flow across national borders.)))
The components of global market assessment include all of the following except(((ethnic analysis.)))
Chris is gathering information about the general economic environment in Nepal. In doing so, he will look for information about the general economic environment, market size and population growth rate, and(((real income.)))
To determine the market potential for its particular product or service, a firm should use(((as many metrics as it can obtain.)))
Manufacturers would prefer to produce in a country with a trade __________, because it signals a greater opportunity to export products to more markets.(((surplus)))
The most common measure of market potential of an economy is a country's(((GDP.)))
GDP is defined as(((the market value of goods and services produced in a country in a year.)))
Gross national income consists of GDP(((plus the net income earned from investments abroad (minus any payments made to nonresidents who contribute to the domestic economy).)))
The Big Mac Index is a novel measure of(((purchasing power parity.)))
According to purchasing power parity theory, if __________ is(are) in equilibrium, products will cost the same in each country.(((exchange rates)))
Economic measures like GDP and GNI do not fully account for a country's economic health because they measure only(((material output.)))
Today, many developed countries are experiencing __________ population growth.(((zero or negative)))
Cory is working on a global marketing assessment team looking out well into the future to help determine the most attractive market areas around the world. He is evaluating market sizes and growth rates. Based on population growth rates in different regions, he should consider that(((countries with high purchasing power today may not continue to show the same growth in the future.)))
The shift of population from rural to urban areas in countries such as India helps global marketers by(((simplifying the supply chain needed to make goods and services available.)))
When considering global marketing opportunities in Bangladesh, Tom asked the question, "How will we get it there?" Tom is concerned about __________ capabilities in Bangladesh.(((infrastructure)))
As part of efforts to stimulate economic development in Africa, the Gates Foundation announced that it would provide cellular phones to farmer cooperatives. The Gates Foundation recognized that problems in __________ exist in many African markets.(((communication)))
When Ben evaluated the commercial infrastructure in Mauritius, he considered the island's(((legal, banking, and regulatory systems.)))
Changes in tariffs and quotas are(((government actions that reduce competition from international firms.)))
Tariffs protect domestic producers by(((making imported products more expensive.)))
In most cases, countries use tariffs to reduce foreign competition, but tariffs are also used(((as a response to perceived unfair trade practices.)))
A __________ limits the quantity of imported merchandise, thus minimizing competition faced by domestic products.(((quota)))
When the U.S. government determined the prices of solar panels imported from China were artificially low due to illegal subsidies, it imposed a ______ to help domestic firms compete.(((tariff)))
When the value of the dollar declines in relation to other currencies, it benefits U.S. marketers who(((export goods to other countries.)))
All of the following are major trade agreements affecting global marketing except(((GNI.)))
Marketers considering operations and trade with a specific country must consider whether or not the country belongs to a trading bloc. A trading bloc is a group of countries(((that have established a formal agreement to manage trade activities.)))
Which of these trade agreements represents the highest level of integration among participating nations?(((EU)))
Global businesses often find it particularly difficult to understand the __________ of a country's culture.(((underlying values)))
Chris laughed at some of the cultural mistakes companies made in advertising and promotion in international trade while he was in school. Now he was trying to determine what had gone wrong with the campaign he had planned in Latin America for his company's product, and it didn't seem quite as amusing. He narrowed the issues to sociocultural factors. He was looking at both __________ and __________.(((visible artifacts; underlying values)))
Geert Hofstede's cultural dimensions concept focuses on five dimensions of __________ in a country.(((underlying values)))
All of the following are included in Hofstede's cultural dimensions except(((certainty assurance.)))
Marketers sometimes use Hofstede's cultural dimensions to design marketing campaigns(((consistent with underlying cultural values in a country.)))
Culture affects(((every aspect of consumers' purchase decisions.)))
Generally, firms entering foreign markets begin with(((less risky strategies first.)))
When entering a foreign market, the least risky strategy is(((exporting.)))
Global expansion often begins with(((exporting of goods.)))
Many of the best-known American retailers, like Starbucks and McDonald’s, have contractual agreements with another firm or individuals, allowing its businesses to operate overseas. These companies expanded globally using (((franchising.)))
Gerald is assessing global entry strategies for his gourmet sandwich business. He does not want to take a lot of risk and he is willing to limit his control of international stores. Gerald will most likely use a(n) __________ strategy.(((franchising)))
Domestic firms developing a global entry strategy might consider franchising; however, the disadvantages need to be considered. All of the following are disadvantages of franchising except(((Franchising is the riskiest way to enter a foreign market.)))
When a firm pools its resources with that of a local firm to enter a new market, they create a(n)(((joint venture.)))
India, like some other countries, may require entering firms to create _________ when expanding into their markets, limiting outsiders' control of businesses.(((joint ventures)))
Of the five strategies for entering new markets, direct investment creates the(((greatest potential risk.)))
NCD Company wants to expand into the Mexican market. It has the financial resources, wants to control business operations, and has had considerable success marketing to Hispanics in the United States. NCD will likely use __________ to expand into the Mexican market.(((direct investment)))
Global segmentation, targeting, and positioning (STP) are more complicated than local STP, in part because(((consumers may view their roles differently in different countries.)))
As noted in your text, global segmenting, targeting, and positioning are more complicated than domestic segmenting and positioning because of cultural nuances, significant subcultures within countries, and(((differences in the way consumers see themselves and in the way they see products and services.)))
Cultural nuances, subcultures, and consumers' different views of their roles in different countries can make __________ complicated.(((segmentation, targeting, and positioning)))
Which of the following statements regarding global segmentation, targeting, and positioning is true?(((Companies must continually adjust products and marketing strategies to meet the changing needs of global markets.)))
Which of the following are the two components of a global marketing strategy?(((determining which target markets to pursue and developing a marketing mix to obtain a competitive advantage)))
The most important consideration when a firm chooses a global product strategy should be(((the needs of the target market.)))
Tariffs, quotas, and currency exchange policies affect global(((pricing strategies.)))
Global pricing strategies should strive to be consistent with(((positioning strategies.)))
Global marketers are under constant pressure to simplify distribution channels in order to(((reduce costs.)))
Global marketers typically find distribution in developing countries is more complex because(((they must go through many different types of distribution channels.)))
Graham had developed an extremely successful advertising and promotion campaign for a client in the United States. The client wanted to roll out the same campaign to markets worldwide, but Graham cautioned against doing this, most likely because(((differences in languages, customs, and culture might make the campaign meaningless and ineffective in some markets.)))
Celia's firm has developed a breakfast cereal targeted toward children. Rather than compete in the mature U.S. market, she has decided instead to introduce the product in Europe, where she feels it will be innovative. Her advertising agency urged caution because(((advertising regulations differ in other countries, including advertising to children.)))
Brands can be extremely valuable domestically, but challenging internationally. Companies can help overcome language difficulties in using brands by(((developing brand names that have no preexisting meaning in any known language.)))
Which of the following is one of the global entry strategies?(((direct investment)))
When a company decides to minimize risk and enter a global market by shipping its products to buyers in other countries, this is known as(((exporting.)))
Which of the following best describes the direct investment global entry strategy? (((With direct investment, a firm maintains total ownership of its plants, operation facilities, and offices in a foreign country.)))
The term trade deficit refers to (((higher levels of imports than exports.)))
When shopping for a car you notice a significant price gap between domestic and imported cars, with the imported cars being much more expensive. This could be the result of(((a tariff.)))
When entering into a franchise agreement, what term is used to refer to the firm that is granted the right to operate a business using the franchise name and business concept?(((franchisee)))
Which of the following statements best describes global expansion through a strategic alliance?(((A strategic alliance is a relationship in which two firms collaborate on a business opportunity, but do not invest in each other.)))
The XYZ Company is collaborating with a competitor on a globally based opportunity for mutual benefit, but the two competitors are not investing in one another. This is an example of(((a strategic alliance.)))
The United States imports more goods from China than it exports to China. This is known as(((a trade deficit.)))
When Ford Motor Company decided to sell the Fiesta—in the same form and design—around the globe, instead of selling different versions in different countries, this was part of Ford's global ________ strategy.(((product)))
When Cisco Systems Inc. of San Jose, California, and Tata Consultancy Services of Mumbai, India, entered into a relationship, they both continued to develop market-ready infrastructure and network solutions for customers, but they relied on each other to provide the training and skills that one or the other might have lacked. This relationship is best described as(((a strategic alliance.)))
Ford Motor Company decided to sell the Fiesta around the globe. Which of the following would be an example of glocalization of the Fiesta?(((The same product design and features in all countries, with variations in the promotional campaigns country by country.)))
One Laptop Per Child is a nonprofit initiative with the goal of making extremely low-cost laptops available to children in the developing world, with the goal of helping them learn skills needed in today's workforce. If some of the low-cost technology developed for this laptop found its way into laptops created for U.S. consumers, this would be an example of(((reverse innovation.)))
Despite multiple upturns and downturns in its economy, Russia's overall growth prospects appear promising, especially as a consumer market. However, there are still multiple dilemmas for firms trying to market their goods and services. These dilemmas include all of the following Russia's(((involvement in China.)))
What do the BRIC countries have in common?(((They are experiencing significant levels of economic growth.)))
Which of the following is currently a negative factor for foreign investment in Russia?(((Russia is known for corruption, creating ethical dilemmas for firms.)))
Which of the following is currently a negative factor for foreign investment in India?(((India's retail environment lacks modern supply chain management facilities and systems.)))
Which of the following is a potential negative factor for foreign investment in China?(((China's population is aging and is likely to continue to do so for many years.)))
A company is assessing opportunities in the BRIC companies and determines that _________ is one of the youngest populations in the world and is increasingly adopting global attitudes.(((India)))
Why should marketers be aware of the BRIC countries?(((They are likely to be the source of most market growth.)))
Which statement about India's population is true?(((India's workforce is highly skilled, particularly in technical fields.)))
A publishing company plans to outsource its production-related tasks to a BRIC country. They are assessing opportunities and are attracted to this country due to its population of young, well-educated, technically-skilled workers who are fluent in English.(((India)))
Which of the following statements is true with regard to growth in global markets?(((Changes in technology, especially communications, have been a driving force for growth in global markets for decades.)))
A U.S. company is analyzing its business prospects in Brazil. Marketing executives understand which of the following statements to be true regarding this market?(((In Brazil, Facebook has 65 million members, making it the company's second-largest market, behind the United States.)))
Which country is Europe's largest Internet market, with Internet users growing at a rate of 10 percent annually?(((Russia)))
A U.S. firm is analyzing its business prospects in China. Marketing executives understand which of the following to be true of this potential market?(((China's strict censorship rules present serious challenges; a post on Facebook could lead to jail time.)))
Which country has embraced market-oriented economic development in spite of maintaining communist political ideals?(((China)))
Which country has a rapidly aging population due to its one-child policy?(((China)))
Once a firm has done an analysis of the most viable markets for its products, then it must next(((conduct an internal assessment of its capabilities.)))
Mary wants to sell her products in Europe, since they are doing well in the United States. She does not have a lot of capital and is risk-averse, so she most likely would choose to begin with(((exporting her products.)))
Franco, a former retailer, has been living in the United States for five years and wants to start a business. He does not have an existing firm or a product, and he doesn't have a lot of capital, but since he loves McDonald's food, he decides to(((open a McDonald's franchise.)))
Sydney's Emporium has 59 stores in the United States and wants to expand globally. Sydney's wants to achieve the highest possible returns, and is not concerned about pursuing a high-risk strategy as long as it maintains complete control over its stores. The best global entry strategy for Sydney's is most likely(((direct investment.)))
When Porsche filmed an advertisement in which its vehicles ran over the Great Wall of China, Chinese consumers were left more confused than intrigued. To address or avoid such issues, one important cultural classification scheme that firms can use is Geert Hofstede’s cultural dimensions concept. Hofstede proposes that cultures differ on all of the following dimensions except(((collectivism.)))
If you visit a Kentucky Fried Chicken restaurant in China, along with KFC's regular menu items, you will find congee, a rice porridge that can feature pork, pickles, mushrooms, and preserved egg, on the menu. This is an example of which global product strategy?(((sell totally new products or services)))
According to Hofstede's cultural dimensions concept, which BRIC country posts notably high scores in the dimensions of uncertainty avoidance and power distance?(((Russia)))
Gandolph's Tires sells the same tire globally, but it uses different advertisements based on the country and culture. This is an example of(((glocalization.)))
Unilever discovered that people in emerging economies could not afford to buy standard sizes of toothpaste or shampoo, so Unilever started selling single-serve packets at very low prices. Later, Unilever discovered that the same approach worked in the United States and started also selling them there. This is an example of(((reverse innovation.)))
Core Publishing Company learned that when selling overseas, local fulfillment can be more cost-effective, and it also can decrease delivery time and improve customer service. This is an example of a global _________ strategy.(((distribution)))
In China, state control of media is high, so companies are challenged to find ways to get their message to customers. This demonstrates one of the difficulties in crafting a global ________ strategy.(((communication)))
China has three main languages, and many more dialects. This presents a particular challenge to developing a global ________ strategy.(((communication)))
The term social media refers exclusively to social networking sites like Facebook.(((FALSE)))
Firms should not attempt to educate customers about products and services using social media, since customers are busy interacting with friends and will reject attempts to educate them.(((FALSE)))
The focus of social media efforts targeted at customer interaction is building connections.(((TRUE)))
The objectives of the 4E framework for social media are excite, educate, engage, and evaluate.(((FALSE)))
Positive engagement through social media often makes customers more profitable for the firm.(((TRUE)))
The dynamic effect of social media engagement expands the impact of the network effect by examining how people flow in and out of networked communities as their interests change.(((TRUE)))
The connected effect is the outcome of social media engagement in which every time a firm or person posts information, it is transferred to the poster's vast connections across social media, causing the information to spread instantaenously.(((FALSE)))
The Wheel of Social Media Engagement includes the following five effects as drivers of social media: Information, Connected, Network, Dynamic, and Timeliness.(((TRUE)))
A blog is an example of a thought-sharing site.(((TRUE)))
The most popular microblogging site is Google+.(((FALSE)))
An advantage of a corporate blog is that all content on the blog can be completely controlled by the company.(((FALSE)))
Of the types of social media identified in the text, Flickr is considered a thought-sharing site.(((FALSE)))
Of the more than 184 million people who have smartphones in the United States, approximately 60 percent of them make purchases on these devices.(((TRUE)))
In Candy Crush Saga, you get five lives to play in the game. When you lose a life, it takes 30 minutes in real time to get that life back, or you can spend $0.99 to get those lives back. Candy Crush Saga is an example of a paid app.(((FALSE)))
When discussing the pricing models of paid apps and freemium apps, it currently appears that the best pricing model is the paid app.(((FALSE)))
A disadvantage of the current design of mobile apps is that they are able to fulfill only one need at a time.(((FALSE)))
One of the seven primary needs that apps meet is the need to deliver.(((FALSE)))
The three-stage process that firms use to engage customers through social and mobile media involves listening, analyzing, and correcting.(((FALSE)))
In order for eHarmony to understand what its clients want, both in a service provider and in a mate, the company uses metrics such as the bounce rate.(((TRUE)))
When designing a social media marketing campaign, the first step is to identify the target audience.(((FALSE)))
The development of strong, eye-catching images and designs is important in the "monitor and change" step of developing a social media campaign.(((FALSE)))
More than 12 million of LinkedIn's users are small business owners, making it an excellent resource for entrepreneurs to network with like-minded firms, identify the best vendors, or build brand reputation.(((TRUE)))
The connected effect of social media engagement is achieved when individuals post a picture to Instagram, upload a video to YouTube, or share a link to an article they have liked on Facebook.(((TRUE)))
If Katy Perry tweets a close-up of her eyelashes, lengthened by using a Cover Girl mascara product, then Cover Girl will have reached more than 80 million of Katy Perry’s followers, even though Cover Girl has fewer than 2,000 Twitter followers.(((TRUE)))
To achieve their objectives, marketers rely on three types of social media: social networking sites, media-sharing sites, and thought-sharing sites.(((TRUE)))
The 4E framework of social media includes(((excite, educate, experience, engage. )))
How does your text describe how social media firms help marketers connect with their customers?(((by helping build connections between customers and firms )))
The 4E framework for social media guides marketers in using social media effectively to build and deepen customer relationships. All of the following are included in the 4E framework except(((energy. )))
Derek just posted an entry in his blog talking about how happy he is with his new GoPro camera. In the blog he also posted videos that he took with the camera. As a customer of the product, which facet of the 4E framework for social media is Derek most exhibiting?(((engagement)))
In terms of the 4E framework, in order for a firm's offer to excite its targeted customers, the offer must be(((relevant. )))
A Groupon offer for discounted theater tickets was sent to targeted customers who had previously purchased movie or theater tickets. This best describes which of the 4E frameworks?(((excite )))
While waiting to be seated at a restaurant, Joylee receives a customer loyalty coupon through an app on her mobile phone for half off a dessert with the purchase of an entrée. This use of a location-based software application is designed to promote which of the 4E framework objectives?(((excite )))
While waiting in line to make a purchase at Best Buy, Joanie sees signs reminding her to visit the company's website for online specials and to sign up on her smartphone for the Best Buy Loyalty Rewards Program. Others signs remind her that Best Buy will match competitors' prices on any item they sell. This is an example of which 4E framework?(((educate)))
Which of the following social media tools is a type of all-in-one marketing software that can be used to educate customers?(((HubSpot )))
Alan wants to upgrade his look before a job interview. A friend tells him about a men's clothing line that allows potential customers to virtually try on suits and see what looks best. It even offers an interactive "custom fit" feature. After checking out his options, Alan takes his measurements and orders a suit without ever leaving his office. Virtually trying on a suit best describes which of the 4E framework objectives?(((experience)))
It is not unusual to see firms offering free product samples in grocery stores to let customers try the product before buying it. Similarly, in today's world of social media, one can read a chapter of a book before buying it, or watch YouTube videos on different ways to use a product. All of these scenarios are aimed at which aspect of the 4E framework?(((experience)))
Gene's Complete Cameras offers its customers an interactive website to help them choose the best camera for their lifestyle. But the site's most-used feature is its user blog in which customers—both satisfied and unsatisfied—talk about products they have purchased and the service received at Gene's. The user blog best describes which of the 4E frameworks for the store's customers?(((engage )))
When the Minnesota Timberwolves encouraged Facebook fans to post a great shot of a dunk onto its Pinterest page for a chance to win tickets to a game, which dimension of the 4E framework was being targeted?(((excite)))
Which aspect of the 4E framework is aimed at action, the potential for a relationship, and possibly even loyalty and commitment?(((engage)))
An example of the ___________ effect is when BMW gained in-depth information about visitors to a popular Chinese social media site, and used that information to determine which visitors were likely to be luxury car buyers and targeted advertising toward them.(((information)))
Individuals achieve the _____________ effect when they check in, post a picture to Instagram, upload a video to YouTube, or share a link to an article they have liked on Facebook.(((connected )))
Coca-Cola relies on _________ technology to offer their customers a free Coke at the moment they walk through a movie theater.(((beacon)))
At the Sol Wave House hotel, located on the beautiful island of Majorca, Spain, a Twitter concierge stays in constant contact with its guests. If a guest has a problem or question, the answer is just a hashtag away. This is an example of the impact of which of the following?(((dynamic effect)))
For marketers, media-sharing sites such as YouTube or Flickr are social media tools best used to(((encourage customers to engage with the firm and experience its products. )))
Trina is developing a marketing campaign for an organic stain-removal product. As "stain removal" is rather dull, she is trying to educate consumers about the value and uniqueness of the product, and appeal to the environmentally conscious. Which of the following is the best social media application for this task?(((blog )))
When the luxury auto brand Mercedes-Benz needed to target affluent potential customers with the launch of its new E- and C-class models, it turned to __________ because its users spend more online—more than 2.5 times as much as an average U.S. consumer—which likely signals their relative affluence.(((LinkedIn)))
Staples provides their loyal customers with a relevant coupon based on previous purchases through their mobile phone, while they are in the store. This represents the _____ aspect of the 4E framework of social media marketing.(((excite )))
Marketers rely on the three types of social media: social networking sites, thought-sharing sites, and(((media-sharing sites. )))
Marketers rely on social media to achieve three objectives: members promoting themselves to gain more friends, sites promoting to get more members, and(((outside companies promoting their products and services to appeal to potential customers. )))
LinkedIn is an example of which type of social media site?(((social networking)))
Your text notes that the networking benefits of LinkedIn are particularly beneficial for which group?(((small-business owners )))
What feature unique to Google+ lets people interested in similar topics form their own groups?(((Communities)))
Instagram is an example of which type of social media site?(((media sharing)))
YouTube is an example of which type of social media site?(((media sharing)))
The company Forever 21 posts company updates, photos, and videos, and participates in a discussion board on its “wall.” The social media site used is:(((Facebook)))
Lauren decides to use Twitter to market her hand-painted scarves to other women on her campus. She sets up a Twitter account with the name @uniquedesignsbylauren and posts this on her Facebook page, making sure to tweet every few days. Soon Lauren is selling a lot of scarves. Lauren's Twitter account is an example of(((microblogging. )))
This type of thought-sharing site typically provides the highest level of control for companies.(((corporate blog )))
From a marketing perspective, what role is played by professional bloggers?(((reviewing and giving product and service recommendations)))
Of the various types of blogs, which one do marketers have the lowest level of control over?(((personal )))
What is Google+?(((a social networking site )))
Suppose that Nike wanted to use Facebook to increase awareness of a new line of tennis shoes. Which of the following methods would allow Nike to specifically target Facebook users who have mentioned tennis in their profiles?(((placing a Facebook ad)))
What is a microblog?(((a blog service that supports only short posts)))
Southwest Airlines operates a blog called Nuts About Southwest. A group of employees share the responsibility for posting items of potential interest to customers—for example, short videos about Southwest's travel destinations, articles (with photos) about special events, and news about airline promotions. Nuts About Southwest is an example of a(((corporate blog. )))
Jordan loves to read and write. She started a blog about books, including writing hints and book reviews. The readership of her blog gradually increased, and eventually publishers started sending her new books, hoping she would review them. Over time, Jordan's blog has changed from a __________ blog to a __________ blog.(((personal; professional)))
All of the following are primary needs that apps meet except(((the need to share. )))
Which of the following is a pricing model for apps discussed in your text?(((ad-supported)))
Of the seven primary motivations for mobile app usage, which one do people spend the most amount of time on each month?(((need for "me time" )))
In a process called _______, a customer visits a store to touch, feel, and even discuss a product's features with a sales associate, and then instantly compares the prices online to see whether a better deal is available.(((showrooming)))
Which of the following apps would help consumers fulfill their "need to accomplish"?(((MyFitnessPal )))
Apps like calendars, trip planners, and flight trackers help consumers fulfill which primary motivation?(((to prepare)))
_______ are free to download, but appear as ads on the screen ; they generate revenue while users interact with the app.(((Ad-supported apps )))
Current research points to which pricing model for apps as the most effective?(((freemium apps)))
The elements of the social media engagement process are(((listening, analyzing, and doing.)))
Companies can find out a lot about customers using sentiment analysis on sites like Facebook and Twitter, or put another way, by(((listening. )))
One important measure of social media's effectiveness is the percentage of site visitors who take the action the site owner hoped for—making a purchase, subscribing to a service, or donating money, for example. This measure is called the site's(((conversion rate. )))
When a company that uses social media runs a contest online, it will measure its effectiveness in a variety of ways. One such measure is the conversion rate. The conversion rate for the contest promotion would be(((the percentage of visitors to the page describing the contest who entered the contest. )))
Suppose that Burger King wanted to evaluate social media content to find out how well its most recent advertising campaign was being received by consumers. This could be done using(((sentiment analysis. )))
Despite its vivid design, the website for Lolly's Bookstore did not seem to attract customers who lingered. In fact, most website visitors left the site before they made a purchase. Which measure does the owner need to address?(((bounce rates )))
Companies that specialize in sentiment analysis are least likely to look at __________ for data.(((YouTube)))
One measure of traffic from visitors on sites, the total requests for a page, is measured in units called(((hits.)))
Darren has developed a better type of medication vial for travelers. He is not sure how to develop a marketing program for his product, as there are a few similar ones on the market. What technique can Darren use to analyze data from his competitor's websites, particularly to learn how people search for similar products online?(((keyword analysis)))
What type of analytic is used to understand what’s popular and what’s not on a firm’s website, including page load times and site navigation?(((content )))
How is the cost to companies and entrepreneurs using Google Analytics calculated?(((free for everyone)))
The first step in the process of creating a digital marketing campaign is to (((set goals.)))
The final step in the process of creating a social media campaign is to(((monitor the program.)))
When developing a social media campaign, what is unique about the copy and images to be used that is more critical with social media that other forms of IMC?(((They need to be updated almost constantly. )))
What is the disadvantage for companies in using a "daily budget" such as that offered by Facebook to its advertisers?(((may limit exposure if ad is very successful )))
One of the benefits of marketing research that it reduces the uncertainty under which managers make decisions.(((TRUE)))
Marketing research should be used only to produce favorable recommendations for senior management to consider.(((FALSE)))
The marketing research process follows five steps, and to be effective they must be followed in order without omitting any steps.(((FALSE)))
Before conducting marketing research, it is important to establish in advance exactly what problem needs to be solved.(((TRUE)))
Briena is looking at the results of a syndicated study conducted two years ago. Briena is looking at primary data.(((FALSE)))
Primary data include external as well as internal data sources.(((FALSE)))
In the marketing research process, data collection happens after research design.(((TRUE)))
Warren is conducting a marketing research project. He is not sure what questions to ask or what types of consumers he should talk to. Therefore, Warren should begin by conducting quantitative research.(((FALSE)))
Quentin is in a group of 8 to 12 people and is being asked questions by a trained moderator. Quentin is part of an in-depth interview group.(((FALSE)))
Donald is analyzing and interpreting data. In the process, he is converting data into information.(((TRUE)))
Data that have been collected prior to the start of the current research project are considered primary data.(((FALSE)))
The terms external secondary data and syndicated data mean the same thing. (((FALSE)))
When companies store information on customers' purchase histories in large computer files, this is known as data warehousing.(((TRUE)))
Panel data are always secondary data.(((FALSE)))
Fingerprints are an example of biometric data.(((TRUE)))
When survey data are analyzed in depth to determine consumer attitudes toward a product, this is an example of sentiment mining.(((FALSE)))
A disadvantage to using secondary data is that they might not be precisely relevant to the information needed.(((TRUE)))
In the United States, the federal government has enacted comprehensive privacy laws for the Internet.(((FALSE)))
Neuromarketing is the process of examining consumer’s brain patterns to determine their responses to marketing communications, products, or services for the purpose of developing marketing tactics or strategies. (((TRUE)))
Companies are legally required to disclose their privacy practices to customers on an annual basis.(((TRUE)))
Marketing research includes all of the following except (((creating data.)))
Political consultants have been using marketing research for decades to help their candidates understand(((who makes up the voting public and how to reach them.)))
Paul subscribes to an Internet service that alerts him whenever other firms in his industry are quoted in the media. Paul is using this type of marketing research primarily to(((monitor his competitors.)))
Every month, Dr. Combahee takes her staff to lunch and asks them to share patients' comments and concerns. Dr. Combahee uses the lunches as an informal marketing research effort intended primarily to(((help her understand the needs of her customers.)))
Through analysis of sales data, Price-Cutters retail store found that customers who bought peanut butter also tended to buy bananas. Price-Cutters was engaged in(((data mining.)))
Walmart is known for its efficient logistical systems. Every time consumers buy something, that purchase is recorded and sent to company headquarters, where it is used to generate reorders to vendors. In addition, customers' billions of purchases are analyzed to uncover patterns of consumers' purchasing behavior. This is an example of(((data mining.)))
If a firm has demographic and purchasing information about its customers, the firm can use data mining techniques to(((build separate marketing programs for different demographic segments.)))
From charitable giving to medical records to Internet tracking, consumers are more anxious than ever about(((their fundamental right to privacy.)))
When conducting a survey about choosing vacation destinations, Hillary will need to __________ in order to get reluctant respondents to provide honest information.(((assure consumers that their individual responses will be kept confidential)))
The marketing research process follows five steps, and researchers(((may not always go through them in the exact sequence if the situation changes or new information is discovered.)))
The first question a marketing researcher should ask before embarking on a research study is(((Will the research be useful?)))
Assuming that a marketing research study will answer important questions and reduce uncertainty associated with the proposed project, a major question that needs to be addressed before starting the study is(((Is top management committed to the project and willing to abide by the results of the research?)))
Just as marketers create value by meeting the needs and wants of consumers, marketing researchers create value if(((the results will be used in making management decisions.)))
All of the following are considered qualitative research except(((experiments.)))
Wanda and Jim are working on a research project to anticipate customer attitudes toward a proposed new product line for their company. They have worked with the marketing manager to determine the answers that are needed and have created a detailed design of the project. Their next logical step will be to(((begin to collect data.)))
Academic researchers often jump at the opportunity to conduct a research study, curious to learn more and address unanswered questions for the sake of general knowledge. Businesspeople tend to be more cautious before embarking on a marketing research study, recognizing that research is often(((expensive and time-consuming.)))
When Betty decided to open up a tea shop in the center of town, she chose a group of customers who represent her customers of interest. This group is called a(((sample.)))
You work for a company that sells baby products. Your manager asks you to conduct research on automotive sales in your area. The problem with this research objective is that(((it is irrelevant to the baby products company.)))
The manager of a company selling cellular phones in rural U.S. markets asks you to conduct survey research into the question, What percentage of people in Kansas live in communities of less than 10,000 people? The problem with this research objective is that(((it is already known and available from the U.S. Census Bureau.)))
After defining objectives and research needs, the next step in the marketing research process involves(((research design.)))
During the research design step of the marketing research process, researchers identify the type of data needed and(((the type of research necessary to collect the data.)))
Victor's custom auctioneering service provides online auction selling assistance. Victor plans to conduct marketing research to determine which auction site is best for his customers. His next step is to(((identify the type of data he needs.)))
Company sales invoices, census data, and trade association statistics are examples of(((secondary data.)))
Commercial research firms such as IRI, the National Purchase Diary Panel, and Nielsen are sources of(((syndicated data.)))
Jaleel is the marketing manager for a moderately well-known rock band. He wants to know more about industry trends including sales by different musical styles, online downloads, and concert attendance. Jaleel will most likely use __________ to gather this type of data.(((syndicated data)))
Once a marketing researcher is ready to move beyond preliminary insights to specific, informed questions, the researcher is ready to conduct(((quantitative research.)))
A marketing research project often begins with a review of the relevant __________ data.(((secondary)))
Joe is reviewing secondary data his company collected about seasonal variations in consumer spending because he is thinking about developing a new product line. The advantages of using these data include(((they can be quickly accessed at a relatively low cost.)))
Omar is responsible for marketing and marketing research for a midsized manufacturer of assemblies for the housing market. His boss has asked him to cut back expenses, especially in marketing research: "Why can't you just use information off the Internet? There's plenty out there." What is Omar's best response to try to get his boss to change his mind?(((Those are secondary data, and they may not be as timely, accurate, and relevant as what we need.)))
Marvin is looking for data to help with a new marketing research study. When reviewing existing secondary data from a past research study, Marvin should pay careful attention to(((how the secondary data were collected.)))
After reviewing the existing data on seasonal spending by his company's customers, Marvin decided he needed new information collected to address his research questions. Marvin will need __________ data to address the questions in his marketing research study.(((primary)))
A major advantage of primary data collection is(((it offers behavioral insights generally not available from secondary research.)))
A major disadvantage of primary data collection is(((it requires more sophisticated training and experience to design the study and collect data.)))
Caroline needs to find information about income and age distribution in Orange County, California. The best source of secondary research of use to Caroline is likely to be(((U.S. Census data.)))
Data collection begins(((only after completing the research design process.)))
When the marketing research problem is not clearly defined, a researcher will likely engage in __________ research.(((qualitative)))
Recently, the number of students enrolled in the marketing program dropped while enrollment in the psychology program increased. The chair of the marketing department will probably use __________ as a first step to gain a better understanding of why enrollments are changing.(((qualitative research)))
Quantitative research offers a means to confirm ideas through(((surveys or experiments.)))
Bianca's discount home furnishings store is in a strip mall. She wants to know what other businesses in the strip mall her customers visit when they come to her store. To collect information for this objective, Bianca will most likely use(((observation.)))
Which of the following is true about quantitative research?(((It confirms insights and hypotheses generated via qualitative research.)))
The purpose of __________ is that it provides information needed to confirm insights and hypotheses generated and helps managers pursue appropriate courses of action.(((quantitative research)))
When consumers are __________, observation becomes particularly useful in understanding consumers' preferences.(((unable to articulate their experiences)))
When consumers are unable to articulate their experiences, __________ becomes particularly useful in understanding consumers' preferences.(((observation)))
Using __________, trained researchers ask questions, listen to and record the answers, and then pose additional questions to clarify or expand on a particular issue.(((in-depth interviews)))
Social media monitoring, in-depth interviews, and focus groups are all __________ research methods.(((qualitative)))
Zan wants to collect considerable information about the current opinions of his ten most important customers. Zan will probably use the __________ research method.(((in-depth interview)))
Which of the following research methods would be best in helping a marketer understand how people feel about a product or service on an individual, detailed level?(((in-depth interviews)))
When the detailed opinions of a few industry experts or experienced consumers are needed, __________ is(are) often the best qualitative research method.(((in-depth interviews)))
Randall wants to do an online survey of college professors about the factors that influence their textbook selection. He would like to use a structured survey but is not sure what responses to include for each question. Randall could use __________ to help him develop his survey.(((in-depth interviews)))
Frederica manages an upscale women's clothing store. She wants more information about her customers' general feelings about upcoming fall fashions. Frederica will most likely use __________ to gather this type of data.(((focus group interviews)))
A(n) __________ is a small group of people brought together for an intensive discussion of a topic.(((focus group)))
In a focus group, researchers usually videotape or audiotape the session in order to(((catch any patterns of verbal or nonverbal responses.)))
Tyree is concerned about a competitor's new line of outdoor barbeque grills that provide most of the same features as his products. Tyree might consider using __________ to gather qualitative information from consumers about the competitor's offerings.(((focus groups)))
Martin has hired a marketing research company to bring together a small group of soft drink consumers and get feedback on the three new advertising slogans his firm is considering. The marketing research firm might conduct a(n) __________ to provide the information Martin has requested.(((focus group)))
In the infamous Coke-New Coke taste test, 54 percent of consumers, using a blind taste test, preferred the New Coke formula to the existing formulation. This is an example of a(n) __________ marketing research method.(((quantitative)))
Malcolm is the campaign manager for a congressional candidate. He wants to know how voters across his district feel about recent sexual harassment scandals involving politicians. Malcolm will most likely use __________ to gather this type of data.(((surveys)))
When conducting a research study attempting to understand what features were most important to automobile consumers, Gary's Research Company used a questionnaire containing __________ questions, with a predetermined set of response options.(((structured)))
Quincy had no idea how consumers would respond to a survey about attitudes toward a program opening up space travel to private citizens. He could use __________ to allow respondents to answer in their own words.(((a survey with open-ended questions)))
The owner of A. C. Flora Ford Dealership wanted to know why consumers chose his company to purchase a car. He was fairly sure that most customers had one of three reasons—service, reputation, or location—but wanted to know which was revealed as the most frequent reason for purchasing. A. C. Flora will probably use a survey with __________ questions to address his research problem.(((closed-ended)))
All of the following are guidelines for developing a marketing research questionnaire except(((Sensitive questions should be asked first.)))
In questionnaire design, a question such as "When was the first time you went to a dentist?"(((is a question that respondents cannot easily or accurately answer.)))
In questionnaire design, a question such as "Marketing is the best part of the business program, isn't it?"(((leads respondents to a particular response.)))
In questionnaire design, a question such as "Do you like Wendy's hamburgers and fries?"(((asks two questions at once.)))
In questionnaire design, a question such as "Do anti-lock braking systems reduce car accidents?"(((is a question that respondents cannot easily or accurately answer.)))
Charles wants to survey recent customers about the quality of service they received at his small auto service dealership. He has customers' mailing and e-mail addresses. Charles will likely use an online survey primarily because it offers(((fast responses at a lower cost.)))
__________ research is a type of quantitative research that manipulates variables to help determine cause and effect.(((Experimental)))
Each time you go to the grocery store and have your purchases scanned while using a loyalty or bonus reward card, you are contributing to a database that can help marketers determine all of the following except(((other stores where you buy similar products.)))
Supermarkets collect information about individual customers through their use of loyalty cards, and then analyze the data to look for patterns in purchases. This is an example of(((data mining.)))
When a research team has gathered data for specific research needs, this is known as(((primary data.)))
Which of the following statements best describes secondary data?(((Secondary data are pieces of information that have been collected prior to the start of the focal research project.)))
Which of the following types of research would be considered quantitative research?(((experimental research)))
Imagine that you currently operate a clothing boutique, and you want to add another boutique in a new location. You need to obtain data to determine the size of your potential market. Which data source would you use?(((U.S. Bureau of the Census)))
With more than 200 million active customers and billions of pieces of shopping data, Amazon qualifies as a(((big data user.)))
All of the following are guidelines provided by the American Marketing Association for conducting marketing research except(((personal information about research participants should not be shared, except with a designated member of senior management.)))
The number of participants who discontinue their use of a service divided by the average number of total participants is called(((churn.)))
All of the following data sources would be considered external secondary data except(((a survey.)))
Which of the following types of data would be the best choice for Wendy's fast-food restaurants if the headquarters office wants to know how many hamburgers versus chicken sandwiches it has sold in past years?(((internal secondary data)))
A restaurant chain is working on improving the quality of its food and service. To track its progress it recruits customers who agree to respond to customer satisfaction surveys once every three months over the next two years. What kind of data is the restaurant chain collecting?(((panel data)))
Which of the following is an unstructured question?(((What are the most important characteristics for choosing a brand of shampoo?)))
Which of the following provides the information needed to confirm insights and hypotheses and help managers pursue appropriate courses of action?(((quantitative research)))
Coca-Cola’s experiments using ____________ record participants’ faces as they watch advertisements or prototypes, then assess how their eyes moved, when they smiled or frowned, and so on.(((facial recognition software)))
What is neuromarketing?(((the process of examining consumers’ brain patterns to determine their responses to marketing communications, products, or services for the purpose of developing marketing tactics or strategies)))
McDonald's introduced a Favorites Under 400 Calorie Menu as part of an attempt to reverse the perception that McDonald's sells only unhealthy food. Suppose that McDonald's, as a follow-up, collects and analyzes social media posts from Facebook, Twitter, and similar sites, hoping to understand whether or not consumer perceptions are improving. This would be an example of(((sentiment mining.)))
How might a book publisher use blogs in its marketing research efforts?(((as a source of reviews of its latest releases)))
What type of interview contains a small group of participants and a trained moderator that guides the conversation?(((focus group)))
Which of the following is an example of secondary research?(((journal articles)))
Digital billboards embedded with ________ software can identify passersby and then display ads targeted to them, based on their age, gender, and attention level.(((facial recognition)))
Mary's Bakery is trying to determine what price to charge for its cookies, so Mary places a low price on them and raises it each week, keeping track of how many she sells. Then she determines what her cost was for the cookies and determines which price was the most profitable. What form of research was Mary conducting?(((an experiment)))
Qualtrics, SurveyMonkey, and Zoomerang are examples of(((online survey software.)))
Which of the following is an example of an unstructured question?(((Why did you buy this product?)))
A typical focus group has ________ members.(((8 to 12)))
Nadia stood outside the mall and asked people which stores they visited and if they bought anything. If they said yes, she asked them what they bought and how they came to the decision to buy that item. What form of research was Nadia most likely conducting?(((in-depth interviews)))
Firms that are engaged in sentiment mining are analyzing data collected from(((social media sites.)))
The term _______ refers to the enormous amount of information that firms now have access to but cannot handle using conventional data management and data mining software.(((big data)))
Mattel wanted to determine if a new toy would appeal to preschoolers, so it put six 4-year-olds in a room with several toys and waited to see which ones they played with. What form of research is this?(((observation)))
If a cable television company had 100 million subscribers at the end of the year, and 2 million of its subscribers left its service, it would have a churn rate of(((2 percent.)))
Before deciding on a new promotional campaign, Jeffrey's Muffins looked at its customers' buying patterns over a 15-year period as determined by their use of a loyalty card. What method did Jeffrey's use to make sense of the data that were available?(((data mining)))
Scanner data are used in quantitative research obtained from scanner readings of Universal Product Code (UPC) labels at checkout counters. The data from these purchases are likely to be acquired by leading marketing research firms such as ___________, which use this information to help leading consumer packaged-goods firms (e.g., Kellogg’s, Pepsi, Kraft) assess what is happening in the marketplace.(((IRI and Nielsen)))
Both major political parties have developed proprietary databases that contain vast information about voters, broken down by demographic and geographic information. This kind of information is(((secondary data.)))
Marketing executives at Diet Pepsi want to analyze the number of Diet Coke customers who switch to its product when Diet Pepsi is offered at a deep discount. What data would likely show this information?(((secondary panel data)))
______is a disadvantage of secondary research, while ________ is a disadvantage of primary research.(((Potential bias; cost)))
"Which of the following is one of the four major growth strategies marketers typically utilize? (( market penetration ))
Manufacturers, retailers, and service providers have created and maintain websites, blogs, and _________ to enable customers to interact with them over the Internet. ((a social media presence))
Which of the following is a traditional contextual element of website design? ((color))
Stakeholders typically include the firm’s employees and their families, customer groups, members of the community, the environment, and the firm’s partners, and competitors. ((true))
Simons & Co. prides itself on a strong ethical climate. Compared to many companies, Simons ((is likely more socially responsible.))
In the ethical decision-making framework, brainstorming occurs immediately following the identification of issues. ((False))
The decision-making metric includes ((the publicity test.))
The value of an influencer may be determined by the number of followers they have. ((True))
An example of a popular microblogging site is (((twitter)))
________ entails a sense of purpose for the firm that is higher than simply making a profit by selling products and services. ((Conscious marketing))
Which of the following is the most powerful and influential online marketer in the United States? ((Amazon))
This type of blog typically provides the highest level of digital marketing control for companies. ((corporate blog))
Which of the following would be considered informational content on a website? ((Audio))
Twitter is considered a ((microblog))
The final step in the process of creating a digital marketing campaign is to ((set goals.))
The type of influencer that has a large following and is widely recognized is usually known as a ________ influencer. ((celebrity))
The objectives of the 4E framework for digital marketing are excite, educate, engage, and evaluate. ((False))
In Candy Crush Saga, you get five lives to play in the game. When you lose a life, it takes 30 minutes in real-life time to get that life back, or you can spend $0.99 to get all five lives back. Candy Crush Saga is an example of ((freemium app.))
What is the basis of any marketing strategy? ((goals))
A ________ is an example of a thought-sharing site. ((blog or microblog))
Firms that hire (or encourage) well-known names to promote brand messages to their networks of followers are using ((influencer marketing.))
What tools help marketers make sense out of the data and use it to make appropriate business decisions? ((marketing analytics))
A marketing research project often begins with a review of the relevant ________ data. ((secondary))
Tariffs artificially lower prices and make goods more competitive. ((False))
The first step of the marketing research process is (((defining the objectives and needs.)))
The components of global market assessment include ((infrastructure and technological analysis.))
The most common measure of market potential of an economy is a country’s ((GDP))
There is only one global product strategy: to sell a product or service similar to that sold in the home country but include minor adaptations.((False))
________ is the process of examining consumers’ brain patterns to determine their responses to marketing communications, products, or services for the purpose of developing marketing tactics or strategies. ((Neuromarketing))
A ________ limits the quantity of imported merchandise, thus minimizing competition faced by domestic products. ((quota))
Global expansion often begins when a firm receives an order for its product or service from another country. ((True))
In a(n) ________, the burden of ownership, control, and profits are shared. ((joint venture))
Which of the following are considered qualitative research? ((focus groups))
A company is assessing opportunities in the BRIC countries and determines that ________ is one of the youngest populations in the world and is increasingly adopting global attitudes. ((India))
Which app pricing model is used by the app Candy Crush Saga? (( freemium apps))
What attributes are product or service features that are important to the buyer and on which competing brands or stores are perceived to differ? (( determinant ))
According to Maslow’s hierarchy of needs, which needs are the needs that people first seek to meet? ((physiological))
An entertaining post likely spreads further than a basic, informational one. This factor contributes to (( reach ))
The definition of corporate social responsibility addresses (( environmental impact. ))
American Express Open Forum site invites business experts to share their wisdom in various posts. This is an example of what type of blog? (( corporate blog ))
The most loyal customers use (( multiple channels. ))
Thought-sharing sites are also known as (( blogs ))
Another name for physiological risk is (( safety risk ))
In a process called _______, a customer visits a store to touch, feel, and even discuss a product’s features with a sales associate, and then instantly compares the prices online to see whether a better deal is available. ((Showrooming))
A(n) ________ is one or more persons whom an individual uses as a basis for comparison regarding beliefs, feelings, and behaviors. (( reference group ))
For every consumer who purchases a pair of TOMS shoes, the company promises that a needy child will receive a pair of shoes. TOMS shoes is actively engaging in ((corporate social responsibility. ))
Economics is best defined as the study of:((how people make choices in the face of scarcity and the implications of those choices for society as a whole.))
Economics is best defined as the study of:((choice in the face of limited resources.))
Economists recognize that because people have limited resources:((they have to make trade-offs.))
The Scarcity Principle states that:((with limited resources, having more of one thing means having less of another.))
An implication of scarcity is that:((people must make trade-offs.))
If all the world's resources were to magically increase one hundredfold, then:((people would still have to make trade-offs.))
The Scarcity Principle applies to:((everyone.))
Forest lives in complete isolation in Montana. He is self-sufficient and feeds himself through hunting, fishing, and farming. Which of the following statements about Forest is true?((Forest has to make trade-offs.))
The Scarcity Principle applies to:((all decisions.))
Chris has a one-hour break between classes every Wednesday. Chris can either stay at the library and study or go to the gym and work out. The decision Chris must make is:((an economic problem because Chris has only one hour, and engaging in one activity means giving up the other.))
Josh wants to go to the football game this weekend, but he has a paper due on Monday. It will take him the whole weekend to write the paper. Josh decides to stay home and work on the paper. According to the Scarcity Principle, the reason Josh doesn't go to the game is that:((Josh can't go the game and finish the paper.))
Whether studying the output of the U.S. economy or how many classes a student will take, a unifying concept is that:((wants are unlimited and resources are scarce, so trade-offs have to be made.))
The Cost-Benefit Principle indicates that an action should be taken if:((its extra benefit is greater than or equal to its extra cost.))
If a person takes an action if, and only if, the extra benefits from taking that action are at least as great as the extra costs, then that person is:((following the Cost-Benefit Principle.))
Choosing to study for an exam until the extra benefit (e.g., improved score equals the extra cost (e.g., the value of foregone activities is:((an application of the Cost-Benefit Principle.))
The Scarcity Principle tells us ________, and the Cost-Benefit Principle tells us ________.((that choices must be made; how to make good choices))
According to the Cost-Benefit Principle, you should go see the latest Star Wars movie with your friends this weekend if:((the extra benefits of seeing the movie are greater than the extra costs of seeing the movie.))
A rational person:((makes choices based on added benefits and added costs.))
Suppose that the extra cost to Tim of a third glass of soda is zero because he's at a restaurant that gives free refills. According to the Cost-Benefit Principle Tim should:((drink a third glass of soda if the extra benefit of doing so is positive.))
Janie must choose to either mow the lawn or wash clothes. If she mows the lawn, she will earn $30, and if she washes clothes, she will earn $45. She dislikes both tasks equally and they both take the same amount of time. Janie will therefore choose to ________ because it generates a ________ economic surplus.((wash clothes; bigger))
Dean should play golf instead of preparing for tomorrow's exam in economics if:((the economic surplus from playing golf is greater than the economic surplus from studying.))
Larry was accepted at three different graduate schools, and must choose one. Elite U costs $50,000 per year and did not offer Larry any financial aid. Larry values attending Elite U at $60,000 per year. State College costs $30,000 per year, and offered Larry an annual $10,000 scholarship. Larry values attending State College at $40,000 per year. NoName U costs $20,000 per year, and offered Larry a full $20,000 annual scholarship. Larry values attending NoName at $15,000 per year. Larry's opportunity cost of attending Elite U is:(($70,000 ))
Larry was accepted at three different graduate schools, and must choose one. Elite U costs $50,000 per year and did not offer Larry any financial aid. Larry values attending Elite U at $60,000 per year. State College costs $30,000 per year, and offered Larry an annual $10,000 scholarship. Larry values attending State College at $40,000 per year. NoName U costs $20,000 per year, and offered Larry a full $20,000 annual scholarship. Larry values attending NoName at $15,000 per year. Larry's opportunity cost of attending State College is:(($35,000 ))
Larry was accepted at three different graduate schools, and must choose one. Elite U costs $50,000 per year and did not offer Larry any financial aid. Larry values attending Elite U at $60,000 per year. State College costs $30,000 per year, and offered Larry an annual $10,000 scholarship. Larry values attending State College at $40,000 per year. NoName U costs $20,000 per year, and offered Larry a full $20,000 annual scholarship. Larry values attending NoName at $15,000 per year. Larry's opportunity cost of attending State NoName U is:(($20,000 ))
Larry was accepted at three different graduate schools, and must choose one. Elite U costs $50,000 per year and did not offer Larry any financial aid. Larry values attending Elite U at $60,000 per year. State College costs $30,000 per year, and offered Larry an annual $10,000 scholarship. Larry values attending State College at $40,000 per year. NoName U costs $20,000 per year, and offered Larry a full $20,000 annual scholarship. Larry values attending NoName at $15,000 per year. Larry maximizes his economic surplus by attending:((State College.))
Larry was accepted at three different graduate schools, and must choose one. Elite U costs $50,000 per year and did not offer Larry any financial aid. Larry values attending Elite U at $60,000 per year. State College costs $30,000 per year, and offered Larry an annual $10,000 scholarship. Larry values attending State College at $40,000 per year. NoName U costs $20,000 per year, and offered Larry a full $20,000 annual scholarship. Larry values attending NoName at $15,000 per year. What is Larry's economic surplus from attending State College instead of his next best alternative?(($5,000 ))
Jen spends her afternoon at the beach, paying $1 to rent a beach umbrella and $11 for food and drinks rather than spending an equal amount of money to go to a movie. Her opportunity cost of going to the beach is:((the value she places on seeing the movie.))
All else equal, relative to a person who earns minimum wage, a person who earns $30 per hour has:((a higher opportunity cost of taking the day off work.))
The opportunity cost of an activity includes the value of:((the next-best alternative that must be foregone.))
Amy is thinking about going to the movies tonight. A movie ticket costs $15, and she'll have to cancel a $20 dog-sitting job that she would have been willing to do for free. The opportunity to Amy cost of going to the movies is:(($35.00 ))
The economic surplus of an action is:((the difference between the benefit and the cost of taking an action.))
You are trying to decide whether to purchase the latest Harry Potter book online or borrow it from the library. There is no charge for borrowing a book from the library, but going to the library takes more time than ordering a book online. Regardless of how you get the book, its benefit to you is the same. If the cost of buying the book online is $13, then you should:((borrow the book from the library if the cost of doing so (in terms of the extra time it takes) is less than $13.))
Alex received a four-year scholarship to State U. that covered tuition and fees, room and board, and books and supplies. If Alex becomes a full-time student, then:((the opportunity cost of attending State U. includes the money Alex could have earned working for four years.))
Suppose Mary is willing to pay up to $15,000 for a used Ford pick-up truck. If she buys one for $12,000, her ________ would be ________.((economic surplus; $3,000))
If individuals are rational, they should choose actions that yield the:((largest economic surplus.))
Suppose the most you would be willing to pay for a plane ticket home is $250. If you buy one for $175, then your economic surplus is:(($75.00 ))
The Cost-Benefit Principle:((provides an abstract model of how people should choose between alternatives.))
Economists believe the Cost-Benefit Principle is:((a simple but useful model of how people should make choices.))
The cost-benefit model used by economists is:((useful because most people follow it most of the time.))
Economists use abstract models because:((they are useful for describing general patterns of behavior.))
The fact that most people make some decisions based on intuition rather than calculation is:((consistent with the cost-benefit model because most people intuitively weigh costs and benefits.))
Moe has a big exam tomorrow. He considered studying this evening, but decided to hang out with Curly instead. If neither activity involves any explicit costs, and Moe always chooses rationally, it must be true that:((Moe gets more benefit from spending time with Curly than from studying.))
If one fails to account for implicit costs in decision making, then applying the cost-benefit rule will be flawed because:((the costs will be understated.))
Your classmates from the University of Chicago are planning to go to Miami for spring break, and you are undecided about whether you should go with them. The round-trip airfare is $600, but you have a frequent-flyer coupon worth $500 that you could use to pay part of the airfare. All other costs for the vacation are exactly $900. The most you would be willing to pay for the trip is $1,400. Your only alternative use for your frequent-flyer coupon is for your trip to Atlanta two weeks after the break to attend your sister's graduation, which your parents are forcing you to attend. The Chicago-Atlanta round-trip airfare is $450. If you do not use the frequent-flyer coupon to fly to Miami, should you go to Miami?((No, your benefit is less than your cost.))
Your classmates from the University of Chicago are planning to go to Miami for spring break, and you are undecided about whether you should go with them. The round-trip airfare is $600, but you have a frequent-flyer coupon worth $500 that you could use to pay part of the airfare. All other costs for the vacation are exactly $900. The most you would be willing to pay for the trip is $1,400. Your only alternative use for your frequent-flyer coupon is for your trip to Atlanta two weeks after the break to attend your sister's graduation, which your parents are forcing you to attend. The Chicago-Atlanta round-trip airfare is $450. What is the opportunity cost of using the coupon for the Miami trip?(($450 ))
Your classmates from the University of Chicago are planning to go to Miami for spring break, and you are undecided about whether you should go with them. The round-trip airfare is $600, but you have a frequent-flyer coupon worth $500 that you could use to pay part of the airfare. All other costs for the vacation are exactly $900. The most you would be willing to pay for the trip is $1,400. Your only alternative use for your frequent-flyer coupon is for your trip to Atlanta two weeks after the break to attend your sister's graduation, which your parents are forcing you to attend. The Chicago-Atlanta round-trip airfare is $450. Should you use the frequent flyer coupon to go to Miami?((No, your benefit is less than your cost.))
Your classmates from the University of Chicago are planning to go to Miami for spring break, and you are undecided about whether you should go with them. The round-trip airfare is $600, but you have a frequent-flyer coupon worth $500 that you could use to pay part of the airfare. All other costs for the vacation are exactly $900. The most you would be willing to pay for the trip is $1,400. Your only alternative use for your frequent-flyer coupon is for your trip to Atlanta two weeks after the break to attend your sister's graduation, which your parents are forcing you to attend. The Chicago-Atlanta round-trip airfare is $450. If the Chicago-Atlanta round-trip air fare were $350, should you use the coupon to go to Miami?((Yes, your economic surplus would be $50.))
Pat earns $25,000 per year (after taxes), and Pat's spouse, Chris, earns $35,000 (after taxes). They have two pre-school-aged children. Childcare for their children costs $12,000 per year. Given that Chris doesn't want to stay home with the kids, regardless of what Pat does, Pat should stay home with the kids if, and only if, the value of Pat spending more time with the kids is greater than:(($13,000 per year.))
You paid $35 for a ticket (which is non-refundable to see SPAM, a local rock band, in concert on Saturday. Assume that $35 is the most you would have been willing to pay for a ticket. Your boss called, and she is looking for someone to cover a shift on Saturday at the same time as the concert. You would have to work 4 hours and she would pay you $11/hr. The psychic cost to you of working is $2/hr. Should you go to the concert instead of working Saturday?((No, the benefit of going to the concert is less than the cost.))
You paid $35 for a ticket (which is non-refundable to see SPAM, a local rock band, in concert on Saturday. Assume that $35 is the most you would have been willing to pay for a ticket. Your boss called, and she is looking for someone to cover a shift on Saturday at the same time as the concert. You would have to work 4 hours and she would pay you $11/hr. The psychic cost to you of working is $2/hr. What is the opportunity cost of going to the concert?(($36 ))
You paid $35 for a ticket (which is non-refundable to see SPAM, a local rock band, in concert on Saturday. Assume that $35 is the most you would have been willing to pay for a ticket. Your boss called, and she is looking for someone to cover a shift on Saturday at the same time as the concert. You would have to work 4 hours and she would pay you $11/hr. The psychic cost to you of working is $2/hr. What is your opportunity cost of going to work on Saturday?(($35 ))
You paid $35 for a ticket (which is non-refundable to see SPAM, a local rock band, in concert on Saturday. Assume that $35 is the most you would have been willing to pay for a ticket. Your boss called, and she is looking for someone to cover a shift on Saturday at the same time as the concert. You would have to work 4 hours and she would pay you $11/hr. The psychic cost to you of working is $2/hr. Your economic surplus from going to work instead of seeing SPAM on Saturday is:(($1 ))
Matt has decided to purchase his textbooks for the semester. His options are to purchase the books online with next day delivery at a cost of $175, or to drive to campus tomorrow to buy the books at the university bookstore at a cost of $170. Last week he drove to campus to buy a concert ticket because they offered 25 percent off the regular price of $16. The benefit to Matt of buying his books at the university bookstore instead of online is:(($5 ))
Matt has decided to purchase his textbooks for the semester. His options are to purchase the books online with next day delivery at a cost of $175, or to drive to campus tomorrow to buy the books at the university bookstore at a cost of $170. Last week he drove to campus to buy a concert ticket because they offered 25 percent off the regular price of $16. The benefit to Matt of driving to campus to buy the concert ticket last week was:(($4 ))
Matt has decided to purchase his textbooks for the semester. His options are to purchase the books online with next day delivery at a cost of $175, or to drive to campus tomorrow to buy the books at the university bookstore at a cost of $170. Last week he drove to campus to buy a concert ticket because they offered 25 percent off the regular price of $16. Given that driving to campus to buy the concert ticket was rational for Matt, Matt should:((drive to campus to buy the books because the $5 he would save is more than he saved by driving to campus to buy the concert ticket.))
Matt has decided to purchase his textbooks for the semester. His options are to purchase the books online with next day delivery at a cost of $175, or to drive to campus tomorrow to buy the books at the university bookstore at a cost of $170. Last week he drove to campus to buy a concert ticket because they offered 25 percent off the regular price of $16. The benefit to Matt of driving to campus to buy the concert ticket last week was:(($4 ))
Matt has decided to purchase his textbooks for the semester. His options are to purchase the books online with next day delivery at a cost of $175, or to drive to campus tomorrow to buy the books at the university bookstore at a cost of $170. Last week he drove to campus to buy a concert ticket because they offered 25 percent off the regular price of $16. Given that driving to campus to buy the concert ticket was rational for Matt, Matt should:((drive to campus to buy the books because the $5 he would save is more than he saved by driving to campus to buy the concert ticket.))
Matt has decided to purchase his textbooks for the semester. His options are to purchase the books online with next day delivery at a cost of $175, or to drive to campus tomorrow to buy the books at the university bookstore at a cost of $170. Last week he drove to campus to buy a concert ticket because they offered 25 percent off the regular price of $16. Assume the minimum that Matt would be willing to accept to drive to campus is equal to the $4 he saved on the concert ticket. What would his economic surplus be if he bought his textbooks at the university bookstore rather than online?(($1 ))
The marginal benefit of an activity is the:((extra benefit associated with an extra unit of the activity.))
Suppose the total benefit of watching 1 baseball game is 100, the total benefit of watching 2 games is 120, and the total benefit of watching 3 games is 125. In this case, the marginal benefit of watching the 3rd game is:((5))
The extra benefit that results from carrying out one additional unit of an activity is the ________ of the activity.((marginal benefit))
The marginal cost of an activity is the:((change in the total cost of the activity that results from carrying out an additional unit of the activity.))
The extra cost that results from carrying out one additional unit of an activity is the ________ of the activity.((marginal cost))
Dividing the total cost of undertaking n units of an activity by n reveals the:((average cost.))
For the fall semester, you had to pay a nonrefundable fee of $600 for your meal plan, which gives you up to 150 meals. If you eat all of the meals, your average cost for a meal is:(($4.00 ))
For the fall semester, you had to pay a nonrefundable fee of $600 for your meal plan, which gives you up to 150 meals. If you eat 100 meals, your average cost for a meal is:(($6.00 ))
For the fall semester, you had to pay a nonrefundable fee of $600 for your meal plan, which gives you up to 150 meals. If you eat 100 meals, your marginal cost of the 100th meal is:(($0.00 ))
The average benefit of n units of an activity is the:((total benefit of n units divided by n.))
You save $10 on gas every week because you take the bus to school. You have class 5 days a week. What is your average benefit per day of taking the bus to school?(($2 ))
Your scholarship depends on your maintaining a 3.5 cumulative GPA. Your GPA for last semester was 3.6, which brought your cumulative GPA down. What must be true?((Last semester's grades were lower than your overall GPA.))
Refer to the accompanying table below. The average cost of 4 units of this activity is:(($25 ))
Refer to the accompanying table below. The marginal cost of the 3rd unit of this activity is:(($20 ))
Refer to the accompanying table below. The average benefit of 2 units of activity is:(($80 ))
Refer to the accompanying table below. The marginal benefit of the 5th unit of activity is:(($10 ))
Refer to the accompanying table below. According to the Cost-Benefit Principle, how many units of this activity should be carried out?((3))
Refer to the accompanying table below. The average cost of 5 units of activity is:(($6 ))
Refer to the accompanying table below. The marginal cost of the 4th unit of activity is:(($8 ))
Refer to the accompanying table below. The average benefit of 4 units of activity is:(($9 ))
Refer to the accompanying table below. The marginal benefit of the 6th unit of activity is:(($2 ))
Refer to the accompanying table below. According to the Cost-Benefit Principle, how many units of this activity should be carried out?((3))
The accompanying table below shows how total donations, average donations, total labor costs and average labor costs vary depending on the number of employees State U hires for its fundraising activities.(($51,963.00 ))
The accompanying table below shows how total donations, average donations, total labor costs and average labor costs vary depending on the number of employees State U hires for its fundraising activities. The total labor cost with 4 employees is:(($38,000.00 ))
The accompanying table below shows how total donations, average donations, total labor costs and average labor costs vary depending on the number of employees State U hires for its fundraising activities. The President of State U decides to hire fundraising employees as long as their average benefit exceeds their average cost. This results in ________ employees being hired and a net benefit (total donations minus total labor costs of ________.((5; $17,080))
The accompanying table below shows how total donations, average donations, total labor costs and average labor costs vary depending on the number of employees State U hires for its fundraising activities. The marginal benefit (in terms of extra donations of the 2nd employee is:(($12,426.00 ))
The accompanying table below shows how total donations, average donations, total labor costs and average labor costs vary depending on the number of employees State U hires for its fundraising activities. The marginal cost of the 4th employee is:(($11,000.00 ))
The accompanying table below shows how total donations, average donations, total labor costs and average labor costs vary depending on the number of employees State U hires for its fundraising activities. The Chair of the Economics Department at State U says that fundraising employees should be hired as long as their marginal benefit exceeds their marginal cost. If the University follows this advice, then ________ employee(s will hired, and the net benefit (total donations minus total labor costs will be ________.((2; $25,426))
The accompanying table below shows how total donations, average donations, total labor costs and average labor costs vary depending on the number of employees State U hires for its fundraising activities. The net benefit of hiring fundraisers is largest when ________ employees are hired.((2))
Ginger bought a phone that came with a $10 rebate. Ginger should fill out and mail in the rebate form if:((the opportunity cost of the time and trouble of sending in the rebate form is less than $10.))
Tony notes that an electronics store is offering a flat $20 off all prices in the store. Tony reasons that if he wants to buy something with a price of $50, then it is a good offer, but if he wants to buy something with a price of $500, then it is not a good offer. This is an example of:((inconsistent reasoning; saving $20 is saving $20.))
Suppose a retail store was offering 10 percent off list prices on all goods. The benefit of the 10 percent savings is:((positively related to the list price of the good.))
A firm pays Pam $40 per hour to assemble personal computers. Each day, Pam can assemble 4 computers if she works 1 hour, 7 computers if she works 2 hours, 9 computers if she works 3 hours, and 10 computers if she works 4 hours. Pam cannot work more than 4 hours day. Each computer consists of a motherboard, a hard drive, a case, a monitor, a keyboard, and a mouse. The total cost of these parts is $600 per computer. What is the marginal cost of producing the computers that Pam can assemble during her 3rd hour of work?(($1,240 ))
A firm pays Pam $40 per hour to assemble personal computers. Each day, Pam can assemble 4 computers if she works 1 hour, 7 computers if she works 2 hours, 9 computers if she works 3 hours, and 10 computers if she works 4 hours. Pam cannot work more than 4 hours day. Each computer consists of a motherboard, a hard drive, a case, a monitor, a keyboard, and a mouse. The total cost of these parts is $600 per computer. What is the marginal cost of producing the computers that Pam can assemble during her 2nd hour of work?(($1,840 ))
A firm pays Pam $40 per hour to assemble personal computers. Each day, Pam can assemble 4 computers if she works 1 hour, 7 computers if she works 2 hours, 9 computers if she works 3 hours, and 10 computers if she works 4 hours. Pam cannot work more than 4 hours day. Each computer consists of a motherboard, a hard drive, a case, a monitor, a keyboard, and a mouse. The total cost of these parts is $600 per computer. If the firm sells each computer for $625, then how many hours a day should the firm employ Pam to maximize its net benefit from her employment?((3 hours))
A firm pays Pam $40 per hour to assemble personal computers. Each day, Pam can assemble 4 computers if she works 1 hour, 7 computers if she works 2 hours, 9 computers if she works 3 hours, and 10 computers if she works 4 hours. Pam cannot work more than 4 hours day. Each computer consists of a motherboard, a hard drive, a case, a monitor, a keyboard, and a mouse. The total cost of these parts is $600 per computer. If the firm sells each computer for $650, then how many hours a day should the firm employ Pam to maximize its net benefit from her employment?((4 hours))
If Jane works for 6 hours she can rent out 9 apartments, and if she works for 7 hours she can rent out 12 apartments. The marginal benefit of Jane's 7th hour of work equals:((3 apartments))
The following accompanying table shows the relationship between the speed of a computer's CPU and its benefits and costs. Assume that all other features of the computer are the same (that is, CPU speed is the only source of variation), and only the CPU speeds listed below are available for purchase. The marginal benefit of upgrading from a 2.0GHz computer to a 2.5GHz computer is:(($400.00 ))
The following accompanying table shows the relationship between the speed of a computer's CPU and its benefits and costs. Assume that all other features of the computer are the same (that is, CPU speed is the only source of variation), and only the CPU speeds listed below are available for purchase. The total benefit of a 3.0GHz computer is:(($1,700.00 ))
The following accompanying table shows the relationship between the speed of a computer's CPU and its benefits and costs. Assume that all other features of the computer are the same (that is, CPU speed is the only source of variation), and only the CPU speeds listed below are available for purchase. The total cost of a 2.5GHz computer is:(($1,000.00 ))
The following accompanying table shows the relationship between the speed of a computer's CPU and its benefits and costs. Assume that all other features of the computer are the same (that is, CPU speed is the only source of variation), and only the CPU speeds listed below are available for purchase. The marginal cost of upgrading from a 2.5GHz to 3.0GHz computer is:(($200.00 ))
The following accompanying table shows the relationship between the speed of a computer's CPU and its benefits and costs. Assume that all other features of the computer are the same (that is, CPU speed is the only source of variation), and only the CPU speeds listed below are available for purchase. Application of the Cost-Benefit Principle would lead one to purchase a ________ computer.((3.0GHz))
The following accompanying table shows the relationship between the speed of a computer's CPU and its benefits and costs. Assume that all other features of the computer are the same (that is, CPU speed is the only source of variation), and only the CPU speeds listed below are available for purchase. Choosing a 3.5GHz computer would be irrational because:((its marginal benefit is less than its marginal cost.))
Jack has a ticket to see the Foo Fighters for which he paid $30 yesterday. He takes an unpaid day off from work to get ready for the concert. When he arrives at the concert, five different people offer him $70 for his ticket. Jack decides to keep his ticket. At the time he makes this decision, his opportunity cost of seeing the Foo Fighters is:(($70.00 ))
Jody has purchased a non-refundable $75 ticket to attend a Miley Cyrus concert on Friday night. Subsequently, she is asked to go to out dinner at no expense to her. If she uses cost-benefit analysis to choose between going to the concert and going out to dinner, the opportunity cost of going out to dinner should include:((only the entertainment value of the concert.))
You won a free ticket to see the latest Star Wars movie this Friday night (which you can costlessly resell for its face value of $15). Your favorite band is also performing on Friday and is your only alternative activity. Friday is your last chance to see either the movie or the band. Tickets to see your favorite band cost $30, and on any given day, you would be willing to pay as much as $50 for a ticket. Based on this information, what is your opportunity cost of going to see the Star Wars movie on Friday?(($35 ))
You want to buy a TV that regularly costs $250. You can either buy the TV from a nearby store or from a store that's downtown. Relative to going to the nearby store, driving downtown involves additional time and gas. The downtown store, however, has a 10 percent off sale this week. Last week you drove downtown to save $20 on some concert tickets, a 15 percent savings. Should you drive downtown to buy the TV?((Yes, because you will save more than $20.))
The accompanying table below shows the relationship between the number of times you get your car washed each month and your total monthly benefit from car washes. Each car wash costs $15. What's the average benefit of 4 car washes per month?(($14 ))
The accompanying table below shows the relationship between the number of times you get your car washed each month and your total monthly benefit from car washes. Each car wash costs $15. What is the marginal cost of the 5th car wash each month?(($15 ))
The accompanying table below shows the relationship between the number of times you get your car washed each month and your total monthly benefit from car washes. Each car wash costs $15. What is the marginal benefit of the 3rd car wash each month?(($12 ))
The accompanying table below shows the relationship between the number of times you get your car washed each month and your total monthly benefit from car washes. Each car wash costs $15.((2))
Frank owns an apple farm and plans to spend 4 hours today picking apples. The number of apples he can pick per hour depends on the total number of hours he spends working in either the east orchard or the west orchard in the manner shown in the accompanying table below. If Frank spends 2 hours picking apples in the east orchard and 2 hours picking apples in the west orchard, how many apples in total will Frank be able to pick today?((84))
Frank owns an apple farm and plans to spend 4 hours today picking apples. The number of apples he can pick per hour depends on the total number of hours he spends working in either the east orchard or the west orchard in the manner shown in the accompanying table below. What is the opportunity cost to Frank of spending an additional hour picking apples in the East orchard?((10 apples))
Frank owns an apple farm and plans to spend 4 hours today picking apples. The number of apples he can pick per hour depends on the total number of hours he spends working in either the east orchard or the west orchard in the manner shown in the accompanying table below. What is the marginal benefit to Frank of the 2nd hour he spends picking in the east orchard?((24 apples))
Frank owns an apple farm and plans to spend 4 hours today picking apples. The number of apples he can pick per hour depends on the total number of hours he spends working in either the east orchard or the west orchard in the manner shown in the accompanying table below. How should Frank divide his time between the east and the west orchard?((He should spend 3 hours in the east orchard and 1 hour in the west orchard.))
Positive economic principles are those that:((predict how people will behave.))
One thing that distinguishes normative economic principles from positive economic principles is that:((normative principles tell us how people should behave, and positive principles tell us how people will behave.))
Normative economic principles are concerned with how people ________ make decisions while positive economic principles are concerned with how people ________ make decisions.((should; do))
An editorial in the paper argues that a person only should be allowed to attend school if the marginal cost of educating that person is less than the marginal benefit of educating that person. The writer's reasoning is an application of:((normative economics.))
The Incentive Principle states that a person:((is more likely to take an action if its benefit increases.))
The Incentive Principle is an example of:((a positive economic principle.))
According to the Incentive Principle, people will be less likely to smoke if the government:((increases taxes on cigarettes, effectively raising the price.))
According to the Incentive Principle:((people tend to do more of something when its benefits are greater.))
Microeconomics differs from macroeconomics in that microeconomics focuses on:((the choices made by individuals and the implications of those choices.))
Macroeconomics differs from microeconomics in that macroeconomics focuses on:((the performance of national economies and ways to improve that performance.))
The study of individual choice and its implications for the behavior of prices and quantities in individual markets is:((microeconomics.))
Which branch of economics is most likely to study differences in countries' growth rates?((macroeconomics))
Which of the following would not be studied in microeconomics?((Whether the federal budget should be balanced.))
Which of the following would not be studied in macroeconomics?((How a sharp increase in gasoline prices is likely to affect SUV sales.))
By convention, there are two major divisions of economics, called:((microeconomics and macroeconomics.))
A study that deals with the salaries of university professors would be considered:((microeconomics))
In deciding how many guitars to buy for his shop before the Christmas season, Mark is making a(n ________ decision.((microeconomic))
A study of the impact of various government policies on economic growth would be considered:((macroeconomics.))
The field of economics that would be most concerned with a recent fall in interest rates is:((macroeconomics.))
An economic naturalist is someone who:((applies economic insights to understand everyday life.))
With ATMs, it is possible to retrieve cash from the bank at any time. One hundred years ago, one could only get cash from the bank during business hours, say, 9 am to 3 pm. The present availability of 24-hour service has arisen because:((the cost of providing 24-hour service is much lower today.))
The number of U.S. households with access to the Internet has grown is growing rapidly. Compared to 50 years ago, one would predict that when considering a major purchase, people today will gather:((more information because the Internet has lowered the cost of gathering information.))
Every time you go to the grocery store, you try to wait in the shortest line. But the lines always seem to be roughly the same length. Why?((Other people are trying to choose the shortest line too.))
Suppose there are two parallel highways between two cities with approximately equal traffic. What would you expect to happen if the state began charging tolls to drive on one of those highways?((More drivers would drive on the non-toll road, making the toll road less congested.))
During times of high unemployment, colleges often observe an increase in enrollment even if tuition remains unchanged. Why?((The opportunity cost of attending college is lower when unemployment is high.))
The last time gas prices increased drastically, sales of large Sport Utility Vehicles (SUVs fell. Why?((Higher gas prices increased the cost of driving a SUV.))
Due to the fact that Curly used his frequent flyer miles to fly to visit Moe, Curly told Moe that it didn't cost him anything to visit. Is Curly correct?((No, because Curly could have used his frequent flyer miles to go somewhere else instead.))
Pat can either drive to work, which takes half an hour and uses $1.50 worth of gas, or take the bus, which takes an hour and costs $1.00. How should Pat get to work?((Pat should drive if saving half an hour is worth $0.50 or more.))
You have two options for how to spend the afternoon. You can either go see a movie with your roommate or work as a tutor for the Math Department. From experience, you know that going to see a movie gives you $20 worth of enjoyment, and with your student discount, a movie ticket only costs $12. If you spend the afternoon working as a math tutor, you will get paid $45. On a typical day, you wouldn't be willing to spend the afternoon working as a math tutor for less than $35. What is your opportunity cost of seeing a movie this afternoon?(($22 ))
You have two options for how to spend the afternoon. You can either go see a movie with your roommate or work as a tutor for the Math Department. From experience, you know that going to see a movie gives you $20 worth of enjoyment, and with your student discount, a movie ticket only costs $12. If you spend the afternoon working as a math tutor, you will get paid $45. On a typical day, you wouldn't be willing to spend the afternoon working as a math tutor for less than $35. What is your economic surplus from working as a math tutor instead of going to the movies?(($2 ))
You have two options for how to spend the afternoon. You can either go see a movie with your roommate or work as a tutor for the Math Department. From experience, you know that going to see a movie gives you $20 worth of enjoyment, and with your student discount, a movie ticket only costs $12. If you spend the afternoon working as a math tutor, you will get paid $45. On a typical day, you wouldn't be willing to spend the afternoon working as a math tutor for less than $35. Should you go see a movie or work as a math tutor?((You should work as a math tutor.))
If a country's economic decisions are made by an individual or small number of individuals, then it has a:((centralised economy.))
A good example of central planning at work in the U.S. is:((New York City's rent control program.))
The entire group of buyers and sellers of a particular good or service makes up:((a market.))
Suppose you bought three tickets to a concert in advance at the University ticket window. At the last minute one friend cancelled, so you could use only two of the tickets. You sold the third ticket just outside the entrance to the concert for more than the price you had originally paid. Which transaction occurred in a market?((Both the purchase at the University ticket window and the sale at the concert entrance were market transactions.))
Suppose you camped out in front of an electronics store to be one of the 200 lucky people able to purchase the latest gaming system. You bought the system for $350. Two weeks later you see that the same system can be sold on e-Bay for $600, so you sell your system. Your market role was as a:((consumer at the electronics store and a seller on e-Bay.))
To understand how the price of a good is determined in a free market, one must account for the desires of:((buyers and sellers.))
Buyers and sellers of a particular good make up the:((market for the good.))
All else constant, consumers will purchase more of a good as the price falls. This statement reflects the behavior underlying:((the demand curve.))
The demand curve illustrates the fact that consumers tend to purchase:((more of a good as its price falls.))
Which of the following is NOT true of a demand curve?((It reflects sellers' reservations prices.))
A demand curve is ________ sloping because ________.((downward; fewer people are willing to buy an item at higher prices))
As coffee becomes more expensive, Joe starts drinking tea instead of coffee. This is called:((the substitution effect of a price change.))
Suppose that as the price of apples rises, people switch from eating apples to eating oranges. This is known as:((the substitution effect of a price change.))
You can spend $10 for lunch and you would like to purchase two cheeseburgers. When you get to the restaurant, you find out the price for cheeseburger has increased from $5 to $6, so you decide to purchase just one cheeseburger. This is best described as:((the income effect of a price change.))
After the price of Revlon nail polish increased, Jen stopped buying Revlon nail polish and started buying a cheaper brand of nail polish instead. This is called:((the substitution effect of a price change.))
When the price of a good changes, the amount of that good that buyers wish to buy changes:((because of both the substitution and the income effects.))
The buyer's reservation price for a particular good or service is the:((largest price the buyer would be willing to pay for it.))
Shelly purchases a leather purse for $400. One can infer that:((her reservation price was at least $400.))
Gertie saw a pair of jeans that she was willing to buy for $35. The price tag, though, said they were $29.99. Therefore:((Gertie should buy the jeans because the price is less than her reservation price.))
One reason the demand curve slopes ________ is that as prices fall ________.((downward; more people find that the price is now less than their reservation price))
When a slice of pizza at the student union sold for $2, Moe did not purchase any. When the price fell to $1.75, Moe purchased a slice each day for lunch. Thus, we can infer that Moe's reservation price for a slice of pizza is:((at least $1.75 but less than $2.))
The quantity that sellers wish to sell tends to ________ as price increases, and so the supply curve is ________ sloping.((increase; upward))
The supply curve illustrates that firms:((increase the quantity supplied of a good when its price rises.))
As the price of a good rises:((more firms can cover their opportunity cost of producing the good.))
Jessica's marginal cost for producing a pitcher of lemonade is $0.25. Therefore, $0.25 is her:((reservation price.))
Suppose that the market price for hot dogs sold by street vendors has just risen from $4.50 to $5.00, and that in response Curly has now begun operating a hot dog cart. We can assume that Curly's reservation price for hot dogs is:((greater than $4.50 but no more than $5.00.))
A seller's reservation price is generally equal to:((the seller's opportunity cost of producing an additional unit.))
A seller's reservation price is generally equal to:((the seller's marginal cost.))
Which of the following is NOT a characteristic of a market in equilibrium?((Neither buyers nor sellers want the price to change.))
When a market is in equilibrium:((there is neither excess demand nor excess supply.))
Equilibrium price and quantity are determined by:((both supply and demand.))
Excess demand occurs:((when price is below the equilibrium price.))
If price is above the equilibrium price, then there will be:((excess supply.))
The price of bananas will increase in response to:((an excess demand for bananas.))
If there is an excess supply of sport utility vehicles, then:((quantity supplied is greater than quantity demanded.))
Refer to the accompanying figure. The equilibrium price is ________, and the equilibrium quantity is ________.(($6; 4))
Refer to the accompanying figure. At a price of $9, there will be:((an excess supply of 5 units.))
Refer to the accompanying figure. At a price of $3, there will be:((an excess demand of 5 units.))
Refer to the accompanying figure. If the price is $4 today and there is no change in either supply or demand, one would expect the price in the future to be:((greater than $4.))
When the current price of a good is below the equilibrium price:((buyers have an incentive to offer to pay sellers more than the current price.))
In a free market, if the price of a good is below the equilibrium price, then;((buyers, hoping to ensure they acquire the good, will bid the price higher.))
In a free market, if the price of a good is above the equilibrium price, then;((sellers, dissatisfied with growing inventories, will lower their prices.))
Which of following is NOT true of the equilibrium price?((It is fair in the sense that everyone can afford basic goods and services.))
When a market is not in equilibrium:((the economic motives of sellers and buyers will move the market to its equilibrium.))
Suppose that when the price of oranges is $3 per pound, the quantity demanded is 4.7 tons per day and the quantity supplied is 3.9 tons. In this case:((excess demand will lead the price of oranges to rise.))
Suppose that when the price of broccoli is $4 per pound, buyers wish to buy 500 pounds per day and sellers wish to sell 800 pounds per day. In this case:((excess supply will lead the price of broccoli to fall.))
Suppose you bought a concert ticket from Ticketmaster for $50, but when you get to the concert, there are a large number of people waiting outside who offer to pay you more than $50 for your ticket. What is probably true?((There is an excess demand for tickets at the Ticketmaster price.))
You have noticed that there is a persistent shortage of teachers in an inner-city school district in your state. Based on this observation, you suspect that:((the wage for teachers in that district is lower than the equilibrium wage.))
Suppose the market demand curve is given by Qd = 80 - 10P, and the market supply curve is given by Qs = 10 + 15P. What is the equilibrium price and quantity?((P* = $2.80 and Q* = 52))
Suppose you drive a car that gets good gas mileage, and you notice that more and more people are driving gas-guzzling cars. Their increased demand for gas:((is likely to cause the price you pay for gas to increase.))
Refer to the accompanying figure. The equilibrium price is ________, and the equilibrium quantity is ________.(($35; 20))
Refer to the accompanying figure. If the current market price were $20:((there would be an excess demand of 25 units.))
Refer to the accompanying figure. Suppose all the sellers in this market started out charging a price of $45 per unit. What is the most likely result?((They would lower their prices because at $45 there would be excess supply.))
Refer to the accompanying figure. If the government imposed a price ceiling of $40, what would happen in this market?((The price ceiling would have no effect.))
Refer to the figure below. There would be an excess supply of 25 at a price of ________.(($50 ))
The tendency of markets to automatically gravitate toward equilibrium is an application of the:((Incentive Principl.))
Refer to the given table. Suppose the columns in this table reflect demand and supply. If the current market price is $50, then you would expect:((the market price to rise.))
Which of the following is NOT a characteristic of rent controls?((Greater availability of apartments))
Suppose one knows two facts: first, the market for prescription drugs experiences chronic shortages and second, the government sets the price for prescription drugs. One can conclude that the government has:((set the price below the equilibrium price.))
A price ceiling that is set above the equilibrium price:((will lead to a black market.))
In a market in which the government has set a price ceiling below the equilibrium price:((a black market might develop.))
According to the textbook, government price controls fail because:((legislation cannot alter basic economic incentives.))
A movement along a demand curve from one price-quantity combination to another is called a:((change in quantity demanded.))
As the price of personal computers continues to fall, demand increases. This headline is inaccurate because:((a falling price of personal computers increases the quantity demanded, not demand.))
An increase in the quantity of tea demanded occurs if:((the price of the tea falls.))
If the demand for a good decreases as income decreases, then the good is a(n):((normal good.))
If the demand for gadgets increases as a result of a decrease in the price of widgets, the widgets and gadgets are:((complementary goods.))
It is likely that for most people:((coffee and tea are substitutes.))
Office workers and word processing programs are complements if:((a decrease in the wage paid to office workers leads to an increase in the demand for word processing programs.))
What might cause a demand curve to shift to the right?((An increase in the price of a substitute))
If the demand for olives falls when the price of cheese falls, then we know that cheese and olives are:((substitutes.))
If the demand for cucumbers falls when the price of tomatoes rises, then we know that tomatoes and cucumbers are:((complements.))
If the demand for steak increases as income increases, then steak is a(n):((normal good.))
If an increase in the price of good X leads to a decrease in the demand for good Y, then:((good X and good Y are complements.))
Two goods are complements if:((an increase in the price of one good leads to a decrease in demand for the other.))
A decrease in the price of pizza will lead to a(n):((increase in the quantity of pizza demanded.))
If the demand for computers increases as consumers' incomes rise, then computers are:((a normal good.))
If an increase in income leads to a decrease in the demand for ground beef, then ground beef is a(n):((inferior good.))
If the demand curve for bologna shifts to the right as income falls then bologna is a(n):((inferior good.))
Suppose sport utility vehicles get poor gas mileage compared to other available cars. If the price of gasoline increases, then one would expect:((the demand for sport utility vehicles to decrease.))
Suppose the residents of Metropolis travel to work either by bus or train. If the price of train tickets increases, then:((the demand for bus tickets will increase.))
If the price of doughnuts decreases, then one would expect the:((quantity of doughnuts supplied to decrease.))
Suppose that the price of doughnuts decreases. Given that doughnut holes are a by-product of producing doughnuts, one would expect:((the supply of doughnut holes to decrease.))
For two goods, X and Y, to be classified as substitutes, it must be the case that:((when the price of X rises, the demand for Y increases.))
At the beginning of the fall semester, college towns experience large increases in their populations, causing a(n):((increase in the demand for apartments.))
Suppose one observes that when the price of peanut butter increases, the demand for jelly increases. One should conclude that:((peanut butter and jelly are substitutes.))
Refer to the accompanying figure. Suppose the solid line shows the current demand curve for coffee. In response to an announcement that much of next year's coffee crop has been destroyed by a storm in Brazil, you should expect:((the demand curve to shift to D(in anticipation of higher future prices.))
Refer to the accompanying figure. Suppose the solid line shows the current demand for coffee. In response to a news story explaining that coffee causes heart disease, you should expect:((the demand curve to shift to D(because some people will stop drinking coffee.))
Refer to the accompanying figure. Suppose the solid line shows the current demand for coffee. In response to news that next year's coffee harvest will be extremely good due to favorable weather conditions, you should expect:((the demand curve to shift to D(in anticipation of lower future prices.))
Refer to the accompanying figure. Suppose the solid line shows the demand for coffee. If coffee and tea are substitutes, and the price of tea falls, then you would expect:((the demand curve to shift to D(A).))
Suppose that a disease that affects people who consume beef has been discovered in the United States. One likely result is:((a decrease in demand for beef.))
Assume consumers eat either rice or pasta for dinner every night. If the price of rice increases, then one would expect to see:((an increase in the demand for pasta.))
Suppose that recent studies conclude that high-fiber diets do not reduce the risk of developing colon cancer as was previously thought. The likely result will be that the:((demand for high-fiber foods will decrease.))
Which of the following is likely to lead to a decrease in the demand for tennis balls?((An increase in the price of tennis racquets.))
If fast food is an inferior good then:((the demand for fast food will fall as income rises.))
Refer to the accompanying figure. Moving from demand curve D1 to demand curve D2 illustrates a(n):((increase in demand.))
Refer to the accompanying figure. Moving from demand curve D2 to demand curve D1 could be caused by a(n):((increase in the price of a complement.))
Refer to the accompanying figure. Moving from demand curve D1 to demand curve D2 could be caused by a(n):((increase in the price of a close substitute.))
All else equal, a decrease in the demand for oranges will lead to a(n ________ in equilibrium price and a(n ________ in equilibrium quantity.((decrease; decrease))
As the price of cookies increases, firms that produce cookies will:((increase the quantity of cookies supplied.))
Which of the following would cause an increase in quantity of wheat supplied?((The price farmers receive for their wheat rises))
If the price of rubber (an input to the production of tires increases:((the supply of tires will decrease.))
As the price of flour (an input in the production of cookies increases, firms that produce cookies will:((decrease the supply of cookies.))
Suppose that the technology used to manufacture laptops has improved. The likely result would be:((an increase in supply of laptops.))
Which of the following factors will lead to a decrease in the current supply of a good?((A belief that the price of a good or service will go up in the future))
What might cause a decrease in current supply of a product?((New information that leads sellers to believe that the product's price will rise in the future))
Refer to the accompanying figure. Suppose the solid line represents the current supply of Star Wars action figures. If retailers learn that a new Star Wars movie will be released in several months, this news is likely to cause:((current supply to shift to S(in anticipation of higher future prices.))
Refer to the accompanying figure. Suppose the solid line represents the current supply of Star Wars action figures. If the price of the plastic used to make action figures rises, current supply will:((shift to S(A).))
Refer to the accompanying figure, which shows the market for cups of coffee. At the original market equilibrium:((40 cups are sold per hour at a price of $2.00 each.))
Refer to the accompanying figure, which shows the market for cups of coffee. What might cause a shift from the original demand curve to the new demand curve?((An increase in consumers' tastes for coffee.))
Refer to the accompanying figure, which shows the market for cups of coffee. What might cause a shift from the original supply curve to the new supply curve?((A new technology that reduces amount of coffee beans needed to make a good cup of coffee))
Refer to the accompanying figure, which shows the market for cups of coffee. If all buyers' reservation prices increase by $1.00, then the equilibrium price of coffee would:((increase by less than $1.00.))
Refer to the accompanying figure, which shows the market for cups of coffee. Consider the original supply and the original demand curve. If the government imposes a price ceiling of $1.00 on a cup of coffee, then there would be:((an excess demand for coffee.))
Refer to the accompanying figure. An increase in demand is represented by a shift from:((curve C to curve D.))
Refer to the accompanying figure. A decrease in demand is represented by a shift from:((curve D to curve C.))
Refer to the accompanying figure. An increase in supply is represented by a shift from:((curve A to curve B.))
Refer to the accompanying figure. A decrease in supply is represented by a shift from:((curve B to curve A.))
Refer to the given table. Relative to column A, column B represents:((an increase in demand.))
Refer to the given table. Relative to column C, column D represents:((a decrease in supply.))
Refer to the given table. Suppose the columns in this table reflect demand and supply. At a price of $30:((there will be an excess demand of 45 units.))
Refer to the given table. Suppose the columns in this table reflect demand and supply. At a price of $50:((there will be an excess supply of 5 units.))
Refer to the given table. The equilibrium price in this market is:((between $40 and $50.))
An increase in the demand for GM automobiles results in:((an increase in the quantity supplied of GM automobiles.))
Which of the following is NOT a determinant of the demand for gasoline?((The supply of gasoline))
When the supply of a good decreases, there will be a(n):((decrease in the quantity demanded.))
When the supply curve shifts to the left and there is no change in demand:((the equilibrium price will rise.))
If the supply curve and the demand curve both shift to the left, then the new equilibrium:((quantity will be lower, but the direction of the price change is uncertain.))
If supply increases, then:((the quantity demanded will increase.))
Suppose demand decreases, but there is no change in supply. As the market reaches its new equilibrium:((excess supply will lead the price to fall.))
Suppose supply decreases, but there is no change in demand. As the market reaches its new equilibrium:((excess demand will lead the price to rise.))
When the demand curve shifts to the right and supply doesn't change:((equilibrium quantity will rise.))
A decrease in both the equilibrium price and the equilibrium quantity of rice is best explained by:((a decrease in the demand for rice.))
An increase in both the equilibrium price and the equilibrium quantity of DVD players is best explained by:((an increase in the demand for DVD players.))
Suppose we observe an increase in both the equilibrium price and quantity of bread. This is best explained by:((a decrease in the price of butter, a complement to bread.))
Suppose that the equilibrium price of T-shirts increases and the equilibrium quantity of T-shirts decreases. This is best explained by:((a decrease in the supply of T-shirts.))
Suppose that the equilibrium price of apples decreases and the equilibrium quantity of apples increases. This is best explained by:((an increase in the supply of apples.))
Suppose you observe a decrease in the equilibrium price and quantity of corn. Of the options listed below, this is best explained by:((a fall in consumer income assuming corn is a normal good.))
Suppose we observe a decrease in the equilibrium price of tuna and an increase in the equilibrium quantity of tuna. This is best explained by:((a decrease in the cost of fuel used by tuna fishing boats.))
Suppose you observe an increase in the equilibrium price of coffee and a decrease in the equilibrium quantity of coffee. Of the options listed below, this is most consistent with:((an increase in the cost of producing coffee.))
If pencils and paper are complements for most consumers, then if the price of paper increases, you would expect:((the equilibrium price and quantity of pencils to fall.))
Refer to the accompanying figure. Assume demand remains unchanged at D1. If supply shifts from S1 to S2, then the equilibrium price will ________ and the equilibrium quantity will ________.((fall; rise))
Refer to the accompanying figure. Assume demand remains unchanged at D1. If supply shifts from S2 to S1, then the equilibrium price will ________ and the equilibrium quantity will ________.((rise; fall))
Refer to the accompanying figure. If demand shifts from D1 to D2, and at the same time, supply shifts from S1 to S2, then according to the figure:((the equilibrium quantity will increase and the equilibrium price will increase.))
If demand increases and supply decreases, the change in the equilibrium price will be ________, and the change in the equilibrium quantity will be ________.((positive; uncertain))
If supply and demand both increase, the new equilibrium price will be ________ and the new equilibrium quantity will be ________.((uncertain; higher))
If supply and demand both decrease, the new equilibrium price will be ________ and the new equilibrium quantity will be ________.((uncertain; lower))
If supply increases and demand decreases, the new equilibrium price will be ________ and the new equilibrium quantity will be ________.((lower; uncertain))
Suppose the equilibrium price and quantity of ketchup fall. The most likely explanation for these changes is:((a decrease in the demand for ketchup.))
Suppose that the equilibrium price of pickles falls while the equilibrium quantity rises. The most likely explanation for these changes is:((an increase in the supply of pickles.))
Suppose that the equilibrium price of french fries rises while the equilibrium quantity falls. The most likely explanation for these changes is:((a decrease in the supply of french fries.))
Assume the demand for coffee increases and the supply of coffee decreases. Which of the following outcomes is certain to occur?((The equilibrium price of coffee will rise.))
Assume the demand for sugar decreases and the supply of sugar increases. Which of the following outcomes is certain to occur?((The equilibrium price of sugar will fall.))
Assume both the demand for beef and the supply of beef decrease. Which of the following outcomes is certain to occur?((The equilibrium quantity of beef will fall.))
Assume both the demand for bagels and the supply of bagels increase. Which of the following outcomes is certain to occur?((The equilibrium quantity of bagels will rise.))
Suppose a new study highlights the health benefits of eating bacon. At the same time, suppose the cost of producing bacon falls. Given these changes, you should expect to see:((an increase in the equilibrium quantity of bacon, but it's hard to say what will happen to the equilibrium price.))
Refer to the accompanying figure. Assume the market is originally at point W. Movement to point X is the result of:((an increase in demand and an increase in quantity supplied.))
Refer to the accompanying figure. Assume the market is originally at point W. Movement to point Y is the result of:((an increase in supply and an increase in quantity demanded.))
Refer to the accompanying figure. Assume the market is originally at point W. Movement to point Z is a combination of:((an increase in supply and an increase in demand.))
Suppose that both the supply of iPads and the demand for iPads decrease. One can predict that the:((equilibrium quantity will fall, but the change in equilibrium price is uncertain.))
Suppose rice is a normal good. If consumers' incomes fall, and a new technology is introduced that lowers the marginal cost of producing rice, then the equilibrium:((price of rice will fall, but we cannot say for sure what will happen to the equilibrium quantity.))
Suppose Bianca buys a used a textbook from Sebastian for $55. If Bianca's surplus from this transaction was $10, we can infer that:((Bianca's reservation price was $65.))
Suppose that Tom bought a bike from Helen for $195. If Helen's reservation price was $185, and Tom's reservation price was $215, the seller's surplus from this transaction was:(($10 ))
Suppose that Tom bought a bike from Helen for $195. If Helen's reservation price was $185, and Tom's reservation price was $215, the buyer's surplus from this transaction was:(($20 ))
Suppose that Tom bought a bike from Helen for $195. If Helen's reservation price was $185, and Tom's reservation price was $215, the total economic surplus from this transaction was:(($30 ))
An outcome is socially optimal if it:((maximizes total economic surplus.))
Efficiency occurs if the:((socially optimal quantity of goods and services is being produced.))
Efficiency is an important social goal because:((movements toward economic efficiency make the total economic pie larger.))
Assume that Joe is willing to produce a hamburger for $1, and Mary is willing to pay $3 for a hamburger. Which of the following is true?((Joe and Mary can make a mutually beneficial exchange.))
When two people agree to a price in a negotiation, we can assume that:((both parties will benefit.))
If there are no unexploited opportunities for individuals in a particular market, then one can conclude that:((the market is in equilibrium.))
The situation described in the book as "smart for one, dumb for all" occurs when:((individuals act rationally, but there are still unexploited opportunities for society as a whole.))
If the local slaughterhouse gives off an unpleasant stench, then the equilibrium quantity of meat will be ________ the quantity that maximizes total economic surplus.((higher than))
If the equilibrium quantity of a good is also the socially optimal quantity, then:((total economic surplus has been maximized.))
If the production of oranges reduces global warming, then the equilibrium quantity of oranges will be ________ the socially optimal quantity.((lower than))
A market equilibrium might not maximize total economic surplus because:((sometimes goods entail costs and benefits that do not fall on buyers and sellers.))
Everyone in the neighborhood has been complaining about the deteriorating condition of the park, but nobody has cleaned it up. Why not?((No single person's benefit from cleaning the park exceeds that person's cost of cleaning it.))
The Equilibrium Principle asserts that in a market equilibrium:((no unexploited opportunities exist for individuals.))
A market equilibrium:((might not maximize total economic surplus.))
A market equilibrium:((leaves no unexploited opportunities for individuals.))
The market equilibrium quantity:((is sometimes the socially optimal quantity.))
Suppose quantity demanded is given by Qd = 100 - P, and quantity supplied is given by Qs = 20 + 3P. In this case, equilibrium price, P*, and equilibrium quantity, Q*, are as follows:((P*= 20, Q*= 80))
Economic theory assumes that a firm's goal is to:((maximize its economic profit.))
Explicit costs:((measure the payments made to the firm's factors of production.))
Which of the following is NOT an example of an explicit cost?((The income the owner could have earned in his or her next best employment opportunity.))
Which of the following statements about explicit costs is true?((They appear on the firm's balance sheet.))
Which of the following statements about implicit costs is true?((They measure the forgone opportunities of the firm's owners.))
Accounting profit is equal to:((total revenue minus explicit costs.))
An example of an implicit cost is:((the value of a spare bedroom turned into a home office.))
If you were to start your own business, your implicit costs would include the:((opportunity cost of the time you spend working at the business.))
Suppose you own a small business. Last month, your total revenue was $6,000. In addition, you paid: $1,000 in monthly rent for office space, $200 in monthly rent for equipment, $3,000 to your workers in wages for the month, and $1,000 for the supplies you used that month. If you correctly determine that your economic profit last month was negative $200, then it must be true that:((your implicit costs are $1,000 per month.))
Curly told Larry about his new business venture: Curly pays Acme International $1,000 per month for supplies, works out of his apartment on his own computer and earns a monthly revenue of $1,500. Should Larry quit his job and do what Curly is doing?((Not if Larry is earning more than $500 per month at his current job.))
If a firm is earning zero economic profit, then:((the firm's accounting profit is equal to the firm's implicit costs.))
Which of the following would not be included in the calculation of accounting profit?((The salary the owner could have earned working elsewhere.))
Economic profit is equal to:((total revenue minus the sum of explicit and implicit costs.))
Accounting profit minus implicit costs equals:((economic profit.))
Which of the following statements is true?((Accounting profit is greater than or equal to economic profit.))
A firm earns a normal profit when its:((economic profit is zero.))
Last year Christine worked as a consultant. She hired an administrative assistant for $15,000 per year and rented office space (utilities includefor $3,000 per month. Her total revenue for the year was $100,000. If Christine hadn't worked as a consultant, she would have worked at a real estate firm earning $40,000 a year. Last year, Christine's explicit costs were ________, and her implicit costs were ________.(($51,000; $40,000))
Last year Christine worked as a consultant. She hired an administrative assistant for $15,000 per year and rented office space (utilities included)for $3,000 per month. Her total revenue for the year was $100,000. If Christine hadn't worked as a consultant, she would have worked at a real estate firm earning $40,000 a year.(($40,000 ))
Last year Christine worked as a consultant. She hired an administrative assistant for $15,000 per year and rented office space (utilities included)for $3,000 per month. Her total revenue for the year was $100,000. If Christine hadn't worked as a consultant, she would have worked at a real estate firm earning $40,000 a year.(($49,000; $9,000))
Last year Christine worked as a consultant. She hired an administrative assistant for $15,000 per year and rented office space (utilities includefor $3,000 per month. Her total revenue for the year was $100,000. If Christine hadn't worked as a consultant, she would have worked at a real estate firm earning $40,000 a year. For Christine to earn a normal profit as a consultant, her accounting profit would have to be ________.(($40,000 ))
Pat used to work as an aerobics instructor at the local gym earning $35,000 a year. Pat quit that job and started working as a personal trainer. Pat makes $50,000 in total annual revenue. Pat's only out-of-pocket costs are $12,000 per year for rent and utilities, $1,000 per year for advertising and $3,000 per year for equipment. Pat's explicit costs are ________, and Pat's implicit costs are ________.(($16,000; $35,000))
Pat used to work as an aerobics instructor at the local gym earning $35,000 a year. Pat quit that job and started working as a personal trainer. Pat makes $50,000 in total annual revenue. Pat's only out-of-pocket costs are $12,000 per year for rent and utilities, $1,000 per year for advertising and $3,000 per year for equipment. Pat's accounting profit is ________, and Pat's economic profit is ________.(($34,000; −$1,000))
Pat used to work as an aerobics instructor at the local gym earning $35,000 a year. Pat quit that job and started working as a personal trainer. Pat makes $50,000 in total annual revenue. Pat's only out-of-pocket costs are $12,000 per year for rent and utilities, $1,000 per year for advertising and $3,000 per year for equipment.(($35,000 ))
If a firm is earning zero economic profit, then its accounting profit will:((be positive.))
If the owners of a business are receiving total revenues just sufficient to cover all of their explicit and implicit costs, then they are:((earning a normal profit.))
Suppose Juliana owns a small business making handbags. Each month she makes 18 handbags, which she sells for $100 each. The materials used to make each handbag cost $50. In addition, Juliana uses a spare room in her house to make the handbags and store her supplies. If she were not using the spare room for her business, she would use it as a guest room, an option that Juliana would value at $250 per month. If Juliana weren't making handbags, she would work at Trader Joe's earning $800 per month. What is Juliana's accounting profit each month?(($900 ))
Suppose Juliana owns a small business making handbags. Each month she makes 18 handbags, which she sells for $100 each. The materials used to make each handbag cost $50. In addition, Juliana uses a spare room in her house to make the handbags and store her supplies. If she were not using the spare room for her business, she would use it as a guest room, an option that Juliana would value at $250 per month. If Juliana weren't making handbags, she would work at Trader Joe's earning $800 per month. What is Juliana's economic profit each month?((−$150))
Refer to the table below. An output level of 25 units, this firm's accounting profit is ________, and its economic profit is ________.((zero; −$8))
Refer to the table below. An output level of 15 units, this firm's accounting profit is ________, and its economic profit is ________.(($12; $6))
Suppose Lando Calrissian owns a smuggling business whose total revenue is $30,000 per month. The accompanying table shows Lando's monthly expenses. If Lando weren't a smuggler, he would earn $6,000 per month working for the Rebel Alliance. Apart from pay, Lando is indifferent between working as a smuggler and working for the Rebel Alliance. What is Lando's accounting profit each month?(($5,000 ))
Suppose Lando Calrissian owns a smuggling business whose total revenue is $30,000 per month. The accompanying table shows Lando's monthly expenses. If Lando weren't a smuggler, he would earn $6,000 per month working for the Rebel Alliance. Apart from pay, Lando is indifferent between working as a smuggler and working for the Rebel Alliance. In the long run, we would expect Lando to:((join the Rebel Alliance since his economic profit from smuggling is negative.))
Refer to the table below. At what output level or levels are this firm's owners doing as well as or better than they could do with the next best use of their resources?((10, 15, and 20 units))
Adam Smith's theory of the invisible hand posits the actions of independent, self-interested buyers and sellers will ________ lead to the most efficient allocation of resources.((often))
Adam Smith's theory of the invisible hand posits that the most efficient allocation of resources is often achieved by:((the actions of independent, self-interested buyers and sellers.))
Refer to the table below. Suppose all firms in this industry have identical costs to this firm and are producing 15 units of output. One can predict that:((new firms will enter the industry.))
The role that prices play in distributing scarce goods and services to those consumers who value them the most highly is known as the ________ function of price.((rationing))
The role that prices play in directing resources away from overcrowded markets and towards markets that are underserved is known as the ________ function of price.((allocative))
Which of the following is an example of the rationing function of price?((Bill Gates purchasing the Mona Lisa for $5 billion))
The allocative function of price is to:((direct resources away from markets that are overcrowded and toward markets that are underserved.))
Generally, ________ motivates firms to enter an industry, while ________ motivates firms to exit an industry.((economic profit; economic loss))
Suppose farmers in a given market can either grow soy beans or corn on their land. In addition, suppose an increase in the demand for corn causes the price of corn to increase. All else equal, an increase in the price of corn creates an incentive for farmers to:((switch away from growing soy beans and into growing corn.))
Suppose farmers in a given market can either grow soy beans or corn on their land. In addition, suppose an increase in the demand for corn causes the price of corn to increase. In the long run, this increase in the demand for corn is likely to ________ the price of soy beans.((lead to an increase in))
Suppose farmers in a given market can either grow soy beans or corn on their land. In addition, suppose an increase in the demand for corn causes the price of corn to increase. As a result of the increase in the price of corn, farmers who were already growing corn will earn an:((economic profit in the short run.))
If the firms in a market are earning an economic profit, then, in the long run, the market ________ curve will shift to the ________.((supply; right))
Suppose all firms in a perfectly competitive industry are earning an economic profit. One would expect that, over time, the number of firms in the industry will ________ and the market price will ________.((rise; fall))
If all firms in a perfectly competitive industry are earning a normal profit, then:((there is no incentive for firms to enter or exit the industry.))
If all firms in a perfectly competitive industry are experiencing economic losses, then:((some firms will exit the industry, until economic profit equals zero.))
Entry into a perfectly competitive industry occurs whenever:((economic profit is greater than zero.))
In an industry with free entry and exit, positive economic profit:((cannot be sustained indefinitely.))
In the long run, in a perfectly competitive industry:((economic profit and loss are driven to zero by entry and exit.))
Assume that all firms in this industry have identical cost curves, and that the market is perfectly competitive. The long-run equilibrium price in this industry is:(($10 ))
Assume that all firms in this industry have identical cost curves, and that the market is perfectly competitive. If the market supply curve is given by S3, then in the long run firms will:((exit the market, leading the market supply curve to shift back to S2.))
Assume that all firms in this industry have identical cost curves, and that the market is perfectly competitive. If the market supply curve is given by S1, then in the long run firms will:((enter the market, leading the market supply curve to shift out to S2.))
Assume that all firms in this industry have identical cost curves, and that the market is perfectly competitive. In the short run, firms in this market will shut down if the market price is:((less than $5))
Assume that all firms in this industry have identical cost curves, and that the market is perfectly competitive. The firm depicted in the graph on the right faces a demand curve that is:((horizontal at the market price.))
Assume that all firms in this industry have identical cost curves, and that the market is perfectly competitive. In the long run, there will be ________ firms in this market.((10))
In perfectly competitive markets, an implication of entry and exit in response to economic profit and loss is that:((firms will produce the quantity that minimizes average total costs in the long run.))
One difference between the long run and the short run in a perfectly competitive industry is that:((firms necessarily earn zero economic profit in the long run but may earn positive or negative economic profit in the short run.))
The figure below depicts the short-run market equilibrium in a perfectly competitive market and the cost curves for a representative firm in that market. Assume that all firms in this market have identical cost curves. The long-run market equilibrium quantity in this industry is:((500))
The figure below depicts the short-run market equilibrium in a perfectly competitive market and the cost curves for a representative firm in that market. Assume that all firms in this market have identical cost curves. Given that the current equilibrium price is $8, what will happen to the number of firms in this market in the long run?((The number of firms in the market will rise as firms enter the market in response to positive economic profit.))
The figure below depicts the short-run market equilibrium in a perfectly competitive market and the cost curves for a representative firm in that market. Assume that all firms in this market have identical cost curves. In the long run equilibrium in this market:((price will equal $5, and there will be 20 firms in the industry.))
The figure below depicts the short-run market equilibrium in a perfectly competitive market and the cost curves for a representative firm in that market. Assume that all firms in this market have identical cost curves. A starting assumption about this industry was that all of the firms in the market had identical cost curves. This assumption is:((realistic because any cost-saving innovation adopted by one firm will be quickly adopted by others.))
Which of the following best describes how a perfectly competitive industry would respond to a sudden increase in popularity of the product? The market demand curve would shift to the right, leading to:((a higher equilibrium price in the short run and entry into the market in the long run.))
One assumption of the perfectly competitive model is free entry and exit. This assumption most directly leads to the implication that:((positive economic profit is only possible in the short run.))
Free entry and exit of firms is a characteristic of:((perfectly competitive industries.))
The allocative function of price cannot operate unless there is:((both free entry and free exit.))
In a market with barriers to entry:((economic profit will not fall to zero in the long run.))
Assume that all firms in this industry have identical cost curves, and that the market is perfectly competitive. If S3 is the market supply curve, then in the short run, the profit-maximizing level of output for a single firm in this market is ________ gallons per week.((8000))
Assume that all firms in this industry have identical cost curves, and that the market is perfectly competitive. If the market supply curve is given by S3, then we would expect firms to:((exit the market in the long run.))
Assume that all firms in this industry have identical cost curves, and that the market is perfectly competitive. If S3 is the market supply curve, then each firm in this market will earn an economic loss of ________ per week.(($2,000 ))
Assume that all firms in this industry have identical cost curves, and that the market is perfectly competitive. If the market supply curve is given by S3, then what will happen to the market supply curve in the long run?((It will shift to S2.))
Assume that all firms in this industry have identical cost curves, and that the market is perfectly competitive. In the long run, the equilibrium price will be ________ per gallon, and each firm's profit-maximizing quantity will be ________ gallons per week.(($15; 300))
Assume that all firms in this industry have identical cost curves, and that the market is perfectly competitive. In the long run, how much profit will each firm in this industry earn each week?(($0 ))
According to the theory of the invisible hand, if buyers and sellers are free to pursue their own self-interest, the result often will be:((an efficient allocation of resources.))
E-commerce and an internet presence are important to many firms, requiring employees with specialized skills that are in short supply. The invisible hand solves the employment problem by:((giving workers an incentive to acquire the necessary skills on their own in order to receive higher wages.))
In a free market economy, the decisions of buyers and sellers are:((guided by prices.))
Some people have argued that the government should provide free medical care to everyone. Under this system:((prices will not ration medical care so some other rationing method will be used.))
If resources are misallocated in a perfectly competitive market, then, in the long run, profit opportunities will:((bring about a more efficient allocation of resources.))
Barriers to entry are forces that:((limit new firms from joining an industry.))
Economic rent is:((the difference between the payment made to the owner of a factor of production and the owner's reservation price.))
Mary Jane is willing to babysit for $6 an hour. Her neighbor has asked her to babysit for $8 an hour. Assuming Mary Jane accepts the offer:((her economic rent will be $2 per hour.))
Which of the following is a characteristic of economic rent?((It can never be negative.))
Angelina Jolie's economic rent from starring in a movie is equal to the difference between:((her final salary and the least she would be willing to accept to star in the movie.))
Superstar professional athletes can sustain their economic rents because:((they have unique talents that they can sell to the highest bidder.))
Professor Plum, who earns $100,000 per year, read in the paper today that the university pays its basketball coach one million dollars per year in exchange for his agreement to remain at the university for at least three more years. The coach earns more than Professor Plum because:((the coach is able to earn economic rent due to his unique talents.))
Unlike economic profit, economic rent:((may not be driven to zero by competition.))
Duke is a highly skilled negotiator who could work for many law firms. The law firm that hires Duke is able to collect twice as much revenue per hour of Duke's time than it can for any other negotiator in town. The increased revenue will:((all go to Duke because, if it didn't, another firm could hire Duke away.))
Factors of production are the most likely to earn economic rent when they:((cannot easily be duplicated.))
The supplier of a factor of production has a reservation price of $100. The purchaser of the factor of production has a reservation price of $200. If the factor of production is unique, then:((a transaction will occur, and the price paid for the factor of production will be $200.))
Consider a perfectly competitive industry in a long-run equilibrium. If a single firm in that industry discovers a significant cost-saving production technology, then:((the rest of the industry will quickly adopt the new technology.))
Suppose several United States software design companies compete with each other in a perfectly competitive environment. If one company decides to move some of its offices to a low-wage country in order to reduce operating costs, then:((other companies will have an incentive move to the low-wage country in order to remain competitive.))
A cost-saving innovation in a perfectly competitive industry will lead to:((economic profits for a few firms for a short time.))
The statement, "If a deal is too good to be true, then it probably is not true," is most closely related to which core economic principle?((The No-Cash-on-the-Table Principle))
The No-Cash-on-the-Table Principle states that there are:((never unexploited opportunities available to individuals in equilibrium.))
If the market supply curve does not capture all of the costs to society of producing an additional unit of good, then:((the market equilibrium will not be efficient.))
If the market demand curve does not capture all of the benefits to society of buying an additional unit of good, then:((the market equilibrium will not be efficient.))
The phrase "smart for one, but dumb for all" refers to the idea that the individual pursuit of self-interest:((doesn't always lead to an efficient outcome.))
Adam Smith believed that the individual pursuit of self-interest:((often promotes the broader interests of society.))
If it is possible to make a change that will help some people without harming others, then the situation is:((inefficient.))
A situation is efficient if it is:((not possible to find a transaction that will make at least one person better off without harming others.))
If there is excess demand in a market, then this suggests that:((there is an opportunity for mutually beneficial trades.))
Which of the following describes a surplus-enhancing transaction?((A person pays $10.00 to buy a scoop of ice cream at a baseball game.))
Suppose your economics professor has an extra copy of textbook that he or she would like to give to a student in the class. Which of the following schemes is the most likely to result in an efficient outcome?((Auctioning off the textbook to the highest bidder.))
Ingrid has been waiting for the show "Mamma Mia!" to come to town. When it finally does come, tickets cost $60. Ingrid's reservation price is $75. But when Ingrid tries to buy a ticket, they are sold out. Ingrid decides to try to buy a ticket from a scalper (a person who purchased extra tickets at the box office with the intent to resell them at a higher price). If Ingrid finds someone who is willing to sell her a ticket for $70, she should:((purchase the ticket because doing so will make her $5 better off.))
Ingrid has been waiting for the show "Mamma Mia!" to come to town. When it finally does come, tickets cost $60. Ingrid's reservation price is $75. But when Ingrid tries to buy a ticket, they are sold out. Suppose Steven was able to purchase a ticket at the box office for $60. Steven's reservation price for the ticket is $65. If Steven attends "Mamma Mia!" and Ingrid does not, then this situation is:((inefficient because Steven and Ingrid could have made a mutually beneficial trade.))
Suppose the market for coffee is in equilibrium at a price of $5 per pound. This means that:((potential consumers not buying coffee value it at less than $5 per pound.))
The figure below shows the supply and demand curves for oranges in Smallville. At the price of $4 per pound, sellers offer ________ pounds of oranges per day, and buyers want to purchase ________ pounds of oranges a day.((10; 30))
The figure below shows the supply and demand curves for oranges in Smallville. The marginal buyer values the tenth pound of oranges at ________.(($12 ))
The figure below shows the supply and demand curves for oranges in Smallville. At a price of $4 per pound there will be an excess ________ of ________ pounds of oranges per day.((demand; 20))
The figure below shows the supply and demand curves for oranges in Smallville. What is the marginal cost of producing the tenth pound of oranges?(($4 ))
The figure below shows the supply and demand curves for oranges in Smallville. When this market is in equilibrium, total economic surplus is ________ per day.(($160 ))
The figure below shows the supply and demand curves for jeans in Smallville. At the price of $60 per pair, sellers offer ________ pairs of jeans per day, and buyers wish to purchase ________ pairs of jeans a day.((24; 8))
The figure below shows the supply and demand curves for jeans in Smallville. At a price of $60 per pair, there will be an excess ________ of ________ pairs of jeans per day.((supply; 16))
The figure below shows the supply and demand curves for jeans in Smallville. Suppose jeans initially sell for $60 per pair. If the price of jeans falls to $40 per pair, then total economic surplus will increase by ________ per day.(($160 ))
The figure below shows the supply and demand curves for jeans in Smallville. The equilibrium price will NOT lead to the largest possible total economic surplus if:((the production of jeans generates air pollution.))
A market equilibrium is only efficient if:((all relevant costs and benefits are reflected in the market supply and demand curves.))
Suppose the production of cotton causes substantial environmental damage because the pesticides used by cotton farmers often make their way into nearby rivers and streams, and are very harmful to fish and other wildlife. Economists would consider the environmental damage that results from the production of cotton to be a(n):((relevant cost of production.))
Suppose the production of cotton causes substantial environmental damage because the pesticides used by cotton farmers often make their way into nearby rivers and streams, and are very harmful to fish and other wildlife. If cotton farmers do not have to pay for the environmental damage caused by the pesticides used to grow cotton, then the market equilibrium price will be ________ and the market equilibrium quantity will be ________.((inefficiently low; inefficiently high))
Which of the following statements best expresses why economic efficiency should be society's first goal?((Efficiency maximizes total economic surplus and thereby allows other goals to be more easily achieved.))
If the demand curve fails to capture all of the benefits of consumption, then the:((equilibrium price will be inefficiently low.))
Which of the following is NOT necessarily true in a market equilibrium?((Both rich and poor have adequate access to the good.))
Efficiency is an important goal because when markets are efficient:((there are more resources available to achieve other goals.))
A price ceiling that is set below the equilibrium price will result in:((a shortage of the good.))
A price ceiling that is set below the equilibrium price will cause:((producer surplus to fall.))
A price ceiling that is set above the equilibrium price will result in:((no change in total economic surplus.))
If an individual consumer is willing to pay $11 for one unit of a good but is able to purchase it for $7, then his or her consumer surplus from the purchase of that unit would be:(($4.00 ))
If an individual producer is willing to produce one unit of a good for $2.50 but is able to sell it for $7.50, then his or her producer surplus from the sale of that unit would be:(($5 ))
The cumulative difference between the price producers actually receive for a good and the lowest price for which they would have been willing to sell it is called:((producer surplus.))
The sum of producer surplus and consumer surplus is:((total surplus.))
Suppose a market is in equilibrium. The area below the demand curve and above the market price is:((consumer surplus.))
Suppose a market is in equilibrium. The area below the market price and above the supply curve is:((producer surplus.))
Consumer surplus is the cumulative difference between:((the amount consumers are willing to pay and the price they actually pay.))
In a perfectly competitive market, if supply and demand fully reflect all of the costs and benefits associated with production and consumption, then total economic surplus is maximized when:((the market is in equilibrium.))
Refer to the figure below. When the market is unregulated, consumer surplus is represented by the area:((ABC))
Refer to the figure below. When the market is unregulated, producer surplus is represented by the area:((DBC))
Refer to the figure below. If a price ceiling were imposed at point G, then excess demand would be measured by the distance between points:((F and I))
Refer to the figure below. If a price ceiling were imposed at point G, then producer surplus would be represented by the area ________.((DGF))
Refer to the figure below. If a price ceiling were imposed at point G, the consumer surplus would be represented by the area ________.((GAEF))
Refer to the figure below. If a price ceiling were imposed at point G, the loss in total economic surplus would be represented by the area ________.((FEC))
Refer to the figure below. If this market is unregulated, the economic surplus received by consumers is:(($16.00 ))
Refer to the figure below. If this market is unregulated, the economic surplus received by producers is:(($16.00 ))
Refer to the figure below.If this market is unregulated, total economic surplus is:(($32.00 ))
Refer to the figure below. If a price ceiling were imposed at $4, consumer surplus would be:(($20.00 ))
Refer to the figure below. If a price ceiling were imposed at $4, producer surplus would be:(($4.00 ))
Refer to the figure below. If a price ceiling were imposed at $4, total economic surplus would be ________, which is ________ less than when the market is unregulated market.(($24; $8))
Subsidies are most likely to:((reduce total economic surplus.))
Suppose that in an effort to help single parents, the government decides to pay part of the cost of childcare. This measure would increase ________ in the market for childcare.((consumer surplus))
Suppose a small island nation imports sugar for its population at the world price of $1,500 per ton. The domestic market for sugar is shown below. With no subsidy, the equilibrium price of sugar is ________ per ton, and the equilibrium quantity is ________ tons per day.(($1,500; 8))
Suppose a small island nation imports sugar for its population at the world price of $1,500 per ton. The domestic market for sugar is shown below. With no subsidy, what is consumer surplus?(($4,000 per day))
Suppose a small island nation imports sugar for its population at the world price of $1,500 per ton. The domestic market for sugar is shown below. With no subsidy, what is producer surplus?(($0 per day))
Suppose a small island nation imports sugar for its population at the world price of $1,500 per ton. The domestic market for sugar is shown below. If the government provides a subsidy of $500 per ton, the equilibrium price of sugar will be ________ per ton, and the equilibrium quantity will be ________ tons per day.(($1,000; 12))
Suppose a small island nation imports sugar for its population at the world price of $1,500 per ton. The domestic market for sugar is shown below. If the government provides a subsidy of $500 per ton, then consumer surplus will be ________ per day.(($9,000 ))
Suppose a small island nation imports sugar for its population at the world price of $1,500 per ton. The domestic market for sugar is shown below. If the government provides a subsidy of $500 per ton, then producer surplus will be ________ per day.(($0 ))
Suppose a small island nation imports sugar for its population at the world price of $1,500 per ton. The domestic market for sugar is shown below. If the government provides a subsidy of $500 per ton, then relative to before the subsidy, consumer surplus will ________ by ________ per day.((increase; $5,000))
Suppose a small island nation imports sugar for its population at the world price of $1,500 per ton. The domestic market for sugar is shown below. If the government provides a subsidy of $500 per ton, then the cost of subsidy, which must be borne by taxpayers, will be ________ per day.(($6,000 ))
Suppose a small island nation imports sugar for its population at the world price of $1,500 per ton. The domestic market for sugar is shown below. If the government provides a subsidy of $500 per ton, then relative to before the subsidy, total economic surplus will ________ by ________ per day.((decrease; $1,000))
The main problem with price subsidies is that they:((lower total economic surplus.))
The fact that price subsidies reduce economic surplus implies that:((we can find an alternative policy that will make both the rich and the poor better off.))
In most markets, scarce goods or services are rationed among competing users using:((monetary prices.))
In some countries, medical care is provided free-of-charge to citizens and is paid for by the government. In those countries, medical care:((is rationed by some non-monetary method.))
Joe is shopping for a new computer. A computer can be delivered to Joe's home for $1,200. Alternatively, Joe can pick up the same computer at the warehouse for $1,000. How should Joe buy the computer?((Joe should drive to the warehouse if his cost of driving to the warehouse is less than $200.))
The tendency for consumers to purchase more of a good or service as its price falls is captured by the:((law of demand.))
The law of demand indicates that as the cost of an activity:((rises, less of the activity will occur.))
According to the law of demand, when the price of shoes ________, people will consume ________ shoes.((falls; more))
As the price of computers falls, the quantity of computers demanded increases. This is an application of:((the law of demand.))
After subsistence levels of food, shelter, and clothing have been provided, economists refer to all other goods and services as:(("wants".))
Taking a limousine to a five-star restaurant in New York is a:((want to both Joe Average and Richie Rich.))
Shelter for homeless people is an example of:((a need.))
Higher education is a ________ and assigned textbooks are ________.((want; wants))
Growing rice requires extensive irrigation in California. Economists consider water to be a ________ for rice farmers in California.((want))
According to economists, the satisfaction people get from their consumption activities is called:((utility.))
The goal of utility maximization is to allocate your ________ in order to maximize your ________.((resources; satisfaction))
During Thanksgiving you participated in a pumpkin-pie eating contest. You really enjoyed the first two pies, the third one was okay, but as soon as you ate the fourth one you became ill and lost the contest. Your total utility ________ with the first three pies you ate.((increased))
During Thanksgiving you participated in a pumpkin-pie eating contest. You really enjoyed the first two pies, the third one was okay, but as soon as you ate the fourth one you became ill and lost the contest. After the third pie, your total utility:((decreased.))
During Thanksgiving you participated in a pumpkin-pie eating contest. You really enjoyed the first two pies, the third one was okay, but as soon as you ate the fourth one you became ill and lost the contest. You got ________ utility from eating the first pie than from eating the third pie.((more))
During Thanksgiving you participated in a pumpkin-pie eating contest. You really enjoyed the first two pies, the third one was okay, but as soon as you ate the fourth one you became ill and lost the contest. You got ________ utility from eating the fourth pie than from eating the second pie.((less))
Sven likes to water ski, but can only water ski during the one week each year when he is on vacation. Therefore, he plans to ski every day, for eight hours a day. The first day, Sven skied for eight hours and enjoyed every hour. The second day, Sven slept in and then skied for seven hours, which was fun but not as much fun as the first day. The third day, Sven skied for six hours, but was starting to get a bit bored by the end. The fourth day, Sven skied for four hours and then took a nap. On the fifth day of Sven's vacation, Sven went blueberry picking all day. Sven's total utility ________ with each hour that he skied.((increased))
Sven likes to water ski, but can only water ski during the one week each year when he is on vacation. Therefore, he plans to ski every day, for eight hours a day. The first day, Sven skied for eight hours and enjoyed every hour. The second day, Sven slept in and then skied for seven hours, which was fun but not as much fun as the first day. The third day, Sven skied for six hours, but was starting to get a bit bored by the end. The fourth day, Sven skied for four hours and then took a nap. On the fifth day of Sven's vacation, Sven went blueberry picking all day. On the fifth day of Sven's vacation, he had decided that another hour of skiing would yield:((less utility than blueberry picking.))
Sven likes to water ski, but can only water ski during the one week each year when he is on vacation. Therefore, he plans to ski every day, for eight hours a day. The first day, Sven skied for eight hours and enjoyed every hour. The second day, Sven slept in and then skied for seven hours, which was fun but not as much fun as the first day. The third day, Sven skied for six hours, but was starting to get a bit bored by the end. The fourth day, Sven skied for four hours and then took a nap. On the fifth day of Sven's vacation, Sven went blueberry picking all day. Sven's marginal utility from his first hour of skiing was ________ his marginal utility from his tenth hour of skiing.((greater than))
Sven likes to water ski, but can only water ski during the one week each year when he is on vacation. Therefore, he plans to ski every day, for eight hours a day. The first day, Sven skied for eight hours and enjoyed every hour. The second day, Sven slept in and then skied for seven hours, which was fun but not as much fun as the first day. The third day, Sven skied for six hours, but was starting to get a bit bored by the end. The fourth day, Sven skied for four hours and then took a nap. On the fifth day of Sven's vacation, Sven went blueberry picking all day. Sven's vacation convinced him that:((even for activities he really enjoys, diminishing marginal utility eventually sets in.))
The additional utility gained from consuming an additional unit of a good is called:((marginal utility.))
Pat's total utility after eating 99 Reese's Peanut Butter Cups was greater than his total utility after eating 100 Reese's Peanut Butter Cups. Therefore, Pat's marginal utility from the 100th peanut butter cup was:((negative.))
The term marginal utility denotes the amount by which ________ changes when consumption changes by ________ unit(s).((total utility; 1))
As Lynn eats more pizza, we would typically expect her marginal utility from eating pizza to:((decrease.))
Laura's total utility from consuming 8, 9, and 10 bonbons is 35, 42, and 45, respectively. Her marginal utility from the 9th bonbon is ________.((7))
When Lorenzo eats 1 slice of pizza for lunch, his total utility is 23, and when he eats 2 slices of pizza for lunch, his total utility is 42. Assuming that Lorenzo's marginal utility from eating pizza is always positive, we can infer that his total utility from eating 3 slices of pizza at lunch is:((greater than 42 and less than 61.))
Suppose Chris's marginal utility from the first taco he eats is 15, and his marginal utility from the second taco he eats is 12. One can infer that:((Chris's total utility from eating two tacos is 27.))
If marginal utility is positive, then as consumption increases:((total utility will increase.))
Refer to the accompanying figure. The total utility of consuming 4 pizzas a week is:((90))
Refer to the accompanying figure. The marginal utility of the 6th pizza is:((5))
Refer to the accompanying figure. The total utility of consuming two pizzas a week is:((55))
Refer to the accompanying figure. The marginal utility of the 7th pizza is((-5))
Refer to the accompanying figure. Total utility increases with each additional pizza up to the ________ and then declines, but marginal utility ________ with each additional pizza consumed.((6th pizza; decreases))
Blair has vast riches and consumes thousands of dollars' worth of consumer goods each week, yet she is never satisfied. Why not?((People's wants are unlimited.))
It is impossible for total utility to be ________ when marginal utility is ________.((decreasing; positive))
For Michael, the first cup of coffee he drinks every morning is heavenly. The second one is pretty good, but not as good as the first, and if he drinks a third cup he feels jittery and sick. For Michael, the marginal utility from drinking a cup of coffee is clearly:((decreasing.))
Refer to the accompanying table. The total utility of consuming two dinners out per week is:((175))
Refer to the accompanying table. The marginal utility of the 3rd dinner is:((60))
Refer to the accompanying table. The marginal utility of the 4th dinner out per week is:((15))
Refer to the accompanying table. The marginal utility of consuming the 2nd apple is:((15))
Refer to the accompanying table. The marginal utility of consuming apples is:((decreasing after the first apple.))
Refer to the accompanying figure. The marginal utility of the 5th apple is:((-5))
Refer to the accompanying table. Total utility ________ as additional apples are consumed, but marginal utility ________ with each additional apple consumed.((first increases then decreases; decreases))
If Terry's total utility is maximized when he owns 10 pairs of shoes, then Terry's total utility from owning 7 pairs of shoes is ________ Terry's total utility from owning 8 pairs.((less than))
The tendency for marginal utility to decline as consumption increases beyond some point is called:((the law of diminishing marginal utility.))
If the marginal utility of the 3rd cup of coffee is 23 and the marginal utility of the 4th cup is 15, then:((there is evidence of diminishing marginal utility.))
Joaquin's marginal utility from an additional slice of pumpkin pie is 4 utils and his marginal utility from an additional slice of pecan pie is 6 utils. If a slice of pumpkin pie costs $2.50, and a slice of pecan pie costs $3.00, then Joaquin:((should reallocate his spending towards pecan pie and away from pumpkin pie.))
According to the law of diminishing marginal utility:((as you consume less of something, your marginal utility from consuming that good will increase.))
Cory gets 18, 23, and 25 units worth of total utility from consuming 10, 11, and 12 raw oysters, respectively, and the price per oyster is 25 cents. Thus, one can infer that Cory:((is experiencing diminishing marginal utility.))
Alex wants to maximize his utility. At his current level of consumption, Alex's marginal utility from an additional cup of coffee is 15 utils, and his marginal utility from an additional can of soda is 11 utils. If the price of a cup of coffee is $3 and the price of a can of soda is $2, Alex should:((reallocate his spending away from coffee and towards soda.))
Refer to the accompanying figure. As Jeff watches more movies, his marginal utility:((gets smaller))
Refer to the accompanying figure. When Jeff goes from watching 3 to 4 movies a day, his total utility:((increases.))
Refer to the accompanying figure. For Jeff, the consumption of movies reflects the law of:((diminishing marginal utility.))
Assume that Dusty has $30 in income, the price of a loaf of bread is $1.50, and the price of a jar of peanut butter is $3. Dusty can buy a maximum of ________ loaves of bread or a maximum of ________ jars of peanut butter.((20; 10))
Assume that Dusty has $30 in income, the price of a loaf of bread is $1.50, and the price of a jar of peanut butter is $3. If Dusty's income rises to $45, Dusty can now buy a maximum of ________ loaves of bread or a maximum of ________ jars of peanut butter.((30; 15))
Assume that Dusty has $30 in income, the price of a loaf of bread is $1.50, and the price of a jar of peanut butter is $3. At the original income of $30, if the price of a loaf of bread decreased to $1 and the price of a jar of peanut butter increased to $5, then Dusty could buy a maximum of ________ loaves of bread or a maximum of ________ jars of peanut butter.((30; 6))
Assume that Dusty has $30 in income, the price of a loaf of bread is $1.50, and the price of a jar of peanut butter is $3. Suppose that at the original income of $30, the price of a loaf of bread increased to $3 and the price of a jar of peanut butter decreased to $2. Dusty can buy a maximum of ________ loaves of bread or a maximum of ________ jars of peanut butter.((10; 15))
Casey earns $150 a week and consumes only fish and shrimp. The price of fish is $3 a pound and the price of shrimp is $5 a pound. Casey can buy a maximum of ________ pounds of fish or a maximum of ________ pounds of shrimp.((50; 30))
Casey earns $150 a week and consumes only fish and shrimp. The price of fish is $3 a pound and the price of shrimp is $5 a pound. If Casey's income rises to $210, he could buy a maximum of ________ pounds of fish or a maximum of ________ pounds of shrimp.((70; 42))
Casey earns $150 a week and consumes only fish and shrimp. The price of fish is $3 a pound and the price of shrimp is $5 a pound. Suppose the price of fish decreased to $1.50 a pound, and the price of shrimp remains $5 a pound. At Casey's original income of $150 per week, Casey can now buy a maximum of ________ pounds fish or a maximum of ________ pounds shrimp.((100; 30))
The ________ combination of goods is the combination that yields the highest total utility given a consumer's income.((optimal))
At his current level of consumption, Evan gets twice as much marginal utility from an additional bottle of water as that from an additional bottle of soda. If the price of soda is $1.00 per bottle, then Evan is maximizing utility if the price of a bottle of water is:(($2.00 ))
Suppose that Cathy spends all of her income on 20 units of good X and 25 units of good Y. Cathy's marginal utility from the 20th unit of good X is 9 utils, and her marginal utility from the 25th unit of good Y is 19 utils. If the price of good X is $0.50 per unit and the price of good Y is $1.00 per unit, then to comply with the rational spending rule, Cathy should:((purchase less than 20 units of good X and more than 25 units of good Y.))
Suppose that Fiona spends all of her income on 10 units of good X and 14 units of good Y. Fiona's marginal utility from the 10th unit of good X is 24 utils, and her marginal utility from the 14th unit of good Y is 20 utils. If the price of good X is $8 per unit and the price of good Y is $5 per unit, then to comply with the rational spending rule, Fiona should:((purchase less than 10 units of good X and more than 14 units of good Y.))
Suppose that each week Henry buys 12 peaches and 3 apples at his local farmer's market. Both kinds of fruit cost $1 each. From this we can infer that:((if Henry is maximizing his utility, then his marginal utility from the 12th peach he buys must equal his marginal utility from the 3rd apple he buys.))
Suppose that Coke and Sprite each sell for $2 a can. Each month Joe buys 6 cans of Sprite and 30 cans of Coke. From this we can infer that:((Joe is maximizing his utility if his marginal utility from the 6th can of Sprite he buys is equal to his marginal utility from the 30th can of Coke he buys.))
Suppose a cup of tea costs $0.60 and a scone costs $1.20. If Edith spends all of her income on these two goods, and at her current level of consumption, she receives a marginal utility of 6 utils from the last cup of tea she buys and a marginal utility of 24 utils from the last scone she buys, then Edith should:((buy less tea and more scones.))
At her current level of consumption, Jess gets half as much marginal utility from an additional bagel as from an additional muffin. If the price of muffin is $2 each, then Jess is maximizing her utility if the price of a bagel is:(($1.00 ))
At his current level of consumption, Cameron gets 3 times more marginal utility from an additional game of pinball than from an additional game of ping pong. If the price of a ping pong game is $0.50, then he is maximizing utility if the price of a pinball game is:(($1.50 ))
Suppose that at Miles's current level of consumption, his marginal utility from a pizza is 10 utils, and his marginal utility from a pint of ice cream is 16 utils. If the price of a pizza is $8, and the price of a pint of ice cream is $5, is Miles maximizing his utility?((No. He should shift his spending away from frozen pizza and towards ice cream.))
Angela is currently playing five games of pool and bowling three games. At this level of consumption, her marginal utility from a game of pool is 10 and her marginal utility from bowling a game is 6. If both pool and bowling cost $1 a game, Angela should:((bowl less and play pool more.))
Suppose the price of an apple is $0.75, and the price of a banana is $0.50. If Hugh is maximizing his utility, and his marginal utility from consuming an apple is 24 utils, then his marginal utility from consuming a banana must be:((16 utils.))
Jamie's marginal utility from muffins and from doughnuts (in utils) is shown in the accompanying table. Jamie spends a total of $8 on muffins and/or doughnuts every morning. The price of each muffin is $2 and the price of each doughnut is $1. If Jamie consumes 3 muffins a day, then what is her marginal utility per dollar spent on the third muffin?((10))
Jamie's marginal utility from muffins and from doughnuts (in utils) is shown in the accompanying table. Jamie spends a total of $8 on muffins and/or doughnuts every morning. The price of each muffin is $2 and the price of each doughnut is $1. If Jamie consumes 2 doughnuts a day, then what is her marginal utility per dollar spent on the second doughnut?((20))
Jamie's marginal utility from muffins and from doughnuts (in utils) is shown in the accompanying table. Jamie spends a total of $8 on muffins and/or doughnuts every morning. The price of each muffin is $2 and the price of each doughnut is $1.((2 muffins, 4 doughnuts))
Taylor's marginal utility from watching movies and from eating out (in utils) is shown in the accompanying table. Taylor spends exactly $100 every month on these two forms of entertainment; the price of each movie is $10 and the price of each dinner is $20. If Taylor watches 2 movies a month, Taylor will eat out ________ times per month, and have marginal utility per dollar of ________ from movies and a marginal utility per dollar of ________ from eating out.((4; 5; 5))
Taylor's marginal utility from watching movies and from eating out (in utils) is shown in the accompanying table. Taylor spends exactly $100 every month on these two forms of entertainment; the price of each movie is $10 and the price of each dinner is $20. If Taylor eats out 3 times a month, Taylor will watch ________ movies, and have marginal utility per dollar of ________ from eating out and a marginal utility per dollar of ________ from going to the movies.((4; 6; ½))
Taylor's marginal utility from watching movies and from eating out (in utils) is shown in the accompanying table. Taylor spends exactly $100 every month on these two forms of entertainment; the price of each movie is $10 and the price of each dinner is $20.((2 movies and 4 dinners.))
Let MUc denote the marginal utility that Pablo receives from a cup of coffee, and let Pc denote the price of a cup of coffee. We typically expect that as Pablo buys more coffee:((MUc/Pc will fall.))
Purchasing goods such that the ratio of marginal utility to price is equal across all goods results in the:((greatest total utility.))
If a consumer buys two different goods, the rational spending rule requires that the:((ratio of marginal utility to price be equal for the two goods.))
For two goods, A and B, the rational spending rule is expressed as:(((MUA/P= (MUB/PB).))
Suppose Suzanne allocates her spending on apples and bananas according to the rational spending rule. If the price of apples is less than the price of bananas, then at Suzanne's optimal consumption bundle, her marginal utility from apples will be:((less than her marginal utility from bananas.))
You are trying to decide how to spend your last lunch dollar. You should use that dollar to buy more of the item:((that gives you the highest marginal utility per dollar.))
Suppose you want to maximize your total utility. If your marginal utility per dollar spent is higher for one good than for all others, then you should:((reallocate your spending towards that good.))
For two goods, coffee and scones, suppose that MU(coffee)/P(coffe= 4 and MU(scones)/P(scones) = 3. To maximize your total utility from these two goods, you should purchase:((more coffee and fewer scones.))
Suppose that at your current consumption of two goods, A and B, MUA/PA = 25 and MUB/PB = 20. In order to maximize, your utility you should:((purchase more of A and less of B.))
The rational spending rule is derived from the consumer's desire to:((maximize utility.))
When the price of a good rises, marginal utility per dollar spent on that good ________, leading consumers to purchase ________ of that good.((falls; less))
When the price of a good falls, marginal utility per dollar spent on that good ________, prompting consumers to purchase ________ of that good.((rises; more))
Refer to the accompanying table. The law of diminishing marginal utility:((applies to both Good A and Good B.))
Refer to the accompanying table. If the price of Good A is $1 and the price of Good B is $3, then the rational spending rule is satisfied when the consumer purchases ________ units of Good A and ________ units of Good B.((4; 3))
Refer to the accompanying table. If the price of Good A is $5 and the price of Good B is $4, then the rational spending rule is satisfied when the consumer purchases ________ units of Good A and ________ units of Good B.((1; 3))
Refer to the accompanying table. If the price of Good A is $2 and the price of Good B is $6, then the rational spending rule is satisfies when the consumer purchases ________ units of Good A and ________ units of Good B.((4; 3))
Refer to the accompanying table. The law of diminishing marginal utility:((applies to both t-shirts and sweaters.))
Refer to the accompanying table. If the price of a t-shirt is $6 and the price of a sweater is $80, then the rational spending rule is satisfied when the consumer purchases ________ t-shirts and ________ sweaters.((4; 1))
Refer to the accompanying table. If the price of a t-shirt is $10 and the price of a sweater is $40, then the rational spending is satisfied when the consumer purchases ________ t-shirts and ________ sweaters.((2; 3))
If a consumer reallocates his or her spending away from Good A and towards Good B, then the consumer's total utility will increase if:((MUA/PA < MUB/PB.))
If a consumer reallocates his or her spending away from Good B and towards Good A, then the consumer's total utility will increase if:((MUA/PA > MUB/PB.))
The dollar price of a good relative to the average dollar price of all other goods is the good's:((real price.))
The absolute price of a good in dollar terms is the good's:((nominal price.))
The fact that the average price of a gallon of gasoline is much higher in England than in the U.S. leads to which of the following predictions?((Large cars with poor gas mileage will be less popular in England than in the U.S.))
Fran runs a doughnut shop in a tiny 3-person town. The accompanying table shows the quantity demand by the three townspeople at various prices. When the price of a doughnut is 50 cents, what is the market demand for doughnuts?((9 doughnuts))
Fran runs a doughnut shop in a tiny 3-person town. The accompanying table shows the quantity demand by the three townspeople at various prices.((16 doughnuts))
Suppose the market for shoes consists of three consumers. The accompanying table shows the quantity demanded at various prices for each consumer: What is the market demand for shoes when the price is $50 a pair?((11 pairs))
Suppose the market for shoes consists of three consumers. The accompanying table shows the quantity demanded at various prices for each consumer: At $100 per pair, market demand:((is the same as Leigh's demand.))
Suppose the market for shoes consists of three consumers. The accompanying table shows the quantity demanded at various prices for each consumer:((has a greater demand for shoes than either Pat or Chris does.))
Refer to the accompanying figure. If Laura and Chris are the only two consumers in this market, then at a price of $2.00 per pound, the market demand for hamburger is:((6 pounds per week.))
Refer to the accompanying figure. If Laura and Chris are the only two consumers in this market then at a price of $2.50 per pound, the market demand for hamburger is:((4.5 pounds per week.))
Refer to the accompanying figure. If Laura and Chris are the only two consumers in this market, then the market demand for hamburger will be 9 pounds per week when the price of hamburger is:(($1.00 per pound.))
Refer to the accompanying figure. If Laura and Chris are the only two consumers in this market, then when the price of hamburger decreases from $2.50 to $2.00 per pound, the quantity demanded in the market will ________ by ________ pound(s) per week.((increase; 1.5))
Refer to the accompanying figures. If Mallory and Rick are the only two consumers in this market and the price of soda is $0.25 per can, then what will be the market demand for soda per month?((130 cans))
Refer to the accompanying figures. If Mallory and Rick are the only two consumers in this market and the price of soda is $0.75 per can, then what will be the market demand for soda each month?((50))
Refer to the accompanying figures. If Mallory and Rick are the only two consumers in this market, then the market demand for soda will be 90 cans per month when the price of a can of soda is ________.(($0.50 ))
Refer to the accompanying figures. If Mallory and Rick are the only two consumers in this market and the price of soda increases from $0.75 to $1.00 per can, the quantity of soda demanded in the market will ________ by ________ cans per month.((decrease; 40))
Refer to the accompanying figures. If the price of soda is $1.25 per can, then the quantity of soda demanded in the market each month is:((0))
Suppose that at a price of 25 cents per orange, 500 consumers each demand 4 oranges, and at a price of 20 cents per orange, 750 consumers each demand 5 oranges. Therefore, the market demand for oranges is ________ at a price of 25 cents per orange and ________ at a price of 20 cents per orange.((2,000; 3,750))
The accompanying figure shows the annual demand for haircuts for one person in the town of Beaumont. There are a total of 300 residents in Beaumont, each with this same demand curve. If the residents of Beaumont purchase a total of 1,800 haircuts each year, then the market price for a haircut must be:(($20.00 ))
The accompanying figure below shows a single consumer's demand for ice cream at the student union. During a regular semester, there are 500 students on campus. Each student's weekly demand for ice cream is shown above. When the price of ice cream is $2.00 per scoop, those 500 students purchase a total of ________ scoops per week from the student union.((3,000))
The accompanying figure shows a a single consumer's demand for ice cream at the student union.((1,800))
The accompanying figure shows a single consumer's demand for ice cream at the student union. If there are 1,000 students on campus during a regular semester, and each student's weekly demand for ice cream is shown above, then when the student union charges $3.00 per scoop, those 1,000 students purchase a total of ________ scoops per week.((4,000))
The accompanying figure shows a single consumer's demand for ice cream at the student union. The student union sells a total of 7,200 scoops of ice cream each week at a price of $2.00 per scoop. If every student who buys ice cream has the demand curve shown above, then there must be ________ students purchasing ice cream each week.((1,200))
The accompanying figure shows a single consumer's demand for ice cream at the student union. An increase in the number of students on this campus would cause:((the market demand curve for ice cream from the student union to shift to the right.))
The accompanying figure shows a single consumer's demand for ice cream at the student union. Fran is one of the students whose demand curve for ice cream is shown above. When price is $4.00, Fran demands ________ scoops a week, and when price is $2.00, Fran demand ________ scoops.((2; 6))
Consumer surplus measures:((the difference between the most a buyer would be willing to pay for a product and the price actually paid.))
Moe's reservation price for his economics textbook is $100. The week before the semester begins, Moe finds a copy of his textbook online for $75. Moe's consumer surplus from buying the textbook online is:(($25.00 ))
The accompanying figure shows the demand curve for a product that can be sold only in whole-number amounts. What is the maximum price that any buyer would be willing to pay for the first unit?((40))
The accompanying figure shows the demand curve for a product that can be sold only in whole-number amounts. At a price of $15 per unit, what would be the total consumer surplus in this market each day?(($75 ))
The accompanying figure shows the demand curve for a product that can be sold only in whole-number amounts. At a price of $25 per unit, what would be the total consumer surplus in this market each day?(($30 ))
Refer to the accompanying figure. What is the equilibrium quantity of bananas in this market?((5 pounds/day))
Refer to the accompanying figure. What is the equilibrium price of bananas in this market?(($5/pound))
Refer to the accompanying figure. At the equilibrium price, total consumer surplus is:(($7.50 per day.))
Refer to the accompanying figure. What is total consumer surplus at the market equilibrium?(($80 per day))
Refer to the accompanying figure. Suppose the dairy lobby convinces the government to impose price controls in this market. If the government requires cheese to be sold for a price of $8 per pound, then, relative to before the price controls, total consumer surplus in the market would:((decrease.))
Refer to the accompanying figure. Suppose the dairy lobby convinces the government to impose price controls in this market. If the government requires cheese to be sold for a price of $8 per pound, total consumer surplus in the market would equal:(($20 per day.))
Refer to the accompanying figure. Suppose a consumer protection group convinces the government to impose price controls in this market. If the government requires cheese to be sold for less than the equilibrium price, then, relative to before the price controls, total consumer surplus in the market:((could either increase or decrease.))
Refer to the accompanying figure. At the equilibrium price, total consumer surplus in this market is:(($12,000 per day.))
Refer to the accompanying figure. Suppose Zeynep's budget constraint is initially given by BC1. If her budget constraint shifts to BC2, this could be the result of:((a decrease in the price of cheese sandwiches and an increase in income.))
Refer to the accompanying figure. If Zeynep's current budget constraint is BC2, and her budget for ham sandwiches and cheese sandwiches is $120, then the price of a cheese sandwich is:(($6.00 ))
At her current level of consumption, Wiebke's marginal utility from an additional pencil is 15 utils, and her marginal utility from an additional pen is 10 utils. This implies that Wiebke would be willing to give up at most ________ pencils for an additional pen.((0.66))
Refer to the accompanying figure. Suppose that James' initial budget constraint shifts from BC1 to BC2, and as a result, his best affordable bundle moves from point A to point B. According to this figure:((spinach and potatoes are substitutes.))
Refer to the accompanying figure. From the figure, we can infer that James' marginal rate of substitution is:((higher at point A than point B.))
A rational seller will sell another unit of output:((if the cost of making another unit is less than the revenue gained from selling another unit.))
Your neighbors have offered to pay you to look after their dog while they are on vacation. It will take you one hour per day to feed, walk, and care for the dog, which you can do either before or after you go to work. Your regular job pays $10 per hour, and you can work up to eight hours per day. The smallest amount of money you would accept to look after your neighbor's dog each day is equal to:((the value you place on one hour of leisure.))
Which of the following best explains why you are more likely to see a poor person than a wealthy person picking up aluminum cans to sell?((The opportunity cost of picking up cans is higher for wealthy people than for poor people.))
John is trying to decide how to divide his time between his job as a stocker in the local grocery store, which pays $7 per hour for as many hours as he chooses to work, and cleaning windows for the businesses downtown. He makes $2 for every window he cleans. John is indifferent between the two tasks, and the number of windows he can clean depends on how many hours he spends cleaning in a day, as shown in the accompanying table. What is John's opportunity cost of cleaning windows for an hour?(($7 ))
John is trying to decide how to divide his time between his job as a stocker in the local grocery store, which pays $7 per hour for as many hours as he chooses to work, and cleaning windows for the businesses downtown. He makes $2 for every window he cleans. John is indifferent between the two tasks, and the number of windows he can clean depends on how many hours he spends cleaning in a day, as shown in the accompanying table. John's benefit from his first hour cleaning windows is:(($14 ))
John is trying to decide how to divide his time between his job as a stocker in the local grocery store, which pays $7 per hour for as many hours as he chooses to work, and cleaning windows for the businesses downtown. He makes $2 for every window he cleans. John is indifferent between the two tasks, and the number of windows he can clean depends on how many hours he spends cleaning in a day, as shown in the accompanying table. The first hour John spends cleaning windows costs him ________ that he could have earned in the grocery store.(($7 ))
John is trying to decide how to divide his time between his job as a stocker in the local grocery store, which pays $7 per hour for as many hours as he chooses to work, and cleaning windows for the businesses downtown. He makes $2 for every window he cleans. John is indifferent between the two tasks, and the number of windows he can clean depends on how many hours he spends cleaning in a day, as shown in the accompanying table. A second hour cleaning windows will yield additional earnings of ________.(($8 ))
John is trying to decide how to divide his time between his job as a stocker in the local grocery store, which pays $7 per hour for as many hours as he chooses to work, and cleaning windows for the businesses downtown. He makes $2 for every window he cleans. John is indifferent between the two tasks, and the number of windows he can clean depends on how many hours he spends cleaning in a day, as shown in the accompanying table. Should John spend a third hour cleaning windows?((No, because the additional amount he would earn is $6, which is less than his opportunity cost of $7.))
John is trying to decide how to divide his time between his job as a stocker in the local grocery store, which pays $7 per hour for as many hours as he chooses to work, and cleaning windows for the businesses downtown. He makes $2 for every window he cleans. John is indifferent between the two tasks, and the number of windows he can clean depends on how many hours he spends cleaning in a day, as shown in the accompanying table. How many hours a day should John spend cleaning windows?((2))
John is trying to decide how to divide his time between his job as a stocker in the local grocery store, which pays $7 per hour for as many hours as he chooses to work, and cleaning windows for the businesses downtown. He makes $2 for every window he cleans. John is indifferent between the two tasks, and the number of windows he can clean depends on how many hours he spends cleaning in a day, as shown in the accompanying table. What is the lowest price per window that would induce John to spend at least one hour per day cleaning windows?(($1 ))
John is trying to decide how to divide his time between his job as a stocker in the local grocery store, which pays $7 per hour for as many hours as he chooses to work, and cleaning windows for the businesses downtown. He makes $2 for every window he cleans. John is indifferent between the two tasks, and the number of windows he can clean depends on how many hours he spends cleaning in a day, as shown in the accompanying table. What is the lowest price per window that John would be willing to accept to spend 4 hours per day cleaning windows?(($3.50 ))
John is trying to decide how to divide his time between his job as a stocker in the local grocery store, which pays $7 per hour for as many hours as he chooses to work, and cleaning windows for the businesses downtown. He makes $2 for every window he cleans. John is indifferent between the two tasks, and the number of windows he can clean depends on how many hours he spends cleaning in a day, as shown in the accompanying table. If we plot John's opportunity cost per window on the vertical axis and the number of windows cleaned each day on the horizontal axis, we will have John's ________ curve for window-cleaning services.((supply))
For a given seller, the accompanying figure shows the relationship between the number of units produced and the opportunity cost of producing an additional unit of output. What is this seller's reservation price for the 250th unit?(($4 ))
For a given seller, the accompanying figure shows the relationship between the number of units produced and the opportunity cost of producing an additional unit of output. If the market price of this good is $6, how many units would this seller produce?((300))
For a given seller, the accompanying figure shows the relationship between the number of units produced and the opportunity cost of producing an additional unit of output. As the market price of this good increases, the quantity produced by this seller will ________.((increase))
For a given seller, the accompanying figure shows the relationship between the number of units produced and the opportunity cost of producing an additional unit of output. If the market consists of 50 identical sellers, each with the same opportunity cost as the seller depicted in the figure, then how many units would be supplied in the market at a price of $14 per unit?((17,500))
A seller's supply curve shows the seller's:((opportunity cost of producing an additional unit of output at each quantity.))
Individual supply curves generally slope ________ because ________.((upward; of increasing opportunity costs.))
As the market price of a service increases, more potential sellers will decide to perform that service because:((more potential sellers will find that the market price exceeds their reservation price.))
The primary objective of most private firms is to:((maximize profit.))
The most important challenge facing a firm in a perfectly competitive market is deciding:((how much to produce.))
Total revenue minus both explicit and implicit costs defines a firm's:((profit.))
A firm's profit equals:(((P − AT× Q [(price minus average total cost) times the quantity sold].))
Which of the following statements is true for both Microsoft and a locally owned restaurant?((Both seek to maximize profits.))
Which of the following is a defining characteristic of all perfectly competitive markets?((All firms sell the same standardized product.))
Which of the following is NOT true of a perfectly competitive firm?((It seeks to maximize revenue.))
Which of the following is NOT a characteristic of a perfectly competitive market?((Each firm in the market sells a somewhat different variant of the good.))
If the market for butter is perfectly competitive, then the demand curve facing a firm that produces butter will be:((perfectly elastic.))
In which of the following markets do firms sell the same standardized product?((2% milk))
Last year, Casey grew fresh vegetables, which she sold at her local farmers market, but this year, Casey did not plant any vegetables and went to work at a bank instead. If Casey's decision to change careers did not affect the price of vegetables at the farmers market, then this suggests that:((the market for vegetables is perfectly competitive.))
Last year, Casey grew fresh vegetables, which she sold at her local farmers market, but this year, Casey did not plant any vegetables and went to work at a bank instead. Which of the following best explains Casey's career change?((Casey's opportunity costs of gardening exceeded Casey's opportunity costs of working at the bank.))
A price-taker faces a demand curve that is:((horizontal at the market price.))
One implication of the shape of the demand curve facing a perfectly competitive firm is that:((if the firm increases its price above the market price, it will earn zero revenue.))
An imperfectly competitive firm is one that:((has at least some influence over the market price.))
A profit-maximizing perfectly competitive firm must decide:((only on how much to produce, taking price as fixed.))
Jenny sells lemonade in front of her house in the summer. Several other kids in Jenny's neighborhood also run lemonade stands in the summer. The lemonade market in Jenny's neighborhood is more likely to be perfectly competitive if:((each lemonade stand sells the same kind of lemonade.))
Jenny sells lemonade in front of her house in the summer. Several other kids in Jenny's neighborhood also run lemonade stands in the summer. If the lemonade market is perfectly competitive, and Jenny is charging the equilibrium price, then Jenny can increase her revenue if she:((keeps the price of her lemonade the same and increases the output.))
Jenny sells lemonade in front of her house in the summer. Several other kids in Jenny's neighborhood also run lemonade stands in the summer. Suppose that the first week of summer, Jenny charged 25 cents for an 8-ounce cup of lemonade, her next-door neighbor Sam charged 50 cents for an 8-ounce cup of lemonade, and Alex across the street charged 15 cents for an 8-ounce cup of lemonade. Assuming the market for lemonade is perfectly competitive, what is most likely to happen?((Eventually prices will equalize across all three lemonade stands.))
Which of the following would be considered a factor of production in the provision of bus service?((Bus drivers))
The short run is best defined as:((a period of time sufficiently short that at least one factor of production is fixed.))
A fixed factor of production:((is fixed only in the short run.))
A variable factor of production:((is variable in both the short run and the long run.))
The long run is best defined as:((a period of time sufficiently long that all factors of production are variable.))
According to the law of diminishing returns, when some factors of production are fixed, in order to increase production by a given amount, a firm will eventually need to add successively:((larger and larger quantities of the variable factors of production.))
Which of the following is the most likely to be a fixed factor of production at a farm?((The land on which the farm is located))
Which of the following is the most likely to be a fixed factor of production at a pizza restaurant?((The size of the seating area))
Which of the following is the most likely to be a variable factor of production at a university?((The number of librarians))
One reason that variable factors of production tend to show diminishing returns in the short run is that:((there is only so much that can be produced using additional variable inputs when some factors of production are fixed.))
To produce 150 units of output, a firm must use 3 employees per day. To produce 300 units of output, the firm must use 8 employees per day. Apparently, the firm is:((experiencing diminishing returns.))
Suppose 30 employees per day can produce 50 units of output. Assuming the law of diminishing marginal returns is present, to produce 100 units of output would require:((more than 30 additional employees per day.))
Refer to the accompanying table. As the firm increases the number of employees per day each day from 1 to 2, output increases by:((33 units.))
Refer to the accompanying table. To increase output from 33 to 66 units requires ________ extra employee(s) per day; to increase output from 66 to 99 units requires ________ extra employee(s) per day.((1; 2))
Refer to the accompanying table. To increase output from 99 to 132 units requires ________ extra employees per day; to increase output from 132 to 165 units requires ________ extra employees per day.((3; 4))
Refer to the accompanying table. The law of diminishing marginal returns becomes evident after ________ units of output are produced.((66))
The accompanying table describes the relationship between the number of workers hired by a call center each hour and the number of calls the call center can make each hour. The call center has only 1 telephone. The telephone costs the firm $5/hour (regardless of how many calls are made), and each worker is paid $10 per hour. What is the total cost of making 6 calls an hour?(($65 ))
The accompanying table describes the relationship between the number of workers hired by a call center each hour and the number of calls the call center can make each hour. The call center has only 1 telephone. The telephone costs the firm $5/hour (regardless of how many calls are made), and each worker is paid $10 per hour. Average variable cost is minimized when output is approximately:((22 units.))
The accompanying table describes the relationship between the number of workers hired by a call center each hour and the number of calls the call center can make each hour. The call center has only 1 telephone. The telephone costs the firm $5/hour (regardless of how many calls are made), and each worker is paid $10 per hour. If the price of a telephone increases to from $5 to $10 an hour and nothing else changes, then:((marginal cost would not change.))
The accompanying table describes the relationship between the number of workers hired by a call center each hour and the number of calls the call center can make each hour. The call center has only 1 telephone. The telephone costs the firm $5/hour (regardless of how many calls are made), and each worker is paid $10 per hour. Given the information in the table above, what is the call center's marginal cost when it goes from making 6 to 16 calls an hour?(($2 ))
If a firm spends $400 to produce 20 units of output and spends $880 to produce 40 units, then between 20 and 40 units of output, the marginal cost of production is:(($24.00 ))
Marginal cost is calculated as:((the change in total cost divided by the change in output.))
If a production process exhibits diminishing returns, then as output rises:((marginal cost will eventually increase.))
Assume that each day a firm uses 13 employee-hours per day and an office to produce 100 units of output. The price of each unit output is $5, the hourly wage rate is $10, and rent on the office is $200 per day. Each day the firm earns a ________ of ________.((profit; $170))
In general, when the price of a variable factor of production increases:((marginal cost rises.))
In general, when the price of a fixed factor of production increases:((marginal cost is unchanged.))
Suppose a profit-maximizing firm in a perfectly competitive market is earning an economic profit of $1,345. If the firm's fixed cost increases from $200 to $300, the firm will:((earn a smaller profit.))
Suppose a profit-maximizing firm in a perfectly competitive market is collecting $1,999 in total revenues. If the total cost of its fixed factors of production falls from $500 to $400, the firm will:((earn greater profits or smaller losses.))
Suppose a firm uses workers and office space to produce output. The firm is locked into a year-long lease on its office space, but it can easily vary the number of employee-hours it uses each day. The accompanying table describes the relationship between the number of employee-hours the firm uses each day and the firm's daily output. Each unit of output sells for $2, the hourly wage rate is $14, and the rent on the office space is $50 per day. This firm's fixed cost each day is:(($50.00 ))
Suppose a firm uses workers and office space to produce output. The firm is locked into a year-long lease on its office space, but it can easily vary the number of employee-hours it uses each day. The accompanying table describes the relationship between the number of employee-hours the firm uses each day and the firm's daily output. Each unit of output sells for $2, the hourly wage rate is $14, and the rent on the office space is $50 per day. When the firm uses 9 employee-hours per day, its total variable cost each day is:(($126.00 ))
Suppose a firm uses workers and office space to produce output. The firm is locked into a year-long lease on its office space, but it can easily vary the number of employee-hours it uses each day. The accompanying table describes the relationship between the number of employee-hours the firm uses each day and the firm's daily output. Each unit of output sells for $2, the hourly wage rate is $14, and the rent on the office space is $50 per day. When the firm uses 9 employee-hours per day, its total cost each day is:(($176.00 ))
Suppose a firm uses workers and office space to produce output. The firm is locked into a year-long lease on its office space, but it can easily vary the number of employee-hours it uses each day. The accompanying table describes the relationship between the number of employee-hours the firm uses each day and the firm's daily output. Each unit of output sells for $2, the hourly wage rate is $14, and the rent on the office space is $50 per day. When the firm uses 9 employee-hours per day, its total revenue each day is:(($240.00 ))
Suppose a firm uses workers and office space to produce output. The firm is locked into a year-long lease on its office space, but it can easily vary the number of employee-hours it uses each day. The accompanying table describes the relationship between the number of employee-hours the firm uses each day and the firm's daily output. Each unit of output sells for $2, the hourly wage rate is $14, and the rent on the office space is $50 per day. When the firm uses 9 employee-hours per day, it earns a daily ________ of ________.((profit; $64))
Suppose a firm uses workers and office space to produce output. The firm is locked into a year-long lease on its office space, but it can easily vary the number of employee-hours it uses each day. The accompanying table describes the relationship between the number of employee-hours the firm uses each day and the firm's daily output. Each unit of output sells for $2, the hourly wage rate is $14, and the rent on the office space is $50 per day. What is the marginal cost of production between 80 and 120 units of output each day?(($1.75 ))
Refer to the accompanying table. It is clear that diminishing returns sets in after ________ workers per day.((4))
The accompanying table shows a pizzeria's fixed cost and variable cost at different levels of output. Pizzas sell for $20 each. When the pizzeria makes 100 pizzas a day, its fixed cost is ________ and its total cost is(($500; $1,350))
The accompanying table shows a pizzeria's fixed cost and variable cost at different levels of output. Pizzas sell for $20 each. When the pizzeria makes 50 pizzas a day, its average total cost is ________.(($15 ))
The accompanying table shows a pizzeria's fixed cost and variable cost at different levels of output. Pizzas sell for $20 each. When the pizzeria makes 50 pizzas a day, its average total cost is ________.(($5 ))
The accompanying table shows a pizzeria's fixed cost and variable cost at different levels of output. Pizzas sell for $20 each. When the pizzeria makes 25 pizzas a day, its average fixed cost is ________.(($20 ))
The accompanying table shows a pizzeria's fixed cost and variable cost at different levels of output. Pizzas sell for $20 each. Between 25 and 50 pizzas per day, the pizzeria's marginal cost is ________.(($4 ))
The accompanying table shows a pizzeria's fixed cost and variable cost at different levels of output. Pizzas sell for $20 each. When the pizzeria makes 125 pizzas per day, its total revenue is ________.(($2,500 ))
The accompanying table shows a pizzeria's fixed cost and variable cost at different levels of output. Pizzas sell for $20 each. When the pizzeria makes 100 pizzas per day, it earns an economic ________ of ________.((profit; $650))
Refer to the accompanying figure. At quantities less than 50 doughnuts per day:((average cost is declining because marginal cost is less than average cost.))
Refer to the accompanying figure. If the market for doughnuts is perfectly competitive, then assuming this firm can earn enough revenue to cover its variable cost, it should produce:((the quantity of doughnuts at which marginal cost equals the market price.))
Refer to the accompanying figure. If the market for doughnuts is perfectly competitive, and the price of a doughnut is 10 cents, then this firm:((should produce 50 doughnuts.))
Refer to the accompanying figure. If the market for doughnuts is perfectly competitive, and the price of a doughnut is 25 cents, then this firm should:((produce 80 doughnuts.))
Refer to the accompanying figure. If the market for doughnuts is perfectly competitive, and the price of a doughnut is 25 cents, then at this firm's profit maximizing level of output, the firm will earn an economic ________ of ________ per day.((profit; $8))
The price equals marginal cost rule for profit maximization is a specific example of which core principle?((The Cost-Benefit Principle))
Suppose a firm produces the level of output at which the marginal cost of the last unit produced equals the price of the good. Which of the following statements is always true?((The firm should shutdown if its total revenue is less than its variable cost.))
Even when a firm produces the level of output at which price equals marginal cost, it should shut down if its total revenue is less than its:((variable cost.))
Suppose that when a firm produces the level of output at which price equals marginal cost, the firm's total revenue is less than its variable cost. In this case, the firm should:((shut down.))
Suppose that when a perfectly competitive firm produces 500 units of output a day, it earns an economic loss. If the price of each unit of output is $1.50, then, in the short run, it's clear that this firm:((should not shut down if its total variable cost is less than $750.))
Suppose that at a firm's profit-maximizing level of output, its total revenue is $1,250, the total cost of its variable factors of production is $1,000, and its total fixed cost is $500. This firm will ________ in the short run, and will ________ in the long run.((not shut down; exit the industry))
If a firm shuts down in the short run, then its:((economic loss will equal its fixed costs.))
Suppose that when a perfectly competitive firm produces 1,000 units of output, its total variable cost is $1,900. If the marginal cost of producing the 1,000th unit is $1.70, and if the market price of each unit of output is $1.70, then the firm should:((shut down.))
Fred runs a fishing lodge and has a very profitable business during the summer. In the fall, the number of guests at the lodge starts to decline. Fred should keep the lodge open:((only during those months in which his total revenue exceeds his variable cost.))
Average variable cost is defined as:((variable cost divided by total output.))
Average total cost is defined as:((total cost divided by total output.))
Suppose Chris is a potter who makes mugs. His total costs depend on the number of mugs he makes each day, as shown in the accompanying table. Chris's fixed cost is ________ per day.(($10 ))
Suppose Chris is a potter who makes mugs. His total costs depend on the number of mugs he makes each day, as shown in the accompanying table. When Chris produces 5 mugs per day, his average variable cost is ________.(($6 ))
Suppose Chris is a potter who makes mugs. His total costs depend on the number of mugs he makes each day, as shown in the accompanying table. When Chris produces 5 mugs per day, his average variable cost is ________.(($7 ))
Suppose Chris is a potter who makes mugs. His total costs depend on the number of mugs he makes each day, as shown in the accompanying table. If the market for mugs is perfectly competitive, and mugs sell for $7.50 each, then Chris should make ________ mugs per day.((4))
Suppose Chris is a potter who makes mugs. His total costs depend on the number of mugs he makes each day, as shown in the accompanying table. If the market for mugs is perfectly competitive, and mugs sell for $7.50 each, then at his profit-maximizing level of output, Chris will earn a ________ of ________ per day.((loss; $2))
Suppose Chris is a potter who makes mugs. His total costs depend on the number of mugs he makes each day, as shown in the accompanying table. If Chris's fixed costs decrease, then in the short run, his profit-maximizing level of output will:((not change.))
Suppose Ben owns a small company that makes kites. The market for kites is perfectly competitive, and kites sell for $25 each. Ben's total production costs vary depending on the number of kites he makes each day, as shown in the accompanying table. When Ben makes 2 kites per day, what is his average variable cost?(($13 ))
Suppose Ben owns a small company that makes kites. The market for kites is perfectly competitive, and kites sell for $25 each. Ben's total production costs vary depending on the number of kites he makes each day, as shown in the accompanying table. What is the profit-maximizing number of kites for Ben to make each day?((4))
Suppose Ben owns a small company that makes kites. The market for kites is perfectly competitive, and kites sell for $25 each. Ben's total production costs vary depending on the number of kites he makes each day, as shown in the accompanying table. What is Ben's economic profit at his profit-maximizing level of output?((−$72))
Suppose Ben owns a small company that makes kites. The market for kites is perfectly competitive, and kites sell for $25 each. Ben's total production costs vary depending on the number of kites he makes each day, as shown in the accompanying table.((No, because he can earn enough revenue to cover his variable cost.))
Suppose Ben owns a small company that makes kites. The market for kites is perfectly competitive, and kites sell for $25 each. Ben's total production costs vary depending on the number of kites he makes each day, as shown in the accompanying table. If Ben's fixed cost rises, then in the short run, his:((profit-maximizing level of output will not change.))
Suppose Sarah owns a small company that makes wedding cakes. The accompanying table shows how Sarah's total cost varies depending on the number of wedding cakes she makes each day. Sarah's fixed cost is ________ per day.(($100 ))
Suppose Sarah owns a small company that makes wedding cakes. The accompanying table shows how Sarah's total cost varies depending on the number of wedding cakes she makes each day. When Sarah produces 2 cakes per day, her average variable cost is ________.(($60 ))
Suppose Sarah owns a small company that makes wedding cakes. The accompanying table shows how Sarah's total cost varies depending on the number of wedding cakes she makes each day. The marginal cost of the 4th wedding cake per day is ________.(($100 ))
Suppose Sarah owns a small company that makes wedding cakes. The accompanying table shows how Sarah's total cost varies depending on the number of wedding cakes she makes each day. If the market for wedding cakes is perfectly competitive, and wedding cakes sell for $95 each, then Sarah should produce ________ cakes per day.((3))
Suppose Sarah owns a small company that makes wedding cakes. The accompanying table shows how Sarah's total cost varies depending on the number of wedding cakes she makes each day. If the market for wedding cakes is perfectly competitive, and wedding cakes sell for $95 each, then at her profit-maximizing level of output, Sarah will earn a ________ of ________ per day.((loss; $15))
Suppose Sarah owns a small company that makes wedding cakes. The accompanying table shows how Sarah's total cost varies depending on the number of wedding cakes she makes each day. If the market for wedding cakes is perfectly competitive, and wedding cakes sell for $125 each, then Sarah should produce ________ cakes per day.((5))
Suppose Sarah owns a small company that makes wedding cakes. The accompanying table shows how Sarah's total cost varies depending on the number of wedding cakes she makes each day. If the market for wedding cakes is perfectly competitive, and wedding cakes sell for $125 each, then at her profit-maximizing level of output, Sarah's profit will be ________ per day. If the market for wedding cakes is perfectly competitive, and wedding cakes sell for $125 each, then at her profit-maximizing level of output, Sarah's profit will be ________ per day.(($105 ))
Suppose Sarah owns a small company that makes wedding cakes. The accompanying table shows how Sarah's total cost varies depending on the number of wedding cakes she makes each day. If Sarah's fixed costs double, then in the short run, her profit-maximizing level of output:((will not change.))
When Acme Dynamite produces 250 units of output, its variable cost is $2,000, and its fixed cost is $500. It sells each unit of output for $25.When Acme Dynamite produces 250 units of output, its average variable cost is ________ and its average total cost is ________.(($8; $10))
When Acme Dynamite produces 250 units of output, its variable cost is $2,000, and its fixed cost is $500. It sells each unit of output for $25. When Acme Dynamite produces 250 units of output, its profit is:(($3,750.00 ))
When Acme Dynamite produces 250 units of output, its variable cost is $2,000, and its fixed cost is $500. It sells each unit of output for $25. If the price of dynamite drops to $10, should Acme Dynamite continue to operate in the short run?((Yes, because price is greater than average variable cost.))
In general, perfectly competitive firms maximize their profit by producing the level of output at which:((marginal cost equals price.))
If a perfectly competitive firm produces an output level at which price is greater than marginal cost, then the firm should:((expand output to earn greater profits or smaller losses.))
If a perfectly competitive firm produces an output level at which price is less than marginal costs, then the firm should:((reduce output to earn greater profits or smaller losses.))
An increase in the price a firm receives for its output will lead the firm to:((expand output.))
A decrease in the price a firm receives for its output will lead the firm to:((reduce output.))
Suppose a perfectly competitive firm is producing 37 units output, and the marginal cost of the 37th unit is $3. If the firm can sell each unit of output for $5 and the firm's revenue is sufficient to cover its variable cost, the firm should:((increase production.))
Suppose a perfectly competitive firm is producing 77 units of output, and the marginal cost of the 77th unit is 11. If the firm can sell each unit of output for $8 and the firm's revenue is sufficient to cover its variable cost, the firm should:((decrease production.))
Suppose a perfectly competitive firm is producing 1,000 units of output and the marginal cost of the 1,000th unit is $7. If the firm can sell each unit of output for $7 and the firm's revenue is sufficient to cover its variable cost, the firm should:((leave production unchanged.))
The accompanying graph shows the cost curves for Moe's mushroom gathering business, which is perfectly competitive. In the graph above, the average variable cost curve is labeled ________, the average total cost curve is labeled ________, and the marginal cost curve is labeled ________.((C; B; A))
The accompanying graph shows the cost curves for Moe's mushroom gathering business, which is perfectly competitive.((of diminishing returns to Moe's variable factors of production.))
The accompanying graph shows the cost curves for Moe's mushroom gathering business, which is perfectly competitive. If mushrooms sell for $10 per bushel, and Moe chooses the profit-maximizing quantity, he will gather:((zero bushels.))
The accompanying graph shows the cost curves for Moe's mushroom gathering business, which is perfectly competitive. If mushrooms sell for $40 per bushel, and Moe chooses the profit-maximizing quantity, he will:((earn positive profit.))
The accompanying graph shows the cost curves for Moe's mushroom gathering business, which is perfectly competitive. Moe's short run supply curve is:((curve A above curve C.))
A profit-maximizing firm will only produce a positive amount of output if:((its total revenue is greater than or equal to its variable cost.))
When plotting marginal and average cost curves, the ________ cost curve always crosses the ________ cost curve at its ________.((marginal; average total; minimum))
Refer to the accompanying graph. If this firm is a price taker, then when the price of each unit of output is $30, this firm's profit-maximizing level of output is ________.((80))
Refer to the accompanying graph. If this firm is a price taker, then when the price of each unit of output is $30, this firm's total revenue at its profit-maximizing level of output is ________.(($2,400 ))
Refer to the accompanying graph. If this firm is a price taker, then when the price of each unit of output is $30, this firm's total cost at its profit-maximizing level of output is ________.(($1,600 ))
Refer to the accompanying graph. If this firm is a price taker, then when the price of each unit of output is $30, how much profit does this firm earn at its profit-maximizing level of output?(($800 ))
Refer to the accompanying graph. If this firm is a price taker, then when the price of each unit if output is $15, what is this firm's profit-maximizing level of output?((60))
Refer to the accompanying graph. If this firm is a price taker and the price of each unit of output is $15, then at this firm's profit-maximizing level of output, it will earn a ________ of ________.((loss; $60))
Refer to the accompanying graph. If this firm is a price taker and the price of each unit of output is $15, then this firm should:((produce 60 units of output.))
Refer to the accompanying graph. If this firm is a price taker and the price of each unit of output is $9, then this firm should:((shut down in the short run.))
Refer to the accompanying graph. If this firm is a price taker and the price of each unit of output is $9, then at its profit-maximizing level of output, this firm will earn a ________ of ________.((loss; $300))
When the price of a perfectly competitive firm's output rises:((the firm will produce more.))
A perfectly competitive firm's supply curve is the portion of its ________ cost curve that lies above its ________ cost curve.((marginal; average variable))
A technological innovation that reduces a firm's cost of producing additional units of output will lead to:((an increase in the firm's supply.))
If crude oil is a variable factor of production for a firm, then an increase in the price of crude oil will lead to:((a decrease in the firm's supply.))
When more firms enter an industry:((the industry supply curve will shift right.))
Assume that the production technology required to produce goods X and Y is very similar. If a firm that is producing good X notices that the market price of good Y is rising, it will:((shift into producing good Y.))
Which of the following will cause an increase market supply?((A technological innovation that lowers the marginal cost of producing the good.))
Which of the following will cause a decrease in the supply of jeans?((An increase in the wages paid to workers who make jeans.))
An increase in consumers' demand for espresso will lead to an increase in ________, while an increase in the number of firms producing espresso will lead to a(n) ________.((quantity supplied; increase in supply))
As price increases, firms in a perfectly competitive market find that it is:((beneficial to produce more units of output.))
If a perfectly competitive firm can sell each unit of output for $9, and the marginal cost of the last unit produced is $8.50, then the:((extra benefit of the last unit produced is greater than the extra cost.))
The difference between the price a seller actually receives for a good and the seller's reservation price is:((producer surplus.))
Refer to the accompanying figure. The equilibrium price in this market is ________ and the equilibrium quantity is ________.(($30; 25))
Refer to the accompanying figure. When this market is in equilibrium, total producer surplus in the market is ________ per day.(($250 ))
Refer to the accompanying figure. Suppose a law is passed requiring restaurants to charge no more than $25 per meal. This law would:((decrease producer surplus.))
Refer to the accompanying figure. In this market, equilibrium price is ________ and equilibrium quantity is ________.(($25; 10))
Refer to the accompanying figure. Total producer surplus in this market is:(($125 ))
The psychological bias known as illusion of control refers to a ((belief that one can influence events even when one has no influence over what will happen.)
Decision makers who search for __________ solutions use ideas they have tried before or follow the advice of others who have faced similar problems. ((ready-made))
In the context of the strategic planning process, a critical task in environmental analysis is __________ future trends. ((forecasting)))
__________ is a condition that occurs when a decision-making group loses sight of its original purpose and a new, less important aim emerges. ((Goal displacement))
Discounting the future refers to ((weighing short-term benefits more heavily than long-term benefits.))
Integrated marketing communications represents the ________ element in the six Ps of a firm’s marketing mix. ((Promotion))
Twitter is a form of ________ and is increasingly used to reach current or potential customers. ((Blogging))
The IMC communication process includes a(n) (( the transmitter. ))
Tariffs, quotas, antidumping laws, and currency exchange policies affect global ((pricing strategies.))
Companies are legally required to disclose their privacy practices to customers on a(n) ______ basis. ((annual))
Commercial research firms, such as IRI and Nielsen, are sources of ((syndicated data.))
1) A company's fiscal year must correspond with the calendar year. (FALSE)
1) Accounting is an information and measurement system that identifies, records, and communicates an organization's business activities. (TRUE)
1) Accounts that appear in the balance sheet are often called temporary accounts. (FALSE)
1) Business transactions and events are the starting points of financial statements. (TRUE)
1) Merchandise inventory refers to products that a company owns and plans to sell to customers. (TRUE)
1. When preparing an unadjusted trial balance using a periodic inventory system, the amount shown for Merchandise Inventory is: (The beginning inventory amount.)
10) A company had a gross profit of $300,000 based on sales of $400,000. Its cost of goods sold equals $700,000. (FALSE)
10) Cash withdrawn by the owner for personal expenses should be treated as an expense of the business. (FALSE)
10) Closing entries are designed to transfer the end-of-period balances in the revenue accounts, the expense accounts, and the withdrawals account to owner's capital. (TRUE)
10) Internal users include lenders, shareholders, brokers and nonexecutive employees. (FALSE)
10) The revenue recognition principle requires that revenue be recorded when goods or services are provided, and at an amount expected to be received. (TRUE)
10. Cushman Company had $800,000 in sales, sales discounts of $12,000, sales returns and allowances of $18,000, cost of goods sold of $380,000, and $275,000 in operating expenses. Net income equals: ($115,000.)
100) A double-entry accounting system is an accounting system: (That records the effect of each transaction in at least two accounts with equal debits and credits.)
100) Accrued revenues: (At the end of one accounting period result in cash receipts in a future period)
100) Tara Westmont, the proprietor of Tiptoe Shoes, had annual revenues of $185,000, expenses of $103,700, and withdrew $18,000 from the business during the current year. The owner's capital account before closing had a balance of $297,000. The entry to close the Income Summary account at the end of the year, after revenue and expense accounts have been closed, is: (Debit Income Summary $81,300, credit T. Westmont, Capital $81,300)
100) The accounting principle that requires accounting information to be based on actual cost and requires assets and services to be recorded initially at the cash or cash-equivalent amount given in exchange, is the: (Measurement (Cost) principle.)
100) The credit terms 2/10, n/30 are interpreted as: (2% cash discount if the amount is paid within 10 days, or the balance due in 30 days.)
101) A trade discount is: (A reduction in selling price below the list price.)
101) An account linked with another account that has an opposite normal balance and is subtracted from the balance of the related account is a(n): (Contra account)
101) Edison Consulting received a $300 utilities bill and immediately paid it. Edison's general journal entry to record this transaction will include a: (Debit to Utilities Expense for $300.)
101) Tara Westmont, the proprietor of Tiptoe Shoes, had annual revenues of $185,000, expenses of $103,700, and withdrew $18,000 from the business during the current year. The owner's capital account before closing had a balance of $297,000. The Net Income for the year is: ($81,300)
101) The rule that (1) requires revenue to be recognized when goods or services are provided to customers and (2) at the amount expected to be received from the customer is called the: (Revenue recognition principle.)
102) GreenLawn Co. provides landscaping services to clients. On May 1, a customer paid GreenLawn $60,000 for 6-months services in advance. GreenLawn's general journal entry to record this transaction will include a: (Credit to Unearned Revenue for $60,000.)
102) Jasper Company is a wholesaler that buys merchandise in large quantities. Its supplier's catalog indicates a list price of $500 per unit on merchandise Jasper intends to purchase, and offers a 30% trade discount for large quantity purchases. The cost of shipping for the merchandise is $7 per unit. Jasper's total purchase price per unit will be: ($357.)
102) Tara Westmont, the proprietor of Tiptoe Shoes, had annual revenues of $185,000, expenses of $103,700, and withdrew $18,000 from the business during the current year. The owner's capital account before closing had a balance of $297,000. The ending owner's capital balance after closing is: ($360,300)
102) The question of when revenue should be recognized on the income statement according to GAAP is addressed by the: (Revenue recognition principle.)
102) The total amount of depreciation recorded for an asset for all periods for which an asset has been used: (Is referred to as accumulated depreciation)
103) A company had revenues of $75,000 and expenses of $62,000 for the accounting period. The owner withdrew $8,000 in cash during the same period. Which of the following entries could not be a closing entry? (Debit Income Summary $75,000; credit Revenues $75,000.)
103) Fragment Company is a wholesaler that sells merchandise in large quantities. Its catalog indicates a list price of $300 per unit on a particular product and a 40% trade discount is offered for quantity purchases of 50 units or more. The cost of shipping the merchandise is $7 per unit under terms FOB shipping point. If a customer purchases 100 units of this product, what is the amount of sales revenue that Fragment will record from this sale? ($18,000)
103) The periodic expense created by allocating the cost of plant and equipment to the periods in which they are used, representing the expense of using the assets, is called: (Depreciation expense)
103) The Superior Company acquired a building for $500,000. The building was appraised at a value of $575,000. The seller had paid $300,000 for the building 6 years ago. Which accounting principle would require Superior to record the building on its records at $500,000? (Measurement (Cost) principle.)
103) Willow Rentals purchased office supplies on credit. The general journal entry made by Willow Rentals will include a: (Credit to Accounts Payable.)
104) An asset created by prepayment of an insurance premium is: (Recorded as a debit to Prepaid Insurance.)
104) On December 15 of the current year, Conrad Accounting Services signed a $40,000 contract with a client to provide bookkeeping services to the client in the following year. Which accounting principle would require Conrad Accounting Services to record the bookkeeping revenue in the following year and not the year the cash was received? (Revenue recognition principle.)
104) Prior to recording adjusting entries, the Office Supplies account had a $359 debit balance. A physical count of the supplies showed $105 of unused supplies available. The required adjusting entry is: (Debit Office Supplies Expense $254 and credit Office Supplies $254)
104) The amount recorded for merchandise inventory includes all of the following except: (Freight costs paid by the seller.)
104) The following information is available from the adjusted trial balance of the Harris Vacation Rental Agency. After closing entries are posted, what will be the balance in the Sue Harris, Capital account? ($130,000.)
105) A company uses the perpetual inventory system and recorded the following entry: (Payment of the account payable less a 2% cash discount taken.)
105) If throughout an accounting period the fees for legal services paid in advance by clients are recorded in an account called Unearned Legal Fees, the end-of-period adjusting entry to record the portion of those fees that has been earned is: (Debit Unearned Legal Fees and credit Legal Fees Earned)
105) Marsha Bogswell is the owner of Bogswell Legal Services. Which accounting principle requires Marsha to keep her personal financial information separate from the financial information of Bogswell Legal Services? (Business entity assumption.)
105) The following information is available for the Higgins Travel Agency. After closing entries are posted, what will be the balance in the C. Higgins, Capital account? ($160,500.)
105) Victor Cruz contributed $70,000 in cash and land worth $130,000 to open a new business, VC Consulting. Which of the following general journal entries will VC Consulting make to record this transaction? (Debit Cash $70,000; Debit Land $130,000; Credit Cruz, Capital, $200,000.)
106) A limited liability company (LLC): (Has owners called members.)
106) Green Cleaning purchased $500 of office supplies on credit. The company's policy is to initially record prepaid and unearned items in balance sheet accounts. Which of the following general journal entries will Green Cleaning make to record this transaction? (Debit Office supplies, $500; credit Accounts payable, $500.)
106) On April 1, a company paid the $1,350 premium on a three-year insurance policy with benefits beginning on that date. What amount of the insurance expense will be reported on the annual income statement for the year ended December 31? ($337.50)
106) The following information is available for the Noir Detective Agency. After closing entries are posted, what will be the balance in the G. Noir, Capital account? ($239,400.)
106) Which of the following is not included on a purchase invoice? (Arrival date of items ordered.)
107) A company purchased $1,800 of merchandise on July 5 with terms 2/10, n/30. On July 7, it returned $200 worth of merchandise. On July 8, it paid the full amount due. The amount of the cash paid on July 8 equals: ($1,568.)
107) A partnership: (Has unlimited liability for its partners.)
107) Alicia Tax Services paid $500 to settle an account payable. Which of the following general journal entries will Alicia Tax Services make to record this transaction? (Debit Accounts payable, $500; credit Cash, $500.)
107) On July 1, a company paid the $2,400 premium on a one-year insurance policy with benefits beginning on that date. What will be the insurance expense on the annual income statement for the first year ended December 31? ($1,200)
107) The F. Mercury, Capital account has a credit balance of $37,000 before closing entries are made. If total revenues for the period are $55,200, total expenses are $39,800, and withdrawals are $9,000, what is the ending balance in the F. Mercury, Capital account after all closing entries are made? ($43,400.)
108) A company had no office supplies available at the beginning of the year. During the year, the company purchased $250 worth of office supplies. On December 31, $75 worth of office supplies remained. How much should the company report as office supplies expense for the year? ($175)
108) A company purchased $1,800 of merchandise on July 5 with terms 2/10, n/30. On July 7, it returned $200 worth of merchandise. On July 28, it paid the full amount due. The amount of the cash paid on July 28 equals: ($1,600.)
108) A law firm billed a client $1,800 for work performed in the current month. Which of the following general journal entries will the firm make to record this transaction? (Debit Accounts Receivable, $1,800; credit Legal Fees Revenue, $1,800.)
108) The F. Mercury, Capital account has a credit balance of $37,000 before closing entries are made. Total revenues for the period are $55,200, total expenses are $39,800, and withdrawals are $9,000. What is the correct closing entry for the revenue accounts? (Debit Revenue accounts $55,200; credit Income Summary $55,200.)
108) Which of the following accounting principles require that all goods and services purchased be recorded at actual cost? (Measurement (Cost) principle.)
109) A company purchased $1,800 of merchandise on July 5 with terms 2/10, n/30. On July 7, it returned $200 worth of merchandise. On July 28, it paid the full amount due. Assuming the company uses a perpetual inventory system, and records purchases using the gross method, The correct journal entry to record the purchase on July 5 is: (Debit Merchandise Inventory $1,800; credit Accounts Payable $1,800.)
109) A law firm collected $1,800 on account for work performed in the previous month. Which of the following general journal entries will the firm make to record this collection of cash? (Debit Cash, $1,800; credit Accounts Receivable, $1,800.)
109) On January 1, a company purchased a five-year insurance policy for $1,800 with coverage starting immediately. If the purchase was recorded in the Prepaid Insurance account, and the company records adjustments only at year-end, the adjusting entry at the end of the first year is: (Debit Insurance Expense, $360; credit Prepaid Insurance, $360)
109) The F. Mercury, Capital account has a credit balance of $37,000 before closing entries are made. Total revenues for the period are $55,200, total expenses are $39,800, and withdrawals are $9,000. What is the correct closing entry for the expense accounts? (Debit Income Summary $39,800; credit Expense accounts $39,800.)
109) Which of the following accounting principles prescribes that a company record its expenses incurred to generate the revenue reported? (Expense recognition (Matching) principle.)
11) A merchandising company's operating cycle begins with the purchase of merchandise and ends with the collection of cash from the sale. (TRUE)
11) Opportunities in accounting include auditing, consulting, market research, and tax planning. (TRUE)
11) The cash basis of accounting commonly increases the comparability of financial statements from period to period. (FALSE)
11) The Income Summary account is a permanent account that will be carried forward period after period. (FALSE)
11) When a company provides services for which cash will not be received until some future date, the company should record the amount billed as accounts receivable. (TRUE)
11. A company purchased $10,000 of merchandise on June 15 with terms of 3/10, n/45. On June 20, it returned $800 of that merchandise. On June 24, it paid the balance owed for the merchandise taking any discount it was entitled to. The cash paid on June 24 equals: ($8,924.)
110) A company purchased $1,800 of merchandise on July 5 with terms 2/10, n/30. On July 7, it returned $200 worth of merchandise. On July 28, it paid the full amount due. Assuming the company uses a perpetual inventory system, and records purchases using the gross method, the correct journal entry to record the merchandise return on July 7 is: (Debit Accounts Payable $200; credit Merchandise Inventory $200.)
110) A law firm collected $1,800 in advance for work to be performed in three months. Which of the following general journal entries will the firm make to record this transaction? (Debit Cash, $1,800; credit Unearned Legal Fees Revenue, $1,800.)
110) Revenue is properly recognized: (When goods or services are provided to customers and at the amount expected to be received from the customer.)
110) The Income Summary account is used to: (Close the revenue and expense accounts.)
110) Unearned revenue is reported in the financial statements as: (A liability on the balance sheet)
111) A company purchased $1,800 of merchandise on July 5 with terms 2/10, n/30. On July 7, it returned $200 worth of merchandise. On July 28, it paid the full amount due. Assuming the company uses a perpetual inventory system, and records purchases using the gross method, the correct journal entry to record the payment on July 28 is: (Debit Accounts Payable $1,600; credit Cash $1,600.)
111) All of the following are external users of accounting information except: (Chief executive officer (CEO).)
111) Jen Rogers withdrew a total of $35,000 from her business during the current year. The entry needed to close the withdrawals account is: (Debit Jen Rogers, Capital and credit Jen Rogers, Withdrawals for $35,000.)
111) Specter Consulting purchased $7,000 of supplies and paid cash immediately. Which of the following general journal entries will Specter Consulting make to record this transaction? (Debit Supplies, $7,000; credit Cash, $7,000.)
111) Which of the following assets is not depreciated? (Land)
112) A company purchased $1,800 of merchandise on July 5 with terms 2/10, n/30. On July 7, it returned $200 worth of merchandise. On July 12, it paid the full amount due. Assuming the company uses a perpetual inventory system, and records purchases using the gross method, the correct journal entry to record the payment on July 12 is: (Debit Accounts Payable $1,600; credit Cash $1,568.)
112) A company's ledger accounts and their end-of-period balances before closing entries are posted are shown below. What amount will be posted to Wilson Peters, Capital in the process of closing the Income Summary account? ($16,780 credit.)
112) All of the following are external users of accounting information except: (human resource managers.)
112) Jose Consulting paid $500 cash for utilities for the current month. Determine the general journal entry that Jose Consulting will make to record this transaction. (Debit Utilities Expense, $500; credit Cash, $500.)
112) Which of the following does not require an adjusting entry at year-end? (Cash invested by owner)
113) A company purchased $4,000 worth of merchandise. Transportation costs were an additional $350. The company returned $275 worth of merchandise and then paid the invoice within the 2% cash discount period. The total cost of this merchandise is: ($4,000.50.)
113) Alejandro Consulting paid $2,500 cash for a 5-month insurance policy that begins on March 1. Given the choices below, determine the general journal entry that Alejandro Consulting will make to record the cash payment. (Debit Prepaid Insurance, $2,500; credit Cash, $2,500.)
113) If a company uses $1,300 of its cash to purchase supplies, the effect on the accounting equation would be: (One asset increases $1,300 and another asset decreases $1,300, causing no effect.)
113) It is obvious that an error occurred in the preparation and/or posting of closing entries if: (all balance sheet accounts have zero balances.)
113) On May 1, a two-year insurance policy was purchased for $18,000 with coverage to begin immediately. What is the amount of insurance expense that would appear on the company's income statement for the first year ended December 31? ($6,000)
114) A buyer of $7,000 in merchandise inventory failed to take advantage of the vendor's credit terms of 2/15, n/45, and instead paid the invoice in full at the end of 45 days. By not taking advantage of the cash discount, the buyer lost the discount of: ($140.)
114) At the beginning of the year, a company's balance sheet reported the following balances: Total Assets = $225,000; Total Liabilities = $125,000; and Owner's Capital = $100,000. During the year, the company reported revenues of $46,000 and expenses of $30,000. In addition, owner's withdrawals for the year totaled $20,000. Assuming no other changes to owner's capital, the balance in the owner's capital account at the end of the year would be: (Ending Owner's Capital $96,000.)
114) Fragmental Co. leased a portion of its store to another company for eight months beginning on October 1, at a monthly rate of $800. Fragmental collected the entire $6,400 cash on October 1 and recorded it as unearned revenue. Assuming adjusting entries are only made at year-end, the adjusting entry made by Fragmental Co. on December 31 would be: (A debit to Unearned Rent and a credit to Rent Revenue for $2,400)
114) If a company receives $12,000 from the owner to establish a proprietorship, the effect on the accounting equation would be: (Assets increase $12,000 and equity increases $12,000.)
114) Ted Catering received $800 cash in advance from a customer for catering services to be provided in three months. Determine the general journal entry that Ted Catering will make to record the cash receipt. (Debit Cash, $800; credit Unearned Catering Revenue, $800.)
115) At the beginning of the year, Sigma Company's balance sheet reported Total Assets of $195,000 and Total Liabilities of $75,000. During the year, the company reported total revenues of $226,000 and expenses of $175,000. Also, owner withdrawals during the year totaled $48,000. Assuming no other changes to owner's capital, the balance in the owner's capital account at the end of the year would be: (Ending Owner's Capital $123,000.)
115) Gloria Catering provided $1,000 of catering services and billed its client for the amount owed. Determine the general journal entry that Gloria Catering will make to record this transaction. (Debit Accounts Receivable, $1,000; credit Catering Revenue, $1,000.)
115) If a company purchases equipment costing $4,500 on credit, the effect on the accounting equation would be: (Assets increase $ 4,500 and liabilities increase $4,500.)
115) On May 1, Sellers Marketing Company received $1,500 from Franco Marcelli for a marketing campaign effective from May 1 this year to April 30 of the following year. The Cash receipt was recorded as unearned fees and at year-end on December 31, $1,000 of the fees had been earned. Assuming adjustments are only made at year-end, the adjusting entry on December 31 would be: (A debit to Unearned Fees and a credit to Fees Earned for $1,000)
115) Sales returns: (Refer to merchandise that customers return to the seller after the sale.)
116) Adriana Graphic Design receives $1,500 from a client billed in a previous month for services provided. Which of the following general journal entries will Adriana Graphic Design make to record this transaction? (Debit Cash, $1,500; credit Accounts Receivable, $1,500.)
116) After preparing and posting the closing entries for revenues and expenses, the income summary account has a debit balance of $33,000. The entry to close the income summary account will be: (Debit Owner Capital $33,000; credit Income Summary $33,000.)
116) An example of a financing activity is: (Obtaining a long-term loan.)
116) Incurred but unpaid expenses that are recorded during the adjusting process with a debit to an expense and a credit to a liability are: (Accrued expenses)
116) Which of the following statements regarding sales returns and allowances is not true? (Sales returns and allowances do not have an impact on gross profit.)
117) A debit to Sales Returns and Allowances and a credit to Accounts Receivable: (Recognizes that a customer returned merchandise and/or received an allowance.)
117) An example of an operating activity is: (Paying wages.)
117) J. Smith withdrew $100 from Jay's Limo Services for personal use. Which of the following general journal entries will Jay's Limo Services make to record this transaction? (Debit J. Smith, Withdrawals, $100; credit Cash, $100.)
117) The adjusting entry at the end of an accounting period to record the unpaid salaries of employees for work provided is: (Debit Salaries Expense and credit Salaries Payable)
117) The trial balance prepared after all closing entries have been journalized and posted is called the: (Post-closing trial balance.)
118) A company pays each of its two office employees each Friday at the rate of $100 per day for a five-day week that begins on Monday. If the monthly accounting period ends on Tuesday and the employees worked on both Monday and Tuesday, the month-end adjusting entry to record the salaries earned but unpaid is: (Debit Salaries Expense $400 and credit Salaries Payable $400)
118) Jay's Limo Services paid $300 cash to employees for work performed in the current period. Which of the following general journal entries will Jay's Limo Services make to record this transaction? (Debit Salaries Expense, $300; credit Cash, $300.)
118) Operating activities: (Involve using resources to research, develop, purchase, produce, distribute and market products and services.)
118) Sales less sales discounts, less sales returns and allowances equals: (Net sales.)
118) Which of the following accounts showing a balance on the post-closing trial balance indicate an error? (Depreciation Expense-Office Equipment.)
119) A company pays its employees $4,000 each Friday, which amounts to $800 per day for the five-day workweek that begins on Monday. If the monthly accounting period ends on Thursday and the employees worked through Thursday, the amount of salaries earned but unpaid at the end of the accounting period is: ($3,200)
119) An example of an investing activity is: (Purchase of land.)
119) Garza Company had sales of $135,000, sales discounts of $2,000, and sales returns of $3,200. Garza Company's net sales equals: ($129,800.)
119) Russell Co. received a $400 utility bill for the current month's electricity. It is not due until the end of the next month which is when they intend to pay it. Which of the following general journal entries will Russell Co. make to record the receipt of the bill? (Debit Utilities Expense, $400; credit Accounts Payable, $400.)
119) Which of the following accounts showing a balance on the post-closing trial balance indicate an error? (S. Stills, Withdrawal.)
12) Closing entries are necessary so that owner's capital will begin each period with a zero balance. (FALSE)
12) Ethics is defined as maximizing personal wealth, regardless the cost. (FALSE)
12) Merchandise inventory is reported in the long-term assets section of the balance sheet. (FALSE)
12) Owner withdrawals always decrease equity. (TRUE)
12) Under the cash basis of accounting, no adjustments are made for prepaid, unearned, and accrued items. (TRUE)
12. A company purchased $10,000 of merchandise on June 15 with terms of 3/10, n/45, and FOB shipping point. The freight charge, $500, was added to the invoice amount. On June 20, it returned $800 of that merchandise. On June 24, it paid the balance owed for the merchandise taking any discount it is entitled to. The cash paid on June 24 equals: ($9,424.)
120) A post-closing trial balance reports: (All permanent ledger accounts with balances.)
120) Net Income: (Is the excess of revenues over expenses.)
120) On May 1, Shilling Company sold merchandise in the amount of $5,800 to Anders, with credit terms of 2/10, n/30. The cost of the items sold is $4,000. Shilling uses the perpetual inventory system and the gross method. The journal entry or entries that Shilling will make on May 1 is (are): (Accounts receivable 5,800 Sales 5,800 Cost of goods sold 4,000 Merchandise Inventory 4,000)
120) Sharp Services provided $800 of consulting work and $100 of design work to the same client. It billed the client for the total amount and is expecting to collect from the customer next month. Which of the following general journal entries did Sharp Services make to record the billing of the customer? (Debit Accounts Receivable, $900; credit Consulting Revenue, $800; credit Design Revenue, $100.)
120) The adjusting entry to record the salaries earned due to employees for services provided but unpaid at the end of the accounting period affects the accounts in which of the following ways? (Debit Salaries Expense and credit Salaries Payable)
121) If equity is $300,000 and liabilities are $192,000, then assets equal: ($492,000.)
121) On January 1, Eastern College received $1,200,000 from its students for the spring semester that it recorded in Unearned Tuition and Fees. The term spans four months beginning on January 2 and the college spreads the revenue evenly over the months of the term. Assuming the college prepares adjustments monthly, what amount of tuition revenue should the college recognize on February 28? ($300,000)
121) On May 1, Anders Company purchased merchandise in the amount of $5,800 from Shilling, with credit terms of 2/10, n/30. Anders uses the perpetual inventory system and the gross method. The journal entry that Anders will make on May 1 is: (Merchandise Inventory 5,800 Accounts payable 5,800)
121) Silvia's Studio provided $150 of dance instruction and rented out its dance studio to the same client for another $100. The client paid cash immediately. Identify the general journal entry below that Silvia's Studio will make to record the transaction. (Debit Cash, $250; credit Rental Revenue, $100; credit Instruction Revenue, $150.)
121) Which of the following statements is true? (A post-closing trial balance should include only permanent accounts.)
122) Geoff Parker, the owner of Parker Tax Services, started the business by investing $10,000 cash and a building worth $20,000. Identify the general journal entry below that Parker Tax Services will make to record the transaction. (Debit Cash, $10,000; debit Building, $20,000; credit G. Parker, Capital, $30,000.)
122) If assets are $300,000 and liabilities are $192,000, then equity equals: ($108,000.)
122) On February 3, Smart Company sold merchandise in the amount of $5,800 to Truman Company, with credit terms of 2/10, n/30. The cost of the items sold is $4,000. Smart uses the perpetual inventory system and the gross method. Truman pays the invoice on February 8, and takes the appropriate discount. The journal entry that Smart makes on February 8 is: (Cash 5,684 Sales discounts 116 Accounts receivable 5,800)
122) On January 1, Fashion Forward Magazine received $15,000 from subscribers for the annual subscriptions that it recorded in Unearned Subscription Revenue. The issues of the magazine are mailed to subscribers quarterly. What amount of subscription revenue should the magazine recognize on March 31 when the first issue is sent in March? ($3,750)
122) Reversing entries: (Are optional.)
123) A company provided $12,000 of consulting services on account. The customer promises payment in 30 days. Identify the journal entry below that properly records this transaction. (Debit Accounts Receivable, $12,000; credit Consulting Services Revenue, $12,000.)
123) An adjusting entry was made on year-end December 31 to accrue salary expense of $1,200. Assuming the company does not prepare reversing entries, which of the following entries would be prepared to record the $3,000 payment of salaries in January of the following year? (Salaries Payable $1,200; Salaries Expense $1,800; Cash $3,000)
123) Kline Company accrued wages of $7,350 that were earned by employees unpaid at the year-end. Assuming Kline uses reversing entries, which of the following entries correctly reverses the accrued wages at the beginning of the following year? (Debit Wages Payable $7,350; credit Wages Expense $7,350.)
123) On July 1, Ferguson Company sold merchandise in the amount of $5,800 to Tracey Company, with credit terms of 2/10, n/30. The cost of the items sold is $4,000. Ferguson uses the perpetual inventory system and the gross method. On July 5, Tracey returns some of the merchandise. The selling price of the merchandise is $500 and the cost of the merchandise returned is $350. The entry or entries that Ferguson must make on July 5 is (are): (Sales returns and allowances 500 Accounts receivable 500 Merchandise inventory 350 Cost of goods sold 350)
123) Resources a company owns or controls that are expected to yield future benefits are: (Assets.)
124) A company provided $12,000 of consulting services and was immediately paid in cash by the customer. Identify the journal entry below that properly records this transaction. (Debit Cash, $12,000; credit Consulting Services Revenue, $12,000.)
124) Increases in equity from a company's sales of products or services are: (Revenues.)
124) Juniper Company uses a perpetual inventory system and the gross method of accounting for purchases. The company purchased $9,750 of merchandise on August 7 with terms 1/10, n/30. On August 11, it returned $1,500 worth of merchandise. On August 16, it paid the full amount due. The amount of the cash paid on August 16 equals: (Cash Paid = ($9,750 − $1,500) * 0.99 = $8,167.50)
124) The difference between the cost of an asset and the accumulated depreciation for that asset is called: (Book Value)
124) Which of the following is not true? (Reversing entries should not be the exact opposite of previous period adjusting entries.)
125) A company purchased a new delivery van at a cost of $45,000 on July 1. The delivery van is estimated to have a useful life of 6 years and a salvage value of $3,000. The company uses the straight-line method of depreciation. How much depreciation expense will be recorded for the van during the first year ended December 31? ($3,500)
125) Juniper Company uses a perpetual inventory system and the gross method of accounting for purchases. The company purchased $9,750 of merchandise on August 7 with terms 1/10, n/30. On August 11, it returned $1,500 worth of merchandise. On August 26, it paid the full amount due. The amount of the cash paid on August 26 equals: (Cash Paid = ($9,750 − $1,500) = $8,250)
125) Molly Martin, the owner of Smart Consulting, withdrew $2,000 cash from the company for personal use. Identify the general journal entry below that Smart Consulting will make to record the transaction. (Debit M. Martin, Withdrawals, $2,000; credit Cash, $2,000.)
125) Reversing entries: (Will often result temporarily in abnormal account balances in some accounts.)
125) The difference between a company's assets and its liabilities, or net assets is: (Equity.)
126) A company's Office Supplies account shows a beginning balance of $600 and an ending balance of $400. If office supplies expense for the year is $3,100, what amount of office supplies was purchased during the period? ($2,900)
126) Creditors' claims on the assets of a company are called: (Liabilities.)
126) Juniper Company uses a perpetual inventory system and the gross method of accounting for purchases. The company purchased $9,750 of merchandise on August 7 with terms 1/10, n/30. On August 11, it returned $1,500 worth of merchandise. On August 16, it paid the full amount due. The correct journal entry to record the purchase on August 7 is: (Debit Merchandise Inventory $9,750; credit Accounts Payable $9,750)
126) Matthew Martin, the owner of Innovation Consulting, started the business by investing $40,000 cash. Identify the general journal entry below that Innovation Consulting will make to record the transaction. (Debit Cash, $40,000; credit M. Martin, Capital, $40,000.)
126) The purpose of reversing entries is to: (Simplify a company's recording of certain journal entries in the future.)
127) Decreases in equity from costs of providing products or services to customers are called: (Expenses.)
127) If a company records prepayment of expenses in an asset account, the adjusting entry when all or part of the prepaid asset is used or expired would: (Result in a debit to an expense and a credit to an asset account.)
127) If cash is received from customers in payment for services that have not yet been performed, the business would record the cash receipt as: (A credit to an unearned revenue account.)
127) Juniper Company uses a perpetual inventory system and the gross method of accounting for purchases. The company purchased $9,750 of merchandise on August 7 with terms 1/10, n/30. On August 11, it returned $1,500 worth of merchandise. The correct journal entry to record the merchandise return on August 11 is: (Debit Accounts Payable $1,500; credit Merchandise Inventory $1,500)
127) Which of the following is not true? (A worksheet is not useful in planning and organizing an audit of financial statements.)
128) A company recorded 2 days of accrued salaries of $1,400 for its employees on January 31. On February 9, it paid its employees $7,000 for these accrued salaries and for other salaries earned through February 9. Assuming the company does not prepare reversing entries, the January 31 and February 9 journal entries are: (Salaries Expense 5,600; Salaries Payable 1,400; Cash 7,000.)
128) Juniper Company uses a perpetual inventory system and the gross method of accounting for purchases. The company purchased $9,750 of merchandise on August 7 with terms 1/10, n/30. On August 11, it returned $1,500 worth of merchandise. On August 16, it paid the full amount due. The correct journal entry to record the payment on August 16 is: (Cash Paid = ($9,750 − $1,500) * 0.99 = $8,167.50)
128) On May 31, the Cash account of Tesla had a normal balance of $5,000. During May, the account was debited for a total of $12,200 and credited for a total of $11,500. What was the balance in the Cash account at the beginning of May? (A $4,300 debit balance.)
128) The description of the relation between a company's assets, liabilities, and equity, which is expressed as Assets = Liabilities + Equity, is known as the: (Accounting equation.)
128) Which of the following is not true? (The balances in the Income Statement credit column are unearned revenues.)
129) A company records the following journal entry: debit Cash $1,470, debit Sales Discounts $30, and credit Accounts Receivable $1,500. This means that a customer has taken what percentage cash discount for early payment? (Discount = $30/$1,500 = 2%)
129) If accrued salaries were recorded on December 31 with a debit to Salaries Expense and a credit to Salaries Payable, and no reversing entries were made on January 1, the entry to record payment of these wages on the following January 5 would include: (Debit to Salaries Payable and a credit to Cash.)
129) On April 30, Gomez Services had an Accounts Receivable balance of $18,000. During the month of May, total credits to Accounts Receivable were $52,000 from customer payments. The May 31 Accounts Receivable balance was $13,000. What was the amount of credit sales during May? ($47,000.)
129) Revenues are: (The increase in equity from a company's sales of products and services.)
129) Temporary accounts include all of the following except: (Prepaid rent.)
13) Cash sales shorten the operating cycle for a merchandiser; credit sales lengthen operating cycles. (TRUE)
13) Expenses always decrease equity. (TRUE)
13) Permanent accounts carry their balances into the next accounting period. (TRUE)
13) Since the revenue recognition principle requires that revenues be recorded when earned, there are no unearned revenues in accrual accounting. (FALSE)
13) The Sarbanes-Oxley Act (SOX) requires documentation and verification of internal controls. It also emphasizes effective internal controls. (TRUE)
13. A company's current assets are $23,420, its quick assets are $13,890, and its current liabilities are $12,220. Its acid-test ratio equals: (1.14.)
130) During the month of February, Rubio Services had cash receipts of $7,500 and cash disbursements of $8,600. The February 28 cash balance was $1,800. What was the February 1 beginning cash balance? ($2,900.)
130) If assets are $99,000 and liabilities are $32,000, then equity equals: ($67,000.)
130) On December 1, Simpson Marketing Company received $3,600 from a customer for a 2-month marketing plan to be completed January 31 of the following year. The cash receipt was recorded as unearned fees. The adjusting entry for the year ended December 31 would include: (Debit to Unearned Fees for $1,800.)
130) Permanent accounts include all of the following except: (Depreciation Expense—Equipment.)
130) Which of the following statements regarding inventory shrinkage is not true? (Inventory shrinkage is recognized by debiting an operating expense.)
131) Another name for equity is: (Net assets.)
131) Frisco Company's Merchandise Inventory account at year-end has a balance of $62,115, but a physical count reveals that only $61,900 of inventory exists. The adjusting entry to record this $215 of inventory shrinkage is: (Cost of goods sold 215 Merchandise Inventory 215)
131) The following transactions occurred during July:What was the amount of revenue for July? ($1,275.)
131) Which of the following statements about a company's operating cycle is not true: (Non-current items are those expected to come due within one year or the company's operating cycle.)
131) Wilson Company paid $4,800 for a 4-month insurance premium in advance on November 1, with coverage beginning on that date. The balance in the prepaid insurance account before adjustment at the end of the year is $4,800 and no adjustments had been made previously. The adjusting entry required on December 31 is: (Debit Insurance Expense, $2,400; credit Prepaid Insurance, $2,400.)
132) If Tyrol Willow, the owner of Willow Hardware, withdraws cash of the business to purchase a family car, the business should record this use of cash with an entry to: (Debit T. Willow, Withdrawals and credit Cash.)
132) What is the proper adjusting entry at December 31, the end of the accounting period, if the balance in the prepaid insurance account is $7,750 before adjustment, and the unexpired amount per analysis of policies is $3,250? (Debit Insurance Expense, $4,500; credit Prepaid Insurance, $4,500.)
132) When expenses exceed revenues, the result is called: (Net loss.)
132) Which of the following accounts would be closed at the end of the accounting period with a debit? (Sales.)
132) Which of the following types of businesses might have an operating cycle longer than one year? (Commercial airplane manufacturer.)
133) A resource that the owner takes from the company is called a(n): (Withdrawal.)
133) An income statement that includes cost of goods sold as another expense and shows only one subtotal for total expenses is a: (Single-step income statement.)
133) Marco Nelson opened a frame shop and completed these transactions:What was the balance of the cash account after these transactions were posted? ($40,300.)
133) On April 1, Griffith Publishing Company received $1,548 from Santa Fe, Inc. for 36-month subscriptions to several different magazines. The subscriptions started immediately. What is the amount of revenue that should be recorded by Griffith Publishing Company for the first year of the subscription assuming the company uses a calendar-year reporting period? ($387.)
133) Use the information in the adjusted trial balance presented below to calculate current assets for Taron Company: ($45,600.)
134) At the beginning of January of the current year, Sorrel Co.'s ledger reflected a normal balance of $52,000 for accounts receivable. During January, the company collected $14,800 from customers on account and provided additional services to customers on account totaling $12,500. Additionally, during January one customer paid Mikey $5,000 for services to be provided in the future. At the end of January, the balance in the accounts receivable account should be: ($49,700.)
134) Distributions of cash or other resources by a business to its owners are called: (Withdrawals.)
134) Expenses that support the overall operations of a business and include the expenses relating to accounting, human resource management, and financial management are called: (General and administrative expenses.)
134) On April 1, Griffith Publishing Company received $1,548 from Santa Fe, Inc. for 36-month subscriptions to several different magazines. The subscriptions started immediately. What is the amount of revenue that should be recorded by Griffith Publishing Company for the second year of the subscription assuming the company uses a calendar-year reporting period? ($516.)
134) Use the information in the adjusted trial balance presented below to calculate the current ratio for Taron Company: (1.87.)
135) During the month of March, Harley's Computer Services made purchases on account totaling $43,500. Also during the month of March, Harley was paid $8,000 by a customer for services to be provided in the future and paid $36,900 of cash on its accounts payable balance. If the balance in the accounts payable account at the beginning of March was $77,300, what is the balance in accounts payable at the end of March? ($83,900.)
135) On April 1, Griffith Publishing Company received $1,548 from Santa Fe, Inc. for 36-month subscriptions to several different magazines. The company credited Unearned Fees for the amount received and the subscriptions started immediately. Assuming adjustments are only made at year-end, What is the adjusting entry that should be recorded by Griffith Publishing Company on December 31 of the first year? (Debit Unearned Fees, $387; credit Fees Earned, $387.)
135) Prentice Company had cash sales of $94,275, credit sales of $83,450, sales returns and allowances of $1,700, and sales discounts of $3,475. Prentice's net sales for this period equal: (Net Sales = $94,275 + $83,450 − $1,700 − $3,475 = $172,550)
135) The assets of a company total $700,000; the liabilities, $200,000. What is the amount of equity? ($500,000.)
135) Use the information in the adjusted trial balance presented below to calculate total current liabilities for Taron Company: ($24,400.)
136) Based on the following information from Schrute Company's balance sheet, calculate the current ratio. (2.23.)
136) Multiple-step income statements: (Contain more detail than a simple listing of revenues and expenses.)
136) On April 1, Griffith Publishing Company received $1,548 from Santa Fe, Inc. for 36-month subscriptions to several different magazines. The company credited Unearned Fees for the amount received and the subscriptions started immediately. Assuming adjustments are only made at year-end, What is the adjusting entry that should be recorded by Griffith Publishing Company on December 31 of the second year? (Debit Unearned Fees, $516; credit Fees Earned, $516.)
136) On January 1 of the current year, Jimmy's Sandwich Company reported owner's capital totaling $122,500. During the current year, total revenues were $96,000 while total expenses were $85,500. Also, during the current year Jimmy withdrew $20,000 from the company. No other changes in equity occurred during the year. The change in owner's capital during the year was: (A decrease of $9,500.)
136) On May 31 of the current year, the assets and liabilities of Riser, Inc. are as follows: Cash $20,500; Accounts Receivable, $7,250; Supplies, $650; Equipment, $12,000; Accounts Payable, $9,300. What is the amount of owner's equity as of May 31 of the current year? ($31,100.)
137) Andrea Apple opened Apple Photography on January 1 of the current year. During January, the following transactions occurred and were recorded in the company's books: Based on this information, the balance in the cash account at the end of January would be: ($12,225.)
137) Expenses to promote sales by displaying and advertising merchandise, make sales, and deliver goods to customers are known as: (Selling expenses.)
137) On April 1, Santa Fe, Inc. paid Griffith Publishing Company $1,548 for 36-month subscriptions to several different magazines. Santa Fe debited the prepayment to a Prepaid Subscriptions account, and the subscriptions started immediately. What amount should appear in the Prepaid Subscription account for Santa Fe, Inc. after adjustments on December 31 of the first year assuming the company is using a calendar-year reporting period and no previous adjustment has been made? ($1,161.)
137) On August 31 of the current year, the assets and liabilities of Gladstone, Inc. are as follows: Cash $30,000; Supplies, $600; Equipment, $10,000; Accounts Payable, $8,500. What is the amount of owner's equity as of August 31 of the current year? ($32,100.)
137) The following information is available for Zephyr Company before closing the accounts. After all of the closing entries are made, what will be the balance in the Zephyr, Capital account? ($186,000.)
138) A company has net sales of $752,000 and cost of goods sold of $543,000. Its net income is $17,530. The company's gross margin and operating expenses, respectively, are: (Gross Margin = Net Sales − Cost of Goods Sold; $752,000 − $543,000 = $209,000 Operating Expenses = Gross Margin − Net Income; $209,000 − $17,530 = $191,470)
138) Andrea Apple opened Apple Photography on January 1 of the current year. During January, the following transactions occurred and were recorded in the company's books: Based on this information, the balance in the A. Apple, Capital account reported on the Statement of Owner's Equity at the end of the month would be: ($40,175.)
138) Assets created by selling goods and services on credit are: (Accounts receivable.)
138) On April 1, Santa Fe, Inc. paid Griffith Publishing Company $1,548 for 36-month subscriptions to several different magazines. Santa Fe debited the prepayment to a Prepaid Subscriptions account, and the subscriptions started immediately. What amount should appear in the Prepaid Subscription account for Santa Fe, Inc. after adjustments on December 31 of the second year assuming the company is using a calendar-year reporting period and the previous year adjustment had been made? ($645.)
138) The following information is available for Brendon Company before closing the accounts. What will be the amount in the Income Summary account that should be closed to Brendon, Capital? ($64,400.)
139) An exchange of value between two entities that yields a change in the accounting equation is called: (An external transaction.)
139) For the year ended December 31, a company had revenues of $187,000 and expenses of $109,000. The owner withdrew $37,000 during the year. Which of the following entries could not be a closing entry? (Debit Income Summary $187,000; credit revenues $187,000.)
139) On April 1, Santa Fe, Inc. paid Griffith Publishing Company $1,548 for 36-month subscriptions to several different magazines. Santa Fe debited the prepayment to a Prepaid Subscriptions account, and the subscriptions started immediately. What adjusting entry should be made by Santa Fe, Inc. for the adjustment on December 31 of the first year assuming the company is using a calendar-year reporting period and no previous adjustments had been made? (Debit Subscription Expense $387 and credit Prepaid Subscriptions $387.)
139) The debt ratio is used: (To assess the risk associated with a company's use of liabilities.)
139) Which of the following accounts is used in the periodic inventory system but not used in the perpetual inventory system? (Purchases)
14) Cost of goods sold is an expense, and is reported on the income statement. (TRUE)
14) If a company plans to continue business into the future, closing entries are not required. (FALSE)
14) Revenues always increase equity. (TRUE)
14) The expense recognition (matching) principle requires that expenses get recorded in the same accounting period as the revenues that are earned as a result of the expenses, not when cash is paid. (TRUE)
14) The fraud triangle asserts that the three factors that must exist for a person to commit fraud are opportunity, pressure, and rationalization. (TRUE)
14. Using the following year-end information for Bauman, LLC, calculate the current ratio and acid-test ratio: (3.16 and 0.97)
140) A company made no adjusting entry for accrued and unpaid employee salaries of $9,000 on December 31. Which of the following statements is true? (It will understate expenses and overstate net income by $9,000.)
140) Flagg records adjusting entries at its December 31 year-end. At December 31, employees had earned $12,000 of unpaid and unrecorded salaries. Prepare the January 1 journal entry to reverse the effect of the December 31 salary expense accrual. (Debit Salaries payable $12,000, credit Salaries expense $12,000.)
140) Identify the correct formula below used to calculate the debt ratio. (Total Liabilities/Total Assets.)
140) Saddleback Company paid off $30,000 of its accounts payable in cash. What would be the effects of this transaction on the accounting equation? (Assets, $30,000 decrease; liabilities, $30,000 decrease.)
141) Flagg records adjusting entries at its December 31 year-end. At December 31, employees had earned $12,000 of unpaid and unrecorded salaries. Prepare the journal entry on January 3 to record payment assuming the adjusting and reversing entries were made on December 31 and January 1. (Debit Salaries expense $30,000; credit Cash $30,000.)
141) If Houston Company billed a client for $10,000 of consulting work completed, the accounts receivable asset increases by $10,000 and: (Revenue increases $10,000.)
141) Langley has a debt ratio of 0.3 and its competitor, Appleton, has a debt ratio equal to 0.7. Determine the statement below that is correct: (Appleton's financial leverage is greater than Langley's financial leverage.)
141) The correct adjusting entry for accrued and unpaid employee salaries of $9,000 on December 31 is: (Debit Salary Expense, $9,000; credit Salaries Payable, $9,000.)
142) A company purchased new furniture at a cost of $14,000 on September 30. The furniture is estimated to have a useful life of 8 years and a salvage value of $2,000. The company uses the straight-line method of depreciation. How much depreciation expense will be recorded for the furniture for the first year ended December 31? ($375.00)
142) Alpha Company has assets of $600,000, liabilities of $250,000, and equity of $350,000. It buys office equipment on credit for $75,000. What would be the effects of this transaction on the accounting equation? (Assets increase by $75,000 and liabilities increase by $75,000.)
142) Identify the statement that is incorrect: (Risk is higher if a company has more assets.)
142) Which of the following accounts would be included in a post-closing trial balance? (Accounts Receivable.)
143) A company purchased new furniture at a cost of $14,000 on September 30. The furniture is estimated to have a useful life of 8 years and a salvage value of $2,000. The company uses the straight-line method of depreciation. What is the book value of the furniture on December 31 of the first year? ($13,625.00)
143) Contessa Company collected $42,000 cash on its accounts receivable. The effects of this transaction as reflected in the accounting equation are: (Total assets, total liabilities, and total equity are unchanged.)
143) Palmer Company is at the end of its annual accounting period. The accountant has journalized and posted all external transactions and all adjusting entries, has prepared an adjusted trial balance, and completed the financial statements. The next step in the accounting cycle is: (Close temporary accounts.)
143) The debt ratio of Company A is 0.31 and the debt ratio of Company B is 0.21. Based on this information, an investor can conclude: (Company B has less financial leverage.)
144) A company purchased new furniture at a cost of $16,000 on January 1. The furniture is estimated to have a useful life of 6 years and a $1,000 salvage value. The company uses the straight-line method of depreciation. What is the book value of the furniture on December 31 of the first year? ($13,500)
144) For the year ended December 31, a company has revenues of $317,000 and expenses of $196,000. The owner withdrew $50,000 during the year. The balance in the owner's capital account before closing is $81,000. Which of the following entries would be used to close the withdrawal account? (Debit Owner's Capital $50,000; credit Owner Withdrawals $50,000.)
144) If the liabilities of a business increased $75,000 during a period of time and the owner's equity in the business decreased $30,000 during the same period, the assets of the business must have: (Increased $45,000.)
144) The debt ratio of Braun is 0.9 and the debt ratio of Kemp is 1.0. Based on this information, an investor can conclude: (Kemp has the exact same dollar amount of total liabilities and total assets.)
145) If the assets of a business increased $89,000 during a period of time and its liabilities increased $67,000 during the same period, equity in the business must have: (Increased $22,000.)
145) Jennings Co. has total assets of $425 million. Its total liabilities are $110.5 million. Its equity is $314.5 million. Calculate the debt ratio. (26%.)
145) The balances in Sanchez Accounting Services' office supplies account on February 1 and February 28 were $1,200 and $375, respectively. If the office supplies expense for the month is $1,900, what amount of office supplies was purchased during February? ($1,075)
145) Which of the following accounts could not be classified as a current liability? (Notes payable (due in 5 years).)
146) If Regent Tax Services' office supplies account balance on March 1 was $1,400, the company purchased $675 of supplies during the month, and a physical count of supplies on hand at the end of March indicated $1,250 unused, what is the amount of the adjusting entry for office supplies on March 31? ($825)
146) If the liabilities of a company increased $74,000 during a period of time and equity in the company decreased $19,000 during the same period, what was the effect on the assets? (Assets would have increased $55,000.)
146) Sanders Co. has total assets of $385 million. Its total liabilities are $100.1 million and its equity is $284.9 million. Calculate its debt ratio. (26%.)
147) A physical count of supplies on hand at the end of May for Masters, Inc. indicated $1,250 of supplies on hand. The general ledger balance before any adjustment is $2,100. What is the adjusting entry for office supplies that should be recorded on May 31? (Debit Supplies Expense $850 and credit Supplies $850.)
147) All of the following statements accurately describe the debt ratio except: (The ratio is computed by dividing total equity by total liabilities.)
147) If a company paid $38,000 of its accounts payable in cash, what was the effect on the accounting equation? (Assets would decrease $ 38,000, liabilities would decrease $38,000, and equity remains unchanged.)
148) At the end of the current year, James Co. reported total liabilities of $300,000 and total equity of $100,000. The company's debt ratio was: (75%.)
148) If assets are $365,000 and equity is $120,000, then liabilities are: ($245,000.)
148) Which of the following statements is not true? (An unadjusted trial balance shows the account balances after they have been revised to reflect the effects of end-of-period adjustments.)
149) A trial balance prepared after adjustments have been recorded is called a(n): (Adjusted trial balance.)
149) At the beginning of the current year, Snell Co. total assets were $248,000 and its total liabilities were $174,200. During the year, the company reported total revenues of $93,000, total expenses of $76,000 and owner withdrawals of $5,000. There were no other changes in owner's capital during the year and total assets at the end of the year were $260,000. The company's debt ratio at the end of the current year is: (67%.)
149) Rushing had net income of $240 million and average total assets of $2,000 million. Its return on assets is: (12%.)
15) A periodic inventory system requires updating of the inventory account only at the beginning of an accounting period. (FALSE)
15) Management is not responsible for implementing internal controls and does not need to issue a report on internal controls. (FALSE)
15) Owner investments always decrease equity. (FALSE)
15) The cash basis of accounting is a system in which revenues are recorded when earned and expenses are recorded when incurred. (FALSE)
15) The first step in the accounting cycle is to analyze transactions and events to prepare for journalizing. (TRUE)
15. A company's net sales are $775,420, its costs of goods sold are $413,890, and its net income is $117,220. Its gross margin ratio equals: (46.6%.)
150) A trial balance prepared before any adjustments have been recorded is: (An unadjusted trial balance.)
150) Cage Company had net income of $160 million and average total assets of $2,000 million. Its return on assets (ROA) is: (8%.)
150) The process of transferring general journal entry information to the ledger is called: (Posting.)
151) A column in journals and ledger accounts that is used to cross reference journal and ledger entries is the: (Posting reference column.)
151) Speedy has net income of $18,955, and assets at the beginning of the year of $200,000. Assets at the end of the year total $246,000. Compute its return on assets. (8.5%.)
151) The adjusted trial balance contains information pertaining to: (All general ledger accounts.)
152) A complete record of each transaction in one place is called a(n): (Journal.)
152) Chou Co. has a net income of $43,000, assets at the beginning of the year are $250,000 and assets at the end of the year are $300,000. Compute its return on assets. (15.6%.)
152) Financial statements are typically prepared in the following order: (Income statement, statement of owner's equity, balance sheet.)
153) A balance sheet that places the assets above the liabilities and equity is called a(n): (Report form balance sheet.)
153) A general journal provides a place for recording all of the following except: (The balance in each account.)
153) U. S. treasury bonds are: (Low-risk and low-return investments.)
154) A balance sheet that places the liabilities and equity to the right of the assets is a(n): (Account form balance sheet.)
154) Risk is: (The uncertainty about the return we will earn.)
154) The balance column in a ledger account is: (A column for showing the balance of the account after each entry is posted.)
155) The statement of cash flows reports all of the following except: (The net increase or decrease in assets for the period reported.)
155) Under the alternative method for accounting for unearned revenue, which of the following pairs of journal entry formats is correct? (Cash, Consulting Revenue; Consulting Revenue, Unearned Revenue)
155) Which of the following is not one of the four steps of processing transactions? (Ensure assets are equal to liabilities.)
156) A record in which the effects of transactions are first recorded and from which transaction amounts are posted to the ledger is a(n): (Journal.)
156) The basic financial statements include all of the following except: (Statement of Changes in Assets.)
156) Under the alternative method for recording prepaid expenses, which is the correct set of journal entries? (Insurance Expense, Prepaid Insurance; Cash, Insurance Expense)
157) Centurion Co. had the following accounts and balances at December 31:Using the information in the table, calculate the company's reported net income for the period. ($5,500.)
157) The statement of owner's equity: (Reports how equity changes over a period of time.)
157) Which of the following statements related to U.S. GAAP and IFRS is not true? (U.S. GAAP does not require items to be separated by current and noncurrent classifications on the balance sheet.)
158) Jackson Services had the following accounts and balances at December 31:Using the information in the table, calculate the company's reported net income for the period. ($16,800.)
158) On December 1, Milton Company borrowed $300,000, at 8% annual interest, from the Tennessee National Bank. Interest is paid when the loan matures one year from the issue date. What is the adjusting entry for accruing interest that Milton would need to make on December 31, the calendar year-end? (Debit Interest Expense, $2,000; credit Interest Payable, $2,000.)
158) On March 12, Klein Company sold merchandise in the amount of $7,800 to Babson Company, with credit terms of 2/10, n/30. The cost of the items sold is $4,500. Klein uses the perpetual inventory system and the gross method of accounting for sales. The journal entry or entries that Klein will make on March 12 is (are) (Accounts receivable 7,800 Sales 7,800 Cost of goods sold 4,500 Merchandise Inventory 4,500).
158) The financial statement that reports whether the business earned a profit and also lists the revenues and expenses is called the: (Income statement.)
159) A balance sheet lists: (The types and amounts of assets, liabilities, and equity of a business as of a specific date.)
159) Cloud Solutions had the following accounts and balances as of December 31:Using the information in the table, calculate the total assets reported on the balance sheet for the period. ($24,900.)
159) On March 12, Klein Company sold merchandise in the amount of $7,800 to Babson Company, with credit terms of 2/10, n/30. The cost of the items sold is $4,500. Klein uses the perpetual inventory system and the gross method of accounting for sales. Babson pays the invoice on March 17, and takes the appropriate discount. The journal entry that Klein makes on March 17 is (Cash 7,644 Sales discounts 156 Accounts receivable 7,800).
159) On September 1, Kennedy Company loaned $100,000, at 12% annual interest, to a customer. Interest and principal will be collected when the loan matures one year from the issue date. Assuming adjustments are only made at year-end, what is the adjusting entry for accruing interest that Kennedy would need to make on December 31, the calendar year-end? (Debit Interest Receivable, $4,000; credit Interest Revenue, $4,000.)
16) A partnership is a business owned by two or more people. (TRUE)
16) A perpetual inventory system continually updates accounting records for merchandising transactions. (TRUE)
16) The accounting cycle refers to the sequence of steps used in preparing the work sheet. (FALSE)
16) The cash basis of accounting recognizes revenues when cash payments from customers are received. (TRUE)
16) Unearned revenue is a liability that is settled in the future when a company delivers its products or services. (TRUE)
16. Which of the following statements related to the multiple-step income statement is not true? (Shows only one total for expenses.)
160) A financial statement providing information that helps users understand a company's financial status, and which lists the types and amounts of assets, liabilities, and equity as of a specific date, is called a(n): (Balance sheet.)
160) A roofing company collects fees when jobs are complete. The work for one customer, whose job was bid at $3,000, has been completed as of December 31, but the customer has not yet been billed. Assuming adjustments are only made at year-end, what is the adjusting entry the company would need to make on December 31, the calendar year-end? (Debit Accounts Receivable, $3,000; credit Roofing Fees Revenue, $3,000.)
160) At the end of its first month of operations, JMP Consulting reported net income of $25,000. They also had account balances of: Cash, $18,000; Office Supplies, $2,000 and Accounts Receivable, $10,000. The owner's total investment for this first month was $5,000. There were no owner withdrawals in the first month. Calculate the ending balance in the Owner's Capital account to be reported on the Statement of Owner's Equity. ($30,000)
160) On March 12, Klein Company sold merchandise in the amount of $7,800 to Babson Company, with credit terms of 2/10, n/30. The cost of the items sold is $4,500. Klein uses the perpetual inventory system and the gross method of accounting for sales. On March 15, Babson returns some of the merchandise, which is not defective. The selling price of the returned merchandise is $600 and the cost of the merchandise returned is $350. The entry or entries that Klein must make on March 15 is (are) (Sales returns and allowances 600 Accounts receivable 600 Merchandise inventory 350 Cost of goods sold 350).
161) Identify the accounts that would normally have balances in the debit column of a business's trial balance. (Assets and expenses.)
161) On March 12, Klein Company sold merchandise in the amount of $7,800 to Babson Company, with credit terms of 2/10, n/30. The cost of the items sold is $4,500. Klein uses the perpetual inventory system and the gross method of accounting for sales. On March 15, Babson returns some of the merchandise. The selling price of the merchandise is $600 and the cost of the merchandise returned is $350. Babson pays the invoice on March 20, and takes the appropriate discount. The amount that Klein receives from Babson on March 20 is (Cash = $7,200 − $144 = $7,056).
161) The financial statement that identifies a company's cash receipts and cash payments over a period of time is the: (Statement of cash flows.)
161) Which of the following is not true regarding prepaid expenses? (The adjusting entry for prepaid expenses increases expenses and decreases liabilities.)
162) An annual reporting period consisting of any twelve consecutive months is known as: (Fiscal year.)
162) Identify the accounts that would normally have balances in the credit column of a business's trial balance (Revenues and liabilities.)
162) On March 12, Klein Company sold merchandise in the amount of $7,800 to Babson Company, with credit terms of 2/10, n/30. The cost of the items sold is $4,500. Klein uses the perpetual inventory system and the gross method of accounting for sales. On March 15, Babson returns some of the merchandise. The selling price of the merchandise is $600 and the cost of the merchandise returned is $350. Babson pays the invoice on March 20, and takes the appropriate discount. The journal entry that Klein makes on March 20 is (Cash 7,056 Sales discounts 144 Accounts receivable 7,200).
162) The financial statement that shows the beginning balance of owner's equity; the changes in equity that resulted from new investments by the owner; net income (or net loss); withdrawals; and the ending balance, is the: (Statement of owner's equity.)
163) Cash investments by owners are listed on which of the following statements? (Statement of owner's equity and statement of cash flows.)
163) Two accounting principles central to accrual accounting basis that are relied on in the adjusting process are: (Revenue recognition and Expense recognition (matching).)
163) Which of the following is not a step in the accounting process? (Ensure all cash is distributed to owners at the end of each period.)
163) Zenith Company's Merchandise Inventory account at year-end has a balance of $91,820, but a physical count reveals that only $90,450 of inventory exists. The adjusting entry to record this $1,370 of inventory shrinkage is (Cost of goods sold 1,370 Merchandise inventory 1,370).
164) A bookkeeper has debited an asset account for $3,500 and credited a liability account for $2,000. Which of the following would be an incorrect way to complete the recording of this transaction? (Debit another asset account for $1,500.)
164) Accounts payable appear on which of the following statements? (Balance sheet.)
164) Which of the following is not true regarding unearned revenues? (The adjusting entry for unearned revenues increases assets and increases revenues.)
165) A list of all ledger accounts and their balances at a point in time is called a(n): (Trial balance.)
165) Assuming prepaid expenses are originally recorded in balance sheet accounts, the adjusting entry to record use of a prepaid expense is: (Increase an expense; decrease an asset.)
165) The income statement reports all of the following except: (Assets owned by a business.)
166) Assuming unearned revenues are originally recorded in balance sheet accounts, the adjusting entry to record earning of unearned revenue is: (Decrease a liability; increase revenue.)
166) Identify the statement below that is true. (The trial balance is a list of all accounts from the ledger with their balances at a point in time.)
166) Use the following information as of December 31 to determine equity. (297,000.)
167) Netherland Corporation has the following unadjusted balances: Accounts Receivable, $80,000 (debit), and Allowance for Sales Discounts $300 (credit). Of the receivables, $50,000 of them are within the 2% discount period, and Netherland expects buyers to take $1,000 in future-period discounts ($50,000 × 2%) arising from this period's sales. The adjusting entry or entries to estimate sales discounts is (are) (Sales Discounts 700 Allowance for Sales Discounts 700).
167) The adjusting entry to record an accrued expense is: (Increase an expense; increase a liability.)
167) Use the following information for Meeker Corp. to determine the amount of equity to report. (270,000.)
167) While in the process of posting from the journal to the ledger, a company failed to post a $500 debit to the Equipment account. The effect of this error will be that: (The trial balance will not balance.)
168) A $15 credit to Sales was posted as a $150 credit. By what amount is the Sales account in error? ($135 overstated.)
168) An expense resulting from failing to take advantage of cash discounts when using the net method of recording purchases is called (Discounts lost).
168) Determine the net income of a company for which the following information is available for the month of July. (190,000.)
168) The adjusting entry to record an accrued revenue is: (Increase an asset; increase revenue.)
169) A company that uses the net method of recording purchases and a perpetual inventory system purchased $1,800 of merchandise on July 5 with terms 2/10, n/30. On July 7, it returned $200 worth of merchandise. On July 28, it paid the full amount due. The correct journal entry to record the payment on July 28 is (Debit Accounts Payable $1,568; debit Discounts Lost $32; credit Cash $1,600).
169) At year-end, a trial balance showed total credits exceeding total debits by $4,950. This difference could have been caused by: (The balance of $5,500 in the Office Equipment account being entered on the trial balance as a debit of $550.)
169) Determine the net income of a company for which the following information is available for the month of September. (167,800.)
169) On October 1, Goodwell Company rented warehouse space to a tenant for $2,500 per month. The tenant paid five months' rent in advance on that date, with the lease beginning immediately. The cash receipt was credited to the Unearned Rent account. The company's annual accounting period ends on December 31. Assuming adjustments are only made at year-end, the adjusting entry needed on December 31 is: (Debit Unearned Rent, $7,500; credit Rent Earned, $7,500.)
17) A company's chart of accounts is a list of all the accounts used and includes an identification number assigned to each account. (TRUE)
17) Beginning inventory plus net purchases equals merchandise available for sale. (TRUE)
17) Owners of a corporation are called shareholders or stockholders. (TRUE)
17) The accrual basis of accounting recognizes revenues when cash is received from customers. (FALSE)
17) The first five steps in the accounting cycle include analyzing transactions, journalizing, posting, preparing an unadjusted trial balance, and recording adjusting entries. (TRUE)
17. A company purchases merchandise with a catalog price of $20,000. The company receives a 35% trade discount from the seller. The seller also offers credit terms of 2/10, n/30. Assuming no returns were made and that payment was made within the discount period, what is the net cost of the merchandise? ($12,740.)
170) A company acquires equipment for $75,000 cash. This represents a(n): (Investing activity.)
170) Identify the item below that would cause the trial balance to not balance? (The cash payment of a $750 account payable was posted as a debit to Accounts Payable and a debit to Cash for $750.)
170) Morgan, Inc. uses a perpetual inventory system and the net method of recording purchases. On May 12, a merchandise purchase of $15,000 was made on credit, 2/10, n/30. The journal entry to record this purchase is (Merchandise Inventory 14,700 Accounts Payable 14,700).
170) On October 1, Goodwell Company rented warehouse space to a tenant for $2,500 per month and received $12,500 for five months' rent in advance on that date, with the lease beginning immediately. The cash receipt was credited to the Unearned Rent account. The company's annual accounting period ends on December 31. The Unearned Rent account balance at the end of December, after adjustment, should be: ($5,000.)
171) A company borrows $125,000 from the Northern Bank and receives the loan proceeds in cash. This represents a(n): (Financing activity.)
171) Sanborn Company rents space to a tenant for $2,200 per month. The tenant currently owes rent for November and December. The tenant has agreed to pay the November, December, and January rents in full on January 15 and has agreed not to fall behind again. Assuming adjustments are only made at year-end, the adjusting entry needed on December 31 is: (Debit Rent Receivable, $4,400; credit Rent Earned, $4,400.)
171) The credit purchase of a new oven for $4,700 was posted to Kitchen Equipment as a $4,700 debit and to Accounts Payable as a $4,700 debit. What effect would this error have on the trial balance? (The total of the Debit column of the trial balance will exceed the total of the Credit column by $9,400.)
171) The net method of recording purchases refers to recording (Purchases at the invoice price less any cash discounts.)
172) On a trial balance, if the Debit and Credit column totals are equal, then: (Equal debits and credits have been recorded for transactions.)
172) On March 15, Babson returns some of the merchandise, which is not defective. The entry or entries that Klein must make on March 15 is (are) (Sales returns and allowances 588 Accounts receivable 588 Merchandise inventory 350 Cost of goods sold 350)
172) Sanborn Company has 10 employees, who earn a total of $1,800 in salaries each working day. They are paid on Monday for the five-day workweek ending on the previous Friday. Assume that year ended December 31, is a Wednesday and all employees will be paid salaries for five full days on the following Monday. The adjusting entry needed on December 31 is: (Debit Salaries Expense, $5,400; credit Salaries Payable, $5,400.)
172) Zippy had cash inflows from operations of $60,500; cash outflows from investing activities of $47,000; and cash inflows from financing of $25,000. The net change in cash was: (38,500 increase.)
173) Identify which error will cause the trial balance to be out of balance. (A $100 cash receipt from a customer in payment of her account posted as a $100 debit to Cash and a $10 credit to Accounts Receivable.)
173) On January 1, Imlay Company purchases manufacturing equipment costing $95,000 that is expected to have a five-year life and an estimated salvage value of $5,000. Imlay uses the straight-line depreciation method to allocate costs, and only prepares adjustments at year-end. The adjusting entry needed on December 31 of the first year is: (Debit Depreciation Expense, $18,000; credit Accumulated Depreciation, $18,000.)
173) On September 12, Ryan Company sold merchandise in the amount of $5,800 to Johnson Company, with credit terms of 2/10, n/30. The journal entry or entries that Ryan will make on September 12 is (are) (Accounts receivable 5,684 Sales 5,684)
173) Zapper has beginning equity of $257,000, net income of $51,000, withdrawals of $40,000 and investments by owners of $6,000. Its ending equity is: (274,000.)
174) A $130 credit to Supplies was credited to Fees Earned by mistake. By what amounts are the accounts under- or overstated as a result of this error? (Supplies, overstated $130; Fees Earned, overstated $130.)
174) Cragmont has beginning equity of $277,000, net income of $63,000, withdrawals of $25,000 and no additional investments by owners during the period. Its ending equity is: (315,000.)
174) Holman Company owns equipment with an original cost of $95,000 and an estimated salvage value of $5,000 that is being depreciated at $15,000 per year using the straight-line depreciation method, and only prepares adjustments at year-end. The adjusting entry needed to record annual depreciation is: (Debit Depreciation Expense, $15,000; credit Accumulated Depreciation, $15,000.)
174) On September 12, Ryan Company sold merchandise in the amount of $5,800 to Johnson Company, with credit terms of 2/10, n/30. The journal entry that Johnson will make on September 12 is (Purchases 5,684 Accounts payable 5,684)
175) All of the following are asset accounts except: (Supplies expense.)
175) On November 1, Jovel Company loaned another company $100,000 at a 6.0% interest rate. The note receivable plus interest will not be collected until March 1 of the following year. The company's annual accounting period ends on December 31. The amount of interest revenue that should be reported in the first year is: ($1,000.)
175) On September 12, Ryan Company sold merchandise in the amount of $5,800 to Johnson Company, with credit terms of 2/10, n/30. The journal entry that Ryan makes on September 18 is (Cash 5,684 Accounts receivable 5,684)
175) Rent expense appears on which of the following statements? (Income statement.)
176) A company's balance sheet shows: cash $22,000, accounts receivable $16,000, office equipment $50,000, and accounts payable $17,000. What is the amount of owner's equity? (71,000.)
176) Compare the list of accounts below and choose the list that contains only accounts that would be classified as asset accounts on the Chart of Accounts. (Cash; Prepaid Insurance; Equipment.)
176) On November 1, Jovel Company loaned another company $100,000 at a 6.0% interest rate. The note receivable plus interest will not be collected until March 1 of the following year. The company's annual accounting period ends on December 31, and adjustments are only made at year-end. The adjusting entry needed on December 31 is: (Debit Interest Receivable, $1,000; credit Interest Revenue, $1,000.)
176) On September 12, Ryan Company sold merchandise in the amount of $5,800 to Johnson Company, with credit terms of 2/10, n/30. The journal entry that Johnson makes on September 18 is (Accounts payable 5,684 Cash 5,684)
177) A company reported total equity of $145,000 at the beginning of the year. The company reported $210,000 in revenues and $165,000 in expenses for the year. Liabilities at the end of the year totaled $92,000. What are the total assets of the company at the end of the year? (282,000.)
177) On December 1, Casualty Insurance Company borrowed $50,000 at a 6.0% interest rate from One Mutual Bank. The note payable plus interest will not be paid until April 1 of the following year. The company's annual accounting period ends on December 31, and adjustments are only made at year-end. The adjusting entry needed on December 31 is: (Debit Interest Expense, $250; credit Interest Payable, $250.)
177) On September 12, Ryan Company sold merchandise in the amount of $5,800 to Johnson Company, with credit terms of 2/10, n/30. The entry or entries that Ryan must make on September 14 is (are) (Sales returns and allowances 490 Accounts receivable 490)
177) Which financial statement reports an organization's financial position at a single point in time? (Balance sheet.)
178) Flitter reported net income of $17,500 for the past year. At the beginning of the year the company had $200,000 in assets and $50,000 in liabilities. By the end of the year, assets had increased to $300,000 and liabilities were $75,000. Calculate its return on assets: (7.0%.)
178) Jeff Jackson opened Jackson's Repairs on March 1 of the current year. During March, the following transactions occurred: Based on this information, net income for March would be: ($10,300.)
178) On September 12, Ryan Company sold merchandise in the amount of $5,800 to Johnson Company, with credit terms of 2/10, n/30. The journal entry that Ryan makes on September 18 is (Cash 5,194 Accounts receivable 5,194)
178) Which of the following statements is not true? (Financial statements should be prepared directly from information in the unadjusted trial balance.)
179) Dawson Electronic Services had revenues of $80,000 and expenses of $50,000 for the year. Its assets at the beginning of the year were $400,000. At the end of the year assets were worth $450,000. Calculate its return on assets. (7.1%.)
179) On December 31, Carmack Company received a $215 utility bill for December that it will not pay until January 15. The adjusting entry needed on December 31 to accrue this expense is: (Debit Utilities Expense $215; credit Accounts Payable $215.)
179) Web Consulting received $3,000 from a customer for services provided. The general journal entry to record this transaction will be: (Debit Cash, credit Services Revenue.)
18) An account's balance is the difference between the total debits and total credits for the account, including any beginning balance. (TRUE)
18) In the partnership form of business, the owners are called stockholders. (FALSE)
18) The accrual basis of accounting recognizes expenses when cash is paid. (FALSE)
18) The acid-test ratio is also called the quick ratio. (TRUE)
18) The last four steps in the accounting cycle include preparing the adjusted trial balance, preparing financial statements, and recording closing and adjusting entries. (FALSE)
18. A company has net sales of $825,000 and cost of goods sold of $547,000. Its net income is $98,500. The company's gross margin and operating expenses, respectively, are: ($278,000 and $179,500.)
180) On December 31, Winters Company received a $385 bill for the purchase of supplies in December that it will not pay for until January 15. Winters follows a policy of recording all prepaid expenses to asset accounts at the time of cash payment. The adjusting entry needed on December 31 to accrue this cost is: (Debit Supplies $385; credit Accounts Payable $385.)
180) Rico's Taqueria had cash inflows from operating activities of $27,000; cash outflows from investing activities of $22,000, and cash outflows from financing activities of $12,000. Calculate the net increase or decrease in cash. (7,000 decrease.)
181) Charlie's Chocolates' owner made investments of $50,000 and withdrawals of $20,000. The company has revenues of $83,000 and expenses of $64,000. Calculate its net income. (19,000.)
181) On December 31, Carmack Company's Prepaid Insurance account had a balance before adjustment of $6,000. The insurance was purchased on July 1 of the same year for one year of insurance coverage, with coverage beginning on that date. Assuming adjustments are only made at year-end, the adjusting entry needed on December 31 is: (Debit Insurance Expense $3,000; credit Prepaid Insurance $3,000.)
182) On December 31, Winters Company's Prepaid Rent account had a balance before adjustment of $6,000. Three months' rent was paid in advance on December 1, the first day of the lease term. The adjusting entry needed on December 31 is: (Debit Rent Expense $2,000; credit Prepaid Rent $2,000.)
182) Savvy Sightseeing had beginning equity of $72,000; revenues of $90,000, expenses of $65,000, and withdrawals by owners of $9,000. Calculate the ending equity. (88,000.)
183) Doc's Ribhouse had beginning equity of $52,000; net income of $35,000, and withdrawals by the owner of $12,000. The owner made no investments during the year. Calculate the ending equity. (75,000.)
184) A company's balance sheet shows: cash $24,000, accounts receivable $30,000, equipment $50,000, and equity $72,000. What is the amount of liabilities? (32,000.)
185) If a company has excess space in its building that it rents to another company for $700, what is the effect on the accounting equation during the first month? (Assets would increase $700 and equity would increase $700.)
186) All of the following are classified as assets except: (Accounts Payable.)
187) Which of the following accounts is not included in the calculation of a company's ending owner's equity? (Cash.)
188) All of the following are classified as liabilities except: (Accounts Receivable.)
189) Billington Corp borrows $80,000 cash from Second National Bank. How does this transaction affect the accounting equation for Billington? (Assets would increase $80,000 and liabilities would increase $80,000.)
19) A classified balance sheet organizes assets and liabilities into important subgroups that provide more information to decision makers. (TRUE)
19) Quick assets include cash and cash equivalents, inventory, and current receivables. (FALSE)
19) Recording revenues early overstates current-period income; recording revenues late understates current period income. (TRUE)
19) The balance sheet shows a company's net income or loss over a period of time. (FALSE)
19) The right side of an account is called the debit side. (FALSE)
190) If the assets of a company increase by $55,000 during the year and its liabilities increase by $25,000 during the same year, then the change in equity of the company during the year must have been: (An increase of $30,000.)
191) All of the following are classified as assets except: (Accounts Payable.)
192) Grandmark Printing pays $2,000 rent to the landlord of the building where its facilities are located. How does this transaction affect the accounting equation for Grandmark? (Assets would decrease $2,000 and equity would decrease $2,000.)
193) Atkins Company collected $1,750 as payment for the amount owed by a customer from services provided the prior month on credit. How does this transaction affect the accounting equation for Atkins? (One asset would increase $1,750 and a different asset would decrease $1,750, causing no net change in the accounting equation.)
194) The accounting equation for Ying Company shows a decrease in its assets and a decrease in its equity. Which of the following transactions could have caused that effect? (Advertising expense for the month was paid in cash.)
195) The accounting equation for Long Company shows an increase in its assets and an increase in its liabilities. Which of the following transactions could have caused that effect? (Equipment was purchased on credit.)
196) The expense recognition principle, also called the matching principle: (Prescribes that a company record the expenses it incurred to generate the revenue reported.)
197) The measurement principle, also called the cost principle: (Prescribes that accounting information is based on actual cost.)
198) The revenue recognition principle: (Provides guidance on when a company must recognize revenue.)
199) The full disclosure principle: (Prescribes that a company report the details behind financial statements that would impact users' decisions.)
2) A service company earns net income by buying and selling merchandise. (FALSE)
2) Income Summary is a temporary account only used for the closing process. (TRUE)
2) Preparation of a trial balance is the first step in processing a financial transaction. (FALSE)
2) Recordkeeping, or bookkeeping, is the recording of transactions and events, either manually or electronically. This is just one part of accounting. (TRUE)
2) The time period assumption assumes that an organization's activities can be divided into specific time periods such as months, quarters, or years. (TRUE)
2. On September 12, Vander Company sold merchandise in the amount of $5,800 to Jepson Company, with credit terms of 2/10, n/30. The cost of the items sold is $4,000. Vander uses the periodic inventory system and the gross method of accounting for sales. The journal entry or entries that Vander will make on September 12 is (are): (Accounts receivable 5,800 Sales 5,800)
20) An unclassified balance sheet provides more information to users than a classified balance sheet. (FALSE)
20) In a double-entry accounting system, total debits must equal total credits for all entries, and total debit account balances in the ledger must equal total credit account balances. (TRUE)
20) Recording expenses early overstates current-period income; recording expenses late understates current period income. (FALSE)
20) The acid-test ratio is defined as current assets divided by current liabilities. (FALSE)
20) The Financial Accounting Standards Board (FASB) is a group tasked with setting generally accepted account principles (GAAP). (TRUE)
200) The materiality constraint: (Prescribes that only information that would influence the decisions of a reasonable person need be disclosed.)
201) The going concern assumption: (Means that accounting information reflects a presumption that the business will continue operating instead of being closed or sold.)
202) The monetary unit assumption: (Means that we can express transactions and events in monetary, or money, units.)
203) The time period assumption: (Presumes that the life of a company can be divided into time periods, such as months and years, and that useful reports can be prepared for those periods.)
204) The business entity assumption: (Means that a business is accounted for separately from other business entities, including its owner.)
205) Congress passed the Dodd-Frank Wall Street Reform and Consumer Protection Act (or Dodd-Frank). Which of the following are two of the important provisions of Dodd-Frank? (Clawback and whistleblower.)
206) Clawback provisions and whistleblower provisions are components of which legislation? (Dodd-Frank Act.)
207) Which of the following accounts is not included in the asset section of the balance sheet? (Services revenue.)
208) Which of the following accounts is not included in the asset section of the balance sheet? (Wages expense.)
209) Which of the following accounts is not included in the liability section of the balance sheet? (Accounts receivable.)
21) A company with an acid-test ratio of 4.1 is unlikely to face near-term liquidity problems. (TRUE)
21) Current assets and current liabilities are expected to be used up or come due within one year or the company's operating cycle whichever is longer. (TRUE)
21) Increases in liability accounts are recorded as debits. (FALSE)
21) Prior to recording adjusting entries at the end of an accounting period, some accounts may not show correct balances even though all transactions were properly recorded. (TRUE)
21) The business entity principle means that accounting information reflects a presumption that the business will continue operating instead of being closed or sold. (FALSE)
210) Which of the following accounts is not included in the calculation of net income? (Cash.)
211) Which of the following combinations results in a net loss reported on the income statement? (Total revenues of $70,000 and total expenses of $74,000.)
212) Which of the following combinations does not result in the same amount of net income reported on the income statement? (Total revenues of $70,000 and total expenses of $60,000.)
22) A company paid $9,000 for a twelve-month insurance policy on February 1. The policy coverage began on February 1. On February 28, $750 of insurance expense must be recorded. (TRUE)
22) Debits increase asset and expense accounts. (TRUE)
22) GAAP aims to make information relevant, reliable, and comparable. (TRUE)
22) Intangible assets are long-term resources that benefit business operations and usually lack physical form. (TRUE)
22) Successful use of a just-in-time inventory system can narrow the gap between the acid-test and the current ratio. (TRUE)
23) A company's quick assets are $147,000 and its current liabilities are $143,000. This company's acid-test ratio is 1.03. (TRUE)
23) Assets are often classified into current assets, long-term investments, plant assets, and intangible assets. (TRUE)
23) Credits always increase account balances. (FALSE)
23) On October 15, a company received $15,000 cash as a down payment on a consulting contract. The amount was credited to Unearned Consulting Revenue. By October 31, 10% of the services required by the contract were completed. The company will record consulting revenue of $1,500 from this contract for October. (TRUE)
23) The business entity assumption means that a business is accounted for separately from other business entities, including its owner or owners. (TRUE)
24) A company's current ratio is 1.2 and its quick ratio is 0.25. This company is probably an excellent credit risk because the ratios reveal no indication of liquidity problems. (FALSE)
24) An expense account normally has a credit balance. (FALSE)
24) As a general rule, revenues should not be recognized in the accounting records when earned, but rather when cash is received. (FALSE)
24) Current liabilities are cash and other resources that are expected to be sold, collected or used within one year or the company's operating cycle whichever is longer. (FALSE)
24) The accrual basis of accounting reflects the principle that revenue is recorded when it is earned, not when cash is received. (TRUE)
25) A revenue account normally has a debit balance. (FALSE)
25) Long-term investments can include land held for future expansion. (TRUE)
25) Specific accounting principles are basic assumptions, concepts, and guidelines for preparing financial statements and arise out of long-used accounting practice. (FALSE)
25) The accrual basis of accounting requires adjustments to recognize revenues in the periods they are earned and to match expenses with revenues. (TRUE)
25) The gross margin ratio is defined as gross margin divided by net sales. (TRUE)
26) Adjusting entries are designed primarily to correct accounting errors. (FALSE)
26) Asset accounts are decreased by debits. (FALSE)
26) Intangible assets are assets that are long-term, have physical form, and are used to produce or sell products and services. (FALSE)
26) Limited liability and indefinite business life are characteristics of a corporation. (TRUE)
26) The profit margin ratio is the same as the gross profit ratio. (FALSE)
27) A company had net sales of $340,500, its cost of goods sold was $257,000, and its net income was $13,750. The company's gross margin ratio equals 24.5%. (TRUE)
27) A sole proprietorship is a business with multiple owners. (FALSE)
27) Adjustments are necessary to bring an asset or liability account to its proper amount and also update a related expense or revenue account. (TRUE)
27) Current liabilities include accounts receivable, unearned revenues, and salaries payable. (FALSE)
27) Debit means increase and credit means decrease for all accounts. (FALSE)
28) Asset accounts normally have debit balances and revenue accounts normally have credit balances. (TRUE)
28) Cash and office supplies are both classified as current assets. (TRUE)
28) Each adjusting entry will affect a balance sheet account. (TRUE)
28) The Merchandise Inventory account balance at the beginning of the current period is equal to the amount of ending Merchandise Inventory from the previous period. (TRUE)
28) Unlimited liability and separate taxation of the business are advantages of a sole proprietorship. (FALSE)
29) A partnership must pay an additional business income tax. (FALSE)
29) Adjusting entries always affect the cash account. (FALSE)
29) An owner's withdrawal account normally has a debit balance. (TRUE)
29) Credit terms for a purchase include the amounts and timing of payments from a buyer to a seller. (TRUE)
29) Plant assets are usually listed by how quickly they can be converted to cash. (FALSE)
3) An accounting system captures relevant data about transactions and then classifies, records, and reports data. (TRUE)
3) Gross profit is also called gross margin. (TRUE)
3) Interim financial statements report a company's business activities for a one-year period. (FALSE)
3) Revenue accounts are temporary accounts that should begin each accounting period with zero balances. (TRUE)
3) Source documents identify and describe transactions and events entering the accounting process. (TRUE)
3. On September 12, Vander Company sold merchandise in the amount of $5,800 to Jepson Company, with credit terms of 2/10, n/30. The cost of the items sold is $4,000. Jepson uses the periodic inventory system and the gross method of accounting for purchases. The journal entry that Jepson will make on September 12 is: (Purchases 5,800 Accounts payable 5,800)
30) A debit entry always increases an account. (FALSE)
30) Accrued expenses at the end of one accounting period are expected to result in cash payments in a future period. (TRUE)
30) Objectives, qualitative characteristics, elements, and recognition and measurement are components of the FASB conceptual framework. (TRUE)
30) Purchase returns refer to merchandise a buyer purchases but then returns to the seller. (TRUE)
30) The current ratio is used to help assess a company's ability to pay its debts in the near future. (TRUE)
31) A transaction that credits an asset account and credits a liability account must also affect one or more other accounts. (TRUE)
31) Accrued revenues at the end of one accounting period are expected to result in cash collections in a future period. (TRUE)
31) Objectivity means that financial information is supported by independent, unbiased evidence; it demands more than a person's opinion. (TRUE)
31) Purchase allowances refer to merchandise a buyer acquires but then returns to the seller. (FALSE)
31) The current ratio is computed by dividing current liabilities by current assets. (FALSE)
32) A transaction that decreases a liability and increases an asset must also affect one or more other accounts. (TRUE)
32) Each adjusting entry affects one or more income statement account, one or more balance sheet account, and never cash. (TRUE)
32) Purchase allowances refer to a price reduction (allowance) granted to a buyer of defective or unacceptable merchandise. (TRUE)
32) The going-concern assumption presumes that a business will continue operating instead of being closed or sold. (TRUE)
32) Trekker Bikes' current assets are $300 million and its current liabilities are $125 million. Its current ratio is 0.417. (FALSE)
33) Accrued expenses reflect transactions where cash is paid before a related expense is recognized. (FALSE)
33) If a company has current assets of $15,000 and current liabilities of $9,500, its current ratio is 1.6. (TRUE)
33) If insurance coverage for the next two years is paid for in advance, the amount of the payment is debited to an asset account called Prepaid Insurance. (TRUE)
33) The measurement principle prescribes that accounting information is based on subjective opinion rather than cost. (FALSE)
33) Under the perpetual inventory system, the cost of merchandise purchased is recorded in the Merchandise Inventory account. (TRUE)
34) Credit terms of 2/10, n/30 imply that the seller offers the purchaser a 2% cash discount if the amount is paid within 10 days of the invoice date. Otherwise, the full amount is due in 30 days. (TRUE)
34) Flo's Flowers' current ratio is 1.3. The industry average for the current ratio is 1.2. This indicates that Flo's can cover its short-term liabilities with its short-term assets. (TRUE)
34) The monetary unit assumption means that companies should express transactions in terms such as "a lot" or "very little". (FALSE)
34) The purchase of supplies on credit should be recorded with a debit to Supplies and a credit to Accounts Payable. (TRUE)
34) Under the accrual basis of accounting, adjustments are often made for prepaid expenses and unearned revenues. (TRUE)
35) A benefit of using a work sheet is that it aids in the preparation of the financial statements. (TRUE)
35) If a company purchases equipment paying cash, the journal entry to record this transaction will include a debit to Cash. (FALSE)
35) Sellers always offer a discount to buyers for prompt payment toward purchases made on credit. (FALSE)
35) The entry to record a cash receipt from a customer when the service is to be provided in a future period involves a debit to an unearned revenue account. (FALSE)
35) The International Accounting Standards Board (IASB) issues International Financial Reporting Standards (IFRS) that identify preferred accounting practices. (TRUE)
36) A limited liability company offers the limited liability of a partnership or proprietorship and the tax treatment of a corporation. (FALSE)
36) Adjustments must be entered in the journal and posted to the ledger after the work sheet is prepared. (TRUE)
36) Costs incurred during an accounting period but unpaid and unrecorded are accrued expenses. (TRUE)
36) If a company provides services to a customer on credit, the company providing the service should credit Accounts Receivable. (FALSE)
36) Purchase discounts are the same as trade discounts. (FALSE)
37) A limited liability company offers the limited liability of a corporation and the tax treatment of a partnership or proprietorship. (TRUE)
37) An adjusting entry often includes an entry to Cash. (FALSE)
37) If a company sells merchandise with credit terms 2/10 n/60, the credit period is 10 days and the discount period is 60 days. (FALSE)
37) The work sheet is a required report made available to external decision makers. (FALSE)
37) When a company bills a customer for $700 for services performed, the journal entry to record this transaction will include a $700 debit to Services Revenue. (FALSE)
38) A work sheet contains all of the balances for each account and therefore may be used as a substitute for the set of financial statements. (FALSE)
38) Before an adjusting entry is made to recognize the cost of expired insurance for the period, Prepaid Insurance and Insurance Expense are both overstated. (FALSE)
38) The debt ratio helps to assess the risk a company has of failing to pay its debts and is helpful to both its owners and creditors. (TRUE)
38) The Securities and Exchange Commission (SEC) is a U.S. government agency that oversees proper use of GAAP by companies that sell stock and debt to the public. (TRUE)
38) The seller is responsible for paying shipping charges and bears the risk of damage or loss in transit if goods are shipped FOB destination. (TRUE)
39) All necessary amounts needed to prepare the income statement can be taken from the income statement columns of the work sheet, including the net income or net loss. (TRUE)
39) Before an adjusting entry is made to accrue employee salaries, Salaries Expense and Salaries Payable are both understated. (TRUE)
39) If goods are shipped FOB destination, the seller does not record revenue from the sale until the goods arrive at their destination because the transaction is not complete until that point. (TRUE)
39) The four common forms of business ownership include sole proprietorship, partnership, corporation, and non-profit. (FALSE)
39) The higher a company's debt ratio, the lower the risk of a company not being able to pay its debts. (FALSE)
4) A fiscal year refers to an organization's accounting period that spans twelve consecutive months or 52 weeks. (TRUE)
4) Closing revenue and expense accounts at the end of the accounting period serves to make the revenue and expense accounts ready for use in the next period. (TRUE)
4) Cost of goods sold is also called cost of sales. (TRUE)
4) Financial accounting is the area of accounting aimed at serving external users by providing them with general-purpose financial statements. (TRUE)
4) Items such as sales receipts, bank statements, checks, and purchase orders are examples of a business's source documents. (TRUE)
4. On September 12, Vander Company sold merchandise in the amount of $5,800 to Jepson Company, with credit terms of 2/10, n/30. The cost of the items sold is $4,000. Vander uses the periodic inventory system and the gross method of accounting for sales. Jepson pays the invoice on September 18, and takes the appropriate discount. The journal entry that Vander makes on September 18 is: (Cash 5,684 Sales discounts 116 Accounts receivable 5,800)
40) Failure to record depreciation expense will overstate assets and understate expenses. (TRUE)
40) If goods are shipped FOB shipping point, the seller does not record revenue from the sale until the goods arrive at their destination because the transaction is not complete until that point. (FALSE)
40) On a work sheet, if the Debit total exceeds the Credit total of the Income Statement columns, a net loss is indicated. (TRUE)
40) The debt ratio is calculated by dividing total assets by total liabilities. (FALSE)
40) The four common forms of business ownership include sole proprietorship, partnership, limited liability company (LLC), and corporation. (TRUE)
41) A buyer using a perpetual inventory system records the costs of shipping merchandise it purchases in a Delivery Expense account. (FALSE)
41) A company that finances a relatively large portion of its assets with liabilities is said to have a high degree of financial leverage. (TRUE)
41) A company's month-end adjusting entry for Insurance Expense is $1,000. If this entry is not made then expenses are understated by $1,000 and net income is overstated by $1,000. (TRUE)
41) If all columns of a completed work sheet balance, you can be sure that no errors were made in its preparation. (FALSE)
41) The three major types of business activities are operating, financing, and investing. (TRUE)
42) A buyer of $5,000 in merchandise inventory does not take advantage of a supplier's credit terms of 2/10, n/30, and instead pays the invoice in full at the end of 30 days. The buyer will pay $4,900. (FALSE)
42) If a company is highly leveraged, this means that it has relatively high risk of not being able to repay its debt. (TRUE)
42) Normally closing entries are first entered in the general journal and then posted to the work sheet. (FALSE)
42) Planning is a part of each business activity (operating, investing, and financing), and gives each activity meaning and focus. (TRUE)
42) Profit margin can also be called return on sales. (TRUE)
43) Adjusting entries are usually entered in the work sheet before they are entered in the general journal. (TRUE)
43) Financing activities provide the resources organizations use to pay for resources such as land, buildings, and equipment. (TRUE)
43) FOB shipping point means that the buyer accepts ownership when the goods arrive at the buyer's place of business. (FALSE)
43) Profit margin measures the relation of debt to assets. (FALSE)
43) Stark Co. has liabilities of $105 million and total assets of $350 million. Its debt ratio is 40.0%. (FALSE)
44) A journal entry that affects only two accounts is called a compound entry. (FALSE)
44) Each sales transaction for a seller that uses a perpetual inventory system involves recognizing both revenue and cost of merchandise sold. (TRUE)
44) Investing activities include long-term borrowing and repaying of cash from lenders. (FALSE)
44) On a work sheet, the adjusted balances of revenues and expenses are sorted to the Income Statement columns of the work sheet. (TRUE)
44) Profit margin reflects the percent of profit in each dollar of revenue. (TRUE)
45) Investing activities are the acquiring and disposing of resources that an organization uses to acquire and sell its products or services. (TRUE)
45) Offering sales discounts on credit sales can benefit a seller by decreasing the delay in receiving cash and reducing future collections efforts. (TRUE)
45) On the work sheet, net income is entered in the Income Statement Credit column as well as the Balance Sheet or Statement of Owner's Equity Credit column. (FALSE)
45) Posting is the transfer of journal entry information to the ledger. (TRUE)
45) Profit margin is calculated by dividing net sales by net income. (FALSE)
46) All necessary amounts to prepare the balance sheet, including ending owner's capital, can be found in the Balance Sheet columns of the work sheet. (FALSE)
46) Owner financing refers to resources contributed by creditors or lenders. (FALSE)
46) Sales Discounts is added to the Sales account when computing a company's net sales. (FALSE)
46) Torsten had total assets of $149,501,000, net income of $6,242,000, and net sales of $209,203,000. Its profit margin was 2.98%. (TRUE)
46) Transactions are recorded first in the ledger and then transferred to the journal. (FALSE)
47) A contra account is an account linked with another account; it is added to that account to show the proper amount for the item recorded in the associated account. (FALSE)
47) A general journal gives a complete record of each transaction in one place, and shows the debits and credits for each transaction. (TRUE)
47) A worksheet can be helpful in showing the effects of proposed or "what if" transactions but not in helping to prepare financial statements. (FALSE)
47) Revenues are increases in equity (via net income) from a company's sales of products and services to customers. (TRUE)
47) Sales discounts has a normal debit balance because it decreases Sales, which has a normal credit balance. (TRUE)
48) A net loss occurs when revenues exceed expenses. (FALSE)
48) Because it is a necessary financial statement, the work sheet must be prepared according to specified accounting procedures. (FALSE)
48) If a company reporting on a calendar year basis, paid $18,000 cash on January 1 for one year of rent in advance (lease beginning January 1), and adjusting entries are made at the end of each month, the balance remaining in Prepaid Rent on December 1 should be $1,500. (TRUE)
48) The general journal is a collection of all accounts and their balances. (FALSE)
48) Under a perpetual inventory system, when a credit customer returns non-defective merchandise to the seller, the seller debits Sales Returns and Allowances and credits Accounts Receivable and also debits Merchandise Inventory and credits Cost of Goods Sold. (TRUE)
49) Accumulated depreciation is shown on the balance sheet as a subtraction from the cost of its related asset. (TRUE)
49) An expense account is normally closed by debiting Income Summary and crediting the expense account. (TRUE)
49) At a given point in time, a trial balance is a list of all ledger accounts and their balances. (TRUE)
49) Net income occurs when revenues exceed expenses. (TRUE)
49) The perpetual system requires that each sale of merchandise has two entries: the revenue side and the cost side. (TRUE)
5) A wholesaler buys products from manufacturers or other wholesalers and sells them to consumers. (FALSE)
5) Adjusting entries are made after the preparation of financial statements. (FALSE)
5) An account is a record of increases and decreases in a specific asset, liability, equity, revenue, or expense item. (TRUE)
5) Internal users of accounting information do not directly run the organization and have limited access to its accounting information. (FALSE)
5) The closing process takes place before financial statements have been prepared. (FALSE)
5. On September 12, Vander Company sold merchandise in the amount of $5,800 to Jepson Company, with credit terms of 2/10, n/30. The cost of the items sold is $4,000. Jepson uses the periodic inventory system and the gross method of accounting for purchases. Jepson pays the invoice on September 18, and takes the appropriate discount. The journal entry that Jepson makes on September 18 is: (Accounts payable 5,800 Purchases discounts 116 Cash 5,684)
50) A journal entry with a debit to cash of $980, a debit to Sales Discounts of $20, and a credit to Accounts Receivable of $1,000 means that a customer has taken a 10% cash discount for early payment. (FALSE)
50) A salary owed to employees is an example of an accrued expense. (TRUE)
50) Errors made in journalizing transactions, posting to the ledger, and preparing the trial balance can still exist in a balanced trial balance. (TRUE)
50) Liabilities are the owner's claim on assets. (FALSE)
50) The withdrawals account is normally closed by debiting it. (FALSE)
51) After posting the entries to close all revenue and expense accounts, the Income Summary account of Cleaver Auto Services has a $4,000 debit balance. This result implies that Cleaver earned a net income of $4,000. (FALSE)
51) Assets are the resources a company owns or controls that are expected to yield future benefits. (TRUE)
51) In accrual accounting, accrued revenues are recorded as liabilities. (FALSE)
51) Sales of $350,000 and net sales of $323,000 could reflect sales discounts of $27,000. (TRUE)
51) The trial balance can serve as a replacement for the balance sheet, since total debits must equal total credits. (FALSE)
52) A balanced trial balance is proof that no errors were made in journalizing transactions, posting to the ledger, and preparing the trial balance. (FALSE)
52) A perpetual inventory system is able to directly measure and monitor inventory shrinkage and there is no need for a physical count of inventory. (FALSE)
52) After posting the entries to close all revenue and expense accounts, Marker Company's Income Summary account has a credit balance of $6,000, and its Marker, Withdrawals account has a debit balance of $2,500. These balances indicate that net income for the current accounting period amounted to $3,500. (FALSE)
52) Depreciation expense is an example of an accrued expense. (FALSE)
52) Owner withdrawals are subtracted as expenses in the calculation of net income. (FALSE)
53) Earned but uncollected revenues are recorded during the adjusting process with a credit to a revenue account and a debit to an expense account. (FALSE)
53) If cash was incorrectly debited for $100 instead of correctly debiting accounts receivable for $100, assuming no other errors, the trial balance will balance. (TRUE)
53) Sales Discounts and Sales Returns and Allowances are contra revenue accounts that are debited to close the accounts during the closing process. (FALSE)
53) The accounting equation can be restated as: Assets − Equity = Liabilities. (TRUE)
53) When there is a net loss, the Income Summary account would have a credit balance. (FALSE)
54) Cost of Goods Sold is debited to close the account during the closing process. (FALSE)
54) Depreciation expense is the portion of a plant asset's cost that is allocated to that period. (TRUE)
54) The accounting equation implies that: Assets + Liabilities = Equity. (FALSE)
54) The financial statement that summarizes how equity changes over the reporting period is called the balance sheet. (FALSE)
54) The Income Summary account is used to close the permanent accounts at the end of an accounting period. (FALSE)
55) All plant assets, including land, are depreciated. (FALSE)
55) An income statement reports revenues earned minus expenses incurred over a period of time. (TRUE)
55) In a perpetual inventory system, the Merchandise Inventory account must be closed at the end of the accounting period. (FALSE)
55) Owner's investments are increases in equity from the sale of products or services. (FALSE)
55) The steps in the closing process are (1) close credit balances in revenue accounts to Income Summary; (2) close debit balances in expense accounts to Income Summary; (3) close Income Summary to Owner's Capital; (4) close Withdrawals to Owner's Capital. (TRUE)
56) During the closing process, Owner's Capital is closed to the Owner's Withdrawals account. (FALSE)
56) Every business transaction leaves the accounting equation in balance. (TRUE)
56) Net income for a period will be understated if accrued revenues are not recorded at the end of the accounting period. (TRUE)
56) The adjusting entry to reflect inventory shrinkage is a debit to Income Summary and a credit to Inventory Shrinkage Expense. (FALSE)
56) The detail of individual revenue and expense accounts is reported on the balance sheet. (FALSE)
57) A multiple-step income statement format shows detailed computations of net sales and other costs and expenses, and reports subtotals for various classes of items. (TRUE)
57) A post-closing trial balance is a list of permanent accounts and their balances after all closing entries. (TRUE)
57) An external transaction is an exchange within an entity that may or may not affect the accounting equation. (FALSE)
57) Depreciation measures the decline in market value of an asset. (FALSE)
57) The heading on every financial statement lists the three W's—Who (the name of the business); What (the name of the statement); and Where (the organization's address). (FALSE)
58) A company owes its employees $5,000 for the year ended December 31. It will pay employees on January 6 for the previous two weeks' salaries. The year-end adjusting entry on December 31 will include a debit to Salaries Expense and a credit to Cash. (FALSE)
58) From an accounting perspective, an event is a happening that affects the accounting equation, but cannot be measured. (FALSE)
58) If an owner's capital account had a $10,000 credit balance at the beginning of the period, and during the period, the owner invests an additional $5,000, the balance in the capital account listed on the trial balance will be equal to a debit balance of $5,000. (FALSE)
58) Operating expenses are classified into two categories: selling expenses and cost of goods sold. (FALSE)
58) The aim of a post-closing trial balance is to verify that (1) total debits equal total credits for temporary accounts, and (2) all temporary accounts have zero balances. (FALSE)
59) A company purchased $6,000 worth of supplies in August and recorded the purchase in the Supplies account. On August 31, the fiscal year-end, the physical count of supplies indicates the cost of unused supplies is $3,200. The adjusting entry would include a $2,800 debit to Supplies. (FALSE)
59) A company's post-closing trial balance has total debits of $40,560 and total credits of $40,650. Accordingly, the company should review for errors in the closing process. (TRUE)
59) A merchandiser's classified balance sheet reports merchandise inventory as a current asset. (TRUE)
59) Owner's equity is increased when cash is received from customers in payment of previously recorded accounts receivable. (FALSE)
59) Owner's withdrawals are not reported on a business's income statement. (TRUE)
6) A customer's promise to pay on credit is classified as an account payable by the seller. (FALSE)
6) A retailer buys products from manufacturers and sells them to wholesalers. (FALSE)
6) Adjusting entries result in a better matching of revenues and expenses for the period. (TRUE)
6) Auditors verify the effectiveness of internal controls. (TRUE)
6) Revenue and expense accounts are permanent accounts and should not be closed at the end of the accounting period. (FALSE)
6. On September 12, Vander Company sold merchandise in the amount of $5,800 to Jepson Company, with credit terms of 2/10, n/30. The cost of the items sold is $4,000. Vander uses the periodic inventory system and the gross method of accounting for sales. On September 14, Jepson returns some of the non-defective merchandise, which is restored to inventory. The entry or entries that Vander must make on September 14 is (are): (Sales returns and allowances 500 Accounts receivable 500)
60) A company performs 20 days of work on a 30-day contract before the end of the year. The total contract is valued at $6,000 and payment is not due until the contract is fully completed. The required adjusting entry includes a $4,000 credit to Unearned Revenue. (FALSE)
60) An income statement reports the revenues earned minus expenses incurred by a business over a period of time. (TRUE)
60) An owner's investment increases equity via net income. (FALSE)
60) Expenses related to accounting, human resource management, and financial management are known as selling expenses. (FALSE)
60) Reversing entries are optional. (TRUE)
61) A company performs 20 days of work on a 30-day contract before the end of the year. The total contract is valued at $6,000, with payment received in advance. The $6,000 cash receipt was initially recorded as Unearned Revenue. The required adjusting entry includes a $4,000 debit to Unearned Revenue. (TRUE)
61) Return on assets is often stated in ratio form as the amount of average total assets divided by revenue. (FALSE)
61) Reversing entries are recorded in response to external transactions that were created in error during the prior accounting period. (FALSE)
61) The balance sheet reports the financial position of a company at a point in time. (TRUE)
61) When a company has no reportable non-operating activities, its income from operations is simply labeled net income. (TRUE)
62) A company entered into a 2-month contract for $50,000 on April 1. It earned $25,000 of the contract services in April and billed the customer. The company should recognize the revenue when it receives the customer's check. (FALSE)
62) A single-step income statement includes cost of goods sold as another expense and shows only one subtotal for total expenses. (TRUE)
62) Return on assets is often stated in ratio form as the amount of income divided by assets invested. (TRUE)
62) Reversing entries simplify the entries to pay accrued liabilities from the previous accounting period. (TRUE)
62) The same four basic financial statements are prepared by both U.S. GAAP and IFRS. (TRUE)
63) Neither U.S. GAAP nor IFRs require the use of accrual basis accounting. (FALSE)
63) Return on assets is useful in evaluating management, analyzing and forecasting profits, and planning activities. (TRUE)
63) The adjusted trial balance must be prepared before the adjusting entries are made. (FALSE)
63) Under a periodic inventory system, purchases, purchases returns and allowances, purchase discounts, and transportation-in transactions are recorded in the Merchandise Inventory account. (FALSE)
63) Which of the following accounts is a temporary account (Salaries expense).
64) An unadjusted trial balance is a list of accounts and balances prepared before adjustments are recorded. (TRUE)
64) Arrow's net income of $120 million and average total assets of $1,500 million results in a return on assets of 8%. (TRUE)
64) The amount of net income is added on the statement of owner's equity. (TRUE)
64) The periodic inventory system requires updating the inventory account only at the end of the period. (TRUE)
64) Which of the following accounts is a permanent account (Accounts payable).
65) Financial statements can be prepared directly from the information in the adjusted trial balance. (TRUE)
65) In a periodic inventory system, cost of goods sold is recorded as each sale occurs. (FALSE)
65) Risk is the uncertainty about the return we will earn. (TRUE)
65) The accounting process begins with: (Analysis of business transactions and source documents.)
65) When closing entries are made (All temporary accounts are closed but permanent accounts are not closed).
66) Asset and liability balances are transferred from the adjusted trial balance to the income statement. (FALSE)
66) Generally, the lower the risk, the higher the return that can be expected. (FALSE)
66) Revenues, expenses, and withdrawals accounts, which are closed at the end of each accounting period are (Temporary accounts).
66) Under both the periodic and perpetual inventory systems, the temporary account Purchases Returns and Allowances is used to accumulate the cost of all returns and allowances for a period. (FALSE)
66) Which of the following statements is not true: (Accounts receivable are increased by customer payments.)
67) A business's source documents may include all of the following except: (Ledgers.)
67) Asset and liability balances are transferred from the adjusted trial balance to the balance sheet. (TRUE)
67) Delivery expense is reported as part of general and administrative expense in the seller's income statement. (FALSE)
67) U.S. Government Treasury bonds provide low return and low risk to investors. (TRUE)
67) Which of the following statements is incorrect (Permanent account is another name for revenue accounts).
68) A business's source documents: (Provide objective evidence that a transaction has taken place.)
68) Assets, liabilities, and equity accounts are not closed; these accounts are called (Permanent accounts).
68) New revenue recognition rules require that sellers report sales net of expected sales discounts. (TRUE)
68) Revenue and expense balances are transferred from the adjusted trial balance to the income statement. (TRUE)
68) The four basic financial statements include the balance sheet, income statement, statement of owner's equity, and statement of cash flows. (TRUE)
69) A record of the increases and decreases in a specific asset, liability, equity, revenue, or expense is known as a(n): (Account.)
69) An income statement reports on investing and financing activities. (FALSE)
69) Closing the temporary accounts at the end of each accounting period does all of the following except (Has no effect on the owner's capital account).
69) In preparing statements from the adjusted trial balance, the balance sheet must be prepared first. (FALSE)
69) Under new revenue recognition rules, the gross method requires a period-end adjusting entry to estimate future sales discounts. (TRUE)
7) Closing entries result in the owner's capital account being transferred into net income or net loss for the period ending. (FALSE)
7) Cost of goods sold represents the expense of buying and preparing merchandise for sale. (TRUE)
7) External auditors examine financial statements to verify that they are prepared according to generally accepted accounting principles. (TRUE)
7) Two main accounting principles used in accrual accounting are expense recognition and full closure. (FALSE)
7) Withdrawals by the owner are a business expense. (FALSE)
7. On September 12, Vander Company sold merchandise in the amount of $5,800 to Jepson Company, with credit terms of 2/10, n/30. The cost of the items sold is $4,000. Vander uses the periodic inventory system and the gross method of accounting for sales. On September 14, Jepson returns some of the merchandise. The selling price of the merchandise is $500 and the cost of the merchandise returned is $350. Jepson pays the invoice on September 18, and takes the appropriate discount. The journal entry that Vander makes on September 18 is: (Cash 5,194 Sales discounts 106 Accounts receivable 5,300)
70) A balance sheet covers activities over a period of time such as a month or year. (FALSE)
70) An account used to record the owner's investments in a business is called a(n): (Capital account.)
70) Inventory Returns Estimated, which reflects an adjustment to inventory for expected future returns, is a liability account reported in the balance sheet, usually under Current Liabilities. (FALSE)
70) It is acceptable to record prepayment of expenses as debits to expense accounts if an adjusting entry is made at the end of the period to bring the asset account balance to the correct unused or unexpired amount. (TRUE)
70) Journal entries recorded at the end of each accounting period to prepare the revenue, expense, and withdrawals accounts for the upcoming period and to update the owner's capital account for the events of the period just finished are referred to as (Closing entries).
71) Identify the account used by businesses to record the transfer of assets from a business to its owner for personal use: (The owner's withdrawals account.)
71) Inventory Returns Estimated is a current asset account used in a period-end adjusting entry to reflect the inventory estimated to be returned in the future. (TRUE)
71) It is acceptable to record cash received in advance of providing products or services to revenue accounts if an adjusting entry is made at the end of the period to bring the liability account balance to the correct unearned amount. (TRUE)
71) The closing process is necessary in order to (Ensure that net income or net loss and owner withdrawals for the period are closed into the owner's capital account).
71) The income statement describes revenues earned and expenses incurred along with the resulting net income or loss over a specified period of time. (TRUE)
72) Closing entries are required (If the temporary accounts are to reflect correct amounts for each accounting period).
72) Identify the statement below that is correct: (Promises of future payment by the customer are called accounts receivable.)
72) The statement of cash flows shows the net effect of revenues and expenses for a reporting period. (FALSE)
72) The time period assumption assumes that an organization's activities may be divided into specific reporting time periods including all of the following except: (Days)
72) Under the net method, when a company uses a perpetual inventory system, an invoice for $2,000 with terms of 2/10, n/30 should be recorded with a debit to Merchandise Inventory and a credit to Accounts Payable of $2,000. (FALSE)
73) A broad principle that requires identifying the activities of a business with specific time periods such as months, quarters, or years is the: (Time period assumption)
73) The income statement shows the financial position of a business on a specific date. (FALSE)
73) The steps performed each reporting period in preparing financial statements, starting with analyzing and recording transactions in the journal and continuing through the post-closing trial balance, is referred to as the (Accounting cycle).
73) Unearned revenues are generally: (Liabilities created when a customer pays in advance for products or services before the revenue is earned.)
73) When purchases are recorded at net amounts, any discounts lost as a result of late payments are reported as an expense. (TRUE)
74) Interim financial statements refer to financial reports: (That cover less than one year, usually spanning one, three, or six-month periods)
74) The first section of the income statement reports cash flows from operating activities. (FALSE)
74) The net method records the invoice at its net amount (net of any cash discount). (TRUE)
74) Unearned revenues refer to a(n): (Liability that is settled in the future when a company delivers its products or services.)
74) Which of the following is the usual final step in the accounting cycle (Preparing a post-closing trial balance).
75) A classified balance sheet (Organizes assets and liabilities into important subgroups that provide more information).
75) Either the gross method or net method may be used to record sales with cash discounts, but the net method requires a period-end adjusting entry to estimate expected future sales discounts taken. (FALSE)
75) Prepaid accounts (also called prepaid expenses) are generally: (Assets that represent prepayments of future expenses.)
75) The 12-month period that ends when a company's sales activities are at their lowest level is called the: (Natural business year)
75) The balance sheet is based on the accounting equation. (TRUE)
76) A company's written promise to pay (in the form of a promissory note) a future amount is a(n): (Note payable.)
76) Investing activities involve the buying and selling of assets such as land and equipment that are held for long-term use in the business. (TRUE)
76) The assets section of a classified balance sheet usually includes the subgroups (Current assets, long-term investments, plant assets, and intangible assets).
76) The length of time covered by a set of periodic financial statements, primarily a year for most companies, is referred to as the: (Accounting period)
76) Under the net method of recording purchases, the Discounts Lost account is used when the purchaser fails to take a discount offered by the seller. (TRUE)
77) A merchandiser: (Earns net income by buying and selling merchandise.)
77) Operating activities include long-term borrowing and repaying cash from lenders, and cash investments or withdrawals by the owner. (FALSE)
77) The accounting principle that requires revenue to be recorded when earned is the: (Revenue recognition principle)
77) The record of all accounts and their balances used by a business is called a: (Ledger (or General Ledger).)
77) The usual order for the asset subgroups of a classified balance sheet is (Current assets, long-term investments, plant assets, intangible assets).
78) A classified balance sheet differs from an unclassified balance sheet in that (A classified balance sheet presents information in a manner that makes it easier to calculate a company's current ratio).
78) A company's ledger is: (A record containing all accounts and their balances used by the company.)
78) Adjusting entries: (Affect both income statement and balance sheet accounts)
78) Cost of goods sold: (Is the term used for the expense of buying and preparing merchandise for sale.)
78) The purchase of supplies appears on the statement of cash flows as an investing activity because it involves the purchase of assets. (FALSE)
79) A company has sales of $695,000 and cost of goods sold of $278,000. Its gross profit equals: ($417,000.)
79) A company's list of accounts and the identification numbers assigned to each account is called a: (Chart of accounts.)
79) The income statement reports on operating activities at a point in time. (FALSE)
79) The main purpose of adjusting entries is to: (Record internal transactions and events)
79) Two common subgroups for liabilities on a classified balance sheet are (Current liabilities and noncurrent liabilities).
8) A company had sales of $350,000 and cost of goods sold of $200,000. Its gross profit equals $150,000. (TRUE)
8) Adjusting entries are necessary so that asset, liability, revenue, and expense account balances are correctly recorded. (TRUE)
8) External users include lenders, shareholders, customers, and regulators. (TRUE)
8) The closing process is a step in the accounting cycle that prepares accounts for the next accounting period. (TRUE)
8) The purchase of land and buildings will generally be recorded in the same ledger account. (FALSE)
8. Cushman Company had $800,000 in net sales, $350,000 in gross profit, and $200,000 in operating expenses. Cost of goods sold equals: ($450,000.)
80) A company has sales of $375,000 and its gross profit is $157,500. Its cost of goods sold equals: ($217,500.)
80) A credit: (Is the right-hand side of a T-account.)
80) The broad principle that requires expenses to be reported in the same period as the revenues that were earned as a result of the expenses is the: (Expense recognition (Matching) principle)
80) The statement of cash flows identifies cash flows separated into operating, investing, and financing activities over a period of time. (TRUE)
80) Which of the following is classified as a current asset (Accounts receivable).
81) A debit: (Is the left-hand side of a T-account.)
81) Ending capital reported on the statement of owner's equity is calculated by adding owner investments and net losses and subtracting net income and withdrawals. (FALSE)
81) The system of preparing financial statements based on recognizing revenues when the cash is received and reporting expenses when the cash is paid is called: (Cash basis accounting)
81) Which of the following is classified as a plant asset (Equipment).
81) Which of the following statements regarding gross profit is not true? (Gross profit is not calculated on the multiple-step income statement.)
82) Adjusting entries made at the end of an accounting period accomplish all of the following except: (Assuring that external transaction amounts remain unchanged)
82) The cost-benefit constraint prescribes that only information with benefits of disclosure less than the costs of providing it, need be disclosed. (FALSE)
82) The current ratio (Is used to help assess a company's ability to pay its debts in the near future).
82) The right side of a T-account is a(n): (Credit.)
82) Which of the following statements regarding merchandise inventory is not true? (Merchandise inventory appears on the balance sheet of a service company.)
83) All of the following regarding the current ratio are true except (Current ratio does not affect a creditor's decision on whether to allow a company to buy on credit).
83) Identify the statement below that is incorrect: (The normal balance of an expense account is a credit.)
83) The approach to preparing financial statements based on recognizing revenues when they are earned and matching expenses to those revenues is: (Accrual basis accounting)
83) The cost-benefit constraint says that information disclosed must have benefits to the user that are greater than the costs of providing it. (TRUE)
83) Which of the following statements regarding the operating cycle of a merchandising company is not true? (The operating cycle is shortened by credit sales.)
84) A credit is used to record an increase in all of the following accounts except: (Wages Expense.)
84) Merchandise inventory: (Is a current asset.)
84) Net income is sometimes called earnings or profit. (TRUE)
84) Prepaid expenses, depreciation expense, accrued expenses, unearned revenues, and accrued revenues are all examples of: (Items that require adjusting entries)
84) The Unadjusted Trial Balance columns of a company's work sheet shows the Store Supplies account with a balance of $750. The Adjustments columns show a credit of $425 for supplies used during the period. The amount shown as Store Supplies in the Balance Sheet columns of the work sheet is ($325 debit).
85) A debit is used to record an increase in all of the following accounts except: (Accounts Payable.)
85) Accounting is an information and measurement system that does all of the following except: (Eliminates the need for interpreting financial data.)
85) An optional aid used to prepare a company's unadjusted trial balance, adjusting entries, adjusted trial balance, and financial statements is a(n) (Work sheet).
85) The accrual basis of accounting: (Is generally accepted for external reporting because it is more useful than cash basis for most business decisions)
85) The operating cycle for a merchandiser that sells only for cash moves from: (Purchases of merchandise to inventory to cash sales.)
86) Accumulated Depreciation and Fees Earned would be sorted to which respective columns in completing a work sheet (Balance Sheet and Statement of Owner's Equity-Credit; and Income Statement-Credit).
86) Identify the account below that is classified as a liability in a company's chart of accounts: (Unearned Revenue.)
86) In its first year of operations, Grace Company reports the following: Earned revenues of $60,000 ($52,000 cash received from customers); incurred expenses of $35,000 ($31,000 cash paid toward them); prepaid $8,000 cash for costs that will not be expensed until next year. Net income under the accrual basis of accounting is: ($25,000)
86) Technology: (Reduces the time, effort and cost of recordkeeping.)
86) The current period's ending inventory is: (The next period's beginning inventory.)
87) Beginning inventory plus net purchases is: (Merchandise (goods) available for sale.)
87) Identify the account below that is classified as an asset in a company's chart of accounts: (Accounts Receivable.)
87) In its first year of operations, Grace Company reports the following: Earned revenues of $60,000 ($52,000 cash received from customers); incurred expenses of $35,000 ($31,000 cash paid toward them); prepaid $8,000 cash for costs that will not be expensed until next year. Net income under the cash basis of accounting is: ($13,000)
87) The primary objective of financial accounting is to: (Provide accounting information that serves external users.)
87) Which of the following statements is incorrect (On the work sheet, the adjusted amounts are sorted into columns according to whether the accounts are used in preparing the unadjusted trial balance or the adjusted trial balance).
88) A company shows a $600 balance in Prepaid Rent in the Unadjusted Trial Balance columns of the work sheet. The Adjustments columns show expired rent of $200. This adjusting entry results in ($200 decrease in net income).
88) Identify the account below that is classified as an asset account: (Supplies.)
88) The acid-test ratio: (Is also called the quick ratio.)
88) The area of accounting aimed at serving the decision making needs of internal users is: (Managerial accounting.)
88) Which of the following statements is incorrect? (Adjusting entries affect only balance sheet accounts)
89) An adjusting entry could be made for each of the following except: (Owner investments)
89) External users of accounting information include all of the following except: (Purchasing managers.)
89) Identify the account below that is classified as a liability account: (Accounts Payable.)
89) Quick assets are defined as: (Cash, short-term investments, and current receivables.)
89) Statements that show the financial statements as if proposed transactions had already occurred are called (Pro forma statements).
9) A company had net sales of $545,000 and cost of goods sold of $345,000. Its gross margin equals $890,000. (FALSE)
9) Closing entries are required at the end of each accounting period to close all ledger accounts. (FALSE)
9) Regulators often have legal authority over certain activities of organizations. (TRUE)
9) The expense recognition (matching) principle does not aim to record expenses in the same accounting period as the revenue earned as a result of these expenses. (FALSE)
9) Unearned revenues are classified as liabilities. (TRUE)
9. Cushman Company had $800,000 in sales, sales discounts of $12,000, sales returns and allowances of $18,000, cost of goods sold of $380,000, and $275,000 in operating expenses. Gross profit equals: ($390,000.)
90) A company made no adjusting entry for accrued and unpaid employee wages of $28,000 on December 31. This oversight would: (Overstate net income by $28,000)
90) Identify the account below that impacts the equity of a business: (Utilities Expense.)
90) In preparing a work sheet an adjusted trial balance amount is mistakenly sorted to the wrong work sheet column. Upon completion of the worksheet, the Balance Sheet columns will balance, but with the wrong net income, if the amount sorted in error is (A liability amount placed in the Income Statement Credit column).
90) KLM Corporation's quick assets are $5,888,000, its current assets are $11,700,000 and its current liabilities are $8,000,000. Its acid-test ratio equals: (0.74.)
90) Which of the following is not true regarding a Certified Public Accountant? (Cannot hold any certificate other than a CPA.)
91) A company's current assets are $17,980, its quick assets are $11,420 and its current liabilities are $12,190. Its quick ratio equals: (0.94.)
91) If a company mistakenly forgot to record depreciation on office equipment at the end of an accounting period, the financial statements prepared at that time would show: (Assets, net income, and equity overstated)
91) If the Balance Sheet and Statement of Owner's Equity columns of a work sheet fail to balance when the net income is added to the Balance Sheet and Statement of Owner's Equity Credit column, the cause could be (An expense entered in the Balance Sheet and Statement of Owner's Equity Credit column).
91) Which of the following factors is not a component of the fraud triangle? (All of the above are components of the fraud triangle.)
91) Which of the following is NOT an equity account: (Unearned Revenue.)
92) A company's December 31 work sheet for the current period appears below. Based on the information provided, what is net income for the current period ($1,855).
92) If a company failed to make the end-of-period adjustment to move the amount of management fees that were earned from the Unearned Management Fees account to the Management Fees Revenue account, this omission would cause: (An overstatement of liabilities)
92) Liquidity problems are likely to exist when a company's acid-test ratio: (Is substantially lower than 1.)
92) Which of the following is NOT an asset account: (Services Revenue.)
92) Which of the following is not true regarding ethics: (Ethics do not affect the operations or outcome of a company.)
93) A business uses a credit to record: (A decrease in an asset account.)
93) A company records the fees for legal services paid in advance by its clients in an account called Unearned Legal Fees. If the company fails to make the end-of-period adjusting entry to move the portion of these fees that has been earned to a revenue account, one effect will be: (An understatement of equity)
93) A company's December 31 work sheet for the current period appears below. Based on the information provided, what is net income for the current period ($3,225).
93) A corporation is: (A business legally separate from its owners.)
93) The acid-test ratio differs from the current ratio in that: (Prepaid expenses and inventory are excluded from the calculation of the acid-test ratio.)
94) A tool that represents a ledger account and is used to show the effects of transactions is called a: (T-account.)
94) Profit margin is defined as: (Net income divided by net sales)
94) The group that sets international preferred accounting practices is called the: (IASB.)
94) Using the following year-end information for Calvin's Clothing, calculate the current ratio and acid-test ratio for the business: (Current Ratio = 3.50 and Acid-Test Ratio = 0.90.)
94) Which of the following errors would cause the Balance Sheet and Statement of Owner's Equity columns of a work sheet to be out of balance? (Entering a revenue amount in the Balance Sheet and Statement of Owner's Equity Debit column.)
95) A company earned $3,000 in net income for October. Its net sales for October were $10,000. Its profit margin is: (30%)
95) Identify the statement below that is correct: (The left side of a T-account is the debit side.)
95) The gross margin ratio: (Indicates the percent of sales revenue remaining after covering the cost of the goods sold.)
95) The Securities and Exchange Commission (SEC) has given the task of setting GAAP to the: (FASB.)
95) The Unadjusted Trial Balance columns of a work sheet total $84,000. The Adjustments columns contain entries for the following: ($85,700.)
96) A company's gross profit (or gross margin) was $83,750 and its net sales were $347,800. Its gross margin ratio is: (24.1%.)
96) An account balance is: (The difference between the total debits and total credits for an account including the beginning balance.)
96) The accounting concept that requires every business to be accounted for separately from other business entities, including its owner or owners is known as the: (Business entity assumption.)
96) The balances in the unadjusted columns of a work sheet will agree with: (the balances reflected in the company's unadjusted trial balance.)
96) Which of the following statements regarding profit margin is not true? (Profit margin is not a useful measure of a company's operating results)
97) A company had $7,000,000 in net income for the year. Its net sales were $15,200,000 for the same period. Calculate its profit margin. (46.1%)
97) A company's net sales were $676,600, its cost of goods sold was $236,810 and its net income was $33,750. Its gross margin ratio equals: (65%.)
97) In the process of completing a work sheet, the accountant determines that the Income Statement debit column totals $83,000, while the Income Statement credit column totals $65,000. To enter net income (or net loss) for the period into the work sheet would require an entry to (the Balance Sheet & Statement of Owner's Equity debit column and the Income Statement credit column.)
97) Select the account below that normally has a credit balance: (Wages Payable.)
97) The rule that requires financial statements to reflect the assumption that the business will continue operating instead of being closed or sold, unless evidence shows that it will not continue, is the: (Going-concern assumption.)
98) A company had net sales of $752,000 and cost of goods sold of $543,000. Its net income was $17,530. The company's gross margin ratio equals: (27.8%.)
98) A debit is used to record which of the following: (An increase in the owner's withdrawals account.)
98) If a company is considering the purchase of a parcel of land that was acquired by the seller for $85,000, is offered for sale at $150,000, is assessed for tax purposes at $95,000, is considered by the purchaser as easily being worth $140,000, and is purchased for $137,000, the land should be recorded in the purchaser's books at: ($137,000.)
98) On July 1 Plum Co. paid $7,500 cash for management services to be performed over a two-year period. Plum follows a policy of recording all prepaid expenses to asset accounts at the time of cash payment. On July 1 Plum should record: (A debit to a prepaid expense and a credit to Cash for $7,500)
98) The special account used only in the closing process to temporarily hold the amounts of revenues and expenses before the net difference is added to (or subtracted from) the owner's capital account is the: (Income Summary account.)
99) A credit entry: (Decreases asset and expense accounts, and increases liability, owner's capital, and revenue accounts.)
99) K. Canopy, the proprietor of Canopy Services, withdrew $5,700 from the business during the current year. The entry to close the withdrawals account at the end of the year is: (Debit K. Canopy, Capital $5,700; credit K. Canopy, Withdrawals $5,700)
99) Mega Skateboard Supplier had net sales of $2.8 million, its cost of goods sold was $1.6 million, and its net income was $0.9 million. Its gross margin ratio equals: (43%.)
99) On July 1 of the current calendar year, Plum Co. paid $7,500 cash for management services to be performed over a two-year period beginning July 1. Plum follows a policy of recording all prepaid expenses to asset accounts at the time of cash payment. The adjusting entry on December 31 of the current year for Plum would include: (A debit to an expense and a credit to a prepaid expense for $1,875)
99) To include the personal assets and transactions of a business's owner in the records and reports of the business would be in conflict with the: (Business entity assumption.)
Shopping Cart